You are on page 1of 1971

D. A.

Evans An Introduction to Frontier Molecular Orbital Theory-1 Chem 206

http://www.courses.fas.harvard.edu/~chem206/ ■ Problems of the Day


The molecule illustrated below can react through either Path A or Path B to
form salt 1 or salt 2. In both instances the carbonyl oxygen functions as the
http://evans.harvard.edu/problems/ nucleophile in an intramolecular alkylation. What is the preferred reaction path
for the transformation in question?
O
Chemistry 206 Path A
1 Br +
O N O

Br Br H Br
Advanced Organic Chemistry N O
H O
Path B
2
+ Br
Lecture Number 1 N O
H Br –
This is a "thought" question posed to me by Prof. Duilo Arigoni at the ETH in
Introduction to FMO Theory Zuerich some years ago
■ General Bonding Considerations

■ The H2 Molecule Revisited (Again!)

■ Donor & Acceptor Properties of Bonding & Antibonding States

■ Hyperconjugation and "Negative" Hyperconjugation (First hr exam, 1999)


The three phosphites illustrated below exhibit a 750–fold span in reactivity with a
■ Anomeric and Related Effects test electrophile (eq 1) (Gorenstein, JACS 1984, 106, 7831).

■ Reading Assignment for week: +


(RO)3P + El(+) (RO)3P–El (1)
Kirby, Stereoelectronic Effects
OMe O
Carey & Sundberg: Part A; Chapter 1
O P OMe
Fleming, Chapter 1 & 2 O P P O
O O O
Fukui,Acc. Chem. Res. 1971, 4, 57. (pdf) A B C
Curnow, J. Chem. Ed. 1998, 75, 910 (pdf)
Rank the phosphites from the least to the most nucleophilic and
Alabugin & Zeidan, JACS 2002, 124, 3175 (pdf) provide a concise explanation for your predicted reactivity order.

Monday,
D. A. Evans September 15, 2003

1-01-Cover Page 9/15/03 8:56 AM


D. A. Evans An Introduction to Frontier Molecular Orbital Theory-1 Chem 206

Universal Effects Governing Chemical Reactions ■ Stereoelectronic Effects


There are three:
■ Steric Effects Geometrical constraints placed upon ground and transition states
by orbital overlap considerations.
Nonbonding interactions (Van der Waals repulsion) between
substituents within a molecule or between reacting molecules Fukui Postulate for reactions:
Me Me "During the course of chemical reactions, the interaction of
S N2

Nu: C Br Nu C R Br: the highest filled (HOMO) and lowest unfilled (antibonding)
R
R R molecular orbital (LUMO) in reacting species is very important
to the stabilization of the transition structure."
RO RO H
O O major
H H
■ General Reaction Types
Me
Me2CuLi RO Me
p. O
minor
Radical Reactions (~10%): A• + B• A B
p.
H
H Polar Reactions (~90%): A(:) + B(+) A B
■ Electronic Effects (Inductive Effects):
The effect of bond and through-space polarization by
heteroatom substituents on reaction rates and selectivities Lewis Acid
Lewis Base
Inductive Effects: Through-bond polarization
Field Effects: Through-space polarization
FMO concepts extend the donor-acceptor paradigm to
non-obvious families of reactions
■ Examples to consider
Me +
S N1 R
C Br C Me + Br:– H2 + 2 Li(0) 2 LiH
R R
R
CH3–I + Mg(0) CH3–MgBr
rate decreases as R becomes more electronegative
"Organic chemists are generally unaware of the impact of
electronic effects on the stereochemical outcome of reactions."
"The distinction between electronic and stereoelectronic effects is
not clear-cut."
1-02-Introduction-1 9/12/03 4:44 PM
D. A. Evans Steric Versus Electronic Effects; A time to be careful!! Chem 206

■ Steric Versus electronic Effects: Some Case Studies OSiR3


O
When steric and electronic (stereoelectronic) effects R3SiO
TiCl4 diastereoselection
lead to differing stereochemical consequences
>94:6
Woerpel etal. JACS 1999, 121, 12208. EtO Nu
OSiR3 OSiR3
R3Si
O OAc O O
SnBr4
O O
H
Me Me Me AlCl3 diastereoselection
93:7
SiMe3 stereoselection 99:1
H
OSiR3 OSiR3
stereoselection >95:5
O OAc O O
SnBr4
Danishefsky et al JOC 1991, 56, 387

BnO BnO BnO


p.
p.

O
EtO2C
(R)2CuLi diastereoselection O AcO
O H
8:1 AcO
EtO2C N H
Bu Ph N
OTBS N N
O
O N only diastereomer
O
EtO2C
OTBS Bu3Al N
only diastereomer Ph
O O
OAc
Bu H OAc
R3 OTBS
Ph N H OAc
O Al O OAc H
Yakura's O O
EtO H 60-94%
rationalization:
N
Ph
R O
O TBS
Al Yakura et al
R R Tetrahedron 2000, 56, 7715
Mehta et al, Acc Chem. Res. 2000, 33, 278-286

1-03-Introduction-1a 9/15/03 8:14 AM


D. A. Evans The H2 Molecular Orbitals & Antibonds Chem 206

The H2 Molecule (again!!) Linear Combination of Atomic Orbitals (LCAO): Orbital Coefficients
Let's combine two hydrogen atoms to form the hydrogen molecule. ■ Rule Two:
Mathematically, linear combinations of the 2 atomic 1s states create Each MO is constructed by taking a linear combination of the
two new orbitals, one is bonding, and one antibonding: individual atomic orbitals (AO):

■ Rule one: A linear combination of n atomic states will create n MOs.


Bonding MO σ = C1ψ1 + C2ψ2

σ∗ (antibonding) Antibonding MO σ∗ = C*1ψ1 – C*2ψ2

The coefficients, C1 and C2, represent the contribution of each AO.


∆E
Energy

H 1s 1s H ■ Rule Three: (C1)2 + (C2)2 = 1


ψ1 ψ2 The squares of the C-values are a measure of the electron population
∆E
in neighborhood of atoms in question
p.
p.

σ (bonding) ■ Rule Four: bonding(C1)2 + antibonding(C*1)2= 1

In LCAO method, both wave functions must each contribute


one net orbital
Let's now add the two electrons to the new MO, one from each H atom:
Consider the pi–bond of a C=O function: In the ground state pi-C–O
σ∗ (antibonding) is polarized toward Oxygen. Note (Rule 4) that the antibonding MO
is polarized in the opposite direction.

∆E1
Energy

C O π∗ (antibonding)
H 1s 1s H
ψ1 ψ2
∆E2
Energy

σ (bonding) C
O

Note that ∆E1 is greater than ∆E2. Why?


π (bonding)
C O

1-04-Introduction-2 9/15/03 8:38 AM


D. A. Evans Bonding Generalizations Chem 206
■ Bond strengths (Bond dissociation energies) are composed of a ■ Orbital orientation strongly affects the strength of the resulting bond.
covalent contribution (δ Ecov) and an ionic contribution (δ Eionic).
For σ Bonds:
Bond Energy (BDE) = δ Ecovalent + δ Eionic (Fleming, page 27) Better
A B A B
than
When one compares bond strengths between C–C and C–X, where X
is some other element such as O, N, F, Si, or S, keep in mind that
covalent and ionic contributions vary independently. Hence, the
mapping of trends is not a trivial exercise. For π Bonds: Better A B
A B than
Useful generalizations on covalent bonding
This is a simple notion with very important consequences. It surfaces in
■ Overlap between orbitals of comparable energy is more effective the delocalized bonding which occurs in the competing anti (favored)
than overlap between orbitals of differing energy. syn (disfavored) E2 elimination reactions. Review this situation.

p.
For example, consider elements in Group IV, Carbon and Silicon. We ■ An anti orientation of filled and unfilled orbitals leads to better overlap.
p. know that C-C bonds are considerably stronger by Ca. 20 kcal mol-1 This is a corrollary to the preceding generalization.
than C-Si bonds. There are two common situations.

C C C C better than C Si C Si
Case-1: Anti Nonbonding electron pair & C–X bond
σ∗ C–C
σ∗ C–Si

X X X
σ* C–X lone pair σ* C–X
Si-SP3 LUMO HOMO LUMO
A C A C Better A C
C-SP3 C-SP3 C-SP3 than
•• lone pair
σ C–Si
HOMO
σ C–C
H3C–CH3 BDE = 88 kcal/mol H3C–SiH3 BDE ~ 70 kcal/mol Case-2: Two anti sigma bonds
Bond length = 1.534 Å Bond length = 1.87 Å

X Y X
This trend is even more dramatic with pi-bonds: X σ* C–X Better σ C–Y σ* C–X
LUMO than HOMO LUMO
π C–C = 65 kcal/mol π C–Si = 36 kcal/mol π Si–Si = 23 kcal/mol C C
A C C C
■ Weak bonds will have corresponding low-lying antibonds. σ C–Y
Y HOMO
Formation of a weak bond will lead to a corresponding low-lying antibonding Y
orbital. Such structures are reactive as both nucleophiles & electrophiles

1-05-Introduction-3 9/12/03 4:36 PM


D. A. Evans Donor-Acceptor Properties of Bonding and Antibonding States Chem 206
Donor Acceptor Properties of C-C & C-O Bonds Hierarchy of Donor & Acceptor States
Consider the energy level diagrams for both bonding & antibonding Following trends are made on the basis of comparing the bonding and
orbitals for C–C and C–O bonds. antibonding states for the molecule CH3–X where X = C, N, O, F, & H.
σ* C-C
σ-bonding States: (C–X)
σ* C-O
CH3–CH3
CH3–H
3
C-SP C-SP3 CH3–NH2
very close!!
3 CH3–OH
O-SP

decreasing σ-donor capacity CH3–F

σ C-C poorest donor


σ C-O
■ The greater electronegativity of oxygen lowers both the bonding σ-anti-bonding States: (C–X)
p.
p. & antibonding C-O states. Hence: For the latest views, please read
■ σ C–C is a better donor orbital than σ C–O CH3–H Alabugin & Zeidan, JACS 2002, 124, 3175 (pdf)

■ σ∗C–O is a better acceptor orbital than σ∗C–C CH3–CH3


CH3–NH2

Donor Acceptor Properties of CSP3-CSP3 & CSP3-CSP2 Bonds CH3–OH

σ* C–C CH3–F

σ* C–C better acceptor Increasing σ∗-acceptor capacity best acceptor

The following are trends for the energy levels of nonbonding states
C-SP3 C-SP3 of several common molecules. Trend was established by
photoelectron spectroscopy.
C-SP2
Nonbonding States

σ C–C
better donor σ C–C H3P:
H2S:
■ The greater electronegativity of CSP2 lowers both the bonding & H3N:
antibonding C–C states. Hence: H2O:
■ σ CSP3-CSP3 is a better donor orbital than σ CSP3-CSP2 HCl:
decreasing donor capacity poorest donor
■ σ∗CSP3-CSP2 is a better acceptor orbital than σ∗CSP3-CSP3

1-06-donor/acceptor states 9/12/03 5:16 PM


D. A. Evans Hybridization vs Electronegativity Chem 206
Electrons in 2S states "see" a greater effective nuclear charge
There is a linear relationship between %S character &
than electrons in 2P states.
Pauling electronegativity

This becomes apparent when the radial probability functions for S 5


and P-states are examined: The radial probability functions for the
N
hydrogen atom S & P states are shown below. SP

4.5
100 % 100 %

Pauling Electronegativity
Radial Probability

4
1 S Orbital

Radial Probability
N
SP2
N
SP3
3.5
C
SP

3
2 S Orbital 2 S Orbital
C
SP2
2.5
p.
p. 2 P Orbital C
SP3

2
Å Å 20 25 30 35 40 45 50 55
% S-Character

3 S Orbital 3 P Orbital There is a direct relationship between %S character &


hydrocarbon acidity

60

CH (56)
4
55
S-states have greater radial penetration due to the nodal properties of the wave
function. Electrons in S-states "see" a higher nuclear charge.
50

Pka of Carbon Acid


Above observation correctly implies that the stability of nonbonding electron 45
pairs is directly proportional to the % of S-character in the doubly occupied orbital C H (44)
6 6

40

Least stable Most stable


35
CSP3 CSP2 CSP
PhCC-H (29)
30
The above trend indicates that the greater the % of S-character at
a given atom, the greater the electronegativity of that atom. 25
20 25 30 35 40 45 50 55
% S-Character

1-07-electroneg/hybrization 9/12/03 4:49 PM


D. A. Evans Hyperconjugation: Carbocation Stabilization Chem 206
■ The interaction of a vicinal bonding orbital with a p-orbital is referred
to as hyperconjugation. Physical Evidence for Hyperconjugation
This is a traditional vehicle for using valence bond to denote charge
delocalization. ■ Bonds participating in the hyperconjugative interaction, e.g. C–R,
will be lengthened while the C(+)–C bond will be shortened.
R R+
+
H H
H C C H C C First X-ray Structure of an Aliphatic Carbocation
H H
H H

The graphic illustrates the fact that the C-R bonding electrons can
"delocalize" to stabilize the electron deficient carbocationic center.
+ 1.431 Å [F5Sb–F–SbF5]–

Note that the general rules of drawing resonance structures still hold:
the positions of all atoms must not be changed. +
C

p.
p. 100.6 ° 1.608 Å Me
Stereoelectronic Requirement for Hyperconjugation: Me

Syn-planar orientation between interacting orbitals Me

The Molecular Orbital Description


σ∗ C–R σ∗ C–R T. Laube, Angew. Chem. Int. Ed. 1986, 25, 349

+ + The Adamantane Reference


H H (MM-2) 1.528 Å
C C
H H
H

110 °
σ C–R σ C–R Me 1.530 Å
Me

Me
■ Take a linear combination of σ C–R and CSP2 p-orbital:

"The new occupied bonding orbital is lower in energy. When you


stabilize the electrons is a system you stabilize the system itself."

1-08-Hyperconj (+)-1 9/12/03 4:53 PM


D. A. Evans "Negative" Hyperconjugation Chem 206
antibonding σ∗ C–R
■ Delocalization of nonbonding electron pairs into vicinal antibonding Syn Orientation
orbitals is also possible
R: – R
R ●●
●●
filled
R ●● R ●●
H C X H C X+ H C X hybrid orbital

H C X H H C X H H H H
H H H H

This decloalization is referred to as "Negative" hyperconjugation antibonding σ∗ C–R


Anti Orientation

R R: – R
Since nonbonding electrons prefer hybrid orbitals rather that P filled
H C X H C X+
orbitals, this orbital can adopt either a syn or anti relationship H C X hybrid orbital
to the vicinal C–R bond. H
●●
H ●●
H
The Molecular Orbital Description
■ Overlap between two orbitals is better in the anti orientation as
σ∗ C–R stated in "Bonding Generalizations" handout.

The Expected Structural Perturbations


X Nonbonding e– pair
●●
Change in Structure Spectroscopic Probe

■ Shorter C–X bond X-ray crystallography

■ Longer C–R bond X-ray crystallography


As the antibonding C–R orbital
σ C–R decreases in energy, the magnitude ■ Stronger C–X bond Infrared Spectroscopy
of this interaction will increase
■ Weaker C–R bond Infrared Spectroscopy

Note that σ C–R is slightly destabilized


■ Greater e-density at R NMR Spectroscopy

■ Less e-density at X NMR Spectroscopy


1-09-Neg-Hyperconj 9/12/03 4:53 PM
D. A. Evans Lone Pair Delocalization: N2F2 Chem 206

The interaction of filled orbitals with adjacent antibonding orbitals can Now carry out the same analysis with the same 2
The trans Isomer orbitals present in the trans isomer.
have an ordering effect on the structure which will stabilize a particular
geometry. Here are several examples:
filled
This molecule can exist as either cis or N-SP2 F
Case 1: N2F2 σ∗ N–F
trans isomers antibonding
N N σ∗ N–F (LUMO)
F F F F filled
N N N N N-SP2
(HOMO)
F
There are two logical reasons why the trans isomer should be more ■ In this geometry the "small lobe" of the filled N-SP2 is required to
stable than the cis isomer. overlap with the large lobe of the antibonding C–F orbital. Hence, when
the new MO's are generated the new bonding orbital is not as stabilizing
■ The nonbonding lone pair orbitals in the cis isomer will be destabilizing as for the cis isomer.
due to electron-electron repulsion.
Conclusions
■ The individual C–F dipoles are mutually repulsive (pointing in same
direction) in the cis isomer. ■ Lone pair delocalization appears to override electron-electron and
dipole-dipole repulsion in the stabilization of the cis isomer.
In fact the cis isomer is favored by 3 kcal/ mol at 25 °C. ■ This HOMO-LUMO delocalization is stronger in the cis isomer due
to better orbital overlap.
Let's look at the interaction with the lone pairs with the adjacent C–F
antibonding orbitals. Important Take-home Lesson

The cis Isomer Orbital orientation is important for optimal orbital overlap.

F σ∗ N–F
F (LUMO)
antibonding
N N σ∗ N–F A B forms stronger pi-bond than A B
filled
filled N-SP2
N-SP2
(HOMO)
A forms stronger
B A B
sigma-bond than
■ Note that by taking a linear combination of the nonbonding and
antibonding orbitals you generate a more stable bonding situation.
This is a simple notion with very important consequences. It surfaces in
■ Note that two such interactions occur in the molecule even though the delocalized bonding which occurs in the competing anti (favored)
only one has been illustrated. syn (disfavored) E2 elimination reactions. Review this situation.

1-10- N2F2 9/12/03 4:59 PM


D. A. Evans Lone Pair Delocalization: The Gauche Effect Chem 206
The interaction of filled orbitals with adjacent antibonding orbitals can The closer in energy the HOMO and LUMO the better the resulting
have an ordering effect on the structure which will stabilize a particular
stabilization through delocalization.
conformation.
Here are several examples of such a phenomon called the gauche effect:
■ Hence, N-lone pair ↔ σ∗ N–H delocalization better than
Hydrazine can exist in either gauche or anti σ N–H ↔ σ∗ N–H delocalization.
Hydrazine
conformations (relative to lone pairs).
●● ●● ■ Hence, hydrazine will adopt the gauche conformation where both
H H H H H ●● N-lone pairs will be anti to an antibonding acceptor orbital.
N N anti gauche
N N
H H The trend observed for hydrazine holds for oxygen derivatives as well
H H H H
observed HNNH H
●●
dihedral angle Ca 90°
H2O2 can exist in either gauche or anti
There is a logical reason why the anti isomer should be more stable than Hydrogen peroxide conformations (relative to hydrogens).
the gauche isomer. The nonbonding lone pair orbitals in the gauche The gauche conformer is prefered.
isomer should be destabilizing due to electron-electron repulsion. ●● ●●
In fact, the gauche conformation is favored. Hence we have neglected H
O O H ●● ●●
●●
an important stabilization feature in the structure. gauche
H anti O O
●●
HOMO-LUMO Interactions observed HOOH ●● H H
dihedral angle Ca 90° ●● H
Orbital overlap between filled (bonding) and antibonding states is
best in the anti orientation. HOMO-LUMO delocalization is possible ■ Major stabilizing interaction is the delocalization of O-lone pairs into
between: (a) N-lone pair ↔ σ∗ N–H; (b) σ N–H ↔ σ∗ N–H the C–H antibonding orbitals (Figure A). Note that there are no such
stabilizing interactions in the anti conformation while there are 2 in the
gauche conformation.
H H Figure A Figure B
filled σ∗ N–H σ∗ O–H
σ∗ N–H
N-SP3 N N (LUMO) σ N–H (LUMO)
N N (LUMO)
(HOMO) (HOMO)

H (HOMO)
(HOMO) filled
O-SP3
σ∗ N–H filled
(LUMO) O-SP3
filled
N-SP3 ■ Note that you achieve no net stabilization of the system by generating
(HOMO) molecular orbitals from two filled states (Figure B).
Problem: Consider the structures XCH2–OH where X = OCH3 and F.
better stabilization σ N–H
What is the most favorable conformation of each molecule? Illustrate the
dihedral angle relationship along the C–O bond.
(HOMO)
1-11 Gauche Effect 9/11/01 11:27 PM
D. A. Evans The Anomeric Effect: Negative Hyperconjugation Chem 206

Useful LIterature Reviews


http://www.courses.fas.harvard.edu/~chem206/
Kirby, A. J. (1982). The Anomeric Effect and Related Stereoelectronic Effects at
Oxygen. New York, Springer Verlag.
Box, V. G. S. (1990). “The role of lone pair interactions in the chemistry of the
Chemistry 206 monosaccharides. The anomeric effect.” Heterocycles 31: 1157.

Box, V. G. S. (1998). “The anomeric effect of monosaccharides and their


Advanced Organic Chemistry derivatives. Insights from the new QVBMM molecular mechanics force field.”
Heterocycles 48(11): 2389-2417.

Lecture Number 2 Graczyk, P. P. and M. Mikolajczyk (1994). “Anomeric effect: origin and
consequences.” Top. Stereochem. 21: 159-349.
Juaristi, E. and G. Cuevas (1992). “Recent studies on the anomeric effect.”
Stereoelectronic Effects-2 Tetrahedron 48: 5019.
Plavec, J., C. Thibaudeau, et al. (1996). “How do the Energetics of the
■ Anomeric and Related Effects Stereoelectronic Gauche and Anomeric Effects Modulate the Conformation of
Nucleos(t)ides?” Pure Appl. Chem. 68: 2137-44.
■ Electrophilic & Nucleophilic Substitution Reactions Thatcher, G. R. J., Ed. (1993). The Anomeric Effect and Associated
Stereoelectronic Effects. Washington DC, American Chemical Society.
■ The SN2 Reaction: Stereoelectronic Effects

■ Olefin Epoxidation: Stereoelectronic Effects


Problem 121 http://evans.harvard.edu/problems/
■ Baeyer-Villiger Reaction: Stereoelectronic Effects
Sulfonium ions A and B exhibit remarkable differences in both reactivity
and product distribution when treated with nucleophiles such as cyanide
■ Hard & Soft Acid and Bases (Not to be covered in class) ion (eq 1, 2). Please answer the questions posed in the spaces provided
below.

KCN
Reading Assignment: Kirby, Chapters 1-3 S + PhCH2CN (1)
S A rel. rate = 8000 Et
Et

BF4
D. A. Evans Wednesday, KCN
September 17, 2003 S Et S + MeCH2CN (2)
rel. rate = 1
B

2-00-Cover Page 9/17/03 8:35 AM


D. A. Evans The Anomeric Effect: Negative Hyperconjugation Chem 206

The Anomeric Effect ■ Since the antibonding C–O orbital is a better acceptor orbital than the
antibonding C–H bond, the axial OMe conformer is better stabilized by
It is not unexpected that the methoxyl substituent on a cyclohexane ring this interaction which is worth ca. 1.2 kcal/mol.
prefers to adopt the equatorial conformation. Other electronegative substituents such as Cl, SR etc also participate in
H anomeric stabilization.
H ●● H
OMe
1.781 Å
O H
∆ Gc° = +0.6 kcal/mol
OMe Cl H O O
What is unexpected is that the closely related 2-methoxytetrahydropyran Cl
prefers the axial conformation: 1.819 Å Cl
This conformer
H preferred by 1.8 kcal/mol Why is axial C–Cl bond longer ?

O H axial O lone pair↔σ∗ C–Cl


OMe
O σ∗ C–Cl
OMe
∆ Gp° = –0.6 kcal/mol ●●

O H
That effect which provides the stabilization of the axial OR
conformer which overrides the inherent steric bias of the
substituent is referred to as the anomeric effect. O
Cl HOMO

Let anomeric effect = A


∆ Gp° = ∆ Gc° + A σ C–Cl
The Exo-Anomeric Effect
A = ∆ Gp° – ∆ Gc°
■ There is also a rotational bias that is imposed on the exocyclic
A = –0.6 kcal/mol – 0.6 kcal/mol = –1.2 kcal/mol C–OR bond where one of the oxygen lone pairs prevers to
be anti to the ring sigma C–O bond
Principal HOMO-LUMO interaction from each conformation is
illustrated below: H
O
H O R O
●●

OMe O O R O
O H R
O favored
●● OMe
A. J. Kirby, The Anomeric and Related Stereoelectronic Effects at Oxygen,
axial O lone pair↔
↔σ∗ C–H axial O lone pair↔
↔σ∗ C–O Springer-Verlag, 1983
E. Jurasti, G. Cuevas, The Anomeric Effect, CRC Press, 1995
2-01-Anomeric Effect-1 9/16/03 2:40 PM
D. A. Evans The Anomeric Effect: Carbonyl Groups Chem 206
Do the following valence bond resonance structures
have meaning? Aldehyde C–H Infrared Stretching Frequencies

R R Prediction: The IR C–H stretching frequency for aldehydes is lower


O C O than the closely related olefin C–H stretching frequency.
C ●● For years this observation has gone unexplained.
X X
R R R
Prediction: As X becomes more electronegative, the IR frequency C O C C
should increase H H R
-1
O
ν C–H = 2730 cm ν C–H = 3050 cm -1
O O
Sigma conjugation of the lone pair anti to the H will weaken the bond.
Me CH3 Me CBr3 Me CF3 This will result in a low frequency shift.
υC=O (cm-1) 1720 1750 1780
Infrared evidence for lone pair delocalization into
vicinal antibonding orbitals.
Prediction: As the indicated pi-bonding increases, the X–C–O The N–H stretching frequency of cis-methyl diazene is 200 cm-1 lower
bond angle should decrease. This distortion improves overlap. than the trans isomer.
Me H
Me H
N N antibonding
R R
N N σ∗ N–H
C O C O filled
N-SP2
ν N–H = 2188 cm -1
X
X Me
Me antibonding
σ* C–X →O lone pair N N σ∗ N–H
N N

Evidence for this distortion has been obtained by X-ray crystallography


H filled .. H
N-SP2
ν N–H = 2317 cm -1
Corey, Tetrahedron Lett. 1992, 33, 7103-7106 ■ The low-frequency shift of the cis isomer is a result of N–H bond
weakening due to the anti lone pair on the adjacent (vicinal) nitrogen
which is interacting with the N–H antibonding orbital. Note that the
orbital overlap is not nearly as good from the trans isomer.
N. C. Craig & co-workers JACS 1979, 101, 2480.
2-02-Anomeric Effect-2 9/16/03 2:41 PM
D. A. Evans The Anomeric Effect: Nitrogen-Based Systems Chem 206
Observation: C–H bonds anti-periplanar to nitrogen lone pairs are
spectroscopically distinct from their equatorial C–H bond counterparts CMe3
Me3C N N
N CMe3 Me3C N N
σ∗ C–H N
Me3C
H N Me3C
H
H ∆G° = – 0.35kcal/mol
H
H N A. R. Katritzky et. al., J. Chemm. Soc. B 1970 135
HOMO

Favored Solution Structure (NMR)


σ C–H
Me
MeN NMe N N
Me
Spectroscopic Evidence for Conjugation MeN NMe N N Me
Me
Infrared Bohlmann Bands
J. E. Anderson, J. D. Roberts, JACS 1967 96 4186
Characteristic bands in the IR between 2700
and 2800 cm-1 for C-H4, C-H6 , & C-H10 stretch
Bohlmann, Ber. 1958 91 2157 Favored Solid State Structure (X-ray crystallography)
Reviews: McKean, Chem Soc. Rev. 1978 7 399
L. J. Bellamy, D. W. Mayo, J. Phys. 1.484
1.453 Bn
Chem. 1976 80 1271
N N
NMR : Shielding of H antiperiplanar to N lone pair Me
Me
H10 (axial): shifted furthest upfield N N
1.453
H6, H4: ∆δ = δ Haxial - δ H equatorial = -0.93 ppm 1.459 1.457
Bn
Protonation on nitrogen reduces ∆δ to -0.5ppm

A. R. Katrizky et. al., J. C. S. Perkin II 1980 1733


H. P. Hamlow et. al., Tet. Lett. 1964 2553
J. B. Lambert et. al., JACS 1967 89 3761

2-03-Anomeric Effect-3 9/16/03 2:43 PM


D. A. Evans Anomeric Effects in DNA Phosphodiesters Chem 206
Calculated Structure of ACG–TGC Duplex The Phospho-Diesters Excised from Crystal Structure

Guanine

Cytosine 1B

Cytosine

2B

1A

Phosphate-1A Phosphate-1B
p.
p. Thymine

Adenine
The Anomeric Effect
Acceptor orbital hierarchy: δ* P–OR * > δ* P–O–

R R

O δ– O
O
δ– O O
P O P
R R

δ– O
δ O
Gauche-Gauche conformation

O Phosphate-2A Phosphate-2B
δ– O R δ– O R O
P O O
P
R R Oxygen lone pairs may establish a simultaneous hyperconjugative
δ– O δ– O relationship with both acceptor orbitals only in the illustrated
Anti-Anti conformation conformation.
Plavec, et al. (1996). “How do the Energetics of the Stereoelectronic Gauche &
Gauche-Gauche conformation affords a better donor-acceptor relationship Anomeric Effects Modulate the Conformation of Nucleos(t)ides?
” Pure Appl. Chem. 68: 2137-44.

2-04-DNA Duplex/Anomeric 9/17/03 9:25 AM


D. A. Evans Carboxylic Acids (& Esters): Anomeric Effects Again? Chem 206

■ Conformations: There are 2 planar conformations. ■ Hyperconjugation: Let us now focus on the oxygen lone pair in the hybrid
orbital lying in the sigma framework of the C=O plane.
O O (Z) Conformer
(Z) Conformer R' (E) Conformer
R O R O R σ* C–O
C O In the (Z) conformation this
O O R' •• O lone pair is aligned to overlap O
Specific Case:
Methyl Formate Me R with σ* C–O.
H O H O ∆G° = +4.8 kcal/mol R
R O
Me
The (E) conformation of both acids and esters is less stable by 3-5 kcal/mol. If (E) Conformer
this equilibrium were governed only by steric effects one would predict that the
(E) conformation of formic acid would be more stable (H smaller than =O). R
C O In the (E) conformation this
Since this is not the case, there are electronic effects which must also be R O lone pair is aligned to overlap
considered. These effects will be introduced shortly.
•• with σ* C–R. σ* C–R
■ Rotational Barriers: There is hindered rotation about the =C–OR bond. O
R
These resonance structures suggest O C Since σ* C–O is a better acceptor than σ* C–R O
hindered rotation about =C–OR bond. O R
barrier ~ 10-12 (where R is a carbon substituent) it follows that
This is indeed observed: kcal/mol R the (Z) conformation is stabilized by this interaction. R
O O O
R' R'
R O R O R O
Energy

O
R Esters versus Lactones: Questions to Ponder. O
R
Rotational barriers are ~ 10-12 R O ∆G° ~ 2-3 1 Et
kcal/mol CH3CH2 O
kcal/mol. This is a measure of the Esters strongly prefer to adopt the (Z) conformation while
strength of the pi bond. O
small-ring lactones such as 2 are constrained to exist in the
(Z) conformation. From the preceding discussion explain the O
following: 2
■ Lone Pair Conjugation: The oxygen lone pairs conjugate with the C=O.
1) Lactone 2 is significantly more susceptible to nucleophilic
versus
attack at the carbonyl carbon than 1? Explain.
•• R The filled oxygen p-orbital interacts with pi (and pi*) 2) Lactone 2 is significantly more prone to enolization than 1?
C O In fact the pKa of 2 is ~25 while ester 1 is ~30 (DMSO). Explain.
•• O C=O to form a 3-centered 4-electron bonding system.
R 3) In 1985 Burgi, on carefully studying O O O β
α β α β α
the X-ray structures of a number of
SP2 Hybridization lactones, noted that the O-C-C (α) & O O
O
O-C-O (β) bond angles were not equal.
■ Oxygen Hybridization: Note that the alkyl oxygen is Sp2. Rehybridization Explain the indicated trend in bond
is driven by system to optimize pi-bonding. angle changes. α−β = 12.3 ° α−β = 6.9 ° α−β = 4.5 °

2-05 RCO2R Bonding 9/16/03 2:50 PM


D. A. Evans Three-Center Bonds Chem 206

Consider the linear combination of three atomic orbitals. The resulting


molecular orbitals (MOs) usually consist of one bonding, one nonbonding Case 3: 2 p-Orbitals; 1 s-orbital
and one antibonding MO.
antibonding
Case 1: 3 p-Orbitals
pi-orientation antibonding 2
+ nonbonding
Energy

3 nonbonding bonding

Case 4: 2 s-Orbitals; 1 p-orbital Do this as an exercise


bonding

Note that the more nodes there are in the wave function, the higher its energy. Examples of three-center bonds in organic chemistry
+ A. H-bonds: (3–center, 4–electron)
H 2C CH CH2 Allyl carbonium ion: both pi-electrons in bonding state
O H O
● The acetic acid dimer is
H 2C CH CH2 Allyl Radical: 2 electrons in bonding obital plus one in CH3 CH3
stabilized by ca 15 kcal/mol
nonbonding MO. O H O
– Allyl Carbanion: 2 electrons in bonding obital plus 2 in
H 2C CH CH2 nonbonding MO.
B. H-B-H bonds: (3-center, 2 electron)

Case 2: 3 p-Orbitals H H H H H
B B B H H B
sigma-orientation H H H H
antibonding H
diborane stabilized by 35 kcal/mol
3
Energy

nonbonding C. The SN2 Transition state: (3–center, 4–electron)


The SN2 transition state approximates a case 2
H situation with a central carbon p-orbital
The three orbitals in reactant molecules used are:
Nu C Br 1 nonbonding MO from Nucleophile (2 electrons)
bonding 1 bonding MO σ C–Br (2 electrons)
H H
1 antibonding MO σ* C–Br
2-06 3-center bonds/review 10/28/03 12:00 PM
D. A. Evans Substitution Reactions: General Considerations Chem 206

Why do SN2 Reactions proceed with backside displacement? Electrophilic substitution at saturated carbon may occur
with either inversion of retention

R R R
δ– δ– Inversion
Nu: – C X Nu C X Nu C – ‡
H H X: Ra Ra Ra
H H H δ+ δ+
H El(+) C M Nu C M Nu C M+
H H
Rb H Rb
Given the fact that the LUMO on the electrophile is the C–X antibonding Rb
orblital, Nucleophilic attack could occur with either inversion or retention.
Retention
Inversion Retention δ+ ‡
Ra Ra M Ra
R R
El(+) H
C M
H
C
H
C El M+
Nu ●● C X C X Rb Rb El δ+
Rb
H H ●●
H H
HOMO LUMO Ra
Nu
Constructive overlap between Overlap from this geometry results Ra C ●●
M
Nu & σ*C–X in no net bonding interaction H
El(+) C ●●
M Rb
H
LUMO Rb HOMO
Expanded view of σ*C–X
El(+)
Inversion
Retention
LUMO C X
Examples
antibonding bonding

●●
HOMO Br2 CO2
H Li CO2Li

Nu Br H H
predominant inversion predominant retention
Fleming, page 75-76
Stereochemistry frequently determined by electrophile structure
See A. Basu, Angew. Chem. Int. Ed. 2002, 41, 717-738
2-07-SN2-1 9/18/03 12:38 PM
D. A. Evans SN2 Reaction: Stereoelectronic Effects Chem 206
The reaction under discussion: The use of isotope labels to probe mechanism.

R R 1 and 2 containing deuterium labels either on the aromatic ring or on the methyl
R
δ– δ– group were prepared. A 1:1-mixture of 1 and 2 were allowed to react.
Nu: – C X Nu C X Nu C X: –
H H
■ If the rxn was exclusively intramolecular, the products would only contain
H H H
H only three deuterium atoms:
O O
■ The Nu–C–X bonding interaction is that of a 3-center, 4-electron bond. The SO3–
S CH3
frontier orbitals which are involved are the nonbonding orbital from Nu as well as O (CD3–Ar–Nu–CH3)
σC–X and σ∗C–X: exclusively CH3
– intramolecular D 3C Nu
σ∗C–X D 3C Nu:
1
O O

S CD3 SO3
O (CH3–Ar–Nu–CD3
exclusively
energy

– CD3
Nu: – Me Nu: intramolecular H 3C Nu
2
σC–X ■ If the reaction was exclusively intermolecular, products would only contain
δ– δ–
Nu C X RCH2–X differing amounts of D-label depending on which two partners underwent reaction.
The deuterium content might be analyzed by mass spectrometry. Here are the
■ Experiments have been designed to probe inherent requirement for achieving possibilities:
1 + 1 D3-product 2 CD3–Ar–Nu–CH3
a 180 ° Nu–C–X bond angle: Here both Nu and leaving group are constrained to
be part of the same ring. 2 + 2 D'3-product 2 CH3–Ar–Nu–CD3
R D6-product1 CD3–Ar–Nu–CD3
R 1 + 2
δ– δ–
– D0-product1 CH3–Ar–Nu–CH3
Nu: C X Nu C X
H
Hence, for the strictly intermolecular situation one should see the following ratios
H H H
D0 : D3 : D'3 : D6 = 1 : 2 : 2 : 1.
The product isotope distribution in the Eschenmoser expt was found to be
"tethered reactants" "constrained transition state" exclusively that derived from the intermolecular pathway!

The Eschenmoser Experiment (1970): Helv. Chim Acta 1970, 53, 2059 Other Cases:
+ –
exclusively SO3CH3 SO3
■ The reaction illustrated below proceeds exclusively through bimolecular pathway (CH3)2N (CH3)3N
intermolecular
in contrast to the apparent availability of the intramolecular path.
O O
– SO3CH3 –
S CH3 SO3 16% intramolecular SO3
O 84% intermolecular +
N(CH3)2 N(CH3)3
CH3
Nu:– Nu Hence, the Nu–C–X 180 ° transition state bond angle must be rigidly
maintained for the reaction to take place.
2-08-The SN2 RXN-FMO 9/16/03 2:56 PM
D. A. Evans Intramolecular methyl transfer: Speculation on the transition structures Chem 206

SO3CH3 – + –
SO3 SO3CH3 SO3
(CH3)2N (CH3)3N
N(CH3)2
+
N(CH3)3

16% intramolecular; 84% intermolecular exclusively intermolecular

00000 9- membered cyclic transition state 8- membered cyclic transition state


00000
00000
00000
00000

000000
000000
000000
000000
000000
est C–O bond est C–O bond
length 2.1 Å length 2.1 Å

174°
174°
00
00
00
00

est C–N bond


00000
00000
00000
00000 000000

length 2.1 Å est C–N bond


00000
00000 0000

length 2.1 Å
000000
0000
0000
00
00
00
00

Approximate representation of the transition states of the


intramolecular alkylation reactions. Transition state C–O and C–N
bond lengths were estimated to be 1.5x(C–X) bond length of 1.4 Å

2-09-Intra alk TS's 9/16/03 2:56 PM


D. A. Evans Olefin Epoxidation via Peracids: An Introduction Chem 206
■ The General Reaction:
Per-arachidonic acid Epoxidation
R R O R R O
+ ●
OH ● + O
R O R OH Me ●
R R R R
O H
HOMO LUMO note labeled oxygen is transferfed
πC–C σ*O–O O-O bond energy: ~35 kcal/mol

■ Reaction rates are governed by olefin nucleophilicity. The rates of


epoxidation of the indicated olefin relative to cyclohexene are provided
below: OH
OH OAc

1.0 0.6 0.05 0.4


■ The indicated olefin in each of the diolefinic substrates may be oxidized
selectively.
Me Me Me

Me

Me
H
Me Me Me Me

■ The transition state:

O
Me H
● O

E. J. Corey, JACS 101, 1586 (1979)


For a more detailed study see P. Beak, JACS 113, 6281 (1991)

View from below olefin For theoretical studies of TS see R. D. Bach, JACS 1991, 113, 2338
R. D. Bach, J. Org. Chem 2000, 65, 6715
2-10 Epoxidation-1 9/16/03 2:58 PM
D. A. Evans Olefin Epoxidation with Dioxiranes Chem 206
■ The General Reaction: Asymmetric Epoxidation with Chiral Ketones
R R R R Review: Frohn & Shi, Syn Lett 2000, 1979-2000
R O O
+ ● +
R ●
O R R
Me
Me
R R R R O
O O
HOMO LUMO note labeled oxygen is transferfed chiral catalyst
πC–C σ*O–O O-O bond energy: ~35 kcal/mol O
O
Me O
R2 O R2
■ Synthesis of the Dioxirane Oxidant Me

H R1 R2 oxone, CH3CN-H2O R1 R2
O R
pH 7-8
O
O K+ O– R O Me
S O R O
O O H R ●
O Ph Me Ph
R R SO3 Ph Ph Ph
(Oxone)
>95% ee 84% ee 92% ee
Synthetically Useful Dioxirane Synthesis
Question: First hour Exam 2000 (Database Problem 34)
O oxone O O co-distill to give Question 4. (15 points). The useful epoxidation reagent dimethyldioxirane (1) may be
~0.1 M soln of prepared from "oxone" (KO3SOOH) and acetone (eq 1). In an extension of this epoxidation
Me Me Me Me dioxirane in acetone concept, Shi has described a family of chiral fructose-derived ketones such as 2 that, in the
presence of "oxone", mediate the asymmetric epoxidation of di- and tri-substituted olefins
O with excellent enantioselectivities (>90% ee) (JACS 1997, 119, 11224).
oxone O O co-distill to give
~0.6 M soln of dioxirane
F 3C CF3 in hexafluoroacetone Me
F 3C CF3 Me Me Me
KO3SOOH O
O O
Curci, JOC, 1980, 4758 & 1988, 3890; (1) O O
CH3CN-H2O O
JACS 1991, 7654. Me pH 10.5 Me 2
1
Transition State for the Dioxirane Mediated Olefin Epoxidation R2 O R2 O
1 equiv 2 O
(2) O
oxone, Me
R1 R2 R1 R2
O R O R CH3CN-H2O Me
pH 10.5 >90% ee
planar R R
O O
spiro
Part A (8 points). Provide a mechanism for the epoxidation of ethylene with
rotate 90° dimethyldioxirane (1). Use three-dimensional representations, where relevant, to illustrate
the relative stereochemical aspects of the oxygen transfer step. Clearly identify the frontier
orbitals involved in the epoxidation.

stabilizing Olp → π* C=C Part B (7 points). Now superimpose chiral ketone 2 on to your mechanism proposed
cis olefins react ~10 times faster than trans above and rationalize the sense of asymmetric induction of the epoxidation of trisubstituted
olefins (eq 2). Use three-dimensional representations, where relevant, to illustrate the
Houk, JACS, 1997, 12982. absolute stereochemical aspects of the oxygen transfer step.

2-11 Epoxidation-2 9/16/03 3:01 PM


D. A. Evans The Baeyer-Villiger Reaction: Stereoelectronic Effects Chem 206
Let RL and RS be Sterically large and small substituents.
CMe3
O O O Me
+ RCO3H O O Me O
Me3C
C RL C + C RS + RCO3H O - MeCO2H
RL RS – RCO2H O RS RL O O
Me CMe3 O O
major minor R
H O H
O R
The major product is that wherein oxygen has been inserted into
theRL–Carbonyl bond.
kR O R kR / KMe Favored Migrating group
R C H
O Me CMe3
CH3CH2 72
O major O
O
150 CMe3
C + CF3CO3H CH3(CH2)2 Me OH Me O
R Me O
O
O (CH3)3C 830
R
kMe C Me
R O >2000 Conformer A
PhCH2
minor
H
RS OH Disfavored Migrating group

The Intermediate C O R Me
RL O O
O
O Me3C OH Me
O CMe3
O
O
The important stereoelectronic components to this rearrangement:
R
1. The RL–C–O–O dihedral angle must be180° due to the HOMO The destabilizing
Conformer B gauche interaction
LUMO interaction σ-RL–C↔σ∗−O–O.

2. The C–O–O–C' dihedral angle will be ca. 60° due to the gauche Steric effects destabilize Conformer B relative to Conformer A;
effect (O-lone pairs↔σ∗−C–O). hence, the reaction is thought to proceed via a transition
state similar to A.
This gauche geometry is probably reinforced by intramolecular
For relevant papers see:
hydrogen bonding as illustrated on the opposite page:
Crudden, Angew. Chem. Int. Ed 2000, 39, 2852-2855 (pdf)
Kishi, JACS 1998, 120, 9392 (pdf)
2-12- Baeyer Villiger Rxn 9/16/03 5:33 PM
D. A. Evans The Baeyer-Villiger Reaction: Stereoelectronic Effects Chem 206

CMe3 Conformer A in three dimensions


Me
O O Me O
Me3C
+ RCO3H O - MeCO2H
O
Me CMe3 O O
H O R H
O R

Favored Migrating group


H
CMe3
O
O
Me OH CMe3
Me O
O
O

R
Conformer A
H
Disfavored Migrating group

Me

O
O 1
Me3C OH Me
O CMe3
O
O 2
3

R
The destabilizing
Conformer B gauche interaction 4

Steric effects destabilize Conformer B relative to Conformer A;


hence, the reaction is thought to proceed via a transition 2–3 dihedral angle ~ 178° from Chem 3D
state similar to A.
For relevant papers see:
Crudden, Angew. Chem. Int. Ed 2000, 39, 2852-2855 (pdf)
Kishi, JACS 1998, 120, 9392 (pdf)
2-13- Baeyer Villiger Rxn-2 9/16/03 5:41 PM
B. Breit FMO-Theory/HSAB Principle 1 Chem 206

Hard and Soft Acids and Bases (HSAB-Principle)


FMO-Theory and Klopman-Salem equation provide an understanding of this empirical
Reading Assignment: Fleming, Chapter 3, p33-46 principle:
Pearson, JACS 1963, 85, 3533.
Hard Acids have usually a positive charge, small ion radii (high charge density), energy rich
Hard Acids prefer to interact with hard bases (high lying) LUMO.
Soft acids prefer to interact with soft bases. Soft Acids are usually uncharged and large (low charge density), they have an energy poor
(low lying ) LUMO (usually with large MO coefficient).
Softness: Polarizability; soft nucleophiles have electron clouds, which can be Hard Bases usually have a negative charge, small ion radii (high charge density), energy
polarized (deformed) easily. poor (low lying) HOMO.
Soft Bases are usually uncharged and large (low charge density), energy rich (high lying)
Hardness: Charged species with small ion radii, high charge density. HOMO (usually with large MO coefficient).

Qualitative scaling possible:


Molecular Orbital Energies of an
idealized Soft Species idealized Hard Species

E
E

small
HOMO/LUMO gap large HOMO/LUMO gap

FMO-Theory for interaction:

Soft-Soft Hard-Hard

E
E

Acid Base

Acid Base

Significant Energy gain Only neglectable


through HOMO/LUMO energy gain through
interaction orbital interaction.

2-14-FMO HSAB 1 9/20/00 8:30 AM


B. Breit FMO-Theory/HSAB Principle 2 Chem 206

Klopman-Salem Equation for the interaction of a Nucleophile N


(Lewis-Base) and an Electrophile E (Lewis-Acid).

QNQE 2(cNcEβ)2
∆E =
εRNE EHOMO(N) - ELUMO(E)

Coulomb Term Frontier Orbital Term

Q: Charge density
ε: Dielectricity constant
R: distance (N-E)
c: coefficient of MO
β: Resonance Integral
E: Energy of MO

Soft-Soft Interactions: Coulomb term small (low charge


density). Dominant interaction is the frontier orbital interaction
because of a small ∆E(HOMON/LUMOE).
⇒ formation of covalent bonds

Hard-Hard Interactions: Frontier orbital term small because of


large ∆E(HOMON/LUMOE). Dominant interaction is described
by the Coulomb term (Q is large for hard species), i.e.
electrostatic interaction.
⇒ formation of ionic bonds

Hard-Soft Interactions: Neither energy term provides


significant energy gain through interaction. Hence, Hard-Soft
interactions are unfavorable.

2-15-FMO HSAB 2 9/20/00 8:27 AM


B. Breit FMO-Theory/HSAB Principle 3 Chem 206

HSAB principle - Application to Chemoselectivity Issues


(c) SN2 vs E2
(a) Enolate Alkylation
CO2R
soft
O CO2R
soft HC(COOR)2 S N2
hard C-Alkylation
MeI H
O Me
soft C Br hard
C E2
OTMS
hard OC2H5
O-Alkylation
TMSCl

(d) Ambident Nucleophiles


(b) 1,2- vs. 1,4-addition to α,β-unsaturated carbonyl compounds

+ 0.29 - 0.48 soft


soft H3 C S C N S-Alkylation
O O Ag MeI

S C N O
H H
+ 0.01 + 0.62 Na hard
hard S C N R N-Acylation
Charge density LUMO-coefficients RCOX

OH
hard soft
hard Me H3 C NO2
MeLi MeI
N-Alkylation
soft O Ag
O
O N
1,2-Addition
Na hard
ONO O-Alkylation
hard soft t-BuCl
soft Me O

Me2CuLi

Conjugate Addition

2-16-FMO HSAB 3 9/20/00 8:27 AM


D. A. Evans Rules for Ring Closure: Introduction Chem 206

http://www.courses.fas.harvard.edu/~chem206/ The Primary Literature


Baldwin, J. Chem. Soc., Chem. Comm. 1976, 734, 736.
Baldwin, J. Chem. Soc., Chem. Comm. 1977 233.
Chemistry 206 Baldwin, J. Org. Chem. 1977, 42, 3846.
Baldwin, Tetrahedron 1982, 38, 2939.
Advanced Organic Chemistry

Lecture Number 3

Stereoelectronic Effects-3

"Rules for Ring Closure: Baldwin's Rules"


Kirby, "Stereoelectronic Effects" Chapters 4, 5

Useful LIterature Reviews


Johnson, C. D. (1993). “Stereoelectronic effects in the formation of 5- ■ Problems of the Day
and 6-membered rings: the role of Baldwin's rules.”
Acc. Chem. Res. 26: 476-82. (Handout) Propose mechanisms for the following reactions
Beak, P. (1992). “Determinations of transition-state geometries by the
endocyclic restriction test: mechanisms of substitution at R HO H+ R O
nonstereogenic atoms.” Acc. Chem. Res. 25: 215. (Handout) O +
R HO R O

Friday, O
D. A. Evans September 19, 2003 NH2–NH2 HN NH
Me OMe Me O
3-00-Cover Page 9/19/03 8:36 AM
D. A. Evans, J. Johnson Rules for Ring Closure: Introduction Chem 206

Ring Closure and Stereoelectronic Connsiderations C. Nucleophilic ring closures sub-classified according to hybridization
An Examination of Baldwin's Rules state of electrophilic component:
(tetrahedral = tet; trigonal = trig; digonal = dig)
"Baldwin's Rules" provides a qualitative set of generalizations on the
probability of a given ring closure. D. Nucleophilic ring closures further subclassified according to size of
the fomed ring. For example:
There are circumstances where the "rules" don't apply.

■ They do not apply to non-first-row elements participating in the X 5-exo-tet


X
cyclization event. The longer bond lengths and larger atomic radii of Y Y–
2nd row elements result in relaxed geometrical constraints.

For example, a change in a heteroatom from O to S could result in 5-exo-trig


relaxation of a given geometric constraint. X X
Y Y–
endo
X = O vs S 5-exo-dig
X Y X
••

Y X
X
Y Y–
■ The "rules" do not apply to electrocyclic processes.
Nomenclature Required trajectories (Baldwin):

Classes of Ring Closing Processes α α


α = 180 ο
X Y X Y
A. Exo-cyclization modes identified by the breaking bond
being positioned exocyclic to the forming cycle.
exo
X
α X α
X X α = 109 ο
••

Y Y Y
Y
B. Endo-cyclization modes identified by the breaking bond
Will come
being positioned endocyclic to the forming cycle. back to this
X case later
α ο X
endo * α ≈ 120 * α α
X Y X α
••

Y Y Y
X = first-row element
N, O Baldwin, J. Chem. Soc., Chem. Commun., 1976, 734.
3-01-Baldwin Rules-1 9/18/03 3:38 PM
D. A. Evans, J. Johnson Rules for Ring Closure: SP3 Carbon & Related Systems Chem 206

FÜRST-PLATTNER RULE
Tetrahedral Carbon
In this simple model, the transition-state leading to 1 involves the
All exo cyclization modes are allowed: (n-exo-tet, n = 3→
→) diaxial orientation of nucleophile and leaving group. This orientation
affords the best overlap of the anti-bonding C–Y orbital and the
nonbonding electron pairs on the nucleophile O–.
exo

C X C In the formation of the diastereomeric epoxide 2, the proper alignment


X
••

– of orbitals may only be achieved by cyclization through the


Y Y
less-favored boat conformer. Accordingly, while both cyclizations are
There are stereoelectronic issues to consider for n-exo-tet cyclizations
"allowed", there are large rate differences the the rates of ring closure.
Formation of 3-Membered Rings (3-exo-tet)
While the FÜRST-PLATTNER RULE deals wilth the microscopic
H H H ‡ reverse, in the opening of epoxides by nucleophiles, the
C Y X CH2
••

X X C Y stereoelectronic arguments are the same.


+ Y–
C C H
C
H2 H2 H2 Stereoelectronic Effects in Epoxide Ring Cleavage
Nu
Conformational Effects in Epoxide Ring Formation/cleavage Nu-
Me3C Me3C H
H
O
Those stereoelectronic effects that operate in ring cleavage also
influence ring formation. Consider a rigid cyclohexene oxide system: H H HO

HO
Y δ– ‡ O
Y Nu-
faster H Me3C Me3C H
H
H
H H H H H Nu
– 1 O
O– δ O
Y δ– ‡ Me Me
Nu
H slower Nu-
O
Y
H HO
H H H
O– H – O
O δ
chair boat 2 "The diaxial nucleophilic ring cleavage of epoxides"
For more information on epoxide cleavage see Handout 03A.

3-02-Baldwin Rules-2 9/18/03 3:39 PM


D. A. Evans, J. Johnson Rules for Ring Closure: SP3 Carbon & Related Systems Chem 206

Tetrahedral Carbon Case 2: King, J.C.S. Chem. Comm., 1979, 1140.

O
Endo cyclization modes that are disallowed O
O
O
S 8-endo-tet
(n-endo-tet, n = 3→
→∼9) disfavored
S
_
O O
Me
NMe2
Rxn exclusively
NMe3+
intermolecular
endo

X X Y 8-endo-tet
••

••
SO2OMe disfavored SO3–
C C(SP3) NMe3+
NMe2
Rxn exclusively
The stereoelectronic requirement for a 180° X–C–Y bond angle is only intermolecular
met when the endo cyclization ring size reaches 9 or 10 members.
9-endo-tet SO3–
SO2OMe
Case 1: Eschenmoser, Helvetica Chim. Acta 1970, 53, 2059. borderline
NMe3+
NMe2
O O O O 84% intermolecular,
S S 16% intramolecular
O O-
NaH
CX3 CX3 Conclusions
6-endo-tet
O S O disfavored O S O Allowed endo cyclization modes will require transition state ring sizes
of at least nine members.
Rxn exclusively Intramolecular epoxidation has also been evaluated
intermolecular
(lecture 2) Beak, JACS 1991, 113, 6281.
CY3 CY3 O
Cl
8-endo-tet
Cyclization exclusively intermolecular. However the exocyclic analog disfavored Cl CO2H
O–OH
is exclusively intramolecular
O O O O n n O
S S
O -
O n = 1: rxn exclusively intermolecular
CX2I NaH CX2 n = 9: rxn is intramolecular
6-exo-tet Beak states that the conclusions made with carbon
O S O O S O
favored substitution also hold for oxygen atom transfer.
Rxn exclusively
intramolecular Beak, P. (1992). “Determinations of transition-state geometries by the
endocyclic restriction test: mechanisms of substitution at nonstereogenic
CY3 CY3 atoms.” Acc. Chem. Res. 25: 215.
3-03-Baldwin Rules-3 9/18/03 4:07 PM
D. A. Evans, J. Johnson Rules for Ring Closure: SP2 Carbon & Related Systems Chem 206

Trigonal Carbon
MeO2C CO2Me
Endo cyclization modes that are disallowed
(3 to 5-endo-trig) NH2

n-endo-trig

X Y X Y–
••

C C

X = first-row element

The 5-endo-trig cyclization is a watershed case


distance from reacting centers: 2.77 Å
Case 1: Baldwin, J. Chem. Soc., Chem. Commun., 1976, 734.

CO2Me base CO2Me


X It is possible that a "nonvertical"
OH 5-endo-trig O trajectory is operational like that
Disfavored suspected in C=O addition
however
CO2Me base CO2Me

SH S

Second row atom relaxes the cyclization geometrical requirement

Case 2: Baldwin, J. Chem. Soc., Chem. Commun., 1976, 736.

MeO2C CO2Me MeO2C CO2Me


X
NH2 HN
5-endo-trig
0%
MeO2C
5-exo-trig
HN 100%
O

3-04-Baldwin Rules-4 9/18/03 4:07 PM


D. A. Evans, J. Johnson Rules for Ring Closure: SP2 Carbon & Related Systems Chem 206

Case 2: continued... Apparent exceptions to disallowed 5-endo-trig cyclization process


MeO2C CO2Me MeO2C CO2Me
O
X
NH2 HN +
N CH3CO2H N OH N O
5-endo-trig
0%
MeO2C Filer, J. Am. Chem. Soc. 1979, 44, 285.
5-exo-trig
HN 100% CO2Me CO2Me CO2Me
O 1
R HC
t
KO Bu R1 R 1

N R2 2
Control experiment: Intermolecular reaction favors conjugate addtion. HN R HN CO2Me
CO2Me CO2Me 3:1 R2
Me Me Me
PhCH2NH2 H H R1 = aryl, R2 = aryl, alkyl
Ph N N Ph
CO2Me CO2Me Grigg, J. Chem. Soc., Chem. Commun. 1980, 648.
O 0%
100%

Case 3: Does the illustrated R (CH2OH)2 R O


O
Ph
ketalization process O
NH2NH2 O necessarily violate "the R H+ R O
Ph OMe
HN NH rules"?
65 oC

R (CH2OH)2 R R R
O Ph O OH H+ OH
1) EtO2CCl, pyridine R
O
2) NH2NH2
HO O ( )2 –H2O O ( )2
Ph OK NH R
+
H2N
MeI H+ disfavored ? 5-endo-trig
200 oC X 5-endo-trig
O
R R O
R OH 5-exo-tet
Ph OMe
Ph +
NH2NH2 CO2Me Ph O HO O ( )2 O
favored ? R
HN
HN NH
65 oC NH2
5-exo-trig
Johnson, C. D. (1993). “Stereoelectronic effects in the formation of 5- and
6-membered rings: the role of Baldwin's rules.”
Acc. Chem. Res. 26: 476-82.
3-05-Baldwin Rules-5 9/18/03 4:08 PM
D. A. Evans, J. Johnson Rules for Ring Closure: SP2 Carbon & Related Systems Chem 206
More Exceptions Bu
Zard, Org. Lett. 2002, 4, 1135 Bu
X
MeO NaH
O
Y Y
S OEt O HO DMF, 60 °C
MeO N O
N
S
ROOR X Y Cond Yield
heat
F F DMF, 60 °C, 2 h 80
O O 80% O O
F H DMF, 80 °C, 43 h 17
MeO MeO Cl Cl DMF, 60 °C, 8 h ––
O O
Br Br DMF, 60 °C, 5 h 15
MeO N MeO N
5-endo-trig Ichikawa, et al Synthesis 2002, 1917-1936, PDF on Course Website

Numerous other cases are provided in this review.


O O O O

Br H Revisiting Case 2 with Fluorines


O O
Bu3SnH
O
O AIBN O MeO2C
MeO2C MeO2C CO2Me
H OMe
82% N
5-exo-trig TsHN N
Ts O 5-endo-trig
Ts
Favored Not Observed
H
O O
5-endo-trig O
O MeO2C CF2
O MeO2C MeO2C CO2Me
OMe
N
5-exo-trig TsHN CF2 N
Ts O 5-endo-trig
Ts F
Chem. Comm 2088, 28
Not Observed Favored
Review: "5-Endo-Trig Radical Cyclizatons" Ishibashi, et al Synthesis 2002,
695-713, PDF on Course Website

3-06-Baldwin Rules-6 9/18/03 5:10 PM


D. A. Evans, J. Johnson Rules for Ring Closure: SP2 Carbon & Related Systems Chem 206

Trigonal Carbon: Exocyclic Enolate Alkylation


distance between reacting
exo centers: 3.37Å

C C C C

O C O C Y-
Y Br
X
■ By definition, an exo-tet cyclization, but stereoelectronically
MO
behaves as an endo trig. O

Me Me The overlap for C-alkylation is poor due to


Br Me geometrical constraints of 5-membered ring
Me (1)
Me X Me
O
MO
only observed O
product distance between reacting
centers: 3.04Å
However:
Me Me The relaxed geometrical constraint
KOt-Bu or LDA
provided by the added CH2 group
Me Me > 95% by NMR now renders the 6-membered
O Me Br O cyclization possible

Baldwin, J. Chem. Soc., Chem. Commun. 1977, 233.

■ Given the failure of the enolate alkylation shown above (eq 1),
explain why these two cyclizations are successful. MO Br O

Br
base
NHAr N
O O Ar
Favorskii Rearrangement (Carey, Pt B, pp 609-610)
R Your thoughts on the mechanism
R R O
O R O CO2Me
base
N Cl MeO– MeO –
NH OMs O
Ar Ar
–HCl

3-07-Baldwin Rules-7 9/18/03 4:09 PM


D. A. Evans, J. Johnson Rules for Ring Closure: SP2 & SP Carbon & Related Systems Chem 206

Trigonal Carbon: Intramolecular Aldol Condensations Digonal Carbon: Cyclizations on to Acetylenes


Baldwin, Tetrahedron 1982, 38, 2939
DIGONAL: Angle of approach for attack on triple bonds

(Enolendo)-Exo-trig Baldwin:
MO X
O
X Nu-
YM
R Y R 120°
- 3 and 4-Exo-dig are disfavored
Favored: 6-7-(enolendo)-exo-trig - 5 to 7-Exo-dig are favored
Disfavored: 3-5-(enolendo)-exo-trig 120°
- 3 to 7-Endo-dig are favored
E+

X
(Enolexo)-exo-trig X
Ab initio SCF 4-31G calculations for the interaction of
MO O YM
Y
hydride with acetylene:
R R
_
H
Favored: 3-7-(enolexo)-exo-trig H
2.13 127 o 148o H 4-31G basis set
H H C C
Houk, J.ACS.1979, 101, 1340.
O 5-(Enolendo)-Exo-trig 6-(Enolendo)-Exo-trig H 156o 1.22
Me Me O
Me O
O H STO-3G minimal basis set
1.5-2.0 110o -120o
I Me
favored H C C H Dunitz, Helv Chim. Acta
Me 1978, 61, 2538.
Me O O Me O
Me Me
III
O Crystal Structures do not support Baldwin
O Statistical Distribution, (I + II)/III = 2:1
II
Experimental Distribution, = 0:100
N
N 2.92
(KOH, MeOH, r.t., 5 min, 77% y.) 104o
O- N + O N
2.44
+ o
N 93 o
86
Caution: Baldwin's conclusions assume that the RDS is ring closure; N
however, it is well known (by some!) that the rate determining step is
dehydration in a base-catalyzed aldol condensation.
J. Dunitz and J. Wallis J. C. S. Chem. Comm. 1984, 671.

3-08-Baldwin Rules-8 9/18/03 5:49 PM


D. A. Evans, J. Johnson Rules for Ring Closure: SP Carbon & Related Systems Chem 206

■ Indole synthesis:
Endo Digonal versus Endo Trigonal Cyclizations
CH3 CH2R
2 equiv. LDA

5-endo-trig + 2 equiv. RX
N - -78 oC
+
N
C C-
R = Me, Bu, CO2Me
Y
LiTMP
X:
R
In-plane approach; Out-of-plane approach;
nucleophile lone pair is nucleophile lone pair can't Saegusa, J. Am. Chem. Soc. 1977, 99, 3532. _
orthogonal to π* achieve Bürgi-Dunitz angle
N Li+
■ Spiro dihydrofuranones:
5-endo-dig
Li
O
HO O
:X MeO OMe KOtBu
Y Allowed due to in-plane pi orbitals OMe

n
X n = 1,2
n n

For an opposing viewpoint to Baldwin's view of nucleophile trajectories, see Developing negative charge on the central allenic carbon is
Menger's article on directionality in solution organic chemistry: in the same plane as the OMe group
Tetrahedron 1983, 39, 1013.
Magnus, J. Am. Chem. Soc. 1978, 100, 7746.
O O
Me
NaOMe Me
Me
MeOH Me
HO Ph O Ph
5-endo-dig
5-exo-dig
O O
NaOMe Li
R X R Ph Li
OH 5-endo-trig O
Ph
R = H, OMe Ph
however, the acid catalyzed version does cyclize 4-endo-dig Li
Baldwin, J. Chem. Soc., Chem. Commun., 1976, 736. X
Johnson, Can. J. Chem. 1990, 68, 1780 Ph
Li
J. Am. Chem. Soc. 1983, 105, 5090
J. Chem. Soc., Chem. Commun. 1982, 36.

3-09-Baldwin Rules-9 9/19/03 8:38 AM


D. A. Evans, J. Johnson Rules for Ring Closure: SP Carbon & Related Systems Chem 206

Digonal Cyclizations: Interesting Examples

MeO2C Et3N, Toluene, reflux ■ Trost, J. Am. Chem. Soc., 1979, 101, 1284.
CN
12 h, 65-70% y.
CN Proposes E-olefin geometry, E/Z > 95:5
5-exo-dig
O CO2Me R 30-40 kcal/mol R
O

:
:
R' R'
?
OH
H

O
HO2C
H Conclusions and Caveats
Hirsutic Acid C

■ Baldwin's Rules are an effective first line of analysis in


OTBS evaluating the stereoelectronics of a given ring closure
LiCH2NC;
O TBS-Cl OTBS 1) RCOCl
2) AgBF4
■ Baldwin's Rules have provided an important foundation for the
71% Me study of reaction mechanism
Me Me 86% N+
+
N C
R
■ Competition studies between different modes of cyclization only
C- give information about relative rates, and are
O not an absolute indicator of whether a process is "favored" or
"disfavored"
Works for varying ring sizes and R groups; acylnitrilium
ion can also work as an electophile in a Friedel-Crafts ■ Structural modifications can dramatically affect the cyclization
type of reaction 5-endo-dig mode; beware of imines and epoxides
EXO ENDO
■ Livinghouse, Tetrahedron 1992, 48, 2209.
Tet Trig Dig Tet Trig Dig

O
3 √ √ X X √
4 √ √ X X √
H Me
R
5 √ √ √ X X √
N 6 √ √ √ X √ √
O 7 √ √ √ X √ √

3-10-Baldwin Rules-10 9/18/03 5:21 PM


D. A. Evans Acyclic Conformational Analysis-1 Chem 206

http://www.courses.fas.harvard.edu/~chem206/ Professor Frank Weinhold


Univ. of Wisconsin, Dept of Chemistry
B.A. 1962, University of Colorado, Boulder

Chemistry 206 A.M. 1964, Harvard University


Ph.D. 1967, Harvard University
Physical and Theoretical Chemistry.
Advanced Organic Chemistry

Lecture Number 4 Useful LIterature Reviews


Eliel, E. L., S. H. Wilen, et al. (1994). Stereochemistry of Organic Compounds.
Acyclic Conformational Analysis-1 New York, Wiley.

Juaristi, E. (1991). Introduction to Stereochemistry and Conformational Analysis.


■ Ethane, Propane, Butane & Pentane Conformations
New York, Wiley.

■ Simple Alkene Conformations Juaristi, E., Ed. (1995). Conformational Behavior of Six-Membered Rings: Analysis,
Dynamics and Stereochemical Effects. (Series: Methods in Stereochemical
Analysis). Weinheim, Germany, VCH.
■ Reading Assignment for week Schweizer, W. B. (1994). Conformational Analysis. Structure Correlation, Vol
1 and 2. H. B. Burgi and J. D. Dunitz. Weinheim, Germany, V C H
A. Carey & Sundberg: Part A; Chapters 2 & 3 Verlagsgesellschaft: 369-404.

R. W. Hoffmann, Angew. Chem. Int. Ed. Engl. 2000, 39, 2054-2070 Kleinpeter, E. (1997). “Conformational Analysis of Saturated Six-Membered
Conformation Design of Open-Chain Compounds (handout) Oxygen-Containing Heterocyclic Rings.” Adv. Heterocycl. Chem. 69: 217-69.

Glass, R. R., Ed. (1988). Conformational Analysis of Medium-Sized Ring


The Ethane Barrtier Problem Heterocycles. Weinheim, VCH.
F. Weinhold, Nature 2001, 411, 539-541 Bucourt, R. (1973). “The Torsion Angle Concept in Conformational Analysis.”
"A New Twist on Molecular Shape" (handout) Top. Stereochem. 8: 159.
F. M. Bickemhaupt & E. J. Baerends, Angew. Chem. Int. Ed. 2003, 42,
4183-4188,"The Case for Steric Repulsion Causing the Staggered
Conformation in Ethane" (handout)
F. Weinhold,, Angew. Chem. Int. Ed. 2003, 42, 4188-4194,"Rebuttal of the ■ Problems of the Day
Bikelhaupt–Baerends Case for Steric Repulsion Causing the staggered O
Connformation of Ethane" (handout)
Predict the most stable conformation of the
indicated dioxospiran? O
Monday,
D. A. Evans September 22, 2003

4-00-Cover Page 9/22/03 9:01 AM


D. A. Evans Acyclic Conformational Analysis-1 Chem 206

The following discussion is intended to provide a general Ethane Rotational Barrier: The FMO View
F. Weinhold, Angew. nature 2001, 411, 539-541"A New Twist on Molecular Shape"
overview of acyclic conformational analysis
One can see from the space-filling models that the Van der Waals radii of the
hydrogens do not overlap in the eclipsed ethane conformation. This makes the
steric argument for the barrier untenable.
Ethane & Propane
One explanation for the rotational barrier in ethane is that better overlap is
possible in the staggered conformation than in the eclipsed conformation as
The conformational isomerism in these 2 structures reveals a gratifying level of shown below.
internal consistency.
In the staggered conformation there are 3 anti-periplanar C–H Bonds
H RH
H eclipsed
C
conformation H
σ* C–H σ∗ C–H
H
H LUMO
H
C C C C
p. ∆ E = +3.0 kcal mol-1 (R = H) Van derWaals radii of vicinal hydrogens σ C–H
p. do not overlap in ethane
∆ E = +3.4 kcal mol-1 (R = Me) H HOMO
H σ C–H

R In the eclipsed conformation there are 3 syn-periplanar C–H Bonds


H H
C staggered
conformation H H σ∗ C–H
H H
H H
H C C σ C–H σ* C–H
HOMO LUMO
In propane there is a discernable
C C
interaction σ C–H

Following this argument one might conclude that:


For purposes of analysis, each eclipsed conformer may be broken up into its
component destabilizing interactions. ■ The staggered conformer has a better orbital match between bonding and
antibonding states.
Incremental Contributions to the Barrier.
Eclipsed atoms δ E (kcal mol -1) ■ The staggered conformer can form more delocalized molecular orbitals.
Structure
J. P. Lowe was the first to propose this explanation
ethane 3 (H↔H) +1.0 kcal mol -1 "A Simple Molecuar Orbital Explanation for the Barrier to Internal
Rotation in Ethane and Other Molecules"
propane 2 (H↔H) +1.0 kcal mol -1 J. P. Lowe, JACS 1970, 92, 3799
1 (H↔Me) +1.4 kcal mol -1 Me Calculate the the rotational barrier about the C1-C2
Me bond in isobutane
4-01-introduction 9/22/03 8:28 AM Me
D. A. Evans Acyclic Conformational Analysis: Butane Chem 206

The 1,2-Dihaloethanes Butane


X H
H H
Using the eclipsing interactions extracted from propane & ethane we should be
H H X = Cl; ∆H° = + 0.9–1.3 kcal/mol able to estimate all but one of the eclipsed butane conformations
C C X = Br; ∆H° = + 1.4–1.8 kcal/mol
H H H X X = F; ∆H° = – 0.6-0.9 kcal/mol
X X Me Me
H H H H
staggered H eclipsed
Observation: While the anti conformers are favored for X = Cl, Br, the gauche C C conformation
conformation H H
conformation is prefered for 1,2-difluroethane. Explain. ∆E=?
Me MeH
Discuss with class the origin of the gauche stabiliation of the difluoro anaolg.

Recent Article: Chem. Commun 2002, 1226-1227 (handout) Eclipsed atoms δ E (kcal mol -1)

1 (H↔H) +1.0 kcal mol -1


Relationship between ∆G and Keq and pKa 2 (H↔Me) +2.8 kcal mol -1

p. ∆ E est = 3.8 kcal mol -1


p. Recall that: ∆ G° = –RT Ln K
or
The estimated value of +3.8 agrees quite well with the value of +3.6 reported
∆ G˚ = –2.3RT Log10K by Allinger (J. Comp. Chem. 1980, 1, 181-184)

At 298 K: 2.3RT = 1.4 (∆G in kcal Mol–1 )

n-Butane Torsional Energy Profile


∆ G˚298 = –1.4 Log10Keq
H HH
Since pKeq = – Log10Keq H
E2 C
E1
H Me H Me Me
∆ G˚298 = 1.4 pKeq H
C
energy

H
MeH H H
Me
Hence, pK is proportional to the free energy change H H C
H +5.1
C Me
Keq pKeq ∆G˚ H H +3.6 Me G
Me A
1.0 0 0 +0.88
Ref = 0
10 –1 –1.4
100 –2 –2.8 kcal /mol
4-02-introduction-2 9/22/03 8:33 AM
D. A. Evans Acyclic Conformational Analysis: Butane Chem 206
Butane continued Me Me Me
H H H Me Me H
From the torsional energy profile established by Allinger, we should be able to Nomenclature for C
C C
extract the contribution of the Me↔Me eclipsing interaction to the barrier: staggered conformers: H H H H
H H
Me H H
H Me trans or t gauche(+) gauche(-)
H Me H Me
staggered C
H
C
eclipsed or (anti) or g+ or g-
conformation Me H conformation Conformer population
at 298 K: 70% 15% 15%
H HH
∆ E = +5.1 kcal mol-1 General nomenclature for diastereomers resulting from rotation about a
single bond (Klyne, Prelog, Experientia 1960, 16, 521.)
Let's extract out the magnitide of the Me–Me interaction
RR

2 (H↔H) + 1 (Me↔Me) = +5.1 C


1 (Me↔Me) = +5.1 – 2 (H↔H)

1 (Me↔
↔Me) = +3.1 R R
R R
p.
p. C C
-60° sp +60°
Incremental Contributions to the Barrier. sc sc
Eclipsed atoms δ E (kcal mol -1)
2 (H↔H) +2.0 ac ac
+3.1 R -120° +120° R
1 (Me↔Me)
ap
R C C
Eclipsed Butane R
conformation
180°
From the energy profiles of ethane, propane, and n-butane, one may extract
the useful eclipsing interactions summarized below: R

C
Hierarchy of Eclipsing Interactions
X Y δ E kcal mol -1 R
X n-Butane
Y +1.0 Torsion angle Designation Symbol
H H Conformer
C C Energy Maxima 0 ± 30° ± syn periplanar ± sp E2
H H H Me +1.4 Energy Minima +60 ± 30° + syn-clinal + sc (g+) G
H H Me Me +3.1 +120 ± 30° + anti-clinal + ac E1
180 ± 30° antiperiplanar ap (anti or t) A
-120 ± 30° - anti-clinal - ac E1
-60 ± 30° - syn-clinal - sc (g-) G
4-03-butane 9/26/03 1:51 PM
D. A. Evans Acyclic Conformational Analysis: Pentane Chem 206
n-Pentane
Rotation about both the C2-C3 and C3-C4 bonds in either direction (+ or -): The double-gauche pentane
conformation
Me Me

Me H H Me
g+g- The new high-energy conformation: (g+g–)
H Me Me H
g+t
Me Me tg- Me Me

H H Me Me H H
t,t Me g-g- Estimate of 1,3-Dimethyl Eclipsing Interaction
g+g+
Me

Me Me H Me
Me H Y
X
H H g-t
tg+
g-g+
1
p.
p. 3
1 5 3 5

∆ G° = +5.5 kcal mol -1


∆ G° = X + 2Y where:
1 (t,t) 2 (g+t) X = 1,3(Me↔Me) & Y = 1,3(Me↔H)
Anti(2,3)-Anti(3,4) Gauche(2,3)-Anti(3,4) 1,3(Me↔H) = Skew-butane = 0.88 kcal mol -1
1,3(Me↔Me) = ∆ G° – 2Y = 5.5 –1.76 = + 3.7 kcal mol -1

1 1,3(Me↔Me) = + 3.7 kcal mol -1


1 5 3

Estimates of In-Plane 1,2 &1,3-Dimethyl Eclipsing Interactions


3
5 Me Me Me Me Me Me Me Me
4 (g+g–) 3 (g+g+)

Gauche(2,3)-Gauche'(3,4) Gauche(2,3)-Gauche(3,4)
double gauche pentane
3.1 ~ 3.7 ~3.9 ~ 7.6
From prior discussion, you should be able to estimate energies of 2 & 3 (relative to 1).
On the other hand, the least stable conformer 4 requires additional data before is
relative energy can be evaluated. It may be concluded that in-plane 1,3(Me↔Me) interactions are Ca +4
kcal/mol while 1,2(Me↔Me) interactions are destabliizing by Ca 2.2 kcal/mol.

4-04-pentane 9/26/03 11:23 AM


D. A. Evans Acyclic Conformational Analysis: Natural Products Chem 206

The syn-Pentane Interaction - Consequences


Lactol & Ketol Polyether Antibioitics
R. W. Hoffmann, Angew. Chem. Int. Ed. Engl. 2000, 39, 2054-2070
Conformation Design of Open-Chain Compounds (handout) The conformation of these structures are strongly influenced by the
acyclic stereocenters

Me Me Me Me
R R' R R' Me Me O OH O
≡ or
Et
Me Me Me H H Me H RR' H HO O O O
tt g -g - OH H H Me OH Et
R Me Me Et OH
R R' R Me Me R'
≡ or
Ferensimycin B, R = Me
Me H H R' H H R H Lysocellin, R = H
Me Me
tg gt

The conformation of these structures are strongly influenced by the


Consequences for the preferred conformation of polyketide natural products acyclic stereocenters and internal H-bonding
p.
p.
Analyze the conformation found in the crystal state of a bourgeanic acid derivative!
Alborixin R = Me; X-206 R = H

Internal H-Bonding

OH O Me Me Me Me
Me R
OH
Me H
OR Me
O O O O O
Me Me Me H OH OH Me
C Me OH Me OH O Me
Bourgeanic acid O OH
OH
Et

Metal ion ligation sites (M = Ag, K)

Me Me Me Me
Me R
OH
H
Me
O O O O O
H OH OH Me
C Me OH Me OH O Me
O O
OH
M Et

4-05-Natural Products 9/22/03 8:42 AM


D. A. Evans Conformational Analysis: Ionophore X-206/X-rays Chem 206

X-ray of Ionophore X-206 ⋅ H2O X-ray of Ionophore X-206 - Ag+ - Complex


Internal H-Bonding
Me Me Me Me Metal ion ligation sites (M = Ag, K)
Me
OH
H
Me Me Me Me Me
O O O Me R
O O OH
H OH OH Me H
C Me OH Me OH O Me Me
O OH O O O O O
OH H OH OH Me
Et Me OH O
C Me OH Me
O O
OH
M Et

p.
p.

"The Total Synthesis of the Polyether Antibiotic X-206". Evans, D. A.; Bender,
S. L.; Morris, J. J. Am. Chem. Soc. 1988, 110, 2506-2526.

4-06-X-206 conformation 9/22/03 8:42 AM


D. A. Evans Conformational Analysis: Ionophore X-206/X-ray overlay Chem 206

p.
p.

4-07-X-206 overlay 9/19/01 11:57 AM


D. A. Evans Stabilized Eclipsed Conformations in Simple Olefins Chem 206

Simple olefins exhibit unusal conformational Butane versus 1-Butene


properties relative to their saturated counterparts
Me
H Me H Me
Propane versus Propene staggered H Me eclipsed
C C
conformation H H conformation
109° 120°
H Me H H H
CH2 H
∆ G° = +4 kcal mol-1
H H H
H H Me
H H Me
CH2 CH2
staggered C
H eclipsed
C
Hybridilzation change opens up the C–C–C bond angle conformation H H conformation
H H H
H Φ = 50 Φ=0
■ The Propylene Barrier CH2 ∆ G° = –0.83 kcal mol-1
staggered
C conformation
H H The Torsional Energy Profile
p.
p.
H Φ = 50 Φ = 180
CH2 +2.0 kcal/mol
H Me
C H
H Me
eclipsed H H C C H
conformation H C C H H
H H
H H

Φ=0 H
H +1.33
New destabilizing effect C C H kcal
H Me Me
H +1.32 kcal
H repulsive interaction between H
π–C–X & σ–C–H C C H
H H Φ = 120
+0.49 kcal
H
X C H H
H Φ=0 Φ = 180
Conforms to ab initio (3-21G) values:
H X C H Wiberg, K. B.; Martin, E. J. Am. Chem. Soc. 1985, 107, 5035.

■ Acetaldehyde exhibits a similar conformational bias


H
K. Wiberg, JACS 1985, 107, 5035-5041 O O O O
K. Houk, JACS 1987, 109, 6591-6600 H Me H Me
H H Me Me
H H H H H H H H

The low-energy conformation in each of above cases is eclisped


4-08-simple alkenes 9/22/03 8:54 AM
Useful Destabilizing Interactions to Remember

Hierarchy of Vicinal Eclipsing Interactions


X Y δ E kcal mol -1
X Y +1.0
H H
H C C +1.4
H H Me
H H Me Me +3.1

Estimates of In-Plane 1,2 &1,3-Dimethyl Eclipsing Interactions

Me Me Me Me Me Me Me Me

~ 3.1 ~ 3.7 ~3.9 ~ 7.6

It may be concluded that in-plane 1,3(Me↔Me) interactions are Ca +4


kcal/mol while 1,2(Me↔Me) interactions are destabliizing by Ca +3 kcal/mol.

0000 0000 000 0000


0000
0000 0000
0000 000
000 0000
0000
00
00 00
00 0
0
0 00 0 minimized structure
0000
0000 00000000 0000
0000 00000000 0000
00000000 0000

4-09-Destabilizing Effects 9/20/01 5:33 PM


D. A. Evans Acyclic Conformational Analysis-2 Chem 206

■ Problems of the Day:


http://www.courses.fas.harvard.edu/~chem206/
Can you predict the stereochemical outcome of this reaction?

O OTs OLi OTs


Chemistry 206 EtO
Me
LiNR2
Me
EtO 1 + 2
n-C4H9
Advanced Organic Chemistry H n-C4H9
H 98:2

Lecture Number 5 D. Kim & Co-workers, Tetrahedron Lett. 1986, 27, 943.

Acyclic Conformational Analysis-2


■ Conformations of Simple Olefinic Substrates
OH
■ Introduction to Allylic Strain CH2OBn B2 H 6 ✻ CH2OBn
O O ✻
H2O2
Me Me Me Me
■ Introduction to Allylic Strain-2: Amides and Enolates
diastereoselection 8:1

■ Reading Assignment for week Y. Kishi & Co-workers, J. Am. Chem. Soc. 1979, 101, 259.

A. Carey & Sundberg: Part A; Chapters 2 & 3


Me Me
R. W. Hoffmann, Angew. Chem. Int. Ed. Engl. 2000, 39, 2054-2070 Me Me
Conformation Design of Open-Chain Compounds (handout) ✻ ✻
PhNCO
R. W. Hoffmann, Chem. Rev. 1989, 89, 1841-1860 Et3N O
Allylic 1-3-Strain as a Controlling Element in Stereoselective Transformations N
(handout) NO2
only one isomer
F. Weinhold, Nature 2001, 411, 539-541
"A New Twist on Molecular Shape" (handout) A. Kozikowski & Co-workers, Tetrahedron Lett. 1982, 23, 2081.

Wednesday,
D. A. Evans September 24, 2001

5-00-Cover Page 9/24/03 8:46 AM


D. A. Evans Stabilized Eclipsed Conformations in Simple Olefins Chem 206

Simple olefins exhibit unusal conformational Butane versus 1-Butene


properties relative to their saturated counterparts
Me
H Me H Me
Propane versus Propene staggered H Me eclipsed
C C
conformation H H conformation
109° 120°
H Me H H H
CH2 H
∆ G° = +4 kcal mol-1
H H H
H H Me
H H Me
CH2 CH2
staggered C
H eclipsed
C
Hybridilzation change opens up the C–C–C bond angle conformation H H conformation
H H H
H Φ = 50 Φ=0
■ The Propylene Barrier CH2 ∆ G° = –0.83 kcal mol-1
staggered
C conformation
H H The Torsional Energy Profile
p.
p.
H Φ = 50 Φ = 180
CH2 +2.0 kcal/mol
H Me
C H
H Me
eclipsed H H C C H
conformation H C C H H
H H
H H

Φ=0 H
H +1.33
New destabilizing effect C C H kcal
H Me Me
H +1.32 kcal
H repulsive interaction between H
π–C–X & σ–C–H C C H
H H Φ = 120
+0.49 kcal
H
X C H H
H Φ=0 Φ = 180
Conforms to ab initio (3-21G) values:
H X C H Wiberg, K. B.; Martin, E. J. Am. Chem. Soc. 1985, 107, 5035.

■ Acetaldehyde exhibits a similar conformational bias


H
K. Wiberg, JACS 1985, 107, 5035-5041 O O O O
K. Houk, JACS 1987, 109, 6591-6600 H Me H Me
H H Me Me
H H H H H H H H

The low-energy conformation in each of above cases is eclisped


5-01-simple alkenes 9/22/03 8:54 AM
Evans, Duffy, & Ripin Conformational Barriers to Rotation: Olefin A-1,2 Interactions Chem 206
5 5
1-butene 2-propen-1-ol

4 H H 4 H H
H H
C C H C C H
H H
Φ Φ
Me OH

E (kcal/mol)
E (kcal/mol)

3 3

2 2

1 1

0 0
-180 -90 0 90 180 -180 -90 0 90 180
Φ (Deg) Φ (Deg)

The Torsional Energy Profile The Torsional Energy Profile H


H OH
C C H
Φ = 50 Φ = 180 H
Me H H
H Me Φ = 60 Φ = 180
H C C H
C C H H HO
H H
H H H
C C H
H H
Φ = 120
Φ=0 H H OH +2.00
H H kcal
+1.33 C C H
C C H
H Me kcal Φ=0 H H
H +1.32 kcal Me H
+1.18 kcal H
H H
C C H C C H
H H H HO
+0.49 kcal Φ = 120 H +0.37 kcal
H
Φ=0 Φ = 180 Φ=0 Φ = 180
Conforms to ab initio (3-21G) values:
Wiberg, K. B.; Martin, E. J. Am. Chem. Soc. 1985, 107, 5035.
5-02-1-butene & 2-propen-1-ol 9/23/03 2:59 PM
Evans, Duffy, & Ripin Conformational Barriers to Rotation: Olefin A-1,2 Interactions-2 Chem 206
5 5

2-methyl-1-butene 2-methyl-2-propen-1-ol

4 H H 4 H H
H H
C C Me C C Me
H H
Φ Φ OH
E (kcal/mol)

Me

E (kcal/mol)
3 3

2 2

1 1

0 0
-180 -90 0 90 180 -180 -90 0 90 180
Φ (Deg) Φ (Deg)

The Torsional Energy Profile The Torsional Energy Profile


H
H Me H
Φ = 180 C C Me H OH
H Φ = 180 C C Me
H
H Φ = 60
Φ = 50 H
HO
Me
H
H C C Me
C C Me H H
H H
H Φ = 120
H
+2.68 OH +2.01
Φ = 110
Φ=0 kcal H kcal
H Me
+1.39 kcal Φ=0 +1.16 kcal C C Me
H H H
H H
C C Me C C Me H H
H Me H H C C Me
H H HO
H +0.21 kcal
+0.06 kcal H
Φ=0 Φ = 180 Φ=0 Φ = 180

5-03-methylbutene etal 9/23/03 3:00 PM


Evans, Duffy, & Ripin Conformational Barriers to Rotation: Olefin A-1,3 Interactions Chem 206
5 5
(Z)-2-pentene (Z)-2-buten-1-ol

4 4 H H
Me
C C H
H H H
Me
C C H Φ OH
E (kcal/mol)

E (kcal/mol)
3 H 3

Φ Me

2 2

1 1

0 0
-180 -90 0 90 180 -180 -90 0 90 180

Φ (Deg) Φ (Deg)

H The Torsional Energy Profile The Torsional Energy Profile


Me
C C H Φ=0 Φ=0 H
H Me
H
Me
C C H
H HO Φ = 180
H
H
Me OH
C C H
H
Φ = 180 H
+3.88 kcal
H
Φ = 90 Me Me +1.44 kcal Φ = 120
C C H
Me H OH
+0.86
Me H Me kcal
C C H C C H
H H H
+0.52
H H H
kcal
Φ=0 Φ = 180 Φ=0 Φ = 180
Values calculated using MM2 (molecular mechanics) force fields
via the Macromodel multiconformation search. Review: Hoffman, R. W. Chem. Rev. 1989, 89, 1841.
5-04-2-pentene/ z-2-buten-1-ol 9/23/03 3:00 PM
Evans, Duffy, & Ripin Conformational Barriers to Rotation: Olefin A-1,3 Interactions-2 Chem 206
5
(Z)-2-hydroxy-3-pentene OH
Me
Rotate clockwise C C H 4.6 kcal/mol
Me H Me
4
H
OH Me
HO H
Me
C C H
E (kcal/mol)

3 100 °
H Me
Φ Me Me
C C H 0.3-0.4 kcal/mol
2 H H OH

1 H Me 30 °
Me
C C H
H

OH
0
-180 -90 0 90 180 Lowest energy conformer
Φ (Deg)
The Torsional Energy Profile Me
Me 60 °
Me
OH H H
Φ=0 OH 2.7 kcal/mol C C Me
Me H
C C H
H H Me H OH
Me Φ = -140
C C H
H HO Me
Needs to be redone

Φ = 110 A(1,3) interaction 4.0 kcal/mol HO H


+2.72 ????? +4.68 Me H
30 °
Me C C Me
R2 3 R1 H
Φ = -80 C C H
HO
Φ = 80 Me H H Y
H OH Φ = 150 Me
Me Me X * R large
C C H C C H R3 2
H H H Me 1 Lowest energy conformer
OH Me
Me H +0.66H C C H R small
+0.34 +0.40 kcal HO
Φ=0 Φ = 180 A(1,2) interaction 2.7 kcal/mol (MM2)
5-05-z-2-hydroxy-3-pentene 9/23/03 6:22 PM
D. A. Evans Acyclic Conformational Analysis: Allylic Strain Chem 206
The Definition of Allylic Strain
Can you predict the stereochemical outcome of this reaction?
F. Johnson, Chem. Rev. 1968, 68, 375; Allylic Strain in Six-Membered Rings
R. W. Hoffmann, Chem. Rev. 1989, 89, 1841-1860 (handout)
Allylic 1-3-Strain as a Controlling Element in Stereoselective Transformations D. Kim & Co-workers, Tetrahedron Lett. 1986, 27, 943.
Houk, Hoffmann JACS 1991, 113, 5006

R2 3 R1 O OTs OLi OTs


Consider the illustrated general structure Y Me Me
where X & Y are permutations of C, N, and O: EtO LiNR2 EtO
X R large 1 + 2
R3 2 1 n-C4H9
H n-C4H9 98:2
H
Typical examples: R small
O
Me
R2 R1 R2 R1 R2 R1 R2 R1 ■ Relevant enolate EtO
conformations 1 major
✻ R large N ✻ R large N ✻ R large – N ✻ R large n-C4H9
R3 R + O + H
p.
p. R small R small R small R small

Olefin Imine Imonium ion Nitrone (CH2)4OTs (CH2)4OTs


TsO(H2C)4 H OR Bu OR
OR
In the above examples, the resident allylic stereocenter (✻) and its associated Me C C Me C C Me C C
substituents frequently impart a pronounced bias towards reactions occuring at OLi
Bu OLi OLi
the pi-bond. H
A(1,3) A1 Bu B1 C1 H
Nonbonding interactions between the allylic R2 3 R1 interaction
Y critical conformations
substituents (Rlarge, Rsmall) & substituents at
the 2- & 3-positions play a critical role in X R large H Bu
defining the stereochemical course of such R3 2 1 H Bu
OR OR OR
reactions R small Me C Me C C Me C
A(1,2) C C
OLi OLi OLi
interaction TsO(H2C)4 Bu H (CH2)4OTs
(CH2)4OTs
A2 B2 C2
Representative Reactions controlled by Allylic Strain Interactions
HO HO
H H
O R O R
Hg(OAc)2 Me
NaBH4 EtO2C
Me HO Me
OBn OBn 2 minor
diastereoselection 10:1 n-C4H9
H
M. Isobe & Co-workers, Tetrahedron Lett. 1985, 26, 5199.

5-06-Allylic Strain-1 9/23/03 6:24 PM


D. A. Evans Allylic Strain & Enolate Diastereoface Selection Chem 206

O OTs O Ph OM Ph O
Me NH4Cl
Me
EtO LiNR2 EtO Me3Si OMe Me3Si OMe
diastereoselection 98:2
R R
n-C4H9 n-C4H9
H H R-substituent diastereoselection

R = Me 87:13
O O
Me R = Et 80:20

EtO R = CHMe2 40:60 major diastereomer opposite


EtO LiNR2 diastereoselection 89:11
to that shown

I. Fleming & Co-workers, Chem. Commun. 1985, 318.


MeI n-C4H9 Y. Yamamoto & Co-workers, Chem. Commun. 1984, 904.
n-C4H9 H
H
D. Kim & Co-workers, Tetrahedron Lett. 1986, 27, 943. Ph O Ph O
LiNR2 H
Me3Si OBn Me–CHO Me3Si OBn diastereoselection 90:10 at C3
one isomer at C2
MeO MeO 71% yield Me OH
RO2C O RO2C O
p.
p. LiNR2
H "one isomer" I. Fleming & Co-workers, Chem. Commun. 1986, 1198.
Br
H 95% yield H
CO2Me CO2Me Me O S Me O S
H
G. Stork & Co-workers, Tetrahedron Lett. 1987, 28, 2088. Bn Sn(OTf)2 Bn
N N S N N S diastereoselection >95%
R–CHO
Boc Boc
91-95% R OH
Me CH2 Me CH2
TBSOCH2 T. Mukaiyama & Co-workers, Chem. Letters 1986, 637
LiNR2 TBSOCH2
"one isomer"
MeI Me H O Me O
Ph(MeS)2C–Li MeS
H H MeS
CO2Me CO2Me Me OMe Me–I OMe diastereoselection 99:1
86% MeS Me
T. Money & Co-workers, Chem. Commun. 1986, 288.
K. Koga & Co-workers, Tetrahedron Letters 1985, 26, 3031.

R OM R O
MeI
I
PhMe2Si OEt PhMe2Si OEt OLi
CO2Et H
Me R = Me: diastereoselection 99:1 R CO2Et
O-t-Bu R = H: one isomer
R = Ph: diastereoselection 97:3
KOt-Bu CO2-t-Bu R = Me: > 15 :1
I. Fleming & Co-workers, Chem. Commun. 1984, 28. THF -78 °C H
R
Y. Yamaguchi & Co-workers, Tetrahedron Letters 1985, 26,1723.

5-07-A-strain enolates 9/20/01 4:45 PM


D. A. Evans Allylic Strain & Olefin Hydroboration Chem 206

■ The basic process Hydroborations dominated by A(1,3) Strain


S S ‡ OH
H CH2 OBn B2 H6 CH2 OBn
B H O
H B H H2 B H O
H H2 O2
H Me Me Me Me
R C C R
R R R R diastereoselection 8:1
C C C C R R
R R R R OMe OMe OH
Me B2 H6
O O OH
H2 O2
■ Response to steric effects: Here is a good calibration system: Me Me Me Me Me
OH
A diastereoselection 12:1

Oxidant Ratio, A:E Reference Y. Kishi & Co-workers, J. Am. Chem. Soc. 1979, 101, 259.
CH2
MCPBA 69:31 JOC, 1967, 32, 1363
Me3 C JOC, 1970, 35, 2654 OH
BH3, H2 O2 34:66
E B2 H6
H BnO OH BnO OH
H2 O2
Me Me Me Me Me Me
■ Acyclic hydroboration can be controlled by A(1,3) interactions:
Diastereoselection = 3:1
OH
C. H. Heathcock et. al. Tetrahedron Lett 1984 25 243.
RL R2 BH RL
OH OH major diastereomer
H2 O2
RM Me RM Me
R
R Me Me
B H Me Me
A(1,3) allylic strain RM H ThexylBH2 ,
Me TrO OTr
control elements Steric effects; RL vs RM H C C
then BH3 OH OH OH
Staggered transition states CH2 OR OH
TrO OTr
RL Diastereoselection; 5:1

OH
RL Me R2 BH RL Me Me Me Me ThexylBH2 , Me Me Me Me
OH major diastereomer
H2 O2
RM RM Me then BH3 TrO OTr
OH
R OH OH OH OH OH OH OH OH
R TrO OTr
B H Diastereoselection; 4: 1
RM H
CH2 OR
See Houk, Tetrahedron 1984, 40, 2257 H C C Still, W.C.; Barrish, J. C. J. Am. Chem. Soc. 1983, 105, 2487.
Me
RL
5-07a-A-strain hydroboration 9/24/03 9:45 AM
D. A. Evans Allylic Strain & Amide Conformation Chem 206

The selection of amide protecting group may be done with the knowledge that
Consider the resonance structures of an amide: R2 3 R1 altered conformational preferences may result:
Y
O O
O R1 –O R1 X R large H H
C C R3 2 1
N R N R R small Favored for Favored for
R3 R3 + R = H, alkyl R = COR
1 1
N N
R R H H
R R
A(1,3) interactions between the "allylic substituent" and the R1 moiety will H
strongly influence the torsion angle between N & C1. O O
Favored N H Disfavored
N
O Me R
C H H R
N Me H
Me O
O
Me H
Disfavored N Favored
N
O C C R
■ conformations of cyclic amides H H
p.
p. R O
–O R R
C H
R N N N
+ R C R C
R H
O O strongly favored R
R
A(1,3) 2 –O 2
A(1,3) interaction between the C2 & amide O 1 1
H Me C R C R
Chow H Me substituents will strongly influence the torsion
Can. J. Chem. 1968, 46, 2821 angle between C1 & C2. N N
Me H R R R + R
N N
R C R C
Me H
O O As a result, amides afford (Z) enolates under all conditions
strongly favored
H ‡
O O H
OM
published X-ray structure of this amide shows chair L
Me N Me diaxial conformation base L Me L
O C N O C N
L N
Quick, J. Org. Chem. 1978, 43, 2705 Me H favored Me H L
Ph O
H L
■ Problem: Predict the stereochemical outcome of this cyclization. (Z)-Enolate
H H ‡
OH H
HOCO ✻
HCO2H ✻
OM
L base L
D. Hart, JACS 1980, 102, 397 N O C N O C N H L
N N
H Me L disfavored H Me L
Ph O Ph O Me L
diastereoselection >95% identify HOMO-LUMO pair (E)-Enolate
5-08-A-strain Amides-1 9/23/03 6:09 PM
D. A. Evans Allylic Strain & Amide Conformation Chem 206
A(1,3) Strain and Chiral Enolate Design Polypropionate Biosynthesis: The Acylation Event

O O M O
O O O O OH O
Me LDA
N Me R SR Acylation Reduction
O
or NaNTMS2 N O enolization selectivity R SR R SR
>100:1 – CO2
O O Me Me
Bn Bn
HO SR
‡ El(+) Me
H
JACS. 1982,104, 1737.
O O
L First laboratory analogue of the acylation event
O C N Me
Me H L N O Li
O O O O
El O
O ❋
favored Bn Me
N O N O
enolization geometry Et Cl
Me Me
p.
p. ■ In the enolate alkylation process product epimerization is a serious R R
problem. Allylic strain suppresses product enolization through the
intervention of allylic strain with M. Ennis JACS 1984, 106, 1154. Diastereoselection ~ 97 : 3

H
El
L Me El
O C N L L Why does'nt the acylation product rapidy epimerize at the exocyclic
O C N O
L C N
El Me Me H L L
stereocenter??
B A C H R H
R R
While conformers B and C meet the stereoelectronic requirement for O C N O C N
enolization, they are much higher in energy than conformer A. Further, as R R
Me H R Me
deprotonation is initiated, A(1,3) destabilization contributes significantly to
favored
reducing the kinetic acidity of the system

These allylic strain attributes are an integral part of the design criteria of
chiral amide and imide-based enolate systems
O O
O CH2OH O Me
Me
Me N O Me
N N
Bn Me OH

Evans Evans Myers


Tetr Lett. 1977, 29, 2495 JACS 1982,104, 1737. JACS 1997, 119, 6496 X-ray structure

5-09-A-strain Amides-2 9/20/01 5:22 PM


D. A. Evans Discodermolide Chem 206

hinge

Me Me Me
16
HO
Me 17

O O H OH O NH2
Me Me
OH O
Me Me
OH
- immunosuppressive activity
- potent microtubule-stabilizing agent
(antitumor activity similar to that of taxol)

The epimers at C16 and C17 have no or almost no biological activity.

The conformation about C16 and C17 is critical to discodermolide's biological activity.

S. L. Schreiber et al. JACS 1996, 118, 11061.

5-10-Discodermolide 9/19/01 12:14 AM


D. A. Evans Conformational Analysis - Discodermolide X-ray 1 Chem 206

Me Me Me
HO
Me
O O H OH O NH2
Me Me
OH O
Me Me
OH

5-11-Discodermolide X-ray1 9/21/01 8:34 AM


D. A. Evans Conformational Analysis - Discodermolide X-ray 2 Chem 206

Me Me Me
16
HO
Me
O O H OH O NH2
Me Me
OH O
Me Me
OH

16

5-12-Discodermolide X-ray2 9/21/01 8:40 AM


D. A. Evans Conformational Analysis: Part–3 Chem 206

Conformational Analysis of Cyclic Systems


http://www.courses.fas.harvard.edu/~chem206/

Three Types of Strain:


Chemistry 206 Prelog Strain: van der Waals interactions

Baeyer Strain: bond angle distortion away from the ideal


Advanced Organic Chemistry
Pitzer Strain: torsional rotation about a sigma bond

Lecture Number 6 Baeyer Strain for selected ring sizes

size of ring Ht of Combustion Total StrainStrain per CH2 "angle strain"


(kcal/mol) (kcal/mol) (kcal.mol) deviation from 109°28'
Conformational Analysis-3 3 499.8 27.5 9.17 24°44'
4 656.1 26.3 6.58 9°44'
5 793.5 6.2 1.24 0°44'
■ Conformational Analysis of C4 → C6 Rings 6 944.8 0.1 0.02 -5°16'
7 1108.3 6.2 0.89
1269.2 9.7 1.21
■ Reading Assignment for week 8
9 1429.6 12.6 1.40
10 1586.8 12.4 1.24
A. Carey & Sundberg: Part A; Chapter 3 11 1743.1 11.3 1.02
12 1893.4 4.1 0.34
Eliel & Wilen, "Stereochemistry of Organic Compounds, "Chapter 11, 13 2051.9 5.2 0.40
Configuration and Conformation of Cyclic Molecules, Wiley, 1994 14 2206.1 1.9 0.14
15 2363.5 1.9 0.13
Eliel, E. L., Wilen, S. H. Stereochemistry of Organic Compounds
Chapter 11, John Wiley & Sons, 1994.
Ribeiro & Rittner, "The Role of Hyperconjugation in the Conformational
Analysis of Methylcyclohexane and Methylheterocyclohexanes"
■ Baeyer "angle strain" is calculated from the deviation of the
planar bond angles from the ideal tetrahedral bond angle.
J. Org. Chem., 2003, 68, 6780-6787 (handout)
■ Discrepancies between calculated strain/CH2 and the "angle
de Meijere, "Bonding Properties of Cyclopropane & their Chemical strain" results from puckering to minimize van der Waals or
Characteristics" eclipsing torsional strain between vicinal hydrogens.
Angew Chem. Int. Ed. 1979, 18, 809-826
■ Why is there an increase in strain for medium sized rings even
Friday, though they also can access puckered conformations free of
September 26, 2003 angle strain? The answer is transannular strain- van der Waals
D. A. Evans interactions between hydrogens across the ring.
6-00-Cover Page 9/26/03 8:44 AM
Evans, Kim, Breit Cyclopropane: Bonding, Conformation, Carbonium Ion Stabilization Chem 206

Cyclopropane Carbocation Stabilization via Cyclopropylgroups

H ■ Necessarily planar.
■ Subtituents are therefore eclipsed.
H ■ Disubstitution prefers to be trans. C

H
ϕ = 120 °
H ■ Almost sp2, not sp3 Me
A rotational barrier of about NMR in super acids
υ = 3080 cm-1 13.7 kcal/mol is observed in
Me
δ(CH3) = 2.6 and 3.2 ppm
following example:
Walsh Model for Strained Rings: H
O
■ Rather than σ and σ* c-c bonds, cyclopropane has sp2 and p-type
X-ray Structures support this orientation 1.222 Å
orbitals instead. 1.517 Å
R
1.302 Å 1.478 Å 1.474 Å

1.464 Å
σ (antibonding) σ (antibonding)
1.541 Å
π (antibonding)
H
1.409 Å
3 Nonbonding 1.444 Å
H
H
side view 1.534 Å O

Ph

π (bonding) π (bonding)

σ–1 (bonding)
de Meijere, "Bonding Properties of Cyclopropane & their Chemical Characteristics"
Angew Chem. Int. Ed. 1979, 18, 809-826 (handout)
de Meijere, A.; Wessjohann, L. "Tailoring the Reactivity of Small Ring Building R. F. Childs, JACS 1986, 108, 1692
Blocks for Organic Synthesis." Synlett 1990, 20.

6-01-Cyclopropane 9/25/03 7:55 PM


Evans, Kim, Breit Conformational Analysis: Cyclic Systems-2 Chem 206

Cyclobutane Cyclopentane H H
H H H
145-155° H
H H H
H H H H
H H H
ax ax ■ Eclipsing torsional strain overrides H H H
H H H H
eq increased bond angle strain by puckering. H H
eq eq H H H H
eq H
ax ■ Ring barrier to inversion is 1.45 kcal/mol. CsEnvelope C2 Half-Chair CsEnvelope
ϕ = 28 ° ax
■ Two lowest energy conformations (10 envelope and 10 half chair conformations
Cs favored by only 0.5 kcal/mol) in rapid conformational flux (pseudorotation)
which causes the molecule to appear to have a single out-of-plane atom "bulge"
which rotates about the ring.

■ Since there is no "natural" conformation of cyclopentane, the ring conforms to


(MM2) minimize interactions of any substituents present.

H
H H
CsEnvelope H
H
(MM2) H
H H H

■ A single substituent prefers the equatorial position of the flap of the envelope
(barrier ca. 3.4 kcal/mol, R = CH3).
■ ∆ G = 1 kcal/mol favoring R = Me equatorial
X
■ 1,2 Disubstitution prefers
■ 1,3 Disubstitution prefers cis diequatorial to trans for steric/torsional
trans by 0.58 kcal/mol for di-bromo cmpd. reasons (alkyl groups) and
dipole reasons (polar groups).

Me X

Me ■ 1,3 Disubstitution: Cis-1,3-dimethyl


■ 1,2 Disubstitution prefers trans diequatorial to cyclopentane 0.5 kcal/mol more stable than trans.
cis by 1.3 kcal/mol for diacid (roughly equivalent to H H
the cyclohexyl analogue.) ■ A carbonyl or methylene prefers the planar position of
the half-chair (barrier 1.15 kcal/mol for cyclopentanone).
6-02-Conform/cyclic-2 9/25/03 7:55 PM X
Evans, Kim, Breit Conformational Analysis: Cyclic Systems-3 Chem 206
Methylenecyclopentane and Cyclopentene Cyclohexane Energy Profile (kcal/mol)
Strain trends:
Half-Chair
■ Decrease in eclipsing strain
more than compensates for the
≈ > >
increase in angle strain. Boat
+1.0–1.5
10.7-
Relative to cyclohexane derivatives, those of cyclopentane prefer an sp2 11.5
center in the ring to minimize eclipsing interactions.
"Reactions will proceed in such a manner as to favor the formation or retention +5.5
of an exo double bond in the 5-ring and to avoid the formation or retention of
the exo double bond in the 6-ring systems." Brown, H. C., Brewster, J. H.;
Shechter, H. J. Am. Chem. Soc. 1954, 76, 467. +5.5
Chair Inverted Chair
Examples: H
Twist Boat

H O H
NaBH4
OH
H k6 H
H H
H H H k6
H = 23
H k5
NaBH4 H H
O
H k5
OH
H
H

Brown, H. C.; Ichikawa, K. Tetrahedron 1957, 1, 221. ∆E = +6.5-7.0

O O
O O
hydrolyzes ∆E = +5.5
13 times faster than

Conan, J-Y.; Natat, A.; Priolet, D. Bull. Soc. Chim., Fr. 1976, 1935.
O OH O The barrier: +10.7-11.5
O

OEt
OEt
E=0
95.5:4.5 keto:enol 76:24 enol:keto

Brown, H. C., Brewster, J. H.; Shechter, H. JACS 1954, 76, 467.

6-03-Conform/cyclic-3 9/25/03 7:57 PM


Evans, Breit Conformational Analysis: Cyclic Systems-4 Chem 206

Monosubstituted Cyclohexanes: A Values A Values depend on the relative size of the particular substituent.
R H
Keq H H H Me H Me Me Me
H H Me Me
H R ∆G° = –RTlnKeq

H H H H
■ Me–axial has 2 gauche butane interactions more than Me–equatorial.
Expected destabilization: ≈ 2(0.88) kcal/mol = ~1.8 kcal/mol; A–Value 1.74 1.80 2.15 5.0
Observed: 1.74 kcal/mol
Me Me The "relative size" of a substituent and the associated A-value may not correlate.
H Me H
H For example, consider the –CMe3 and –SiMe3 substituents. While the –SiMe3
C H C
C H substituent has a larger covalent radius, it has a smaller A-value:
H H
H H
Me Me Me
■ The A– Value, or -∆G°, is the preference of the substituent for the Me Me Me
equatorial position. C Me Si Me Sn Me

H H H

A–Value 4.5-5.0 2.5 1.1

Can you explain these observations?

■ The impact of double bonds on A-values:


Lambert, Accts. Chem. Res. 1987, 20, 454

R H

H R

A-value
substituent −∆G° (cyclohexane)

R = Me 0.8 1.74
R = OMe 0.8 0.6
R = OAc 0.6 0.71
The Me substituent appears to respond strictly to the decrease in nonbonding interactions
in axial conformer. With the more polar substituents, electrostatic effects due to the
trigonal ring carbon offset the decreased steric environment.

6-04-Conform/cyclic-4 9/25/03 7:59 PM


Rigberio & Rittner, “The Role of Hyperconjugation in the Conformational
Analysis of Methylcyclohexane and Methylheterocyclohexanes”
JOC 2003, 68, 6780

Commentary by Ken Houk University of California, Los Angeles


Department of Chemistry
Dear David,
“The calculations in the Ribeiro article look fine, but I am not convinced by
the interpretation. It does seem to work pretty well for many systems, but
not obviously for the isomeric 1,3-dioxane cases they note early on. There
seems no explanation of why C–H hyperconjugates better than C–C.
Further, the results with alkyls larger than methyl still require traditional
steric arguments. I would say that the equatorial methyl preference has
been attributed in part to hyperconjugative effects that occur when the CH
bonds are anti-periplanar. But I would not yet go much beyond that!
Best regards,

(b. 1943) A.B. 1964, Ph.D. 1968, Harvard


University; Assistant-Full Professor, Louisiana
State University, 1968-1980; Alfred P. Sloan
Fellow, 1975-1977; Camille and Henry Dreyfus
Teacher-Scholar, 1972-1977; LSU Distinguished
Research Master, 1978; Professor, University of
Pittsburgh, 1980-1985; Alexander von Humboldt
Senior U.S. Scientist Award, 1982; Akron Section,
American Chemical Society Award, 1983; Arthur
C. Cope Scholar Award, 1988; Director, Chemistry Division, National
Science Foundation, 1988-1990; James Flack Norris Award in Physical
Organic Chemistry, 1991; Schrödinger Medal, World Association of
Theoretically Oriented Chemists, 1998; Tolman Medal, Southern California
Section, American Chemical Society, 1999; Fellow of the American
Academy of Arts and Sciences, 2002; American Chemical Society Award
for Computers in Chemical and Pharmaceutical Research, 2003;
International Academy of Quantum Molecular Science, 2003.
D. A. Evans Bond Lengths and A-Values of Methyl Halides Chem 206

C–F: 1.39 Å C–Cl: 1.79 Å C–Br: 1.95 Å C–I: 2.16 Å

F A-value: Cl A-value: Br A–value: I A-value:


0.25–0.42 0.53–64 0.48-0.67 0.47–0.61

p.
p.

Chem 3D Pro (Verson 5.0)


6-04a methyl halides 9/26/01 8:26 AM
Evans, Breit Conformational Analysis: Cyclic Systems-5 Chem 206

The impact of trigonal Carbon Polysubstituted Cyclohexane A Values


■ Let's now compare look at the carbonyl analog in the 3-position ■ As long as the substituents on the ring do not interact in either
Me conformation, their A-values are roughly additive
H

H Me 1,4 Disubstitution: A Values are roughly additive.


O O
Me Me
∆G° = –1.36 kcal/mol
versus –1.74 for cyclohexane ∆G° = 0 kcal/mol
Me Me

Me
■ Let's now compare look at the carbonyl analog in the 2-position
Me
Me H Me
base
epimerization
Me ∆G° = –2(1.74) = –3.48 kcal/mol
Me3C H Me3C Me

O O
1,3 Disubstitution: A Values are only additive in the trans diastereomer
∆G° = –1.56 kcal/mol
versus –1.74 for cyclohexane X
H
H X
H ∆G° = A(Me) – A(X)
However, the larger alkyl groups do not follow the expected trend.
Can you explain? (see Eliel, page 732) Me Me H

CHMe2 base H
epimerization H
H Me3C CHMe2 H
Me3C H
X H
O O
Me Me X
∆G° = –0.59 kcal/mol
versus –2.15 for cyclohexane The new interaction!

For X = Me
CMe3 base H H
H
epimerization H
Me3C H Me3C CMe3 H
Me H
O O Me H
+ 0.88 + 0.88
∆G° = –1.62 kcal/mol Me Me
versus –5.0 for cyclohexane
∆G° = 2(.9) + 1(+3.7)= 5.5 kcal/mol
+ 3.7
6-05-Conform/cyclic-5 9/25/03 8:01 PM
Evans, Breit Conformational Analysis: Cyclic Systems-6 Chem 206
Let's now consider geminal substitution
Me
Let's now consider vicinal substitution
Me Me
Ph
H
Ph
Case 1: HH Me
The prediction: ∆G° = A(Ph) – A(Me) Me H
Me
∆G° = +2.8 – 1.7 = +1.1 kcal/mol
The prediction: ∆G° = 1 gauche butane – 2A(Me)
Observed: ∆G° = –0.32 kcal/mol
∆G° = +0.88 – 2(1.74) = +2.6 kcal/mol
Hence, when the two substituents are mutually interacting you can predict neither
the magnitude or the direction of the equilibrium. Let's analyze this case. Observed: ∆G° = +2.74 kcal/mol

Allinger, Tet. Lett. 1971, 3259 If the added gauche butane destabilization in the di-equatorial
conformer had not been added, the estimate would have been off.
000000
00
0
000000
000000

00
0
00
00000
0
000
000
0000
000

Case 2:
00
00
00

Me
0
0

H Me
OH H
A B
H
∆G° = +2.8 OH Me
∆G° = –0.32 H
H Me
H
The conformer which places the isopropyl group equatorial is much more
strongly preferred than would be suggested by A- Values. This is due to
a syn pentane OH/Me interaction.

Problem:
Can you rationalize the stereochemical outcome of this reaction?
0
0
0000000
000000

000000
0

O O
000000
000

Me

EtO EtO
LiNR2
C
00
00
00

D MeI
00

n-C4H9 n-C4H9
H H

diastereoselection 89:11
Note the difference in the Ph substituent in A & B. D. Kim & Co-workers, Tetrahedron Lett. 1986, 27, 943.
6-06-Conform/cyclic-6 9/25/03 8:13 PM
Evans, Breit Conformational Analysis: Cyclic Systems-7 Chem 206

Heteroatom-Substituted 6-Membered Rings A-Values for N-Substituents in Piperidine


■ A-values at the 2-position in both the O & N heterocycles are larger than
expected. This is due to the shorter C–O (1.43 Å), and C–N (1.47 Å) bond H The Reference:
lengths relative to carbon (C–C; 1.53 Å). The combination of bond length and N N
bond angle change increases the indicated 1,3-diaxial interaction (see eq 1, 4). H ∆G° = –0.36 kcal/mol

Me
Me Reference: H
N N
Me ∆G° = –3.0 kcal/mol
H Me −∆G° = 1.74 kcal/mol

■ Hydrogen is "bigger" than the N–lone Pair.


Me H
H ■ The A-value of N–substituents is slightly larger than the corresponding
(1) H Me −∆ G° = 2.86 kcal/mol cyclohexane value. Rationalize
O O

Me H
Me H

H Me −∆G° = 1.43 kcal/mol O O −∆G° = 4.0 kcal/mol


(2) O O H Me
O O

Me H

H Me −∆G° = 1.95 kcal/mol O O


(3) H Me
O O
O O −∆G° = 0.8 kcal/mol
Me H
Me H
H
■ The indicated distance is 2.05 Å. The analogous H–H distance in
(4) H Me −∆ G° = 2.5 kcal/mol
N H N H Cyclohexane is 2.45 Å

Me H

N H N Me −∆G° = 1.6 kcal/mol


(5)
H H
Me H

H Me −∆G° = 1.9 kcal/mol


(6) H N H
N

2.05 Å 2.45 Å
6-07-Conform/cyclic-7 9/25/03 8:02 PM
Evans, Breit Conformational Analysis: Bicyclic Ring Systems Chem 206

Conformations of Bicyclic Ring Systems Estimate the energy difference between the two methyl-decalins
shown below.
Me Me Me

Me H 11
1 13
17
H H H H
10
O see Elier, p 780
14
7
4
Hydrindane Ring System (6/5)
The steroid nucleus provided the stimulation for the development of H H
conformational analysis, particularly of polycyclic ring systems. D. H. R. Barton
was awarded a Nobel prize in 1969 for his contributions in this area.
flexible rigid
Decalin Ring System (6/6)
H H
H H ∆G° = –0.5 kcal/mol
mobile H rigid
■ The turnover to favor the cis fusion results from the entropic preference for the
less ordered cis isomer.
H
The 5-5 Ring System

H H
H

H favored

H H
2.4 kcal/mol 0 Relative ∆G°
∆G° = +6.4 kcal/mol
Let's identify the destabilizing gauche butane interactions in the cis isomer
H H
H Me H R
3
Gauche-butane interactions Me H R
C D
C D
H C1 → C2 A B
A B
H H H
4 2 C1 → C3 H H
A/B Trans H
C4 → C3 A/B Cis
1
∆G°(est) = 3(0.88) = 2.64 kcal/mol
Rationalize the conformational flexibility of a A/B Trans vs. A/B Cis Steroid!
6-08-Conform/bicyclics 9/25/03 8:04 PM
Evans, Breit Conformational Analysis: Axial vs Equatorial Reactivity Chem 206

Different reactivity for axial and equatorial substituents ■ SN2 Reactions (Displacement with Ph–S–)
H OTs
Axial substituents are more hindered, thus less reactive in many
transformations Me3C Me3C
OTs H

■ Acetylation with Ac2O/Py k rel 1 31


H OH The axial diastereomer is not always slower reacting:
OH H ■ Alcohol Oxidation with Cr(6+)
k rel 1 0.13 H OH
H OH
Me3C OH Me3C H
Me3C OH Me3C H
k rel
k rel 1 3.2
1 0.27
Me H Me OH
■ Acid-catalyzed esterification Me OH Me H
H CO2H Me
k rel Me
1 3.36
CO2H H
The rate-determining step is breakdown of the chromate ester. This is an
k rel 1 0.04 apparent case of strain acceleration

H CO2H Steric Hindrance and Steric Assistance


TS
Me3C CO2H Me3C H TS
k rel
1 0.05 ∆∆G > ∆∆G° TS ∆∆G < ∆∆G°

■ Ester Saponification

H CO2Et

Me3C CO2Et Me3C H ∆G A ∆G B


∆G ref

k rel 20 1

For a more detailed discussion of this topic see:


Eliel, E. L., S. H. Wilen, et al. (1994). Stereochemistry of Organic GS
Compounds pp 720-726 GS ∆∆G°
∆∆G°

∆GA > ∆G ref GS


Reference Case ∆GB < ∆G ref
6-09-Axial-equat react 9/25/03 8:05 PM
D. A. Evans Conformational Transmission Chem 206
Observation: Let Φ be defined as the torsion or dihedral angle for butane
Relative enolate stability correlates to ring junction stereochemistry
Me Φ
H
A Me
Me
H H H
C B Φ illustrated may be designated as Φ B.
base H H
C Me
O LiO LiO H
H H H
ratio: 13 : 87
Let's now consider cyclohexane
H H H

base Perfect chair real chair

O LiO LiO Φ = 60° Φ = 56°


H H H
ratio: 70 : 30 CCC∠ 109° 28' CCC∠ 111°
House, JOC 1965, 30, 1341
Given cyclohexane with an identified torsion angle ΦR, if ΦR either increases
Observation: or decreases wht effects in angle change are transmitted to ΦO, ΦM, and ΦP?
Relative enolate stability seems to be correlated to position of C=C
ΦR = 56° ΦR = 0° [∆]
H H H
ΦO 56 15 –41°
base ΦR ΦP
ΦM 56 44 –11°
O LiO LiO ΦM
H H H ΦO ΦP 56 61 +6°
ratio: 10 : 90
H H H [∆] = ΦR( 0°) – ΦR( 56°)
Operation:
base 56 56 15° 44°
–H2
O LiO LiO 56 56 0* 61°
H H H
ratio: 92 : 8 56 56

How do we explain the experimental observations shown above? Hence, relative to cyclohexane, the following notation for torsion angle change
may be denoted:

Readings: Velluz etal, Angew. Chemie, Int Ed. 1965, 4, 181-270 –11°
+6°

For ΦP For ΦM

6-10-Conformal transmission 9/25/03 8:09 PM


D. A. Evans Conformational Transmission-2 Chem 206

Operation: H H H
H H base
R Me
C O LiO LiO
H H H
R Me ratio: 10 : 90
H
H effects opposing effects reinforcing
Operation:
H H
R H
C H H H
R Me base
H
Me
O LiO LiO
H H H
Simple Application: Reinforcing Torsional Effects ratio: 92 : 8
effects reinforcing effects opposing
R R

Question: Which is the more stable C=C isomer in the two THC structures?
A B versus A B
Me Me
R R
OH OH
Which C=C bond isomer is more stable?

R Me Me
1) C=C will open up ring=B torsion angle O C5H11 O C5H11
Me Me
B 2) Ring B will resist increase in its ring fusion torsion angle
R. W. Kierstead, JACS 1967, 89, 5934
R 3) Therefore torsion effects are opposed
Question: Which enol acetate is more stable?

OBz OBz
R OBz
1) C=C will close down ring=B torsion angle
Ac2O
2) Ring B will accomodate decrease in its ring fusion torsion angle TSOH
or
3) Therefore torsion effects are reinforcing
R
O AcO AcO

6-11-Conformal transmission-2 9/25/03 8:12 PM


Evans, Breit Conformational Analysis: Cylcoheptane Chem 206

Cycloheptane Cyclooctane

5
Chair-Boat
Lowest-energy conformation

1 3
Transannular strain
Chair (2.16 kcal/mol) Twist-Chair (0 kcal/mol) between C3 & C7

Ring strain originates in eclipsing interactions and transannular


van der Waals interactions

Methyl position 1 2 3 4 5
(pseudoeqatorial)
∆G 1.8 2.8 >4.5 -0.3 6.1
(pseudoaxial) (kcal/mol)

Underside view of boat-chair C3 & C7 eclipsing interactions


Boat (3.02 kcal/mol) Twist-Boat (2.49 kcal/mol)
Hendrickson, J. B. JACS 1961, 83, 4537. 7
See Eliel, page 762+
Olefins are preferentially orientated to eliminate eclipsing interactions.

1 7

3 3

6-12-Conform/cycloheptane 9/24/01 7:43 AM


Evans, Breit Conformational Analysis: Cyclooctane Chem 206

Cyclooctane continued...
5
Chair-Boat (BC)
Lowest-energy conformation

1 3
Transannular strain
between C3 & C7 Chair-Boat (CB) conformation Chair-Chair (CC) conformation
reference structure (+1-1.6 kcal/mol)

Methyl position 1 2 3 4 5
(pseudoeqatorial)
∆G 1.8 2.8 >4.5 -0.3 6.1
(pseudoaxial) (kcal/mol)

Cyclooctane derivatives

Boat-Boat (BB) conformation


(>+ 8 kcal/mol)
O
Carbonyl is positioned at C3 or C7 Olefin is positioned at C3-C4 or C6-C7
Me Me

O O
LINR2 Predict
Nu stereochemistry
X MeI
❋ Me

Me

O O
X LiCuMe2 ❋ Predict
Nu
stereochemistry
Disubstitution occurs at C4 or C6 SN2 occurs at C1 and C5 Me Me

Still, W. C.; Galynker, I. Tetrahedron 1981, 37, 1981.

6-13-Conform/cyclooctane 9/24/01 7:43 AM


D. A. Evans Conformational Analysis: Part–4 Chem 206

■ Other Reading Material


http://www.courses.fas.harvard.edu/~chem206/
The Curtin-Hammett Principle
Chemistry 206
J. I. Seeman, J. Chem. Ed. 1986, 63, 42-48.

Advanced Organic Chemistry J. I. Seeman, Chem Rev. 1983, 83, 83-134.


Leading References:
Eliel, pp. 647-655
Lecture Number 7
Carey & Sundberg,Part A, CH 4, pp 187-250

Conformational Analysis-4
■ Problems of the Day: (To be discussed)

■ Ground State Torsional Effects (Conformational Transmission) Predict the stereochemical outcome of this reaction. The
diastereoselection is 99:1
■ Conformational Analysis of C6 → C8 Rings continued O
mCPBA
■ Transition State Torsional Effects H H

■ Curtin–Hammett Principle (Will not cover in lecture) N


N
Ph Ph
■ Reading Assignment for week O O
Eliel & Wilen, Stereochemistry of Carbon Compounds" Martinelli, et.al. Tett. Lett. 1989, 30, 3935
Chapter 11
Configuration and Conformation of Cyclic Molecules (handout)
Rationalize the stereochemical outcome of this reaction.
A. Carey & Sundberg: Part A; Chapter 4
"Study & Descrption of Reaction Mechanisms" Me
Me O CO2Me Me O CO2Me
LiNR2
K. Houk, Science. 1986, 231, 1108-1117 Me-I
Theory & Modeling of Stereoselective Organic Reactions (Handout) Me O Me Me O Me
diastereoselection is 8:1.
Monday,
D. A. Evans September 29, 2003
Ladner, et.al. Angew. Chemie, Int. Ed 1982, 21, 449-450
7-00-Cover Page 9/29/03 8:57 AM
D. A. Evans Conformational Transmission Chem 206
Observation: Let Φ be defined as the torsion or dihedral angle for butane
Relative enolate stability correlates to ring junction stereochemistry
Me Φ
H
A Me
Me
H H H
C B Φ illustrated may be designated as Φ B.
base H H
C Me
O LiO LiO H
H H H
ratio: 13 : 87
Let's now consider cyclohexane
H H H

base Perfect chair real chair

O LiO LiO Φ = 60° Φ = 56°


H H H
ratio: 70 : 30 CCC∠ 109° 28' CCC∠ 111°
House, JOC 1965, 30, 1341
Given cyclohexane with an identified torsion angle ΦR, if ΦR either increases
Observation: or decreases wht effects in angle change are transmitted to ΦO, ΦM, and ΦP?
Relative enolate stability seems to be correlated to position of C=C
ΦR = 56° ΦR = 0° [∆]
H H H
ΦO 56 15 –41°
base ΦR ΦP
ΦM 56 44 –11°
O LiO LiO ΦM
H H H ΦO ΦP 56 61 +6°
ratio: 10 : 90
H H H [∆] = ΦR( 0°) – ΦR( 56°)
Operation:
base 56 56 15° 44°
–H2
O LiO LiO 56 56 0* 61°
H H H
ratio: 92 : 8 56 56

How do we explain the experimental observations shown above? Hence, relative to cyclohexane, the following notation for torsion angle change
may be denoted:

Readings: Velluz etal, Angew. Chemie, Int Ed. 1965, 4, 181-270 –11°
+6°

For ΦP For ΦM

7-01-Conformal transmission 9/25/03 8:09 PM


D. A. Evans Conformational Transmission-2 Chem 206

Operation: H H H
H H base
R Me
C O LiO LiO
H H H
R Me ratio: 10 : 90
H
H effects opposing effects reinforcing
Operation:
H H
R H
C H H H
R Me base
H
Me
O LiO LiO
H H H
Simple Application: Reinforcing Torsional Effects ratio: 92 : 8
effects reinforcing effects opposing
R R

Question: Which is the more stable C=C isomer in the two THC structures?
A B versus A B
Me Me
R R
OH OH
Which C=C bond isomer is more stable?

R Me Me
1) C=C will open up ring=B torsion angle O C5H11 O C5H11
Me Me
B 2) Ring B will resist increase in its ring fusion torsion angle
R. W. Kierstead, JACS 1967, 89, 5934
R 3) Therefore torsion effects are opposed
Question: Which enol acetate is more stable?

OBz OBz
R OBz
1) C=C will close down ring=B torsion angle
Ac2O
2) Ring B will accomodate decrease in its ring fusion torsion angle TSOH
or
3) Therefore torsion effects are reinforcing
R
O AcO AcO

7-02-Conformal transmission-2 9/25/03 8:12 PM


Evans, Breit Conformational Analysis: Cylcoheptane Chem 206

Cycloheptane Cyclooctane

Chair-Boat 5
Lowest-energy conformation

1 3
Transannular strain
Chair (2.16 kcal/mol) Twist-Chair (0 kcal/mol) between C3 & C7

Ring strain originates in eclipsing interactions and transannular


van der Waals interactions

Methyl position 1 2 3 4 5
(pseudoeqatorial)
∆G 1.8 2.8 >4.5 -0.3 6.1
(pseudoaxial) (kcal/mol)

Underside view of boat-chair C3 & C7 eclipsing interactions


Boat (3.02 kcal/mol) Twist-Boat (2.49 kcal/mol)
Hendrickson, J. B. JACS 1961, 83, 4537. 7
See Eliel, page 762+
Olefins are preferentially orientated to eliminate eclipsing interactions.

1 7

3 3

7-03-Conform/cycloheptane 9/26/03 1:26 PM


Evans, Breit Conformational Analysis: Cyclooctane Chem 206

Cyclooctane continued...
5
Chair-Boat (BC)
Lowest-energy conformation

1 3
Transannular strain
between C3 & C7 Chair-Boat (CB) conformation Chair-Chair (CC) conformation
reference structure (+1-1.6 kcal/mol)

Methyl position 1 2 3 4 5
(pseudoeqatorial)
∆G 1.8 2.8 >4.5 -0.3 6.1
(pseudoaxial) (kcal/mol)

Cyclooctane derivatives

Boat-Boat (BB) conformation


(>+ 8 kcal/mol)
O
Carbonyl is positioned at C3 or C7 Olefin is positioned at C3-C4 or C6-C7
Me Me

O O
LINR2 Predict
Nu stereochemistry
X MeI
❋ Me

Me

O O
X LiCuMe2 ❋ Predict
Nu
stereochemistry
Disubstitution occurs at C4 or C6 SN2 occurs at C1 and C5 Me Me

Still, W. C.; Galynker, I. Tetrahedron 1981, 37, 3981.

7-04-Conform/cyclooctane 9/26/03 2:31 PM


D. A. Evans Ground State Torsional Effects Chem 206

Torsional Effects Relevant Orbital Interactions:


Torsional Strain: the resistance to rotation about a bond H
H
H H H H
Torsional energy: the energy required to obtain rotation about a bond C C H
C C H
H H
Torsional Angle: also known as dihedral angle
B A
H H
Torsional steering: Stereoselectivity originating from transition state
torsional energy considerations H H
H H H
C C H C C H
H H
H H
σ C–H & π electrons are σ C–H's properly aligned for π∗ overlap
destabilizing hence better delocalization
∆G = +3 kcal mol-1
Dorigo, A. E.; Pratt, D. W.; Houk, K. N. JACS 1987, 109, 6591-6600.

Conformational Preferences: Acetaldehyde


staggered eclipsed
H H
Torsional Strain (Pitzer Strain): Ethane H H
A O H O H B

H
H H
CH2 staggered The eclipsed conformation (conformation A) is preferred.
C conformation Polarization of the carbonyl decreases the 4 electron destabilizing
H H Rotational barrier: 1.14 kcal/mol
H
H Houk, JACS 1983, 105, 5980-5988.
CH2 +2.0 kcal/mol
H
eclipsed C Conformational Preferences
conformation H 1-Butene (X = CH2); Propanal (X = O)
H
H H
Wiberg K. B.; Martin, E. J. Amer. Chem. Soc. 1985, 107, 5035-5041. Me Me
A X H X H B

See Lecture 4 for previous discussion H


H
Conformation A is preferred. There is little steric repulsion between the
methyl and the X-group in conformation A.
7-05-torsional effects-1 9/29/03 8:14 AM
D. A. Evans Transition State Torsional Effects According to Houk Chem 206

Houk: "Torsional effects in transition states are more important than in ground states"
H* Transition states
H* H*
H H-radical and H-anion: antiperiplanar σ∗C–R orbital stabilized the TS
H H H
H illustrated for Nu addition
H 2C C H H2C C H H2C C H
H H H RL
H σ∗C-RL
lumo σ∗C-RL
C C
σC-Nu σC-Nu
0° 30° 60° 90° 120° homo
no H* Nu
+1.6
Forming bond
Forming bond
+ +4.7 Same trends are observed for H+ addition
H
Houk, Science 1981, 231, 1108-1117
"The Theory and Modeling of Stereoselective Organic Reactions"
2 Kcalmol-1
0
H• +2.4 Nu
0 H H
H σ∗C-RL
+5.3 Transition state C C H
H- H σC-Nu
RL
0
Nu
0° 30° 60° 90° 120° H H
σ∗C-RL
Houk, JACS 1981, 103, 2438 Ground state C
Houk, JACS 1982, 104, 7162 R H
RL
σC-Nu
7-06-Torsional effects-2 9/29/03 8:17 AM
D. A. Evans Transition State Torsional Effects: Olefin Additions Chem 206

■ Olefin Addition Reactions: Case one ■ Olefin Addition Reactions: Case two

How do we account for the high exo selectivities in addition How do we account for the high selectilvities in the oxidation of
reactions to norbornene? the indicated olefin?

exo OH O
Highly exo selective for electrophilic, OsO4
nucleophilic and cycloaddition reactions OH mCPBA
H
H H
H
Rate enhancement due to strain
N N
H
N Ph
Ph
endo Ph O O
O 98 : 2 99 : 1
diastereoselectivity diastereoselectivity
The Controversy over origin of high exoselectivities
Nitrogen protecting group does not affect selectivities
Steric effects

Least nuclear motion


A
Orbital distortion

Schleyer: torsional steering

B B

Favored

Schleyer, P. R. J. Amer. Chem. Soc. 1967, 89, 701. Martinelli, et.al. Tett. Lett. 1989, 30, 3935

Addition from exo face avoids eclipsing A & B hydrogens Addition from indicated olefin face avoids eclipsing A & B H's
(better hyperconjugative stabilization of transition state) (better hyperconjugative stabilization of transition state)

Martinelli has carried out further studies on related structures...........

7-07-torsional effects-3 9/29/03 8:18 AM


D. A. Evans Transition State Torsional Effects: Olefin Additions Chem 206

Martinelli: Torsional steering important in selectivity


O
mCPBA
H
89°

99 : 1 diastereoselectivity
major

O
mCPBA

Me 62°
Me

50 : 50 diastereoselectivity
63°

O
mCPBA
H

major
99 : 1 diastereoselectivity

40°

74°

Authors propose that diastereoselection controlled by


TS torsional effects
Martinelli & Houk, J. Org. Chem. 1994, 59, 2204.

7-08-torsional effects-4 9/29/03 8:18 AM


D. A. Evans Cyclohexanone Addition Reactions: Hydride Reduction Chem 206
HA
Stereoselective Reductions: Cyclic Systems Nu: Attack Trajectories for Cyclohexanone
H OH (Torsional Argument)
O HE
[H] OH H
Me3C Me3C Me3C

H H H
3
% Axial Diastereomer HE This approach favored
0 10 20 30 40 50 60 70 80 90 100 sterically
Nu:
HA

DIBAL-H 72:28 L-Selectride 8:92 The steric hindrance encountered by Nu-attack from the axial C=O face by the axial
K-Selectride 3:97
ring substituents (hydrogens in this case) at the 3-positions is more severe than the
NaBH4 79:21
steric hinderance encountered from Nu-attack from the equatorial C=O face.
Me
LiAlH(Ot-Bu)3 92:8 –
H B C CH2Me
LiAlH4 93:7 M+ H
3
Axial Attack
Observation: Increasingly bulky hydride reagents prefer to attack from H: –
the equatorial C=O face.
O–C–C–He dihedral:
The most stereoselective Reductions
+63.0 °
H OH
O
[H] OH H
Me3C Me3C Me3C O–C–C–He dihedral:
+4.0 °
H H H H: –
Reagent Ratio Equatorial Attack O–C–C–He dihedral:
KBH(s-Bu)3 03 :97 –56.0 °
Li in NH3 99 :01 The Issues Associated with the Reduction Process
H NHR
NPh
Steric Effects: Attack across equatorial C=O face sterically more favorable.
Me3C [H] NHR Me C H
Me3C 3
Torsional Effects: However, attack across the axial face of the C=O group avoids
H H H development of eclipsing interactions in the transition state.
(Note the dihedral angle sign changes between reactants &
(R) Reagent Ratio products shown above). These "torsional effects" favor axial attack.
Ganem, Tet. Let 1981, 22, 3447 R = Bn LiBH(s-Bu)3 03 :97 For "small" hydride reagents such as LiAlH4, torsional effects
Hutchins, JOC 1983, 48, 3412 R = Ph LiBH(s-Bu)3 01 :99 Prediction are felt to be dominant and this explains the predisposition for
private communication Al/Hg/MeOH ~90 :10 axial attack.

For "large" hydride reagents such as H–BR4, steric effects


Prediction now are dominant and this explains the predisposition for
equatorial attack.
7-09-Cyclohexanon/C=O-1 9/29/03 8:21 AM
K. A. Beaver, D. A. Evans Conformational Analysis and Reactivity: Curtin-Hammett Principle Chem 206
Leading References: J. I. Seeman, J. Chem. Ed. 1986, 63, 42-48. Case 1: "Kinetic Quench"
J. I. Seeman, Chem Rev. 1983, 83, 83-134.
See also Eliel, pp. 647-655
k1, k2 >> kA, kB: If the rates of reaction are faster than the rate of
interconversion, A and B cannot equilibrate during the course of the
reaction, and the product distribution (PB/PA) will reflect the initial
composition.
How does the conformation of a molecule effect its [PB]
=
[B]o
[PA] [A]o
reactivity?

Consider the following example:

Me Me
N N
13 13
Me–I Me–I
∆GAB‡

Energy
∆G2‡
∆G1‡ PB
∆G°
p.
p. PA A B
Do the two different conformers react at the same rate, or different rates?
What factors determine the product distribution?
Rxn. Coord.

The situation: In this case, the product distribution depends solely on the initial ratio
of the two conformers.
Consider two interconverting conformers, A and B, each of which can steric
undergo a reaction resulting in two different products, PA and PB. hindrance Me Me
N
Me
less stable H more stable
kA Me H
k1 k2
PA A B PB (1) N O O
-78°C
kB
major minor
∆G = -3.0 kcal/mol
Me–Br (ab initio calculations) Me–Br
We'll consider two limiting cases:

(1) The rate of reaction is faster than the rate of conformational interconversion Me Me

(2) The rate of reaction is slower than the rate of conformational interconversion Me N Me N major product
Me H
minor product
If the rates of conformationall interconversion and reaction are comparable, the O H O Me
reactants are not in equilibrium during the course of the reaction and complex
mathmatical solutions are necessary. See Seeman, Chem. Rev. 1983, 83 - 144 for While enolate conformers can be equilibrated at higher temperatures, the products of
analytical solutions. alkylation at -78° C always reflect the initial ratio of enloate isomers.

Padwa, JACS 1997 4565


7-10- Curtin-Hammett Intro 9/26/03 2:49 PM
K. A. Beaver, D. A. Evans Curtin - Hammett: Limiting Cases Chem 206
Case 2: Curtin-Hammett Conditons
k1, k2 << kA, kB: If the rates of reaction are much slower than the rate of To relate this quantity to ∆G values, recall that ∆Go = -RT ln Keq or Keq =
interconversion, (∆GAB‡ is small relative to ∆G1‡ and ∆G2‡), then the ratio of e-∆G°/RT, k1 = e-∆G1‡/RT, and k2 = e-∆G2‡/RT. Substituting this into the above
A to B is constant throughout the course of the reaction. equation:
[PB] k2 e-∆G2/RT
= Keq = (e-∆G°/RT) = e-∆G2/RTe-∆G°/RTe∆G1/RT (4)
k1 kA k2 (1) [PA] k1
PA A B PB e-∆G1/RT
slow kB slow
Combining terms:
fast

∆∆G‡ [PB] [PB]


= e-(∆G2 + ∆G°-∆G1)/RT or =
[PA] [PA] e-∆∆G/RT

∆G1‡ ∆G2‡
∆GAB‡ Where ∆∆G‡ = ∆G2‡+∆G°-∆G1‡
Energy

PB Curtin - Hammett Principle: The product composition is not solely


p.
p. A
∆G°
dependent on relative proportions of the conformational isomers in the
PA B
substrate; it is controlled by the difference in standard Gibbs energies
of the respective transition states.
major minor
Rxn. Coord.

The Derivation: Within these limits, we can envision three scenarios:


d[P ] d[PB]
Using the rate equations dt A = k1[A] and dt = k2[B] we can write:
• If both conformers react at the same rate, the product distribution will be
the same as the ratio of conformers at equilibrium.
d[PB] k2[B] k2[B]
= or d[PB] = d[PA] (2)
d[PA] k1[A] k1[A] • If the major conformer is also the faster reacting conformer, the
product from the major conformer should prevail, and will not reflect the
[B] equilibrium distribution.
Since A and B are in equilibrium, we can substitute Keq =
[A]
• If the minor conformer is the faster reacting conformer, the product
ratio will depend on all three variables in eq (2), and the observed product
k2 [PB] k2 distribution will not reflect the equilibrium distribution.
d[PB] = Keq d[PA] Integrating, we get = K (3)
k1 [PA] k1 eq

This derivation implies that you could potentially isolate a product


When A and B are in rapid equilibrium, we must consider the rates of which is derived from a conformer that you can't even observe in the
reaction of the conformers as well as the equilibrium constant when ground state!
analyzing the product ratio.

7-11-C-H Derivation 9/29/03 8:39 AM


K. A. Beaver, D. A. Evans Some Curtin-Hammett Examples Chem 206

Tropane alkylation is a well-known example.


Enantioselective Lithiation:
Me N
Me N
N i-Pr2N O
H i-Pr2N O N
less stable more stable Li
s-BuLi (-)-Sparteine
Me
Me

13
faster Me–I 13
Me–I slower Because sparteine is
chiral, these two
(-)-Sparteine complexes are
13
Me + Me Me + 13
Me diastereomeric and
N N have different
major product minor product properties.
i-Pr2N O i-Pr2N
Li•sparteine O
Li•sparteine

Me
Me
The less stable conformer reacts much faster than the more stable
p.
p. conformer, resulting in an unexpected major product!
faster Cl
JOC 1974 319 slower

Oxidation of piperidines: i-Pr2N O


i-Pr2N O
Me
Me
N N Me
Me3C Me3C Me
less stable more stable
H Keq = 10.5 H
82 - 87% ee

slower k1 H2O2 k2 faster


Enantioselectivities are the same, regardless of whether or not the starting material is
Me O– chiral, even at low temperatures. Further, reaction in the absence of (-)-sparteine
results in racemic product.
N+ N
Me3C O– Me3C + Me
Note that the two alkyllithium complexes MUST be in equilibrium, as the
H H
minor product enantioselectivity is the same over the course of the reaction. If they were not
Ratio: 5 : 95 major product equilibrating, the enantioselectivity should be higher at lower conversions.

When the equilibrium constant is known, the Curtin-Hammett derivation This is a case of Dynamic Kinetic Resolution: Two enantiomeric alkyl
can be used to calculate the relative rates of reaction of the two lithium complexes are equilibrating during the course of a reaction with an
electrophile.
conformers. Substituting the above data into [PB]/[PA] = k2K/k1, the ratio
k2/k1 ~ 2.
Beak, Acc. Chem. Res, 1996, 552
Note that in this case, the more stable conformer is also the faster reacting conformer!
Tet. 1972 573
7-12-Simple C-H examples 9/29/03 8:38 AM Tet. 1977 915
K. A. Beaver, D. A. Evans Mechanism of Asymmetric Hydrogenation Chem 206

P S MeO2C NHAc
The asymmetric hydrogenation of prochiral olefins catalyzed by
Rh
Rhodium is an important catalytic process.
S,S P S Ph

coordination coordination
MeO2C NHAc [L2Rh]+ MeO2C NHAc
MeO2C CO2Me
NH HN
Ph Ph P P
> 95% ee Rh minor major Rh
Ph Ph
P P
O O
Enantioselectivities are generally very high when the ligand is a chelating Me
Me
diphosphine. (ee's are given for S,S-CHIRAPHOS)

When a chiral ligand is used, there are two diastereomeric complexes which hydrogen hydrogen
addition faster slower addition
may be formed:
MeO2C CO2Me MeO2C CO2Me
NH HN P NH HN P
P P
Rh H P
p.
p. * Ph Ph Rh
* Rh Ph Ph Rh
P P
O O P H
Me O O
Me major complex H
minor complex 1 Me Me H
2
(NMR, X-Ray)

H2 migration +S +S migration
fast slow H2

P CH2Ph P
PhH2C
MeO2C NHAc H CO2Me MeO2C
MeO2C NHAc H
R Rh R S Rh
S P S S P
Ph
Ph O NH HN O
observed product
Me Me
Observations:

• Complex 2 is the only diasteromer observed for the catalyst-substrate complex


(1HNMR, X-Ray crystallography) in the absence of hydrogen reductive -L2RhS2 reductive
-L2RhS2 elimination
elimination
• The enantioselectivity is strongly dependant on the pressure of H2, and
degrades rapidly at higher hydrogen pressures
• The observed enantiomer is exclusively derived from the minor complex 2 MeO2C NHAc MeO2C NHAc
R S
These observations may be explained using the Curtin - Hammett Principle
Ph Ph
> 95% ee Halpern, Science, 217, 1982, 401
7-13-Halpern Mechanism 9/25/01 9:59 PM
K. A. Beaver, D. A. Evans Reactions Involving Interconverting Isomers Chem 206

The Curtin-Hammett treatment can be extended to ANY case where different


products are formed from two rapidly intereconverting starting materials,
whether they are conformers, tautomers or isomers.

kA "It was pointed out by Professor L. P. Hammett in 1950 (private


k1 k2
PA A B PB communication) that ..."
kB
major minor
David Y. Curtin, 1954

Stannylene ketals provide an efficient way to acylate the more hindered site of 1,2-diols.
" Because Curtin is very generous in attributing credit, this is
Ph O
O O sometimes referrred to as the Curtin-Hammett principle rather
Cl
SnBu2 Cl than the Curtin principle."
O
O 2N
O 2N
Louis Plack Hammett, 1970

p.
p. Ph OCOAr Ph Ph OSnBu2Cl
O
Ar
OSnBu2Cl
OSnBu2Cl O OCOAr Curtin - Hammett Principle: The product composition is
more stable less stable not solely dependent on relative proportions of the
Ratio 2:1 conformational isomers in the substrate; it is controlled by
Ar= p-NO2C6H4 the difference in standard Gibbs energies of the
TMS-Cl faster slower TMS-Cl respective transition states.

Ph OCOAr Ph OTMS

Product Ratio 22:1


OTMS OCOAr
THE TAKE-HOME LESSON:

The two stannyl esters are in equilibrium at room temperature, and the Never assume that the most stable conformation of a
more stable isomer is initially formed more slowly. The stannyl esters are compound is the most reactive. It may be, but then again,
allowed to equilibrate before quenching with TMS-Cl, which reacts more it may not.
rapidly with the less hindered primary alkoxystannane.

JOC 1996, 5257

7-14-Isomer examples 9/25/01 10:00 PM


D. A. Evans Olefin Addition Reactions: Part–1 Chem 206

■ Problems of the Day: (To be discussed)


http://www.courses.fas.harvard.edu/~chem206/ Rationalize the stereochemical outcome of these reactions.

OH OH OH
9-BBN
Chemistry 206 Me2CH H2O2
Me2CH diastereoselection 24:1
Me Me
Advanced Organic Chemistry
W. C. Still & J. C. Barrish, J. Am. Chem. Soc. 1983, 105, 2487.

Lecture Number 8 Me NHCONHPh m-CPBA


Me NHCONHPh
Ph Ph
CH2Cl2, 0 °C
75 % O
Olefin Addition Reactions–1
Roush, J. Org. Chem. 1987, 52, 5127. Diastereoselection = 95 : 5
■ Hydroboration

■ Epoxidation & Directed Epoxidation Web Problem 163: The following is an idea that has been proposed to you by a fellow
student. The proposal is based on the fact that borane–methyl sulfide complex is an
effective hydroboration reagent (eq 1). It is proposed that homoallylic sulfides such as
■ Reading Assignment for week that illustrated below should be capable of "directing" the hydroboration process from
this substituent through the borane-substrate complex.

A. Carey & Sundberg: Part B; Chapter 4 H 2B


"Electrophilic Additions to C–C Multilple Bonds" further reaction with (1)
-SMe2 cyclohexene
K. Houk, Science. 1986, 231, 1108-1117
Theory & Modeling of Stereoselective Organic Reactions (Handout)
Me2S–BH3 + SMe Me BH3 Me
S S
K. Houk, Tetrahedron. 1984, 40, 2257-2274
Theoretical Studies of Stereoselective Hydroboration Reactions (Handout) –Me2S directed
hydroboration (2)
Hoveyda, Evans, Fu, Chem Rev. 1993, 93, 1307-1370 +Me2S
Substrate-Directable Chemical Reactions ??? BH2
(Electronic Handout)
Me Me Me
E. Vedejs, JACS 2003, 125, 10502-3
A Mechanistic Alternative for the Intramolecular Hydroboration of Homoallylic + Part A. In order to begin your critique, you must possess a good working knowledge of the
Amine and Phosphine Borane Complexes details of the hydroboration of olefins with borane–methyl sulfide. Provide a clear depiction
(Electronic Handout) of the transition state for the hydroboration process using ethylene as the olefinic substrate
and borane–methyl sulfide as the hydroborating agent.
Wednesday,
D. A. Evans October 1, 2003 Part B. Now, based on your knowledge of the hydroboration reaction and the principles
learned thus far in Chem 206, critique the idea proposed in Eq 2. You must concisely state
the logic upon which you base your assessment. Pictures speak a thousand words.

8-00-Cover Page 10/1/03 8:02 AM


D. A. Evans Olefin Addition Reactions: Introduction Chem 206

Representative Trans-Addition Processes


Representative Cis-Addition Processes
■ Hydrometallation ■ Halogenation
M H Br R
R R H
C C R + M H R C C R C C R + Br Br R C C
H H H H
M = B, Al, etc H H H Br

■ Hydrogenation
H H ■ Oxy–metallation (M = Hg(II), Tl(III)
R M-catalyst
C C R + H H R C C R
H H RO R
H H R H
C C R + Hg(OR)2 R C C
H H
■ Group Transfer (epoxidation) H Hg–OR
O
R
C C R + RO2H R C C R ■ Oxy–sulfenation (M = S(II), Se(II)
H H –ROH H H
RO R
■ Group Transfer (dihydroxylation) R
C C R
H
O O + R–S–X R C C
H H
Os H S–R
R
C C R + OsO4
O O
Attributes:
H H –N2 R C C R
H H Each process may proceed via an bridged X
■ Group Transfer (cyclopropanation) intermediate where X is the initiating electrophile R C C R
R2
C H H
R M-catalyst Olefin substitution may disrupt bridging
C C R + R2C=N2 R C C R
H H –N2 H H
■ Addition of hydrogen halides
■ Cycloadditions (one of many!) R O
R H R
R H R
C C R + R2C=C=O R C C R
R
C C R H H
H H + H–X R C C + H C C
–N2 H H H
H
H X R X
Attributes: Attributes:

Each process adds to the C=C via a stereospecific process Process may proceed via an bridged intermediate H
where H+ is the initiating electrophile
R C C R
Intermediates may be involved in some of the indicated reactions H
H
Olefin substitution, reaction conditions as well as
halide type may disrupt bridging
8-01-Olefin addn-intro 9/29/03 11:29 AM
D. A. Evans Allylic Strain & Olefin Hydroboration Chem 206

The basic process Hydroborations dominated by A(1,3) Strain


S S ‡ OH
H B CH2OBn B2 H 6 CH2OBn
H H H O O
H B H 2B H2O2
H δ– Me Me
H Me Me
R C C R
R R R R diastereoselection 8:1
C C C C R R
R R R δ+ R OMe OMe OH
Me B2 H 6
O H2O2 O OH
Response to steric effects: Here is a good calibration system: Me Me
OH Me Me Me
A diastereoselection 12:1
Oxidant Ratio, A:E Reference Y. Kishi & Co-workers, J. Am. Chem. Soc. 1979, 101, 259.
CH2
MCPBA 69:31 JOC, 1967, 32, 1363
Me3C JOC, 1970, 35, 2654 OH
BH3, H2O2 34:66
p.
p. E B2 H 6
H BnO OH BnO OH
H2O2
Acyclic hydroboration can be controlled by A(1,3) interactions: Me Me Me Me Me Me
Diastereoselection = 3:1
OH
B2 H 6
RL RL C. H. Heathcock et. al. Tetrahedron Lett 1984 25 243.
OH OH major diastereomer
H2O2
RM Me RM Me

A(1,3) allylic strain Me Me


control elements Me Me
Steric effects; RL vs RM ThexylBH2,
TrO OTr
Staggered transition states then BH3 OH OH OH
OH
H TrO OTr Diastereoselection; 5:1
H RM
B H
RM H Me
Me
H C C Me Me Me Me ThexylBH2, Me Me Me Me
major H C C minor
CH2OR CH2OR
RL H
then BH3 TrO OTr
RL B H
H
H OH OH OH OH OH OH OH OH
TrO OTr
Diastereoselection; 4: 1
Houk, "Theoretical Studies of Stereoselective Hydroboration Reactions"
Tetrahedron 1984, 40, 2257 (Handout) Still, W.C.; Barrish, J. C. J. Am. Chem. Soc. 1983, 105, 2487.

8-02-hydroboration-1 9/29/03 11:31 AM


D. A. Evans Allylic Strain & Olefin Hydroboration Chem 206
Hydroborations dominated by A(1,2) Strain ■ Case II: Dialkylboranes
R
Me R
R2BH
H B Me
RL OH RM H R2BH
H2O2 TS1 H RL OH
Me Me C C H2O2
R2BH structure is major
RL RM H
a potential variable RM favored for R2BH
Me RL
RM R2BH R
RL OH R
H2O2 B H
RM H Me
RM H R2BH
TS2 C C Me RL OH
H H2O2
Houk's rules: Orient RL anti-periplanar to incoming reagents to avoid TS eclipsing: minor
RM
RL
■ Case I: Borane H
Midland finds that TS1 favored for R2BH reagents, but TS1 ~ TS2 for BH3
H B H
RM H Me Others have found that TS1 favored over TS2 for BH3
B BH3
TS2 H RL OH
C C Me H2O2
major H Representative Examples
p.
p. RM
RL favored for BH3
H
A(1,2) Strain H Me Me
H B Me CH2 CH2OH
A RM H BH3 Me Me
TS1 H RL OH H 9-BBN H
Me C C H2O2
minor H H2O2 H
H RM
RL H H

from Lecture 4: The Torsional Energy Profile R2BH diastereoselection


H
Me borane methylsulfide 1:1
Φ = 180 H C C Me M. M. Midland & Co-workers,
thexylborane 4:1
H J. Am. Chem. Soc. 1983, 105, 3725.. 14 : 1
9-BBN
H dicyclohexylborane 26 : 1
Φ = 50
Me OH OH OH
H 9-BBN R = n-Bu: diastereoselection 11:1
C C Me
H H R H2O2
R R = CHMe2: diastereoselection 24:1
H Me Me
Φ = 110 +2.68
Φ=0 kcal Model is consistent if you presume HO = RM: R = RL
H +1.39 kcal Me
H H
C C Me C C Me
H Me H H
H
+0.06 kcal H W. C. Still & J. C. Barrish, J. Am. Chem. Soc. 1983, 105, 2487.
Φ=0 Φ = 180

8-03-hydroboration-2 10/1/03 8:17 AM


D. A. Evans Allylic Strain & Olefin Hydroboration Chem 206

■ Case I: Dialkylboranes R ■ Case II: Borane H


R
H B H
Me H B
RM H R2BH Me
TS1 RL OH A RM H BH3
H TS1 H RL OH
major Me C C H2O2 H2O2
Me C C
H RM favored for R2BH minor
H RM
RL
R RL
H
R B H
Me H B H
RM H Me
H R2BH B RM H
TS2 C C Me RL OH TS2 H
BH3
minor C C Me RL OH
H H2O2 H2O2
major H
RM
RL RM
RL
favored for BH3
OMe OMe
Me Me OMe
Me Me
Me Me
A B O D
HO2C E OMe Evans, Ratz, Huff, Sheppard, JACS 1995, 117, 3448-3467.
O 9 O
C O O
p.
p. OH H H Me H F
Me Me O
Me
Lonomycin A Me OH
C-9 → C10

OMe
5 H H
Me Me H
O OH OH H B H H B
RO H RO H
BH3•SMe2 10 H
H
85% XP 1 O 9 C C Me Me C C
OMe H H H
Me Me Me
OMe RL RL
5 diastereoselection TS1 favored TS2 disfavored
Me Me 92 : 8
O O OH
10 OMe R R
O N 1 O 9 5
OMe H Me Me R B H H B R
Me Me Me O OH OH RO H RO H
Bn 10 H H
XP 1 O 9 C C Me Me C C
OMe H H H
9-BBN Me Me Me RL RL
60%
diastereoselection TA1 disfavored TA2 favored
> 95 : 5

8-04-hydroboration-3 9/29/03 11:33 AM


D. A. Evans Represetative Hydroboration Examples: Acyclic Control Chem 206

Me O
For each of the examples shown below, attempt to rationalize the stereochemical Me CO2H H
1. 9-BBN
outcome of the reaction in terms of one of the models presented in the discussion. O
CH2
2. H2 O2 , NaOH
Me H
Me Me Me
OH Wolinsky, J.; Eustace, E. J.
J. Org. Chem. 1972, 37, 3376. Diastereoselection = 7:1
B2H6
Me R
Et O O H2O2 Et O
Et H Et Et O
Et OH Et Me CO2H Me
1. 9-BBN
"one isomer" O
Y. Kishi & Co-workers, J. Am. Chem. Soc. 1978, 100, 2933. CH2
2. H2 O2 , NaOH
Me H
Me
Me Wolinsky, J.; Nelson, D.
Me
O Me Tetrahedron. 1968, 25, 3767. Diastereoselection = 10:1
O Me
Me O BH3 .THF
Me O
Me Me
p.
p. B2H6 / [ O ] H
HO Me Me
Mori, K.
Tetrahedron 1976, 32, 1979 diastereoselection 12:1
OH CH2 OH
OH
Diastereoselection = 19:1
R
R
H
O H Me Me
O HO O
. O
CH3 BH3 THF B2H6 / [ O ] H
CH3 Me Me
O
CH3 O
OBn CH3 OH OH
OBn CH2 OH
Diastereoselection = 32:1
Oikawa et. al. R=H; Diastereoselection = 6.8:1
Tetrahedron Lett. 1983, 19, 1987. R=OBn Diastereoselection = 6.6:1
Me Me

B2H6 / [ O ] H
Me Me
R OH R OH
OH CH2 OH
BH3.THF OH
OH Diastereoselection = 10:1
H
Me
Me
Birtwistle et. al.
R = CH3; Diastereoselction = 6.7:1
Me H
Tetrahedron Lett. 1986, 25, 243. R = isopropyl "One Compound" B2H6 / [ O ] Me
OH CH2
Schulte-Ette, K.H.; Ohloff, G. OH
OH
Helv. Chim. Acta 1967, 50, 153.
Diastereoselection = 4.6:1
8-05-hydroboration-4 9/29/03 12:38 PM
D. A. Evans Representative Hydroboration Examples: Cyclic Systems Chem 206

CH2 OH OH
BH3.THF OH H H
O O
BnO BH3.THF BnO
HO H HO H
Me3C Me3C
OH
Diastereoselection = 2.1:1 O O
Me Me
B. Fraser-Reid et. al.
J. Am. Chem. Soc. 1984, 106, 731.
Major isomer; no ratio given.

CH2
BH3.THF OH
Me Me
Me3C Me3C CH3
Me Me H CH3
O
Diastereoselection = 1.2:1 N O H
N
H H BH3.THF
H H
CH2 OH
O
O O
p. O
p. CH2 .
BH3 THF OH
Sallay, S. I.
J. Am. Chem. Soc. 1967, 89, 6762. 90% yield, no diastereoselection given
Me3C Me Me3C Me
Diastereoselection = 3.3:1

Me Me
BH3.THF
CH2
BH3.THF OH Me Me OH
O CH2 O H
H H
Me3C Me Me3C Me CO2Me CO2Me
Diastereoselection = 2.4:1
Ley, S. et.al. 55% yield with the diastereomeric alcohol
J. Chem. Soc. Chem. Commun. 1983 630.
produced in an unspecified amount.
Recycling of the minor isomer further
provided 15% of the desired material

CH2
BH3.THF OH N–NHAr N–NHAr
Me . Me
BH3 THF
Me Me
Diastereoselection = 4.9:1 Me Me
Y. Senda et. al. (Compare with H.C. Brown's
Tetrahedron 1977, 33, 2933. case, with 9-BBN; 1.5:1) H H
Me Me OH
McMurry, J. E.
J. Am. Chem. Soc. 1968, 90, 6321. Minor diastereomer not detected
8-06-hydroboration-5 9/29/03 12:39 PM
D. A. Evans Directed Reactions: An Introduction Chem 206

Stereochemical Control Elements for all reactions Heteroatom-directed Reactions


Mechanism-based: (HO & C=C must be allylic)
■ Steric & Electronic Factors
■ Stereoelectronic Considerations O
[3,3]
■ Associative Substrate-Reagent Interactions R R
R

OH O CH2 R
■ Steric control: A B
disfavored
R Claisen Rearrangement
R R
A via Reagent Ligation
A B C C
H H
H Et2Zn
B
favored CH2
CH2I2
favored product A B R R
R
OH O CH2I
Zn OH
p.
p.
Nonbonding Interactions disfavor the syn diastereoface ? Simmons-Smith Reaction

Directed Reactions Hydroxyl-directed Reactions


Review: Hoveyda, Evans, Fu Chem. Reviews 1993, 93, 1307 MCPBA
ratio 92 : 8 Henbest
Cl CO3H J. Chem. Soc. 1958, (1957)
■ Associative Substrate-Reagent Interactions
A B O
X X A B X favored
A OH
A B t-BuOOH
Sharpless
C C
VO(acac)2
ratio 98 : 2 JACS 95, 6136, (1973)
H H OH
B H
favored product
disfavored
Noncovalent Interaction favors the syn diastereoface CH2
A B Et2Zn

Directed Oxidations CH2I2 Winstein


OH
Epoxidation JACS 91, 6892, (1969)
Hydroboration ratio 90 : 10
Agenda Directed Reductions
Me Me (Ir+) Stork
Hydrogenation M(I) + H2
Hydride reduction JACS 105, 1072 (1983)
Directed C–C Bond Constructions (Rh+) Evans
OH OH JACS 106, 3866 (1984)
8-07-Directed Rxns/intro 10/1/03 8:19 AM
D. A. Evans Directed Reactions: An Introduction Chem 206

Peracid Epoxidation
Orientation of the Directing Group A. Rao in Comprehensive Organic Synthesis, Trost, Ed., 1992, Vol 7, Chapter 3.1
H
reagent X ■ General Reaction:
H Me Me
X = O, CH2 R R O R R O
Me OH Me OH + ●
OH ●
O +
R O R OH
R R R R
‡ ‡ HOMO LUMO note labeled oxygen is transferred
∅maj Me H πC–C σ*O–O O-O bond energy: ~35 kcal/mol
H
C C H
LUMO HOMO
Me
π∗C–C O lone pair
Reag OH Reag
OH
■ Transition state: What about lone pairs. [Consider ● to be Sp2 hybridized].
H
C C H ∅min R
Me R R
H Me
R O
O O O
TSmajor TSminor O O
O ••
•• O H ●
O
H ●
O H ●
O
•• ● ••
H O
R R C H C H
Orientation of directing group is not the same for all reactions C H C H R C R C
R C R C H H
H H
HOMO πC–C HOMO: O lone pair
Reagent Selectivity Φ Estimate LUMO: σ∗O–O LUMO:π∗ C–C
t-BuO2H, V +5 71 : 29 ~ 50 °
■ Reaction rates governed by olefin nucleophilicity. The rates of epoxidation
RCO3H 95 : 5 ~ 120 ° of the indicated olefin relative to cyclohexene are provided
CH2I2, Zn–Cu > 99 : 1 ? below: OH
OH OAc

The transition state bite angles for the above reactions are either 1.0 0.6 0.05 0.4
not known or have been only crudely estimated.
■ The indicated olefin in each of the diolefinic substrates may be oxidized
The "best guesses" are provided. selectively.
Me Me Me

Me

Me
H
Me Me Me Me
8-08-Directed Rxns/intro-2 10/1/03 9:27 AM
D. A. Evans Diastereoselective Peracid Epoxidation Chem 206

The Directed Peracid Epoxidation


Stereoelectronic Implications of intramolecular Peracid Epoxidation
■ Transition State Hydrogen Bonding: Substrate as H-bond donor (Henbest)
■ Per-arachidonic acid Epoxidation: Corey, JACS 101, 1586 (1979) R R R

O O O O O
O
O ●
O H
H H ●
O
O R H
R●
H
O H •• •• C O
O H
H
C
R C O
R H C R C
H
C H2 C
O H2
R C
C
H2 require allylic or homoallylic alcohol

■ Transition State Hydrogen Bonding: Peracid as H-bond donor (Ganem)

p.
p. CF3 CF3 ‡
CF3

O O O O O O

H ●
O H ●
O H
H R
H●
H R O C O
R O R R R
C O C
C C
C C H2 H C C
H2 H H
H2

require more acidic peracid both allylic alcohols and ethers OK

Syn : Anti Syn : Anti Syn : Anti Syn : Anti


OH (m-CPBA) (CF3CO3H) OTBS (m-CPBA) (CF3CO3H)

24 : 1 50 : 1 1:7 5:1

OH OTBS

O
H 24 : 1 100 : 1 1:8 12 : 1
O O Me3C Me3C
OH OTBS

5:1 100 : 1 1:4 1:6


Me3C Me3C
Ganem Tet. Let. 1985, 26, 4895
8-09-peracid epoxid-2 9/29/03 12:52 PM
D. A. Evans Diastereoselective Peracid Epoxidation Chem 206

Epoxidation of Cyclic Olefins with Amide &Urethane Directing Groups Epoxidation of Cyclic Homoallylic Alcohols

Substrate Major Product Selectivity Major


Substrate Selectivity
O O
Product

HN Me HN Me OH O OH 9:1

HO HO "highly Me Me
OH OH
O selective"
HO HO "highly
O O selective"
HO HO
Et Me O
HN Ph HN Ph Et Me

Me Me
"highly
O selective"
p.
p. 16 : 1
HO HO
O O O

O R O R a. R = NH2 3:1 Me Me

b. R = NHBn 5:1 1:1


O HO HO
10 : 1
c. R = NMe2 O
Me Me
Me Me
a. R = OCONHBn >20 : 1 21 : 1
b. R = OCONMe2 >20 : 1 HO HO
R R O
O
HOH2C HOH2C
RO RO
a. R = CONH2 6:1 5:1

b. R = CONHBn >10 : 1 AcO AcO


O
AcO AcO 2:1
O c. R = CONMe2 Conditions: Perbenzoic acid, or meta-chloroperbenzoic acid
Conditions: Perbenzoic acid, or meta-chlorobenzoic acid in benzene. in benzene or cyclopentane.

(Table 11, p1316, from the Evans, Hoveyda, Fu review article) (Table 14, p1318, from the Evans, Hoveyda, Fu review article)

8-10-peracid epoxid-3 9/27/01 10:16 PM


D. A. Evans Sharpless Epoxidation (V+5) Chem 206

The Sharpless Epoxidation ■ The literature precedent: Sheng, Zajecek, J. Org. Chem. 1970, 35, 1839

OR R TBHP
RO

O+ OR ●
O ●
O
O
O RO RDS RO OH 80 oC OH OH
V
HO
V ●
O V
O VO(acac)2 4:1
O ●
O Catalyst
O O Mo(CO)6 1:1

–ROH ■ Next step: Sharpless, Michaelson JACS 1973, 95, 6136


ROOH
OR
OH OH
RO Me Me

O VO(acac)2
HO
O V ●
O ●
O Regioselection 20:1
HO Me TBHP Me
OR
80 °C
Me Me

Aldrichimica Acta, 12, 63 (1979) OH OH


p.
p. Mo(CO)6
Stereoselection 98:2
TBHP (90 % yield)
80 °C ●
O
O–C2–C3–C4 = 41°
The Sharpless estimate: ~50°
Relative Rates (Diastereoselectivities) for the Epoxidation of
Cyclohexene Derivatives JACS 1973, 95, 6136

krela,b (diastereoselectivityc )
t-BuOOH t-BuOH Substrate
peracid Mo(CO)6 VO(acac)2
HO
HO O
1.00 1.00 1.00

OR OH
2 tBu
O-OtBu 0.55 (92 : 8) 4.5 (98 : 2) >200 (98 : 2)
1 O O V
3 RO
4 V O O OAc
O
O 0.046 (37 : 63) 0.07 (40 : 60) --
tBu
OH
O
Chem 3D Transition State slow RO 0.42 (60 : 40) 11.0 (98 : 2) 10.0 (98 : 2)
V O
O
O a,b
The relative rate data apply only to a given column.
Values in parenthesis refer to the ratio of syn:anti epoxide.
8-11-Sharpless-1 9/29/03 1:07 PM
D. A. Evans Epoxidation of Acyclic Alcohols Chem 206

■ Allylic Alcohols: OH OH OH
Reagent
Me OH Me OH Me OH Me Me Me Me + Me Me
Reagent
+ ●
O ●
O
Me Me Me

O ●
O Reagent Ratio
threo erythro m-CPBA 64 : 36
t-BuOOH / VO(acac)2 29 : 71
Φ Estimate Reagent Ratio
t-BuOOH / Mo(CO)6 62 : 38
~ 120 ° m-CPBA 95 : 5 t-BuOOH / (t-BuO)3Al 64 : 36
40-50 ° t-BuOOH / VO(acac)2 71 : 29
t-BuOOH / Mo(CO)6 84 : 16 OH OH OH
t-BuOOH
■ RCO3H Transition States: Φ ~ 120 ° R1 R2 R1 R2 + R1 R2
OH VO(acac)2 ●
O ●
O
H SiMe3 SiMe3 SiMe3
TSmajor H H
CH C H C C H TSminor R1 R2 Yield Ratio
Me Me Me Oshima, Tetrahedron Lett. 1982, 23, 3387.
H Bu 84 % 99 : 1
p.
p. Me OH 70 % 99 : 1
C5H11 Me
■ V(+) Transition States: Φ ~ 45 °
Me OH OH
HO
t BuOOH
TSmajor H Me H O O
C C H C C H TSminor VO(acac)2 ●
O only isomer
Me Me H O O
EtO OEt Me 60 % EtO OEt Me
HO Me Me
H
OH OH
t-BuOOH
O ● O
Me OH Me OH Me OH VO(acac)2 O
Reagent O O only isomer
+ EtO OEt Me 60 % EtO OEt Me
Me Me Me Me Me Me Me Me

O ●
O
Depezay, Tetrahedron Lett. 1978, 19, 2869.
threo erythro

O
t-BuOOH
Reagent Ratio Me Me
K. Oshima & Coworkers VO(acac)2
m-CPBA 95 : 5 OH OH
Tetrahedron Lett. 1980, 21, 1657, 4843. HO 60 % HO
t-BuOOH / VO(acac)2 86 : 14 Boeckman, JACS 1977, 99, 2805. Diastereoselection = 7 : 1
K. B. Sharpless & Coworkers t-BuOOH / Mo(CO)6 95 : 5
Tetrahedron Lett. 1979, 20, 4733. t-BuOOH / (t-BuO)3Al 100 : 0
Me NHCONHPh Me NHCONHPh Me NHCONHPh
m-CPBA
Ph Ph + Ph
CH2Cl2, 0 °C
75 % ●
O ●
O

8-12-Acyclic substrates 9/29/03 1:01 PM Roush, J. Org. Chem. 1987, 52, 5127. Diastereoselection = 95 : 5
D. A. Evans Epoxidation of Acyclic Homoallylic Alcohols Chem 206

Anti diastereomer
Homoallylic Alcohols (Mihelich, JACS 1981, 103, 7690)
OH OH OH

O ●
O

O ●
O t-BuOOH
t-BuOOH R2 R2 + R2
OH OH + OH VO(acac)2
VO(acac)2
R1 Me R1 Me R1 Me
Me Me 90 % Me Me Me Me

Diastereoselection > 400 : 1 R1 R2 Yield Ratio


Control Elements
C6H13 Me 92 % 104 : 1
A(1,3) Strain H L
L' Me i-Pr 97 % > 400 : 1
Syn diastereomer
Directed Rxn HV O R ●
O
H O
Me ●
O H HO OH OH OH
Me Me ●
O ●
O
H t-BuOOH
Me R2 R2 + R2
VO(acac)2
R1 Me R1 Me R1 Me
R1 R2 Yield Ratio
OH OH OH

O ●
O C6H13 Me 73 % 70 : 1
p.. t-BuOOH
Me Me + Me Me Me 70 % 85 : 1
VO(acac)2
Et Et Et Me C5H11 81 % 16 :1
Diastereoselection 12 : 1
OH ●O OH OH
●O
Control Elements t-BuOOH
H L Me Me +
L' OH Me
VO(acac)2
Directed Rxn HV O R ●
O
Me C5H11 Me C5H11
O Me C5H11
Me ● H
O Me
Et
E. D. Mihelich & Coworkers Diastereoselection = 211 : 1
J. Am. Chem. Soc. 1981, 103, 7690.
Et

Epoxidation of Homoallylic Alcohols with TBHP, VO(acac)2


Prediction Substrate Product Selectivity
OH OH
H L
L' H L
L' ●O
HV HV 2:1
O R O R Me Me
R2 O Anti should be more R2 H O
● H
O ●
O H
R1 diastereoselective OH OH
than syn ●O
H R1 4.6 : 1
Me Me
Me Me
Anti diastereomer Syn diastereomer Me Me
OH OH
●O
R R
Hex Hex
1.4 : 1
R = (CH2)7CO2Me
8-13-Homoallylic ROHs 9/29/03 1:03 PM
D. A. Evans Epoxidation of Acyclic Homoallylic Alcohols Chem 206

Bishomoallylic Alcohols (Kishi, Tet. Lett. 1978, 19, 2741) Epoxidation & Cyclization of Bishomoallylic Alcohols
R
R Et R Et
OH OH VO(acac)2 AcOH

O ●
O Me
t-BuOOH, VO(acac)2
Me CHMe2 Me CHMe2 iPr OH Me iPr OH Me iPr O
C6H6, RT Et
Et Et ●
OH
diastereoselection ~ 9 : 1
The Kishi Lasalocid Synthesis (JACS 1978, 100, 2933)
H Et Me Me
Et OH O
H Me A H
R Me
R H
● HO O
O ●
O H Et B
V OH CO2H Me Me Et O OH
O O R
Et
Me
Me Et Me
OH OH H

O VO(acac)2 A
Et
p.. t-BuOOH, VO(acac)2 Ar OH Ar O Diastereoselection 8:1
Me CHMe2 Me CHMe2 TBHP Et
C6H6, RT
AcOH ●
OH
Et Me Et Me Me Et
Ar = p-MeOPh
diastereoselection ~ 20 : 1 Me

H Et
2nd stereocenter Me Et Evans X-206 Synthesis JACS 1988, 110, 2506.
Me Me
is reinforcing R H R Me
Me Me Me Me

O

O Me
V OH OH
O O R A B C D E H
Me
O O O O O
H OH OH Me OH Me F
Me OH O Me
OH OH O OH

O OH
t-BuOOH, VO(acac)2 Et
Me CHMe2 Me CHMe2
C6H6, RT
Et Me Et Me Me O OH O OH
diastereoselection ~ 6 : 1 ●
O
Ph VO(acac)2
N Et XN Et
TBHP
O OBn Me C6H6, RT OBn Me
Me Et
O
Me Et HOAc
H Me
R H R Me

O

O
O
V OH D
O O R diastereoselection 20 : 1 Et
(89 %) XN O
Me
OBn ●
OH
8-14-Bishomoallylic ROHs 9/29/03 1:04 PM
D. A. Evans Olefin Addition Reactions: Part–2 Chem 206

■ Problems of the Day: (To be discussed)


http://www.courses.fas.harvard.edu/~chem206/
Predict the stereochemical outcome of the indicated reaction.
O
Chemistry 206 Me
HOOC O
I2, NaHCO3
Me
Advanced Organic Chemistry Me Me Me
Me I
Kinetic Control: 3 eq. I2, MeCN, NaHCO3, 0°C diastereoselection: 20:1
Lecture Number 9

Bartlett, P. A.; Richardson, D.; Myerson, J. Tetrahedron 1984, 12, 2317


Olefin Addition Reactions–2

■ Epoxidation & Directed Epoxidation


Rationalize the stereochemical outcome of the indicated reaction.
■ Hydrogenation
■ Olefin Bromination N M–H N H Me N OH
Me Me
■ Reading Assignment for week O
OH H
A. Carey & Sundberg: Part B; Chapter 4
"Electrophilic Additions to C–C Multilple Bonds" R2AlH 97 : 3
R. Noyori
Hoveyda, Evans, & Fu (1993). Substrate-directable chemical Bull. Chem. Soc. Japan 47, 2617, (1974) LiAlH4 28 : 72
reactions. Chem. Rev. 93: 1307-70 (Handout)

J. M. Brown, Angew. Chem. Int. Edit. 26, 190-203 (1987) (Handout)

Investigation of the early Steps in Electrophilic Bromination through the


Study of the Reaction of Sterically Encumbered Olefins
R. S. Brown, Accts. Chem. Res. 1997, 30, 131 (handout)
Bromoniun Ions or β-Bromocarbocations in Olefin
Bromination. A Kinetic Approach to Product Selectivities
M-F. Ruasse, Accts. Chem. Res. 1990, 23, 87 (handout)

Friday,
D. A. Evans October 3, 2003

9-00-Cover Page 10/2/03 3:38 PM


D. A. Evans Diastereoselective Hydrogenation: Introduction Chem 206

The Hydrogenation Reaction Polar functional groups may play a role in controlling the diastereoselectivity
of the hydrogenation process;
Relevant Review articles: J. M. Brown, Angew. Chem. Int. Edit. 26, 190-203 (1987). however, the control elements were not well-defined.

CHMe2 CHMe2
■ General Mechanism
M
M
R R R
M(0) + C C R R C C R C C O H2, Pd-C O
H H H H only isomer
H H
H H
CH3 CH3
H
H H H M H M H however CHMe2 CHMe2
M(0) + R C C R R
R C C R C C R
H H H H H H OH
H2, Pd-C OH
trans:cis = 55:45
Historically, primary stereochemical control designed around analysis of H H
steric environment in vicinity of C=C. CH3 CH3
p..
However, the influence of polar effects was documented
J. E. McMurry & Co-workers, Tetrahedron Lett.. 3731 (1970)

O CO2Et O CO2Et

O H2 Pd-C O H
HO
EtOH 10% Pd-C
H H H
H2 N
OMe OMe Steric Control
H
LiAlH4 trans : cis HO H
sole product
Pd(0) Pd(II) 85 : 15 H OH
N

O CH2OH O CH2OH H
HO
O H2 Pd-C O O
H H
EtOH 5% Pd-Al2O3 N
H Directed ?
H2
OMe OMe 12 : 1
H
OH
trans : cis
Thompson, J.Org. Chem. 36, 2577 (1971) 5 : 95
Y. Kishi & Co-workers, J. Am. Chem. Soc. 102, 7156 (1980)

9-01-Hydrogenation-1 10/2/03 3:22 PM


D. A. Evans Diastereoselective Hydrogenation: Introduction-2 Chem 206

The first rational attempt to identify those FGs which will direct the reaction Cationic Hydrogenation Catalysts
H

O H2, 5% Pd-C O

Rh BF4
CH3O CH3O –
R O R O Ph2P PPh2 Ir PF6
10 Py PCy3
(CH2)n
R cis : trans Schrock & Osborne, R. Crabtree
J. Am. Chem. Soc. 91, 2816 (1969) J. Organomet. Chem. 168, 183 (1979)
CH2OH 95 : 5
CHO 93 : 7
CN 75 : 25
55 : 45 S S S S = solvent
COONa
18 : 82 H2 S
COOH Rh

BF4 H2
COOMe 15 : 85 Ph2P PPh2 H Rh PPh2
COMe 14 : 86 H
10 : 90 (CH2)n
Ph2P
CONH2 16-e– 18-e-
p.. H. Thompson & Co-workers, J. Am. Chem. Soc. 95, 838 (1973) Rh(+I): d8

The first rational attempt to associate catalyst with substrate: Mechanism of Hydrogenation Cationic Rhodium-(I) Catalysts.

CH2OK CH2OH S = solvent


S S
(Ph3P)3RhCl CH2=CH2 S
S Rh PPh2 Rh PPh2
H2 100 psi H
MeO 50 °C, C6H6 MeO Ph2P Ph2P

cis : trans H2
>98 : 2 CH3–CH3 Oxidative Addition
Reductive Elimination (–S)
(+S)

CH2O–Rh(PPh3)3 H
H
S
H S H Rh PPh2
Rh PPh2
Rxn Catalytic in Rh (4 mol%) H 2C
Ph2P
Ph2P
MeO

Thompson & Coworkers, J. Am. Chem. Soc. 97, 6232 (1974)

9-02-Hydrogenation-2 10/2/03 3:24 PM


D. A. Evans Diastereoaselective Hydrogenation: Cationic Catalysts Chem 206

D. A. Evans & M. M. Morrissey JACS 106, 3866 (1984)


Mechanism of Hydrogenation Cationic Rhodium-(I) Catalysts.
OH OH

S S S H2

2H2 S CH2Cl2
Rh

BF4 H2 CH3 CH3
H Rh PPh2
+2 S Ph2P PPh2

Rh BF4 H
+ Catalyst Mol% Catalyst H2 Pressure trans:cis (Yield)
Ph2P PPh2 - Ph2P
16-e- 18-e
17.5 15 psi H2 200 : 1 (89%)
P Rh(DIPHOS-4)+
P 3.5 375 psi H2 300 : 1 (95%)
+
HA Rh OH
H2 HB 20.0 15 psi H2 50 : 1 (82%)
R2
OH H Ir(pyr)PCy3
C C R2 2.5 15 psi H2 150 : 1 (85%)
CH2 H H H
p..
P Which hydrogen
Rh migrates ?? OH OH CH2OH
P Rh + CH2OH
P Rh +
H2
+ CH3 H2
CH3 CH3 CH3
P Rh OH 65 : 1
R2 CH3 H 19 : 1
OH Rh(DIPHOS-4)+ H2 1000 psi CH2Cl2
C
CH3 H H
R2 Excessive Steric Hindrance
Me
Me H Me Me
A potential stereoelectronic effect H
Me
OH OH
P P Me H H
CO2H Me Me
P P Me Me Me
+ }

HA Rh OH HB Rh Retigeranic Acid
+ H
HB HA O Me Me
H HA R2
C C R2 Rh +
H H H H H2 75 : 1 (95%)
H OH OH
H
Me Me
That H atom lying parallel to the pi-system (HA) should migrate preferentially Me Me
if the dihydride is an intermediate. Rh(DIPHOS-4)+ H2 800 psi THF
THF is important to success of rxn to buffer the Lewis acidity of the catalyst which
causes elimination of ROH under normal conditions
9-03-cationic H2 mech 10/2/03 3:25 PM
D. A. Evans Diastereoaselective Hydrogenation: Cyclic Substrates Chem 206

Polar functional groups other than OH may also


direct the process O O

CO2Me CO2Me X X Diastereoselection


Ir(pyr)Pcy3+ X
Rh(DIPHOS-4)+ diastereoselection OMe 55:45
H2
91 : 9 H2
NC4H8 99:1
H 2C CH3
Me Me

CO2Me CO2Me
X Diastereoselection
O O
Ir(pyr)Pcy3+
diastereoselection CH3 CH3 OMe 99:1
X Ir(pyr)Pcy3+ X
H2 89:11
CH3 CH3 CH3 CH3 NC4H8 >99:1
H2

A.G. Schultz and P.J. McCloskey, J. Org. Chem., 1985, 50, 5907.
p.. O N O N
Ir(pyr)Pcy3+
diastereoselection
H2 >99:1
Me Me
H OCH3 15 psi H2 OCH3
J.M. Brown and S.A. Hall, J. Organomet. Chem., 1985, 285, 333. Ir(pyr)Pcy3+
diastereoselection
O
>99:1
Me O Me CH3 CH3
H H
N Ir(pyr)Pcy3+ N
diastereoselection R.H. Crabtree and M.W. Davis, J. Org. Chem., 1986, 51, 2655.
H2 >99:1
H N H
N H
H O O
H H
CONC4H8 CONC4H8
CH2OMe CH2OMe 15 psi H2
N N Ir(pyr)Pcy3+
Ir(pyr)Pcy3+ diastereoselection
CH3 O CH3 O >99:1
CH3
diastereoselection
CH3
H2 >99:1 CH3 CH3

A.G. Schultz and P.J. McCloskey, J. Org. Chem., 1985, 50, 5907.
A.G. Schultz and P.J. McCloskey, J. Org. Chem., 1985, 50, 5907.

9-04-Cyclic substrates 10/3/03 8:33 AM


D. A. Evans Diastereoaselective Hydrogenation: Acyclic Substrates Chem 206

Acyclic Allylic Alcohols P OH


OH P +
+ P Rh OH
P P Rh H2 R2 R1
R2 R1
+ OH H
P C C CH2R2 Me
+ P Rh OH CH2 H
P Rh H2 R1 H
R2 R1 anti > 93 : 7
H
favored C C CH2R2 Me
H
R1 H P
anti
OH OH + OH
P P Rh OH
R2 R1 +
R2 R1 P Rh R2 R2 R1
C C CH3 H2
CH2 CH3 H Me
R1 H
R1
syn > 91 : 9
P H OH
H
+ C C CH2R2
P Rh H2
H
R2 R1 OH O Me OH
disfavored +
P Rh OH H2
p.. Me Ph COXn
syn R N Rh(DIPHOS-4)+ R
P
CH2 Me O Me Me
D O anti

low pressure
P
P + OH
+ P Rh OH Me OH
P Rh H2 OH O H2
R2 R1 Rh(DIPHOS-4)+ COXn
favored R2 R N
Ph R
C C CH3 Me
H Me O Me Me
R1 H syn Me syn
OH
O
T
R2 R1 Anti : Syn Ratio
Me Hydroxy-Olefin 15 psi H2 640 psi H2

R = CH3 25 : 75 (23%) 93 : 7
P H R1 OH
R2
+ C C Me D R = (CH3)2CH 52 : 48 (35%) 94 : 6
P Rh H2
H R2 R1
disfavored + R = Ph 71 : 29 (-) 93 : 7
P Rh OH Me

anti R = CH3 13 : 87 (6%) 9 : 91


P
T
R = (CH3)2CH 12 : 88 (8%) 8 : 92
D. A. Evans & M. M. Morrissey JACS 106, 3866 (1984)
R = Ph 21 : 79 (-) 6 : 94
9-05-Acyclic ROH's 10/3/03 8:41 AM
D. A. Evans Diastereoaselective Hydrogenation: Acyclic Substrates Chem 206

Homoallylic Alcohols Evans, Morrissey Tetrahedron Lett. 26 6005 (1985) The Premonensin Synthesis

P OH HO
R H Me
+ H2 Me
P Rh C C H Me
Me R O OH OHMe
favored P Rh C Me
CH2 Me Me
O+ HO O
P Et O
OH H Me Me
syn Me Me
R

Me Me Catalyst Ratio
P H EtO2C Me
OH +
O+ Rh(DIPHOS-4) 85 : 15
disfavored P Rh CH2 H2
H2 Me Me EtO2C Me
R
R
Rh + Rh(–)(BINAP) + 65 : 35
P HO HO
C C H Me Me +
+ Me Me Rh(+)(BINAP) 98 : 2 (90%)
P Rh Me H Me

p.. anti
A(1,3) destabilization Evans, DiMare, JACS, 1986, 108, 2476)

HO OTBS HO OTBS The Ionomycin Synthesis


Me Me Me Me

A syn Me Me
OH OH
Me
Me O O O
HO H Me H Me
HO OTBS HOOC ✻ Me Me OH
Me Me Me O
OTBS anti Me
B Me Me

Me
Olefin Catalyst (H2 Pressure) syn : anti
CH3O2C ✻ H2
OH
A Rh(DIPHOS-4)+ (1000 psi) 95 : 5 Me Me Me Rh(DIPHOS-4)+
Me
CH3O2C ✻
A Ir(pyr)PCy3+ (15 psi, 2.5 mol%) OH
73 : 27
Me Me Me
Rh(DIPHOS-4)+ (1000 psi)
B 9 : 91 Diastereoselection: 94 : 6 (93%)

with Dow, Shih, Zahler, Takacs, JACS 1990, 112, 5290


}

9-06-Acyclic ROH's 10/3/03 8:39 AM


D. A. Evans Olefin Bromination-1 Chem 206

■ Bromonium ion origin of the anti (trans) selectivity first suggested by


R R Br2 R Roberts, JACS 1937, 59, 947
Br R
Introduction C
R R C Br
R R + R
R R Br2 Br R
R Br
■ Reaction is first order in alkene R
C C
C
R R C Br
At low concentrations of Br2, rxn is also first order in Br2 R R R
– R
Br
At higher concentrations of Br2 in nonpolar solvents rxn is second order in Br2.

■ First X-ray Structure of a bromonuium ion: Brown, JACS 1985, 107, 4504
■ Substituent Effects on Bromination Rates
Alkene krel Br3–
Br
CH2=CH2 1 C C 2 eq Br2
CH3CH=CH2 61
-2 eq Br2
n-PrCH=CH2 70
p..
i-PrCH=CH2 57
t-BuCH=CH2 27
(CH3)2C=CH2 5470
cis-CH3CH=CHCH3 2620
trans-CH3CH=CHCH3 1700 Br-3
(CH3)2C=CHCH3 130,000 Br-2
(CH3)2C=C(CH3)2 1,800,000 Br-4
X-ray structure
■ Stereochemical outcome versus structure (Br2 in HOAc @ 25°) Br-1

Alkene % anti addition Alkene % anti addition

H H H Me 2.116 Å 2.194 Å
100% 83%
Me Me Ph H
H Me Me H
100% 63% 1.497 Å
Me H Ph Me
H H Me Me
73% 68%
Ph Me Ph H

9-07-Bromination-1 10/2/03 3:41 PM


D. A. Evans Olefin Bromination-2 Chem 206

■ Calculated Geometries of Substituted Bromonium Ions Overall Reaction Mechanism


Ruasse, Chem Commun. 1990, 898
More recent calculations: Sigalas, Tetrahedron 2003, 59, 4749
Second Order Kinetics
Br Products
Br Br Br A Br–.HOR
2.01 2.70 2.05
1.88
σ-complex
H H H C C Me Me Me Br2
C C C C Br2
1.47 1.51 1.51
H H H Me Me Me
(π-complex)
Note; the C–Br bond lengths in previous X-ray structure are 2.116 Å.
B Br2
■ Bromonium Ions undergo fast exchange with olefins Br Products
Brown, Accts. Chem. Res. 1997, 30, 131 Third Order Kinetics Br3–

Unprecedented until 1991 (Bennet, JACS 1991, 113, 8532) σ-complex


p..

Ad–C C–Ad Ad–C C–Ad Bromination of Cyclohexene Derivatives Pasto, JACS 1970, 92, 7480
Br3– Br Br3–
Br R Br
+
Pyr–Br Br3–
Ad–C C–Ad Ad–C C–Ad Me3C
Me3C R exclusive
H product
X H R = H, Me
X = Br: exchange rate: 2 x 106 M–1 s-1 H Br
Ad–C C–Ad X = I : exchange rate: 8 x 106 M–1 s-1
Diaxial opening of bromonium ions may be viewed as an extension of the
Furst-Plattner Rule for epoxide ring opening (Lecture-3).
OMe
There is an intermediate in the halogen transfer (ab initio calcs): H

H Me3C H
Me3C H
MeOH
R R Pyr–Br+ Br3– Br
H Br 47%
R R R H H
R R R
R R R
R Me3C Br
+ Br + Br + Br Br
R R R R R H H
R Me3C H Me3C
R MeOH H
R R R R R
H 53%
complex TS complex H H OMe

It appears that bromine attack from both olefin faces occurs wilth
near equal probability.
9-08-Bromination-2 10/3/03 8:46 AM
D. A. Evans Olefin Bromination-3 Chem 206
Bromination of Cyclohexene Derivatives Pasto, JACS 1970, 92, 7480
Representative Examples of Diastereoselective Bromination
Diaxial opening of bromonium ions may be viewed as an extension of the
Furst-Plattner Rule for epoxide ring opening. (Lecture-2) Br
Me H Me H
R
OMe Br2
Case A H H
HH R
HOAc
H Me3C H H
Me3C H H Br H
MeOH
Br
Pyr–Br+ Br3– H H H Br 47%
House 2nd Ed, pg 424
Me3C Br
Br H Me H Br Me H
R R
H H Me3C H
MeOH Me3C H Br R H R
Br H
H 53% H H Major Product
H H OMe Minor Product
(7%) H H Br H (70%)
Case B
p.. Pyr–Br+ Br3– OMe
Me How to generate either epoxide from a conformationaly biased olefin
MeOH
Me3C Me3C Me exclusive Me Me
H
product
H RCO3H Epoxidation controlled by steric
H Br H H effects imposed by cis-fused ring
O
Br δ+ Me
δ+
Me How do we construct the other epoxide diastereomer??
Me3C Me3C H
H Me Me H
H H Br
Br2 Me
syn-Unreactive anti-Reactive H H
H 2O
Br Br
Br
OH2
Br OH2
δ+ Me +
Me3C –H not
Me3C Me
H H observed
H Me Me
HOMe H OMe H
syn-Unreactive
H base H Me
From Case A, one assumes that both bromonium ions are formed; however, Br H
H Br
for the syn isomer to react, ring opeing must proceed against the polarization H
due to Methyl substituent. OH OH
O H H minor major H
both bromohydrins afford same product
9-09-Bromination-3 10/3/03 8:47 AM
D. A. Evans Olefin Oxymercuration-1 Chem 206
Oxymercuration Pasto, JACS 1970, 92, 7480
Bromination of Cyclohexene Derivatives Pasto, JACS 1970, 92, 7480
The basic process:
XHg H H Diaxial opening of bromonium ions may be viewed as an extension of the
H H X–Hg–X NaBH4 H
H H Furst-Plattner Rule for epoxide ring opening. (Lecture-2)
C C C C C C
R H ROH H H
R OR R OR OMe
Case A H
Kinetics: Halpern, JACS 1967, 89, 6427 Reduction: Pasto, JACS 199, 91, 719
Overview: B rown, JOC 1981, 46, 3810. H Me3C H
Me3C H
MeOH
Br
Pyr–Br+ Br3– H H H Br 47%
Oxy-Mercuration & bromination follow identical pathways (Pasto) Me3C Br
Br
OH HgOAc
R H H
Hg(OAc)2 Me3C H Me3C
MeOH H
Me3C Me3C R Me3C R
THF, H2O H H H 53%
H H OMe
H H HgOAc H OH
p.. Case B
R=H 41% 48% Pyr–Br+ Br3– OMe
Me
R = Me 100% MeOH
OH Me3C Me3C Me exclusive
Me H
product
Hg(OAc)2
Me3C Me3C Me exclusive H H Br
H
THF, H2O product
H H HgOAc
Br δ+ Me
δ+
Me3C Me H
Hg–X Me Me3C
δ+
H
Me3C δ+ Me Me3C H H Br
H
H syn-Unreactive anti-Reactive
H H Hg–X
syn-Unreactive anti-Reactive
Br
Reduction of the Hg–C bond Br
Me3C δ+ Me –H+ not
NaBH4 Me3C Me
R Hg–X H H observed
R Hg–H R H H
HOMe H OMe
H–CO2– nonstereoselective radical syn-Unreactive
chain process
–CO2
R Hg O From Case A, one assumes that both bromonium ions are formed; however,
Formate is an excellent source of hydride ion for
late transition and heavy main-group metals
for the syn isomer to react, ring opeing must proceed against the polarization
H O
due to Methyl substituent.
9-10-oxymercuration-1 10/3/03 8:21 AM
D. A. Evans Olefin Addition Reactions: Part–3 Chem 206

Bromoniun Ions or β-Bromocarbocations in Olefin Bromination. A Kinetic


Approach to Product Selectivities
http://www.courses.fas.harvard.edu/~chem206/ M-F. Ruasse, Accts. Chem. Res. 1990, 23, 87 (handout)

Investigation of the early Steps in Electrophilic Bromination through the


Chemistry 206 Study of the Reaction of Sterically Encumbered Olefins
R. S. Brown, Accts. Chem. Res. 1997, 30, 131 (handout)

Advanced Organic Chemistry


■ Predict stereochemical outcome
Lecture Number 10
Me
OR OR O
X-206 Synthesis
Me
Olefin Addition Reactions–3 OH Me
(with S. Bender, JACS 1988, 110, 2506)

O OH Me Me
■ Olefin Oxymercuration
■ Halolactonization Hg(OAc)2, CH2Cl2 Me
OR OR O
-78 oC to -20 oC A
■ Simmons-Smith Reaction Me
99% O ❋ Me
16
O 1 OH HgX Me Me
■ Reading Assignment for week 99%, single
HO diastereomer
Me Me
O
A. Carey & Sundberg: Part B; Chapter 4 H Me
OH
"Electrophilic Additions to C–C Multilple Bonds"
Me
BnO O Hg(OAc)2, CH2Cl2
-78 oC to -20 oC
Me
Hoveyda, Evans, & Fu (1993). Substrate-Directable Chemical O
Reactions. Chem. Rev. 93: 1307-70 (Handout) Me
XHg
❋ Me
Me
O O
Me H Me
Me OH
BnO O

Monday, O Me
85%, dr = 93 : 7 Me
D. A. Evans October 3, 2003 Me
Ionomycin Synthesis
(with Dow & Shih, JACS 1990, 112, 5290)

10-00-Cover Page 10/6/03 8:35 AM


D. A. Evans Olefin Oxymercuration-1 Chem 206
Oxymercuration Pasto, JACS 1970, 92, 7480 Bromination of Cyclohexene Derivatives Pasto, JACS 1970, 92, 7480

The basic process: Diaxial opening of bromonium ions may be viewed as an extension of the
XHg H H Furst-Plattner Rule for epoxide ring opening. (Lecture-2)
H H X–Hg–X NaBH4 H
H H
C C C C C C
H ROH H OMe
R H Case A H
R OR R OR
H Me3C H
Kinetics: Halpern, JACS 1967, 89, 6427 Reduction: Pasto, JACS 199, 91, 719 Me3C H
Overview: B rown, JOC 1981, 46, 3810. MeOH
Br
Pyr–Br+ Br3– H H H Br 47%
Me3C Br
Oxy-Mercuration & bromination follow identical pathways (Pasto) Br
H H Me3C H
OH HgOAc MeOH Me3C H
R
Hg(OAc)2 H 53%
Me3C Me3C R Me3C R H H OMe
THF, H2O H H
H H Case B
H HgOAc OH
p.. Me
R=H 41% 48% Me3C Pyr–Br+ Br3–
R = Me 100%
OH H
Me
Me
Hg(OAc)2 Br δ+ δ+
Me3C Me3C Me exclusive
H Me H
THF, H2O product Me3C Me3C
H H HgOAc H Br
H H
syn-Unreactive anti-Reactive
Hg–X Me
δ+
Me3C δ+ Me Me3C H MeOH MeOH
not exclusive
H observed Br OMe
H H Hg–X product
syn-Unreactive anti-Reactive Me3C Me Me3C Me
H H
Reduction of the Hg–C bond
H OMe H Br
NaBH4
R Hg–X R Hg–H R H Br δ+ Me

H–CO2– nonstereoselective radical Me3C Me Me3C


+ A
chain process
–CO2 H
H H
R Hg O Formate is an excellent source of hydride ion for
late transition and heavy main-group metals
From Case A, one assumes that both bromonium ions are formed; however,
H O
for the syn isomer to react, ring opeing must proceed against the polarization
due to Methyl substituent.
10-01-oxymercuration-1 10/5/03 11:20 AM
D. A. Evans Oxymercuration Examples Chem 206

Diastereoselective ring closures via oxymercuration ■ Acyclic allylic alcohols:

OH OH OH
H H Hg(OAc)2 NaBH4
OBn OBn Me
OH Hg(OTFA)2 O R R'OH R HgOAc R
BnO BnO
BnO NaBH4 BnO H α:β = 96 : 4 OR'
OR'
H H
H OC6H11 H OC6H11 R R'OH Ratio yield
-Et HOH 76 : 24 65%
Mukaiyama, Chem. Lett. 1981, 683 Giese, Tet. Lett. 1985, 26, 1197 -Et MeOH 93 : 07 72%
H -Ph HOH 88 : 12 66%
H OR H OR
Hg(OTFA)2 H -tBu HOH 98 : 02 70%
R' OH X
AcNH R' O CO2Me "one isomer"
CO2Me Ph3 SiH AcNH
Hg
H OBn HO H OH
H OBn OH
Hg(OAc)2 H
C C RL HgOAc
RL H H
Sinay, Tet. Lett. 1984, 25, 3071 OH
RL
H HOH
H
O X
OBn O
HO Hg(OAc)2 OBn Hg
OR HO OH HO H OH OH
Me H NaBH4 OR
HO Me H Hg(OAc)2 H
n-Bu Me C C RL Me
H Me
Isobe, Tet. Lett. 1985, 26, 5199 HgOAc
RL HOH
syn:anti = 80 :20
Chamberlin, Tetrahedron 1984, 40, 2297
■ Kinetic vs Thermodynamic control:

H H CH2 –HgX O-acetate participation will turn over the stereochemical course of the rxn
Me Hg(OAc)2 Me Me
XHg–CH2 H OAc
NaBH4
OAc OH
H N N N Hg(OAc)2
BnO2 C H BnO2 C H Me Me
BnO2 C H Et HOH Et Et erythro:threo = 77 : 23
NaBH4
OH OAc
Hg(OAc)2: short rxn times : 40 : 60 erythro
Harding, JOC 1984, 49, 2838
Hg(OTFA)2: longer rxn times : 2 : 98 Me Me
Hg(OAc)2
COOMe BnOH COOMe
BnO O NaBH4
BnO O OBn
With more electrophilic Hg(II) salt, more polar solvents, and
longer rxn times, the rxn may be rendered reversible. Seebach, JACS 1983, 105, 7407 diastereoselection = 83 : 17 (79%)

10-02-oxymercuration-2 10/6/03 8:54 AM


D. A. Evans Oxymercuration Examples Chem 206
Proposed Mechanism
Oxymercuration via Hemiketals & Hemiacetals
J. L. Leighton et. al, Org. Lett. 2000, 2, 3197-3199 ■ Lewis acid catalyzes formation of hemiketal

■ General Reaction: diastereoselection >10:1 H Me H Me


H 5% Yb(OYt)3 H
R OH Me R O Me
R' O
O
OH O HgClOAc O O Yb(X2) Yb(X2)
+
5% Yb(OYt)3 HgCl
R H R' R ■ The Oxymercuration Step (Kinetic Phase)

Me Me H Me
H Me
OH O HgClOAc O O H Me H H
+ H X R O
HgClOAc R O Me Me
5% Yb(OYt)3 HgCl R O Me Hg O
R Me Me R O
O
Yb(X2) HgCl
Yb(X2)
■ Mechanistic Observations: rate-determining
p.. low diastereoselectivity
step
Me Me
H Me X H Me
OH O HgClOAc O O Hg
+ HgClOAc
acetone, 2h rt HgCl R O R O Me
R Me Me Me
R ❉ O O
~1:1-mixture of diastereomers H H Yb(X2)
Yb(X2)
Product formed in low yield.
much recovered starting material
Leighton presumes that mercurium ion formation is rate-determining under
Lewis acid addends were surveyed. the logic for this step was two-fold: kinetic conditions.
At higher temperatures and longer reaction times the products are shown to
(A) Lewis acid would promote the formation of the putative hemiketal imtermediate.
(B) Lewis acid would promote reversability of the oxymercuration process
interconvert.

YbX3
Me Me ClHg H Me HOAc, 5% Yb(OYt)3
HgClOAc Cl
OH O 5% Yb(OYt)3 O O R O Hg H Me
+ Me
acetone, 2 min HgCl O R
Me3C Me Me H O Me
0 °C Me3C ❉
O
~1:1-mixture of diastereomers H
H
H MM-2 H HOAc, 5% Yb(OYt)3 YbX3
Me Me
H Me Cl
Me O Me O Me Me Hg H
Me Me Me
O O O O 93% yield
Me H R O
HgCl Me
Erel = 0 Erel = +5.2 kcal/mol Me3C O
H –
OAc
H
10-03-oxymercuration-3 10/5/03 11:22 AM
D. A. Evans Oxymercuration Examples: X-206 & Lonomycin Syntheses Chem 206

X-206 Synthesis (with S. Bender, JACS 1988, 110, 2506) Ionomycin Synthesis (with Dow & Shih, JACS 1990, 112, 5290)

Me Me Me Me
Me Me
OH
A B C E
H O O
D H Me H Me
Me
O O O O O Me OH
H OH OH Me OH Me F OH Ionomycin Calcium
Me OH O Me
O OH Me Complex
aldol OH OH Ca O
Et

+ Me O O O
O O
D E
H
H O O
Me OMe Me F Me Me Me Me Me
C1-C16 Subunit Me Me OR O Me
OH
Me C17-C37 Subunit Et Me Me
OR OR O O O HO O
A H Me H Me Me H Me
Me Me OH Me OH
p.. O 7 Me OH OH
16
OH Me Me Me Me
O 1
OH OH
R R
Assemblage strategy for Ring A: Me
OR OR O
Me
A H R1 Hg(OAc)2, CH2Cl2
HO
O Me Me
Me H Me
OH -78 oC to -20 oC O
16 Me O H Me
OH Me Me OH
Predicted stereochemical outcome: O 1
H Me
H
H Me
H HO BnO O
H Hg–X
Me 7 Me
OR H Me O
7 Me R2 Me
Me HgX2 O HgX+ RDS Me
OH 9
RL RO2C Me H
9
OR
RDS H 85%, dr = 93 : 7
CO2R H RL
H H R1
O Me
X H
O XHg Me
Hg Me O O
H Hg(OAc)2 H OR H H Me H Me
RO H Me Me OH
Me CH2Cl2 H 7 Hg H BnO O
C C 9 HO X
H Me RL
H 99% Me
O O
RL RO2C Me HgX Me Me
R2 Me
H

H
10-04-oxymerc/lono 10/5/03 5:22 PM
D. A. Evans Related Olefin Addition Rxns: Halogen Electrophiles Chem 206

Other electrophilic olefin addition reactions afford the same Iodine-induced lactonization is also highly stereoselective
stereochemical outcome
X ■ Chamberlin (JACS 1983, 105, 5819)
Hg OH OH
OH HO H A
Hg(OAc)2 I B I
H O OH HO H HO H
n-Bu Me C C n-Bu Me
H I2, HOH/THF H H
RDS Me HgOAc C C C C
RO Me Me
n-Bu HCO3– Me Me Me
ratio = 80 :20 Me R=H CH2 CH2
I OH OAc -O2C -O2C
OH HO H
I2, HOAc H HOAc
C C n-Bu Me As we have seen before, gauche B
n-Bu Me H is more destabilizing than gauche A
RDS Me I HO I HO I
n-Bu Ratio = 98 : 2 (78%)
Me Me
Me O Me
O
I OH OAc Ratio
OH Me HO H t-BuOOH
p.. I2, HOAc HOAc R = OMe O 96 : 4 (85%) O
Me VO(acac)2
n-Bu C C n-Bu Me
RDS H H K2CO3
I MeOH
n-Bu O OH O OH
Ratio = 94 : 6 (85%)
Chamberlin, Tetrahedron 1984, 40, 2297
MeO Me MeO Me
O O
This is an exceptional approach to the creation of either syn or anti Me Me
1,3-dioxygen relationships Lactonization Ratio = 96 : 4
■ Other cases:
Epoxidation Ratio = 3 : 97
Evans, Kaldor, Jones, J. Am. Chem. Soc. 1990, 112, 7001. HO I
O OH
I2, HOH/THF Me Ratio
HO Me HCO3– >95 : 5 (49%)
TIPSO OH TIPSO OH OH O H
I2, THF, 4 °C
Me Me
0.25 M KH2PO4, O
HO I
Me Me I O OH
Me Me I2, HOH/THF R = H: 77 : 23 (74%)
n-Bu3SnH, toluene, 25 °C HO HCO3– Me
TsOH, (CH3)2C(OCH3)2, 25 °C TIPSO O O O R R = Me: 42 : 58 (81%)
Me Me R
O

This methodology superior to oxymercuration Me HO I


67% overall O OH
alternative which was evaluated first I2, HOH/THF R = H: 87 : 13 (41%)
HO HCO3– Me
diastereoselection 96 : 4 O R R = Me: 90 : 10 (94%)
Me R
10-05-iodination 10/5/03 5:40 PM O
D. A. Evans Related Olefin Addition Rxns Chem 206

Halogen-induced heterocyclization in the synthesis of monensin Hypothesis-B: Stereocontrol through Reversal of Bromonium Ion Intermediate

HO Me Me Me
RL H Me
B C D Me Me
Me A E Me OH Br C D
O O O H Me Ar
MeO O Me H Et H O O O
CH2OH H Et H H
O HO Br
RL
Me
bromonium ion-olefin exchange
HO Me Kishi, JACS 1979, 101, 259, 260, 262
Still, JACS 1980, 103, 2117-2121 Me
■ The Kishi Ring D Construction: Me RL Me Me
Me C D
Me Ar
RL O O
Me Me NBS H Et H H
Ar C D only one diastereomer H OH Br Br
MeCN H
O OH Ring D disfavored
H Et 57%

p.. ■ The Still Ring E Construction:


Hypothesis-A: Stereocontrol through A(1,3) Strain??
Me Me
Me
Me Me R D
C Me Me Me Me E Me
Ar D OH
O H HO
O
Me H Et H H D R D O
Br R E Me E Me
Me Me O O
C KO2_ H O H O
Ar D CH2OH
DMSO HO O
O O I
H Et H H 47%
OH
H Me
H
Ag2CO3 50%
Me
Me H
RL O
RL H Stereocontrol through A(1,3) Strain??
OH Br Me
H OH H Me
Me O
Me Me Me Me
KI3
RL RL
D HCO3– D
E Me
E Me
87%
O O HO O O I O
Me RL Me RL Me Me O Me Me O
Me Me
RL RL Incorrect Stereochemistry
H OH H OH Br El(+)-induced Cardillo, Tetrahedron 1990, 46, 3321-3408
H
heterocyclization Bartlett, Asymmetric Synthesis 1984, 3, Chap 6, 411-454

10-06-Monensin examps 10/5/03 5:34 PM


D. A. Evans Applications in Synthesis Chem 206

■ A complete turnover in olefin diastereofacial selectivity is HO I OH OH


observed when adding internal and external nucleophiles Hehre's Analysis H
I2 Me Bu Me
O I
HO I
H O Addition product
H Me
O OH
I2 Me
H
HO Me OH O
O O Nu
cis : trans 95 : 5 R H Me R H Me RDS R H Me
O-
OH2 H H H
H
H H
OH OH OH I+
Bu Me HO HO HO
I2 Favored ground-
Bu Me state conformer I2
Bu Me H
H 2O Disfavored π–complex
I Favored iodonium ion
HO H
ratio 99 : 1
I2
General Observation: I2
p..
For electrophiles that react via onium intermediates (I2, Br2, Hg(OAc)2, PhSeCl), I +
OH Me
the major diastereomer from electrophile-induced cyclization is opposite to that H H Me Me
observed in the analogous intermolecular electrophilic addition. RDS H H H
H H H
OH OH H R
For a review of elctrophilic induced olefin cyclization reactions see: R R
More reactive Nu
G. Cardillo & M. Orena, Tetrahedron 1990, 46, 3321.
ground-state conformer
Favored π–complex Disfavored iodonium ion

Chamberlin & Hehre's Rationalization


■ "Facial preferences in electrophilic addition reactions are not invariant with respect
to the location of the transition state along the reaction coordinate."
■ Change in diastereoselectivity is a consequence of a change in the rate-limiting step HO I
H OH OH
● Addition reactions: Formation of an onium ion intermediate Me
(subsequently trapped by a Nu from the medium) Bu Me
O
● Cyclization reactions: Intramolecular attack on a π–complex (not an onium ion) O I

■ Analysis of the stereoselectivity of electrophilic addition to chiral olefins: Cyclization product

1. Relative abundances of conformational minima Houk: Argument for the "inside alkoxy effect" in π–complex formation
2. Relative reactivities of the available forms
3. Stereoselectivies of the individual conformers ■ π–complex cyclizes if R contains a Nu and its formation is rate determining
■ Onium ion formation is rate determing in the addition reactions
■ "The presence or absence of an internal nucleophile acts to determine the
Chamberlin & Hehre, J. Am. Chem. Soc. 1987, 109, 672-677. stereochemical outcome of the reaction by modifying the nature (timing) of
transition state.

10-07-Monensin examps-2 10/5/03 5:37 PM


D. A. Evans The Simmons-Smith Reaction Chem 206
For a recent general review of the Simmons-Smith reaction see:
Charette & Beauchemin, Organic Reactions, 58, 1-415 (2001) R CH3 CH2 I2 R CH3 R CH3

OH Zn-Cu OH OH
CH2 I2 , Zn-Cu
OH OH >99:1
79 % R Ratio
M. Pereyre and Co-workers
S. Winstein, JACS 1959, 81, 6523; 1961, 83, 3235; 1969, 91, 6892 J. Chem. Res. (S) 1979, 179 CH3 57 : 43
Et 64 : 36
A large rate acceleration relative to simple olefins was observed. tBu 67 : 33

■ The classical mechanism


OR OR CH2 I2 + Zn ICH2 ZnI
CH2 I2 R = OMe: >99:1 Dauben, JACS 1963, 85, 468

CH2 R = OAc: 4:1 Sawada, JOC 1968, 33, 1767 ICH2 ZnI R ZnI R
Zn-Cu R
R CH2 + ZnI2
OH R I R
OH
CH2 I2
9:1 ■ Enantioselective Simmons-Smith Variants: Kobayashi, Tet. Let. 1992, 33, 2575
Zn-Cu CH2
3
Et2 Zn
OH OH CH2 OH CH2 OH
PhCH2 CH2 PhCH2 CH2 80% ee (82% yield)
CH2 I2
1
CH2 I2
Zn-Cu
CH2 >99 : 1 NHSO2 Ar These results suggest that the transition state
might be binuclear.
10 mol%
epoxidation also gives anti adduct O–C1–C2–C3 dihedral = 165 ° NHSO2 Ar Construct a reasonable transition structure which
S. Winstein, JACS, 1969, 91, 6892 accomdates the data

■ Low-valent Samarium Variants: Molander,JOC 1987, 52, 3942


Absolute control of stereochemistry is possible through chiral ketal auxiliaries
R''
BnOH2 C CH2 OBn Me3 C CH3 > 200 : 1 (99%)
BnOH2 C CH2 OBn Me3 C
HO Sm or Sm/Hg OH
O diastereoselection 20:1 R'
O CH2 I2 O O CH2 I2 R' R'
Me Me
Zn-Cu R'' R''
CH2 R''
HO
OH OH
Yamamoto, JACS, 1985, 107, 8254
Mash, JACS, 1985, 107, 8256 R' R" Ratio
Yamamoto, Tetrahedron, 1986, 42, 6458 Isolated alkenes and homoallylic alcohols
are inert to these reaction conditions. Ph nBu 1 : 1.4
Ph iPr > 200 : 1
Ph tBu > 200 : 1
G. A. Molander and J. B. Etter
tBu CH3 1 : 5.1
J. Org. Chem. 1987, 52, 3942
tBu iPr > 200 : 1

10-08-S-Smith rxn 10/6/03 9:23 AM


A. S. Kim, D. A. Evans Redox- Thallium.1 Chem 206

Radical Lead Tetraacetate Mechanism Vinyl and Aryl C-H Oxidations

Thallium
-
ROH + Pb(OAc)4 ROPb(OAc)3 + OAc CHO (TFA)2Tl CHO I CHO
ROPb(OAc)3 RO• + •Pb(OAc)3 Tl(TFA)3, TFA CuI, I2
RO• + R'H ROH + R'• N N DMF N
94%
H H H
R'• + Pb(OAc)4 R'OAc + •Pb(OAc)3
Somei, Heterocycles 1984, 22, 797.

H OAc OAc
Me RCH2 1) Tl(TFA)3, TFA
RCH2 OH LTA RCH2 2) CuSO4•5H2O
Me Somei, Heterocycles, 1988, 27,
Me O R O O 2363.
N 42% N
19% 33% 20% Ac Ac
(racemic) OH
Hauser, Helv. Chim. Acta 1964, 47, 1883.
Me Phenolic Oxidations: For a review of oxidative aryl couplings, see: Dinsmore, C.
O
O Me Evening Seminar, February 1993.
Me Me
LTA Me O Me O

45-72% CF3COO
HO O TTFA
H
75%
Immer, Helv. Chim. Acta 1962, 45, 753.
HO O
OAc Coombs, Chem. Ind., 1972, 169.
LTA O
O
HO OAc O
Br O
28% 42% Haynes, JOC, 1866,31, 3067.
O Ph

O O O
Me
Me Me OAc
N O TTFA N
O O O
56%
Me LTA, BF3•Et2O Me Ph O
HO
PhH, RT
86% Br O
Cocker, JCS, 1965, 6.
AcO AcO Taylor, JACS, 1981, 103, 6856.
H H

10-09-REDOX-Thallium.1 10/6/03 8:36 AM


A. S. Kim, D. A. Evans Redox- Thallium.2 Chem 206

Oxythallation of Ketones (Enols) with Ring Contraction TTN = Tl(NO3)3 Oxythallation of Double and Triple Bonds

TTN CO2Me Me
O MeOH Tl(ClO4)3
84% COMe
McKillop and Taylor, AcOH
Me JACS, 1973, 95, 3340.
O
McKillop and Taylor, 88-92%
ONO2 JOC, 1978, 43, 3773.
TTN OMe

MeCN 97%
OMe
TTN 85%
McKillop and Taylor,
O MeOH JACS, 1973, 95, 3635.
1) TTN, AcOH
O OH OMe
2) NaHCO3
84% 86%
McKillop and Taylor, OMe
JOC, 1972, 37, 3381.

1) TTN, AcOH
COOH O
2) reflux, 30min.
84%
O 1) TTN/HClO4 O
O
O
TTN 2) NaH, n-Pentyl
AcOH COOH Salaun, Tet., 1974, 30, 1423. n-Pentyl
(MeO)2P(O)
95% O
O 70%
Corey, TL, 1971, 4753.
O CO2Me
TTN
Jones, JOC, 1977, 42, 2176.
MeOH O
85% TTN
Jones, JACS, 1976, 98, 8476.
MeOH
Me Me Me R 68%
Me Me
Me
Me Me Me Me Me Me
Tl(OAc)3 Me
TTFA
83% HO2C Inoue, Bull. Chem. Soc.
AcOH Me Jpn. 197, 51, 2439.
O
H H Me
76% Me
Romeo, Tet., 1972, 28, 5337. O

10-10-REDOX-Thallium.2 10/6/03 8:36 AM


A. S. Kim, D. A. Evans Redox- Thallium.3 Chem 206

Oxidative Rearrangements of Styrenes Oxidative Rearrangements of Chalcones


TTN = Tl(NO3)3

MeO OMe
O
H
1) TTN, MeOH Me
Me TTN
2) H3O+ O
MeOH O
MeO 70% MeO
85% McKillop and Taylor, JACS, 1973, 95, 3635.
Antus, Liebigs Ann. Chem. 1980, 1283.
Me
OH
Me O O
1) TTN, MeOH
2) H3O+ O TTN
81% McKillop and Taylor, JACS, 1973, 95, 3635.
MeOH
OH OH 74% O

O Eade,Aust. J. Chem. 1978, 31, 2699.

OM
1) TTN, MeOH O OBn O e
1) TTN,
2) H3O+ MeOH
95% McKillop and Taylor, JACS, 1973, 95, 3635. 2) H3O+ OBn
MOMO OMOM OMe HO O
75%
O van Heerden JCSPT1 1980, 2463.
Me
1) TTN, MeOH
2) H3O+ O OH O O
95%
McKillop and Taylor, TL, 1977, 1827. 2TTN O
MeOH
O O OMe
OH O 65% O
O
Antus, Liebigs Ann. Chem. 1980,1271.
1) TTN, MeOH
2) H3O+ MeO
MeO
O OH OO
100%
McKillop and Taylor, TL, 1977, 1827.
2TTN
MeOH
O O
OH HO 53%
Antus, Liebigs Ann. Chem. 1980,1271.

10-11-REDOX-Thallium.3 10/6/03 8:36 AM


A. S. Kim, D. A. Evans Redox- Thallium.4 Chem 206

Oxidation of Nitrogen Compounds


TTN = Tl(NO3)3

2TTN
N R CO2Me + N2
N O MeOH
Taylor and McKillop, ACIEE 1972, 11, 48.
78-95%
H

R HN COR R
TTN
N O
R' MeOH R'
McKillop and Taylor, JACS 1971, 93, 4918.
R and R' alkyl or aryl, yields 82-95%
R OH R
TTN
N O
R' MeOH R'
R and R' alkyl or aryl, yields 72-96% McKillop and Taylor, JACS 1971, 93, 4918.

Oxidation of Sulfur Compounds

SEt OMe
MeO NO2 MeO NO2
TTN
MeOH
MeO 98% MeO Fujita, JCSCC, 1976, 202.

O O
OMe
SEt OMe
TTN
MeOH Fujita, TL, 1978, 4115.
O Ph 91% O Ph

Me S S TTN Me S S
MeOH
S 97% Smith, Synth. Commun. 1979, 9, 301.
O
S

10-12-REDOX-Thallium.4 10/6/03 8:37 AM


D. A. Evans, T. B. Dunn Pericyclic Reactions: Part–1 Chem 206

■ Other Reading Material:


http://www.courses.fas.harvard.edu/~chem206/
■ Woodward-Hoffmann Theory
R. B. Woodward and R. Hoffmann, The Conservation of Orbital
Symmetry, Verlag Chemie, Weinheim, 1970.
Chemistry 206
■ Frontier Molecular Orbital Theory
I. Fleming, Frontier Orbitals and Organic Chemical Reactions,
Advanced Organic Chemistry John-Wiley and Sons, New York, 1976.

■ Dewar-Zimmerman Theory
T. H. Lowry and K. S. Richardson, Mechanism and Theory in
Lecture Number 11 Organic Chemistry, 3rd Ed., Harper & Row, New York, 1987.

■ General Reference
R. E. Lehr and A. P. Marchand, Orbital Symmetry: A Problem
Pericyclic Reactions–1 Solving Approach, Academic Press, New York, 1972.

■ Introduction to Pericyclic Reactions ■ Problems of the Day:


■ Electrocyclic Reactions
Predict the stereochemical outcome of this reaction.
■ Sigmatropic Reactions
O Ph
■ Cycloaddition Reactions Ph O O O

■ Reading Assignment for week: O
heat ❉
Ph
Carey & Sundberg: Part A; Chapter 11 O
Ph
Concerted Pericyclic Reactions Huisgen, TL, 1964, 3381.

Fleming: Chapter 4
Thermal Pericyclic Reactions
Suggest a mechanism for the following reaction.
CO2Me
CO2Me H
MeO2 C
heat
Wednesday,
Travis Dunn October 7, 2003 CO2Me
H
Bloomfield, TL, 1969, 3719.
11-00-Cover Page 10/6/03 9:26 AM
D. A. Evans, B. Breit, T. B. Dunn Pericyclic Reactions: Introduction Chem 206

Pericyclic Reactions - Introduction/Definitions The Theories:


Three theories are commonly used to explain and predict pericyclic
reactions. We will only concern ourselves with two of these theories.
A pericyclic reaction is characterized as a change in bonding relationships that
takes place as a continuous, concerted reorganization of electrons. 1) Fukui: Frontier Molecular Orbital Interactions
The term "concerted" specifies that there is one single transition state and
therefore no intermediates are involved in the process. To maintain continuous ■ Much easier to use than the original orbital symmetry arguments
electron flow, pericyclic reactions occur through cyclic transition states. ■ HOMO/LUMO interactions
More precisely: The cyclic transition state must correspond to an arrangement of
the participating orbitals which has to maintain a bonding interaction between
the reaction components throughout the course of the reaction. 2) Dewar-Zimmerman: Aromatic Transition States
■ The easiest to apply for all reaction types, but it is not as easy to
Some factors to consider in our analysis: understand why it it valid

The number of electrons involved has a profound influence on reactivity: ■ Aromatic or antiaromatic transition states

3) Woodward-Hoffmann: Conservation of Orbital Symmetry


heat heat ■ First theory to explain and predict the outcome of many reactions
■ Correlation diagrams
rarely often
observed observed On the three methods:
"There are several ways of applying the orbital-symmetry principle to cycloaddition
4 electrons 6 electrons reactions, three of which are used more frequently than others. Of these three, we will
discuss two: the frontier-orbital method and the Möbius-Hückel method. The third, called
the correlation diagram method, is less convenient to apply than the other two."
Pericyclic reactions are stereospecific: Jerry March in "Advanced Organic Chemistry"

A
A A A A The Five Major Categories of Pericyclic Reactions
A
A heat heat A (1) ELECTROCYCLIC RING CLOSURE/RING OPENING:
An electrocyclic ring closure is the creation of a new sigma bond at the expense
of the terminal p orbitals of a conjugated pi system. There is a corresponding
reorganization of the conjugated pi system. We usually classify the reaction
Reactions behave differently depending on the conditions used according to the number of electrons involved.
(i.e. thermal versus photochemical conditions): Examples:
A 4 e- electrocyclic reaction A 6 e- electrocyclic reaction
A
A heat A
hν ∆ or hν ∆ or hν
A
A A
Cyclobutene Butadiene 1,3,5-Hexatriene 1,3-Cyclohexadiene
11-01-Peri 10/11/00 7:53 AM
D. A. Evans, B. Breit, T. B. Dunn Pericyclic Reactions: Major Classes Chem 206

(2) CYCLOADDITION REACTIONS/CYCLOREVERSION REACTIONS: (4) SIGMATROPIC REARRANGEMENTS:


A cycloaddition reaction is the union of two smaller, independent pi systems. A sigmatropic rearrangement is the migration of a sigma bond from one position
Sigma bonds are created at the expense of pi bonds. A cycloaddition can occur in a conjugated system to another position in the system, accompanied by
in an intramolecular sense, but it must be between two independent pi systems. reorganization of the connecting pi bonds. The number of pi and sigma bonds
Cycloaddition reactions are referred to as [m + n] additions when a system of m remains constant. The rearrangement is an [m,n] shift when the sigma bond
conjugated atoms combines with a system of n conjugated atoms. A migrates across m atoms of one system and n atoms of the second system.
cycloreversion is simply the reverse of a cycloaddition. Examples:
2
Examples: 2 [1,3]-shift 2
3 1 R1 [1,5]-shift
3 R2
3 1 1 4 R2
5
O [2+2] A 2+2 cycloaddition. R R
+ O
The Paterno-Büchi R1 H
hν reaction.
2 2
1 3 1 3
X [3,3]-shift X X=CR2, Cope rearrangement
[4+2] 3' 3'
X=O, Claisen rearrangement
A 4+2 cycloaddition. R 1' R 1'
+ The Diels-Alder reaction.

2' 2'

(5) GROUP TRANSFER REACTIONS:


In a group transfer reaction one or more groups get transferred to a second
(3) CHELETROPIC REACTIONS: reaction partner.
Cheletropic reactions are a special group of cycloaddition/cycloreversion
reactions. Two bonds are formed or broken at a single atom. The Examples:
nomenclature for cheletropic reactions is the same as for cycloadditions.
Hydrogen R H R H
Examples:
Transfer: + +
O [4+1] O
+ S S H H
O O
R
H H R
N
+ N2 +
[4+1] N
+ C O O H H R'
R'
Ene Reaction:

R
+ CR2 [2+1]

R +
H
H
11-02-Peri 10/11/00 7:55 AM
D. A. Evans, B. Breit, T. B. Dunn Electrocyclic Reactions Chem 206

ELECTROCYCLIC RING CLOSURE/RING OPENING: Butadiene to cyclobutene: A 4-electron (4q) system

The Stereochemical issues:


Ring closure can occur in two distinct ways. This has consequences with
Me H
regard to:
conrotation
Me H Me
■ The orbital lobes that interact H heat
■ The disposition of substituents on the termini H Me

Conrotatory Closure: The termini rotate in the same direction conrotation


Me Me Me Me
heat

B B Hextriene to cyclohexadiene: A 6-electron (4q+2) system


B B
A B A B
A A A A

disrotation
H H

H H
Me Me
Disrotatory Closure: The termini rotate in opposite directions
Me Me

A B
A B
A B A B disrotation
B A B A

Me Me Me Me

It was also noted that changing the "reagent" from heat to light reversed
this reactivity pattern. Under photochemical conditions 4 electron
Empirical Observations: systems undergo disrotatory motion, while 6 electron systems undergo
conrotatory motion.
It was noted that butadienes undergo conrotatory closure under thermal
conditions, while hexatrienes undergo disrotatory closure under thermal Me
conditions. The microscopic reverse reactions also occur with the same Me
rotational sense (i.e. cyclobutenes open in a conrotatory sense when disrotation controtation
heated, and cyclohexadienes open in a disrotatory sense when heated.) hν hν
Me
Me Me Me Me
Me
11-03-Peri 10/11/00 7:56 AM
D. A. Evans, T. B. Dunn Conjugated pi systems Chem 206

antibonding 6 p-orbitals

5 p-orbitals
4 p-orbitals

3 p-orbitals
Ψ4
2 p-orbitals

Ψ3
π∗
Ψ3

C C
nonbonding Ψ2 nonbonding

Ψ2
π

Ψ1

Ψ1

There are no nodal planes in the most stable bonding MO. With each higher MO, one additional nodal plane is added.
bonding The more nodes, the higher the orbital energy.

11-04-Peri 10/11/00 7:56 AM


D. A. Evans, B. Breit, T. B. Dunn Electrocyclic Reactions: FMO Analysis Chem 206

■ FMO Treatment of Electrocyclic reactions. Photochemical Activation:


When light is used to initiate an electrocyclic reaction, an electron is
■ Examine the interactions that occur in the HOMO as the reaction
excited from Ψ2 to Ψ3. Treating Ψ3 as the HOMO now shows that
proceeds.
disrotatory closure is allowed and conrotatory closure is forbidden.
■ If the overlap is constructive (i.e. of the same phase) then the
reaction is "allowed."
■ If the overlap is destructive (i.e. of different phases) then the Ψ4 Ψ4
reaction is "forbidden." Ψ3 hν Ψ3
Ψ2 Me H Me Photon Me
H Me Ψ2
Thermal Activation: H absorption H
Ψ1 Ψ1
Conrotatory Closure: (Allowed and observed)
Ψ2 (HOMO) Ψ3 (HOMO)
Constructive
overlap
Disrotatory Closure: (Allowed and observed)
Me H
Me H
Me H Me H
H Me H H H
H H Me Me
H H Me
Ψ2 (diene HOMO) Me Me Me Me

Ψ3 (new HOMO) Constructive


Disrotatory Closure: (Forbidden and not observed) overlap

H H H H Conrotatory Closure: (Forbidden and not observed)


Me H Me
H Me Me Me Me H
Me
Me H
Ψ2 (diene HOMO)
Me H Me
Destructive H Me
overlap H H Me
Ψ3 (new HOMO) Destructive
overlap

A similar analysis for the hexatriene system proves that under We have so far proven which ring closures are allowed and which are
forbidden. Do we now have to go back and examine all the ring
thermal conditions, disrotation is allowed and conrotation is openings?
forbidden. NO!
The principle of microscopic reversiblity says that if the reaction is
allowed in one direction, it must be allowed in the other direction.
11-05-electrocyclic/FMO 10/11/00 7:56 AM
D. A. Evans, B. Breit, T. B. Dunn Electrocyclic Reactions: Dewar-Zimmerman Chem 206

The Dewar-Zimmerman analysis is based on identifying transition states Connect


as aromatic or antiaromatic. We will not go into the theory behind why Orbitals
this treatment works, but it will give the same predictions as FMO or
Orbital Symmetry treatments, and is fundamentally equivalent to them.

Using the Dewar-Zimmerman model: Disrotatory Conrotatory


Closure Closure
■ Choose a basis set of 2p atomic orbitals for all atoms involved (1s for
hydrogen atoms).

■ Assign phases to the orbitals. Any phases will suffice. It is not


important to identify this basis set with any molecular orbital.

■ Connect the orbitals that interact in the starting material, before the
reaction begins. Zero Phase Inversions One Phase Inversion
∴Hückel Topology ∴Möbius Topology
■ Allow the reaction to proceed according to the geometry 4 electrons in system 4 electrons in system
postulated. Connect those lobes that begin to interact that were not ∴ Antiaromatic and ∴ Aromatic and
interacting in the starting materials. Forbidden Allowed
Note that I can change the phase of an abitrary orbital and the analysis
■ Count the number of phase inversions that occur as the electrons is still valid!
flow around the circuit. Note that a phase inversion within an orbital is
Connect
not counted. Orbitals
■ Based on the phase inversions, identify the topology of the system.

Odd number of phase inversions: Möbius topology Conrotatory


Even number of phase inversions: Hückel topology Disrotatory
Closure Closure

■ Assign the transition state as aromatic or antiaromatic, based on the


number of electrons present.
System Aromatic Antiaromatic
Hückel 4q + 2 4q
Möbius 4q 4q + 2
Two Phase Inversions Three Phase Inversions
■ If the transition state is aromatic, then the reaction will be allowed ∴Hückel Topology ∴Möbius Topology
thermally. If the transition state is antiaromatic, then the reaction will 4 electrons in system 4 electrons in system
be allowed photochemically. ∴ Antiaromatic and ∴ Aromatic and
Forbidden Allowed
11-06-electrocyclic/DZ 10/11/00 7:57 AM
D. A. Evans, B. Breit, T. B. Dunn [1,3]-Sigmatropic Rearrangements: FMO Analysis Chem 206

The Stereochemical issues:


■ [1,3] Sigmatropic Rearrangements (H migration)
The migrating group can migrate across the conjugated pi system in
one of two ways. If the group migrates on the same side of the system, ■ Construct TS by considering an allyl anion and the proton (or radical
it is said to migrate suprafacially with respect to that system. If the pair):
group migrates from one side of the pi system to the other, it is said to H H H
migrate antarafacially with respect to that system.
X Y •• X Y
X Y
Suprafacial migration: The group moves across the same face.

B A
A B A B
A B H H H
A B A B
X Y X Y X Y

Antarafacial migration: The group moves from one face to the other.
Proton 1S (LUMO)

bonding H antibonding bonding


B
A B B
A B A A B A B
A X Y X H
Y
Ψ2 (allyl anion HOMO)
bonding
■ Sigmatropic Rearrangements: FMO Analysis Suprafacial Geometry Antarafacial Geometry
■ Imagine the two pieces fragmenting into a cation/anion pair, (or a ■ The analysis works if you consider the other ionic reaction, or
pair of radicals) and examine the HOMO/LUMO interaction. consider a radical reaction. In each case it is the same pair of orbitals
interacting.
■ If the overlap is constructive at both termini then the reaction is
allowed. If the overlap is destructive at either terminus then the
reaction is forbidden. ■ The suprafacial migration is forbidden and the bridging distance too
great for the antarafacial migration. Hence, [1,3] hydrogen migrations
■ If the migrating atom is carbon, then we can also entertain the are not observed under thermal conditions.
possiblity of the alkyl group migrating with inversion of configuration
(antarafacial on the single atom). ■ Under photochemical conditions, the [1,3] rearrangement is allowed
suprafacially. How would you predict this using FMO?
■ If the migrating atom is hydrogen, then it cannot migrate with
inversion.
11-07-[1,3]sig/FMO 10/11/00 7:58 AM
D. A. Evans, B. Breit, T. B. Dunn [1,3]-Sigmatropic Rearrangements Chem 206

■ [1,3] Sigmatropic Rearrangements (C migration) ■ Sigmatropic Rearrangements: Dewar-Zimmerman

Dewar-Zimmerman also predicts the [1,3] suprafacial migration to be


CH3 CH3 CH3 forbidden.

X Y •• X Y The basis set of s and p orbitals with arbitrary phase:


X Y

Two Phase Inversions


■ Construct TS by considering an allyl anion and the methyl cation: Hückel Topology
Four Electrons
Forbidden thermally
Retention at carbon Inversion at carbon
H H
2p on Carbon H
H
C H C
Orbital interactions in the Completing the circuit
H antibonding
parent system
bonding across the bottom face
bonding H H bonding

X H H
Y X H H
Y
Ψ2 (allyl anion HOMO)

The [1,5] shift of a hydrogen atom across a diene.


Suprafacial on allyl fragment Suprafacial on allyl fragment

■ The analysis works if you consider the other ionic reaction, or consider H H
a radical reaction. In each case it is the same pair of orbitals interacting.

■ Under photochemical conditions, the [1,3] rearrangement is allowed Zero Phase Inversions
suprafacially with retention of stereochemistry. Hückel Topology
Six Electrons
■ The stereochemical constraints on the migration of carbon Allowed thermally
with inversion of configuration is highly disfavored on the basis of strain.
Such rearrangements are rare and usually only occur in highly strained Orbital interactions in the Completing the circuit
systems. parent system across the bottom face

Using a similar analysis, one can prove that [1,5] hydrogen and alkyl
shifts should be allowed when suprafacial on the pi component and
proceeding with retention. Please refer to Fleming for more applications
of FMO theory to [1,n] sigmatropic shifts.

11-08-[1,3]sig/FMO/DZ 10/11/00 7:58 AM


D. A. Evans, B. Breit, T. B. Dunn [3,3]-Sigmatropic Rearrangements Chem 206

[3,3] Rearrangements:
■ The Toggle Algorithm:
A thermally allowed reaction in either of two geometries, the "chair" or The toggle algorithm is a simple way to take one reaction of each
the "boat" geometry. Depicted below is the "chair" geometry. You class that you remember is allowed (or forbidden) and derive if the
should be able to work out the details of the "boat" geometry yourself. reaction is allowed or forbidden under new conditions.

■ How does it work?
X X X
All of the various parameters of the pericyclic reaction are the
Z Z Z input variables, the "switches."
The output is either "allowed" or "forbidden."
X & Z = C, O, N etc Write out all the relevant parameters of a reaction together with
the known result.
Each time you change a parameter by one incremental value
The FMO Analysis: ("toggle a switch"), the output will switch.
This is the prediction of the reaction under the new parameters.
Bring two Allyl radicals together to access for a possible bonding
interaction between termini. ■ So it's nothing really new, is it?
‡ No, its just a convenient way to rederive predictions without

memorizing a table of selection rules.

An Example:
Take the [1,3] sigmatropic rearrangement of an alkyl group. We
know this is forbidden under thermal conditions in a supra-supra
bonding manner, and so we make it the first entry in the table.
The nonbonding
allyl MO
bonding
Ψ2
Rearrangement Conditions Component 1 Component 2 Output

[1,3] Heat Suprafacial Suprafacial Forbidden


The Dewar-Zimmerman Analysis:
[1,3] Heat Antarafacial Suprafacial Allowed

[1,3] Light Antarafacial Suprafacial Forbidden

[1,5] Heat Suprafacial Suprafacial ?


Two Phase Inversions
Hückel Topology
Six Electrons
Allowed Thermally Each incremental change in the "input" registers changes the "output"
register by one. Multiple changes simply toggle the output back and
forth. What is the prediction in the last line?
11-09-[3,3]sig/FMO/DZ 10/11/00 7:59 AM
D. A. Evans, B. Breit, T. B. Dunn Cycloaddition Reactions Chem 206

The Stereochemical issues:


The [4+2] Cycloaddition: Dewar-Zimmerman
In a cycloaddition, a pi system may be attacked in one of two distinct
ways. If the pi system is attacked from the same face, then the reaction The most well known cycloaddition is the Diels-Alder reaction between
is suprafacial on that component. If the system is attacked from a four pi component (the diene) and a two pi component (the
opposite faces, then the reaction is antarafacial on that component. dienophile). An exhaustive examination of this reaction is forthcoming,
so we will limit ourselves to a simple examination.

Suprafacial
attack Antarafacial
attack

Zero Phase Inversions


Hückel Topology
The [2+2] Cycloaddition: FMO Analysis Six Electrons
Allowed thermally
For the [2+2] cycloaddition two different geometries have to be
considered.

Suprafacial/Suprafacial Antarafacial/Suprafacial
bonding HOMO
HOMO
Summary:
■ There are three fundamentally equivalent methods of analyzing
bonding pericyclic reactions: Two are much simpler than the third.
antibonding
■ Fukui Frontier Molecular Orbital Theory
■ Dewar-Zimmerman Hückel-Möbius Aromatic Transition States
■ Woodward-Hoffmann Correlation Diagrams
LUMO
LUMO bonding ■ Some methods are easier to use than others, but all are equally
correct and no one is superior to another. Conclusions drawn from
Forbidden Allowed
the correct application of one theory will not be contradicted by
another theory.
The simplest approach (Supra/Supra) is forbidden under thermal
activation. The less obvious approach (Antara/Supra) is allowed ■ The principle of microscopic reversibility allows us to look at a
thermally but geometrically rather congested. It is believed to occur in reaction from either the forward direction or the reverse direction.
some very specific cases (e.g. ketenes) where the steric congestion is
reduced. ■ There is a general trend that reactions will behave fundamentally
different under thermal conditions and photochemical conditions.

11-10-Cycloaddition 10/11/00 7:59 AM


D. A. Evans Pericyclic Reactions: Part–2 Chem 206

■ Other Reading Material:


C. Palomo, "Asymmetric Synthesis of β-Lactams by Stauginger Ketene-Imine
http://www.courses.fas.harvard.edu/~chem206/ Cycloaddition Reaction, Eur. J. Org. Chem. 1999, 3223-3235.

R R R R R R
Chemistry 206 N N
Correlate with
O R O R

Advanced Organic Chemistry H


N3 H S N3 S N3
con S
C N N N
O O O
Lecture Number 12

R N3 R R
con N3

N N R
Pericyclic Reactions–2 R O
N R
O
■ Electrocyclic Reactions
■ Cheletropic Reactions ■ Problems of the Day:
Predict the stereochemical outcome of this reaction.
O Ph
Ph O O O

■ Reading Assignment for week: ❉


O
Carey & Sundberg: Part A; Chapter 11 heat ❉
Ph
Concerted Pericyclic Reactions Ph O
Huisgen, TL, 1964, 3381.
Fleming: Chapter 4
Thermal Pericyclic Reactions
Suggest a mechanism for the following reaction.

H CO2Me
CO2Me H
MeO2C
heat
Monday,
D. A. Evans October 12, 2003 CO2Me
H
H
Bloomfield, TL, 1969, 3719.
12-00-Cover Page 10/13/03 9:03 AM
Evans, Breit Electrocyclic Processes-1 Chem 206
Electrocyclic Reaction - Selection Rules Controtation Ψ1 and Ψ2 on to the indicated bonding and anti-bonding
orbitals of cyclobutene:
Ground State Excited State LUMO
(Thermal process) (Photochemical Process)
4n π e-
conrotatory disrotatory Ψ2 Con
(n = 1,2...) HOMO

4n+2 π e-
disrotatory conrotatory
(n = 0,1,2...)

Examples Ground State Excited State


Con LUMO
Ψ1
Conrotatory Disrotatory
HOMO

Disrotatory Conrotatory Activation Energy (kcal/mol)


for electrocyclic ring opening

42 45
Conrotatory Disrotatory

29 27
Disrotatory Conrotatory
H H
Conrotatory Disrotatory
Criegee, Chem. Ber. 1968, 101, 102.

O Ph
Conrotatory Disrotatory Ph O O O

O
Disrotatory Conrotatory Ph
Ph O

O Ph
R R Ph O O O
R Con
Con O
R R Ph
R R R
Ph O
Sterically favored Huisgen, TL, 1964, 3381.
12-01-Electroclycliz-1 10/12/03 4:39 PM
Evans, Breit Electrocyclic Processes-2: Torquoselectivity Chem 206

Torquoselectivilty is defined as the predisposition of a given R Donor substituents prefer con–out mode
substituent for a given conrotatory motion How do we explain?
Pi acceptor substituents prefer con–in mode
Houk et al. Acc. Chem. Res 1996, 29, 471
R
R View the 2 conrotatory modes by looking at
R the breaking sigma bond from this perspective
R con con H
in H out
R R
Donor substituents prefer con–out mode
Examples: H
Pi acceptor substituents prefer con–in mode H

R Outward Motion Inward Motion


R
con R
+
H
H
H H H
H H H
R = Me only none
H
only H H
R = CHO none
LUMO + p
LUMO + p

CH2OBn CH2OBn
CHO
con H
CH2OBn
H +
H CHO A H H H H H B
H H
H
CHO H
ratio: >20:1
HOMO + p HOMO + p

destabilizing 4 electron
Me CN interation for donor
CN As conrotation begins the energy of substituents
con CN Me
+ the breaking sigma bond rises
Me steeply. Hyperconjugation with a pi*
orbital, while possible in both A & B, stabilizing 2 electron
ratio: 4:1 is better in B. (Houk) interation for acceptor
substituents

12-02-Electroclycliz-2 10/12/03 4:40 PM


Evans, Breit Electrocyclic Processes-3: 3-Atom Electrocyclizations Chem 206

Three-Atom Electrocyclizations (2 electrons) Solvolysis of Cyclopropyl Derivatives

A H A
Does solvolysis proceed via cation 1 followed by rearrangement to 2
A
H H (Case 1), or does it proceed directly to 2 (Case 2)?
Dis?? Con??
R R C R
A H slow fast
A H X H
H A Case 1
–X–

1 2
fast +X–
Ψ3
slow fast
Case 2 X
–X– +X–
X
Ψ2 nonbonding 2

TsO TsO Me TsO H


Me H
Ψ1 Me
H H H H
cation anion
H Me
relative rate
1 4 40,000
LUMO DePuy, Accts. Chem. Res. 1967, 1, 33
H H
+ + X H
Ψ1 R A C
R A Dis LUMO Me
H A H C
C Ψ1
A HOMO R
R H
Me +
H C Dis H
Me
Me
Note that there are two disrotatory modes
HOMO
R R R R Me
X
R Dis X R Dis LUMO H
R Me C
Ψ1 R Me
X Sterically favored Favored for R = ring R H +
R C Dis Me
H
H
HOMO
12-03-Electroclycliz-3 10/12/03 4:41 PM
Evans, Breit Electrocyclic Processes-3: 3-Atom Electrocyclizations Chem 206
dis-in dis-out Three-Atom Electrocyclizations (4 electrons)
Solvolysis Summary Unfavorable favorable

TsO TsO A H A A
TsO Me H
Me H H H
Dis?? Con??
R R C R
H H Me
H H A H
H Me A H
H A
relative rate 1 4 40,000

Ring-fused Cyclopropyl Systems


Ψ3
When the cis substiltutents on the cyclopropyl ring are tied together
in a ring the following observsations have been made

Ψ2 nonbonding
TsO TsO

H H H
H
H H Ψ1
favored cation anion
dis-in
relative rate: > 10+6 B
A ••
H 2C B
H R C Con C C
H B
CH2 Ψ2 A A
H R C
TsO H B
H TsO H A
H 2C
disavored
dis-out Observation H CO2Me
CO2Me
•• Con Ar N
Revisiting the Favorski rearrangement: (Carey, Part A, pp 506-8) Ar N (+)
•• (–)
O O– O– O H CO2Me MeO2C
Cl base Cl dis-in
H CO2Me MeO2C

–Cl •• Con
Ar N Ar N
3-exo-tet (+)
•• (–)
MeO2C H
disallowed MeO2C
products
12-04-Electroclycliz-4 10/12/03 4:42 PM
Evans, Breit Electrocyclic Processes-3 Chem 206

Five-Atom Electrocyclizations (4 electrons) The Nazarov Reaction


O O
OH
R R R +H+ –H+
Dis?? ❋ Con??
❋ ❋ A A
A A A A
A A
A A A A
Denmark, S. E. In Comprehensive Organic Synthesis; Trost, B. M.,
Fleming, I., Eds.; Pergamon Press: Oxford, 1991; Vol. 5; pp 751.
O O
Ψ3 nonbonding +
+H predict
stereochemistry

H H

Ψ2
Eight-Atom Electrocyclizations (8 electrons)

Ψ1
A A
Cation Anion A
Dis?? A Con??

Pentadienyl Cation A A
A LUMO H
C HOMO
Con +
Ψ2 ● A Let's use the "Ready" shortcut to find the homo: Nodes will appear at
A
C single bonds

A H
symmetry of homo
Pentadienyl Anion
A H
● C
Dis –
Ψ3 •• A
H
● C Ψ4 Closure should be conrotatory
A A

12-05-Electroclycliz-5 10/12/03 4:44 PM


Evans, Breit Cheletropic Processes-1 Chem 206

CHELETROPIC REACTIONS: [n+1] Cycloadditions (or Cycloreversions)


2 + 1 CheletropicReaction: Olefins + Singlet Carbene
Concerted processes in which 2 σ-bonds are made (or broken) which terminate at
a single atom. R R
C C
[4+2] R
R
+

Linear Approach: 2 HOMO-LUMO Interactions


O [4+1] O
+ S S
O O C R C
R
R R
C C
General
HOMO
LUMO
LUMO HOMO
Reversion Y
π-system

Y π-system

X + X Nonlinear Approach: 2 HOMO-LUMO Interactions


Addition
Z Z
R R R R
Y C C
X C C O N N N N O SO2
C C
Z LUMO
Singlet-Carbene Cycloreversion only Reversion
Addition (and Reversion) and Addition HOMO
LUMO HOMO
Frontier Orbitals ω0 p (empty) E
ω0 Carry out the analysis of the indicated hypothetical transformation
Y
sp2 (filled) ω2 X ω2

Z Me Me
Y Y
C C
Z Z
R
Question: what is orientation of carbene Me
C Me
relative to attacking olefin??
R

predict approach geometry of carbene

12-06-cheletropic-1 10/12/03 4:44 PM


Evans, Breit Cheletropic Processes-2 Chem 206

Let's now consider SO2 as the one-atom component Key step in the Ramberg Bäcklund Rearrangement

R1 R2 R1
E
O O O O S -SO2 R 2
S S S S X O O
O O O O
base base
R1 S R2
4e– in pi system O O
Ψ1 filled Ψ2 filled Ψ3 empty R1 R2
R1 R2
S Z
Me Me -SO2
O O
O
suprafacial Clough, J. M. The Ramberg-Backlund Rearrangement.; Trost, B. M. and Fleming, I.,
S
Ed.; Pergamon Press: Oxford, 1991; Vol. 3, pp 861.
O "The Ramberg-Backlund Rearrangement.", Paquette, L. A. Org. React. (N.Y.) 1977,
O Me 2 5, 1.
Me reactions are:
S stereospecific & reversible
O Me
Me Analysis of the Suprafacial SO2 Extrusion (nonlinear)
O
suprafacial R1
S R1 R2
O + SO2
Me 2 S
Me R
O O

O O O
O
S S S
S O–
O O O–

HOMO
Ψ3 empty (LUMO) Ψ3 empty (LUMO)
HOMO

O O
O O
S S S
O S
O O–
O–
LUMO
Ψ2 filled Ψ1 filled
LUMO
Similar to carbene geometry
12-07-cheletropic-2 10/12/03 4:50 PM
D. A. Evans Sigmatropic Rearrangements-1 Chem 206

■ [1,3] Sigmatropic Rearrangements (C migration)


Sigmatropic rearrangements are those reactions in which a sigma bond
CH3 CH3
(& associated substituent) interchanges termini on a conjugated pi system ∆
consider the 1,3-migration of Carbon
X Y X Y
■ Examples: X X
∆ H3
[1,3] Sigmatropic rearrangement ‡
R Consider the orbitals needed to contruct C
R
the transition state (TS).
X Y
Y:– –:X
X Y

[2,3] Sigmatropic rearrangement ❐ Construct TS by uniting an allyl and Me radicals:
R R
Retention at carbon
X X Inversion at carbon
∆ R
[3,3] Sigmatropic rearrangement X
C R antibonding
H C
X bonding bonding H X bonding
X

H ∆ H X H Y X H Y
[1,5] Sigmatropic rearrangement H H

Suprafacial on allyl fragment Suprafacial on allyl fragment


■ [1,3] Sigmatropic Rearrangements (H migration) Sychronous bonding to both termini Sychronous bonding to both termini
H H cannot be achieved from this geometry is possible from this geometry

consider the 1,3-migration of H
X Y X Y
‡ ❐ The stereochemical constraints on the suprafacial migration of carbon
H with inversion of configuration is highly disfavored on the basis of strain.
Consider the orbitals needed to contruct
the transition state (TS). X Y
[1,3]-Sigmatropic rearrangements are not common
■ Construct TS by uniting an allyl and H radical: D D

bonding no observed scrambling of labels ✻ X ✻
bonding H antibonding Me
H Me ‡
X Y H
H C Me
X H Y Ψ2 (allyl HOMO) 1
1
H 120 °C H
bonding
Suprafacial Geometry Antarafacial Geometry 3 1 3 3

Bridging distance too great for antarafacial migration. These rearrangements are only seen in systems that are highly strained,
an attribute that lowers the activation for rearrangement.
12-08-Sigmatropic-1 10/12/03 5:29 PM
D. A. Evans Sigmatropic Rearrangements-2 Chem 206

SIGMATROPIC REACTIONS - FMO-Analysis ■ [1,5] Sigmatropic Rearrangements (C migration)

2 4 2 4
1
∆/hν 5
1
3 5 R = H, CR3 3
R R [1s,5s] alkyl shift ⇒ RETENTION
■ [1,5] Sigmatropic Rearrangements (H migration) [1a,5a] alkyl shift ⇒ INVERSION
disfavored

Ψ4
■ [1,5] (C migration): Stereochemical Evaluation
Me Me
Me
RETENTION
Ψ3 nonbonding
230-280°C H

[1,5s]C- shift [1,5s]H- shift
Me Me Me
Ψ2

Dewar–Zimmerman Analysis: Retention


Ψ1
R R
thermal photochemical
H H
H
View as cycloadditon between following species: H

R R R
R
H H R
H
H H H
R suprafacial preferred H
R H
H H
H H
+
R
0 phase inversions ⇒ Huckel toplogy
6 electrons
therefore, allowed thermally
either, or pentadienyl radical Ψ3 pentadienyl radical Ψ3

12-09-Sigmatropic-2 10/12/03 5:29 PM


D. A. Evans Sigmatropic Rearrangements-3 Chem 206

■ [1,2] Sigmatropic Rearrangements: Carbon The Wittig Rearrangement [1,2]

"[2,3]-Wittig Sigmatropic Rearrangements in Organic Synthesis.", Nakai,


[1,2] Concerted sigmatropic rearrangements to cationic centers allowed T.; Mikami, K. Chem. Rev. 1986, 86, 885.
Marshall, J. A. The Wittig Rearrangement.; Trost, B. M. and Fleming, I.,
Ed.; Pergamon Press: Oxford, 1991; Vol. 3, pp 975.
R R
+ +
Li
R R
O BuLi O
consider as cycloaddition

C–R homoylsis ●

R
This 1,2-sigmatropic R● R

● + rearrangement is Li
● O Li
non-concerted O
R

olefin radical cation


transition state
The Wittig Rearrangement [2,3]
Li R
[1,2] Concerted sigmatropic rearrangements to carbanionic centers not observed

O R
OLi
R ●●
R
●●
stepwise
C–R homoylsis
● ●●

consider as cycloaddition C O
● ●●


C–R homoylsis C O
R R
antibonding H
●●
ketyl radical
● ●● ●

R Allyl radical
R
H
olefin radical anion transition state
Allyl radical
12-10-Sigmatropic-3 10/12/03 5:29 PM
D. A. Evans Pericyclic Reactions: Part–3 Chem 206

■ Other Reading Material:


http://www.courses.fas.harvard.edu/~chem206/ [2,3] Sigmatropic Rearrangements

Trost, Ed., Comprehensive Organic Synthesis 1992, Vol 6, Chapter 4.6:


Chemistry 206 Nakai, T.; Mikami, K. Org. React. (N.Y.) 1994, 46, 105-209.
Hoffmann, Angew. Chem. Int. Ed. 1979, 18, 563-572 (Stereochemistry of)
Advanced Organic Chemistry Nakai, Chem. Rev. 1986, 86, 885-902 (Wittig Rearrangement)
Evans, Accts. Chem. Res. 1974, 7, 147-55 (Sulfoxide Rearrangement)
Vedejs, Accts. Chem. Res. 1984, 17, 358-364 (Sulfur Ylilde Rearrangements)
Lecture Number 13
[3,3] Sigmatropic Rearrangements

Trost, Ed., Comprehensive Organic Synthesis 1992, Vol 5,


Pericyclic Reactions–3 Chapter 7.1: (Cope, oxy-Cope, Anionic oxy-Cope)
Chapter 7.2, Claisen

■ Introduction to Sigmatropic Rearrangements S. J. Rhoades, Organic Reactions 1974, 22, 1 (Cope, Claisen)
■ [2,3] Sigmatropic Rearrangements
S. R. Wilson, Organic Reactions 1993, 43, 93 (oxy-Cope)

T. S. Ho, Tandem Organic Reactions 1992, Chapter 12 (Cope, Claisen)

Paquette, L. A. (1990). “Stereocontrolled construction of complex cyclic


■ Reading Assignment for week: ketones by oxy-Cope rearrangement.” Angew. Chem., Int. Ed. Engl. 29: 609.
Carey & Sundberg: Part A; Chapter 11
Concerted Pericyclic Reactions
Fleming: Chapter 4 ■ Problems of the Day:
Thermal Pericyclic Reactions
Provide a mechanism for this transformation.
Me Me
heat
S Me Me S Me
Me S
Me Me
Wednesday, PPh3 S=PPh3
D. A. Evans October 14, 2003
For study on this [2,3] rxn See Baldwin JACS 1971, 93, 6307

13-00-Cover Page 10/15/03 8:53 AM


D. A. Evans Sigmatropic Rearrangements-1 Chem 206

■ [1,3] Sigmatropic Rearrangements (C migration)


Sigmatropic rearrangements are those reactions in which a sigma bond
CH3 CH3
(& associated substituent) interchanges termini on a conjugated pi system ∆
consider the 1,3-migration of Carbon
X Y X Y
■ Examples: X X
∆ H3
[1,3] Sigmatropic rearrangement ‡
R Consider the orbitals needed to contruct C
R
the transition state (TS).
X Y
Y:– –:X
X Y

[2,3] Sigmatropic rearrangement ❐ Construct TS by uniting an allyl and Me radicals:
R R
Retention at carbon
X X Inversion at carbon
∆ R
[3,3] Sigmatropic rearrangement X
C R antibonding
H C
X bonding bonding H X bonding
X

H ∆ H X H Y X H Y
[1,5] Sigmatropic rearrangement H H

Suprafacial on allyl fragment Suprafacial on allyl fragment


■ [1,3] Sigmatropic Rearrangements (H migration) Sychronous bonding to both termini Sychronous bonding to both termini
H H cannot be achieved from this geometry is possible from this geometry

consider the 1,3-migration of H
X Y X Y
‡ ❐ The stereochemical constraints on the suprafacial migration of carbon
H with inversion of configuration is highly disfavored on the basis of strain.
Consider the orbitals needed to contruct
the transition state (TS). X Y
[1,3]-Sigmatropic rearrangements are not common
■ Construct TS by uniting an allyl and H radical: D D

bonding no observed scrambling of labels ✻ X ✻
bonding H antibonding Me
H Me ‡
X Y H
H C Me
X H Y Ψ2 (allyl HOMO) 1 1
H 120 °C H
bonding
Suprafacial Geometry Antarafacial Geometry 3 1 3 3

Bridging distance too great for antarafacial migration. These rearrangements are only seen in systems that are highly strained,
an attribute that lowers the activation for rearrangement.
13-01-Sigmatropic-1 10/15/03 8:53 AM
D. A. Evans Sigmatropic Rearrangements-2 Chem 206

SIGMATROPIC REACTIONS - FMO-Analysis ■ [1,5] Sigmatropic Rearrangements (C migration)

2 4 2 4
1
∆/hν 5
1
3 5 R = H, CR3 3
R R [1s,5s] alkyl shift ⇒ RETENTION
■ [1,5] Sigmatropic Rearrangements (H migration) [1a,5a] alkyl shift ⇒ INVERSION
disfavored

Ψ4
■ [1,5] (C migration): Stereochemical Evaluation
Me Me
Me
5 RETENTION
Ψ3 nonbonding 5
230-280°C H
hν 3
1 [1,5s]C- shift 1 [1,5s]H- shift
Me Me Me
Ψ2

Dewar–Zimmerman Analysis: Retention


Ψ1
R R
thermal photochemical
H H
H
View as cycloadditon between following species: H

R R R
R
H H R
H
H H H
R suprafacial preferred H
R H
H H
H H
+
R
0 phase inversions ⇒ Huckel toplogy
6 electrons
therefore, allowed thermally
either, or pentadienyl radical Ψ3 the transiton structure

13-02-Sigmatropic-2 10/15/03 8:53 AM


D. A. Evans Sigmatropic Rearrangements: An Overview Chem 206

[1,2] Sigmatropic Rearrangements: Carbon The Wittig Rearrangement [1,2]

"[2,3]-Wittig Sigmatropic Rearrangements in Organic Synthesis.", Nakai,


[1,2]-Sigmatropic rearrangements to cationic centers allowed. T.; Mikami, K. Chem. Rev. 1986, 86, 885.
Wagner-Meerwein Rearrangement Marshall, J. A. The Wittig Rearrangement.; Trost, B. M. and Fleming, I.,
Ed.; Pergamon Press: Oxford, 1991; Vol. 3, pp 975.
R R
+ +
Li
R R
O BuLi O
consider as cycloaddition

Ea ~16 Kcal/mol
FMO analysis

This 1,2-sigmatropic R● R
R ●
Li
rearrangement is Li
non-concerted O O

● +
R

olefin radical cation


transition state The Wittig Rearrangement [2,3]
R
R
[1,2]-Sigmatropic rearr to carbanionic centers not observed
●●
concerted
O OLi
R ●●
R
●●
stepwise

FMO analysis
consider as cycloaddition ● ●●
● ●●

C O
FMO analysis ●
C O
R
antibonding ketyl radical R
● ●●
● ● ●● H
R
R
H transition state
olefin radical anion transition state
Allyl radical The ∆∆G‡ between concerted and
non-concerted pathways can be quite small
13-03-Sigmatropics-3 10/15/03 8:53 AM
D. A. Evans [2,3]-Sigmatropic Rearrangements: An Introduction Chem 206

[2,3] Sigmatropic Rearrangements


Me Me
O BuLi O OH OH
■ The basic process:
Me Me temp Me Me
R2 R2 ‡ R2 Me
Me
R ✻ R3 R R3 R ✻ R3 Me Me Me Me Me Me Me Me

X :X Y Y Rautenstrauch, Chem Commun. 1979, 1970 –25 °C ~70% ~30%


Y: :X

X & Y = permutations of C, N, O, S, Se, P; however X is usually a heteroatom ■ X - S, Y = C; Sulfonium Ylide Rearrangement:

Attributes: Stereoselective olefin construction & chirality transfer


BuLi [2,3]
■ Representative X-Y Pairs:
S S
N–O (amine oxides) S–P, S–N, S–O (sulfoxides) – Li+ S S
S–C (sulfur ylids) O–P (phosphites) S S
O–C (Wittig rearrangement) N–N, Cl+–C (haloium ylids)
p.
p. N–C (nitrogen ylids) P–C, C–C (homoallylic anions). Lythgoe, Chem Commum 1972, 757

S–S (disulfides)
■ X - N, Y = C; Ammonium Ylide Rearrangement:
An important early paper: Baldwin, J. Chem. Soc., Chem. Comm. 1970, 576
Sommelet-Hauser:
■ General Reviews:
Trost, Ed., Comprehensive Organic Synthesis 1992, Vol 6, Chapter 4.6: [2,3] H base
Nakai, T.; Mikami, K. Org. React. (N.Y.) 1994, 46, 105-209. BuLi
Hoffmann, Angew. Chem. Int. Ed. 1979, 18, 563-572 (Stereochemistry of) –
Nakai, Chem. Rev. 1986, 86, 885-902 (Wittig Rearrangement) Me CH2 CH2 NMe2 Me NMe2
Evans, Accts. Chem. Res. 1974, 7, 147-55 (sulfoxide Rearrangement) N N
Me Me
Vedejs, Accts. Chem. Res. 1984, 17, 358-364 (Sulfur Ylilde Rearrangements) Me Me

■ X - O, Y = C; Wittig Rearrangement: Review, Pines, Org. Rxns 1970, 18, 416

Me Me Me Me Me Me
BuLi [2,3] Modern versions of Stevens:
O O – Li+ LiO Ph R2 R2 R2
Ph Ph R1 R3 R1 R3 [2,3] R1 R3
Baldwin, JACS 1971, 93, 3556 BuLi
R + +
R R• Me N Me N
Li Li –
O O [1,2] Me Me2N CN
BuLi – Li+ Me
O • H O R CN CN
Ph Ph
Ph Ph Buchi, JACS 1974, 96, 7573
Garst, JACS 1976, 98, 1526 Mander, JOC 1973, 38, 2915
important extension lacking CN FG; Sato, JACS 1990, 112, 1999
13-04-[2,3] introduction 10/15/03 8:55 AM
D. A. Evans [2,3]-Sigmatropic Rearrangements: Introduction-2 Chem 206

■ X - O, Y = C; Wittig-like Rearrangements
■ X - S, Y = O; Sulfoxide Rearrangement
R2
R2 R2 R2 R2 R2
R1 R3 140 °C
R1 R3 R1 R3 R1 ✻ R3 R1 ✻ R3 thiophile R1 ✻ R3
OH +
O S O OH
OMe C– O NMe2 Ar O keq < 1 S BuLi

H NMe2 + NMe2 Ar PhSCl

OMe Evans, Accts. Chem. Res. 1974, 7, 147


R2 Buchi, JACS 1974, 96, 5563
R1 R3
■ X - Se, Y = N; Related Rearrangement
In thinking about this rearrangement,
O
C: also consider the carbenoid resonance
form as well R2 R2 R2
NMe2
R1 ✻ R3 R1 ✻ R3 selenophile R1 ✻ R3
■ X - O, Y = C; An all-carbon Rearrangement +
Se N NH–Ts
Ar N– Se Ts
p.
p. R2 R2 R2 Ar Hopkins, Tet Let. 1984, 25, 15
Ts
R1 R3 Cu(I) R1 R3 R1 R3 Hopkins, JOC 1984, 49, 3647
–N2 Hopkins, JOC 1985, 50, 417
O O : C
N O
H
N ROH ■ X - S, Y = N; Related Rearrangement
note that the product contains the retrons R2
for the enolate Claisen rearrangement R1 R3 Na+
TsO –
Ts–N–Cl TsO [2,3] TsO
Smith, Chem. Commun. 1974, 695; Smith, JOC 1977, 42, 3165 O Ts N
N SPh
S Ts S
OR
Ph Ph (MeO)3P
■ X - N, Y = O; Meisenheimer Rearrangement NaOH

R2 R2
Zn/HOAc R2
R1 R3 R1 R3
R1 R3 85% yield overall N TsO –
+ Me N
Me N O Ts
O– N OH Ts
Me Dolle, Tet Let. 1989, 30, 4723
Me
Tanabe, Tet Let. 1975, 3005

13-05-[2,3] intro-2 10/15/03 8:55 AM


D. A. Evans [2,3]-Sigmatropic Rearrangements: Olefin Geometry Chem 206

■ 1,2-Disubstitution: Good Trans Olefin Selectivity


Me Me
RLi
Starting olefin: Trans (E) selectivity: "only isomer"
‡ O -75 to -50 °C
Ra
X Ra Rb HO Ph
favored Ph
Y Rb
H Y Me
:X 2 LDA Me
Rb H
Ra (E) selectivity: 75%
O -75 to -50 °C
X HO CO2H
Y: ‡ CO2H
Ra Nakai, Tet. Lett 1981, 22, 69
H
Rb
disfavored X H R1 R2 R1 R2 (MeO) P
Ra 3 R1 R2
Y
Y :X S MeOH
Rb O
Ar O S OH
Ar (E) selectivity: >95%
Ra & Rb prefer to orient in pseudo-equatorial positions during rearrangement; RLi R1–X
nevertheless, this is a delicately balanced situation
p.
p. R2
Evans, Accts. Chem. Res. 1974, 7, 147-55
Starting olefin: Cis S
Ar O

Ra
X Ra Rb
favored The preceeding transition state models do not explain some of the results:
Y H
H Y
:X
Rb
Ra ❏ Cis selectivity has been observed: Still JACS 1978, 100, 1927.
X Rb Me
Y: ‡ R Me OH R
Ra n-BuLi OH
highly Rb
disfavored Rb -78 °C R
X H O SnBu3 Me
Ra ratio, 65:35
Y
Y :X
H
❏ However, Cis selectivity is dependent on starting olefin geometry

R
Conclusions n-BuLi R
OH only (E) isomer (91%)
❏ Olefin geometry dictates sense of asymmetric induction in rearrangement -78 °C
Bu3Sn O Me
Me
❏ (Z) Olefin rearrangements might exhibit higher levels of 1,3 induction
❏ Product olefin geometry can be either (E) or (Z) from (E) starting material Several theoretical studies have been published: Good reading
❏ Product olefin geometry will be (E) from (Z) starting material Houk JOC 1991, 56, 5657 (Sulfur ylide transition states)
Houk JOC 1990, 55, 1421 (Wittig transition states)
13-06-Stereochemistry-1 10/15/03 8:56 AM
D. A. Evans [2,3]-Sigmatropic Rearrangements: Olefin Geometry Chem 206

■ Starting olefin: (E) Trisubstituted ■ (Z) selectivity has been observed: Still JACS 1978, 100, 1927.
Me
‡ Me OH
R1 Me n-Bu
X R1 R2 95%, >96% (Z)
n-Bu
favored Y R2 O Li Me
Me H Y
:X
R1 R2 H
n-BuLi
Me -78 °C Me
X
Y: Me n-Bu KH n-Bu
R1 ‡ SnBu3
R2
disfavored H OH
X H O SnBu3 halogen
R1 Y
Y :X
R2 Me Still says that the TS is early, so that the 1,2 interactions in the TS are most
important.
destabilizing
R1–Me interaction can destabilize the (E) transition state while (Z) TS might ‡
be destabilized by R1 interactions with both X-Y and allyl moiety. C4H9 Me Me
p.
p. O n-Bu
(E)-path
H 2C H
H CH2
■ Starting olefin: (Z) ‡
Me
Me Li LiO
R1 Me H
X n-Bu
favored R1 R2
Y H O C 4H 9 ‡ Me
Me H Y CH2–Li
:X H
R1 R2 (Z)-path H
O
H2 C R1 CH2
X R2 Li H Me LiO
Y: ‡ Me
highly R1
disfavored R2 R2
X H ■ (Z) selectivity has also been observed by others: Sato JACS 1990, 112, 1999.
R1 Y
Y :X
H Me Me Me
LHMDS, NH3 NMe2
-70 °C
Conclusions
Me N +Me 76%, (Z):(E) 95:5
❏ Olefin geometry dictates sense of asymmetric induction in rearrangement Me
X- Me NMe2
❏ (Z) Olefin rearrangements might exhibit higher levels of 1,3 induction
❏ Product olefin geometry can be either (E) or (Z) from (E) starting material Cs–F in HMPA

❏ Product olefin geometry will be (E) from (Z) starting material N+ 25 °C Me


Me CH2–TMS
Me 61%, (E):(Z) 100:0
X-
13-07-Stereochemistry-2 10/15/03 8:57 AM
D. A. Evans [2,3]-Sigmatropic Rearrangements: Olefin Geometry Chem 206

Trisubstituted olefins via [2,3]-rearrangement of sulfoxides: ■ Trisubstituted olefins via [2,3]-rearrangement of sulfonium ylides:
‡ Me
R1 Me
PhS Me Me Me
(E)-path R1
O H Bu CuSO4 Bu Bu
Me favored H O
S S 100 °C CO2Me
H S – CO2Me
R1 Ph Ph + S
Ph N2 C(COOMe)2
CO2Me
CO2Me Ph
S
Ar O Me
R1 ‡ (E):(Z) > 90:10 (70%)
Grieco, JOC 1973, 38, 2572
(Z)-path H
Ph H
S
R1 O
disfavored O S
H Me
Ph A general procedure for the direct synthesis of sulfur ylides:

■ In contrast to the previous cases exhibiting (Z) selectivity rearrangements R R R R R


base
(E)-selective rearrangments has been observed: S + :CR2 +S C – +S C H
p.
p. R R R R R
Me Me Me Me
pKa ~ 18 (DMSO
α
α
n-BuLi – γ
S S Li N S
N N Me Me
N N Br N
Me Me Me
Me α/γ = 90:10 (95%) ■ Trisubstituted olefins via Wittig [2,3]-rearrangement:
Me Me
Accts. Chem. Res. 1974, 7, 147-55 RCO3H
C5H11 2 LDA C5H11 (E):(Z) > 95:5 (74%)
Me Me O
Me Me HO CO2H Nakai, Tet Let 1981, 22, 69
[2,3] CO2H
OH Et2NH, MeOH
Me N S However, this reaction is not general:
25 °C Me O
(E):(Z) > 97:3 (80-85%)
N Me
Me Me
Me Me LDA
Me (E):(Z) 31:69
(–) is operationally O Me
(–)
S equivalent to: HO CO2Me
N
H OH CO2Me Nakai, Tet Let 1986, 27, 4511
N
Me

13-08-Stereochemistry-3 10/15/03 8:57 AM


D. A. Evans [2,3]-Sigmatropic Rearrangements: Olefin Geometry Chem 206

■ Trisubstituted olefins via [2,3]-rearrangement: An elegant squalene synthesis Ollis, Chem. Commun 1969, 99

RL Me Me Me
X RL
(E)-path S Me
Y H Me S
RM RM Y
RM Me Me Me
H :X
RL For study on this [2,3] rxn See [2,3] heat
(RL = large)
Baldwin JACS 1971, 93, 6307
X Me Me Me
Y: RL ‡
RM S
(Z)-path H Me Me Me S Me
X RM
RL Y
Y Me
H :X
PPh3 → S=PPh3
One might project that the (E) path will be moderately favored with selectivity
depending on size difference between RL & RM Me Me Me

p.
p. Me Me Me Me Me S Me
Me n-BuLi Me
Me
Me S Me S Li F
Me benzyne
Me
MgBr
Me Me Me Me Me Me Me Me
– +
Rautenstrauch, Helv. Chim Acta 1971, 54, 739 Me S Me
SLi Me
Ph
Me For related [2,3] rxns See
Me This rxn is probably not as [2,3]
stereoselective as advertised Baldwin JACS 1968, 90, 4758
Me Me Baldwin JACS 1969, 91, 3646
Me (E):(Z) = 3:2
Me Me Me Me SPh

Me OH Me Me
Me Me
140 °C Me Me Me
OMe Me O
C:
Li/NH3
H NMe2 NMe2 "gave one major product in high yield"
OMe O
Me Me Me Me
Buchi, JACS 1974, 96, 5563 NMe2
Me
Me Me Me
poorly selective Squalene Me Me Me

13-09-Stereochemistry-4 10/15/03 8:57 AM


D. A. Evans [2,3]-Sigmatropic Rearrangements: Chirality Transfer Chem 206

[2,3] Sulfur Ylide Rearrangement Using a Chiral Auxiliary Chiral Auxiliaries can also be used in the Wittig Rearrangement

Kurth JOC 1990, 55, 2286 and TL 1991, 32, 335


BuLi
Me Me Me O 96% de (61%)
CO2H steps Allylation CO2H CH2OR XC
Me CO2H HO
Me Me
O N O
NH2 SH S
ROCH2
Me
steps
Me
Me
BuLi
Me O
Me Cp2ZrCl2 97% syn; 96% de (43%)
CH2OR XC
HBF4 Me O HO
DBU, -78 °C Me O CH2Cl2
O N O
S
S+ O
p.
p. O ROCH2 Katsuki, Tet Lett 1986, 27, 4577
BF4 - Me N2
Me

Me Me Me Internal Relay of Stereochemistry in C–C Constructions


Me O Me O Me O
S + S S Bu3Sn O Me Me Me Me Me
O– O O
C 3H 7 n-BuLi, THF, -78 °C
Me C 3H 7
Me Me OBOM
Me OH OBOM
66%, 94:4
diastereoselection > 100:1 (57%)

Me Me Me Bu3Sn O Me Me Me Me Me

C 3H 7 n-BuLi, THF, -78 °C


Me O Me O Me O C 3H 7
+ OBOM OH OBOM
S S S Me
O– O O
Kallmerten TL 1988, 29, 6901. diastereoselection > 100:1 (64%)
Me Me
Me 64%, 4:93
Kallmerten TL 1993, 34, 753.
See these papers for other applications Kallmerten TL 1993, 34, 749.
Kallmerten SynLet 1992, 845.

13-10-applications 10/15/03 8:58 AM


D. A. Evans [2,3]-Sigmatropic Rearrangements: Chirality Transfer Chem 206

Internal Relay of Stereochemistry in C–O Constructions Cases where the chirality is exocyclic to the rearrangement

Tandem [ 4+2 ] & [ 2,3 ] Process: Evans, Bryan, Sims J. Am. Chem. Soc. 1972, 2891. Me Me Me
O Me O Me O Me
O n-BuLi O O

+ O
HO ratio 79 : 6 HO
N
S Me SnBu3
N
O Ph S Me
O –
Ph
H 2C O
OMe Na2S, MeOH A Felkin analysis predicts the
major product H C C
HO O

O O
HO
Bruckner, Angew. Chem. Int. Ed. 1988, 27, 278
p.
p. MeO
N cepharamine
Me N
Me

Allylic Ethers to Make Three Contiguous Stereocenters


H C5H11 O
O
15 CO2Et
HSPh, KOt-Bu TBSO Me
CO2Et TBSO OH
C5H11 n-BuLi, -78 °C
Me 94%
H – 15 O
SPh SPh
TMS TMS
1) MCPBA

2) P(OMe)3 TBSO OH
O O Me 4%
Nakai TL, 1988, 29, 4587.
CO2H steps CO2Et TMS
C5H11 15 C5H11
Can you rationalize the stereochemical outcome of this reaction?
OH OH

Taber J. Am. Chem. Soc. 1977, 99, 3513.


Kondo Tet. Lett. 1978, 3927.

13-11-applications-2 10/15/03 8:58 AM


D. A. Evans [2,3]-Sigmatropic Rearrangements: Chirality Transfer Chem 206

The Synthesis of Bakkenolide-A (Evans JACS 1977, 99, 5453) [2,3] Sigmatropic rearrangements respond to subtile steric effects
Me Me Me H
Me Me Me
HO
O HO X: favored
H H C: H Me3C
H X Me3C Y
H H
O :Y
O O H H
CH2 Me
Me Ph
S + Ph
■ Candidate processes: N2=CHCO2Et S
SPh
–C Me3C
Cu(I) catalysis
R3 R3 H CO2Et H CO2Et
R3 25 °C
140 °C
O selectivity: 91:9
OH OMe C: O NMe2 CMe3
CMe3
H NMe2 NMe2 Ph
Buchi, JACS 1974, 96, 5563 S
OMe
O
25 °C Me3C OH
R3 R3 Me3C OSPh
p.
p. R3 (MeO)3P
NaH MeOH H H
S SMe 65 °C S selectivity: 92:8
C C: S SMe CMe3
Evans, JACS, 1972, 94, 3672
N SMe
NHTs Baldwin, Chem Comm 1972, 354 +
N
■ The synthesis: – C CN -10 °C
N selectivity: 90:10
Me H Me3C
Me Me
Me Me Me Me CN
H Mander, JOC, 1973, 38, 2915
Me
O CMe3
MgBr
H HO
H H
■ The comparison of analogous [2,3] & [3,3] rearrangements:
Br OEt
Me
–S SMe
C O
Note that rearrangement is not required to proceed
via the carbenoid. propose altenate mechanism N O
NHTs selectivity: 52:48
heat
Me3C OEt
Me
Me Me H
Me
Me CMe3
HgCl2, HOH MeS NaH
Bakkenolide-A H O H O
H2SeO3 H 65 °C
S heat selectivity: 75:25
S H Me3C
Me
C H
65% (no other isomer) N SMe
House, JOC 1975, 40, 86
NHTs CMe3
13-12-applications-3 10/15/03 8:58 AM
D. A. Evans [2,3]-Sigmatropic Rearrangements: Ring Expansion & Contraction Chem 206

Ring expansion reactions have been investigated A ring contraction using the Wittig Rearrangement

Methods based on sulfur ylides: (review) Vedejs, Accts. Chem. Res. 1984, 17, 358 Me Me Me
R2N–Li

Me – Me
Me
CO2Et
Cu(O) O O HO

S +S 50%
S
N2 CO2Et CO2Et Li
With chiral amide Ph N Ph
82%, 69% ee
bases induction
is observed! Me Me
KOt-Bu TfO CO2Et Me
72%
S S DBU S
+ Aristolactone
72% Me
EtO2C Marshall, JACS 1988, 110, 2925
O
p. Me O
p.
O

OH Methynolide has been synthesized by Vedejs


Me A ring contraction using the Stevens Rearrangement
HO using this ring-expansion methodology
O
Me Vedejs, JACS 1989, 111, 8430
Me Me Me
Et O
Me
O MeO–
An early ring expansion using the Sommelet-Hauser Rearrangement Me + 54%
N MeOH O
Me Ph Me N
Me
N Me Ph
Me Me O salts readily separated
Me
+ NaNH2
N+ Br
N liq NH3 N nonselective Ph Me
Me Me Me
CH2 Me N-alkylation
– Me
H MeO–
O
+ 78%
N MeOH O
Me Ph Me N
83% Me
Me Ph
Both rearrangements afford a single isomer
Hauser, JACS 1957, 79, 4449 N Stevenson, Tet. Lett 1990, 31, 4351
Me

13-13-applications-4 10/15/03 8:58 AM


D. A. Evans Pericyclic Reactions: Part–4 Chem 206

■ Other Reading Material:


http://www.courses.fas.harvard.edu/~chem206/ [3,3] Sigmatropic Rearrangements

Trost, Ed., Comprehensive Organic Synthesis 1992, Vol 5,


Chemistry 206 Chapter 7.1: (Cope, oxy-Cope, Anionic oxy-Cope)
Chapter 7.2, Claisen

S. J. Rhoades, Organic Reactions 1974, 22, 1 (Cope, Claisen)


Advanced Organic Chemistry
S. R. Wilson, Organic Reactions 1993, 43, 93 (oxy-Cope)

Lecture Number 14 T. S. Ho, Tandem Organic Reactions 1992, Chapter 12 (Cope, Claisen)

Paquette, L. A. (1990). “Stereocontrolled construction of complex cyclic


Pericyclic Reactions–4 ketones by oxy-Cope rearrangement.” Angew. Chem., Int. Ed. Engl. 29: 609.

■ [3,3] Sigmatropic Rearrangements: Introduction


■ Cope Rearrangements & Variants ■ Problems of the Day:
■ Claisen Rearrangements & Variants Predict the stereochemical outcome of this Claisen rearrangement
Et Et

■ Reading Assignment for week: O O


Carey & Sundberg: Part A; Chapter 11 144 °C, 6h diastereoselection
Concerted Pericyclic Reactions >87:13
Fleming: Chapter 4: Thermal Pericyclic Reactions
CMe3 CMe3
K. Houk, Transition Structures of Hydrocarbon Pericyclic Rxns Ireland, JOC 1983, 48, 1829
Angew Chem. Int. Ed. Engl. 1992, 31, 682-708
K. Houk, Pericyclic Reaction Transition States: Passions & Punctilios, Accts. Provide a mechanism for the indicated fransformation
Chem. Res. 1995, 28, 81-90
Angew Chem. Int. Ed. Engl. 1992, 31, 682-708 OH
O
KH, ∆ THF
Me
H3O+ quench Me
Friday, H
D. A. Evans Me H
October 16, 2003 Me

Schreiber, JACS 1984, 106, 4038


14-00-Cover Page 10/16/03 7:29 PM
D. A. Evans Introduction to [3, 3]-Sigmatropic Rearrangements Chem 206

General Reviews: The CopeTransition States


S. J. Rhoades, Organic Reactions 1974, 22, 1 (Cope, Claisen) ‡
S. R. Wilson, Organic Reactions 1993, 43, 93 (oxy-Cope) X
T. S. Ho, Tandem Organic Reactions 1992, Chapter 12 (Cope, Claisen)
Trost, Ed., Comprehensive Organic Synthesis 1992, Vol 5,
Chapter 7.1: (Cope, oxy-Cope, Anionic oxy-Cope)
Chapter 7.2, Claisen
CHAIR BOAT

X X X Relative Energy ∆∆G‡: 0 + 5.8 kcal/mol
Z Z Z

? • CHAIR BOAT
X
X & Z = C, O, N etc Relative Energy ∆G°: 0 + 5.3 kcal/mol
Z •
p.
p.
The Boat and Chair geometries for these transition structures are well defined.
X X
O O

The FMO Analysis (Fleming page 101)


Cope Rearrangement, Ea = 33.5 kcal/mol Claisen Rearrangement Ea = 30.6 kcal/mol
Bring two allyl radicals together to access for a possible bonding interaction
between termini.
The Reaction Energetics Goldstein, JACS 1972, 94, 7147

‡ •

X •

X

bonding
∆G523‡ = 46.3 The nonbonding
E allyl MO
∆G523‡ = 40.5 bonding
Ψ2

X X X It is evident that synchronous bonding is possible in this rearrangement

14-01-[3,3] intro 10/16/03 7:16 PM


D. A. Evans The Doering–Roth Experiments Chem 206
Doering/Roth Experiments: Tetrahedron 18, 67, (1962): H
The Results H
Me favored trans-trans:
The Geometry of the transltion state (boat vs chair) can be analyzed via the Me
90%
rearrangement of substituted 1,5-dienes: Me Me
H H
Me
Me Me
Results: H less favored cis-cis:
Me Me Threo isomer H
Me 10%
Me H
Threo isomer H Me
Me
Me
Me
Me Me Me
trans-cis:
H disfavored H < 1%
Me
Me H H
Me Me
Meso isomer
Me Me
■ Measure product composition from rearrangement of each diene isomer
Me favored Me trans-cis: 99.7%
p. H
p. H H
Results: H
H H ∆∆G‡
Me favored Me Meso isomer
trans-trans
Me Me ~ 5.7 kcal/mol
Me
H H Me Me disfavored
Me
trans-trans: 0.3%
Me Me
Predictions: H H
H less favored H H
Threo isomer H cis-cis
H H Me
Me
Ring Strain can be employed to drive the Cope process:
Me
Me H disfavored trans-cis H
H
5-20 °C
H H Brown Chem. Commun. 1973, 319
Me Me
H
Me Me H
Me favored Me trans-cis 120 °C Vogel Annalen 1958, 615, 1
Predictions: H
H H H
Meso isomer H
Me H
Me Me disfavored
Me trans-trans 60 °C
Reese Chem. Commun. 1970, 1519
H
H H H H equilibrium stongly favors this isomer

14-02-Doering/Roth 10/16/03 7:17 PM


D. A. Evans Strain Accelerated-Cope Rearrangements Chem 206
Ring Strain can be employed to drive the Cope process: ■ Ring extension via divinylcyclopropane rearrangement
‡ O O Me
20 °C OMe Me
heat
"quantitative" xylene
Me
Me
90%
I
Li+ Me
W. von E. Doering's Bullvalene – Me
(PhS)Cu 1.5 equiv Me

● ● LDA
● Me
MeI
O

Me (EtO)2P
Me OMe Me OMe Me
Me H2
Me Me
Bullvalene: Ea = 13.9 kcal/mol Li (Ph3)3RhCl
EtNH2/THF LDA
At 100 °C one carbon is observed in nmr spectrum O

Carey, Vol 1, page 630-630 Me Me (EtO)2PCl


p.
p. Me
β-himachalene
Piers, Can J. Chem. 1983, 61, 1226, 1239

O O CO2Et
Me Me CO2Et
O Me
■ Position of Equilibrium dictated by ring strain issues: 140 °C
CHO
H Wittig 60-70%

Vogel Angew. Chem. Int. Ed. + S(O)Me2
1963, 2, 739 Marino, J. Org. Chem. 1974, 39, 3175
H
favored
H
Accelerated Cope Rearrangements
Wharton J. Org. Chem.
1973, 38, 4117 ‡
HO HO HO O
H k1
■ However, tautomerism can shift the equilibrium:
H
keq ~ 10+5
220 °C k2
–O –O
3h k2 = 10 + 10 10 + 17
k1
OH OH 90% O
Evans, Golob, JACS 1975, 97, 4765.
Marvell, Tet. Lett. 1970, 509
Energetically, how much does Trost, Ed., Comprehensive Organic Synthesis 1992, Vol 5,
keq ~ 10+5 Chapter 7.1: (Cope, oxy-Cope, Anionic oxy-Cope)
tautomerization give you?
∆G = 1.4(pKeq) = 1.4X(-5) = -7 kcal/mmol "Recent applications of anionic oxy-Cope rearrangements."
Paquette, L. A. Tetrahedron 1997, 5 3, 13971-14020
14-03-Strain Acceleration 10/15/03 12:03 PM
D. A. Evans The Anionic Oxy-Cope Rearrangement Chem 206

The Aborted Oxy-Cope Reaction (circa 1969) Documentation of Alkoxy Substituent Effect

■ The basic reaction –OX Half-life


OX
OH O –OH (66 yrs)
OX
Me H –OLi no rxn
OH Me Me
66 °C
66 °C
THF H –ONa 1.2 hrs
MeO
–OK 1.4 min
■ Actual case studied: OMe
1 –OK 11 hrs
10 +12 rate acceleration 10 °C
Me Me OH O (HMPT) –O K -+ 4.4 min
4 1 1
3 3
3 OH 4 Me
6
6 6 XO
Me
Me Me Me
Me Me 4 66 °C ∆∆G‡estimate = 15 kca mol -1
No Rxn
p.
p.
THF ∆∆G‡experiment = 13 kca mol -1
Prediction of Substituent Effect (circa 1969) MeO

‡ with A. M. Golob, JACS. 1975, 97, 4765.


HO HO HO
∆G‡OH
∆GOH
Origin of the Rate Effect –O ‡

pka (SM) pka (TS)


Φ HO ‡

–O –O –O
∆G‡O –
∆GO –
??
Effect probably comes from both
–O reactant destabilization
&
∆G‡O – = ∆G‡OH+ 2.3RT [ pka TS – pka SM] transition state stabilization

∆G‡O –= ∆G‡OH + 2.3RT [18 – 29] (in DMSO) ∆ –Φ ~ 15 kcal/mol


HO ∆
∆G‡O – = ∆G‡OH + 1.4 [– 11]
∆G‡O – = ∆G‡OH – 15 kcal/mol at 298 K (in DMSO)

Maximal rates are observed under conditions where reactant is maximally destabilized
14-04-oxy-Cope 10/15/03 12:04 PM
D. A. Evans Anionic Substituent Effects: Bond Homolysis Chem 206

Substituent Effects in Bond Homolysis Substituent Effects in Molecular Rearrangements

DI – – – –
HO C CR3 HO C• + •CR3 X X X X

A B • [3,3] R [1,3] R

DI I
–O C CR3 –O C• + •CR3 – – – –
X Y X Y X Y X Y
ketyl [2,3] R R
R R [1,2]

DI – DI I = 2.3 RT[pka (A) – pka (B)]

– –X
X
H Ph
Acidities of these radicals are known in H O
ene
HO C• HO C• H H
p. HO C• Hayon, Accts. Chem. Res. 1974, 7, 114 2
p.
H Ph
(H2O) pKa = 10.7 pKa = 9.2
– C –
In DMSO: ∆D = 2.3 RT[29 – pka 18] = ~ 15 kcal/mol Y X Y X
C
■ Substituent Effect based on ab initio calculations
(Evans, Goddard, JACS 1979, 101, 1994) C
C –

Y X Y X
H H H H C
C
HO C H NaO C H KO C H –O C H
H H H H C
C
BDE = 90.7 BDE = 80.6 BDE = 79.0 BDE = 74.2
(BDE = 91.8 expt) X – C
∆D C
+16.5 kcal/mol X–

– –
Related papers: Evans, Baillargeon, Tet Lett. 1978, 36, 3315, 3319 X C H X C • – – •
X C • X C

R• R–H

14-05-oxy-Cope-2 10/15/03 12:04 PM


D. A. Evans Anionic Oxy-Cope Rearrangement: Applications Chem 206
O
O Periplanone-B Synthesis
MeO OMe O
O O
MeO OMe Me
OMe O
H OMe Me 200 µg from 75,000
NaH O virgin female cockroaches
H2 C Me H
O OH 75% H H Me
MgX
OH OR
H H

KH
Me Me
Jung, JACS 1978, 100, 4309 ∆ THF
H
Jung, JACS 1980, 102, 2463 ROH2C Me ROH2C Me
Still, JACS 1979, 101, 2493
OH OH
OR
KH, THF O Levine, JOC 1981, 46, 2199
KH
Me
Me O
H ∆ THF
Me
OH O Me Me
Me 2 steps
Schreiber, JACS 1984, 106, 4038
H
KH, THF Me Me
Me Gadwood, JOC, 1982, 47, 2268
Me
Synthesis of (+)-CP-263,114: Shair, JACS 2000, 122, 7424-7425.
O
H
O O
O H Me
O
Me
O
O
O
HC CLi 50 °C
OH CO2H
50 % yield H
Me H H MeO2C R
Me H Me OMe
OH O
R H
C R
Me H [3,3] R O
OH OH
OH H
XMgO CH OR –78→23 °C H R
2
O R
XMgO CH OR
2
Me H Me H
Me Me Me Me H H
Me O 53%
poitediol Dieckmann
dactylol CH2OR
Gadwood, JACS, 1986, 108, 6343

14-06-Oxy-Cope appl 10/15/03 12:04 PM


J. Leighton, D. A. Evans Oxy-Cope Problems Chem 206
Propose a synthesis of α-amorphene using 1-methyl-1,3-cyclohexadiene. Me Me
Me Me Me
H H
? Me O Me
α-amorphene
Me
Me H
THPO A THPO B
Me Me
Gregson, R. P.; Mirrington, R. N. J. Chem. Soc., Chem. Commun. 1973, 598. Propose a three step synthesis of B from A.
Koreeda, et al. J. Org. Chem. 1980, 45, 1172.
Et Me By incorporating a carbonyl group into this structure generate Me Me
Me O
all possible oxy-Cope retrons. H
Which is (are) the most reasonable? Me KHMDS
H
THF, 20 °C
Bérubé, G.; Fallis, A. G. Tetrahedron Lett. 1989, 30, 4045. OH
88% H
H OMe
MeO
H By incorporating a double bond into this structure generate all H Me
possible oxy-Cope retrons. Me
Which is (are) the most reasonable? Me
H Me Ireland, et al. J. Org. Chem. 1981, 46, 4863. Me KHMDS O
H O THF, 20 °C
O OH
H H OMe OMe 51%
Et
OMe H
Propose detailed mechanisms for these reactions.
Rationalize the different behavior of these enol ether isomers.
Me
H H Paquette, L. A.; Reagan, J.; Schreiber, S. L.; Teleha, C. A.
By incorporating a double bond into this structure generate all J. Am. Chem. Soc. 1989, 111, 2331-2332.
possible oxy-Cope retrons. Which is (are) the most reasonable? O Me
Paquette, L. A. et al. Tetrahedron Lett. 1987, 28, 31. H
O O
MeO Me O
H Me mesitylene, ↑↓
O
O N
HO 72 h O
H
Li O 48% Me Me
O Me O
H A
Me Me Me
Propose a detailed mechanism paying particular attention
Propose a synthesis of A using the illustrated dihydropyran synthon. to issues of chemo- and stereoselectivity.
Oplinger, J. A.; Paquette, L. A. Tetrahedron Lett. 1987, 28, 5441. Jacobi, P. A.; Selnick, H. G. J. Org. Chem. 1990, 55, 202.
14-07 Oxy-Cope/probs 10/15/03 12:05 PM
D. A. Evans The Claisen Rearrangement Chem 206
■ General Reviews: Recognition Pattern for Organic Synthesis: An Enforced SN2'
S. J. Rhoades, Organic Reactions 1974, 22, 1 (Cope, Claisen)
Trost, Ed., Comprehensive Organic Synthesis 1992, Vol 5, Ch 7.2 R R R
Ziegler, Accts. Chem. Res. 1977, 10, 227 (Claisen)
Bennett, Synthesis 1977, 589 (Claisen) SN2' –
Blechert, Synthesis 1989, 71 (HeteroCope) O Claisen O O
R. K. Hill, Asymmetric Synthesis vol 3, Ch 8, p503 (chirality transfer)
Ziegler, Chem Rev. 1989, 89, 1423 (Claisen) R R
■ The Reaction: X R

O ∆ O Stereochemical outcome is syn and controlled by hydroxyl stereocenter


∆H ~ –20 kcal mol-1
R R 1 O X

There is good thermodynamic driving force for this reaction. 2


-1 -1 R R
Bonds Broken: C-Cπ (65 kcal mol ) & C-Oσ (85 kcal mol ) O
X
Bonds Made: C-Oπ (85 kcal mol-1) and C-Cσ (85 kcal mol-1)
R O
p.
p. 1 O X
■ Themodynamics of Claisen Variants:
Substituent ∆H (kcal mol-1) R
2
R
X X X
X=H –16 Control of stereocenter 2 evolves into a decision how to
O O X = OH –31 establish the hydroxyl-bearing stereocenter
O
X = NH2 –30
(Benson estimates)
Rearrangements of Aryl Allyl Ethers: Traditional Applications
O OH
O
H
H OR 180-200 oC
~ 30 77%
O O ~ 20

O O O OH
H
180-200 oC Cope
~ 20 kcal/mol O H
OR
~ 30 kcal/mol 91%
O

E:Z = 6.7:1
O OH

Heteroatom substitution at the indicated position increases


exothermicity as well as reaction rate N 65% N
14-08-Claisen-1 10/16/03 7:22 PM
D. A. Evans The Claisen Rearrangement-2 Chem 206
Stereoelectronic & steric constraints Synthesis of Allyl Vinyl Ethers

■ Endocyclic Olefins: Ireland, JOC 1983, 48, 1829


Hg(OAc)2 -EtOH
Et O
OH AcOHg O
O 75%
O OEt (solvent) OEt
Et
144 °C, 6h Watanabe, Conlon, JACS 1957, 79, 2828
Me3C H diastereoselection
>87:13 Bronsted acids can also serve as catalysts
H CH2
O CH2
CMe3
for endocyclic olefins, overlap between developing sigma and pi bonds required. Best Ph O Ph O
CH2
overlap for forming chair geometry. As shown below, bring a radical up to either face
of the allylic radical. As the bond is formed, overlap must be maintained. Parh A Cp2Ti AlMe2 96%
evolves into a chair conformation while Path B evolved into a boat conformation. Cl

R•
H Use of Tebbe's Reagent: Evans, Grubbs, J. Am. Chem. Soc. 1980, 102, 3272.
H R H
p.
p. B Me3C (review) S. H. Pines, Organic Reactions 1993, 43, 1
Me3C A
C C Me3C H
H H2C • H
H2C H 2C
R
R•
The Ireland approach to the bicyclic acid A: JOC 1962, 27, 1118
■ Exocyclic Olefins: House, JOC 1975, 40, 86 X H
OEt
O OH
O Me O Me
O Me
O Claisen
OEt
heat X=H
Me3C OEt Me3C HO HO HO
Me Me A Me Me
Me Me
H H
CMe3 ratio 52:48
H
Me
O Me OH
H O OH O
Me Me Me
O EtOCH=CH2
heat Me3C HH
Me3C
H Hg(OAc)2 H ✻
H HO HO 53% overall
H ratio 75:25 O
CMe3 Me Me Me Me
for exoocyclic olefins, overlap between developing sigma and pi bonds is equally good H
from either olefin diastereoface. In this instance, steric effects dominate & this system
shows a modest preference for "equatorial attack." A related case is provided below. The new stereocenter (✻) introduced via the rearrangement had
the wrong configuration!
14-09-Claisen-2 10/16/03 7:23 PM
D. A. Evans The Claisen Rearrangement: Stereoselective Olefin Synthesis Chem 206
Claisen Rearrangement as vehicle for stereoselective olefin synthesis Faulkner suggests that the installation of other substituents on Claisen
transition states will lead to enhanced reaction diastereoselection:
Consider the following rearrangement:
H O
H X
CHO ke X
ke Et O
Me O 110 °C
X
Me Et (E)

Me e‡
e O Me
O
∆G‡a - ∆G‡e = 1.5 kcal/mol Et
O
Me Et X
Me CHO
Me ka Et X
ka Me H O
H O
110 °C (Z)
Me ‡
a
a‡ Me

Faulkner & Perrin (Tet. Lett. 2783 (1969) have made the correlation between The R2↔X interaction should destabilize a as X gets progressively larger.
∆∆G‡ for rearrangement & ∆G° for the corrresponding cyclohexane# equilibria:
X (E):(Z) found
H Me
p.
p. H– 90:10 Faulkner, Tet Let 1969, 3243
Me O H O ‡ Me– >99:1
∆∆G = +1.5 kcal/mol Faulkner, JACS 1973, 95, 553
MeO– >99:1 Johnson, JACS 1970, 92, 741
H Me Me2N– >98:2
Me H ∆G° = +1.75 kcal/mol
■ Another comparison: (DAE) M. DiMare, Ph. D. Harvard University, 1988
#
Note: The A-value of 2-methyl-tetrahydropyran is +2.86 kcal/mol (LectureNo. 6) X ‡
OY
O
They then suggest that there is a good correlation between cyclohexane "A-values" & OPMB
∆∆G‡ for the rearrangement process. Their case is fortified by the following expamples: R
Et Me
Et Me
O CHO CHO
110 °C R2 O
R2 R2 X OPMB
R1 R1 (E) (Z) Et Me
R1
R1 R2 (E):(Z) found (E):(Z) predicted procedure conditions X T, °C (E):(Z) ratio
Me– Et– 90:10 91:9 Y = Ac, Ireland LDA, TMSCl TMSO– -78→+55 97:3
Me– iPr– 93:07 94:6 Y = H, Johnson HC(OMe)3, H+ MeO– 130 94:6
Et– Et– 90:10 91:9 Y = H, Eschenmoser MeC(OMe)2NMe2 Me2N– 80 97.5:2.5
Faulkner, JACS 1973, 95, 553
14-10-Claisen-3 10/16/03 7:24 PM
D. A. Evans Pericyclic Reactions: Part–5 Chem 206

■ Other Reading Material:


http://www.courses.fas.harvard.edu/~chem206/ Enders, D.; Knopp, M.; Schiffers, R. "Asymmetric [3.3]-sigmatropic
rearrangements in organic synthesis."
Tetrahedron: Asymmetry 1996, 7, 1847-1882
Ziegler, F. E. "The Thermal Aliphatic Claisen Rearrangement." Chem. Rev. 1988,
Chemistry 206 88, 1423.
Gajewski, J. J. "The Claisen rearrangement. Response to solvents and
substituents: The case for both hydrophobic and hydrogen bond acceleration in
Advanced Organic Chemistry water and for a variable transition state." Acc. Chem. Res. 1997, 30, 219-225.
Tietze, L. F. "Domino reactions in organic synthesis." Chem. Rev. 1996, 96,
115-136.
Lecture Number 15 Parsons, P. J.; Penkett, C. S.; Shell, A. J. "Tandem reactions in organic synthesis:
Novel strategies for natural product elaboration and the development of new
synthetic methodology." Chem. Rev. 1996, 96, 195-206.
Pericyclic Reactions–5 Pereira, S.; Srebnik, M. "The Ireland-Claisen rearrangement." Aldrichimica Acta
1993, 26, 17.
■ Claisen Rearrangements & Variants
■ Problems of the Day:
■ Reading Assignment for week:
Carey & Sundberg: Part A; Chapter 11 Propose a mechanism for this transformation
Concerted Pericyclic Reactions
N Me O Me
Carey & Sundberg: Part B; Chapter 6 EtN(iPr)2
O N
Cycloadditions, Unimolecular Rearrangements
Thermal Eliminations TiCl4 O Me
O
Me
Fleming: Chapter 4 Cl MacMillan, JACS 1999, 121, 9726
Thermal Pericyclic Reactions
Wipf, P. Claisen Rearrangements.; Trost, B. M. and Fleming, I., Ed.;
Pergamon Press: Oxford, 1991; Vol. 5, pp 827. Predict the stereochemical outcome of this reaction
New Aspects of the Ireland and Related Rearrangements, OH O OH O
Tetrahedron 2002, 58, 2905–2928 (handout) 2 LDA

Me O Me Me OTMS
Wednesday,
October 20, 2003 Me ❉
D. A. Evans
Kurth, JOC 1985, 50, 1840

15-00-Cover Page 10/20/03 9:40 AM


D. A. Evans The Claisen Rearrangement Chem 206
■ General Reviews: Recognition Pattern for Organic Synthesis: An Enforced SN2'
S. J. Rhoades, Organic Reactions 1974, 22, 1 (Cope, Claisen)
Trost, Ed., Comprehensive Organic Synthesis 1992, Vol 5, Ch 7.2 R R R
Ziegler, Accts. Chem. Res. 1977, 10, 227 (Claisen)
Bennett, Synthesis 1977, 589 (Claisen) SN2' –
Blechert, Synthesis 1989, 71 (HeteroCope) O Claisen O O
R. K. Hill, Asymmetric Synthesis vol 3, Ch 8, p503 (chirality transfer)
Ziegler, Chem Rev. 1989, 89, 1423 (Claisen) R R
■ The Reaction: X R

O ∆ O Stereochemical outcome is syn and controlled by hydroxyl stereocenter


∆H ~ –20 kcal mol-1
R R 1 O X

There is good thermodynamic driving force for this reaction. 2


-1 -1 R R
Bonds Broken: C-Cπ (65 kcal mol ) & C-Oσ (85 kcal mol ) O
X
Bonds Made: C-Oπ (85 kcal mol-1) and C-Cσ (85 kcal mol-1)
R O
p.
p. 1 O X
■ Themodynamics of Claisen Variants:
Substituent ∆H (kcal mol-1) R
2
R
X X X
X=H –16 Control of stereocenter 2 evolves into a decision how to
O O X = OH –31 establish the hydroxyl-bearing stereocenter
O
X = NH2 –30
(Benson estimates)
Rearrangements of Aryl Allyl Ethers: Traditional Applications
O OH
O
H
H OR 180-200 oC
~ 30 77%
O O ~ 20

O O O OH
H
180-200 oC Cope
~ 20 kcal/mol O H
OR
~ 30 kcal/mol 91%
O

E:Z = 6.7:1
O OH

Heteroatom substitution at the indicated position increases


exothermicity as well as reaction rate N 65% N
15-01-Claisen-1 10/19/03 5:47 PM
D. A. Evans The Claisen Rearrangement-2 Chem 206
Stereoelectronic & steric constraints Synthesis of Allyl Vinyl Ethers

■ Endocyclic Olefins: Ireland, JOC 1983, 48, 1829


Hg(OAc)2 -EtOH
Et O
OH AcOHg O
O 75%
O OEt (solvent) OEt
Et
144 °C, 6h Watanabe, Conlon, JACS 1957, 79, 2828
Me3C H diastereoselection
>87:13 Bronsted acids can also serve as catalysts
H CH2
O CH2
CMe3
for endocyclic olefins, overlap between developing sigma and pi bonds required. Best Ph O Ph O
CH2
overlap for forming chair geometry. As shown below, bring a radical up to either face
of the allylic radical. As the bond is formed, overlap must be maintained. Parh A Cp2Ti AlMe2 96%
evolves into a chair conformation while Path B evolved into a boat conformation. Cl

R•
H Use of Tebbe's Reagent: Evans, Grubbs, J. Am. Chem. Soc. 1980, 102, 3272.
H R H
p.
p. B Me3C (review) S. H. Pines, Organic Reactions 1993, 43, 1
Me3C A
C C Me3C H
H H2C • H
H2C H 2C
R
R•
The Ireland approach to the bicyclic acid A: JOC 1962, 27, 1118
■ Exocyclic Olefins: House, JOC 1975, 40, 86 X H
OEt
O OH
O Me O Me
O Me
O Claisen
OEt
heat X=H
Me3C OEt Me3C HO HO HO
Me Me A Me Me
Me Me
H H
CMe3 ratio 52:48
H
Me
O Me OH
H O OH O
Me Me Me
O EtOCH=CH2
heat Me3C HH
Me3C
H Hg(OAc)2 H ✻
H HO HO 53% overall
H ratio 75:25 O
CMe3 Me Me Me Me
for exoocyclic olefins, overlap between developing sigma and pi bonds is equally good H
from either olefin diastereoface. In this instance, steric effects dominate & this system
shows a modest preference for "equatorial attack." A related case is provided below. The new stereocenter (✻) introduced via the rearrangement had
the wrong configuration!
15-02-Claisen-2 10/19/03 5:48 PM
D. A. Evans The Claisen Rearrangement: Stereoselective Olefin Synthesis Chem 206
Claisen Rearrangement as vehicle for stereoselective olefin synthesis Faulkner suggests that the installation of other substituents on Claisen transition states
will lead to enhanced reaction diastereoselection:
Consider the following rearrangement:
H O
H X
CHO ke X
ke Et O
Me O 110 °C
X
Me Et (E)

Me e‡
e O Me
O
∆G‡a - ∆G‡e = 1.5 kcal/mol Et
O
Me Et X
Me CHO
Me ka Et X
ka Me H O
H O
110 °C (Z)
Me ‡
a
a‡ Me

Faulkner & Perrin (Tet. Lett. 2783 (1969) have made the correlation between The R2↔X interaction should destabilize a as X gets progressively larger.
∆∆G‡ for rearrangement & ∆G° for the corrresponding cyclohexane# equilibria:
X (E):(Z) found
H Me
p.
p. H– 90:10 Faulkner, Tet Let 1969, 3243
Me O H O ‡ Me– >99:1
∆∆G = +1.5 kcal/mol Faulkner, JACS 1973, 95, 553
MeO– >99:1 Johnson, JACS 1970, 92, 741
H Me Me2N– >98:2
Me H ∆G° = +1.75 kcal/mol
■ Another comparison: (DAE) M. DiMare, Ph. D. Harvard University, 1988
#
Note: The A-value of 2-methyl-tetrahydropyran is +2.86 kcal/mol (Lecture No. 6) X ‡
OY
O
They then suggest that there is a good correlation between cyclohexane "A-values" & OPMB
∆∆G‡ for the rearrangement process. Their case is fortified by the following expamples: R
Et Me
Et Me
O CHO CHO
110 °C R2 O
R2 R2 X OPMB
R1 R1 (E) (Z) Et Me
R1
R1 R2 (E):(Z) found (E):(Z) predicted procedure conditions X T, °C (E):(Z) ratio
Me– Et– 90:10 91:9 Y = Ac, Ireland LDA, TMSCl TMSO– -78→+55 97:3
Me– iPr– 93:07 94:6 Y = H, Johnson HC(OMe)3, H+ MeO– 130 94:6
Et– Et– 90:10 91:9 Y = H, Eschenmoser MeC(OMe)2NMe2 Me2N– 80 97.5:2.5
Faulkner, JACS 1973, 95, 553
15-03-Claisen-3 10/19/03 5:44 PM
D. A. Evans Johnson & Eschenmoser Claisen Rearrangements Chem 206

Johnson Orthoester Claisen Eschenmoser-Claisen


■ Lead paper: Johnson, Faulkner, Peterson, JACS 1970, 92, 741 Eschenmoser, A. Helv. Chem. Acta 1964, 47, 2425; Helv. Chim.Acta 1969, 52, 1030.

OEt
EtCO2H (cat) Xylene, 150oC
Me C OEt Me O O
OH ∆ OH Me OMe
OEt
EtO OEt Et2N OMe NEt2

OEt
O O O
■ Compare the two variants: OEt Et2N OMe NEt2
Me Me Me Me H H
CH3C(OEt)3 Me NMe2
NMe2
CH3C(NMe2)(OMe)2 Et
CH3CH2CO2H (cat) EtO2C Et Et
O O
OH 138oC 92% (E:Z = 98:2) Xylene, 110oC Me Me
p.
p. OH
Faulkner and Peterson High yield, E:Z = 99:1
60% Hg(OAc)2, EVE
CO2Me CO2Me
Me Me Me Me Me
Me Me2N
CH2=C(OMe)(NEt2)
98oC
OHC o 70%
Xylene, 140 C, 14h O
98% (E:Z = 86:14) Me Me
O
OH
■ Synthesis of Amide Acetals
The Saucy Marbet Alternative
Et O OEt OEt
EtO
OMe NaOEt
Me O BF4- + O Et
Me NMe2 Et2O Me NMe2 EtOH Me NMe2
OH Me Et +
O Me Me
Me Me
94% ■ Reactions to ponder:
Me H3PO4 or TsOH Me Me
Me
C 3H 5 R
125oC
Me OLi
diastereoselection 6:1
Me Me CHO OMe Predict the major diastereomer
OH Me O Me +N N O
Me OEt Me Me Me Stevenson, Tet. Let. 1991, 32, 4199
Me H3PO4 Me 60% Me
Me 125oC Me Me OMe O Me
+ Me OLi
Saucy, Marbet, Helv. Chim. Acta 1967, 50, 2091,2095 N Et N
Predict the major diastereomer
Me Welch, JACS 1987, 109, 6716
CMe2(OBn) CMe2(OBn)
15-04-Johnson/Eschenm 10/19/03 5:52 PM
D. A. Evans Ireland Enolate Claisen Rearrangement Chem 206
Ireland-Enolate Claisen Substituted enolates afford an additional stereocenter
Reviews New Aspects of the Ireland and Re;ated Rearrangements, R2 ✽ R1 Me R1 ‡
Tetrahedron 2002, 58, 2905–2928 (handout) O R2 ✽ R1
Me R2
Ireland, R. E.; Mueller, R. H.; Willard, A. K. J. Am. Chem. Soc. 1976, 98, 2868 O H O
(E) H Me
H OTBS
OTBS OTBS
O OTMS OH
LDA conditions (E):(Z) control
Me3SiCl t1/2 (32oC) = 3.5 h O LDA, TBSCl R2 ✽ R1
O Me O Et OEt LDA, TBSCl 94:6 kinetic
66% O
LDA, TBSCl Me O LDA, TBSCl 7:93 thermo
DMPU
DMPU
O

Enolization: Amide Bases & Ireland Enollization Model R2 ✽ R1 H R1 ‡


O R2 R1
R2
O
Stereoelectronic Requirements: α-C-H bond must be able to overlap with π∗ C–O Me Me H O
p.
p. H Me
OTBS (Z) OTBS
Ha OTBS
π∗ C–O Hc Hb key study: Ireland, JOC 1991, 56, 650 and earlier cited papers
base
R C O
– Ha+
Hc Hb R O– Double Claisen Rearrangements are also possible
R
‡ OLi Paterson, Tet Lett 1991, 32, 7601
O
favored H Me Me Me Me Me
Me Me Me Me
R Li R
O N H Me
Me R O O O
R R (E) Geometry OH O OH
Me Me
O
LM–NR2 disfavored Me H OLi O O
Li LDA, TMSCl
R Me Et3N
N H R 1,2 syn aldol relation permuted into
1,5 syn relationship via Claisen rearrangement
R (Z) Geometry Me Me Me Me
The Ireland Model (JACS 1976, 98, 2868); Narula, Tetrahedron Lett. 1981, 22,
Me Me Me Me Me Me 20-60 °C
4119; more recent study: Ireland, JOC 1991, 56, 650 Me Me
RO OR O O O
For a recent study on the effect of amide base structure on (E)/(Z) selectivity in the
cont3ext of the Ireland enolization model see: JOC 1997, 62, 7516. O O O OTBS OTBS
63% yield (diastereoselection 86%)

15-05-Ireland/enolate Claisen 10/20/03 8:54 AM


D. A. Evans Claisen Rearrangement & Chirality Transfer Chem 206

Recent studies on controlling enolization condiltions have apeared These Chelating substituents can be benzyl ethers as well
Yamamoto, JOC 1993, 58, 5301
Kalmerton, Tet Lett 1993, 34, 1103
Me Me BnO OMOM BnO OMOM
O–TMS
N LN(TMS)2/TMSCl
Me Li Me Me Me
(A) OMe (E)
TMSCl, THF CH2N2
O in situ t-BuMe2SiO Me O Me O
OPMB
t-BuMe2SiO O PMB
OMe O O
1) LiN(TMS)2 O–TBS LI
(B)
THF-HMPA t-BuMe2SiO (Z)
OMe BnO OMOM OPMB
1) TBSCl BnO OMOM
OMe Me
OR Et O
(A) O
H Me Me O Me O
O OSiR3 OPMB
H 85% yield
t-BuMe2SiO Et OTBS OTBS OTMS
O Et
ratio 95:5

p. H Et O
p. (B) O
H
OSiR3 A Problem to consider Predict the stereochemical outcome of this reaction
O
Et R OTBS
ratio 99:1 OTBS OH O OTMS O
2 LDA, TMSCl

R O Me R OTMS
2 LDA
Me ❉
Chelating substituents on α-carbon afford (Z)-enolates Li Li
O O ‡ H OTMS
H OTMS H
Fujisawa, Tet Lett 1983, 24, 729 predict product if
this is rearranging R O Me
Me
O
O Me O species
2 LN(TMS)2/TMSCl O Me
HO Me R H
ratio 92:8 (84%) H favored TMSO
O Me CH2N2 OMe TMSO

(E):(Z) ratio 93:7 OH Me3SiCl H ‡ H


H OTMS OTMS
O
(H) Me Me Me
H OTMS O
TMSO larger O
(Me) H OTMS R disfavored Me
R
Me O O H H
TMSO TMSO TMSO
O Me 2 LN(TMS)2/TMSCl smaller
OMe ratio 97:3 (79%) Me
HO CH2N2
O OH A(1,3) strain orients stereocenter
relative size of carbon & oxygen Kurth, JOC 1985, 50, 1840
substituents dictate relative face
selectivity
15-06-Ireland/enolat Claisen-2 10/19/03 5:54 PM
D. A. Evans Applications of the Claisen Rearrangement Chem 206

Johnson Squalene Synthesis: JACS 1970, 92, 741 Faulkner Juvenile Hormone Synthesis: JACS 1973, 95, 553
Me Me Me Et Et Me
O
❉ CO2Me Cecropia Juvenile Hormone
Me
Me Me

Me Me Me
Et Et Me
Observations: Molecule contains an obvious symmetry plane HO
The trisubstituted C=C's are the issue CO2Me
Me

Me OH OH

CHO ❉ MeC(OEt)3
OHC
❉ Me EtCO2H Et Et Me Et Me
❉ HO 3
OH Me 138 °C CO2Me 6 CO2Me
Li Me
6
Me 1O O
Me
❉ LiAlH4 Li
CO2Et
EtO2C Et Me
p. ❉ CrO3-pyr MeC(OEt)3 OR
p. Me Me
87% (E)-(E) 97% EtCO2H
138 °C Me
Me
Me Me CO2Me
CO2Me
❉ Et Me
CO2Et H+
EtO2C OH
❉ 110 °C O 6 CO2Me
Me Me
OMe
O
LiAlH4 Et
CrO3-pyr 85-90% (E)
Et
Et Me NaBH4
Me Me
❉ 6 CO2Me
CHO
OHC
❉ O Et Me
Me Me H+
6 CO2Me
Me 110 °C
+ Et Me
– PPh3 OR OH OMe
Me Me OMe
Et Et Me
Me Me Me HO 3
❉ CO2Me
Me Me Et Me
Me 6 OR
❉ 1O
Me Me Me Et Et Me
HO
NaBH4 CO2Me
Isomeric purity is Ca 95% Me
OH TSCl/pyr Cecropia Juvenile Hormone
nonselective [H]
MeONa
15-07-Applications-1 10/19/03 5:54 PM
D. A. Evans Claisen Rearrangement & Chirality Transfer Chem 206
Chirality transfer via the Claisen rxn is an integral aspect of the
general utility of process Sense of Asymmetric induction may be controlled by olefin geometry
R. K. Hill, Asymmetric Synthesis vol 3, Ch 8, p503 (chirality transfer) R1
O R2 ✽ R1
OEt R2 ✽ R1 MeC(OEt)3 R2
OEt
MeC(OEt)3 H+ H
OH O OH H OEt CO2Et
+ O
H
R2 ✽ R1 R2 ✽ R1
R2 ✽ R1 Na/NH3
✽ R1
Such chirality permutation processes are only as stereoselective as the energy H2 R2
difference between diastereomeric chair transiltion states: OH
Pd
R1 CaCO3
O R2 R1
R2 R1
O R2 ✽ R1
H O ✽ R1 MeC(OEt)3 H
favored
H X
R2 ✽ R1 H+ H
X R2 OH R2 OEt CO2Et

O R2
Since stereoselection in reduction of acetylenes is >98%, either product accessible
p. H R1
p. disfavored X R1
X O
R2 H tocopherol (Vitamin E) Cohen JOC 1976, 41, 3497
O Me JACS 1979, 101, 6710
X
Note that chirality transfer is coupled to olefin geometry in product. Prior arguments HO
(Faulkner) imply that the X substituent will play significant role in promoting selectivity. Me Me Me

Me O Me
Me ✽ Me Me ✽ Me Me
MeC(OEt)3 Me
ee > 90% O Me Me
50%
(E)-selection > 90% 50%
O H+ O
R. K. Hill H Me
X X JOC 1972, 37, 3737 Me MgBr
OH Me Me OH Me Me
CO2Me CO2Me
Me Me
N N
MeC(OEt)3 Me Me
68% enantiomerically pure H2 Pd/CaCO3 NaAlH2(OR)2
Me H+ Me
Uskokovic, Me OH Me Me OH Me Me
OH JACS 1979, 101, 6742
CO2Me
Me Me Me
Me Me Me (Z):(E) = 98:2 (E):(Z) = >99:1
OTMS MeC(OEt)3
Me ✽ ✽ R
Me H+ ETO2C 92%
O Me Me Me
77%
OH O Me O
Heathcock EtO Me
JOC 1988, 53, 1922
diastereoselection ~ 99% from both routes
15-08-Chirality transfer 10/19/03 5:55 PM
D. A. Evans Claisen Rearrangement & Chirality Transfer Chem 206

Boat transition states more accessible in Claisen than in Cope


■ The analysis: H
rearrangements O
H
O
H H
■ A case where the chair-boat preference depends on enol geometry OR
H O Me
Ireland, JACS 1991, 56, 3572 O
Factors controlling diastereoselection OR
Me H
Enolate geometry H H
Chair vs Boat transition states Boat (Z)-enolate Chair (Z)-enolate
Bartlett, JOC 1981, 46, 3896 H H
OR
Ireland, JACS 1991, 56, 3572
H O Me O O
O
LDA,TBSCl A B
Me OR H O O
O
Boat (Z)-enolate Chair (Z)-enolate Me
O O
Ireland study supports Bartlett's conclusions Me Me
conditions A:B TS OR OR
O O LDA,THF 29:71 (E)-boat
LDA,TBSCl A B
p. LDA,THF 86:14 (Z)-boat
p. DMPU H H
Me O O
O O H
(E):(Z) Me Me H
conditions A:B TS OR
LDA,THF 83:17 84:16 (E)-chair OR OR O
H O Me
LDA,THF 04:96 72:28 (Z)-boat Me H OR
DMPU H H
■ A further example: Chair (E)-enolate Boat (E)-enolate
H H
OTBS
OR
Boat geometies can be favored in O Me
H O Me O OMe
these and related systems O
Me H OR OMe boat-preferred
Chair (E)-enolate Boat (E)-enolate RO O Me
H
O Me Me
OMe
■ In this case the boat geometry is preferred from either enol geometry OMe

Me Me O
O O O
LDA,TBSCl A B H
O O H O
? O
O destabilizing H H OTBS
Me
O O TBSO Me Me
conditions A:B TS Me Me
LDA,THF 29:71 (E)-boat OR OR It appears that both of the indicated interactions contribute to the destabilization of
LDA,THF 86:14 (Z)-boat chair geometry
DMPU

15-09-Boat geometries 10/19/03 5:56 PM


D. A. Evans Claisen Rearrangement & Chirality Transfer Chem 206

■ In this case the chair geometry is preferred from either enol geometry The Claisen Rearrangement has been used in fragment coupling
O O CO2H CO2H CO2H
LDA,TBSCl A B Me Me Me
Me O O
Me Me
conditions A:B TS O O 6 O
OR OR
LDA,THF 75:25 (E)-chair O O O
HO RO 3 RO
LDA,THF 40:60 (Z)-chair
O O O O RO O O
DMPU
H H H H H
O O H
R2 1O
R2 OTBS
This destabilizing interaction TBSO R1 Me Me Me
has been attenuated R1
O favored disfavored H H H
H OR Chlorothricolide OR
O [3,3] OR
H
TBSO Me Ireland, J. Org. Chem. 1986, 51, 635 CO2H
Ireland, J. Org. Chem. 1981, 46, 4863
p. Me
p.
■ In this case the boat geometry is preferred from either enol geometry
CO2H O OR
O
O O Me
LDA,TBSCl O O O O
O A B O
CO2H RO
H H
Me O O O O O
Me Me
TS OH O Me
conditions A:B OR OR H H
RO
LDA,THF 43:57 (E)-boat OR H Me
LDA,THF 80:20 (Z)-boat OR
O O
DMPU In the initial approaches to the synthesis, the Schlosser-Wittig H
O O was unsuccessfully attempted for the fragment coupling process. OR
R2 R2 OTBS
R1 OR OR
TBSO R1
Me Me
disfavored favored X = OH
■ Summary:
O O O O O O
O boat-preferred TS from either geometry O O O KHMDS
MeO O
O RO
Me Me TESCl
X = SePh
O O X O O
O O O O H H Bu3SnH
H H
O
Me Me
Me Me 60-72% decarboxylation
chair-preferred TS from either geometry boat/chair TS dependant on enol geometry H H
OMOM OMOM

15-10-Boat geometries-2 10/19/03 5:57 PM key paper for decarboxylation: Ireland, JACS 1985, 107, 3285
D. A. Evans Ireland Claisen Rearrangements: Ionophore Synthesis Chem 206

The Ireland lasalocid synthesis: JACS 1983, 105, 1988 ■ The relevant rearrangements:
Me
Me Me
Me
OH O RO
H A A H
A O Claisen
HO H O RO O
O Et
H Et B H B
CO2H Me Me Et O OH O Et HO OO OR

Et B OR Et
Me O Me
aldol
Me Me Claisen construction Et
O O Me Me
A H
HO H O A H
Me
H Et B OR Me BnO O
CO2H Me Me Et O H
A
CO2H B
O Et O
Et A
Me BnO O O OMOM
H BnO O
Note that neither retron for Claisen Et Cl H LDA Me
exists in this intermediate Et O
Me THF/HMPA 50% overall yield
p.
p. OH B TMSCl
add Claisen retrons O
RO A B
Me O OMOM
O Claisen
H O OMOM Me
Et
A H Me
O
RO O
B OR H B
O Et OR
HO OO The Ireland monensin synthesis:
Et Et
Me Claisen construction
Me
HO Me Me
enolate face selectivity is anticipated
Me problem. This results in CO2H epimers C D
OH Me A B E Me
A O O O
B OR MeO O Me H Et H O
RO O COOR O CH2OH
H O HO
Et Et
Me Me Me→COOH
HO Me JACS 1993, 115, 7152-65
■ Here is another potential Claisen construction
JACS 1993, 115, 7166-72
Me
Methyl-bearing stereocenter to be (and previously cited papers)
established by selective reduction HO
O
A H Et
Me A B O
X O Me O
H Et B Me O
OR MeO O
Et O O O O Me
R Me
A A R
Et
Me O RO O R Me C D
Claisen construction Et
H Et
Me
E Me
Et O O
Et H O
CH2OH
HO
15-11-lasalocid 10/19/03 5:59 PM
D. A. Evans Ireland Claisen Rearrangements: Prostaglandin Synthesis Chem 206

Consider the prostaglandin nucleus Prostaglandin A2 Synthesis, G. Stork, JACS 1976, 98, 1583

O O O H
O H
H H
COOH COOH COOH COOH
Aldol Me 8 MeO2C
O ✻ Me ✻ Me
Me Me
15 H
H
HO H H OH OH
OH H OH
C-8 center can be controlled Dieckmann
Claisen retrons Claisen retrons by equilibration 1O
H
OR
Claisen
O (chirality transfer) MeO 3 COOH
H H
COOH COOH ✻ Me
RO MeO2C
1 OR
O Me H 5
Me OH
H H O 3 COOH
OR OH 3 5
OH
RO ✻ Me
Claisen Claisen
p.
p. 1O OR
O
OR Claisen (trans C=C)
COOH
O COOH
HOOC
OR COOR
Me O Me
O COOH
OH OH
OR OH ✻ Me

O C4H9
OR
O OR
COOR HO OH OR
HO COOH
OR
Me HO MeO2C
HO Me
O OH
HO
OH O O OCO2Me
MgBr
✻ ✻
■ Unrealized plan to generate the required enolate Cl-CO2Me MeC(OMe)3

OCO2Me
O O O O H3O+
+
OMe H
OMe 83% Et3N
Me Me Me Me O O
CONH2 Li(0) CONH2
via erythrose Me Me
O O Me
Me
(MeO)3C (CH2)3CO2Me O
O O O OH (CH2)3CO2Me
M OH MeO
H+
An Application PGA1: Ireland JOC 1976, 41, 986; Aldrichimica Acta 1988, 21, 59 ✻ 12

MeO2C(CH2)2 160 °C, 1h MeO2C
O H O
O O
C12 center and ∆13 C=C 59%
O set in this rxn O
15-12-PG applications 10/19/03 6:00 PM
D. A. Evans Ireland Claisen Rearrangements: Cyclic Enolates Chem 206

■ The previous cases were derived from a connection between R2 &R3 The Indanomycin Synthesis,
OR OR Burke, Tet. Lett. 1985, 26, 1163; ibid, 1986, 27, 6295
R1 R1 O
O O N
O
H H
Me O
R2 R3 N X
OR
H H
R1 Me
O Me
H H
CO2H Et Et
■ Consider consequence of connecting R1 &R2 H
Me O "Right wing"
Et O
R H
R R O CO2H Et
O O
"Left wing"
R1 O ■ The Left Wing: Tet. Lett. 1985, 26, 1163
O O
Me
R3 Me Me
R3 add Claisen H
R2 R3
retrons Claisen O O
Me O Me O
R O O
O equivalent representations H
p.
p. H Me
O CO2H Et CO2H OR O
H
O "Left wing" CO2H
OR Me Me
R3 H
■ Examples: R3 O O
H
O OTMS
Me
LDA/TMSCl 110 °C 4h
Me Me O
Me O
Me 80% yield O
Me O
H H H
Me H OTMS
MeO + H
Ph3C MeO2C BnO BnO
BnO
TBSO -78 °C MeO2C
Me ■ The Right Wing: Tet. Lett. 1986, 27, 6295
O Me OM H
CHO H O Y TMSO
TMSO O
XCO H
Danishefsky, Tet. Let 29, 1371 (1988) X [4+2] O
X Claisen
TMSO2C Me H Et
H Me Me
Me H H H
TMSO Et Et
LDA/TMSCl O CH2X
H H
O H O OTMS
O TMSO
H H
OTMS H H TMSO2C
OTMS O 135 °C 4h
O

Me H Et
Me
65% yield H
Et Et
see also Danishefsky, JACS 1980, 102, 6889, 6891
The "apparent" Claisen process is more complicated than anticipated.
15-13-cyclic enolates 10/19/03 6:00 PM
D. A. Evans Ireland Claisen Rearrangements: Cyclic Enolates Chem 206

HO R R R
■ Now connect R1 &R3 O O O
The FK 506 application
R1
MeO Me O O O
Schreiber Synthesis JACS 1990,112, 5583 H
26 24 22 O
Me R2 R3 R2 R2
O OH
N
H ■ Examples: Funk, JACS 1982, 104, 4030
8 O 19
■ The Schreiber Route: O
O
Me
O H TMSO
H
17 TMSO H
Me O LDA/TMSCl O
O Me FK 506 H
OH
14 O 70%
Claisen disconnection OMe H
OMe O TMSO
R3SiO H
O O O LDA/TMSCl O
OR OR 81%
MeO 29 MeO 29

O O MeO H
p. O
p. TMSO
H H
O O MeO O
TBSOTf O LDA/TMSCl O
110 °C 89%
Et3N OR
H MeO 29 CHMe2
MeO OTBS H
71% O
Me2HC

■ Recent improvments: Funk, JACS 1993, 115, 8847


■ The previous cases were derived from a connection between R2 &R3 O O
OR OR P(OEt)2 P(OEt)2
O O O
R1 R1
O O R1 R1 R1
O O LDA O O
R2 R3 ClPO(OEt)2
OR R2 R2 R2
R1 R3 R3 R3
■ and R1 &R2
chrysanthemic acid application
R
O R O XO TMSO
H H
O
R1 O H Me
O O LDA O
Me Me 65 °C
O ClPO(OEt)2 O 83%
OR Me
R2 R3 Me Me Me Me Me Me H
R3
R3
Me Me
X = TMS; T1/2 = 58 min
X = PO(OEt)2; T1/2 = 10 min
15-14-cyclic enolates-2 10/19/03 6:01 PM
D. A. Evans Claisen Rearrangements: Acyclic Cases with Stereoinduction Chem 206

Exocyclic Stereochemical Issues N-Allylketene-N-O-Acetals: Kurth, JACS 1985, 107, 443


R1 O R1 OR R1 OR R
R R
R2 O ✻ O OH
R2 O R2 O alkylate
N O 185 °C
N O N O
Me base
Me Me ✻
R'
These rearrangements present many of the same issues which were encountered Br R'
R' R'
during our discussion of carbonyl addition with regrad to assymmetric induction.

β-Hydroxy ester enolates: Kurth, JOC 1985, 50, 5769


■ Chelate Control: β-Hydroxy ester enolates: Kurth, JOC 1985, 50, 1840
Me2HC Me2HC
Li Li Me2HC
OH O O O OH O O OH
2 LDA El(+) alkylate 185 °C
R R R N O N
Me O O N O
Me O Me O base
chelate control El Me
OH O Br
R'
Me
O Li OH O Me
O H Me
p. Me O 2 LDA, TMSCl
p. diastereoselection 87:13
H O Me OTMS
H
Me H Me
Me Me2HC Me2HC
Me
chair TS diastereoselection 81:19
N O N O
185 °C
We again see the consequence of chelate-organized asymmetric induction diastereoselection 94:6
H
57%
■ Felkin Control ?: Cha, Tet. Lett. 1984, 25, 5263
Me Me
Me Me Me
Me Me Me Me Me
O O O
O O O
OTMS OTMS OTMS The Claisen Rearrangement is subject to acid catalysis
O 25 °C
O 57:43 O LA + LA +
O LA O O LA O
Me Me
Me Me Me
Me
O OMe O OMe
O O OMe
OTMS O O BF3-HOAc: Bryusova, J. Gen. Chem. (USSR) 1941, 11, 722
OTMS OTMS BCl3: Gerrard, Proc. Chem. Soc. 1957, 19;
O 25 °C Schmid, Helv. Chem. Acta 1973. 56, 14
O 81:19 O Et2AlCl: Sonnenberg, J. Org. Chem. 1970, 35, 3166
THPO TiCl4: Mukaiyama, Chem. Lett. 1975, 35, 1041
THPO
OEt (RO)2AlMe: Yamamoto, JACS. 1988, 110, 7922
OEt
Me Me 75:25 (RO)2AlMe: Yamamoto, Tet. Lett. 1989, 30, 1265
110 °C
O O
(Felkin model, RO = large)
(RO)2AlMe: Yamamoto, JACS. 1990, 112, 316
Takano, Tet. Lett. 1985, 26, 865 LiClO4: Reetz, Tetrahedron. 1993, 49, 6025
15-15-Acyclic/Felkin 10/19/03 6:01 PM
D. A. Evans Lewis Acid Catalyzed Claisen Rearrangements Chem 206

Catalyzed Claisen Rearrangement of Allyl-phenyl Ethers Catalyzed Claisen Rearrangement of Allyl-vinyl Ethers
(RO)2AlMe: Yamamoto, Tet. Lett. 1990, 31, 377 (RO)2AlMe: Yamamoto, JACS. 1990, 112, 316

CMe3 Me3C ■ The thermal process


The Lewis Acid Br O O Br favored R O
(LA)
Al
H X R O X
CMe3 Me Me3C
R
O R O X R
OH X
Me disfavored
Me heat Me H
Me "high yield" O O X

Me
Me ■ The catalyzed process
LA
LA + OH disfavored R O
p. O
p. Me Al
R O H
Me Me H H
Me
+ LA R
R O H R
Me H
Me favored
A H
O O H
-78 °C Al
nonbonding interactions favor this TS
B
+ LA O
Me O R
Me
Me Claisen A:B = 3:4
Cope Me
R O H O H
Me
Me Me
OH 43% 19:81
O Me iBu O H

LA Claisen
40% 07:93
Me O H
Cope
85% Me
TMS
94% 03:97
The hindered Lewis acid will alter the partitioning of the Claisen process O H
to the two ortho positions

15-16-catalysis-1 10/19/03 6:02 PM


D. A. Evans Lewis Acid Catalyzed Claisen Rearrangements Chem 206

Chiral Lewis Acid Promoted Claisen Rearrangements Allylic rearrangements may be included as a subset of other
Yamamoto, JACS. 1990, 112, 7791 sigmatropic processes:
Yamamoto, Tet. Asymmetry 1991, 2, 647-662
Si(t-Bu)Ph2
R1 1 3 R2 R1 3 R2
1
O
X Y
Al Me Note that these reactions are
not catalytic
O R2 R2
R1 R3 [3, 3] R1 R3
Si(t-Bu)Ph2 X = C, N, O, S
(R)-1 X Y X Y
R
R
(R)-1 (1.1-2 equiv.)
R % ee ■ X = O; Y = S Faulkner, Synthesis 1971 175 (see pg 183)
Ph 88%
CH2Cl2, -20°C 71%
O SiMe3 C6H11 Me Me Me
76-95% O SiMe3
Et Et Et
Ph R pyr
p.
p. OH O S O S
(R)-1 (1.1-2 equiv.) R % ee Cl S
SiMe2Ph 90% OPh
93% OPh OPh
CH2Cl2, -40°C GeMe3
O R O R (E) selectivity: 96.5%
68-76%

Enantioselective Claisen Rearrangements: Metal-Centered Chirality ■ A stereochemically related case Johnson, JACS 1970 92, 735
Corey, JACS. 1991, 113, 4026
Ph Ph Me Me Me
CF3
R R R
SOCl2
ArO2S N N SO2Ar Ar =
B OH ether O Cl O Cl
(LA) S S
Br CF3 O
O (E) selectivity: >98%
BL2
O OH
Me ■ X = O; Y = N
L2B–Br O –20 °C Me
O R R
i-Pr2NEt Threo: erythro 99:1
200 °C
CH2Cl2 Me >97% ee (75%) Roberts, JACS 1960 82, 1950
Me O N O N
O Ph
Ph
Me H
O Hill, JOC 1968 33, 1111 H
BL2
O OH R R
Me R R
Me 200 °C H3O+
H
L2B–Br O –20 °C O Threo: erythro 10:90 O N O N
Me Ph O N N
96% ee (65%) Ph Ph
Et3N H Ph
Me Me O ONa
toluene ONa
8 cases reported
15-17-catalysis-2 10/19/03 6:02 PM
D. A. Evans Allylic Rearrangements via [3,3] Processes Chem 206

R2 R2 Me Me
Me Me
R1 R3 [3, 3] R1 R3 O OTBS O OTBS
O O
X = C, N, O, S 138 °C
X Y X Y Me Me
20 h
TBSO TBSO HN O
HN O
■ X = O; Y = N: 90% as a single isomer
CF3 CF3
steps
The Trichloroacetimidate Rearrangement, Overman, JACS 1974, 96, 597
Thomas, Chem. Commun 1989 717
R R R
Cl3C CN heat O OH O
+ O N
O H 3O O N CH2OH
M H H OH O HN HO OH
CO2H
CCl3 CCl3
OH NH2
RO N O N
This reaction is also catalyzed by Hg(II) ion H
OH NH2
C 3H 7 C 3H 7
C 3H7 Hg(OTFA)2 The polyoxins (antifungal antibiotics)
79% OH OH
HN O THF, 0 °C HN O
p.
p. OH
CCl3 (or 138 °C, 9 h) CCl3
■ The "Burgess Reagent" is normally used for alcohol dehydration
Me –O however, rearrangement is obsteved with allylic alcohols
R O +
Hg(OTFA)2 MeO N S NEt3
79%
Me Me OH HN O Me Et
THF, 0 °C O Me Et
Me Et
O
Me (or 138 °C, 6 h) CCl3 N O
OM MeO2C S N
O MeO2C H
For an excellent review see: Overman, Angew. Chem. Int. Ed. 1984, 23, 579 O OM
MeO N S + NEt3
NHBoc E. Burgess, JACS 1970, 92, 5224
NHBoc O

Me PdCl2(MeCN)2
Me 99% anti (45-62%) ■ A new approach to the synthesis of α-amino acids
HN O THF, 25 °C, 3h HN O
O OLi OLi
CCl3 CCl3 R Me R Me
Me N LDA R Me
Me N N
Me
Me Gonda, Synthesis 1993 729
O R'HN O N O N O
O O R' R'
O
heat O OLi OLi
BnO HN O
a 1:1 mixture was obtained +
BnO HN O H 3O
Endo, SynLett 1991 649
O
CCl3 CCl3 R Me
This may not be a concerted process N
Saksena, JOC 1986 51, 5024 H
N moderate yields
R' H

15-18-ALLYLIC rearr 10/19/03 6:03 PM


D. A. Evans Cycloaddition Reactions: Diels-Alder Reaction Chem 206

The Diels-Alder Cycloaddition Reactions


http://www.courses.fas.harvard.edu/~chem206/
"Diels-Alder Reactions". Evans, D. A.; Johnson J. S. In Comprehensive
Asymmetric Catalysis, Jacobsen, E. N.; Pfaltz, A.; and Yamamoto, H.
Editors; Springer Verlag: Heidelberg, 1999; Vol III, 1178-1235
Chemistry 206 (electronic handout)

Advanced Organic Chemistry The Diels-Alder Reaction in Total Synthesis, K. C. Nicolaou,


Angew Chem. Int. Ed. 2002, 41, 1668-1698 (electronic handout)

Catalytic Enantioselective Diels–Alder Reactions: Methods,


Lecture Number 16 Mechanistic Fundamentals, Pathways, and Applications, E. J.
Corey, Angew Chem. Int. Ed. 2002, 41, 1650-1667 (electronic
handout)
Cycloaddition Reactions-1
Chemistry and Biology of Biosynthetic Diels–Alder Reactions
■ Cycloadditions: Introduction Emily M. Stocking and Robert M. Williams, Angew Chem. Int. Ed.
■ Ketene Cycloadditions 2003, 42, 3078-3115 (electronic handout)
■ The Diels-Alder Reaction

Reading Assignment for week:

Carey & Sundberg: Part A; Chapter 11 Problem of the Day:


Concerted Pericyclic Reactions
Rationalize the sense of asymmetric induction for this Diels-Alder
Carey & Sundberg: Part B; Chapter 6 Reaction reported by MacMillan, JACS, 2000, 122, 4243. (pdf)
Cycloadditions, Unimolecular Rearrangements
O Me
Thermal Eliminations catalyst N
Me
Fleming: Chapter 4 PhCH2
Thermal Pericyclic Reactions N
H Me

X X
Wednesday, CHO CHO
D. A. Evans October 22, 2003 5% catalyst
+
R MeOH-H2O R
16-00-Cover Page 10/22/03 11:23 AM
D. A. Evans Cycloaddition Reactions-1 Chem 206

Why does maleic anhydride react easily with 1,3-butadiene, but not with ■ Consider [2 + 2] cycloaddition: Photochemical activation [ π2s + π2s]
ethylene? So what are the "rules"?

O O O new LUMO
π* π* HOMO C
O O X O C
light bonding
[4+2] [2+2]
O O O bonding
C ✻
■ The related reaction of 2 ethylenes is nonconcerted: [2 + 2] cycloaddition π π C
HOMO
Y Y Y
heat light ✻
• C C C C ✻
• C C C C
concerted + energy
X X X +
C C C C
■ We also know that the photochemical variant is concerted C C
p..
The frontier orbitals of the reacting species must have the proper symmetries
[2+2] Cycloaddition - Examples
■ Nomenclature
C C π2a
suprafacial π2s antarafacial hν
Quadricyclane Dauben, Tet. 1961, 15, 197.
C C
[π2s + π2s]

Using this nomenclature, the Diels-Alder reaction is a π4s + π2s cycloaddition Me Me


Me Me
Me
■ Consider [2 + 2] cycloaddition: Thermal activation [ π2s + π2s] Me hν
Me Me
Me [π2s + π2s] Schäfer, AC 1967, 79, 54.
Me
bonding Me
Me
C
Dewarbenzene-Derivative Prismane-Der.
π2s C C
antibonding π2s C π2a
bonding C C
C
[π2s + π2a]
π2s C bonding

[ π2s + π2a] "allowed" H


[ π2s + π2s] "forbidden" must be antarafical for indicated stereochem TL 1967, 4357, 4723.

16-01-Cycloaddition intro-1 10/22/03 6:15 AM


D. A. Evans Cycloaddition Reactions-2 Chem 206

Summary of Ketene Cycloadditions O O O


_
O
O
R R C O
O O H ∆
H
R' R'
or
R O O
O R
R' Ph Zn
R' Br _ ZnBr Ph2C C O
R R Ph Br 2
O O
R' R'
O O
550 °C
H 2C C O
Carbonyl Alkene O
O O
R1 O R1 C
C

R R' R3 or ∆ R3
1,3-Dipole Imine 2
p.. R R
O X O R2
O
Y R' R
X R' hν
R Z N C O
R = -CH=CH2 R or ∆
R' R'
Y N2
R O R
O ∆
C O
X O R' H -CH2CH2
H
R
Y
R'
FMO Analysis
Ketene Preparation
O HOMO
R
R R 3N
X C O Staudinger Reaction (very general)
R' O
R' [π2sπ2a]
H
X = Cl, Ts, AcO, DCC, etc... ∆
O
(CH3CO)2O

H 2C C O + AcOH H LUMO
_
_ O _ R
OH -ArO
RCH2CO2Ar R C O
_H O OAr
2

16-02-Cycloaddition intro-2 10/22/03 8:16 AM


D. A. Evans Cycloaddition Reactions-3 Chem 206

Ketene-Alkene [2+2]
[2+2]: Stepwise Versus Concerted
O Me O
H R H R R' O Me Me ∆ Me
C + Fast
H H
C C O C O Me Me Me Me
R'
R'
H R H R O Me O Me O
R R Me
least hindered C ∆ Me Me
+ +
bond rotation
Me Me Me Me Me Me Me
Stepwise Concerted B
1:2
• Very large polar effects
• E olefins yield a mixture of cis and trans • Ketenes add stereoselectively to Z H Me H Me
products Me
alkenes Me C Me C
• Solvent effects observed, but it could merely
be a ground state effect • Z olefins are much more reactive
+ Me Me O_ Me O_
• KIE seen for many reactions support stepwise
mechanism than E
Me C
• Calculations (Wang and Houk) show a highly C O C
p.. Me Me + H H + Me
asynchronus transition state in the gas phase
reaction
• All stereochemical outcomes can be Frey, H. M.; Isaacs, N. J. J. Chem Soc. B, 1970, 830-832.
rationalized assuming a stepwise mechanism
Ketenes + Aldehydes Afford β-Lactones
Solvent Effects
ab initio Calulations O
O O O
Me X
+ + H H
H H
Me X Me
C O O 38 kcal/mol
endo exo path A
X C
X Solvent endo / exo X Solvent endo / exo O
H H O
Cl hexane 4.3 / 1 Br hexane 0.71 / 1 +
Cl Et3N 2.2 / 1 Br Et3N 0.28 / 1 O 32 kcal/mol

Cl CHCl3 1.6 / 1 Br CH3CN 0.14 / 1 H H path B O


Cl CH3CN 0.59 / 1 O
H H
• Solvent effects implicate a zwitterionic intermediate Pons, J. -M.; et. al. H H
JACS 1997, 119, 3333.
Brady,et. al, JACS 1970, 92, 146-148.
Mechanism and Origin of Stereoselectivity in Lewis Acid Catalyzed [2 2]
Cycloadditions of Ketenes with Aldehydes, Singleton, Angew. Chem. Int. Ed. 2002,
41, 1572
16-03-Cycloaddition intro-3 10/22/03 11:29 AM
D. A. Evans Cycloaddition Reactions-4 Chem 206

Transformations of β-Lactones The Staudinger Reaction


In this process, the illustrated ketene, generated in situ from an acid chloride,
undergoes reaction with the indicated substrates to form β-lactams in a
R2 O R2 stereoselective process. When the azo-methine (RN=CHR) geometry in the
∆ or BF3
reactant is (Z) the product stereochemistry is trans (eq 1). In a complementary
O -CO2 fashion, the (E) imine affords the cis-substituted product (eq 2). While this
R1 R1 transformatlion could be viewed as a [2s+2a] cycloaddition, it is felt that this reaction
is stepwise.
O H H
_ H S R
Me2S +S S
O (1)
O R R H N N
O Et3N O
C
O Cl
O R' O H R H H
CuCN R R
R'Li (2eq) N (2)
R 2N O R 2N CO2H R N
O R
Vederas et al JACS 1987, 107, 4649.
The stepwise mechanism
p..
H H conrotatory H H
R2 O OH O R closure
R S
base S (1)
N
O or acid R1 Nu – + N
R H O O
R1 R2 H S
Nu = OH2, ROH, R2NH C enantiomers
N
O
H H H H
conrotatory R S
R
Application in Natural Product Synthesis: Ginkolide B, Ther are two contortaory modes. closure
If you control the conrotatory mode, S
E. J. CoreyJACS 1988, 110, 649. N N
MeO you control the absolute –
O + O
O stereochemistry of the reaction:
O
O O O
CMe3 1. (COCl)2, PhH, ∆ O H Ar
Et3N O O
CMe3 H H H H
2. NBu3, toluene, ∆ N N Ar Ar
N N
H Bn +
CO2H H Ph Ph N N
O O Cl Bn Ph
O O Bn

Evans, SjogrenTet. Lett. 1985, 26, 3783, 3787. diastereoselection > 95:5
NEt3 See also Evans, Williams, Tet. Lett. 1988, 29, 5065. 80-90% yields
H O
C
CH2COCl O "[2+2] photocycloaddition/fragmentation strategies for the synthesis of natural and unnatural
H products.", Winkler, J. D.; Bowen, C. M.; Liotta, F. Chem. Rev. 1995, 95, 2003.
JOC 1982, 47, 3470.
"Stereoselective intermolecular [2+2]-photocycloaddition reactions and their application in synthesis.",
Bach, T. Synthesis 1998, 683.
16-04-Cycloaddition intro-4 10/22/03 8:24 AM
D. A. Evans Cycloaddition Reactions-4 Chem 206

Enantioselective Ketene-Aldehyde Cycloaddiitons Me Me +


1) O O
O _
O O catalyst (10 mol%) O NN OTf
+ Cu
Me Br H R i-Pr2NEt Me3C H2O OH2 CMe3
R OTf
O O PhMe2Si O
R3N O C +
[RCHO • cat.] OEt 1 mol%, THF, 3Å MS
H
C -78 °C, 24 h O
Me3Si H O EtO2C
R3NH•Br CH2
77% yield, 93% ee
Bn
i-Pr i-Pr PhMe2Si O O
N KF, CH3CN
cat. = 5a: R = Me
N Al N 5b: R = Cl O O
F3CO2S SO2CF3 EtO2C EtO2C
R
p.. 3: >99% yield, 92% ee
catalyst % ee 3
entry Aldehyde 2 (R) [time (h), temp (°C)] % yield (configuration) 2+
Me
O
a 5b (8, -40) 91 92 (R) H
BnOCH2— Me Me3Si O
O N
b PhCH2CH2— 5a (16, -50) 93 92 (S) N Cu O R
RO observed product
OR2 R1 O
PhCH2CH2— 5a (72, -78) 89 95 (S) 2
H R 1 R O
O
c CH2CH(CH2)8— 5b (16, -50) 91 91 (S) H C C O

d Me2CHCH2— 5a (24, -50) 80 93 (S) Me3Si

e BnOCH2CH2— 5b (16, -40) 90 91 (S)


f TBDPSOCH2— 5b (16, -40) 74 89 (R)
Me Me 2+
g BnOCH2 5a (16, -50) 86 93 (R) O O
h Me3C 5a (16, -50) 91 85 (R) N N
Cu
i C6H11— 5b (24, -40) 56 54 (R) Me3C CMe3
H2 O OH2

+ 2 CF3SO3–
Nelson, S. G.; Peelen, T. J.; Wan, Z. JACS, 1999, 121, 9742-9743 X-ray

with J. Janey, Org. Lett. 2001, 3, 2125-2128

16-05-Cycloaddition intro-5 10/22/03 9:14 AM


D. A. Evans The Diels-Alder Reaction Chem 206

Articles and monographs of Significance


■ The Reaction:
Comprehensive Organic Synthesis, Vol. 5, Trost, Ed. 1991 ‡
4.1 Intermolecular Diels-Alder Reactions, W. Oppolzer
4.2 Heterodienophile Additions to Dienes, S. M. Weinreb +
4.3 Heterodiene Additions, D. L. Boger
4.4 Intramolecular Diels-Alder Reactions, W. R. Roush
4.5 Retrogade Diels-Alder Reactions, R. W. Sweger, A. W. Czarnik ■ Representative natural products displaying the Diels-Alder retron:
The Diels-Alder Reaction in Total Synthesis, K. C. Nicolaou, These natural products could well have incorporated the DA rxn into the
Angew Chem. Int. Ed. 2002, 41, 1668-1698 (handout) biosynthesis
HO O
Catalytic Enantioselective Diels–Alder Reactions: Methods, Mechanistic H
Me H
Fundamentals, Pathways, and Applications, E. J. Corey, Angew Chem. Int. O O
O NMe2 O
Ed. 2002, 41, 1650-1667 (handout)
H Et
Et O O
Hetero Diels-Alder Methodology in Organic Synthesis Me H
Boger, D.L. and Weinreb, S.N., Academic Press, 1987 Compactin: R = H Me Me
O H
Natural Products Synthesis Through Pericyclic Reactions Mevinolin: R = Me
p.. Desimoni, Tacconi, Barco, Polini, ACS Monograph 180, 1983, Chapter 5, R
O H H
Asymmetric Diels-Alder Reactions with Chiral Enoates as Dienophiles H
(Biosynthesis) JACS 1985, 107, 3694
Modern Synthetic Methods 1986, Scheffold, Ed. Springer-Verlag, Clive, JACS 1988, 110, 6914
Intramolecular Diels-Alder and Alder Ene Rxns, D. F. Taber, Springer-Verlag, 1984 H O Kozikowski, JOC 1987, 52, 3541
Lepicidin H Keck, JOC 1986, 51, 2487
H
Synthetic Aspects of D-A Cycloadditions with Heterodienophiles H O Me Grieco, JACS 1986, 108, 5908
Weinreb, Tetrahedron, 1982, 38, 3087-3128 H OMe
(Synthesis) Heathcock, JACS 1985, 107, 3731
MeO
The Intramolecular DA Rxn: recent advances and synthetic applications JACS, 1993, 115, 4497 OMe H Girotra, Tet. Let. 1983, 24, 3687
Fallis, Can. J. Chem., 1984, 62, 183-234 Hirama, JACS 1982, 104, 4251
Intramolecular [4 +2] & [3 + 2] Cycloadditions in Organic Synthesis
Oppolzer, Angew. Chem. Int. Ed., 1977, 16, 10-23
HN HO2C
H
Preparation & DA Reactions of Heterosubstituted 1, 3-Dienes Me O
Petrzilka, Synthesis, 1981, 753-786 H H
H H Ph
DA Reactions of Azadienes Me
Boger, Tetrahedron, 1983, 39, 2869-2939 O
H H
COOH Et
Silyloxydienes in Organic Synthesis Ph Endiandric Acid C
Danishefsky, Acct. Chem. Res., 1981, 14, 400-406 X-14547A H Et H

DA Reactions Part I: New Preparative Aspects Roush JOC 1984, 49, 3429 H
Sauer, Angew. Chem. Int. Ed., 1966, 5, 211-230 H
Nicolaou JOC 1985, 50, 1440
Ley Chem. Commun. 1983, 630 H H
DA Reactions Part II: The Reaction Mechanism
Sauer, Angew. Chem. Int. Ed., 1967, 6, 16-33 CO2H
H
Mechanistic Aspects of Diels-Alder Reactions: A Critical Survey
Endiandric Acid B
Sauer, Angew. Chem. Int. Ed., 1980, 19, 779-807
(Syntheses)
Nicolaou, JACS 1982, 104, 5555-5562
16-06-Diels-Alder intro 10/22/03 6:47 AM
D. A. Evans Diels-Alder Reaction-Orbital Symmetry Considerations Chem 206

The Alder Endo Rule The following observation illustrates an example of the Orbital Symmetry Considerations for Diels Alder Reaction
Alder Rule which will be defined below.
If the symmetries of the frontier MO's of reacting partners are "properly matched" the
reaction is referred to as "symmetry-allowed". The Diels-Alder reaction is such a case.
As illustrated, the HOMO and LUMO of both the diene and dienophile, which in this case
H are the same, will constructively overlap as indicated in formation of both sigma bonds.
+
disfavored favored H
H
H
"Exo product" C C HOMO-π2 C C LUMO-π3
"Endo product"
Observation: The endo Diels-Alder adduct is formed faster even though the exo C C C C
product is more stable. There is thus some special stabilization in the transition state
leading to the endo product which is lacking the exo transition state.

C LUMO-π3 C HOMO-π2
Exo TS ‡ C C
C C
Endo TS ‡ C C
p..

■ Primary orbital overlap leads directly to the formation of new chemical bonds.
Energy

Frontier MO Explanation for the Endo Rule


2 H C C LUMO-π3
H ■ Secondary (transient) orbital overlap can also occcur in the C C
stabilization of certain transition state geometries. Such a
H transient stabilizing interaction can occur in the endo, but
H not exo, transition state: C HOMO-π2
C C
■ Of the two possible transition states, the one having the "greatest accumulation C
of interacting double bonds will be preferred" (the Alder Endo Rule).
Secondary orbital overlap is noted below. The Other Dimerization Possibility for Butadiene

Does the possibility for the following ∆


concerted dimerization exist?

Secondary
orbital overlap ■ Note that the termini only match at one end for the C C HOMO-π2
HOMO-LUMO pairing. Hence we say that the symmetry
C C
requirements for the reaction in question are not met.
This does not mean that the reaction will not occur,
only that the reaction will not be concerted. Such reactions
are called "symmetry-forbidden". C C LUMO-π3
C
Exo TS ‡ Endo TS ‡ Additional Reading: Lowry & Richardson,
C
Chapter 10, theory of Pericyclic Rxns pp 839-900
16-07-Diels-Alder intro-2 10/22/03 6:48 AM
D. A. Evans Diels-Alder Reaction: The Transition Structure Chem 206

Transition State Modelling is Coming of Age ■ Lewis Acid Catalysis of the reaction is possible: Yates & Eaton,
JACS 1960, 82, 4436

+ LUMO2
LUMO1
The Critical Energy

energy
Difference:
■ The lengths of the forming C–C bonds are Ca. 1.5 times the normal bond E(LUMO1) - E(HOMO2)
distance. This factor comes out of the ab initio work of Jorgensen & Houk HOMO1 or
Jorgensen, JACS 1993, 115, 2936-2942 E(LUMO2) - E(HOMO1)
leading references: HOMO2
Houk, Jorgensen, JACS 1989, 111, 9172
Transition Structures of Hydrocarbon Pericyclic Reactions Diene Dienophile
Houk Angew. chem. Int. Ed. 1992, 31, 682-708
■ The closer the two orbitals are in energy, the better they interact
■ Bond formation is not synchronus with substituted dienophiles (Jorgensen) ■ As ∆E decreases for the relevant ground state FMOs, rxn rates increase

p..
Ethylene & Butadiene Vs Butadiene & Acrolein
O O

+ + H H

2.193 Å 2.193 Å
2.091 Å 2.325 Å

LUMO2
LUMO1

LUMO3
∆H‡ = 22.5 kcal/mol ∆H‡ = 14.0 kcal/mol E
exptl
∆S‡ = -32.1 eu ∆S‡ = -38.3 eu
HOMO1
rel rate = 1 rel rate = 10+5 HOMO2

■ Diene Reactivity as measured against Maleic anhydride HOMO3


Me
Me ∆E (LUMO3-HOMO1) < ∆E (LUMO2-HOMO1) Rate Acceleration

log k = 4.96 log k = 2.36 log k = 2.19 log k = 2.12 log k = 1.83 Lewis acid catalysis not only dramatically increases rates by ca 10+6
it also improves reaction regiochemistry & endo diastereoselectivity
Sauer, Angew. Chem. Int. Ed., 1980, 19, 779-807

16-08-DA transtion structure 10/22/03 6:52 AM


D. A. Evans Diels-Alder Reaction: Regiochemistry Chem 206

Orientation of Reacting Partners Here is an interesting problem in reaction design


CO2H CO2H CO2H
COX COX RO COX
CO2H CO2H
RO RO
CO2H
favored disfavored
favored disfavored
However, what if you need the disfavored product?
4.5 : 01 @ 100 °C Ni(Raney)

Lewis acid catalysis improves orientation PhS PhS COMe PhS


MgBr2

COX COX Me AcO COMe AcO


COX AcO COMe

Me Me Trost, JACS 1980, 102, 3554 disfavored favored


p.. favored disfavored
By employing a removable substituent, it is possible to access the normally
toluene, 120 °C 59 : 41 disfavored product diastereomer
C6H6, SnCl4, 25 °C 96 : 04
O2N NO2
In general, 1-substituted dienes are more regioselective than their
2-substituted counterparts: Sauer, Angew. Chem. Int. Ed., 1967, 6, 16-33 RO CO2Me RO CO2Me
Danishefsky, JACS 1978, 100, 2918: The NO2 FG completely dominates directivity
Lewis acid catalysis improves endo diastereoselection
NO2
It then can be base
removed by elimination –NO2–
RO CO2Me RO CO2Me
H CO2Me
CO2Me
CO2Me H NO2
or by reduction R3SnH
favored disfavored Ono, Tet. 1985, 4013
RO CO2Me RO CO2Me
CH2Cl2, 0 °C 80 : 20
O O O
C6H6, SnCl4, 25 °C 95 : 05 H H
83% R3SnH
DA Reactions Part II: The Reaction Mechanism, 86%
Sauer, Angew. Chem. Int. Ed., 1967, 6, 16-33 NO2
Me O2N H
Me Me
Ono, Chem. Commun. 1982, 33-34 mixture of ring-fusion
16-09-DA regiochem 10/22/03 6:58 AM isomers
D. A. Evans Diels-Alder Reaction: Regiochemistry Chem 206

MeOCH2 H H CH2OMe
Instructive Issues of Regiocontrol with Quinone Dienophiles
CH2OMe Cu(BF4)2
O O Me O Cl Cl
Me Me 0 °C
Me Cl CN
Me
CN CN
Corey, JACS 1969, 91, 5675 Ratio: 90 : 10
MeO MeO MeO
H H
O O O Me
Diels-Alder Reactions with Chiral Dienes
Orientation of Reacting Partners Conditions Ratio
controlled by Lewis acid structure thermal (100 °) 50 : 50 Comprehensive Organic Synthesis, Vol. 5, Trost, Ed. 1991
Reusch JOC 1980, 45, 5013 BF3•OEt2 (-20 °) 4.1 Intermolecular Diels-Alder Reactions, W. Oppolzer, See page 347
80 : 20
SnCl4 (-20 °) <5 : 95
F 3B O
O O H O
Me H
Me O Me X
Me 25-50 °C
N–Ph Ph–N Ph–N
Me δ+
O selection 80 : 20
p.. O O H H
MeO O
O H
O X X
O
X= Ratio
δ+ Me O
Me –OH 36 : 64
Me Overman, JACS 1988, 110, 4625 –Me 83 : 17
O selection >95 :5
–OMe >97 : 3
Sn O MeO
Me H
Cl4 O Me
Me OR Me OR Me OR
Similar results provided by Stoodley Chem. Comm. 1982, 929 O O O

25-50 °C
N–Ph N–Ph N–Ph
Kelly Tet. Let. 1978, 4311
OMe O OMe O
Me Me O Me O
BF3•OEt2
RO
selection >95 :5 Franck, Tet. Lett. 1985, 26, 3187 R = Me: Ratio; 83 : 17
O 0.4 equiv
Me Me Franck, JACS 1988,110, 3257 R = Me3Si: Ratio; 88 : 12
RO
OH O Me Me
Me RO
O Comments on the Transition State H
OR H
OH O RO Me
MgI2 ■ Avoid Eclipsing allylic substituents
better than Me O
0.5 equiv ■ better donor (Me) anti to forming bond Me O
OMe selection >95 :5 PhN PhN
■ avoid gauche OR interaction
OH O OMe
O O
16-10-DA regiochem-2 10/22/03 7:01 AM
D. A. Evans Cycloaddition Reactions: Diels-Alder Reaction Chem 206

The Diels-Alder Cycloaddition Reactions


http://www.courses.fas.harvard.edu/~chem206/
"Diels-Alder Reactions". Evans, D. A.; Johnson J. S. In
Comprehensive Asymmetric Catalysis, Jacobsen, E. N.; Pfaltz, A.;
Chemistry 206 and Yamamoto, H. Editors; Springer Verlag: Heidelberg, 1999; Vol
III, 1178-1235 (pdf)
Advanced Organic Chemistry "Chiral Bis(oxazoline) Copper (II) Complexes: Versatile Catalysts
for Enantioselective Cycloaddition, Adol, Michael and Carbonyl Ene
Reactions". Johnson, J. S.; Evans, D. A. Acc. Chem. Res. 2000,
Lecture Number 17 33, 325-335. (pdf)

"New Strategies for Organic Catalysis: The first Highly Selective


Organocatalytic Diels-alder Reaction", MacMillan, JACS, 2000,
Cycloaddition Reactions-2 122, 4243. (pdf)
"New Strategies for Organic Catalysis: The first Enantioselective
■ The Diels-Alder Reaction
Organocatalytic 1,3-Dipolar Cycloaddition", MacMillan, JACS, 2000,
122, 9874. (pdf)
■ Reading Assignment for week:
Carey & Sundberg: Part A; Chapter 11
Concerted Pericyclic Reactions ■ Problem of the Day:

Carey & Sundberg: Part B; Chapter 6 O


H
Me Me
Cycloadditions, Unimolecular Rearrangements OMe
Me O toluene, ∆ O
Thermal Eliminations Me O 97% yield
Fleming: Chapter 4 Me
Thermal Pericyclic Reactions OMe Me

Me
Fallis J. Org. Chem. 1993, 58, 2186. H Me

Friday,
D. A. Evans October 24, 2003
Me (+)-Longifolene
17-00-Cover Page 10/23/03 3:34 PM
D. A. Evans Chiral Auxiliary Controlled Diels-Alder Reactions Chem 206

Review: Oppolzer in Comprehensive Organic Synthesis 1992,Vol. 4, 315-399. ■ Non-Chelate Ester-Type Chiral Auxiliaries

LA
■ Ester-Type Chiral Auxiliaries Corey JACS 1975, 97, 6908. O H
S-trans geometry
O RL
Me BnO RS
BnO
O Ph H Me
R
H AlCl3
O +
H
89% Ph rxn from this face
Me
O O
Lewis Acid-Ester Complex Conformation Dictates Diastereoselection
LA
Diastereomer Ratio Not Given O H O H O H
LA
O RL RL
R O R O RL
RS RS RS
R
p..
LA
Chem 3D model
R O H
front Face
AlCl3 O RL
H RS

O O RL R
RS
front Face
See Oppolzer ACIEE 1984, 23, 876.

Me

Me Cl
O
Cl
Cl Cl
Ti Ti
Chem 3D model Cl Cl
Cl
O
Cl Me
diene approach
O

Me X-ray

17-01-Aux-control/DA 10/23/03 4:12 PM


D. A. Evans Lewis Acid–Carbonyl Complexes-3 Chem 206

Representative η1–titanium complexes with organic compounds Representative η1–titanium complexes with organic compounds

Me

O
X-ray structure

Me X-ray structure
O Cl
Cl
Cl Cl Ti
Ti Cl Cl
Cl Cl Cl
Cl Ti Cl
Cl O
Me O O

O OEt
O
Me
Me
p..

X-ray structure

R R
O O
Side view of in-plane Ti
coordination: Ti-O-C-C = 2.86° Ph Ph
Ph Ph
O O
Ti–O–C angle = 152 ° Ti
O O

Ph N O

Cl Cl
Cl Ti Cl
O O

Me O Me
Narasaka JACS, 1989, 111, 5340
A. Jorgensen, JACS, 1995, 117, 4435 X-ray structure

17-02-LA-C=O complexes-3 10/23/03 4:11 PM


D. A. Evans Chiral Auxiliary Controlled Diels-Alder Reactions-2 Chem 206

Ester-Type Chiral Auxiliaries: Chelating Dienophiles Chelating Imide-type Chiral Auxiliaries Evans JACS, 1984, 106, 4261.
Metal ion Dependent Diastereoselection Evans JACS, 1988, 110, 1238.
EtO
Me O O O
TiCl4 Me Me Exo-1
0.7 equiv TiCl4 O R Me N O H H +
O M+ H H Exo-2
Entry A C C
Me2CH
O OR* O XV O XV

R Endo-1 Endo-2
Me O Lewis Acid Endo-1
R Entry Temp Endo/Exo Endo dr
EtO (1.2 equiv) Temp. Endo-2 Σ Endo / Exo
O H A –63 °C 39:1 93:7
O H B –63 °C 39:1 22:78 SnCl4 25 2.7 92 : 8
R
AlEtCl2 SnCl4 –78 93 : 7
Me O 3.1
R
Entry B EtO
p.. O TiCl4 –78 2.6 91 : 9
2.5 equiv
EtAlCl2 O
Cl2EtAl R O OR* AlCl3 –78 1.5 80 : 20

(2.0 equiv) Et2AlCl –78 15.7 >99 : 1


Helmchen Tetrahedron Lett. 1984, 25, 2191.
ACIEE 1985, 24, 112. (0.7 equiv)
Tetrahedron Lett. 1985, 26, 3095. Et2AlCl –78 4.4 92 : 8
Binding Mode Dictates Diastereoselectivity
Me Me
O O 1 equiv Al 1 Point Binding
Cl
+
Me2AlCl O R C 5H 6
Cl R N O
Cl
N Endo/Exo = 4 :1
Cl Ti Cl R Rel. Rate = 1
O
O O R
O

OEt Me Me
O O O Me2AlCl2
> 1 equiv Al 2 Point Binding
Me2AlCl O + O C 5H 6
Me R N O

X-ray structure R N O Endo/Exo = >20:1


R Rel. Rate = 100
Why Imide Dienophiles?? R
They are ~ 100-1000X more reactive
R = CH2Ph much more stereoselective than R = CHMe2
17-03-Aux-control/DA-2 10/23/03 4:11 PM
D. A. Evans Chiral Auxiliary Controlled Diels-Alder Reactions-2 Chem 206

A Case for Π-Stacking: Angew Chem, Int Ed. 1987, 26, 1184
The Chiral Dienophile
Compare the alkylation rxn which is dominated by steric effects with the DA rxn
which may be controlled by both steric and electronic effects
Model
Et Et Me2AlCl2 Me Me
Al Me Al
O O O O Me O O O + O Al
Et2AlCl
*
Me N O Me N O N O
–30˚ CH3 N O

R R Me R
PhCH2
°
O O O Li O O O ~3A
Et Et Et *
N O LDA N O Me–I N O
–30˚ Me A Complex Application
R R R
OTIPS OTIPS
∆∆ G‡ = 2.3 RT Log P1/P2
p..
PLOT ∆∆G‡ FOR EACH RXN AS A FUNCTION OF THE SUBST., R. Et OTES Et OTES
Me Me
20 H H
CH2Ph O O O O
CH2c-C6H11 Me2AlCl, 0 °C O O
O O N 71% H
O
N O
15 Bn H H
CH2p-MeOPh
CH2p-ClPh diastereoselection Bn
H
CH2p-CF3Ph 10:1
A1 Et
Alkylation, 10 OTBS
A2 i-C3H7 OTBS

Me The control experiment with no chiral auxiliary:

5 OTIPS OTIPS
Ph
Et OTES Et OTES
Me Me
H Me2AlCl, 0 °C H
0 O O O O
O O
0 5 10 15 20 25 30 74% H
O O N O
D1 H N O
Conclusions: Diels-Alder, H
D2 diastereoselection H
Steric effects correlate well for the two reactions 6:1
Added electronic effects from Bn group enhance facial bias OTBS
OTBS
Evans, BlackLepicidin Synthesis, JACS, 1993, 115, 4497
17-04-Aux-control/DA-3 10/23/03 4:36 PM
D. A. Evans Diels-Alder Reaction: Intramolecular Reactions Chem 206

Type I intramolecular Diels-Alder Reaction: A Type I Intramolecular Diels-Alder:


CO2H CO2R R3SiO

Me Me
Me

O O
O
n n n O O
HO RO t-Bu
O O
Type II intramolecular Diels-Alder Reaction: O O O OH
OH O
OR

n H Me H Me
n n H Me

H H
OH OR H
SiR3 OR

2 Diels-Alder Retrons
(-)- Chlorothricolide
45% yield

R3SiO R3SiO
A Type II Intramolecular Diels-Alder:
O O
Me Me
O O H
O H H C 5H 9 O
O
O O
O O Me t-Bu
O
O O C8H15
O O OR O t-Bu
Me O O
O
H
toluene, 120 °C Me
O CO2H Me
CO2H
(–)-CP-263,114
Me
Me
O RO2C
Me Me
O SiR3 OR
C 5H 9 SiR3 O OMe
O O O ZnCl2 O O
O O H
CH2Cl2 Roush, Sciotti J. Am. Chem. Soc. 1998, 120, 7411-7419.
C 5H 9 X
O N EtS C8H15
EtS C8H15 Endo T.S.
Bn
approx. 60% yield MeO C CO Me Some Intramolecular Diels-Alder Reviews:
MeO2C CO2Me 2 2

Fukuyama et al JACS 2000, 122, 7825-7826. Shea Angew. Chem. Int. Ed. 2001, 40, 820.
The concept: Evans, et al Angew. Chem. Int. Engl. 1997, 36, 2119-2121. Fallis Acc. Chem. Res. 1999, 32, 464-474.

17-05- Intramolecular DA 10/23/03 4:41 PM


D. A. Evans The Diels-Alder Reaction: Enantioselective Catalysis Chem 206

Articles and monographs of Significance The conformation of the dienophile is also an issue
"Diels-Alder Reactions". Evans, D. A.; Johnson J. S. In Comprehensive
Asymmetric Catalysis, Jacobsen, E. N.; Pfaltz, A.; and Yamamoto, H. Editors; The S-cis versus S-trans dienophile conformation is coupled to the geometry
Springer Verlag: Heidelberg, 1999; Vol III, 1178-1235. of the Lewis acid-dienophile complex & both issues determine face selection
L M
Review: Kagan, H. B.; Riant, O. Chem. Rev. 1992, 92, 1007-1019 M L
M R' O
S O re-face M
Comprehensive Organic Synthesis, Vol. 5, Trost, Ed. 1991 re-face O S
X
4.1 Intermolecular Diels-Alder Reactions, W. Oppolzer X R
4.2 Heterodienophile Additions to Dienes, S. M. Weinreb R X
(E) s-cis
4.3 Heterodiene Additions, D. L. Boger
M L L M R
4.4 Intramolecular Diels-Alder Reactions, W. R. Roush R' O
4.5 Retrogade Diels-Alder Reactions, R. W. Sweger, A. W. Czarnik M M
O S S O
si-face X si-face
Catalytic Asymmetric Synthesis, I. Ojima, Ed. 1993 X R X
Chapter 9, Asymmetric Rxns with Chiral Lewis Acid Catalysts (Z) s-cis R
R
Theoretical Studies on Conformations of Acrolein & Methyl
Chiral Lewis Acids in Catalytic Asymmetric Reactions
p.. Narasaka, Synthesis, 1991, 1-11 Acrylate & their Lewis Acid Complexes
Houk, JACS, 1987, 109, 14-23
(Carbonyl-Lewis Acid Complexes)
Schreiber, Angew. Chem. Int. Ed., 1990, 29, 256-272 Stereoelectronic Effects (?) in Lewis acid-C=O Complexes
Rotational barriers in Aldehydes & Ketones Coordinated to Neutral Lewis Acids
Wiberg, JACS, 1988, 110, 6642 Y Z Let X be the most electronegative ligand in the Lewis acid

Theoretical Studies on Conformations of Acrolein, Acrylic Acid, Methyl Acrylate M The stabilizing hyperconjugative interaction between the
& their Lewis Acid Complexes Houk, JACS, 1987, 109, 14-23 X O O-lone pair and σ* M–X will provide a stabilizing interaction
C2 Symmetry and Asymmetric Induction, Whitesell, Chem. Rev., 1989, 89, 1581-1590 for the illustrated conformation.
H R J. M. Goodman, Tet. Lett. 1992, 33, 7219
The Design of Enantioselective Diels-Alder Catalysts
R' O However, there is no evidence for this orienting effect in this X-ray structure
reported by Reetz, JACS, 1986, 108, 2405
X F

F 3B
O R O+

X R chiral Lewis acid H B


catalyst
R' O
R'
X
However pi-bonding to coordinated C=O-Al complexes has been reported
R Barron, JACS, 1990, 112, 3446, JACS, 1990, 112, 2950

Theory predicts a small rotational barrier about B–O bond: Wiberg JACS,
1988, 110, 6642
17-06-DA enantioselect-1 10/23/03 4:48 PM
D. A. Evans The Diels-Alder Reaction: Enantioselective Catalysis-2 Chem 206

Boron-Based Catalysts: Hawkins JACS 1991, 113, 7794 Titanium-Based Catalysts: Narasaka JACS 1989, 111, 5340.

R Temp (°C) Endo ee Ph Ph


10 mol % O
H -78 97 % Ph O
H H TiCl2
O BCl2 Me -20 91 % Me O
O O O R
CO2Me -78 92 % Ph Ph
OMe
R N O + O
R Endo/Exo Ratios not provided. 10 mol %
R N O
re-face O
CO2Me
R Temp (°C) Endo/Exo Endo ee
H -40 96:4 64 %
reported X-ray similar to this model
Me 0 92:8 91 %
Ph 25 88:12 64 %

10 mol % Ph Ph
p..
re-face O
Ph O CO2Me
R TiCl2
O O O O
CO2Me Me O
MeO2C N O + Ph Ph N O
O
0 °C
94% ee

R R
O O
Ph Ph
R Ph Ph
O O
si-face Ti
O O
CO2Me
Ph N O

X-ray

Chem 3D Models
A. Jorgensen, JACS, 1995, 117, 4435

17-07-DA enantioselect-2 10/23/03 4:49 PM


D. A. Evans The Diels-Alder Reaction: Enantioselective Catalysis-2 Chem 206

Mg(2+)-Based Catalysts: Corey Tetrahedron Lett. 1992, 33, 6807.


10 mol% Me Me O O
O O Me O O Me
H Me N O
Me N N Me
N O + Mg O
Ph Ph
I I
O N O
-78 °C
Stereochemical Model: Endo/Exo = 98:2
Endo ee = 91%

X-ray structure PM3 model

Mg Tetrahedral metal O O R
geometry
p.. H
R N O 5 mol% cat. H
25 °C
H O X
O
PM3 model R time endo ee yield
Re-face
O N O R = Me 8h 96% ee 98%
R = Ph 8h 96% ee 95%
R = Cl 8h 94% ee 95%
Limitations: Scope limited to illustrated reaction
O O R O O
Evans, Miller, Lectka JACS 1993, 115, 6460. R
Cu(2+)-Based Catalysts: Angew. Chem. Int. Engl. 1995, 34, 798-800. N O N O
2-5 mol% cat.
JACS 1999, 121, 7559-7573. CH2Cl2
JACS 1999, 121, 7582-7594
R temp endo ee yield
2+
Me Me
R = Me 25 °C 94% ee 89%
O O
R = Ph -20 °C 97% ee 95%
O O N N + 2 SbF6–
R = OAc 0 °C 97% ee 100%
Cu R
R N O Me3C CMe3
H2 O OH2 O O
+
0 °C N O
O X 10 mol% cat.
CH2Cl2 +25 °C
93% ee, 90% yield O X

17-08-DA enantioselect-3 10/23/03 4:50 PM


D. A. Evans Chiral Cu(2+) Complexes as Chiral Lewis Acids Chem 206

Cu(box) and Cu(pybox) catalyst-substrate complexes implicated in enantioselective reactions.


"Chiral Bis(oxazoline) Copper (II) Complexes: Versatile Catalysts for Enantioselective Cycloaddition, Adol,
Michael and Carbonyl Ene Reactions". Johnson, Evans, Acc. Chem. Res. 2000, 33, 325-335. (pdf)

Me Me 2+ Me Me 2+ Me Me 2+ 2+
O O O O O O
2X – 2 X– 2 X– O
N
O 2 X–
N N N N N N
Cu Cu Cu N Cu N
Me3C CMe3 Me3C O O CMe3 Me3C CMe3
O O O O
Ph O OBn Ph
N
R N O RO2C N N O MeO OMe
H
R
Cycloaddition Reactions
Enol Amination Reactons Michael Reactions Aldol Reactions
Michael Reactions
p..

Me Me 2+ Me Me 2+ Me Me 2+ 2+
O O O O O O
2 X– 2X –
2 X– O O 2 X–
N N N N N N N
Cu Cu Cu N Cu N
Me3C O O CMe3 Me3C O O CMe3 Me3C O O CMe3
Bn X O Bn
P OR'
OR' R OR'
OR'
R R R
R = H, Cycloaddition Reactions
Hetero Diels-Alder Reactions Ene Reactions
Diels-Alder Reactions
R = Alkyl, Cycloaddition Reactions
Aldol Reactions

17-09-DA enantioselect-4 10/23/03 4:51 PM


D. A. Evans Temperature–Enantioselection Profile for [Cu((S,S)-t-Bu-box)](X)2 Catalysts Chem 206

Me Me 2+ Me Me 2+ O O
O O O O (A) cat. 1b
H O
– a: X = OTf N O
2X 2 X– C5 H6
N N b: X = SbF6 N N
Cu Cu O N O
Me3C CMe3 Me3C H O -78 °C: >98% ee
2 OH2 CMe3 25 °C: 94% ee
1 2
OTMS
100 Me
N O CO2Et O O
(B) cat. 1b
(D) O O
(CF3)2CHOH Pyr N O
98
EtO2C N O Me
-78 °C: 98% ee
(E) -20 °C: 94% ee
96
(B) (A)
p..
O
enantiomeric excess (%)

MeO Me OH O
94 Me cat. 1a
(C) OTMS MeO
(C) + StBu
O
StBu -78 °C: 99% ee O
92 25 °C: 92% ee

(A) Ph
O
90 (B) OEt cat. 2a
(D) Ph
(C) O 3Å mol. sieves
EtO2C O OEt
88 (D) OEt -40 °C: 99% ee
25 °C: 94% ee
(E)

86
OTMS

N O Troc O O
(E) cat. 2a
84 N
O O
-80 -70 -60 -50 -40 -30 -20 -10 0 10 20 30 CF3CH2OH Pyr N N O
N H
Troc N N O
temperature (°C) -78 °C: 99% ee
"Chiral Bis(oxazoline) Copper (II) Complexes: Versatile Catalysts for Enantioselective Cycloaddition, Adol, Michael 25 °C: 96% ee
and Carbonyl Ene Reactions". Johnson, Evans, Acc. Chem. Res. 2000, 33, 325-335. (electronic pdf)

17-10-DA enantioselect-5 10/22/03 4:31 PM


D. A. Evans Diels-Alder Reaction: Molecular Orbital Analysis Chem 206
Ethylene & Butadiene Vs Butadiene & Acrolein "Inverse-Electron Demand" Hetero-Diels-Alder Reactions
O O
RO O X RO O X OR OR
+ + H H
+ +

LUMO LUMO
LUMO2
LUMO1
LUMO3
LUMO
E LUMO
E
HOMO1
HOMO HOMO
HOMO2 HOMO
HOMO
HOMO3

∆E (LUMO3-HOMO1) < ∆E (LUMO2-HOMO1) Rate Acceleration

"Inverse-Electron Demand" Diels-Alder Reactions ■ Lewis Acid Catalysis of the reaction is possible:

CO2R
CO2R LA O
RO
RO XOC O OR RO X
O O OR XOC O OR
+ + X
+ +

LUMO LUMO LUMO


LUMO

LUMO
LUMO LUMO
E LUMO
E

HOMO HOMO HOMO HOMO


HOMO
HOMO HOMO
HOMO

17-11-IED-diels alder 10/22/01 8:08 AM


D. A. Evans Hetero-Diels-Alder Reactions – Chiral Catalysis Chem 206

•Heteroatom-substituted reactions are also possible: Chiral Copper Lewis Acids:



O O O
+ Me Me 2+ Me Me 2+
O O O O
2 X–
A: X = OTf
2 X–
N N B: X = SbF6 N N
‡ Cu Cu
O Me3C CMe3 Me3C H O OH CMe3
O O 2 2
+ 1 2

OMe
O O 10 mol% 2B OSiR3
Ph O N O
OSiR3 10 mol% 1A O
EtO2C N O OSiR3 O
Me R3SiO CO2Et
p.. Me Me OMe O
Ph CO2Et
H Me
OEt
O
Endo T.S.
O CO2EtO O OSiR3
Ph O N O
(CF3)2CHOH OMe
Ph N O O
Me Me >99:1 Endo:Exo O O
CO2Et 96% yield CF3CO2H
Evans, Scheidt, Johnston, Willis J. Am. Chem. Soc. 2001, 123, 4480. 99% ee O CO2Et R3SiO CO2Et
Me Me

Me Me Jorgensen J. Am. Chem. Soc. 1998, 120, 8599.


24:1 Endo:Exo
+ 2 mol% 2A 87% yield

OEt EtO2C O OEt 97% ee C4H10OTBS


EtO2C O
C4H10OTBS
5 mol% 1B H
Me Me O O N O
H 16:1 Endo:Exo
2 mol% 2A CH2Cl2 H
+ 96% yield N O O O
O 81% yield
EtO2C O EtO2C O O 97%ee
H 96% ee, 99:1 dr

Evans, Johnson, Olhava J. Am. Chem. Soc. 2000, 122, 1635. Evans, Johnson J. Org. Chem. 1997, 62, 786-787.

17-12-Hetero DA 10/23/03 4:51 PM


D. A. Evans Diels-Alder Reactions: Synthesis of FR182877 Chem 206

Structures of FR182877 & Hexacyclinic acid The Transannular Diels-Alder Step

HO AcO H TBSO CO2Et


H 8 Me Structural Differences: H 8
Me TBSO CO2Et
Acylation at C8 hydroxyl
Ph2Se2O3
TBSO 18 Me
11 11 TBSO Me Me O
Me OH Oxidation at C11 HO2C Me O R3N, RT
OH
H H H H
63% 19 OTBS
2 O H O OTBS
endo vs. exo Diels-Alder retron
H O H O 17
17 Br
Me O H Me OH Br Me
Hydration across O H Me
C2-C17 double bond
Me Me Diels-Alder
50 °C
cycloaddn
(—)-FR182877 Hexacyclinic Acid
Sato, B. J. Antibiot. 2000, 123, 204, 615. TBSO
Hofs, R., et al. Angew. Chem. Int. Ed. 2000, 39, 3258.
Corrected Structure: J. Antibiot. 2002, C-1. H Me Me OMe
Hetero-DA H CO2Me
Hypothesis: A Transannular Diels-Alder Cycloaddition Cascade Br MeO H Me
p.. OTBS Cycloaddiiton
H H H OMe
TBSO CO2Et CO2Et 63% O
H
TBSO Br
TBSO Me H
Me O Me O Me
OTBS Me
1. Diels-Alder cycloaddn
2. Hetero-DA cycloaddiiton C18 & C19 stereocenters exert complete control over the first cycloaddition (Evans)

HO Br
Me
H Me -120
A H CO2R
11 RO OR H H H
X B Me OR MeO 6 CO2Me -130
OH H 2
H H H Me
C H MeO 8 18 Me

kcal/mol (PM3)
O
H
O Me O -140 endo-I
DO
Br endo-II
Me Me 19 OMe -150
O H

Me endo-TS 11
-160
Br
FR182877 Me
-170
Evans & Starr, JACS, 2003, 125, ASAP
(natural configuration)
Sorensen et al, JACS, 2003, 125, 5393 -180
2.1 2.3 2.5 2.7 2.9
∆∆E‡ = 5–9 kcal/mol @ 2.1 Å
π-Face Separation (Å)
17-13-FR DA Rxns 10/22/03 4:01 PM
■ Problem of the Day:

O Me Me Me
H H Me
OMe
Me O toluene, ∆ O
Me Fallis J. Org. Chem. 1993, 58, 2186.
O
Me
OMe Me Me
97% yield (+)-Longifolene

O O O
H
H 1,5-H shift
Me O Me O 1,5-H shift Me O
Me Me Me
Me Me Me
H
OMe OMe OMe

Me Me Me
OMe Me Me
O O
OMe
H O O
Me
O Me Me
Me O
MeO H H

[4+2] Exo T.S.

Me Me Me Me Me
H
OMe OMe
O O
Me O O O

O
Me Me Me
OMe

97% yield

17-14-Fallis Longifolene 10/22/01 8:06 AM


D. A. Evans Dipolar Cycloadditions Chem 206

Dipolar Cycloaddition Reactions

http://www.courses.fas.harvard.edu/~chem206/ General References


Carruthers, W. Cycloaddition Reactions in Organic Synthesis.;
Pergamon: Elmsford, NY, 1990.
Chemistry 206 Padwa, A. 1,3-Dipolar Cycloaddition Chemistry, John Wiley, 1984,
Volumes 1 & 2,
Advanced Organic Chemistry Jorgensen, Asymmetric 1,3-Dipolar cycloadditions, Chem Rev. 1998,
98, 863-909

Padwa, A. Generation and utilization of carbonyl ylides via the tandem


Lecture Number 17 cyclization-cycloaddition method." Acc. Chem. Res. 1991, 24, 22.
(handout)

Confalone, P. N.; Huie, E. M. The [3+2] Nitrone-Olefin Cycloaddition


Dipolar cycloaddition Reactions Reaction Org. React. (N.Y.) 1988, 36, 1.
S. Kanemasa, Metal Assisted 1,3-Dipolar Cycloaddition Reactions,
SynLett 2002, 1371-1387 (handout)

■ Reading Assignment for week:


Carey & Sundberg: Part A; Chapter 11, p 647-648 Problem of the Day
Concerted Pericyclic Reactions
Provide a plausible mechanism for this transformation in the space below. In
attacking this question, it is important that you are aware of the transformation
that transpires when terminal acetylenes are treated with Cu(I) or Ag(I) in the
Carruthers, W. Cycloaddition Reactions in Organic presence of an amine base.
Synthesis.; Pergamon: Elmsford, NY, 1990.pp
294-331 (Handout) O Ph R
N R 1–3% CuX
D. Ripin, W. Use of Isoxazoles as 1,3-Dicarbonyl Ph H Ar major
R3N, RT N
equivalents in Organic synthesis; Evans Group H O Ar
Seminar 1997 (ElectronicHandout)
In 1995 Miura and co-workers reported the remarkable reaction illlustrated below (J.
Org. Chem. 1995, 60, 4999). Recently, Fu has reported an enantioselective variant of
this transformation (J. Am. Chem. Soc. 2002, 124, 4572). In most instances, the cis
Monday, adduct is formed in large excess (>90%). There is really no thoughtful mechanism in
the literature for this transformation. You will be graded on "reasonability"
D. A. Evans October 27, 2003

18-00-Cover Page 10/27/03 9:17 AM


D. A. Evans Dipolar Cycloaddition Reactions: An Introduction Chem 206

The General Reaction Family Classification of 1,3-Dipoles Containing C, N, & O Atoms


B
B B [4πS + 2πS] A C ●
A C A C
D E ●
isoelectronic with
allylanion D E ●


C C
HOMO B LUMO B ●
A A
FMO Analysis

E E
LUMO D HOMO D

p.. The specific set of reaction partners, will define the dominant frontier orbitals

Reaction Stereospecificity: The Dipolariphile (Padwa, Vol 1, pp 61-90)


N ●
Me Me
N
Me Me
MeO2C CO2Me MeO2C CO2Me
H
C N N
Me CO2Me H N
N
Me CO2Me
Me

MeO2C MeO2C Me

Rinehart, JACS 1962, 84, 3736


H H ●

MeO2C CO2Me
H
N
O
nitrone N ●
O MeO2C CO2Me

Fumarate gives trans cycloadduct Huisgen, Chem. Ber. 1969, 102, 736

18-01-Introduction-1 10/27/03 8:56 AM


D. A. Evans Dipolar Cycloaddition Reactions: An Introduction Chem 206
Reaction Stereospecificity: Dipolariphile continued Bn Rng strain factors may control regioselectivity
Bn N
MeO2C H 2C O
Bn
H N 100 ° N
major H
N
R Me
CO2Me CO2Me
O O
Bn Me
Ratio 98:02 H
RO Me N
MeO2C heat in toluene Strain Energy
70%
43.8 kcal/mol
DeShong, JOC 1985, 50, 2309; Tet. Lett. 1986, 27, 3979 R Me
O Me
We will return to the other stereochemical element of this reaction shortly N H
Me MeI

Reaction Regiochemistry (Padwa, Vol 1, pp 135) LiAlH4 Strain Energy


HO H
37.6 kcal/mol

N H H
p.. Ph
N N
Ph N N N CO2Me Here is an elegant example of regiochemical control.
Me While the authors offer no explanation for the
LUMO CO2Me H
N outcome, it appears that the reaction is being
Me governed by transition state steric strain factors.
N Ph Oppolzer, JACS 1978, 98, 6722
H
Ph N N N N N
C 4H 9
O
H
HOMO C 4H 9

Reaction Diastereoselectivity Huisgen et al, Angew.Chem. Int. Ed. 1969, 8, 604


The specific set of reaction partners, will define the dominant frontier orbitals
Ar
Ar Ar
H N CO2Me N
Steric Effects will frequently alter regioselectivity (Padwa, Vol 1, pp 144) 100 ° MeO2C CO2Me
MeO2C N CO2Me
H H CO2Me H
N
N MeO2C CO2Me
C N N
H 100:0
H conrotation
Me MeO2C CO2Me
Me H CO2Me
Me
Ar
Me CO2Me Ar Ar
Me N Me N 100 ° N
H H MeO2C N MeO2C CO2Me
C N N N
Me 96:4
Me Me CO2Me CO2Me CO2Me
Me CO2Me MeO2C CO2Me

Steric effects are also important considerations in reaction regiochemistry

18-02-Introduction-2 10/25/03 5:19 PM


D. A. Evans Dipolar Cycloaddition Reactions: An Introduction Chem 206

Relative Orientation of Reaction Partners Cis Disubstituted Olefins.

DeShong, JOC 1985, 50, 2309; Tet. Lett. 1986, 27, 3979 ‡
O H
N O N O N
H Bn O
MeO2C H H
N Bn N O HH
MeO2C
H
H O
R H Major cycloadduct (30:1)
H
MeO2C O R Me
N Bn The preceding trend appears to be reinforced by cis disubstitution
H H O
H Me Ratio 98:02 I. Washita et al, Chem lett 1979, 1337
R H
H Me H Bn
O MeO2C Intramolecular Reaction Variants N. LeBel et al, JACS 1964, 86, 3759
N Bn N
H MeO2C
R Me Me H Me
H H ‡ H
R Me O
O H Me O
O H
p.. Me N N N
Me
H Me
The above analysis is clouded by the fact that the geometry of the 1, 3-dipole is O
not fixed. H

H CO2Me Orientation probably driven by ring strain as in "highly diastereoselective"


MeO2C H Oppolzer case (previous page)
N Ph N Ph
H H
H H Intramolecular Reaction Variants P. Confalone et al, Tet. Lett. 1984, 25, 4613

Monosubstituted Olefins.
In the following study, 1,3-dipole isomerism is not an issue CO2H H
O ∆ O
CHO
‡ N HNTMS2 H H
Ph Me CO2Me O H Bn N
N O O O
N Bn
N O H 78% 100% 82%
–H2O
H H H highly stereoselective
R R
‡ O ‡

H N O H CH2
O R O –CO2 R CH2
R N 22% 0% 18%
H H
N N N
Ar Bn Ar Bn Bn
R
H H Ar
R H H H
Diastereoselection appears to be dictated by steric effects
Conclusions on Reaction Diastereoselection
Tufariello, Accounts. Chem. Res. 1979, 12, 396-1403
In general reaction diastereoselection appears to be dictated by
steric and torsional rather than electronic factors
18-03-Introduction-3 10/27/03 9:01 AM
D. A. Evans Dipolar Cycloaddition Reactions: Nitrile Oxide Cycloadditions Chem 206
The Basic Reactions Reactions with olefins are stereospecific
Padwa, A. 1,3-Dipolar Cycloaddition Chemistry, John Wiley, 1984, Vol 1, Chapter 3 Carruthers, W. Cycloaddition Reactions in Organic Synthesis.; Pergamon: Elmsford, NY,
1990.Chapter 6, pp2 69–298

N OH Me
R Me O
O
R C N O R Isoxazoles Cl N Et3N N
+
R 0 °C Me
Me CO2Et CO2Et

N
R
O Me
R C N O R Isoxazolines O
Me O O
N Et3N N
R +
PhN=C=O Me
Me
Methods of Generation Me Me
0°→rt
H
OH O
N NCS N Et3N Oxazoline Cleavage
Method A R C N O
R H R Cl
p..
Me Me OH
O H2 Raney Ni
O O O O NHPh N
Et3N O
N N
Method B Me Me
PhN=C=O O Me Me
Non-aldol approach to
R R H aldol adducts
Me Me OH
O H2 Raney Ni
Stability Nitrile oxides are usually prepared in the presence of the olefin or
acetylene acceptor. These intermediates are generall unstable and will N
O
dimerize if not given an alternative reaction course Me Me
Me Me
N R N
R C N O R C O
O N O H2 Raney Ni O O
Non-aldol approach to
R N 1,3-dicarbonyls
R C N O R R R R
R C N O O
Preferred method for reducing oxazoles and oxazolines:
Nittta et al, Chem. Comm. 1982, 877-878: Mo(CO)6 MeCN
Regioselectivity Nitrile oxide cycloadditions with olefins and acetylenes are usually
quite regioselective and in the direction as illustrated above.

DeShong, JOC 1985, 50, 2309; Tet. Lett. 1986, 27, 3979

18-04-Nitrile oxides 10/27/03 9:03 AM


D. A. Evans Nitrile Oxide Cycloadditions: Applications in Synthesis Chem 206
The Intermolecular Case
Miyakolide Synthesis: with David Ripin & David Halstead, JACS 1999, 121, 6816-6826
Me Me
Me
3-ClPhNCO
Me Me
Me i-Pr2NEt
90˚ C
O O O
O O 96% Me
Me O 2N OTBS OTBS BnO 1
O O biomimmetic aldol OMe
H reaction? Me Me Me
OH OH N O OTBS O
Me Me
17 19 O O O O
Me
13
Me
O O BnO Me Me OPMB CO2Me
OH OMe Xp O
Me CO2Me H O
OH Me Me
O O O OTBS O
19
OR
17 Me
Me O OH O
Me OPMB CO2Me
Xp O
p.. O M ‡
13 11

Me O O Me OH CO2Me
The Intramolecular Case
H Me
R
Me OH
H 11 Me
R Me
H
O O Me
Transannular Aldol N
O Me
O O O
19
OR 3-ClPhNCO
19
17 Me Me i-Pr2NEt
Me O OH O O 2N O 90˚ C
OMe
Me Me
O OTES O 68 % Me
Me OH CO2Me
Me

Me OPMB
Competing olefin chlorination eliminated this approach to the nitrile oxide precursor Xp O O OMe O O
N Me
Me Me O O 1
19 OMe
Me Me
Me Me O OTES O
HO HO
N O N O
X 13

H OTES Me OPMB
Cl OTES Xp O O OMe
Me Me

18-05-Nitrile oxide Appl-1 10/26/03 9:01 PM


D. A. Evans Nitrile Oxide Cycloadditions: Applications in Synthesis Chem 206
Reaction Diastereoselectivities
Development of Directed Cycloadditions
Kanemasa at al, JACS 1994, 116, 2324-2339 (electronic handout) N O N O
Kanemasa at al, Metal-Assisted Stereocontrol of 1,3-Dipolar Cycloaddition Reactions Et A Et Et
Ph Ph
SynLett 2002, 1371-1387 (electronic handout) base
OH OH
OH
O
Et3N 67:33 60%
N O N O
Me Me EtMgBr 95:5 75%
Ph R' Ph Me
Ph C N O
O Me R'
OBn N O N O
Me 69:31→74:26 Me A Me Me
Ph Ph
base
conditions; no cat, ZnI2, Ti(OiPr)4 OH
OH OH

While lewis acid activation is known, no change in regiochemistry was noted under
above connditions Et3N 61:39 60%
p..
Magnesium alkoxides found to effect regiochemical control EtMgBr 96:4 75%

N O N O Me
Ph C N O N O N O Me
Me
R OH Ph Ph nBu A nBu nBu
CH2OH R Ph Ph
base
R CH2OH OH OH
OH

Me OH + Et3N 46:54 82%


Et3N 60:40

OH Me OMgBr (2 equiv) >99:1 66% EtMgBr 96:4 85%


N

Stereochemical Rationale
Ph Cl >99:1 68%
Me OMgBr (2 equiv)
N ‡ N

A Ph C
X Ph X
Pr O C O
M M

OMgBr (2 equiv) 98:2 73% H H O H H O


syn product Anti product
H H
N O H R R H
A
Ph Ph Rate acceleratons
base
OH OH O N
OH Me
Et3N 67:33
OH Me OH OH Me OH
EtMgBr 100:0
2030 6900 490 16,000

18-06-Nitrile oxides/Kanemasa 10/27/03 9:20 AM


D. A. Evans Nitrile Oxide Cycloadditions: Applications in Synthesis Chem 206

Applications in Polypropionate Synthesis Applications to the Synthesis of Epothilones A, B

Carreira at al, Angew. Chem. Int. Ed. 2001, 40, 2082–2085 Carreira & Bode JACS 2001, 123, 2082–2085

Epothilone A: R = H
Epothilone B: R = Me
S
TBSO N O Me TBSO N O Me
Me O S
Me 82% 87% Me Me
N Me O
Me OH
Me Me N
Me OH Me OH OH
Me Me
syn–syn Me Me OH
OH anti–syn O Me Me
OH
TBSO N TBSO N Me
O
O OH O
H H
Me Me
N O OTIPS
TBSO N O TBSO N O MgBr OTIPS
73% N chelate control
p.. Me 68% Me Me
Me Me Me HO Me ~10:1
S Me
Me Me OH Me Me OH Me
OH N O M
anti–anti N O
syn–anti O
N O R
a, Oxime Chlorination: t-BuOCl; b, 3 Equiv EtMgBr, room temp, 12 h Me Me
Me Me
S

OH O N O
Oxazoline Reduction O N Me EtO
EtO t-BuOCl P Me
P
TBSO N O H EtMgBr EtO
TBSO O OH EtO OH
Raney Ni Me 79%
Me OH
Me Me
90% syn–anti CHO
Me Me OTES
LiCl, DBU N
Me Me OTES Me
Oxidation
Lit Conditions: Curran, JACS 1983, 105, 5826; JOC 1984, 49, 3474 S

N O
TBSO N O TBSO O
O OH N
Me
Raney Ni
Me anti–syn Me
Me Me
94% S
Me Me OTES Me Me OTES

18-07-Nitrile oxides/Carreira 10/26/03 8:57 PM


D. A. Evans Carbonyl Ylides: Introduction Chem 206

The Basic Reactions


Carbenes Plus Carbonyl Groups
Padwa, A.; Hornbuckle, S. F. Chem. Rev. 1991, 91, 263-309.
Reviews - Ylides Padwa, A.; Krumpe, K. E. Tetrahedron. 1992, 48, 5385-5483.
R
Padwa, A. Acc. Chem. Res. 1991, 24, 22-28. O R O
M
R R
R1 O
R O
A B
A B Tandem Intramolecular Cyclization–Intermolecular Cycloaddition

R R
R
Stabilized (Isolable) Carbonyl Ylides Rh2(OAc)4
O N C CO2Et
NMe2 O N
CHN2 O
N2 Me2N O CO2Et
O
O O O O
F 3C CF3 H3C CH3 hν
F 3C CF3
N N
NPh RCHO
CH3 CH3
p.. F 3C CF3 F 3C CF3
O DMAD R
Arduengo, A. J., III; Janulis, E. P., Jr. J. Am. Chem. Soc. 1983, 105, 5929-5930 R O
H O
R O
O NPh CO2CH3 R
O CH3 H
Hamaguchi, M.; Ibata, T. Tetrahedron Lett. 1974, 4475-4476. N O
O
O H O
CO2CH3
O
O 2N O
O O
O 2N X Padwa, A.; Hornbuckle, S. F.; Fryxell, G. E.; Stull, P. D. J. Org. Chem. 1989, 54, 817-824.

N Cu(acac)2

N2 CH3 PhH, 80 °C O CH3


X Intramolecular Variants
N
X = H, Br, OMe
O
H O
O O
O O 2N X O Rh2(OAc)4 O
C6H4NO2 CHN2
O
CO2CH3
N CH3O2C O 75%
CH3 Ph CO2CH3
CO2CH3
Can make 5-7 membered rings
quantitative

Hamaguchi, M.; Ibata, T. Chem. Lett. 1975, 499-502.

18-08-Carbonyl Ylids 10/26/03 9:30 PM


D. A. Evans Carbonyl Ylides: Applications Chem 206

Carbonyl Ylide Cycloadditions of Diazoimides


Cycloadditions with Oxidopyrylium Ylides
O O O H CH3
Me Rh2(OAc)4
N Me O O
COMe A B
Bn N2 PhCH3, 110 °C O
N
Bn O O O
A B
74%

H Me O
O O O Br2, MeOH 1) H3O+
Rh2(OAc)4 R R
Me O O R
N Me O
COMe -45 °C MeO OMe 2) Ac2O / pyr AcO O
PhCH3, 110 °C Me OH OH
N
Me Bn N2 80% from
Bn O
furfuraldehyde
73%
Maier, M. E.; Evertz, K. Tetrahedron Lett. 1988, 29, 1677-1680.

p.. O O
O DBN, CH2Cl2, RT
Me O O
AcO O
H 75%
O O O
N Rh2(OAc)4 O N H
N Me
PhCH3, 110 °C
N H O O
N2
H
pyrolysis O
"high yield" AcO O
O
Me O

O
N O
O
cat. HOAc / CH3CN O
N O 150 °C, 16h
OH
H 52%

Sammes, P. G.; Street, L. J. J. Chem. Soc., Chem. Commun. 1982, 1056-1057

Hertzog, D. L.; Austin, D. J.; Nadler, W. R.; Padwa, A. Tetrahedron Lett. 1992, 33, 4731-4734.

18-09-Carbonyl Ylids-2 10/26/03 9:47 PM


D. A. Evans Carbonyl Ylides: Applications Chem 206

Cyclizations with 5-Hydroxy-4-Pyrones


Phorbol: The Hydroxypyrone Approach
O H OH
O
OH O OH
O
A B Me OH
Me
O O H
O A B Me
Me Phorbol
H
OH

O OH
OH
O OAc
O
OH O
1) PhH, reflux, 12 h
O
O
2) acetylation
O N
Me H
N H Me
Me t-BuMe2Si O
p.. TBSO Me
55% PhCH3, 200 °C O H

O O O O
OAc
OH
1) CH3CN,reflux, 60 h O OTBS
OTBS
O O
O 2) acetylation O
N
Me N
Me
42%

O O
71%
OH Me Me
OAc
1) heat H O
O Me
( )n O 2) acetylation O O TBSO O
( )n
Z
Z Z O
O
Z H OTBS OTBS
Z = CO2Me
n = 1 : 70%
n = 2 : 65%

Garst, M. E.; McBride, B. J.; Douglass, J. G. III. Tetrahedron Lett. 1983, 24, 1675-1678. Wender, P. A.; McDonald, F. E. J. Am. Chem. Soc. 1990, 112, 4956-4958

18-10-Carbonyl Ylids-3 10/26/03 10:00 PM


D. A. Evans Carbonyl Ylides: Problems Chem 206

Problem 53. Williams recently reported an approach to the synthesis of quinocarcinamide (1) (J. Org. Chem. 1995, 60, 6791). The
pivotal process that establishes the tetracyclic nucleus is the two-step transformation shown below (eq 1).
CO2H
CO2Me
H
Me Me
N N Me
H N
N N (1)
two steps N

OMe CH2OH O OMe CH2OH O


1 A OMe CH2OH O
B
Devise a strategy for transforming A into B and clearly illustrate your answer in the space below. Full credit will be
awarded to concise answers.

Problem 55. The following transformation was recently reported by Heathcock during studies directed toward the synthesis of
sarain A
(Tetrahedron Lett. 1995, 6, 2381). From your knowledge of the functionality present in the starting material, deduce the
structure (including stereochemistry) of the reaction product which has the same molecular weight as the starting material.
Hint. the 1H NMR spectrum of the product reveals that the olefinic resonances have disappeared.

p.. N Ph 110 °C
O N
45-55%
Ts CO2Et
MeO

Your mechanism for the transformation


product structure

Problem 65. The following stereoselective nitrile oxide cycloaddition has been reported by Kozikowski (Tetrahedron Lett. 1982, 23,
2081;
J. Org. Chem. 1984, 49, 2762). Provide the stereostructure of the major product and rationalize the stereochemical outcome
as indicated in the directions.

Me Me PhNCO, Et3N
The product ?
O 2N Stereoselection: 16/1
product structure

Problem 87. The illustrated transformation has been utilized by Coldham (Chem.
Commun. 1999, 1757) to construct the core ring system of the manzamines.

N
H N
H H
S
i S
S Pr2NEt, toluene, ∆
BocN OH
N
CHO 45% S
BocN N
CH2 Me
O N
NHMe•HCl EtO2C
OEt "one diastereomer"

18-11-Problems-1 10/26/03 10:19 PM


D. A. Evans Carbonyl Ylides: Problems Chem 206

Problem 90. Padwa and co-workers recently disclosed the illustrated multistep polycyclisation as a possible route to
the strychnine core (Org. Lett. 2001, ASAP)

O
N2 Ph H
O N Rh2(pfb)4
O
145 °C
O O N
O Ph
In the space below, provide a mechanism for the indicated transformation. Hint: The management suggests that a
carefull bidirectional analysis might help you to arrive at a sollution of this question.

Problem 136. A recent paper by Harwood and Park highlights the rapidity which whch one may assemble complex architecture
in a single chemical operation (Tetrahedron Lett. 1999, 40, 2907 and earlier cited references). The transformation in question is
illustrated below. You are asked to address two aspects of this transformation.
H
Ph N O
benzene, heat H H

p.. H Ph N
O O
H
O O
Part A. Provide a concise mechanism for the indicated transformation. For now, ignore the stereochemical aspects of
the reaction.
Part B. Predict the stereochemical outcome of the reaction at the three new stereocenters, and provide a
three-dimensional drawing of the transition state wherein these centers are produced.

Problem 171. A recent paper by Dolle (Tetrahedron Lett. 1999, 40, 2907) highlights the rapidity which whch one may
assemmble complex architecture in a single chemical operation. The transformation in question is illustrated below.
CO2Me
O
1. BocNHNH2 N
N–Boc 75% yield, one diastereomer
CO2Me 2. EtOH, reflux

Provide a concise mechanism for the indicated transformation. In that step where the complex stereochemical
relationships are established, a carefully rendered three dimensional illustration is requested.

Problem 189. This question is taken from recent work reported by Jack Baldwin (Org. Lett., 1999, 1933 and
1937). Provide a mechanism for the conversion of I to II.

O O
O
Rh2(OAc)4;
O O
H3O+ quench
O

I OH
N2
O
II

18-12-Problems-2 10/26/03 10:18 PM


D. A. Evans Acid-Base Properties of Organic Molecules Chem 206

Articles on the Acidities of Organic Molecules


http://www.courses.fas.harvard.edu/~chem206/
Lowry & Richardson: 3rd Edition, Chapter 3
Chemistry 206 Acids and Bases

Advanced Organic Chemistry


Here is a web site containing Brodwell pKa data

Lecture Number 19 http://www.chem.wisc.edu/areas/reich/pkatable/index.htm

Acid-Base Properties of Organic Molecules ■ Problems of the Day:


Explain why 1 and 3 are ~4 pKa units more acidic than their acyclic
■ Bronsted Acidity Concepts in the Activation of Organic Structures counterparts 2 and 4. (J. Org. Chem. 1994, 59, 6456)
■ Medium Effects on Bronsted Acidity
O
■ Substituent & Hybridization Effects on Bronsted Acidity O
O O
■ Kinetic & Thermodynamic Acidity of Ketones O Me Et HN O Me Et
O N O
■ Kinetic Acidity: Carbon vs. Oxygen Acids
H
■ Tabulation of Acid Dissociation Constants in DMSO
1 2 3 4

The thermodynamic acidities of phenol and nitromethane are both


■ Reading Assignment for this Lecture: ~10; however, using a common base, phenol is deprotonated 10+6
times as fast. Rationalize
Carey & Sundberg: Part A; Chapter 7
Carbanions & Other Nucleophilic Carbon Species
Base
pKa(H2O) O O
"Equilibrium acidities in DMSO Solution", F. G. Bordwell. ~10 H rel rate: 10+6
Acc. Chem. Res. 1988, 21, 456-463. (handout)

Wednesday, O O
D. A. Evans October 29, 2003 pKa(H2O) Base
~10 H 3C N H2C N
O rel rate: 1 O

19-00-Cover Page 10/27/03 11:36 AM


D. A. Evans Acidity Concepts-1 Chem 206
■ Definition of Ka
Activation of Organic Molecules
Let H–X be any Bronsted acid. In water ionization takes place:

■ Base Activation H–X + HOH H3O+ + X–


R1 base R1
[H3O+] [X–]
C H C Nucleophile where Keq = where [HOH] = 55.5 mol L-1 (A)
R2 - H-base R2 [H–X] [HOH]
R3 R3
Since [HOH] is, for all practical purposes, a constant value, the acid
dissociation constant Ka is defined wilthout regard to this entity. e.g.
pKa , describes quantitatively a molecule's propensity to act as an acid, i.e. H–X H+ + X– where H+ = H3O+
to release a proton.
Hence [H+] [X–] (B)
- Medium effects Ka =
- Structural effects (influence of substituents R1) [H–X]

■ Acid Activation From the above definitions, Ka is related to Keq by the relation:

p.. R1 acid (protic or lewis acid) R1 acid Ka(H–X) = 55.5 Keq(H–X) (C)

X X Electrophile
■ Autoionization of water
R2 R2
X = e.g. O, NR ... H3O+ +
HOH + HOH HO–
Keq = 3.3 X 10–18
■ The Aldol Example
M M From Eq C: Ka = 55.5 Keq = 55.5(3.3 X 10–18)
O O O O
base catalysis Hence Ka = 1.8 X 10–16
R +
R H R R R
HOMO Since pKa is defined in the following equation:
R
base pKa = – log10 [Ka] The pKa of HOH is + 15.7
O O O OH
Keep in mind that the strongest base that can exist in water is HO–.
R +
R H R R R
R
acid Lets now calculate the acid dissociation constant for hydronium ion.
Ca 10+6 Activation
LUMO M H3O+
SiMe3
O
SiMe3 H3O+ + H 2O + H 2O
O O O
R + acid catalysis
R H R R R obviously: Keq = 1
R Ka = [HOH] x Keq hence Ka = 55.5
pKa = – log10 Ka = –1.7

The strongest acid that can exist in water is H3O+.


19-01-Acidity Concepts-1 10/27/03 12:51 PM
D. A. Evans Acidity Trends for Carbonyl & Related Compounds Chem 206
■ The Gibbs Relationship ■ Medium Effects on the pKa of HOH
HOH pKa Medium
∆ G° = – RT ln K
2.3 RT = 1.4 ** The gas phase ionization of HOH is 15.7 HOH
or ∆ G° = – 2.3 RT log10 K endothermic by 391 kcal/mol !!!
at T = 298 K 31 DMSO
in kcal ⋅ mol-1 279 (est)**
∆ G°298 = –1.4 log10 Keq Vacuum
■ Representative pKa Data

∆ G°298 = 1.4 pKeq ≈ 1.4 pKa with pK = – log10 K


Substrate DMSO HOH ∆ pKa
Hence, pKa is proportional to the free energy change HOH 31.2 15.7 15.5
HSH 14.7 7.0 7.7
Keq pKeq ∆ G°
MeOH 29.0 15.3 13.7
1 0 0

Energy
A
10 -1 –1.4 C6H5OH 18.0 9.9 8.1
HA ∆ G°
p.. 100 -2 –2.8 kcal/mol O2N–CH3 17.2 10.0 7.2
Reaction coordinate O
Ph C CH3 24.6 17 7.6
■ Medium Effects Consider the ionization process:

H A + solvent A: – + solvent(H+)
The change in pKa in going from water to DMSO is increasingly diminished as
the conjugate base becomes resonance stabilized (Internal solvation!).
In the ionization of an acid in solution, the acid donates a proton to the medium. The
more basic the medium, the larger the dissociation equilibrium. The ability of the
medium to stabilize the conjugate base also plays an important role in the promotion
of ionization. Let us consider two solvents, HOH and DMSO and the performance of Substrate DMSO HOH ∆ pKa
these solvents in the ionization process.
18.1 16.0 2.1
The Protonated Solvent Conjug. Base Stabiliz.
H O O
Water H O O H A EtO OEt 16.4 13.3 3.1
H H
O O
Me
HO S No H-bonding Capacity 13.3 8.9 4.5
DMSO Me Me
Me

As shown above, although HOH can stabilize anions via H-bonding, DMSO cannot. NC CN 11.1 11.2 0
Hence, a given acid will show a greater propensity to dissociate in HOH. As
illustrated below the acidity constants of water in HOH, DMSO and in a vacuum
dramatically reflect this trend.

19-02-Acidity Concepts-2 10/27/03 12:54 PM


D. A. Evans Acidity Trends for Carbonyl & Related Compounds Chem 206

Electrons in 2S states "see" a greater effective nuclear charge


Substituent Effects than electrons in 2P states.

This becomes apparent when the radial probability functions for S and
Electronegativity e.g. Compare Carboxylic Acids vs. Ketones P-states are examined: The radial probability functions for the hydrogen
atom S & P states are shown below.

H O H O Carboxylate ion
O– O– 100 % 100 %
H C C more stabile than
H C C R C enolate because R
O C
H H H CH2–H O more 1 S Orbital
O CH2

Radial Probability
Radial Probability
electronegative than C
(H2O) pKA = 4.8 pKA ≈ 19

(DMSO) pKA = 12.3 pKA ≈ 26.5


2 S Orbital 2 S Orbital

Hybridization - S-character of carbon hybridization


2 P Orbital
p..
Remember: sp3-orbitals 25% s-character
Å Å
sp2-orbitals 33% s-character
3 S Orbital 3 P Orbital
sp-orbitals 50% s-character

Carbon Acids S-states have greater radial penetration due to the nodal properties of
RR the wave function. Electrons in s states see a higher nuclear charge.
R R R
The above observation correctly implies that the stability of nonbonding
R H electron pairs is directly proportional to the % of S-character in the
R H H RR H doubly occupied orbital.

Hybridzation sp sp2 ≈ sp2 sp3 Carbanions CSP3 CSP2 CSP

Bond Angle 180° 120° ≈ 120 109° Least stable Most stable

Carbenium ions CSP3 CSP2 CSP


pKa(DMSO) 23 ≈ 44 ≈ 39 ≈ 60
Most stable Least stable

The above trends indicate that the greater the % of S-character at a given
atom, the greater the electronegativity of that atom.
19-03-Acidity Concepts-3 10/27/03 12:55 PM
D. A. Evans Acidity Trends for Carbonyl & Related Compounds Chem 206
Hybridization vs Electronegativity Substituent Effects
There is a linear relationship between %S character &
Pauling electronegativity ■ Alkyl Substituents on Localized Carbanions are Destabilizilng:
5
Steric hinderance of anion solvation
NSP
Compare: (JACS 1975, 97, 190)
4.5

pKA (DMSO) S H pKA (DMSO)


Pauling Electronegativity

4
PhSO2-CH–H
N 29 31.1
NSP3
SP2
H S H
3.5
PhSO2-CH–Me
C SP 31 S Me
H 38.3
3
S H

C
SP2
2.5
CSP3
■ Heteroatom-Substituents: - 1st row elements of periodic table
p.. 2 pKA (DMSO)
20 25 30 35 40 45 50 55
% S-Character PhSO2-CH-OCH3 30.7
Inductive Stabilization versus
There is a direct relationship between %S character & H Lone Pair Repulsion
hydrocarbon acidity (-I vs +M -Effect)
60 PhSO2-CH-OPh 27.9
H
CH (56)
4
55
PhSO2-CH-NMe3 19.4 Inductive Stabilization

50 H
Pka of Carbon Acid

45
■ Heteroatom-Substituents: - 2nd row elements of periodic table
C H (44)
6 6

40 Strong carbanion stabilizing effect

35 pKA (DMSO)
pKA (DMSO)
PhCC-H (29)
PhSO2-CH-H PhSO2-CH-SO2Ph
30 29 12.2
H H
PhSO2-CH-SPh PhSO2-CH-PPh2
25
20 25 30 35 40 45 50 55
20.5 20.5
% S-Character H H

19-04-Acidity Concepts-4 10/27/03 12:56 PM


D. A. Evans Acidity Trends for Carbonyl & Related Compounds Chem 206
■ Conjugative Stabilization of Conjugate Base
■ Carbanion Stabilization by 2rd–Row Atoms: SR, SO2R, PR3 etc
H H O
Me O S H
O C NO2 C N 17.2 pKA (DMSO)
S CH3 H3C S CH3 H3C S CH3 H H O
Me O S H
18.2 (DMSO) 31 31 35 H O H O
C C C C 26.5
Ph CH3 H CH3
H
Ph P CH3
(JACS 1976, 98, 7498; JACS 1977, 99, 5633; JACS 1978, 100, 200).
Ph H H
22.5 C C N C C N 31.5
H H
The accepted explanation for carbanion stabilization in 3rd row
elements is delocalization into vicinal antibonding orbitals For efficient conjugative stabilization, rehybridization of carbanion orbital from
nsp3 to np is required for efficient overlap with low-lying π*-orbital of stabilizing
group. However, the cost of rehybridization must be considered.
S–Xσ* X
E
p.. Cn S–Xσ* (empty) ■ Stereoelectronic Requirement for Carbanion Overlap:
C S Enolization of Carbonyl Compounds
Cn (filled) Stereoelectronic Requirements: The α-C-H bond must be able to overlap with
π∗ C–O

π∗ C–O Ha
This argument suggests a specific orientation requirement. Hc Hb
CH3 base
This has been noted:
R C O – Ha+
R O
Hc R O–
Anti (or syn) periplanar orientation of Carbanion-orbital and σ* orbital mandatory Hb
for efficient orbital overlap.

H Ha Rates for deprotonation with n-BuLi


pKA pKA (DMSO)
H
Me S He He : Ha = 8.6 5.2 Ph3C–H 31.5
(JACS 1974, 96, 1811) O O
S
Me H

He
S He : Ha = 30 (JACS 1978, 100, 200)
C-H acidity not 47.7
Ha detectable
H O H H
O
19-05-Acidity Concepts-5 10/24/01 8:13 AM
D. A. Evans Phenol Acidity: An Analysis of Resonance & Inductive Effects Chem 206

■ Phenol Acidity: ■ Is the benzene ring somehow special. i.e "larger resonance space."
OH O–
■ Acetone enol:
+ H+ (1)
∆G° FG acetone acetone enol acetone enolate
FG
-8
Me O Keq = 10 H 2C OH pKa = 10.9 H2 C O–
This topic has a number of take-home lessons. Most importantly, is is a useful + H+ (1)
construct on which to discuss the role of FG's in influencing the acidity of this Me Me Me
oxygen acid.
The surprising facts is that the acetone enol has nearly the same pKa as phenol.
■ How does one analyze the impact of structure on pKa of a weak Hence, the answer to the above question is no!
acid (pKa > 0) ? O– ■ How important are inductive effects in the stabilization of C6H5O–?
■ The Approach: Consider the following general oxygen acid X–OH where X can only stabilize
FG
+ solvent(H+) the conjugate base through induction:
For equilibria such as that presented X OH pKa(H2O)
above, analyze the effect of stabilizing X OH X O– + H+
Energy

(or destabilizing) interactions on the CH3 OH 15.5


OH
more energetic constituent which in ∆G° As the electronegativity of X increases
CF3CH2 OH 12.4
this case is the conjugate base. the acidity of X–OH increases.
FG
Cl OH 7.5
If you take the calculated electronegativity of an SP2 carbon (2.75) you can
see that there is a linear correlation between the electronegativity of X and the
■ Why is phenol so much more acidic than cyclohexanol? pKa of X–OH.
This argument suggests that the acidity of acetone enol is largely
OH O– due to inductive stabilization, not resonance.
3.2
+ H+ pKa (H2O) = 10 HOCl (7.5)
∆G°
OH O– 3

+ pKa (H2O) = 17
+ H
∆G° 2.8
acetone enol (10.9)
Loudon (pg 730): "The enhanced acidity of phenol is due largely to stabilization Electronegativity of X
phenol (10.0)
of its conjugate base by resonance."
2.6

O– – O O O
2.4
– –

from previous discussion, ∆ G˚298 = –1.4 Log10 Keq = 1.4 pKeq 2.2
HOH (15.7)

∆G° (stab) = 1.4(Pkaphenol – pKacyclohenanol) = 1.4(-7) = 9.8 kcal/mol


2
6 8 10 12 14 16
pKa of X–OH
19-06-Phenol acidity 10/27/03 12:58 PM
D. A. Evans Weak Acids: Impact of Structure on Acidity Chem 206
■ The General Reaction: Ionization of a weak acid (pKa > 0) Case III: Carboxylic Acids vs Ketones:

O O– H O H O Carboxylate ion
H + solvent + solvent(H+) O – more stabile than O–
R X R X H C C H C C
∆G° R C enolate because R C
H OH H CH2–H O more
O CH2
electronegative than C
pKa = 4.8 pKa ~ 19
Variables: O– +
+ solvent(H )
X = O (carboxylic acid)
R X
X = NH (amide) O
Case IV: Carboxylic Acids, Esters, Amides & Ketones:
X = CH2 (Ketone/ester) R C
Energy
O
CH2–H
∆G°
R = CR3 R XH
R = OR
O O O O
R = NR2 Me C EtO C Me2N C –O C
CH2–H CH2–H CH2–H CH2–H
■ The Question: How does one analyze the impact of structure on pKa ?
pKa ~ 26 pKa ~ 30 pKa ~ 34 pKa > 34 < 40
■ The Approach:

For equilibria such as that presented above, analyze the effect of


stabilizing (or destabilizing) interactions on the more energetic The Analysis:
constituent which in this case is the conjugate base. In this series of compounds, there are two variables to consider:
Case I: Carboxylic Acids: Inductive Effects
■ Inductive Effect: OEt > Me2N > H3C but (O–?)
H O Cl O Cl O Carboxylate ion ■ Resonance Effect: O– O–
H C C Cl C C stabilized by increased ●● +
Cl C C X C X C
H OH Cl OH electron-withdrawing The degree to which substituent X:
Cl O–
CCl3 group. "contributes" electron density into enolate CH2 – CH2
pKa = 4.8 pKa = 0.6 ●●

represents a destabilizing interaction:


Trend: O– > Me2N > OEt
Case II: Carboxylic Acids: Inductive Effects & Carbon Hybridization
■ Resonance donation dominates inductive electron withdrawal as
indicated by the data.
H H O O O Carboxylate ion
H C C C H C C C stabilized by increased
Substituents on the α-carbon: Stabilization by either resonance,
HC C C
H H OH OH induction, or both is observed:
O – electron-withdrawing
SP-hybridized carbon O O O O
pKa = 4.9 pKa = 1.9
Ph C CH2CH3 Ph C CH2OCH3 Ph C CH2Ph Ph C CH2SPh
pKa = 24.4 pKa = 22.9 pKa = 17.7 pKa = 17.1
19-07 Weak Acids/Gen anal 10/24/01 8:16 AM
D. A. Evans Acidity of Carboxylic Acids, Esters, & Lactones: Anomeric Effects Again? Chem 206

■ Conformations: There are 2 planar conformations. ■ Hyperconjugation: Let us now focus on the oxygen lone pair in the hybrid
orbital lying in the sigma framework of the C=O plane.
O O (Z) Conformer
(Z) Conformer R' (E) Conformer
R O R O R σ* C–O
C O In the (Z) conformation this
O O R' •• O lone pair is aligned to overlap O
Specific Case: H R with σ* C–O.
Formic Acid H O H O ∆G° = +2 kcal/mol R
R O
H
The (E) conformation of both acids and esters is less stable by 2-3 kcal/mol. If (E) Conformer
this equilibrium were governed only by steric effects one would predict that the
(E) conformation of formic acid would be more stable (H smaller than =O). R
C O In the (E) conformation this
Since this is not the case, there are electronic effects which must also be R O lone pair is aligned to overlap
considered. These effects will be introduced shortly.
•• with σ* C–R. σ* C–R
■ Rotational Barriers: There is hindered rotation about the =C–OR bond. O
R
These resonance structures suggest O C Since σ* C–O is a better acceptor than σ* C–R O
hindered rotation about =C–OR bond. O R
barrier ~ 10 (where R is a carbon substituent) it follows that
This is indeed observed: kcal/mol R the (Z) conformation is stabilized by this interaction. R
O O– O
R' R'
R O R O R O
Energy

+ O
Lone pair orientation & Impact on pKa (DMSO)
R R
R O ∆G° ~ 2-3 See Bordwell, J. Org. Chem. 1994, 59, 6456-6458
Rotational barriers are ~ 10 kcal/mol kcal/mol
This is a measure of the strength of Meldrum's Acid
the pi bond. O O
O O O O
Et
CH3CH2 O O Me Me
■ Lone Pair Conjugation: The oxygen lone pairs conjugate with the C=O. O O O O

Me Me
pKa ~ 30 pKa = 25.2 pKa = 15.9 pKa = 7.3
•• R The filled oxygen p-orbital interacts with pi (and pi*)
C O
•• O C=O to form a 3-centered 4-electron bonding system.
O O Is this a dipole effect? See Bordwell
R
Me Et
N O HN O
SP2 Hybridization Me O Me O
H +
+
■ Oxygen Hybridization: Note that the alkyl oxygen is Sp2. Rehybridization pKa = 24.5 pKa = 20.6 O O
is driven by system to optimize pi-bonding. R R
+ +
Houk, JACS 1988, 110, 1870
supports the dipole argument E(rel) = 0 E(rel) = +3.8 kcal

19-08-Lone pair orientation 10/27/03 12:59 PM


D. A. Evans Kinetic & Thermodynamic Acidity of Ketones Chem 206
■ Kinetic Acidity: Rates of proton removal
Kinetic & Equilibrium Ratios of Enolates Resulting from Enolization
Consider enolization of the illustrated ketone under non-equilibrating conditions: with LDA & Subsequent Equilibration

OLi O OLi
HA HB HA HB O O
HB Me LiNR2 Me Me O O
LiNR2 (1) Me (90) Me
HB HB (2) (66)
kB (99) (10) (98) (34)
kA

B– K A‡ A–
Kinetic Ratios Equilibrium
Ratios Kinetic Ratios Equilibrium
B ‡ Ratios
∆G‡A
∆G‡B (13) H (47) H
O
O O O
Energy

K (87)
C3H7 CH3 C3H7 CH3
(53)
(16) (84) (87) (13)
p.. LiNR2 H H
B–
Kinetic Ratios Equilibrium Kinetic Ratios Equilibrium
Ratios Ratios

A
O O
Reaction Coordinate
Ph Ph
CH3 CH3
Kinetic acidity refers to the rate of proton removal. e.g. kA vs kB. For example, in (14) (99) (1)
(86)
reading the above energy diagram you would say that HA has a lower kinetic acidity
than HB. As such, the structure of the base (hindered vs unhindered) employed Kinetic Ratios Equilibrium
plays a role in determining the magnitude of kA and kB. For the case shown above, Ratios
∆G‡A will increase more than ∆G‡B as the base becomes more hindered since the
proton HA resides in a more sterically hindered environment. The example shown
■ Note that alkyl substitution stabilizes the enolate (Why??). This effect
below shows the high level of selectivity which may be achieved with the sterically
shows up in the equilibrium ratios shown above.
hindered base lithium diisopropylamide (LDA).

■ Hence, enolization under "kinetic control with LDA allows you to produce
Me O OLi OLi
H H H H the less-substituted enolate while subsequent equilibration by simply
Me Me THF H Me Me heating the enolate mixture allows equilibration to the more substituted
N Li H H enolate.
–78 °C
Me
Me
Kinetic Ratio 99 : 1
LDA
Equilibrium Ratio 10 : 90

19-09-kinetic Acidity-1 10/27/03 1:00 PM


Evans, Annis Kinetic Acidity: Carbon versus Oxygen Acids Chem 206

■ Kinetic Acidity ■ Kinetic Acidity vs. Leaving Group Ability: E1cb Elimination Reactions
Observation: The thermodynamic acidities of phenol and nitromethane are
Stirling, Chem. Commun. 1975, 940
both approximately 10; however, using a common base, phenol is
deprotonated 10+6 times as fast. O O O
base rds
Ph S Ph S Ph S + X–
Base X X
pKa(H2O) O O O O O
~10 H rel rate: 10+6
O O O
Ph S Ph S + Ph S
OPh PPh3 CN
O O O O O
pKa(H2O) Base
H3C N H 2C N
~10 O rel rate: 1 O krel = 1 krel = 10+4 krel = <10–8

pKa HX 10 0 9.5
Proton transfers from C-H Bonds are slow.
■ Why???
The greater the structural reorganization of the leaving group during
p.. E1cb elimination, the slower the rate of elimination.
Most carbon acids are stabilized by resonance. Hence significant
structural reorganization must accompany deprotonation.

■ Protonation of Conjugate bases


Base
O O
H O H O–H
H N Kinetic product
N
H O H O

O-H electron density O-H electron density


is here. is still here.
H3O+ H3O+
OLi O–H O

H O H O Keq ~ 10+5
Base H
N N Kinetic product
H O H O
H

C-H electron density


is here. electron density
now resides here, and nuclei Jack Hine: Least Motion Principle (Adv. Phys. Org. Chem. 1977, 15, 1)
have moved to accomodate Lowry & Richardson, 3rd Edition, pp 205-206
rehybridization.

The greater the structural reorganization during deprotonation, the lower Those elementary reactions that involve the least change in
the kinetic acidity atomic posiitons will be favored

19-10-kinetic Acidity-2 10/27/03 1:00 PM


D. A. Evans Carbonyl and Azomethine Electrophiles-1 Chem 206

Additional Reading Material Provided


http://www.courses.fas.harvard.edu/~chem206/
Additions to 5- & 6-Membered oxocarbenium Ions:
Chemistry 206 Woerpel etal. JACS 1999, 121, 12208 (Handout)
Woerpel etal. JACS 2000, 122, 168 (Handout)
Advanced Organic Chemistry Woerpel etal. JACS 2003, 125, ASAP (Handout)
"From Crystal Statics to Chemical Dynamics", Accounts Chem.
Research 1983, 16, 153. (Electronic Handout)
Lecture Number 20 "Theoretical Interpretation of 1,2-Asymmetric Induction. The
Importance of Antiperiplanarity", N. T. Anh, O. Eisenstein
Nouv. J. Chem. 1977, 1, 61-70. (Handout)
Carbonyl and Azomethine Electrophiles-1
"Around and Beyond Cram's Rule" A. Mengel & O. Reiser, Chem
Rev. 1999, 99, 1191-1223 (Electronic Handout)
R R R R R R R
C O C O C N C N
R R R R R

■ Relevant Dunitz Articles


■ Reactivity Trends
■ C=X Stereoelectronic Effects "Geometrical Reaction Coordinates. II. Nucleophilic Addition to
■ Carbonyl Addition: Theoretical Models a Carbonyl Group", JACS 1973, 95, 5065.
■ The Felkin-Anh-Eisenstein Model for C=O Addition "Stereochemistry of Reaction Paths at Carbonyl Centers",
■ Diastereoselective Ketone Reduction Tetrahedron 1974, 30, 1563

■ Reading Assignment for this Week: "From Crystal Statics to Chemical Dynamics", Accounts Chem.
Research 1983, 16, 153. (Electronic Handout)
Carey & Sundberg: Part A; Chapter 8
Reactions of Carbonyl Compounds "Stereochemistry of Reaction Paths as Determined from Crystal
Structure Data. A Relationship Between Structure and Energy.",
Carey & Sundberg: Part B; Chapter 2 Burgi, H.-B. Angew. Chem., Int. Ed. Engl. 1975, 14, 460.
Reactions of Carbon Nucleophiles with Carbonyl Compounds
Carey & Sundberg: Part B; Chapter 5
Reduction of Carbonyl & Other Functional Groups

Friday,
D. A. Evans October 31, 2003

20-00-Cover Page 10/30/03 11:44 AM


Chemistry Reviews Issue on Diastereoselection in
C=O Addition
Chem Rev. 1999, 99, (5), 1069-1480

Mengel, A. and O. Reiser, “Around and beyond Cram's rule.” Chem. Rev. 1999,
1191-1223.
Reetz, M. T., “Synthesis and diastereoselective reactions of N,N-dibenzylamino
aldehydes and related compounds.” Chem. Rev. 1999,1121-1162.

Dannenberg, J. J., “Using perturbation and frontier molecular orbital theory to


predict diastereofacial selectivity.” Chem. Rev. 1999, 1225-1241.

Tomoda, S., “The exterior frontier orbital extension model.” Chem. Rev.
1999,1243-1263.
Cieplak, A. S., “Inductive and resonance effects of substituents on pi-face selection.”
Chem. Rev. 1999,1265-1336.

Ohwada, T., “Orbital-controlled stereoselections in sterically unbiased cyclic


systems.” Chem. Rev. 1999,1337-1375.

Gung, B. W., “Structure distortions in heteroatom-substituted cyclohexanones,


adamantanones, and adamantanes: Origin of diastereofacial selectivity.” Chem. Rev.
1999,1377-1386.
Kaselj, M., W. S. Chung, et al., “Face selection in addition and elimination in
sterically unbiased systems.” Chem. Rev. 1999, 1387-1413.

Adcock, W. and N. A. Trout, “Nature of the electronic factor governing


diastereofacial selectivity in some reactions of rigid saturated model substrates.”
Chem. Rev. 1999,1415-1435.

Mehta, G. and J. Chandrasekhar, “Electronic control of facial selection in additions to


sterically unbiased ketones and olefins.” Chem. Rev. 1999,1437-1467.

Wipf, P. and J. K. Jung, “Nucleophilic additions to 4,4-disubstituted


2,5-cyclohexadienones: Can dipole effects control facial selectivity?.” Chem. Rev.
1999,1469-1480.

20-000-chem reviews 1999 10/29/03 4:15 PM


D. A. Evans C=X Electrophiles: Carbonyls, Imines & Their Conjugate Acids Chem 206
■ The Set of Functional Groups:
Stereoelectronic Considerations for C=O Addition
R R R R R R R
C O C O C N C N LUMO is π∗ C–O; HOMO Provided by Nu:
R R R R R
π∗ C–O
Aldehyde Oxocarbenium Aldimine Iminium
Ketone ion Ketimine ion
(Imine)
π∗ C–O
These functional groups are among the most versatile sources of electrophilic carbon R
in both synthesis and biosynthesis. The ensuing discussion is aimed at providing a C O LUMO
more advanced discussion of this topic. R
R R
■ C=X Polarization C O C O HOMO
Dunitz-Burgi trajectory (Nu)
R R
Partial Charge: As the familiar polar resonance structure above indicates, the
carbonyl carbon supports a partial positive charge due to the polarization of the sigma π C–O
p..
and pi system by the more electronegative heteroatom. The partial charges for this σ Nu–C
family of functional groups derived from molecular orbital calclulations (ab initio, δ –Nu ~107 ° The forming
3-21(G)*, HF) are illustrated below: bond
R R R R R R R
R δ–
C N C O C N C O C O
R R R R R R
δ + 0.61 (R = H)
δ + 0.33 δ + 0.51 δ + 0.54 δ + 0.63 (R = Me)
electrophilic reactivity ■ What about C=O vs C=O-R(+)?
■ Proton Activation of C=X Functional groups
R R H
H–A + C O C O A– R R
R A C O B C O
δ + 0.51 R δ + 0.61 R
R R
The electrophilic potential of the C=O FG may be greatly increased by either Lewis acid
coordination of by protonation. The magnitide of this increase in reactivity is ~ 10+6.
Among the weakest Bronsted acids that may be used for C=O actilvation (ketalization) The LUMO coefficient on carbon for B will be considerably larger than for
is pyridinium ion (pKa = 5). Hence, the Keq below, while quite low, is still functional.
A. Does this mean that there is a lower constraint on the approach
R R H angle for the attacking nucleophile? There is no experimental proof for
+ C O C O Keq ~ 10-11 this question; however, it is worthy of consideration
N R N
H R
pka = +5 pka = –6 ■ What was the basis for the Dunitz-Burgi analysis?

20-01-Introduction 10/30/03 11:46 AM


D. A. Evans The Dunitz-Burgi Trajectory for C=O Addition Chem 206
■ Relevant Dunitz Articles
"Geometrical Reaction Coordinates. II. Nucleophilic Addition to a
Carbonyl Group", JACS 1973, 95, 5065.
2.56Å
"Stereochemistry of Reaction Paths at Carbonyl Centers", Tetrahedron
1974, 30, 1563

"From Crystal Statics to Chemical Dynamics", Accounts Chem. Research


1983, 16, 153.

"Stereochemistry of Reaction Paths as Determined from Crystal Structure


Data. A Relationship Between Structure and Energy.", Burgi, H.-B.
Angew. Chem., Int. Ed. Engl. 1975, 14, 460.
■ Dunitz Method of Analysis
A series of organic structures containing both C=O and Nu FG's disposed in
a geometry for mutual interaction were designed. These structures positioned In this structure (A), at 2.56Å the C=O is starting to pyramidalize
the interacting FGs an increasingly closer distances. The X-ray structures of Cyclic aminoketones. Medium-ring ketones of various ring sizes were analyzed for
p..
these structures were determined to ascertain the direction of C=O distortion. the interaction of amine an C=O FGs. One example is shown below.
The two families of structures that were evaluated are shown below.

1,8-Disubstituted Naphthalenes. Substituents located at these positions are strongly


interacting as illustrated by the MM2 minimized di-methyl-naphthalene structure shown Nu C O
below.

2.29Å
Dunitz, Helv. Chem. Acta 1978, 61, 2783 Me
N
Me Me

R O R
~ 7.6 kcal/moll
Lecture 5
Analysis of distortion of C=O in this
and related structures formed the
basis of the 107° attack angle. This
value should be taken as
O Me O Me approximate.
Me2N MeO

Sekirkine
A (shown) B Birnbaum JACS 1974, 96 6165

20-02-Dunitz-Burgi 10/29/03 4:38 PM


D. A. Evans The Dunitz-Burgi Trajectory for C=O Addition Chem 206
Cyclic aminoketones. Medium-ring ketones of various ring sizes were Should these crystallographic date be relevant to the addition to
analyzed for the interaction of amine an C=O FGs. Two examples are shown complexed C=O & Iminium Ions?
below.
Sekirkine
Birnbaum JACS 1974, 96 6165 R R R R R R R
C N C O C N C O
R R R R R
δ + 0.61 (R = H)
δ + 0.33 δ + 0.51 δ + 0.54
δ + 0.63 (R = Me)

2.46Å
C O π∗ (antibonding)

Energy
C O
C
p.. R O

π (bonding)
C O

C O π∗ (antibonding)

R R

Energy
C O
C
R O
2.69Å

π (bonding)
C O

DB angel ~107 ° DB angel >107 °??


δ –Nu δ–
Nu
R δ– R
C O C O
R R R

20-03-Dunitz-Burgi-2 10/29/03 4:39 PM


D. A. Evans Stereoelectronic Effects in the Addition to Iminium and Oxo-carbenium Ions Chem 206
■ Pivotal Articles
R. V. Stevens in ■ An early example from Eliel; JACS 1969, 91, 536
"Strategies and Tactics in Organic Synthesis", Vol. 1.
On the Stereochemistry of Nucleophilic Additions to Tetrahydropyridinium Salts: a H Ph
H OMe H H
Powerful Heuristic Principle for the Stereorationale Design of Alkaloid Synthesis.; H H
Lindberg, T., Ed.; Academic Press, 1984; PhMgBr Me O H
Me O Me O H
H O
Eliel etal. , JACS 1969, 91, 536 O O Me
Kishi etal. , JACS 1982, 104, 4976-8 Me Me
dioxolenium ion trans : cis 95:5 (95%)
■ The Proposal for Oxo-carbenium Ions (Eliel, Kishi) Eliel was the first to attibute stereoelectronic factors to the addition of nucleophiles to
cyclic oxo-carbenium ions.
H
Nu
Nu Path A R ■ Kishi Examples; JACS 1982, 104, 4976-8
H O
H OBn
kinetic product OBn
R SiMe3 BnO OBn
C O conformations BnO OBn H O CH2OBn
H H
Path B H BF3•OEt2
p.. H
R R PMBO O CH2OBn
O
Nu O stereoselection 10:1 (55%)
Nu Nu
OBn
OBn
■ The Proposal for Iminium Ions (Stevens) BnO OBn Et3Si–H BnO OBn
O CH2OBn
H BF3•OEt2
Nu H
O CH2OBn
Nu Path A R OH
R stereoselection 10:1 (55%)
H N
H Chair-aixal attack on oxo-carbenium ion occurs for both carbon and hydride nucleophiles
R kinetic product
C N conformations
H ■ Iminium Ions (Stevens) cited reference
R Path B H H
H
R N R NaCNBH3
Nu R N
R N N
Nu Nu
C 4H 9 Me C 4H 9
Me
It was proposed that chair-axial addition would be preferred as a consequence of the only one stereoisomer
intervention of a transition state anomeric effect (Path A). Attack through Path B would
necessitate the generation of the twist-boat kinetic product conformation thus n-PrMgBr
destabilizing attack from the equatorial diastereoface. While Stevens espoused this N N
concept for iminium ions in the late 70's, his untimely death at the age of 42 significantly
delayed his cited publication. C 4H 9
C 4H 9 n-Pr
20-04-cyclic onium addns-1 10/29/03 4:40 PM
D. A. Evans Stereoelectronic Effects in the Addition to Iminium and Oxo-carbenium Ions Chem 206
H
H
5-Membered oxocarbenium Ions: Woerpel etal. JACS 1999, 121, 12208. O Allyl
BF3•OEt2 Me
C O Me
O OAc O Me OAc H O
O
SnBr4 H
SiMe3 trans:cis 99:1 (69%)
Me Me Me
stereoselection 99:1
BnO BnO Allyl
SiMe3 O
BF3•OEt2 H H
BnO OAc C O O
O OAc O O H H
SnBr4
SiMe3 cis:trans 89:11(75%)

BnO BnO BnO O Allyl


stereoselection >95:5 BF3•OEt2 BnO
These cases provide dramatic evidence for the importance of electrostatic effects in C O BnO
OAc H
controlling face selecticity. O
H H
OBn H
p.. 6-Membered oxocarbenium Ions: Woerpel etal. JACS 2000, 122, 168. cis:trans 83:17(84%)
H These cases provide dramatic evidence for the importance of electrostatic effects in
O OAc H controlling face selecticity.
Me O
BF3•OEt2 Me H Are the preceding addition reactions somehow related to the apparently
O C
Me H contrasteric reactions shown below??
Allyl OSiR3
O
SiMe3 cis:trans 94:6 (74%)
R3SiO
HgI2
OBn EtO2C
O OAc EtO
OBn
BF3•OEt2 H >94 : 6 OSiR3
O C O OSiR3
BnO H H
H JOC 1991, 56, 387
Allyl O O
H
trans:cis 99:1 (75%)
AlCl3
This analysis presumes that only pseudo-chair H
transition states need be considered.
O Allyl H
BnO 93 : 7 OSiR3
OSiR3
H Cl H
N H

Woerpel's model states that axial attack from the most stable chair N N
conformer predicts the major product. H 2C Cl Cl
Cl N
exclusive adduct
Tet. Lett. 1988, 29, 6593
20-05-cyclic onium addns-2 10/26/01 8:17 AM
D. A. Evans Diastereoselective Oxocarbenium Ion Additions in the Phorboxazole Synthesis Chem 206

Phorboxazole B B: The C-22 Reduction


Evans, Fitch, Smith, Cee, JACS 2000, 122, 10033
Et3SiH
20
OH H
HO B
13 RO2C H H
Et3SiH H
O A OTPS R
O O Me
C BF3•OEt2 O OTPS
N H O Me Me
H H 9
N H R Me
Br H Me
H H
H 4
H CH2 Me
B H O > 95:5 Diastereoselection 91%
HSi
Me
O MeO
Me H
H H
OH N H O H
H Me
R
O OTPS
O O Me
HO
MeO Me
p.. Stereochemical analogies: N H
Me
Kishi et. al.: JACS 1982, 104, 4976-8
Me O
A: The C-11 Reduction
TIPSO
13
A TIPSO
‡ C: The C-9 C–C Bond Construction
O 13
R Et3SiH
9
H O O
H OH R R OTMS OTMS
H BF3•OEt2
H OAc O
H H C Me Me
CH2 Et3SiH H O
R
H 9 O H
O H O
4 TMSOTf
BnOCH2 O
TIPSO BnOCH2
13

O O
>95:5 Diastereoselection R
9
H O C O 89%
H H Me
H Diastereoselection 9 H
H H O
CH2 89:11
R
O
BnOCH2
20-06-phorboxazole cases 10/25/01 5:06 PM
D. A. Evans Carbonyl Addition Reactions: Transition State Geometry Chem 206

■ 4- vs 6-Membered Transition Structures for C=O Addition ■ Do these results relate to "real" reactions? Yes!
Consider carbonyl hydration: R' R' O Zn–R
slow
C O + R Zn R C
H H R
H2C=O + n HOH H2C(OH)2 + (n-1) HOH
Observation: catalytic amounts of ZnI2 dramatically catalyze
H addition process.
O H R
H C H Zn I
O
H R Zn I
H O bimetallic transition state
T1
H O H T2 H C O
+ HOH R
H C O
+42.2 H ■ 4– Versus 6–Center Transition States for Boron

4-Centered
L Me
+6.7 L L
R2C=O Me B
B Me R C
R C OBL2
H2C=O H2C=O L O
+ HOH + 2 HOH R
R
H2C(OH)2
disfavored : rxn does not proceed!)
H2C(OH)2
Schowen J. Am. Chem. Soc. 105, 31, (1983). +HOH 6–Centered

Overall Process: The valure of the proton shuttle CH CH2
L
L R2C=O H 2C B
R C
‡ B L OBL2
H H R
H L C O R
H C O H C O R
C O fast
H H O H O H favored
H
H O
H O H H O
H 6–Centered R R
H H ‡
H O C O R
C CH2 H H 2C
H O H H R
L R2C=O L R
H H O H R B R C
B H OBL2
L
L H R R
C O
Transiton structure T2 ~40 kcal/mol more stable than transition structure T1. R
favored
20-07-C=O addn TS-1 10/30/03 12:25 PM
D. A. Evans Carbonyl Addition Reactions: Transition State Geometry Chem 206
Carbonyl Addition: The Schlenk Equilibrium
4– Versus 6–Center Transition States for Aluminum R R
R Br Br Me Br Br
Me
L ‡ Mg Mg Mg Mg Mg Mg
L R S Br R S R S
Me Al S Br
4–Centered
R C Solution structure of R–MgBr is in dynamic eqkuilibrium through Schlenck equilib
disfavored O Me
R
L The Bimetallic (Binuclear) Mechanism for C=O Addition
R R C
n L OAlL2 Recent theoretical study: Yamazaki & Yambe, J. Org. Chem. 2002, 67, 9346
Al Me C O ‡
L R
L R Al L Me S Me S S ‡
Me Mg Mg
6–Centered Me Al
Me R Me Mg
fast slow Br
R + Br Br Br
favored C O C O Br Br
R R R
R C O
Mg C O Mg Mg
R
Me R Me Me
The Bimetallic Transition States are preferred S L
S
L L Bicyclic TS
L
L
Me Al L Me
L
Me Al Al
L R C Br
R C O L O S
R C O R L Al
R Al Mg
R Al Me Me
Me Me
Me Me
Me R O Br
4-Centered 6-Centered Boat 6-Centered Chair
R Mg
L L Me
L L L
Me Al
Al Swain JACS 1951, 73, 870
L L Yamazaki TS
Bicyclic TS Me Ashby JACS 1974, 89, 1967
R C O O
Al Al
R
Me R C Me The Mononuclear) Mechanism is now in disfavor
Me R Me R
R S
fast C O
C O + Mg R2 R + solvent (S)
S Mg R2
R O MgBr R S
R Br
Grignard Reagents: C O + R2 Mg Br C Br
R R R2 ‡
R R
C O O MgBr
The molecularity and transition structure for this reaction have not been C O slow R R
carefully elucidated. The fact that the Grignard reagent is not a single R C
Mg Br R2 Mg S R
species in solution greatly complicates the kinetic analysis. S R2
R2 Br

20-08-C=O addn TS-2 10/30/03 1:49 PM


D. A. Evans The Evolution of Models for Carbonyl Addition Chem 206

Mengel, A. and O. Reiser, “Around and Beyond Cram's Rule.”


Chem. Rev. 1999, 1191-1223

Fischer Cram Cornforth Felkin Anh/Eisenstein Cieplak Tomoda

Humor provided by Sarah Siska

20-09 C=O addition Models 10/30/03 2:17 PM


D. A. Evans Evolution of a Model for C=O Addition Chem 206

■ Product Development & Steric Approach Control:


Assumptions in Felkin Model:
Dauben, JACS 1956, 78, 2579
H OH
O ■ Transition states are all reactant-like rather than product-like.
[H] OH Me C H
Me3C Me3C 3 ■ Torsional strain considerations are dominant.
Staggered TS conformations preferred
H H H
■ The principal steric interactions are between Nu & R.
% Axial Diastereomer
0 10 20 30 40 50 60 70 80 90 100 RL RL

R C O predicted to be O C R destabilizing
favored TS interaction
H RM H RM
DIBAL-H 72:28 L-Selectride 8:92 ✓
NaBH4 79:21 K-Selectride 3:97 Nu: Nu:
Me
LiAlH(Ot-Bu)3 92:8 – The flaw in the Felkin model: A problem with aldehydes!!
H B C CH2Me
LiAlH4 93:7 M+ H RL RL
3
predicted to be
Observation: Increasingly bulky hydride reagents prefer to attack from the H C O destabilizing O C H favored TS
equatorial C=O face. interaction
H RM H RM
Assumption: Hindered reagents react through more highly developed wrong prediction
transition states than unhindered reagents Nu: Nu:

Carbonyl Addition: Evolution of Acyclic Models


The HOMO-LUMO interaction dictates the
Stereoelectronic Effect:
H following reaction geometry:
R OH R OH
RL Nu: RL
O RL
Nu Nu π∗ C–O
RM RM RM LUMO
favored disfavored δ –Nu
~107 °

R
RL RL RL C Oδ– HOMO
R
R R Nu:
Nu: C R C O
C The Dunitz-Bürgi Angle
OR M H RM
H O RM H π C–O
Nu: Nu:
attack angle greater than 90 °; estimates place it in the 100–110 ° range
Cram Karabatsos Felkin
JACS 1952, 74, 5828 JACS 1967, 89, 1367 TL. 1968, 2199-2208
Burgi, Dunitz, Acc. Chem. Res. 1983, 16, 153-161

20-10-C=O addn/traj 10/30/03 1:52 PM


D. A. Evans The Felkin-Anh Eisenstein Model Chem 206

The flaw in the Felkin model: A problem with aldehydes!!


"The tendency for the staggering of partially formed
RL Houk:
RL vicinal bonds is greater than for fully formed bonds"
predicted to be Lecture–7
H C O destabilizing O C H favored TS
interaction
H RM H RM Lets begin with ground state effects: Ethane Rotational Barrier
wrong prediction
Nu: Nu:

Anh & Eisenstein Noveau J. Chim. 1977, 1, 61-70


∆ G =+3 kcal mol -1
Anh Topics in Current Chemistry. 1980, No 88, 146-162

RL RL
One explanation for the rotational barrier in ethane is that better overlap is
Nu: H OH achieved in the staggered conformation than in the eclipsed conformation.
H C O C
Felkin RM
H H RM
H Nu In the staggered conformation there are 3 anti-periplanar C–H Bonds
RL Nu: favored
O
σ∗ C–H
RM H H
σ* C–H
RL ‡ RL C C
LUMO

H C C
Nu: HO
O C H C σ C–H
H σ C–H
anti-Felkin H RM H RM HOMO
H
Nu
Nu: disfavored
In the eclipsed conformation there are 3 syn-periplanar C–H Bonds
New Additions to Felkin Model:
H H σ∗ C–H
Y X
■ Dunitz-Bürgi C=O–Nu orientation applied to Felkin model. σ C–H σ* C–H
C C HOMO LUMO
C C
■ The antiperiplanar effect:
Hyperconjugative interactions between C-RL which will lower π*C=O σ C–H
will stablize the transition state.
Following this argument one might conclude that:
Theoretical Support for Staggered Transition states (Lecture 7)
■ The staggered conformer has a better orbital match between bonding
(Read this) Houk, JACS 1982, 104, 7162-6 and antibonding states.
(Read this) Houk, Science 1986, 231, 1108-17 ■ The staggered conformer can form more delocalized molecular orbitals.

20-11-C=O Felkin/Anh 10/30/03 1:55 PM


D. A. Evans The Felkin-Anh Eisenstein Model Chem 206

The tendency for the staggering of partially formed Hierarchy of Donor & Acceptor States
vicinal bonds is greater than for fully formed bonds
The following trends are made on the basis of comparing the bonding and
antibonding states for the molecule CH3-X where X = C, N, O, F, and H.
RL Ground State RL Transition State
σ-anti-bonding States: (C–X)
C C C C
CH3 H
Nu X Nu X CH3 CH3
σ* C–RL σ* C–RL
LUMO LUMO CH3 NH2
C C C C CH3 OH
σ C–Nu σ C–Nu
HOMO HOMO CH3 F
Nu Increasing σ∗-acceptor capacity best acceptor
Nu

σ-bonding States: (C–X)


σ∗ C–RL σ∗ C–RL
Under debate
CH3 CH3 CH3 H
σ C–Nu

σ C–Nu CH3 NH2


CH3 OH

Best acceptor σ* orbital is oriented anti periplanar to forming bond decreasing σ-donor capacity CH3 F
poorest donor

RL The following are trends for the energy levels of nonbonding states of several
σ∗ C RL
common molecules. The trend was established by photoelectron spectroscopy.
H C O C Nu
H RM Nonbonding States
Nu
σC RL
H3P:
■ Theoretical support: H2S:
H3N:
Padden-Row, Chem. Commun. 1990, 456; ibid 1991, 327 H2O:
Houk, J. Am. Chem. Soc. 1991, 113, 5018 HCl:
Frenking & Reetz, Tetrahedron 1991, 47, 8091 decreasing donor capacity poorest donor

20-12-C=O Felkin/Anh-2 10/29/03 4:17 PM


D. A. Evans The Felkin-Anh Eisenstein Model: Verification Chem 206

Addition of Enolate & Enol Nucleophiles ■ This trend carries over to organometallic reagents as well

H Nu
RL OH Me Me + Anti-Felkin Isomer
Nu: Me O Me OH
H C O RL
Nu H H
Felkin RM H
H RM H Reagent Ratio
O
Nu: (Felkin) favored H H ClMg C CEt 1:1
RL
H Li Me
Cram 5:1
RM
RL ‡
C. Djerassi & Co-workers, Me
OH
J. Org, Chem. 1979, 44, 3374.
Nu: RL
O C H Nu
anti-Felkin H RM RM H OH
R–Ti (OiProp)3
disfavored + Anti-Felkin Isomer
Nu: O R
Me Me
For Li enolates, increased steric hindrance at enolate carbon
Trend-1: results in enhanced selectivity M. Reetz & Co-workers,
R-Titanium
Angew Chemie Int. Ed.. 1982, 21, 135. Ratio
OLi R = Me >90 : 10
H OH O (R–MgX gives Ca 3:1 ratios) R = n-Bu >90 : 10
R
+ Anti-Felkin Isomer
O R
Me Me
Trend-2: Lewis acid catalyzed reactions are more diastereoselective
Enolate (R) Ratio OSiMe2tBu
L. Flippin & Co-workers,
Tetrahedron Lett.. 1985, 26, 973. R = Me 3:1 H OH O
R2
OLi R = OtBu 4:1 R1 R1 + Anti-Felkin Isomer
O BF3-Et2O R2
Me -78 °C
OMe Me Me
H OH O
R Me R Ratio
+ Anti-Felkin Isomer Ketone (R1) Enolate (R2)
O OMe Ratio Li enolate
Me MeMe Me R = Ph R = Me 10 : 1 3:1
R = Ph R = t-Bu 24 : 1
R = Ph R = OMe 15 : 1
L. Flippin & Co-workers, Ketone (R) Ratio 36 : 1
R = Ph R = Ot-Bu 4:1
Tetrahedron Lett.. 1985, 26, 973. R = Ph >200 : 1 R = c-C6H11 R = Ot-Bu 16 : 1
R = c-C6H11 9:1
C. Heathcock & L. Flippin J. Am. Chem. Soc. 1983, 105, 1667.
20-13-C=O Felkin/Anh-verify 10/29/03 4:32 PM
D. A. Evans The Felkin-Anh Eisenstein Model: Ketone Reduction Chem 206

Addition of Hydride Nucleophiles O HO


Me Me
Me Me

RL H [H] H
OH
Hydride H H
RL + Anti-Felkin Isomer
R C O R
Felkin RM H H
H RM
O
(Felkin) favored Cram Reagent Ratio TS ‡
RL Nu:
R M. M. Midland & Co-workers,
J. Am. Chem. Soc. 1983, 105, 3725.
RM Li+H–B–(sec-Bu)3 54 : 1 Felkin
RL ‡ Felkin
OH NaBH4 5:1
Hydride RL LiAlH4 3:1 Felkin
O C R R H–B(Sia)2 1 : 10 Anti-Felkin
anti-Felkin H RM RM
disfavored Note: Borane reducing agents do not follow the normal trend
Nu:

R O R OH Transition States for C=O-Borane Reductions


+ Anti-Felkin Isomer
M–H
H2 C (CH2)2Ph H 2C (CH2)2Ph
RL ‡
–78 °C
Me Me OH
R2B–H
R C O RL
Ketone (R) Reagent Ratio
R
Felkin H RM
+ – RM
G. Tsuchihashi & Co-workers, R=H Li H–B (sec-Bu)3 96 : 4
Tetrahedron Lett. 1984, 25, 2479.
O H B R (Felkin) disavored
R=H DIBAL 47 : 53
RL R
R = Me Li+H–B–(sec-Bu)3 >99 : 1 R
R = Me DIBAL 88 : 12 RM
RL ‡
OH
R2B–H RL
O OH OH O C R
M–H R
anti-Felkin H RM
Me Me Me R RM
B H
Me Me Me favored
R
Reagent Ratio TS ‡
M. M. Midland & Co-workers,
J. Am. Chem. Soc. 1983, 105, 3725. Li+H–B–(sec-Bu)3 22 : 1 Felkin Nonspherical nucleophiles are unreliable in the Felkin Analysis
H–B(Sia)2 1:4 Anti-Felkin Exercise: Draw the analogous bis(R2BH)2 transition structures

20-14-C=O Cram reductions 10/30/03 1:58 PM


D. A. Evans The Felkin-Anh Eisenstein Model: A Breakdown Chem 206

Are there cases not handled by the Anh-Eisenstein Model? Felkin-Anh analysis predicts B for R = electronegative substituent.
O
HO Me Me OH
Anh-Eisenstein:
"Best acceptor σ* orbital is oriented anti periplanar to forming bond."
Me-Li
σ∗ CSP3–CSP2 is lower in energy than σ∗ CSP3–CSP3 bond.
R R R
R R R
Ph Cyclohexyl A B
(Felkin-Anh Prediction)
H C O H C O (R) Substituent A/B Ratio
H RM H RM G. Mehta, JACS 1990, 112, 6140 O
Nu Nu C OMe >90:10

σ∗ C Cyclohexyl CH2OMe 34:66


σ∗ C Ph
CH=CH2 27:73
Felkin-Anh analysis
predicts the wrong product! CH2–CH3 17:83
σC Cyclohexyl
σC Ph

Case I: O HO H H OH
O
HO Me Me OH NaBH4

Me-Li CO2Me CO2Me CO2Me


CO2Me CO2Me CO2Me
CO2Me CO2Me CO2Me 70: 30
CO2Me CO2Me CO2Me
A B O HO H H OH
M NaBH4
O
Electronegative -CO2Me substituent
Nu will stabilize both
C–C bonding & antibonding states Et Et Et
Et Et Et
NaBH4 39: 61
CO2Me Felkin-Anh analysis predicts B Me–Li 34: 66
CO2Me
G. Mehta, Chem. Commun. 1992, 1711-2:
"These results can be reconciled in terms of the Cieplak model."
20-15-C=O cases not handled 10/30/03 2:02 PM
D. A. Evans The Felkin-Anh Eisenstein Model: A Breakdown Chem 206

Case II: The Le Noble Examples Le Noble, JACS 1992, 114, 1916
Cieplak Model for C=O Addition
O H OH HO H Cieplak, JACS 1981,103, 4540; Cieplak/Johnson, JACS 1989, 111, 8447

NaBH4
Point A: TS is stabilized by antiperiplanar X
N i-PrOH, 25 °C N N
Me Me Me
allylic bond, but....
Felkin-Anh
Ratio, ≥ 95:5 Prediction Point B: Nature of the stabilizing secondary R C O
orbital interactions differ: H R
Pyramidally distorted C=O ruled out from inspection of X-ray structures.
Nu
O σC X σ∗ C Nu
H OH

NaBH4
+ Syn Isomer σ∗ C X σ∗ C X
i-PrOH, 25 °C
σ∗ C Nu
R R (R) Anti:Syn Ratio
Felkin Anh
R=F 62:38
Cieplak
σC Nu
R = CO2Me 61:39
Le Noble, J. Org. Chem. 1989, 54, 3836
R = SiMe3 45:55 σC X σC X
R = OH 43:57
R R
Point C: C–X Electron donating ability follows the order:
C–H > C–C > C–N > C–O
H (Houk disputes the ordering of C–H, C–C)
O
NaBH4 Point D: Importance of torsional effects
+ Syn Isomer
MeOH, 0 °C (Felkin, Anh, Houk, Padden-Row) disputed.
OH

(R) Anti:Syn Ratio "Structures are stabilized by stabilizing their highest energy filled
states. This is one of the fundamendal assumptions in frontier
R = NO2 79:21 molecular orbital theory." The Cieplak hypothesis is nonsense."
R = Cl 63:37
Halterman, JACS 1990, 112, 6690 "Just because a hypothesis correlates a set of observations doesn't
R = OMe 43:57 make that hypothesis correct."
R = NH2 36:64 The management

20-16-C=O cases not handled 10/30/00 8:06 AM


Quotes for the Day

"Every generation of scientists starts where the previous generation left off,
and the most advanced discoveries of one age constitute elementary axioms
of the next."

Aldous Huxley

“It is a capital mistake to theorise before one has data. Insensibly one begins
to twist facts to suit theories, instead of theories to suit facts.”

(Sherlock Holmes, A Scandal in Bohemia)

Quotes-1 11/3/03 9:18 AM


D. A. Evans Carbonyl and Azomethine Electrophiles-2 Chem 206

1. "Theoretical Interpretation of 1,2-Asymmetric Induction. The Importance of


http://www.courses.fas.harvard.edu/~chem206/ Antiperiplanarity", N. T. Anh, O. Eisenstein Nouv. J. Chem. 1977, 1, 61-70. (pdf)

2. “Structural, mechanistic, and theoretical aspects of chelation controlled


Chemistry 206 carbonyl addition reactions.”Reetz, Acc. Chem. Res. 1993 26: 462. (pdf)

3. "A Stereochemical Model for Merged 1,2- and 1,3-Asymmetric Induction in


Advanced Organic Chemistry Diastereoselective Mukaiyama Aldol Addition Reactions and Related Processes."
Evans, et. al. JACS 1996, 118, 4322-4343. (pdf)

Lecture Number 21 4. “The Exceptional Chelating Ability of Dimethylaluminum Chloride and


Methylaluminum Dichloride. The Merged Stereochemical Impact of α- and β-
Stereocenters in Chelate-Controlled Carbonyl Addition Reactions with Enolsilane
Carbonyl and Azomethine Electrophiles-2 and Hydride Nucleophiles”. Evans, Allison,Yang, Masse, 2001, 123,
10840-10852. (pdf)

R R R R R R R Me2AlCl is the most powerful chelating Lewis acid yet documented


C O C O C N C N Me Me
R R R R R Al 1+
O O
2 Me2 AlCl O O
R N O Me2 AlCl2 – (1)
■ Breakdown in the Felkin-Anh Model R N O
■ Cyclohexanone Revisited Bn
Bn
■ Diastereoselective Additions to Cyclic Ketones
"Asymmetric Diels-Alder Cycloaddition Reactions with Chiral α,β-Unsaturated-N
■ Chelate Controlled Carbonyl Additions Acyloxazolidinones". Evans, D. A.; Chapman, K. T.; Bisaha, J. J. Am. Chem.
Soc. 1988, 110, 1238-1256.
■ Reading Assignment for this Week:
Syn Diastereomer: & Centers Reinforcing
Carey & Sundberg: Part A; Chapter 8 M P
Reactions of Carbonyl Compounds O OP Me2AlCl O O OH OP
Nu
(2)
Carey & Sundberg: Part B; Chapter 2 H Rβ H Rβ Nu Rβ
Reactions of Carbon Nucleophiles with Carbonyl Compounds
Rα A Rα Rα
Carey & Sundberg: Part B; Chapter 5
Reduction of Carbonyl & Other Functional Groups M P
O OP O O OH OP
Me2AlCl Nu
(3)
Monday, H Rβ H Rβ Nu Rβ
D. A. Evans November 3, 2003 Rα B Rα Rα

Anti Diastereomer: & Centers Opposing


21-00-Cover Page 11/3/03 9:33 AM
D. A. Evans The Felkin-Anh Eisenstein Model: A Breakdown Chem 206

Are there cases not handled by the Anh-Eisenstein Model? Felkin-Anh analysis predicts B for R = electronegative substituent.
O
HO Me Me OH
Anh-Eisenstein:
"Best acceptor σ* orbital is oriented anti periplanar to forming bond."
Me-Li
σ∗ CSP3–CSP2 is lower in energy than σ∗ CSP3–CSP3 bond.
R R R
R R R
Ph Cyclohexyl A B
(Felkin-Anh Prediction)
H C O H C O (R) Substituent A/B Ratio
H RM H RM G. Mehta, JACS 1990, 112, 6140 O
Nu Nu C OMe >90:10

σ∗ C Cyclohexyl CH2OMe 34:66


σ∗ C Ph
CH=CH2 27:73
Felkin-Anh analysis
predicts the wrong product! CH2–CH3 17:83
σC Cyclohexyl
σC Ph

Case I: O HO H H OH
O
HO Me Me OH NaBH4

Me-Li CO2Me CO2Me CO2Me


CO2Me CO2Me CO2Me
CO2Me CO2Me CO2Me 70: 30
CO2Me CO2Me CO2Me
A B O HO H H OH
M NaBH4
O
Electronegative -CO2Me substituent
Nu will stabilize both
C–C bonding & antibonding states Et Et Et
Et Et Et
NaBH4 39: 61
CO2Me Felkin-Anh analysis predicts B Me–Li 34: 66
CO2Me
G. Mehta, Chem. Commun. 1992, 1711-2:
"These results can be reconciled in terms of the Cieplak model."
21-01-C=O cases not handled 11/2/03 12:33 PM
D. A. Evans The Felkin-Anh Eisenstein Model: A Breakdown Chem 206

Case II: The Le Noble Examples Le Noble, JACS 1992, 114, 1916
Cieplak Model for C=O Addition
O H OH HO H Cieplak, JACS 1981,103, 4540; Cieplak/Johnson, JACS 1989, 111, 8447

NaBH4
Point A: TS is stabilized by antiperiplanar X
N i-PrOH, 25 °C N N
Me Me Me
allylic bond, but....
Felkin-Anh
Ratio, ≥ 95:5 Prediction Point B: Nature of the stabilizing secondary R C O
orbital interactions differ: H R
Pyramidally distorted C=O ruled out from inspection of X-ray structures.
Nu
O σC X σ∗ C Nu
H OH

NaBH4
+ Syn Isomer σ∗ C X σ∗ C X
i-PrOH, 25 °C
σ∗ C Nu
R R (R) Anti:Syn Ratio
Felkin Anh
Cieplak
R=F 62:38
σC Nu
R = CO2Me 61:39
Le Noble, J. Org. Chem. 1989, 54, 3836
R = SiMe3 45:55 σC X σC X
R = OH 43:57
R R
Point C: C–X Electron donating ability follows the order:
C–H > C–C > C–N > C–O
H (Houk disputes the ordering of C–H, C–C)
O
NaBH4 Point D: Importance of torsional effects
+ Syn Isomer
MeOH, 0 °C (Felkin, Anh, Houk, Padden-Row) disputed.
OH

"Structures are stabilized by stabilizing their highest energy filled states. This
(R) Anti:Syn Ratio
one of the fundamendal assumptions in frontier molecular orbital theory." The
Cieplak hypothesis is nonsense."
R = NO2 79:21
R = Cl 63:37 "Just because a hypothesis correlates a set of observations doesn't make that
Halterman, JACS 1990, 112, 6690 hypothesis correct." The management
R = OMe 43:57
“It is a capital mistake to theorise before one has data. Insensibly one
R = NH2 36:64 begins to twist facts to suit theories, instead of theories to suit facts.”
(Sherlock Holmes, A Scandal in Bohemia)

21-02-C=O cases not handled 10/31/03 11:28 AM


D. A. Evans The Cyclohexanone Case Revisited: Frenking & Houk Chem 206

Observation As R becomes more electronegative, percentage of Houk: Electrostatic rather than covalent considerations may be
axial attack increases. dominant.
Me OH
H O H H
Me–Metal "Equatorial electronegative substituents should interact more strongly with the C(2-3) and C(9-10)
OH Me bonds than axial substituents."
H
H OH
R H R R O NaBH4
% Axial Attack OH H
R MeOH, 25 °C R R
(R) Substituent Me-Li Me2Cu-Li 3
2
H H H
R=H 21% 6% (R) Substituent Product Ratio
R = C6F5 34% 21%
R=H 60:40
R = CF3 50% 42% R = OAc 71:29

Felkin-Anh predicts opposite trend. R = Cl 71:29


Cieplak argument consistent with results. "If nucleophilic addition occurs anti to the better donor bond (Cieplak), the equatorial isomers
should have considerably more axial attack than the parent while the axial isomers should have
only a slight increase in axial attack."
The Frenking Position: H OH
O NaBH4
OH H
H MeOH, 25 °C H H
■ Cieplak stabilizing interaction is "dubious." Why not stabilize the
forming sigma bond? R R R

■ Enhanced rate of axial Nu attack on cyclohexanone is caused by (R) Substituent Product Ratio
better electrostatic interactions of the ketone with the attacking R = OAc 83:17
reagent and not by torsional considerations. "Exactly the opposite is observed."
R = Cl 88:12
■ Nonequivalence of the π*C=O LUMO with a greater extension on
the axial face dictates stereoselection (Klein, 1973).
Axial 4-substituents favor axial attack for electrostatic reasons:
"Since interactions between the π C=O & π∗ C=O and the bonding &
anti-bonding (β) C–H & (β) C–C orbitals are all symmetry allowed, "Disfavored" "Favored" δ – Nu M
it is difficult to predict a priori which interactions are dominant without O
M
carrying out quantum mechanical calculations." O
H δ+ Nu H δ+
δ–
δ–R δ –R
Frenking & Reetz, Angew. Chem. Int. Ed. 1991, 30, 1146
(DAE: Bimetallic transition states were not considered)

K. Houk & Co-workers, J. Am. Chem. Soc. 1991, 113, 5018


21-03-C=O Frencking/Houk 10/31/03 11:31 AM
D. A. Evans The Felkin-Anh Eisenstein Model: Electronic Effects Chem 206

The Polar Felkin-Anh Model


Are there electronic effects in the reaction?
Premise: Transition state hyperconjugation between forming bond (HOMO)
Several cases have already been presented which may be relevant and best antiperiplanar acceptor (σ* C–X, LUMO). Steric effects alre also
considered; X = Halogen, OR, SR etc
L. Flippin & Co-workers, Tetrahedron Lett.. 1985, 26, 973.
X ‡
OH
H OH O
R H C R
O Nu
OMe + Anti-Felkin Isomer
O H R
Me X
Me OLi Me Me
O
Nu favored
Me Ratio 9 : 1 R
OMe R Nucleophile
X
Me X ‡
OH O OH
H
R O C H R
OMe Nu
O + Anti-Felkin Isomer R
Me H X
Me Me Me
Nu disfavored
Ratio > 200 : 1
The molecular volume occupied by cyclohexyl acknowledged to be larger
than that for phenyl. Because of shape phenyl "can get out of the way."
Modified Cornforth Model
Premise: Transition State dipole minimization between polar C–X substituent
■ Anh-Eisenstein Explanation based on HOMO-LUMO Analysis: and the transforming carbonyl function dictate preferred TS geometrics. Steric
"Best acceptor σ* orbital is oriented anti periplanar to forming bond." effects alre also considered; X = Halogen, OR, SR etc
σ∗ CSP3–CSP2 is lower in energy than σ∗ CSP3–CSP3 bond. R ‡
OH
Ph Cyclohexyl R
H C O Nu
X H
H C H C X
O O
O
H H Nu favored
RM RM
R
R Nucleophile
Nu Nu
R
X ‡
OH
σ∗ C Cyclohexyl
σ∗ C C H R
Ph O
Nu
H
X X
Nu disfavored
σC Cyclohexyl
σC Ph
Both models lead to the same stereochemical prediction.
21-04-Felkin-anh Polar Model 11/2/03 12:35 PM
D. A. Evans Carbonyl Addition Reactions: Chelate Organization Chem 206

Chelate organization also provides a powerful control element in


carbonyl addition reactions O SnBu3 OH OH
C6H11 H C C6H11
R Nu R Nu H Lewis Acid H5C3 5 3
R
R R + R OR OR OR
O Nu–M H
O OH
Chelate Felkin: RL=OR
O M O O
R M R
R
(OR) Acid Solv. Ratio
R M R
R M O O O OH R = CH2OBn MgBr2 THF (0°) 20 : 80
O O Nu–M Nu H+ Nu
R = CH2OBn MgBr2 CH2Cl2 (-20°) >99 : 1
R R R R = CH2OBn TiCl4 CH2Cl2 (-78°)
R R R >99 : 1
R = SiMe2(t)Bu BF3-Et2O CH2Cl2 (-78°) 5 : 95
Reviews Reetz, Accts. Chem. Res. 1993, 26, 462-468 (pdf)
Reetz, Angew. Chem. Int. Ed. 1984, 23, 556-569 G. Keck & Co-workers, Tetrahedron Lett. 1984, 25, 265

Me
Me Me
Addition of Carbon Nucleopiles Me
TBSO MgBr
OR ‡ O TBSO
OH OH
Chelate Model
path A OCH2OBn
H C O RL OCH2OBn
Nu diastereoselection 50 : 1
H R
O OR
Me
R Nu: Felkin: RL=OR Me
H TBSO Me-MgBr Me
OR O TBSO
R ‡ Chelate Model OH
OH OCH2OBn
path B OCH2OBn
O C H R W. C. Still & Co-workers,
Nu Tetrahedron Lett. 1980, 21, 1031 diastereoselection >100 : 1
RO H
OR
Nu:
Chelation model MgBr
R ‡ H Me Me Me
OH O O
path C H Et THF H Et
M O C H R O OBn Me OH OBn
Nu
O H "only one isomer"
OR
R Y. Kishi & Co-workers, Chelate Model
Nu: Chelate Tetrahedron Lett. 1978, 19, 2745

21-05-Chelation VS PFA-1 11/2/03 4:29 PM


D. A. Evans Chelate Organization in C=O Addition Chem 206

Chelate organization provides a powerful control element in


carbonyl addition reactions O OBn O OH OBn
TiCl4
O H Me3C
OH OH
Me LiAlH4 Me Me Me Me
Me OTMS
-10 °C R R
Me OR
diastereoselection 95 : 5
OR OR Me3C (Chelation)
Polar Felkin Anh Chelation
O OTBS O OH OTBS
TiCl4
(OR) Solv. Ratio Model Me3C
H
Overman R = CH2OBn THF 30 : 70 Chelate Me Me
Tet Lett. 1982, 23, 2355 Et2O 2 : 98 Chelate
R = CH2OBn
diastereoselection 93 : 7
R = SiPh2(t)Bu THF 95 : 5 F–A: RL=OR (Felkin)
Me Me 1+
2 Me2AlCl TBS
Degree of chelate organization may be regulated by choice of solvent Al
O O O OH OBn
and protecting group. Note that SiPh2(t)Bu group prevents chelation.
H Me3C
See Lecture 17
slide 03 for this Me Me
Case Study
Lewis acid Me2AlCl2– diastereoselection 97 : 3
Nu: ‡
OH OP (Chelation)
(M) Me H Nu:
Felkin M–O C H Me2AlCl & MeAlCl2 only Lewis acids that will chelate strongly to OSiR3 Groups.
Control Nu
Evans, Allison, Yang,Masse, JACS 2001, 123, 10840–10852
Me
D CH2OP
O OP 1,2-Syn (Felkin) O OP OTMS O OH OP O OH OP
H Lewis acid
t t t
(M) H Bu Bu Bu
Me ‡ Lewis Acid
P Me Me Me
1, P = Bn O Me CH2Cl2
2, P = TBS Chelation Felkin
H
M C H 1 P = Bn 9 10
O OH OP 2 P = TBS 11 12
Chelate (M)
Control Nu: Nu 1 P = Bn 2 P = TBS
entry Lewis acidb 9 : 10 (%) 11 : 12 (%)
Me
Me 26 : 74 (76)
M O C 1,2-Anti (Chelation) A BF3•OEt2 09 : 91 (55)
H B SnCl4 50 : 50 (87) 07 : 93 (41)
P O H C TiCl4 97 : 03 (74) 07 : 93 (55)
Nu: D Me2AlCl 90 : 10 (45) 97 : 03 (62)c
E MeAlCl2 78 : 22 (70) 77 : 23 (55)
21-06-Chelation VS PFA-2 11/2/03 8:57 PM
D. A. Evans Carbonyl Addition Reactions: Chelate Organization Chem 206

Kinetic Evidence for Chelate-Controlled C=O Additon Alpha–Versus Beta-Chelation

R R Me
Since much more reactive than Me Me
O M O Me
R R TBSO MgBr
O TBSO
Substrates which can participate in C=O chelation will be more reactive OH
Chelate Model
since the effective concentration of chelated intermediate will be higher. OCH2OBn
OCH2OBn
W. C. Still & Co-workers,
R Tetrahedron Lett. 1980, 21, 1031 diastereoselection 50 : 1
Ketone + R–M O M product
R +
R–M
Me H R-M Me R
+ isomer
O OMgX -78 °C
Me2Mg R rel rate BnO O Chelate Model BnO OH
OR OR
Me k2 Me
Me –Me 213
–Bn 174 M. T. Reetz & Co-workers
J. Am. Chem. Soc.. 1983, 105, 4833. R-M Solv. Ratio
O OMgX –CMe3 9
Me2Mg Me-MgCl THF 40 : 60
Bu Bu –SiMe3 7
Me Me Other nucleophiles reported Me-TiCl3 CH2Cl2 90 : 10
k1 Me
reference rxn –Si-i-Pr3 1
rxn run inTHF at - 78°C
Eliel, Frye, JACS 1992, 114, 1778-84 (read)
Me H SiMe3 Me
+ isomer
However, these trends are not transmitted strongly to β-chelation
Acid -78 °C
BnO O BnO OH
O CH2Cl2
Me2Mg Chelate Model
Me OBn k1 OMgX
k1 M. T. Reetz & Co-workers
= 2.5 Tetrahedron Lett. 1984, 25, 729. Acid Ratio
O
Me OR k2 TiCl4 95 : 5
Me2Mg Me
Me OSi(iPr)3 k2
SnCl4 95 : 5
BF3-OEt2 85 : 15

R
R M ■ Note that beta chelation can be developed as a control element by
R O O varying solvent & Nu.
Hence, organization O better than
through O M R R ■ Note BF3 gives "apparent" chelate control
R

21-07-More chelation 10/31/03 2:41 PM


D. A. Evans Carbonyl Addition Reactions: Chelate Organization Chem 206

1,3-Stereoinduction Models for Chelate & non-Chelate Rxns 1,3-Stereoinduction Polar Model:
1,3-Stereoinduction Polar Model: OTMS
Evans, Dart, Duffy,Yang, JACS 1996, 118, 4322–4343 O OH OP O OH OP
O OP (8)
R R iPr R iPr
1,3-Stereoinduction Chelate Model: H iPr
Evans, Allison, Yang,Masse, JACS 2001, 123, 10840–10852 BF3•OEt2 1,3-Anti 1,3-Syn
1 P = PMB CH2Cl2 3 4
Nu: ‡ 2 P = TBS 5 6
(M) Ha Hb Nu:
1,3-Anti 3:4 5:6
H C O–M entry R
(P = PMB) (%) (P = TBS) (%)
C A t-Bu 89 : 11 (82) 84 : 16 (79)
PO H B i-Pr 92 : 08 (91) 80 : 20 (84)
O OP R OH OP
E C Me 91 : 09 (89) 93 : 07 (87)
H β R H Nu β R
R ‡
P Steric effects appear to play a minor role in stereoinduction:
O
C OTMS O Me 1,3-Anti (58%)
(M) M O H BF3•OEt2
F1 Nu:
1,3-Anti (Chelation) 1,3-Syn (42%)
Me2CH H CH2Cl2
Nu: Me Me (88%)

P O
M O C H
F2 R

Nu: H
O OP MLn OH OP OH OP
1,3-Anti Relationship is favored by either polar or chelate models
H Me Nu Me Nu Me
OTMS P = Bn (benzy) 1,3-Anti 1,3-Syn
O OBn O OH OBn P = MOM (methoxymethyl)
Ph
H Me TiCl4 Ph Me (P = Bn) (P = MOM)
entry conditions anti : syn anti : syn
diastereoselection 92 : 8
A SiMe3 BF3•OEt2 85 : 15
OTMS
O OPMB i O OH OPMB B SnPh3 BF3•OEt2 79 : 29
Pr
i C SnMe3 BF3•OEt2 70 : 30
H R BF3•OEt2 Pr R
R = (CH2)2Ph diastereoselection 81 : 19

21-08-beta–OR Model 11/2/03 6:03 PM


D. A. Evans Carbonyl Addition Reactions: Chelate Organization Chem 206

1,3-Stereoinduction Polar Model The Anti Diastereomer


Evans, Dart, Duffy,Yang, JACS 1996, 118, 4322–4343
O OP OTMS OH OP OH OP

Can one develop a Rational model for α & β Stereocenters? H iPr R Nu iPr Nu iPr
Me BF3•OEt2 Me Me
O OR O OH OR
CH2Cl2 Felkin Anti-Felkin
BF3•OEt2
H R R R 13 P = PMB 15 16
✻ 14 P = TBS
Me OTMS 17 18
Me
15 : 16 17 : 18
R entry R
O OH OR (P = PMB) (P = TBS)
O OR
A t-Bu 99 : 01 99 : 01
BF3•OEt2
H R R ✻ R B i-Pr 98 : 02 95 : 05
Me Me C Me 97 : 03 71 : 29

Which of the two stereochemical representations is reinforcing? Non-reinforcing?


The Syn Diastereomer
Integration of 1,3- Polar Model & Felkin-Anh Model
OTMS
O OP OH OP OH OP
Nu ‡
R
H iPr Nu iPr Nu iPr
O OPMB Ha Hb BF3•OEt2
F3B–O C H O OH OPMB Me Me Me
solvent Felkin
H β R Anti-Felkin
C R β iPr 19 P = PMB 21 22
H OP 20 P = TBS 23 24
OTMS iPr
BF3•OEt2 21 : 22 23 : 24
R entry R Solvent (P = PMB) (P = TBS)
‡ O OH
Nu
O A t-Bu CH2Cl2 96 : 04 96 : 04
α RL
α RL Me H R B t-Bu toluene 88 : 12 94 : 06
H F3B–O C H C i-Pr CH2Cl2 56 : 44 87 : 13
Me
Me D i-Pr toluene 32 : 68 75 : 25
RL
Nu ‡ E Me CH2Cl2 17 : 83 58 : 42
O OR F Me toluene 06 : 94 40 : 60
Me Hb
If both models are correct, they α
should integrate when the two F3B–O C H Conclusions
H β R
stereocenters are reinforcing A: Anti diastereomer is reinforcing. Both models integrate.
C Me
H OP B: Syn diastereomer transitions from Felkin control (Large Nu) to 1,3-control
iPr α & β reinnforcing? (Small Nu).

21-09-Merged Model-1 11/2/03 5:52 PM


D. A. Evans Reinforcing & Non-reinforcing Relationsips Chem 206

The Anti Diastereomer: Both Centers Reinforcing


O OPMB OH OPMB

R H iPr iPr Felkin


O OPMB Bu3Sn OH OPMB Me TBSO Me
Bu3Sn
H iPr R β iPr Diastereoselection > 99 : 1
BF3•OEt2 OTBS
Me Me CH2Cl2
entry Conditions R Felkin : anti-Felkin O OPMB OH OPMB OH OPMB

A BF3•OEt2, toluene Me >99 : 1 H iPr iPr iPr


B BF3•OEt2, toluene H >99 : 1
Me TBSO Me TBSO Me
C Ph3CClO4, CH2Cl2 H >99 : 1 Felkin Anti-Felkin
Diastereoselection 59 : 32 : 9a
a
The third unpictured product is the Felkin-3,4-anti diastereomer.

The Syn Diastereomer: Stereocenters are Non-reinforcing The Syn Diastereomer: Stereocenters are Non-reinforcing

O
O OP OH OP OH OP
R Me
R H iPr Nu iPr Nu iPr
LDA, THF
O OPMB Bu3Sn OH OPMB Me Me Me
Felkin Anti-Felkin
H iPr R β iPr 19 P = PMB 21 22
20 P = TBS 23 24
Me Me
21 : 22 23 : 24
entry Conditions R Felkin : anti-Felkin entry R (P = PMB) (%) (P = TBS) (%)
A BF3•OEt2, toluene Me 20 : 80
BF3•OEt2, toluene A t-Bu 11 : 89 (71) 08 : 92 (91)
B H 13 : 87
B i-Pr 14 : 86 (95) 13 : 87 (64)
C Ph3CClO4, CH2Cl2 H 62 : 38 C Me 22 : 78 (73) 14 : 86 (88)
In this example, the OR substituent is the dominant stereo-control element

21-10-Merged Model-2 11/2/03 8:19 PM


D. A. Evans Carbonyl Addition Reactions: Chelate Organization Chem 206

Beta Chelation with Organometals OTBS OTBS OH


CHO
Me Me O Me
Me M + isomer
H Me-M Me O N O O XV
+ isomer
-78 °C
BnOCH2O O BnOCH2O OH Metal Ratio
R-M Ratio
Me2CH
Chelate Model M = MgCl 70 : 30
MeMgBr 50 : 50 D. A. Evans & E. Sjogren M = ZnCl 97 : 3
Tetrahedron Lett. 1986, 27, 4961.
Me2CuLi 97 : 3
Me Me
Me
H Me2CuLi O Me Me
O + isomer Me O
-78 °C Et2O RO O R RO OH
O O O OH O
Chelate Model TBSO H Me
diastereoselection > 95 : 5 R = BOM Me Me Me
(CH2OBn) Me
Me Me
Me O
Me Me D. A. Evans and S.L. Bender O Metal Ratio
Me H Me2CuLi
+ isomer J. Am. Chem. Soc.. 1988,
-78 °C Et2O Me M = [CuCN]1/2 98 : 2
OH in press.
BnOCH2O O BnOCH2O 33 : 67
M = Li
Chelate Model M Me
diastereoselection > 95 : 5
Me Me OTMS
Me
Me H Me2CuLi Me Me Me
+ isomer Me H Ph Me Ph
-78 °C Et2O + isomer
BnOCH2O O BnOCH2O OH TiCl4 -78 °C
BnO O CH2Cl2 BnO OH O
Chelate Model
diastereoselection 70 : 30 Chelate Model

M. T. Reetz & Co-workers diastereoselection >92 %


Me Me Tetrahedron Lett. 1984, 25, 729.
Me H Me2CuLi + isomer
-78 °C Et2O O
O BnOCH2O OH OH
BnOCH2O
CO2R TiCl4
H CO2R
O Me2Zn Me O
W.C. Still & Co-workers, diastereoselection 50 : 50 H H H H
Tetrahedron Lett. 1980, 21, 1035.
Me Me
chelate model
S. W. Baldwin & Co-workers
J. Org. Chem. 1987, 52, 320. diastereoselection 96 : 4

21-11-Chelation beta-2 10/31/00 8:58 PM


D. A. Evans Carbonyl Addition Reactions: Chelate Organization Chem 206

Beta Chelate-Controlled Reduction Directed reductions of β-hydroxyketones

Me Me Evans, Chapman, Carreira, JACS 110, 3560 (1988)


Ph Zn(BH4)2 Ph
Et2O, 0 °C
+ isomer ‡
OH O H H OAc
MeO O MeO OH O+
91-99% OH OH
diastereoselection 97 : 3 –
Chelate Model R1 B favored
T. Oishi & Co-workers R1 R2 O OAc
H C R1 R2
Chem. Pharm Bull. 1984, 32, 1411. R3 H
R R2 R3
Me4NBH(OAc)3 3
Me Me

Ph Ph LiAlH4 Ph Ph H OAc OH OH
+ isomer R2
Et2O, 0 °C –
NH2 O NH2 OH R1 B
91-99% O OAc R1 R2
diastereoselection 88 : 12 H C
Chelate Model H R3
J. Barluenga & Co-workers
J. Org. Chem. 1985, 50, 4052. R3 O H

O O OH OH O OH OH
OH
Me Me NaBH(OAc)3
M-H Me Me + isomer
OEt OH HOAc, -20 °C
OH
C 3H 5 C 3H 5 Me Me Me Me
C 3H 5

G. R. Brown & Co-workers diastereoselection 96 : 4


M-H Ratio
Chem. Commun. 1985, 455.
Zn(BH4)2 Et2O 100 : 0 OH O OH OH
0 : 100 Me Me NaBH(OAc)3
LiAlH4 THF Me Me + isomer
HOAc, -20 °C
O O Me Me Me
CH2MOM OH O OH O Me Me Me
Me
N Me Me
M-H XC XC
diastereoselection 98 : 2
Me Me OH O
Me Me Me Me OH OH
MOMCH2 NaBH(OAc)3
Me Me Me Me + isomer
M. Yamaguchi & Co-workers HOAc, -20 °C
Tetrahedron Lett. 1985, 26, 4643. M-H Ratio Me Me Me Me Me Me
T. Oishi & Co-workers Zn(BH4)2 Et2O 100 : 0
Tetrahedron Lett. 1980, 21, 1641 KBH3H THF 0 : 100 diastereoselection 98 : 2
(Zn(BH4)2 on esters.
Bu Bu
Propose a mechanism for tihs highly diastereoselective transformation,
B Evans, Hoveyda JACS 112, 6447 (1990)
OH O O O OH OH
NaBH4 O
Ph Ph Ph Ph Ph Ph + isomer OH O
-100 °C R2 O OH
Me R2CHO
K. Narasaka & Co-workers R1 Me diastereoselection
diastereoselection 96 : 4 15% SmX3
Chem. Lett. 1980, 1415. catalyst R1 > 100:1
Me
Me
21-12-Chelation beta-3 11/1/00 9:00 AM
D. A. Evans Remote Heteroatom Effects in Carbonyl Addition Chem 206

Me Me Me O O O
OH OH OH O O
O OH OH
M-H COOH M-H
C +
H H H Me Me Me
Me Me O Me Me O Me Me O
Temp. & Solvent not specified in this study
in first three cases Reagent Ratio Reagent Ratio

Li-AlH4 40 : 60 NaBH2(OR)2 50 : 50
R. Baker & Co-workers R. Frenette & Co-workers
J. Org. Chem. 52, 304 (1987) NaBH4 70 : 30
Chem. Commun. 1984, 74 Na-BH4 50 : 50
DIBAL (2.4 equiv) 95 : 5
Zn-BH4 60 : 40 DIBAL (2.4 equiv) 99 : 1
Li-BHEt3, (THF, -78˚C) with ZnCl2
100 : 0

Me Me Me
Me Me
Et Et Et Me
O OH OH M-H
O O O
M-H COOMe Ph O O Ph O O
+ Ph O Me Me

OMe OMe OMe


Solvent Ratio
R. Frenette & Co-workers
Y. Kishi & Co-workers J. Org. Chem. 52, 304 (1987) THF 50 : 50
Tetrahedron Lett. 1978, 2741. Li-BH(R)3, Et2O, 25˚C 1:1
C 6H 6 70 : 30
LiAlH4, Et2O, 25˚C 3:1
* di-2-(o-toluidinomethyl)pyrrolidine LiAlH4-diamine,* -78˚C 11 : 1 OH
OH
O Me–M
t-BuPhMe2SiO Me + Isomer
OCH2OCH3 OCH2OCH3 OCH2OCH3 -78 °C
Me Me Me t-BuPhMe2SiO
OH
M-H
+ 67-87% yields
O HO HO Reagent Solvent Ratio
MeLi Et2O 1.7 : 1
Li-BH(R)3, 73 : 27 G. Tsuchihashi & Co-workers MeMgBr THF 1.3 : 1
Tetrahedron Lett. 1987,28, 6335.
LiAlH4 76 : 24 Me3Al CH2Cl2 1.1 : 1
MeTiCl3 CH2Cl2 8.4 : 1
MeTi(O-iProp)3 CH2Cl2 12 : 1
OCH2OCH3 OCH2OCH3 OCH2OCH3
Me Me Me
M-H OH
+
MeTi(O-iProp)3 OH OH
O HO HO O
+ Isomer
0 °C R Me
Li-BH(R)3, 28 : 72 R
W. G. Dauben & Co-workers Ratio 4-5.5 : 1
Tetrahedron Lett. 1978, 2741. 72 : 28 R = Me, Ph(CH2)2
LiAlH4

21-13-C=O ADDN (REMOTE-OR) 10/31/00 9:00 PM


Quote for the Day
Richard P. Feynman (as an undergraduate) in Surely you're Joking Mr. Feynman

"When I was an undergraduate student at MIT I loved it. I thought it was a great
place, and I wanted to go to graduate school there too of course. But when I went
to Professor Slater and told him of my intentions, he said,

'We won't let you in here.'

I said, "what"?

Slater asked, 'Why do you think that you should go to graduate school at MIT'?

"Because it is the best school for science in the country."

'You think that'?

"Yeah."

'That's why you should go to some other school. You should find out how the rest of
the world is.'

Quotes-2 11/5/03 9:11 AM


Experimental Support for Cornforth or Felkin–Anh Models
Polar Felkin-Anh Model Cornforth Model
L R' L R'
H O OH H
M O
L M O L
H 2 4 R H
O R' 3 O
X H R X
RZ RZ X RZ
H
I R matched for Cornforth Model III
syn-pentane

L R' L R'
H O OH H
M O M O
L L
RE 2 4 R RE
O R' 3 O
X H R X
H RE X H
R H syn-pentane
II matched for polar Felkin-Anh Model IV
PFA Model Prediction: The (Z) enolate substituent causes a destabilizing syn-pentane interaction (I),
while the (E) enolate substituent experiences no such interaction (II). Therefore, (E) enolates are
predicted to give superior 3,4-anti selectivity relative to (Z) enolates.

Cornforth Model Prediction: The (E) enolate substituent causes a destabilizing syn-pentane interaction
(IV) while the (Z) enolate substituent experiences no such interaction (III). Therefore, (Z) enolates are
predicted to give superior 3,4-anti selectivity relative to (E) enolates. Due to this dichotomy, the
experimentally determined relationship between enolate geometry and aldehyde diastereofacial selectivity
should validate a single model for asymmetric induction.
Angew. Chem. Int. Ed., 2003, 42, 1761-1765.
CornforthSupport 11/5/03 9:27 AM
D. A. Evans Diastereoselective & Enantioselective Carbonyl Addition Chem 206

■ Relevant Problems:
http://www.courses.fas.harvard.edu/~chem206/
Database Problem 27: Chiral amino alcohol 1 efficiently mediates the addition of
diethylzinc to aromatic aldehydes. While a number of other amino alcohols are also
effective in controlling the absolute course of the addition process, this amino alcohol has
Chemistry 206 been the focus of a recent computational investigation that addresses the preferred
transition state geometry for this addition process (Pericas, et al. J. Org. Chem. 2000, 65,
7303 and references cited therein). It should be noted that, while 1 is not the actual catalyst,
Advanced Organic Chemistry it is modified under the reaction conditions to the competent catalytic agent. Provide a
detailed mechanism for the overall transformation. Use 3-dimensional representations to
illustrate the absolute stereochemical aspects of the indicated transformation.

Lecture Number 22
O OH

N 0.06 equiv 1
Et
Enantioselective Carbonyl Addition Ph
H
Et2Zn, 0 °C
(S) 97% ee

■ Enantioselective addition of R2Zn to aldehydes Ph (R) Ph toluene


1
■ Enantioselective Reduction of Ketones & Imines OH
■ Introduction to Enolate-based Nucleophiles Cume Question, 2000: Corey's introduction of chiral oxazaborolidine catalysts 1 in the
borane-mediated enantioselective reduction of ketones represents an important advance in
asymmetric synthesis (Corey & Helal, Angew. Chem. Int. Ed. 1998, 37, 1986-2012).
Provide a detailed mechanism for the overall transformation. Use 3-dimensional
representations to illustrate the absolute stereochemical aspects of the indicated
■ Reading Assignment for this Week: transformation.
Ph OH
H Ph O
Carey & Sundberg: Part A; Chapter 8
Reactions of Carbonyl Compounds O 0.1 equiv 1 (R) Me
N B Me 97% ee
Carey & Sundberg: Part B; Chapter 2 1 equiv BH3•THF
Reactions of Carbon Nucleophiles with Carbonyl Compounds Me
1
Carey & Sundberg: Part B; Chapter 5
Reduction of Carbonyl & Other Functional Groups
Database Problem 151: The following stereoselective transformation has been reported by
Fujisawa (Chem.Lett. 1991, 1555). Given the structure of the product, rationalize the
stereochemical outcome of the process.
Enantioselective Carbonyl Reduction: Corey Angew. Chem. Int Ed. 1998, 37,
1986-2012 (handout)
Ph Ph
Enantioselective Carbonyl Addition (R2Zn): Noyori Angew. Chem. Int Ed. OMe R"-Li OMe
1991, 30, 49-69 (handout) N HN dr> 97:3

R H R R"
Wednesday,
D. A. Evans November 5, 2003

22-00-Cover Page 11/5/03 8:51 AM


D. A. Evans Enantioselective C=O Addition: Noyori Catalyst Chem 206

Catalytic Asymmetric Carbonyl Addition The Catalytic Cycle


Me Me
R Me H
O
Zn I replace with chiral controller C
N Me Me Me Me Me
R Zn I Zn Me Me
O Et
H C Me H N Me N Me Et
O H Zn
R
the catalyst Me H

O Et ●
O
Zn
Noyori & co-workers, J. Am. Chem. Soc. 1986, 108, 6072. Me H
H O H
J. Am. Chem. Soc. 1989, 111, 4028. ● H
C
Review: Noyori Angew. Chem. Int. Ed. 1991, 30, 49
Review: L. Pu, Chem. Reviews 2001, 101, 757-824
Zn–Et
O
H Et2Zn
90-98% ee
Et
Zn
Me2Zn
Zn
PhCHO
p.
p. Me Me Me Me
Me Me Me
N Me Et N Me Et

00000
00
Zn Zn RDS Zn

O ●
O

00000
Me H Me H
O H O H

00
0000
(DAIB-Zn) H H
Zn C Zn C
OH Et Et Et
the catalyst Et
Ar–CHO + R2’Zn
0°C, toluene Ar R’
59 - 97%

C6H5CHO Et2Zn 98% e.e. ■ Catalyst must be sterically hindered so that association is precluded
" Me2Zn 91% e.e.
p-ClC6H4CHO Et2Zn 93% e.e.
Et2Zn R R
p-MeOC6H4CHO 93% e.e. Zn O R'
● R'
Et2Zn Zn ●
O
Cinnamyl 96% e.e. R' R
Et2Zn R' ●
O Zn R
PhCH2CH2CHO Et2Zn 90% e.e. ●
O Zn
61% e.e. 4 R'O ZnR
n-C6H13CHO
R R' ●O Zn
R' R
Zn ●
O Zn ●
O
■ The method is catalytic in aminoalcohol. ●
O Zn
R' R R R'
■ Two zinc species per aldehyde are involved in the alkylation step.
■ Product is taken out of the picture by aggregation

22-01-Et2Zn-1 11/5/03 8:46 AM


D. A. Evans Enantioselective C=O Addition: Noyori Catalyst-2 Chem 206

Other Catalysts for the R2Zn Addition Process Explanation for Nonlinearity of DAIB Catalyst
Ph Me2 Me2
H Ph Me Me
H H N N
Ph H Me Zn Zn
O O N Me O O O O
N N N
Zn Zn Li Zn Zn
Me Et Me N N
Et O Ph
Me3C Me Me Me Me
2 2
97% e.e. (S) 100% e.e. (R) 95% e.e. (S) (S,S) dimer (R,R) dimer
Li
Me
Me N Bu
N Bu
N Me
Me Li N
Me O N Et Zn
Ph Me2 Me2
90% e.e. (R) Li O Ph N N
Zn Me + Zn Me
90% e.e. (R) 90% e.e. (R) O O
p.
p.
(Results are cited for the reaction of benzaldehyde and Et2Zn)
Problem: Rationalize the stereochemical course of each of the catalysts (S) catalyst (R) catalyst

■ Non-linear effects observed with the Noyori Catalyst (DAIB-Zn)

100 Me2
N Me
There is no correlation between (S,R) dimer Zn
catalyst and product ee. O O
Zn
N
Et2Zn + PhCHO Me Me
Product 2
(%ee) Observations
(S) catalyst

■ (S,S) dimer dissociates upon addition of RCHO & effects catalysis


OH
25% ee Catalyst affords ■ (S,R) dimer is overwhelmingly more stable than (S,S) homodimer
product in 95% ee. Ph Me
■ (S,R) dimer is ineffective as a catalyst

%ee of catalyst 100

22-02-Et2Zn-2 11/5/03 8:49 AM


D. A. Evans Enantioselective C=O Addition: Dialkylzinc Addn Scope Chem 206

Me2
N
Improved Selectivity with Aliphatic Aldehydes
Zn Me
Scope of the DAIB Catalyst
O Soai, J. Org. Chem. 1991, 56, 4264
(S) catalyst Bu
Bu
N Me
O OH
Et Zn (6%)
(S) catalyst OH
H Et O Ph
Et2Zn RCHO + Et2Zn
98% ee 0 °C, hexane R Et
70 - 100%
O OH
O O O Me O
H
(S) catalyst
Me
Me2Zn n-Bu H H H Me H
91% ee
OH 88% ee 78% ee 95% ee 93% ee
O
(S) catalyst
H
H
p.
p. Et2Zn
96% ee
OH Lepicidin Application: The reaction functions in complex systems
O
(S) catalyst OR
Bu3Sn C5H11
Bu3Sn H Bu Bu
(n-Pen)2Zn Et 21 O
85% ee N Me
Me
the catalyst
Et Zn H
OH O
O O Ph 3
(S) catalyst O H H
H
H H
Et2Zn
90% ee Evans, Black, JACS 1993, 115, 44974513

H 9 OR
OH
O
(S) catalyst Et 21 OTIPS H 21 OTIPS
n-Bu H
n-Bu H
Et2Zn
60% ee OH 15 OTES
Et2Zn, O 15 OTES
Me Me
H H
MeO 0 °C, hexane MeO
Review: Noyori Angew. Chem. Int. Ed. 1991, 30, 49 3 3
diastereoselection
O O
10:1 (98%)
11
SnBu3 SnBu3
22-03-Et2Zn-Scope 11/5/03 8:49 AM
D. A. Evans Enantioselective C=O Addition: Corey-Itsuno Catalyst Chem 206

Discovery of a Catalytic Process O


Y
O
B X + B X
Enantioselective Carbonyl Reduction: Corey Angew. Chem. Int Ed. 1998, The N BH3 N
37, 1986-2012 (handout)
Catalytic Cycle Y – BH3

■ The Stoichiometric Process: Itsuno, 1983-1985


OBH2 O

RL C C
RS RL RS
Me2HC Ph H
2 equiv BH3 Chiral
Ph Boron Hydride
30 °C, 10 hr Y X Y O – X
H 2N OH ( H–BXC ) O
B + B
N N
+

000
000
000000
O OH R = Me, 94 % ee O RS O RS
( H–BXC ) H 2B –
H R = Et, 94 % ee H 2B H C C
H
R Ph Ph R = n-Bu 100 % ee

000
000

00
00
00
00
R RL RL

000
000
000
00
00
00
Itsuno, Chem. Commun. 1983, 469

00
00
00
00
p.
p. Itsuno, J. Org. Chem. 1984, 49, 555
Itsuno, J. Chem. Soc. Perkin Trans I. 1985, 2615
Ph

+ X
■ The Catalytic Process: Corey, 1987 H
– N O
B B Ph
H Ph B H
H Ph B Me
Ph H O+
Ph BH3
N+ O
N O – B Ph
B

00 00000
00
00
00
00000 00
R=H H 3B
R = Me R
R
Corey, JACS 1987, 109, 5551
Corey, JACS 1987, 109, 7925
O OH Corey, JOC 1988, 53, 2861

00
00
(0.1 equiv)
R Catalyst X-ray, Corey, Tet. Let 1992, 33, 3429
R Me BH3 Me
H
Mathre, JOC 1993, 58, 2880
R = Ph, 97 % ee catalyst prep: Mathre, JOC 1993, 58, 799
H Ph
R = t-Bu, 97 % ee Mathre, JOC 1991, 56, 751
91 % ee Ph
R = c-C6H11 (Review) Martens, Tertrahedron Asymmetry 1992, 3, 1475
N O
B Improved version
But how does it really work ?
22-04-Corey Cat 11/5/03 8:55 AM Me
Chem 206

B
Enantioselective C=O Addition: Corey-Itsuno Catalyst

B
00000
00
000000
000
000
000
000
00000 0000
00000000
000 000
000
000 0000
0000
000000
000 000 000
000
000 000 000
000000
00
000
000 00000 000
00

B
0000
0000
0000
000
0000
000 0000

22-05-Corey Cat 11/3/03 1:51 PM


0000
000
0000

B
Ph

B Ph
O
D. A. Evans

O+
+ X

Ph
Me
– N

H
B
H

p.
p.
D. A. Evans Enantioselective C=O Addition: Catalyst Scope Chem 206

H
Ph Representative Reductions Me Me
Ph
Me Me
O The catalyst Me Me Me Me Me Me
N (R) cat (0.2 equiv)
B O Me Me Me Me
Me
O O 1.5 equiv HO
(S) cat (0.1 equiv) O F F 92% ee
Me O Me
O BH
n-C5H11
O Tet. Let 1992, 33, 2319
BH3 (0.6 equiv)
n-C5H11 OH
THF 23°C, 2 min ArCOO
Fluoxetine (Prozac®) Synthesis
ArCOO O 91 : 9
O O OH
(R)-cat, as above BH3
O Cl Cl
H Ph
Ph 94% ee (>99%)
n-C5H11
0.1 equiv O
N Na–I
Corey, JACS 1987, 109, 7925 B MeNH2
p.
p. ArCOO OH
CF3 Me
90 : 10 OH

O OH O NHMe
NaH
(S) cat NHMe CF3
86% ee
BH3
Tet. Let 1989, 30, 5207
Prozac® Cl

O OH
Br Br An α-Amino Acid Synthesis
(S) cat R ee
91% ee
BH3 O O OH
n-C5H11 95%
BH
R CCl3 R CCl3 c-C6H11 92%
O OH O H Ph
Ph
t-C4H9 98%
O
Me (S) cat Me N
91% ee B
BH3 0.1 equiv n-Bu
Corey, JACS 1992, 114, 1906
Tet. Let 1992, 33, 3435
OH N3
HO – Tet. Let 1992, 33, 3431
R CCl3 N3– rm temp R COOH
22-06-Corey Cat-2 10/31/01 7:43 AM
D. A. Evans Enantioselective C=O Addition: Ketone Reduction Chem 206

OH
Stoichiometric Chloroborane Reducing Agents
Me
NMe2
Enantioselective Reducing Agents Ph B Cl
Me O OH
N-Methylephedrine, 2
LiAlH4, (3,5-xylenol)2 RL RS THF RL RS
Me [LiAl(lig)(OAr)2H] 50 - 90%
O H Me
B Ketone Reaction Conditions % ee
Al -
O OEt Bn
Ph acetophenone –25 °C 98%
+
Li Me2N OH butyrophenone –25 °C 98%
(R)-Alpine Borane
(S)-BINAL-H Darvon alcohol, LiAlH4 2,2-dimethylpropiophenone –25 °C 79%
[LiAl(lig)2H] 3,3-dimethyl-2-butanone 25°C, 12 days 95%
Reviews: Midland, Asymmetric Synthesis, Vol 2, p 45- 2,2-dimethylcyclohexanone 25°C, 2 days 91%
Granbois, Asymmetric Synthesis, Vol 2, p 71-
Brown, Accts. Chem. Res. 1992, 25, 16-24
p.
p. Singh, Synthesis 1992, 605-617

Reductions of Representative Carbonyl Compounds


O Brown's Model
O
O
R
R
R
Reagent Me
Me

78% e.e. R=Me H Me


Alpine-Borane 72 - 92% e.e. 59 - 89% e.e. Large ligand (RL)
90% e.e. R=CO2Me R
B
Cl H
BINAL-H 84 - 96% e.e. >95% e.e. 95 - 100% e.e. O
(71% ee, R=i-Pr) Small ligand (Rs) Favored TS
(57% ee, R=i-Pr)
RS RL
Darvon-LiAlH4 34 - 90% e.e. 25% e.e. 15 - 75% e.e.
Less hindered aliphatic ketones Brown, J. Org. Chem. 1985, 50, 5446
are not reduced with useful Brown, J. Org. Chem. 1986, 51, 3394
N-Methylephedrine- 75 - 90% e.e. levels of enantioselectivity Brown, J. Org. Chem. 1988, 53, 2916
78 - 98% e.e. ----
LiAlH4 (cyclic ketones)

22-07-Asym Redn-1 11/5/03 8:55 AM


D. A. Evans Enolates & Metalloenamines: Introduction Chem 206
Important References Tautomers: Structural isomers generated as a consequence of the
1,3-shift of a proton adjacent to a X=Y bond. for example:
"Structure and Reactivity of Lithium Enolates. From Pinacolone to Selective
C-Alkylations of Peptides. Difficulties and Opportunities Afforded by Complex H H
Structures".
D. Seebach Angew. Chem. Int. Ed. Engl., 27, 1624 (1983). Z X Y Z X Y
"Stereoselective Alkylation Reactions of Chiral Metal Enolates". D. A. Evans
Asymmetric Synthesis, 3, 1 (1984).
Keto-Enol Tautomers: Tautomerism may be catalyzed by either acids
"Generation of Simple Enols in Solution". B. Capon, B.-Z. Guo, F. C. Kwok, A. K. or bases:
Siddhanta, and C. Zucco Acc. Chem. Res. 21, 121 (1988).
O O OH
"pKa and Keto-Enol Equilibrium Constant of Acetone in Aqueous Solution". Y. –H+ +H+
Chiang, A. J. Kresge, and Y. S. Tang J. Am. Chem. Soc. 106, 460 (1984). H H
base catalysis: R CH3 R R
H H
Enols & Enolates are the most important nucleophiles in O H OH
organic & biological chemistry. +H+ O –H+
acid catalysis: H
R CH3 R
R CH3
OH H
H+ El(+)
Rb
O Ra O
Rb Acidity of Keto and Enol Tautomers: Consider Acetone:
Rb Ra
Ra
base O El(+) El OH
O
Rb H
Ra H3C
pK = 8.22 (measured)
H3C CH3
H
Enamines & metalloenamines, their nitrogen counterparts, are OH O
equally important.
H + H pK = 10.94 (measured)
+
H
H3C H3C
enamine
H H
R H
H+ N El(+)
R O
Rb R O
N N +H+ pK = 19.16 (calculated)
Ra H
Rb Rb H3 C
H3 C CH3
Ra Ra
R M H
base N El(+) El
Rb
Ra Kresge, JACS 1984, 106, 460

metalloenamine On the origin of the acidity of enols: Wiberg, JACS 1996, 118, 8291-8299

22-08-Enolates/intro 11/5/03 8:57 AM


D. A. Evans Enolization with Metal Amides bases Chem 206

Tautomeric Equilibria: Ketones vs. Imines ■ Stereochemistry


OLi OLi
O
LM–NR2 Me
O OH Me R + R
Keq ~ 10–3 R
(E) Me
(Z)

Base R-Substituent Ratio, (E):(Z)


Can you rationalize these differences?
MeNH2 –H2O LDA (THF) -OMe, O-t-Bu 95 : 5
H LDA (THF) -S-t-Bu 95 : 5
N N LDA (THF) -Et 77 : 23
Me Me
Keq > 10 LDA (THF) -CHMe2 40 : 60
LDA (THF) -CMe3 0 : 100
The enamine content in an analogous imine is invariably higher than its
carbonyl counterpart. In the case above, ring conjugation now stabilizes the LDA (THF) -C6H5 0 : 100
enamine tautomer as the major tautomer in solution.
LDA (THF) -NEt2 0 : 100
Enolization: Amide Bases R
‡ OLi s-BuLi (THF) -NEt2 25 : 75
O
H Me
R Li R
O H
N Me ■ Solvent Base R-Substituent Ratio, (E):(Z)
Me R (E) Geometry
R R
O LDA (THF) -OMe 95 : 5
LM–NR2 Me H OLi LDA (THF, HMPA) -OMe 16 : 84
R Li Me
H R
N ■ Base Structure Masamune (J. Am. Chem. Soc. 1982, 104, 5526)
R (Z) Geometry O OLi OLi
LiNR2
The Ireland Model (J. Am. Chem. Soc. 1976, 98, 2868) Me R
+ Me
Narula, Tetrahedron Lett. 1981, 22, 4119 R THF, -78 °C R
more recent study: Ireland, JOC 1991, 56, 650 Me
For the latest word on this subject see: Xie, JOC 1997, 62, 7516-9 (E) Geometry (Z) Geometry

Stereoelectronic Requirements: The α-C-H bond must be able to overlap with Base R = Et, (E):(Z) R = Cy, (E):(Z)
π∗ C–O
Ha Li–N(i-Pr)2 70 : 30 39 : 61
π∗ C–O Hc Hb Li–N(SiMe3)2 30 : 70 15 : 85
base
R C O Li–N(SiEt3)2 1 : 99 4 : 96
– Ha+
Hc Hb R O – Li–N(SiMe2Ph)2 0 : 100 0 : 100
at equilibrium 16 : 84
22-09-Enolates/intro-2 11/3/03 2:03 PM
D. A. Evans Enolization with Metal Amides bases Chem 206

Base Structure Corey & Co-workers, Tetrahedron Lett. 1984, 25, 491, 495 Regioselective Enolization
O OLi OLi
O OLi OLi
LiNR2 Me M–base Me Me
Me + Me
Et Et Et A + B
THF, -78 °C
Me
(E) Geometry (Z) Geometry
Base temp control Ratio (A:B)
Li–N(i-Pr)2
Me3C LiN(i-Pr)2 –78 ° kinetic 99:1
(LDA) 77 : 23 Me Me Me
N Me Me LiN(SiMe3)2 –78 ° kinetic 95:5
Me Li Me N Ph3C–Li –78 ° kinetic 90:10
Me Li Me
Ph3C–Li heat thermo 10:90
(LiTMP) 86 : 14 (LOBA) 98 : 2
Na–H heat thermo 26:74
Lithium Halide Effects Collum (J. Am. Chem. Soc. 1991, 113, 9572) K–H heat thermo 38:62
Collum (J. Am. Chem. Soc. 1991, 113, 9575)
Collum (J. Am. Chem. Soc. 1991, 113, 5053) A: Alkyl groups stabilize metal enolate
For the latest in the series of Column papers see: JACS 2000, 122, 2452-2458
A: As M–O bond becomes more ionic A is attenuated
O OLi OLi
LiNR2 Kinetic Selection sensitive to structure
Me + Me
Et Et Et O OLi OLi
THF, -78 °C
Me Ph Ph Ph
Ratio, (E):(Z) M–base
A + B
LiTMP 86 : 14
LiTMP, 10% LiBr 98 : 2
O ratio: 99:1
Enolization in Non-Ethereal Solvents Collum (JACS 2003, 125, ASAP) estimated
(18) Ph
pKa's
(25) A B
O OLi
(Bordwell) pKeq est: 7 (10+7)
Me Me
Na–N(TMS)2
toluene, –78 ° Unsaturated Ketones
6 equiv R3N Me
Me
Me
R 3N none Me2NEt n-Bu3N 1-Bu3N KOt–Bu LiN(i-Pr)2
krel 1 200 3000 1 t-BuOH O LiO
KO
kinetic enolate
Reaction kinetics suggest (TMS2NLi)2(R3N)(Ketone)‡ thermodynamic
enolate see kinetic acidity handout for an
extensive compilation of cases.
22-10-Enolates/intro-3 11/3/03 2:28 PM
D. A. Evans Enolization with Metal Amides bases Chem 206

Kinetic Selection sensitive to structure


Kinetic Selection in Enolization of Unsaturated Ketones
O OLi OLi
LDA
Me Me Me
Me CH2 Me
O OLi
71:29 Me Me
O OLi OLi LDA
LDA only enolate
Me Me Me
Me CH2 Me O O
Me Me 99:1 Me

O OLi OLi Me Me
LDA
Ph Ph Ph LDA only enolate
Me CH2 Me
14:86 O LiO
O
OLi
OLi
Ph LDA O OLi
Ph
Ph
99:1 LDA
only enolate

O Me Me
OLi
OLi
MeO LDA MeO
MeO O OLi
85:15
LDA
only enolate
O OLi
OLi Me Me
LDA
~90:10 O OLi
Me
Me LDA
Me only enolate
O
OLi Me Me Me Me
OLi Me Me
LDA
N ~83:17
Et N
Et N
Et

22-11-Enolates/intro-4 11/4/03 3:14 PM


Quote for the Day

Professor Robert Milikan (1928), Nobel Laureate in Chemistry

"There is no likelihood man can ever tap the power of the atom.
The glib supposition of utilizing atomic energy when our coal has
run out is a completely unscientific Utopian dream, a childish
bug-a-boo. Nature has introduced a few foolproof devices into
the great majority of elements that constitute the bulk of the world,
and they have no energy to give up in the process of disintegration."

Quotes-3 11/7/03 8:42 AM


D. A. Evans Enolates & Metalloenamines-1 Chem 206

■ Assigned Journal Articles


http://www.courses.fas.harvard.edu/~chem206/
"Structure and Reactivity of Lithium Enolates. From Pinacolone to
Selective C-Alkylations of Peptides. Difficulties and Opportunities
Afforded by Complex Structures".
Chemistry 206 D. Seebach Angew. Chem. Int. Ed. Engl., 27, 1624 (1983). (handout)

Advanced Organic Chemistry "Stereoselective Alkylation Reactions of Chiral Metal Enolates".


D. A. Evans Asymmetric Synthesis, 3, 1 (1984). (handout)

Lecture Number 23 ■ Other Useful References


"Recent Advances in Dianion Chemistry". C. M. Thompson and D. L. C. Green
Tetrahedron, 47, 4223 (1991).
Enolates & Metalloenamines-1
The Reactions of Dianions of Carboxylic Acids and Ester Enolates". N.
Petragnani and M. Yonashiro Synthesis, 521 (1982).
M M R
O N "Generation of Simple Enols in Solution". Capon, Guo, Kwok, Siddhanta, and
R R Zucco Acc. Chem. Res. 21, 121 (1988).
R R
"Keto-Enol Equilibrium Constants of Simple Monofunctional Aldehydes and
Ketones in Aqueous Solution". Keeffe, Kresge, and Schepp JACS, 112, 4862
■ Tautomerism in C=O and C=NR Systems (1990).

■ C=O Enolization with Metal Amide Bases "pKa and Keto-Enol Equilibrium Constant of Acetone in Aqueous Solution".
■ C=O Enolization: Kinetic Acidities Chiang, Kresge, and Tang JACS 106, 460 (1984).
■ Mild Methods for Enolate Generation
■ Enolate Structure: A Survey of X-ray Structures ■ Database Problem 314
■ Metallo-Enamine X-ray Structures Kawabata and co-workers recently reported the remarkable enolate
alkylation illustrated below (JACS 2003, 125, 13012). When the indicated
α-aminoacid ester is treated with KHMDS in DMF at —60 °C, the derived
■ Reading Assignment for this Week: cyclic amino acid ester is formed in high yield and enantioselectivity. The
stereochemical outcome represents a formal retention of configuration. This
Carey & Sundberg: Part A; Chapter 7 reaction exhibits some generality as the 4- 5-, 6-, and 7-membered lactams
Carbanions & Other Nucleophilic Carbon Species may be obtained in high ee.
Carey & Sundberg: Part B; Chapter 2 R CO2Et
R R ee
Reactions of Carbon Nucleophiles with Carbonyl Compounds KN(TMS)2 CO2Et
Bn 98 (S)
Br N DMF N
Boc Me2CH 94 (S)
Friday, –60 °C, 30 min Boc
D. A. Evans November 7, 2003
Provide a rationalization of these results. Three-dimensional drawings are recommended.

23-00-Cover Page 11/7/03 8:28 AM


D. A. Evans Enolates & Metalloenamines: Introduction Chem 206

Important References Tautomers: Structural isomers generated as a consequence of the


1,3-shift of a proton adjacent to a X=Y bond. for example:
"Structure and Reactivity of Lithium Enolates. From Pinacolone to Selective
C-Alkylations of Peptides. Difficulties and Opportunities Afforded by Complex H H
Structures".
D. Seebach Angew. Chem. Int. Ed. Engl., 27, 1624 (1983). Z X Y Z X Y
"Stereoselective Alkylation Reactions of Chiral Metal Enolates". D. A. Evans
Asymmetric Synthesis, 3, 1 (1984).
Keto-Enol Tautomers: Tautomerism may be catalyzed by either acids
"Generation of Simple Enols in Solution". B. Capon, B.-Z. Guo, F. C. Kwok, A. K. or bases:
Siddhanta, and C. Zucco Acc. Chem. Res. 21, 121 (1988).
O O OH
"pKa and Keto-Enol Equilibrium Constant of Acetone in Aqueous Solution". Y. –H+ +H+
Chiang, A. J. Kresge, and Y. S. Tang J. Am. Chem. Soc. 106, 460 (1984). H H
base catalysis: R CH3 R R
H H
Enols & Enolates are the most important nucleophiles in O H OH
organic & biological chemistry. +H+ O –H+
acid catalysis: H
R CH3 R
R CH3
OH H
H+ El(+)
Rb
O Ra O
Rb Acidity of Keto and Enol Tautomers: Consider Acetone:
Rb Ra
Ra
base O El(+) El OH
O
Rb H
Ra H3C
pK = 8.22 (measured)
H3C CH3
H
Enamines & metalloenamines, their nitrogen counterparts, are OH O
equally important.
H + H pK = 10.94 (measured)
+
H
H3C H3C
enamine
H H
R H
H+ N El(+)
R O
Rb R O
N N +H+ pK = 19.16 (calculated)
Ra H
Rb Rb H3 C
H3 C CH3
Ra Ra
R M H
base N El(+) El
Rb
Ra Kresge, JACS 1984, 106, 460

metalloenamine On the origin of the acidity of enols: Wiberg, JACS 1996, 118, 8291-8299

23-01-Enolates/intro 11/7/03 8:16 AM


D. A. Evans Enolization with Metal Amides bases Chem 206

Tautomeric Equilibria: Ketones vs. Imines ■ Stereochemistry


OLi OLi
O
LM–NR2 Me
O OH Me R + R
Keq ~ 10–3 R
(E) Me
(Z)

Base R-Substituent Ratio, (E):(Z)


Can you rationalize these differences?
MeNH2 –H2O LDA (THF) -OMe, O-t-Bu 95 : 5
H LDA (THF) -S-t-Bu 95 : 5
N N LDA (THF) -Et 77 : 23
Me Me
Keq > 10 LDA (THF) -CHMe2 40 : 60
LDA (THF) -CMe3 0 : 100
The enamine content in an analogous imine is invariably higher than its
carbonyl counterpart. In the case above, ring conjugation now stabilizes the LDA (THF) -C6H5 0 : 100
enamine tautomer as the major tautomer in solution.
LDA (THF) -NEt2 0 : 100
Enolization: Amide Bases R
‡ OLi s-BuLi (THF) -NEt2 25 : 75
O
H Me
R Li R
O H
N Me ■ Solvent Base R-Substituent Ratio, (E):(Z)
Me R (E) Geometry
R R
O LDA (THF) -OMe 95 : 5
LM–NR2 Me H OLi LDA (THF, HMPA) -OMe 16 : 84
R Li Me
H R
N ■ Base Structure Masamune (J. Am. Chem. Soc. 1982, 104, 5526)
R (Z) Geometry O OLi OLi
LiNR2
The Ireland Model (J. Am. Chem. Soc. 1976, 98, 2868) Me R
+ Me
Narula, Tetrahedron Lett. 1981, 22, 4119 R THF, -78 °C R
more recent study: Ireland, JOC 1991, 56, 650 Me
For the latest word on this subject see: Xie, JOC 1997, 62, 7516-9 (E) Geometry (Z) Geometry

Stereoelectronic Requirements: The α-C-H bond must be able to overlap with Base R = Et, (E):(Z) R = Cy, (E):(Z)
π∗ C–O
Ha Li–N(i-Pr)2 70 : 30 39 : 61
π∗ C–O Hc Hb Li–N(SiMe3)2 30 : 70 15 : 85
base
R C O Li–N(SiEt3)2 1 : 99 4 : 96
– Ha+
Hc Hb R O – Li–N(SiMe2Ph)2 0 : 100 0 : 100
at equilibrium 16 : 84
23-02-Enolates/intro-2 11/7/03 8:16 AM
D. A. Evans Enolization with Metal Amides bases Chem 206

Base Structure Corey & Co-workers, Tetrahedron Lett. 1984, 25, 491, 495 Regioselective Enolization
O OLi OLi
O OLi OLi
LiNR2 Me M–base Me Me
Me + Me
Et Et Et A + B
THF, -78 °C
Me
(E) Geometry (Z) Geometry
Base temp control Ratio (A:B)
Li–N(i-Pr)2
Me3C LiN(i-Pr)2 –78 ° kinetic 99:1
(LDA) 77 : 23 Me Me Me
N Me Me LiN(SiMe3)2 –78 ° kinetic 95:5
Me Li Me N Ph3C–Li –78 ° kinetic 90:10
Me Li Me
Ph3C–Li heat thermo 10:90
(LiTMP) 86 : 14 (LOBA) 98 : 2
Na–H heat thermo 26:74
Lithium Halide Effects Collum (J. Am. Chem. Soc. 1991, 113, 9572) K–H heat thermo 38:62
Collum (J. Am. Chem. Soc. 1991, 113, 9575)
Collum (J. Am. Chem. Soc. 1991, 113, 5053) A: Alkyl groups stabilize metal enolate
For the latest in the series of Column papers see: JACS 2000, 122, 2452-2458
A: As M–O bond becomes more ionic A is attenuated
O OLi OLi
LiNR2 Kinetic Selection sensitive to structure
Me + Me
Et Et Et O OLi OLi
THF, -78 °C
Me Ph Ph Ph
Ratio, (E):(Z) M–base
A + B
LiTMP 86 : 14
LiTMP, 10% LiBr 98 : 2
O ratio: 99:1
Enolization in Non-Ethereal Solvents Collum (JACS 2003, 125, ASAP) estimated
(18) Ph
pKa's
(25) A B
O OLi
(Bordwell) pKeq est: 7 (10+7)
Me Me
Na–N(TMS)2
toluene, –78 ° Unsaturated Ketones
6 equiv R3N Me
Me
Me
R 3N none Me2NEt n-Bu3N 1-Bu3N KOt–Bu LiN(i-Pr)2
krel 1 200 3000 1 t-BuOH O LiO
KO
kinetic enolate
Reaction kinetics suggest (TMS2NLi)2(R3N)(Ketone)‡ thermodynamic
enolate see kinetic acidity handout for an
extensive compilation of cases.
23-03-Enolates/intro-3 11/7/03 8:15 AM
D. A. Evans Enolization with Metal Amides bases Chem 206

Kinetic Selection sensitive to structure


Kinetic Selection in Enolization of Unsaturated Ketones
O OLi OLi
LDA
Me Me Me
Me CH2 Me
O OLi
71:29 Me Me
O OLi OLi LDA
LDA only enolate
Me Me Me
Me CH2 Me O O
Me Me 99:1 Me

O OLi OLi Me Me
LDA
Ph Ph Ph LDA only enolate
Me CH2 Me
14:86 O LiO
O
OLi
OLi
Ph LDA O OLi
Ph
Ph
99:1 LDA
only enolate

O Me Me
OLi
OLi
MeO LDA MeO
MeO O OLi
85:15
LDA
only enolate
O OLi
OLi Me Me
LDA
~90:10 O OLi
Me
Me LDA
Me only enolate
O
OLi Me Me Me Me
OLi Me Me
LDA
N ~83:17
Et N
Et N
Et

23-04-Enolates/intro-4 11/1/01 12:38 PM


D. A. Evans Enolate Structures from X-ray Diffraction Chem 206
M
M O Ab initio calculations (Spartan) indicate O
O M=H – 0.19
R that the partial negatilve charge on the H M = Li – 0.22
Metal Tautomerism R alpha carbon is ~ – 0.22 for the Li enolate Me
R
R δ–
M H

For alkali metal enolates (M = Li, Na, K etc.) the O-metal tautomer is strongly
favored. This generalization holds for most alkaline earth enolates (Mg+2) as
well. These are the generally useful enolate nucleophiles
CMe3
For certain metal enolates from heavy metals such as M = Hg+2 the C-metal O Me2
Me N
tautomer is sometimes favored.
O Li
O H N
M M M Me2
O O O
R R Li Crystallized as
R
R R R N the dimer
●●
Li
– –
Resonance Structures O resonance C resonance
structure structure
Since enolates usually function as carbon nucleophiles, it is therefore of some
interest to assess the relative importance of the illustrated contributing polar
resonance structures. Within the last decade good X-ray crystal structures of a
number of metal enolates have been obtained.
Seebach & co-workers,
One would predict that as the relative importance of the C– structure J. Am. Chem. Soc. 1985,107, 5403.
increases, the C–O bond would shorten and the C–C bond would lengthen.

Me Li
O O
The prediction stated above
1.35 Å
does hold, but the net change in 1.36 Å
H H
the C–C bond length is < 2 % !
R R Mg
1.32 Å H 1.34 Å H
Mg
CMe3
In solution and in the solid state metal enolates have a strong tendency to Br
aggregate into dimers and tetramers to satisfy metal solvation requirements. H
Br O Mg
M M OR Me OEt2
M RO M
R O + R O O R O M Crystallized as
O R
M M O the dimer
M R
23-05 enolate structure-1 11/7/03 8:15 AM
D. A. Evans Enolate Structures: Lithiun Enolates Chem 206

Li Enolates Me Me
Me Me
Si Li Si
O O CMe3 O O CMe3
LDA
Me H 2C
Me Me
Me Me

Williard, P. G.; Hintze, M. J. J. Am. Chem. Soc. 1987, 109, 5539-5541.


O Li
Li O
OLi LiO
O Li
Li O O
Si
Li O
N Li
O Li N
Li

O
Li Si
N Li

Li O Li
O Li O
O Li
Li O
Li O
Li O Li
Li
Li

Li
Li
Li

Williard, P. G.; Carpenter, G. B. J. Am. Chem. Soc. 1986, 108, 462-468.

23-06 Enolate Structrure-2 11/7/03 8:14 AM


D. A. Evans Enolate Structures: The Reformatsky Reagent Chem 206

ZnBr
Zn Enolates O
Me3C THF
Me3C O O Zn Br
O
The THF Complex
O Br O
Zn(0)
Me3C Br
metal tautomers Zn O
CMe3
O THF
O
Br O CMe3 Me3C ZnBr
O
Zn
O O
Zn
Me3C O
Br

Br
Zn

Zn
Br
C

O Br
O
C
O
Zn
Zn 2.0Å

2.0Å
Br

Dekker, J.; Boersma, J.; van der Kerk,


G. J. M. J. Chem. Soc. Chem. Commun. 1983, 553.

Dekker, J.; Budzelaar, J.; Boersma, J.; van der Kerk, G. J. M.


Organometallics 1984, 3, 1403.

23-07 Zn Enolate structure 11/7/03 8:14 AM


D. A. Evans The Reformatsky Reaction Chem 206

The "Classical" Reformatsky Process The Samarium(II) Variant


Review: Comprehensive Organic Synthesis, 1991; Vol 2, Chapter 1.8, pp 277-299 Molander, "Reductions with Samarium (II) Iodide." Org. Reactions1994, 4 6, 211-367.
Fürstner, A. "Recent Advances in the Reformatsky Reaction." Synthesis 1989, 571.

CHO Zn, Et2AlCl R SmI2/0 °C R R = H: 85% yield


O OH
-20 °C O O O R = Me: 82% yield
Br CHO aldol diastereoselection 70:30
Br 48% yield no config assignment
O
O O O OH
H. Nozaki & Co-workers,
J. Am. Chem. Soc. 197, 99, 7705 Both cyclic and acyclic cases studied (11 cases).
R = H: 76% yield
SmI2/0 °C
OH Br CHO R = Me: 82% yield
Me CHO O aldol diastereoselection 62:38
Zn, Et2AlCl Me O
Br diastereoselection 10:1 OH no config assignment
O C6H13 55% yield O R
C6H13 O O R
O O
SmI2/0 °C
T. Nishida & Co-workers, 82% yield
Tetrahedron 1991, 47, 6623. Based on the Nozaki recipe JACS 1977, 99, 7705 O O
Br CHO
O O OH
O O OH O OH T. Tabushi & Co-workers,
Zn in HOH Good entry into prior literature
Br Tetrahedron Lett. 1986, 27, 3889.
Ph 25 °C Ph Ph Ph Ph O O
Me Me Me Br
O O 2 SmI2/THF O
PhCHO diastereoselection 60:40 8: 1 diastereoselection
87% yield Me -20 °C 97% yield
T. H. Chan & Co-workers, R R Et
Rxns carried out in water with either activated Zn or Sn. OH
Chem. Commun. 1990, 505. 19 cases reported. Me Me
Br Br O O
Br
O O 2 SmI2/THF O one isomer? 86% yield
O Me -78 °C
O R R Et
Zn/THF
O OZnBr O O OH
Br 25 °C Me Me
N N
Bn N G. A. Molander & Co-workers,
Bn Bn R(H)
Me Me J. Am. Chem. Soc. 1987, 109, 6556. O
Me
78% yield Proposed Transition R Et
structure
(R)H O
C. H. Heathcock & Co-workers, H
J. Org. Chem. 1987, 52, 5745. O Sm

23-08-Reformatsky Rxn 11/6/03 4:15 PM


D.A. Evans Design of Soft Enolization Systems Chem 206
Some qualitative observations (Evans Group, Unpublished))
Mild Methods for Forming Enolates
Cl4 Cl4
Lewis Acid C=O Complexation Enhances C–H Acidity (Computation)
Ti TiCl4 (1.0 equiv) Ti pKa 9 (DMSO)
O O Et3N (1.0 equiv) O O
∆Hgp H2SO4 = 304
F 3B Me Et3N Me Et3N–H
O O O N O N
BF3 ∆Hgp H–I = 314
Keq~100
H CH3 ∆pKa = –36 H CH3
∆∆Hgp = –50 kcal/mol Bn Bn
∆Hgas phase +366 +316
BF3 estimated pKa (DMSO)???
pK
pKaHOH +17 –7
∆pKa = –24 ∆∆Hgp = –24 kcal/mol
+ Et3N +9
Et3N–H H
BF3 complexation generates a "superacid" comparable to the acidity of H2SO4
Cl4 Cl4
Ren et al, JACS 1999, 121, 2633-2634 (pdf)
Ti TiCl4 (1.0 equiv) Ti
O O Et3N (1.0 equiv) O O
Et3N –2
Me Me
Some qualitative observations (Evans Group, Unpublished)) O N O N
pKa 9 (DMSO)
Xn Bn Bn Et3N–H
M
O O Lewis Acid (1.0 equiv) O O
Et3N (1.0 equiv) Cl4 Cl4
Me Me
O N O N Et3N–H
Ti Ti
O O O O
CH2Cl2
Bn Bn Me Me H pKa ~ +7
O N O N
Lewis Acid (MXn) DCl/D2O
% deuterium
TiCl4 100 Bn Bn
TiCl3(Oi-Pr) 100 O O estimated pKa (DMSO) ~ +7
TiCl4•(THF)2 80
TiCl2(Oi-Pr)2 70 Me
O N Hence TiCl4 complexation lowers acidity by
AlCl3 70 ~20 pka units. this number is the same
D
MgBr2•OEt2* 25 O O mgnitude as the BF3–acetaldehyde case
Bn
TiCl(Oi-Pr)3 10 just discussed
SnCl4, Et2AlCl, ZrCl4 0 Me
*2.0 equiv required O N

In these experiments, the Lewis acid was added first followed by the amine. pKa 9 (DMSO)~ +27
Bn

23-09-soft enoliz-1 11/7/03 8:05 AM


D.A. Evans Design of Soft Enolization Systems Chem 206

Strategy Lithium Enolates


Choose Lewis Acid (LA) which can reversibly associate with amine base (B:).
Horner-Wadsworth-Emmons Reaction.
(–) (+)
LA + B: LA B O O
LiCl, 1.2 equiv
O
Base, 1.0 equiv
(EtO)2P Base = DBU,Me
5 min, 99%, >50:1 E:Z
This system has the potential to enolize carbonyl functional groups: OEt i-PrCHO, 1 equiv OEtE:Z
Base = DIPEA, 7 h, 97%, >50:1
MeCN
1.2 equiv Me
LA LA rt
O +O O (+)
+ LA + B–H pKa 19.2 (DMSO), K+ counterion
R CH3 R CH2–H pKa 12.2 (Diglyme), Li+ counterion
R CH2
+ Roush & Masamune, Tet. Lett. 1984, 25, 2183-2186
B:
(–) (+)
LA B

Useful Lewis Acid Pairs Complexation OTf = –OSO2CF3 NHCbz NHCbz O
Above conditions
MgBr2 + NEt3 Reversible CHO using DIPEA
OEt
Li–X + NR3 Reversible 24 h, rt
Me Me
Sn(OTf)2 + NR3 Reversible (Et3N, EtNi-Pr2)
85% + 10% recovered aldehyde
R2B-OTf + NR3 Reversible (Et3N, EtNi-Pr2)
Conventional methods of deprotonation (NaH) resulted in epimerization
R2BCl + NR3 Reversible (Et3N, EtNi-Pr2) Me N Me (Overman JACS 1978, 5179).
PhBCl2 + NR3 Reversible (Et3N, EtNi-Pr2)
TiCl4 + NR3 Irreversible (Et3N, EtNi-Pr2)
i-PrOTiCl3 + NR3 Reversible (Et3N, EtNi-Pr2) Magnesium Enolates
(i-PrO)2TiCl2 + NR3 Reversible (Et3N, EtNi-Pr2)
(i-PrO)3TiCl + NR3 Reversible (Et3N, EtNi-Pr2)
MgBr2, 1.2 equiv O
O O Et3N, 1.1 equiv
All of the above systems will enolize simple ketones to some extent.
(EtO)2P
O O O O O OEt RCHO, 1 equiv R OEt
1 equiv THF, rt
R= i-Pr, 40% yield
R CH3 RS CH3 PhO CH3 EtO CH3 R2N CH3 R= n-C6H13, 100% yield
100% enolization for B, Sn, Ti Rathke, Nowak J. Org. Chem. 1985, 50, 2624-2626.
partial enolization for Li, Mg

23-10-soft enoliz-2 11/7/03 8:14 AM


D.A. Evans A Survey of 'Soft' Enolization Techniques Chem 206

Magnesium Enolates Titanium Enolates


J. Org. Chem. 1985, 50, 2622-2624.
Rathke J. Org. Chem. 1985, 50, 4877-4879. The Early Literature
Syn. Comm. 1986, 16, 1133-1139.
Lehnert, W. Tetrahedron Lett. 1970, 4723-4724.
Diethylmalonate acylations
O O
MgCl2, 1 equiv O O TiCl4
O O O O O O Pyridine
Et3N, 2 equiv
EtO OEt
EtO OEt
EtO OEt Me Cl MeCN, rt EtO OEt Et Et THF
Et Et
O Me
75% yield
85% yield
Ketone Carboxylation
MgCl2, 2 equiv Harrison, C. R. Tetrahedron Lett. 1987, 28, 4135-4138.
O NaI, 2 equiv
O
Et3N, 4 equiv
COOH TiCl4 O OH
O Et3N
CO2, MeCN 70 % yield + PhCHO
Me Ph Ph
rt Ph CH2Cl2, 0° C
30 min Me
Mg Ketone and aldehyde combined followed 91% yield
O O O O O by sequential addn of TiCl4 and then amine 95:5 syn/anti
MgCl2, 2 equiv
Et3N, 4 equiv MVK
O Me
+CO2 EtOH
-CO2 TiCl4 TiCl4
MeCN O O O
75% yield O +
rt +
Me Me
Ph Ph H Ph Ph H
Michael reaction
O O O O
MgBr2•OEt2
Me CO2Me
O N CO2Me O N Brocchini, Eberle, Lawton J. Am. Chem. Soc. 1988, 110, 5211-5212.
Et3N
CH2Cl2, 0° C Me
Bn Bn O O
Brn TiCl4
OH DIPEA CHO
Mg 73% yield, 93:7 diastereomer ratio S S
O O
THF OH
Me Evans, Bilodeau unpublished results. -40° C
Et3N–H O N
NO2 NO2
10-15:1 Z/E
Bn
Deuterium quench indicates 25% enolization of N-propionyloxazolidinone
23-11-soft enoliz-3 11/7/03 8:14 AM
M. Bilodeau, D.A. Evans A Survey of 'Soft' Enolization Techniques Chem 206

Titanium Enolates Reactions with Representative Electrophiles


Cl Cl4 O
Cl Ti Cl Ti
O CH2OBn
O O O O O Xp
TiCl4 Cl
Me Me R 3N Me Me
O N O N O N O O 99%, >99:1 O
CH2SPh
1 Xp OMe Xp
Bn Bn Bn
Me BOMCl Me 93%, 97:3
■ Enolization process not responsive to tertiary amine structure
■ DIPEA, Et3N, N-Ethylpiperidine all suitable bases. 78%, 98:2 PhSCH2Cl
■ DBU and tetramethylguanidine do not provide enolate.
H2C=CHCO2Me Cln
■ CH2Cl2 is the only suitable solvent for these enolizations.
O Et Ti O
N-Propionyloxazolidone (1) Ethylisopropylketone O O
(CH2)2COEt O
ClCH2NHCOPh CH2NHCOPh
Lewis Acid % Enolization Lewis Acid % Enolization Xp Me Xp
O N
TiCl4 100 TiCl4 100 Me Me
i-PrOTiCl3 100 i-PrOTiCl3 80 88%, >99:1 Bn 89%, 97:3
HC(OMe)3
TiCl4•2THF 80 (i-PrO)2TiCl2 50 MeOCH2NHCbz
(i-PrO)2TiCl2 70
(i-PrO)3TiCl ~10 O O
O O O
Me Me O
CH2NHCBz
■ Order of addition of reagents is important for TiCl4. CH(OMe)2 Xp
Me Xp O
+ - Me
R3N + TiCl4 R3N-TiCl4 Irreversible Complexation Me CH2OH
Xp 91%, 96:4
■ Order of addition of reagents is not important for i-PrOTiCl3 or (i-PrO)2TiCl2. 99%, >99:1
Me
+ -
93%, 99:1
R 3N + i-PrOTiCl3 R3N-TiCl3(OiPr) Reversible Complexation

J. Am. Chem. Soc. 1990, 112, 8215-8216.; J. Org. Chem. 1991, 56, 5750-5752.
■ Enolizable substrates:
O O O O O
Me OMe Me
R t-Bu Me i-Pr Me MeO PhS O O O O O O OH
1. TiCl4
■ Substrates Which present problems: Me DIPEA Me
O O N O N
O O 2. i-PrCHO
Me Me Me Me
Me Bn Bn
MeO R Me
R=Ar, R<i-Pr 86% yield, >99:1
self condensation self condensation Evans, Clark, Metternich, Novack, Sheppard J. Am. Chem. Soc. 1990, 112, 866.

23-12-soft enoliz-4 11/7/03 8:12 AM


D.A. Evans A Survey of 'Soft' Enolization Techniques: Boron Enolates-1 Chem 206

BR2 BR2
Dialkylboron Triflates O O O
R2B–X, R3N
Me Me Me Me Me
Di-n-butylboron triflate
Mukaiyama, Inoue Chem. Lett. 1976, 559-562. Mr
9-BBN-OTf, Et3N ratio ~97: 3
Bull. Chem. Soc. Jpn. 1980, 53, 174-178.
Enolizes ketones with 2,6-lutidine or DIPEA in ethereal solvents. Cy2B-Cl, Et3N ratio 21:79
Bu2B-OTf, Et3N ratio ~97: 3
Diastereoselective Aldol Reactions of Boron Enolates.
Evans, Vogel, Nelson J. Am. Chem. Soc. 1979, 101, 6120. Borane and lutidine or DIPEA form 1:1 complex with L2B-OTf. Complexation
Evans, Nelson, Vogel, Taber J. Am. Chem. Soc. 1981, 103, 3099-3111. reversible as enolization will occur upon addition of ketone. Less hindered
Evans, Bartroli, Shih J. Am. Chem. Soc. 1981, 103, 2127. nitrogen bases - pyridine, Dabco, DBU, irreversibly complex with L2B-OTf.
Masamune, S. et. al. Tetrahedron Lett. 1979, 2225, 2229, 3937.
Masamune, S. et. al. J. Am Chem. Soc. 1981, 103, 1566-1568.
The Ketone-Boron Complexes as enolate precursors:
Chiral dialkylboron triflates Me
R R R R R R
R R
Masamune, Sato, Kim, Wollmann – B B B
J. Am. Chem. Soc. 1986, 108, 8279-8281. TfOB B -X -X– O X
X O O O
Me anti Me +X– Me +X– R
syn
Paterson, I. et. al. R R R
Me
Tetrahedron 1990, 46, 4663-4684. charged Me Me
Tetrahedron Lett. 1989, 30, 997-1000.
Tetrahedron Lett. 1986, 27, 4787-4790. Me BOTf
OBL2 OBL2
(-)-(Ipc)2BOTf
Me R
R
Enolate Stereochemistry Me
Evans, Nelson, Vogel, Taber J. Am. Chem. Soc. 1981, 103, 3099-3111.
anti NR'3 syn NR'3
Goodman, Tetrahedron Lett. 1992, 33, 7219.
Enolization Model: Paterson, Tetrahedron Lett. 1992, 33, 7223. H H

BR2 BR2 R O BL2OTf R O BL2X


O O O H Me Me H
R2B-X, R3N
Me Me Me Me Me

Me Me Me Mr Cy2BCl-ketone complex may deprotonate through syn complex


9-BBN-OTf, Et3N ratio ~ 88: 12
Cy2B-Cl, Et3N ratio ~ 3: 97 R2BOTf-ketone complex may deprotonate through charged complex
with (Z) preference
Brown, J. Org. Chem. 1993, 58, 147-153

23-13-soft enoliz-5 11/7/03 8:13 AM


D. A. Evans C versus O Enolate Reactivity & the Hammond Postulate Chem 206
Question: Why do we generally show enolates reacting with electrophiles The Hammond Postulate is also relevant to this issue and is broadly
at carbon as opposed to oxygen ?? Let's begin the the discussion with an used to make qualitative statements about transition state structure.
observation:
■ "As electrophile reactivity increases, the percentage of reaction at the Hammond, JACS 1955, 77, 334 (handout)
enolate oxygen increases." For example, consider the reactions of cyclo-
hexanone enolate with the two electrophiles, methyl iodide and the much ■ In attempting to grasp the Hammond Postulate, let's consider two
extreme reactions, one which is strongly endothermic and one which is
more reactive acetyl chloride:
strongly exothermic.
O O–El
T‡
1 El(+) C/O Rxn Ratio B
Strongly Exothermic Reactions
El(+) O
∆H° > 20 kcal/mol

Energy
Me C Cl << 1
O O
>> 1 B A
El Me I
: A
2

■ The very reactive acid chloride gives almost exclusively the O-acylation Rxn Coordinate
product while the less reactive methyl iodide affords the alternate
C-alkylation product.
Hammond Postulate
These results may be understood in the context of qualitative statements "For strongly exothermic reactions, the transition state T‡
made by Hammond (The Hammond Postulate) and looks like reactant(s) e.g. B."
Hine (The Principle of Least Motion)
■ As applied to the enolate-electrophile reaction, for very exothermic
reactions, e.g. the reaction with acetyl chloride, the transition state for the
The Principle of Least Motion: process will involve little enolate structural reorganization. Hence in this
"As reactions become more exothermic, the favored reaction becomes instance the electrophile heads for the site of highest electron density
that path which results in the least structural (electronic) reorganization."
Carey & Sundberg: Part A; Chapter 4, pp217-220
for discussion of Hammond's Postulate
See Hine in Advances in Phys. Org. Chem. 1977, 15, 1-61
Based upon the above discussion draw a detailed mechanism for the
protonation of cyclohexanone enolate.
Since the X-ray data clearly support the picture that resonance structure O– O
1 best represents the enolate structure, highly reactive electrophiles will
favor O-attack according to Hine's generalization.
H+

23-14 C vs O Enolate React 11/6/03 5:29 PM


D. A. Evans Enols, Enolates, Enamines & Metalloenamines: Reactivity Hierarchy Chem 206

■ Metalloenamines: Decreasing Nucleophilicity


Imines may be transformed into their conjugate bases (enolate counterparts) Nucleophile
with strong bases: NR O NR2 OMe
R R C C C C C C C C
Metal R
Electrophile
N N N
pKa~ 29-33
Li-NR2 R-MgX Br2, O3 + + + +
The usual bases employed are either lithium amides (LDA) or Grignard
reagents. Note that Grignard reagents do not add to the C=N pi-bond due to H3O+ + + + +
the reduced dipole. With this functional group, deprotonation is observed to be
the preferred reaction. O
■ When to use a metalloenamine: R C Cl + + +

Decreasing Electrophilicity
Metalloenamines are significantly more nucleophilic than ketone or aldehyde O
enolates. They are used when less reactive electrophiles are under
consideration. For example: R C H + + +
OLi O Me
O
X Me R C R + +
no reaction
Me syn
However: relationship Me I + +
Metal R I Me
R
N N Me O

Me
R C OR + +
good yield
O
H 2C CH2 +
Metalloenamines are reactive enough to open epoxides in good yield. Ketone
enolates are only marginally reactive enough for this family of electrophiles. Me2CH I
+
O
Li-NR2 R O
OH R C NR2
N Me N CH2 N
Li R ■ Nature uses enamines, "stabilized" enolates, and enol derivatives in
C–C bond constructions extensively.
23-15 Enolate Reactivity Grid 11/6/03 5:28 PM
Quote for the Day

"640K ought to be enough for anybody." Bill Gates, 1981

Today's astrological forecast, Boston Globe, Monday, November 10, 2003


Capricorn (Dec 22–Jan 19th)

"Be prepared for someone to try to steal your ideas or take credit for your
work. You're on to something tangible and you need to act fast."

Quotes-3 11/10/03 9:12 AM


D. A. Evans Enolates & Metalloenamines-2 Chem 206

■ Assigned Journal Articles


http://www.courses.fas.harvard.edu/~chem206/
"Structure and Reactivity of Lithium Enolates. From Pinacolone to
Selective C-Alkylations of Peptides. Difficulties and Opportunities
Afforded by Complex Structures".
Chemistry 206 D. Seebach Angew. Chem. Int. Ed. Engl., 27, 1624 (1983). (handout)

Advanced Organic Chemistry "Stereoselective Alkylation Reactions of Chiral Metal Enolates".


D. A. Evans Asymmetric Synthesis, 3, 1 (1984). (handout)

Lecture Number 24 ■ Other Useful References


"Advances in Asymmetric Enolate Methodology" Arya, Qin, Tetrahedron 2000,
Enolates & Metalloenamines-2 56, 917-947 (pdf)

"Recent Advances in Dianion Chemistry". C. M. Thompson and D. L. C. Green


M M R Tetrahedron, 47, 4223 (1991).
O N
R R
R R The Reactions of Dianions of Carboxylic Acids and Ester Enolates". N.
Petragnani and M. Yonashiro Synthesis, 521 (1982).
■ Introduction and General Trends
"Generation of Simple Enols in Solution". Capon, Guo, Kwok, Siddhanta, and
■ Enolate Alkylation: Electronic & Steric Control Elements
Zucco Acc. Chem. Res. 21, 121 (1988).
■ Enolate Alkylation: Unusual Cases
■ Chiral Amide Enolates "Keto-Enol Equilibrium Constants of Simple Monofunctional Aldehydes and
■ Chiral Ester Enolates Ketones in Aqueous Solution". Keeffe, Kresge, and Schepp JACS, 112, 4862
■ Chiral Imide Enolates (1990).
■ Chiral Metalloenamines
"pKa and Keto-Enol Equilibrium Constant of Acetone in Aqueous Solution".
■ Reading Assignment for this Week: Chiang, Kresge, and Tang JACS 106, 460 (1984).

Carey & Sundberg: Part A; Chapter 7 Explain why A is favored for X = O while B is favored for X = NNHR
Carbanions & Other Nucleophilic Carbon Species
X X
Carey & Sundberg: Part B; Chapter 2 Me Me base Me Me
Reactions of Carbon Nucleophiles with Carbonyl Compounds
A B
Monday,
D. A. Evans November 10, 2003

24-00-Cover Page 11/9/03 11:46 AM


D. A. Evans Enols, Enolates, Enamines & Metalloenamines: Reactivity Hierarchy Chem 206

■ Metalloenamines: Decreasing Nucleophilicity


Imines may be transformed into their conjugate bases (enolate counterparts)
with strong bases:
Nucleophile – NR O– NR2 OMe
R R C C C C C C C C
Metal R
Electrophile
N N N
pKa~ 29-33
Li-NR2 R-MgX Br2, O3 + + + +
The usual bases employed are either lithium amides (LDA) or Grignard
reagents. Note that Grignard reagents do not add to the C=N pi-bond due to H3O+ + + + +
the reduced dipole. With this functional group, deprotonation is observed to be
the preferred reaction.
O
■ When to use a metalloenamine: R C Cl + + +

Decreasing Electrophilicity
Metalloenamines are significantly more nucleophilic than ketone or aldehyde
O
enolates. They are used when less reactive electrophiles are under consideration.
For example: R C H + + +
OLi O Me
O
X Me R C R + +
no reaction
Me syn
However: relationship Me I + +
Metal R I Me
R
N N Me O

Me
R C OR + +
good yield
O
H 2C CH2 +
Metalloenamines are reactive enough to open epoxides in good yield. Ketone
enolates are only marginally reactive enough for this family of electrophiles. Me2CH I
+
O
Li-NR2 R O
OH R C NR2
N Me N CH2 N
Li R ■ Nature uses enamines, "stabilized" enolates, and enol derivatives in
C–C bond constructions extensively.
24-01 Enolate Reactivity Grid 11/9/03 11:46 AM
D. A. Evans C versus O Enolate Reactivity: Hine "Least Motion Principle" Chem 206

Question: Why do we generally show enolates reacting with C versus O Enolate Reactivity: Enolate Acylation
electrophiles at carbon as opposed to oxygen ?? Let's begin the the
discussion with an observation: ■ Kinetic C–Acylation of ketone enolates can be carried out:
■ "As electrophile reactivity increases, the percentage of reaction at the Li
OLi O O O
enolate oxygen increases." For example, consider the reactions of cyclo- –78 °C
+
hexanone enolate with the two electrophiles, methyl iodide and the much R1 NC OMe R1 OMe 2
fast CN
more reactive acetyl chloride: R2 1 R2
O– O–El
Enolate acylation with 1 is fast slow
1 Intermediate 2 breaks down to product
El(+) C/O Rxn Ratio
more slowly than the acylation step
El(+) O
Under these conditions, proton transfer O O
Me C Cl << 1 from product to enolate does not occur. + LiCN
R1 OMe
O O
See accompanying "Acylation Handout" R2
Me I >> 1
:– El
2
■ Kinetic Acidities nicely illustrate LNM Principle: *Lecture 18"
Base
■ The very reactive acid chloride gives almost exclusively the O-acylation pKa(H2O) O O
product while the less reactive methyl iodide affords the alternate ~10 H rel rate: 10 +6
C-alkylation product.

These results may be understood in the context of qualitative statements pKa(H2O) O Base O
H 3C N H 2C N
made by Hammond (The Hammond Postulate) and ~10 O rel rate: 1 O
Hine (The Principle of Least Motion)
Proton kinetically controlled transfers from C-H Bonds are slow due to
The Principle of Least Nuclear Motion: the extensive reorganization required in conjugate base.

"As reactions become more exothermic, the favored reaction becomes ■ Leaving Group Ability: Stirling, Chem. Commun. 1975, 940
that path which results in the least structural (electronic) reorganization." O O O
base rds
Hine in Advances in Phys. Org. Chem. 1977, 15, 1-61 (handout)
Ph S
X
Ph S Ph S + X–
O X O
O
O O
Since the X-ray data clearly support the picture that resonance structure
1 best represents the enolate structure, highly reactive electrophiles will Ph S Ph S
The greater the structural OPh CN
favor O-attack according to Hine's generalization. reorganization of the leaving group O O
during E1cb elimination, the slower
See reinforcing examples on the accompanying page. krel = 1 krel = <10–8
the rate of elimination.
pKa = 10 pKa = 9.5
24-02 C vs O Enolate React 11/9/03 12:16 PM
D. A. Evans C versus O Enolate Reactivity: "The Hammond Postulate" Chem 206

The Hammond Postulate is also relevant to this issue and is broadly T‡


used to make qualitative statements about transition state structure. Strongly Endothermic Reactions
∆H° > +20 kcal/mol A
Hammond, JACS 1955, 77, 334
■ In attempting to grasp the Hammond Postulate, let's consider two B A

Energy
extreme reactions, one which is strongly endothermic and one which is
strongly exothermic. SN1 reactions are typical examples of
strongly endothermic processes
T‡ B
Hammond: "The transition state will look
like products."
B
Strongly Exothermic Reactions
Rxn Coordinate
∆H° > –20 kcal/mol Energy

G‡
B A
∆GA‡ H
A ∆GE‡
Me3C OTs

Rxn Coordinate ∆G° H

Hammond Postulate Does ∆GE‡ – ∆GA‡ = ∆G° (E – A)??


"For strongly exothermic reactions, the transition state T‡
looks like reactant(s) e.g. B." ∆GA‡: 26.9

■ As applied to the enolate-electrophile reaction, for very exothermic


reactions, e.g. the reaction with acetyl chloride, the transition state for the ∆GA‡:27.6
process will involve little enolate structural reorganization. Hence in this OTs
instance the electrophile heads for the site of highest electron density
Me3C H
Carey & Sundberg: Part A; Chapter 4, pp217-220 H
for discussion of Hammond's Postulate A
∆G° = +0.5–0.7 kcal/mol ∆∆ G‡= +0.7
Based upon the above discussion draw a detailed mechanism for the H
protonation of cyclohexanone enolate.
O O Me3C OTs

H+ H E
Wnstein & Holness, JACS 1955, 55, 5562

24-02a Hammond Postulate 11/10/03 8:31 AM


D. A. Evans Enolate Alkylation: Stereoelectronic Control Elements Chem 206

Review Evans, D. A. Stereoselective Alkylation Reactions of Chiral Metal


Enolates.; Morrison, J. D., Ed.; AP: New York, 1984; Vol. 3, pp 1-110. Examples where stereoelectronic factors are dominant

Stereoelectronic Issues Pilli, Tetrahedron, 1999, 55, 13321



■ Enolization: Breaking C–H bond must overlap with π∗ C–O in TS R R–X ratio
LDA
■ Alkylation: Forming C–El bond must overlap with π∗ C–O in TS‡
Bn–Br >99:1
O N Me R–X O N Me
base
H Boc Boc Allyl–Br 93:7
R R H
C C C C Issue: Degree of rehybridization
H M O good illustration of the impact of allylic strain
M O R in TS‡?
R Me Me
El(+) Me
H
‡ H LDA Boc N C C H
El El H H
N LiO H N
R H R RO O RO O
C C C C H O R
M O H O
M O R R–X
R
■ Cyclohexanone Enolate:
El(+)
a The C19 Angular Methyl Group in the steroid nucleus
H H
El OH
Me3C path A Me
Me3C OH
C C Me
R OLi favored
R H
e O Li/NH3 R
O O H
chair conformation H
El(+) e Me H
H LiO
O
H O H Me-I
R
Me3C path E
Me3C
R OH
disfavored Me
El O Me-I
El
twist boat conformation Me H
R
Metal R-substituent Electrophile Ratio, a:e
H H
Li Me CD3I 70:30 O
Li CO2Me Me-I 83:17 (90%) one isomer

Chair vs boat geometries not stongly reflected in diastereomeric TS s. The The enolate (Chem 3D)
transition states is early and enolate-like.

24-03-Enolate alk-1 11/10/03 8:46 AM


D. A. Evans Enolate Alkylation: Steric Control Elements Chem 206

In this case, both e and a paths are stereoelectronically


Steric Effects
equivalent. Diastereoselectivity is now determined by the Cases with Opposed steric & electronic effects
differential steric effects encountered in the two TS‡s.
El(+)
CO2Me R R R
a OMe
Me3C El(+)
OLi El +
El
Me3C H LiO O O
E H H H
e -78 °C Me3C CO2Me Me El Me Me El
H
El(+) H A Li/NH3
Dominant
Electrophile Ratio, E:A R El(+) Ratio Control element
R
Me-I 84:16 –H Et-I 95:05 stereoelectronic

n-Bu-Br 87:13 –H CD3I 83:17 stereoelectronic

Representative cases O –Me CD3I 07:93 steric


Me –Me Et-I >5:95 steric
CO2Me Me CO2Me Me CO2Me
Me Me Me
LDA
Ratio, 80 : 20
MeI Li
The enolate R = Me
CO2Me Me CO2Me Me CO2Me (Chem 3D)

LDA
Ratio, 90 : 10
MeI
Me Me Me
Based on above data, this case is reasonable:
C 3H 5
Me Me
LDA
O O Ratio, >97 : 3 LiNH2 (58%) >90 : 10
O allyl-Br O Me-I
Ph3COCH2 Ph3COCH2
H H O O H
H
diastereoselectivity depends stongly on O-protecting group CN NC Me
H H Me However
OTHP OTHP
LDA Me Me
O O Ratio, >97 : 3
O MeI NaH
Me Me O (67%) 93 : 7
H H Me-I
O O H
H
CO2Me Me CO2Me
24-04-Enolate alk-2 11/10/03 8:46 AM
D. A. Evans Enolate Alkylation: Unusual Cases Chem 206

Cases which do not appear to give the expected product based on


the analysis of steric effects Sterically Expected Results:
O H O H
OLi
Me O CO2Me Me R C 3H 5 N N R
O t-Bu t-Bu
LDA OMe BnBr 97% ds
MeI >98% ds CO2Me
Me O CO2Me O O
Me O allyl-Br BzCl >95% ds
R H
R = Me, Et, CO2Me Seebach, Helv. Chim. Acta 1987, 70, 1194.
≥ 88 : 12
Contrasteric relatives:
Seebach, Angew. Chem. Int. Ed 1981, 20, 1030
Ladner, Angew. Chem. Int. Ed 1982, 21, 449 OLi
■ However: O O R
t-Bu t-Bu
S-t-Bu MeOD >95%ds COS-t-Bu
Me O CO2Me CO2Me MeO2C Me O O
Me Me BnBr 60% ds
LDA O CO 2 Me Me O acetone 95%ds
Me O acetone OH O
CO2Me Me O Seebach, Helv. Chim. Acta 1987, 70, 1194.
Me O
HMe Me H O
Li ratio, 80 : 20 OLi
O MeI O Me
t-Bu t-Bu
OMe CO2Me 70 : 30
O O
The enolate (MM-2) Me Me

OLi
t-Bu O MeI t-Bu O Me
OMe >95 : 5
Here is another example of a contrasteric alkylation Me O Me O CO2Me

HO2C HO2C Me Me
HO2C
LDA, conditions
Ladner, Chem. Ber. 1983, 116, 3413-3426.
Me Me
(+)-menthyl–O2C CO2R CO2R Those factors defining olefin face selection are currently being
X
Me defined: Would you have predicted the outcome of the following
Me X conditions Ratio reaction? OSiR
O 3
R-Cl THF, 23 °C 80:20 R3SiO
(+)-menthol HgI2
R-Br THF-HMPA 02:98 >94 : 6
EtO2C
OH -78→-20 °C EtO
CMe2 K. Yamada, Organic Synthesis Past Present, and Future, p 525 OSiR3 OSiR3
Danishefsky J. Org. Chem. 1991, 56, 387
24-05-Enolate alk-3 11/10/03 8:47 AM
D. A. Evans Chiral Enolate Enolate Alkylation Circa 1978 Chem 206

■ Enolization selectivity: Amide-based controllers XC limited by


Chiral Enolate Design Requirements Circa 1978 enolization selectivity (Lecture 22)
Overall enantioselection will be the sum total of the O OLi OLi
defects introduced through: LM–NR2 Et Me
Et Me Et
N N + N
Et (Z)
■ Enolization selectivity Et (E) Et Me
R
■ Enolate-electrophile face selectivity H
O ‡ ‡
N R
R Li Me O
■ Racemization attendant with Xc removal N Et N Li
H Me R
Et N Et
Base Ratio, (E):(Z) H H
Et
LDA (THF) 0 : 100 disfavored favored
O OM
enolization s-BuLi (THF) 25 : 75
R R
XC XC
■ Amide Based Chiral Auxiliaries
Li
El(+)
O CH2OH M O O CH2OH
2 LiNR2 O El(+)
Me Me
N Me N
O O N
El
R hydrolysis R
RO XC
El El With Takacs,Tetrahedron Lett. 1980, 4233 diastereoselection Ca 95 %

Allylic Strain controls Enolate Geometry:


■ Enolization selectivity: Ester-based chiral controllers XC limited by
H H
enolization selectivity (Lecture 23)
OLi strongly R R strongly
OLi O C N O C N
O favored R R disfavored
LM–NR2 Me Me H H Me
Me RX + RX
RX (Z)
(E) Me
Allylic Strain Prevents Product Enolization:
Base R-Substituent Ratio, (E):(Z)
El H
LDA (THF) -OMe, O-t-Bu 95 : 5
strongly R R strongly
LDA (THF) -S-t-Bu 95 : 5 O C N O C N
favored R R disfavored
Me H El Me

24-06-Enolate alk-4 11/10/03 8:47 AM


D. A. Evans Enolate Alkylation: Chiral Amide Enolates Chem 206

Chiral Amide Enolates Amide Hydrolysis


HOH2C O
R–X Me (major) O CH2OH O O
N
M R + R R
R H
N N O
O Li
HOH2C O Me HO Me H2 N
O
+
M-NR2 Me
N HCO3-
N
H H2O, 5 min
Me O O
R O
HOH2C O R C H H2O R
OH O N O
Me (minor) O
R–X N Me H H Me HN
R
Evans, Takacs, intramoleclar general base catalysis
Tet. Lett. 1980, 21, 4233-4236 Br 96:4 (98%)

Me
Br 98:2 (84%) Applications in Ionomycin synthesis
Me Me
O O
Chem 3D model Ionomycin Calcium Complex H Me H Me
Li Li Me OH
OH
Li JACS 1990, 112, 5290-5313
Me
OH Ca O
O O
17
Me O O O 1
N O
14 14 12
9
Me2HC
LDA Me Me Me Me Me
Li
O O O O
R–X Me 84% 14
The nature of enolate chelation is ambiguous. Nitrogen chelation is a real possibility. N O Ph N O
14
PhCH=CHCH2Br
Me
Me2HC Me2HC
Me O
diastereoselection 99:1 K
Me O CH2OH Li O
N
14 12
O
OH Me Ph N 83% 14 Me
Myers, JACS 1997, 119, 6496 Me Me Ph I N
diastereoselection 97:3 Me

24-07-Enolate alk-5 11/9/03 9:05 PM


D. A. Evans Enolate Alkylation: Chiral Imide Enolates Chem 206

M O O
Alkali Metal enolates: O O Enolate Amination
O Trisyl-N3
R R JACS 1987,109, 6881.
LDA N O El(+) R M=K N O
XC HOAc JACS 1990,112, 4012-4030
or NaNTMS2 N3
O O El Bn
Me Ph M
R O O
N O diastereoselection 91-99+ %
R
M N O
R1 R2 O O O
R El(+) R Bn O O
N O XC
enolization selectivity BocN=NBoc R
El M = Li N O JACS 1986,108, 3695.
>100:1 Me2CH CHMe2 Tet. 1988, 44, 5525-40
Alkyl Halide BocHNNBoc
Ratio
Bn
Me2HC SO2N3
ArCH2Br 50-120 : 1 diastereoselection 97-99+ %
M = Li, THF < 0 °C CH2C=CHCH2Br 50 : 1 (Trisyl-N3) CHMe2
M = Na, THF -78 to 0 °C ArCH2OCH2Br 50 : 1
marginal reaction CH3CH2I 25 : 1
CH3I 13 : 1
Enolate Hydroxylation

Enolate Acylation O O O O O
JACS. 1982,104, 1737.
R PhHC NSO2Ph R
O O O O O N O N O
Na-N(TMS)2
Me Li–NR2 OH
N O Et N O JACS 1984, 106, 1154.
Me Ph Me Ph
O
Me
Imide (R) Ratio Yield *
Me2CH R Cl Me2CH
Na enolate is required.
Diastereoselection ~ 97 : 3 Why? PhCH2- 94 : 6 86 %
CH2=CHCH2- 95 : 5 91 %
MeO2CCH2CH2CH2- 96 : 4 68 %
Ph- 90 : 10 77 %
Me3C- >99 : 1 94 %
New stereocenter not lost
through enolization JACS. 1985,107, 4346.

For all indicated rxns, as the R on the enolate grp increases in


X-ray structure size enolate-El face selectivity increases. Explain.

24-08-Enolate alk-6 11/9/03 9:06 PM


D. A. Evans Enolate Alkylation: Chiral Ester Enolates Chem 206

Chiral Ester Enolates Helmchen, Angew. Chem. Int.Ed. 1981, 20, 207-208 Chiral β-Keto Ester Enolates
CO2t-Bu
Me Me
SO2Ph Me2HC N O
Li N RO El
Me N Me O THF
CO2t-Bu OMe
Me O LiCHIPA
(LiCHIPA) R Li
Me THF, HMPA Me2HC NH O Me2HC N O A
THF H O 4:1
Me LDA CO2t-Bu
Me Me Me Me OMe OMe
SO2Ph SO2Ph toluene
Me2HC N O
N Me Me N H Me El
O HMPT
O OMe
Me H Me Me
H LiO Me Rationalize the effect of HMPA on B
HLiO Me the stereochemical outcome of reaction.
(E)-enolate (Z)-enolate
El(+) addend Yield Ratio (A:B)
El(+) El(+)
Me–I THF 63% 96:04
Me Me Me Me Koga, JACS 1984, 106, 2718-2719 Me–I HMPT 57% 01:99
SO2Ph SO2Ph
Bn-Br THF 48% 99:01
N Me N El
X X Bn-Br HMPT 77% 15:85
O O
Me El Me Me
H O H O
enolate El(+) Ratio Chiral β-Keto Ester Dienolates
(E) n-C14H29–I 98.5:1.5
Me Me Me Me
(Z) n-C14H29–I 06:94 enolate contamination O
OLi
El(+)
Helmchen, Angew. Chem. Int.Ed. 1984, 23, 60-61 N N Major diastereomer
O O
Helmchen,Tet. Lett. 1983, 24, 1235-1238
Me Me Me
Helmchen,Tet. Lett. 1983, 24, 3213-3216
Me Me t-BuLi E(+) = Me–I, Et–I, Bn–Br
SO2Ph Me Me diastereoselection 98%
SO2PhH
N Me Me Me
O KO-t-Bu N O
Br X
Me O N Rationalize the stereochemical outcome of reaction.
H O Me H O
Me H O
Me
Ratio, 93:7 (74%) Schlessinger,Tet. Lett. 1988, 29, 1489-1492
Helmchen,Tet. Lett. 1985, 26, 3319-3322
24-09-Enolate alk-7 11/9/03 9:09 PM
D. A. Evans Allylic Strain & Enolate Diastereoface Selection Chem 206

O OTs O Ph OM Ph O
Me NH4Cl
Me
EtO LiNR2 EtO Me3Si OMe Me3Si OMe
diastereoselection 98:2
R R
n-C4H9 n-C4H9
H H R-substituent diastereoselection

R = Me 87:13
O O
Me R = Et 80:20

EtO R = CHMe2 40:60 major diastereomer opposite


EtO LiNR2 diastereoselection 89:11
to that shown

I. Fleming & Co-workers, Chem. Commun. 1985, 318.


MeI n-C4H9 Y. Yamamoto & Co-workers, Chem. Commun. 1984, 904.
n-C4H9 H
H
D. Kim & Co-workers, Tetrahedron Lett. 1986, 27, 943. Ph O Ph O
LiNR2 H
Me3Si OBn Me–CHO Me3Si OBn diastereoselection 90:10 at C3
one isomer at C2
MeO MeO 71% yield Me OH
RO2C O RO2C O
p.
p. LiNR2
H "one isomer" I. Fleming & Co-workers, Chem. Commun. 1986, 1198.
Br
H 95% yield H
CO2Me CO2Me Me O S Me O S
H
G. Stork & Co-workers, Tetrahedron Lett. 1987, 28, 2088. Bn Sn(OTf)2 Bn
N N S N N S diastereoselection >95%
R–CHO
Boc Boc
91-95% R OH
Me CH2 Me CH2
TBSOCH2 T. Mukaiyama & Co-workers, Chem. Letters 1986, 637
LiNR2 TBSOCH2
"one isomer"
MeI Me H O Me O
(MeS)3C–Li MeS
H H MeS
CO2Me CO2Me Me OMe Me–I OMe diastereoselection 99:1
T. Money & Co-workers, Chem. Commun. 1986, 288. 86% MeS Me
K. Koga & Co-workers, Tetrahedron Letters 1985, 26, 3031.

R OM R O
MeI
I
PhMe2Si OEt PhMe2Si OEt OLi
CO2Et H
Me R = Me: diastereoselection 99:1 R CO2Et
O-t-Bu R = H: one isomer
R = Ph: diastereoselection 97:3
KOt-Bu CO2-t-Bu R = Me: > 15 :1
THF -78 °C H
I. Fleming & Co-workers, Chem. Commun. 1984, 28. R
Y. Yamaguchi & Co-workers, Tetrahedron Letters 1985, 26,1723.

24-10-enolate alk/A-strain 11/9/03 9:10 PM


D. A. Evans Metalloenamines-1 Chem 206

■ Seminal Paper: Stork & Dowd, JACS, 1963, 85, 2178-2180 ■ Representative Reactions:
Me Me Me
■ Reviews:
Et–MgCl H3O+
Martin in Comprehensive Organic Synthesis, 1991; Vol 2, Chapter 1.16, pp 475-502
Whitesell Synthesis, 1983, 517-535 Me2CH–I
N ∆ N O
Bregbreiter in Asymmetric Synthesis, 1983; Vol 2, Chapter 9, pp 243-273 conditions: 60% overall
Enders in Asymmetric Synthesis, 1984; Vol 3, Chapter 4, pp 275-339 base + R-X in refluxing THF Me Me Me Me

■ Generation & Structure:


Me
M R R Et–MgCl H3O+
N N 83% overall
pKa~ 29-33 Me–I
El N O
El(+)
R (Z) anion syn product Me Me
N
H
base
Me CMe3 Et–MgCl H3O+ 60% overall
N n-Bu–Br
Me O
R M R H3O+ Me
The base: N N
R–Li; RMgX; R2N–Li Stork & Dowd, JACS, 1963, 85, 2178-2180
El
El(+)
(E) anion anti product ■ Nature of N-substituent, base, and solvent additive can play a role in
deprotonation regioselectivity: Hosomi, JACS, 1982, 104, 2081-2082
Acidity Measurements: (Streitwiser, JOC 1991, 56, 1989; Fraser, ibid. 1985, 50, 3234): +
Me
base H3O
+
R Me–I
Kinetic product geometry strongly favors Fraser, JACS 1978, 100, 7999 N O O
the syn isomer (>99%) (Fraser) Fraser, Chem. Commun. 1979, 47 Me Me
Me Me
■ Solid State & Solution Structure: R base ratio

X-ray structure reveals the following: Collum, JACS 1984, 106, 4865-4869 –cyclohexyl s-BuLi 10:90
❐ Anion geometry is (Z) Collum, JACS 1985, 107, 2078-2082 –NMe2 s-BuLi 100:0
❐ For M = Li, anion is delocalized Collum, JACS 1986, 108, 3415-3422
rather than localized as pictured R
Collum, JACS 1993, 115, 789-790 N O O
base H3O+
Bu Bu
Me Me Bn–Br Me
■ Geometry Rationalization: R nonbonding N-lone pair may be
N stabilized by delocalizatin into Bn Bn
antibonding orbital of C=C. R base ratio
Remember, (Z) geometry also –cyclohexyl s-BuLi 74:26
favored for enol ethers + 1 equiv HMPA
–cyclohexyl s-BuLi 100:0
24-11-met-enamines-1 11/9/03 9:11 PM
D. A. Evans Metalloenamines-2 Chem 206

Stereoelectronic Issues: Chiral Metalloenamines:


Meyers, J. Am. Chem. Soc 1976, 98, 3032
Bn Li Bn early papers:
N N Whitesell, J. Org. Chem. 1978, 42, 377-378
H H
Me H
LDA MeI Me3C Me3C Meyers, J. Org. Chem 1978, 43, 892
full papers: Meyers, J. Am. Chem. Soc 1981, 103, 3081
H X Me X Meyers, J. Am. Chem. Soc 1981, 103, 3088
CMe3 CMe3 Ratio, 97:3
R2 R2
Fraser, JACS 1978, 100, 7999 (handout)
O O
Tendency for axial-chair alkylation is significantly greater that for ketones M
R
N R1 N R1 N
H H Me H H
Me Me Me El
LDA base El(+)
Me3C Me3C Me3C
MeI H Me
H X H X H X
The base: Major Product
X Ratio R–Li; RMgX; R2N–Li
Fraser, JACS 1978, 100, 7999 (handout)
X = N-Bn 94:06 MeO
X=O 60:40
N Bn O
NNMe2 El
H H
CN Me CN LDA El(+) H3O+
LDA Me3C Me3C
MeI
NC Me R–X ee
NNMe2 NNMe2
CMe3 Me–I 87
Ratio, 96:04 Meyers, J. Am. Chem. Soc 1981, 103, 3081
Et–I 94
n-Pr–I 99
NNMe2 Me H Chiral Metallated Hydrazones
H H
Me LDA
H Me
MeI N N
Me NNMe2 Me NNMe2 N N
CH2OMe LDA
Ratio, 90:10 R CH2OMe
R–X
Collum, JACS 1984, 106, 4865-4869 (handout)

Enders in Asymmetric Synthesis, 1984; Vol 3, Chapter 4, pp 275-339

24-12-met-enamines-2 11/10/03 8:42 AM


D. A. Evans, K. Scheidt Metalloenamine X-ray Structures Chem 206

LI

LI

LI

Me
Li N Li
N Me N
H H

LI

LI

Collum & Clardy, JACS 1984, 106, 4865


LI

24-13 Metalloenamline X-rays 11/3/00 7:34 AM


D. A. Evans SAMP-Metalloenamine X-ray Structure Chem 206

Chiral Metallated Hydrazones


Me MeO THF deleted
O
N
N Li N N
N LDA N R–X
CH2OMe R
Li

H Me
R O
H
N N R
R N N
Li
Li R
O diastereotopic face
Me A (Enders) attacked by El(+)
B

Which of the reactive chelate conformations are we to begin our analysis from?

Li

diastereotopic face
attacked by El(+)

For a review of this methodology see Enders, D. in Asymmetric Synthesis.;


Morrison, J. D., Ed.; AP: New York, 1984; Vol. 3, p 275-339.

24-14 X-ray 11/10/03 8:37 AM


Chemistry 206

Advanced Organic Chemistry

Handout–24A

Enolate Acylation

D. A. Evans Monday ,
November 10, 2003
H24-00-Cover page 11/10/03 8:03 AM
D. A. Evans Enolate Acylation Acylation & Carboxylation Chem 206

The Reaction: OM O O O Deacylation: When an acyl residue is employed in the one of the
illustrated bond constructions, it may then be removed by
Acylation + nucleophilic deacylation: Several examples are provided.
R1 X R3 R1 R3
R2 R2 O O O O
Deformylation: CHO
OM O O O Me – Me
Me HCO3 Me
Carboalkoxylation +
R1 X OR3 R1 OR3 Me Me
R2 R2
Me Me
Situations where the reaction is employed: competitive ring cleavage not a problem due to more electrophilic formyl C=O

■ Acyl moiety is a constituent of the target structure: Decarboxylation:

O O O O ■ Alkyl-Oxygen Cleavage: tert-butyl esters


O O
OH O R1 OR3 R1 (–) + O
X (+) OR3 NaH, DMF R CF3CO2H
R2 R2 CO2-t-Bu R
R1 OR3 CO2-t-Bu R–Br
O O ∆
O O
R2
R1 OR3 + H H
R1 X OR3 O O O
R2 –CO2
R2
O
■ Acyl moiety employed in assisting bond construction but not part
of the target structure:
O O O O O Decarboxylation in this system is a sigmatropic rearrangement involving
R C=O participation
R (–)
H H representative procedure: Henderson, Synthesis 1983, 996
+ R–X
(+)
■ Alkyl-Oxygen Cleavage: Methyl esters
Me Me O O O
O O R Li–I/H2O ∆ R H3O+
CO2-Me CO2– CO2 R
Me
CO2Me Me N Me
CO2Me
CO2Me
Me Me R=H
O O CO2Me O O leading references
O– HO–
JOC. 1991, 56, 5301-7
C O Tet Let. 1990, 31, 1401-4
OR Acylketene
intermediate
H24-01-Acylation Intro 11/5/00 5:20 PM
D. A. Evans Claisen Condensation & Related Processes Chem 206

■ Claisen Condensation: Condensation of 2 esters ■ Analysis of the two processes:


O O O O Conventional Carbomethoxylation: Equilibrium achieved between all species
R RO–
R CO2Me CO2Me
OR + OR H O+ OR
3 O–
R O O–
R Keq ~ 10+4
■ Intramolecular Variant: Dieckmann Condensation A
Me2CO3
Me Me
O + MeOH
CO2Me RO– Keq ~ 10-2 + MeO–
H3O+ O–
CO2Me CO2Me O Keq > 10+4
O– B
Me2CO3
Strictly speaking, the Claisen and Dieckmann condensations are defined as condensations CO2Me
between ester enolates & ester electrophiles. CO2Me
In this discussion, we choose to liberalize the classifcation to include ketone enolates as well.
Critical issue: Product enolate A is significantly destabilized by peri-interaction
■ Reaction Thermodynamics: Overall Keq ~ 1 with aromatic ring disrupting the required planarity of the delocalized enolate.
O Hence, the greater stability of B dictates the product.
O O
MeO O
RO– + 2 R RO– H
OR OR + O–
O– Keq >> 1
R R B
+4 A
■ Final enolization Step: Keq ~ 10 CO2Me

O O O– O ■ This type of control is general:


– R R + ROH
RO + OR OR CO2Et CO2Et
R pKa 12 pKa 16
R HCO2Et OH
Contrary to popular belief, final enolization step does not render the process irreversible KOtBu
O O
Me Me Me Me
Reaction Control Elements: These reactions can be manipulated to give Meyers, JOC 1976, 41, 1976
either kinetic or thermodynamic control:
CO2Me HCO2Et OH
LDA O -78 °C O Me MeO – Me
kinetic product O benzene O
Piers, Tet. Let 1968, 583
O NC OMe
Me Me
HCO2Et
O O
NaH 0 °C Thermodynamic Me MeO– Me JACS 1965, 87, 5728
product O benzene O
MeO OMe
CO2Me OH

H24-02-Claisen Condensation 11/5/00 5:17 PM


D. A. Evans Kinetic Enolate Acylation: The Mander Reagent Chem 206

■ Kinetic Acylation: Methyl Cyanoformate (1): ■ The Tetrahedral Intermediate 2; Why is it so stable?
Li
OLi O O O Li
-78 °C O O O O
+
R1 NC OMe R1 OMe 2 slow
+ LiCN
fast CN R1 OMe R1 OMe
R2 1 R2 CN
R2 R2
Enolate acylation with 1 is fast 2
slow
Consider this process in the broader context of elimination reactions
Intermediate 2 breaks down to product of the E1cb classification where:
more slowly than the acylation step
Under these conditions, proton transfer O O Y might be either C or some heteroatom
from product to enolate does not occur. X might be various leaving groups such as CN, OR etc.
+ LiCN
R1 OMe R
R R R R
R2 base slow
X Y X Y– + X–
R Y
■ Examples: O O
H
CO2Me
LDA Data is available for the case where X = CN, OR & Y = carbanion:
84%
1 Stirling, Chem. Commun. 1975, 940-941

O O
LDA FG base FG slow
CO2Me FG
65% X X –
Me Me 1 Me
H + X–

O O kX
LDA leaving grp pKa
CO2Me log
Me Me 1 Me (X) H–X kOPh

Mander Tet. Lett. 1983, 24, 5425 –OPh 10 1


OTMS
–CN 9.5 <-7
Me
Me-Li O
–C(Me)2-NO2 ~10 <-9
Me3C Me + isomer
1 –OMe 16 -3.9
H CO2Me 7%
CMe3 75%
Mander, SynLett. 1990, 169
Above data makes the point that CN is a poor LG but it also leads one to the
O O faulty conclusion that 2 should partition to acyl cyanide rather than methyl ester!
CO2Me Li
R2Cu(CN)2Li2
1 O O O O O
O
OTBS
R1 CN R1 OMe R1 OMe
82% Hashimoto, Chem. Lett. 1989, 1063 CN R2
R2 R2
+ LiOMe 2 + LiCN

H24-03-Mander Reagent 11/5/00 5:21 PM


J. L. Leighton, D. A. Evans Carbon Acylation with N-Methoxy-N-methylamides Chem 206

Acylating agents can be desiged where the tetrahedral intermediate O


exhibits exceptional stability: OR O OR O O
Li P(OMe)2
MeO2C Me MeO2C P(OMe)2
O Li N THF/Et2O
O O
Me Nu(-) O Me H3O+ OMe -110 °C to -80 °C
R N R 62% P. Thiesen and C. Heathcock
N R Nu J. Org. Chem. 1988, 53, 2374.
OMe Nu
Me O BrMg OEt O
O CbzHN Me CbzHN OEt
O N
Me R-Li or R-MgBr H3O+ Bn OMe THF, -78 °C
Bn
R' N R' R 73%
THF, 0 °C
OMe M. Angelastro, N. Peet and P. Bey
R = Me, n-Bu, or Ph; yields > 90% J. Org. Chem. 1989, 54, 3913.
O O
Weinreb Tet. Lett. 1981, 22, 3815. O
Nucleophiles: Me Me R1–M Me R2–M
N N H3O+ R1 N H3O+ R1 R2
MeO OMe OMe
Acceptable Unacceptable W. Wipple, H. Reich
Enolates and Metalloenamines: J. Org. Chem. 1991, 56, 2911-2.
R–Li, R–MgX R Li(MgX) R–ZnX & other colalent metal alkyls
OLi O O
Li
OLi LiN O OLi t-Bu t-Bu
O O 83%
Ar S CH2Li other colalent metal enolates
RO R R O THF, -78 °C
O
Me OLi
DIBAL LiAlH4 LiB(R)3H Weak hydride reagents: NaBH4 N O O
OMe Me Me 47%
An excellent review on all aspects of Weinreb amide chemistry: THF, -78 °C to R. T.
M. Sibi, Organic Preparations and Procedures Int., 1993, 25 (1), 15-40.
J. Org. Chem. 1989, 54, 4229.
Hydride Reductions:
Representative Organometals: OMe OMe OMe TBSO O
OTBS O OTBS O O2N OMe DIBAl-H
N
OMe THF, -78 °C
95%
Me N MeMgBr Me Me Me OMe Me Me Me Me
N Me THF, 0 °C N
99% OMe OMe OMe OMe TBSO O
O Ar O Ar
O2N
J. Prasad and L. Liebeskind D. Evans and S. Miller H
Several other examples reported.
Tetrahedron Lett. 1987, 28, 1857. J. Org. Chem. 1993, 58, 471. Me OMe Me Me Me

OMe
H24-04 Weinreb Amides-1 11/5/00 5:22 PM
J. L. Leighton, D. A. Evans Carbon Acylation with N-Methoxy-N-methylamides-2 Chem 206

The Rutamycin B Synthesis, H. Ng, Ph. D. Thesis, Harvard University, 1993 Me


Me 29
OR OR O O O
A E
H
Me Me Me Me D 27
Me
Li NMe2 NMe2 O 7 11 Me + H 17 21 O O
N I O O N O O 16 Me OMe Me F
Me Me Me Me OR O Me
97% O 1 OH 35
Me Me OH
Me Me C1-C16 Subunit C17-C37 Subunit Et
26 32 26 32
Me Me
Problem is to control C=O reactivity on central D-fragment
TESO O LDA Me
O O H
20
O Me
OMe
PMBO 23 N M D
H 28 29 F
17 X OBn
H O Me
21
80% O
Et Me Me
Me Me OMOP O Me
Me Me Et
H NMe2
TESO O N O O
20 O O MeHO Me
PMBO 23 Me D
HF 26 32 21 30
Et H O 28
MeCN-H2O 80% Me Me F Me
Me Me
Me Me Me Me OH O O
O 5 OH
35 OH
HO 32 Me 33 H Et
1
O
H
H O O
H Me OH H Me O Me
H O O 25 The Solution: Me Me
H 8
Me OH Et2O, -78 °C Me 29
20 Me N
PMBO O H Li H O Me
H H
26 O MeO OMOP N
Et O Me C20–C21 NMe2
Me several steps Me CH2
20 12
HO Me
Et Me Me
Me Me O Li
Me MOP = C OMe
Evans, Rieger, Jones, Kaldor, JOC, 1990, 55, 6260-6268 Me MeN O
17 tetrahedral intermediate Me
stable for hours at 0°C H H O Me
Me CH2 OMOP N
The X-206 Synthesis, S. L. Bender, Ph. D. Thesis, Harvard University, 1986 NMe2
Me
Me Me Me Me O H
Me 29 LDA, 0 °C
OH 12
A B C 27 E
H
15 21 D OBn
Me O
O 7 O O O O 83% C –C Me
H OH OH Me OH Me F 28 29
Me OH O Me Et
O 1 OH 35
OH Me Me Me
Et O
Me H
OH OH O OH O MeHO Me H O
A
D H O Me Me
Me 27 NH O OBn
O 11 15 21 30 Me CH2 OMOP
7 O Me
Me F Me NMe2
Me Me Me Me OH O O Et
O 1 OH 35 OH
Evans, Bender, Morris J. Am. Chem. Soc. 1988, 110, 2506.
Et
H24-05-Weinreb Amides-2 11/5/00 5:25 PM
D. H. Ripin, D. A. Evans The Eschenmoser Coupling Reaction Chem 206

Key Bond Construction Needed for the B12 Synthesis:


N (+)
H O N
(+) R 3P R3P=S
N
N
? H
H O H (–) N
The Problem:
? H N
H (–) N
N N S
H
N
The Solution: N
O S
O
Me N H C
O Me
(CH2)2CO2Me O H S N
Me (CH2)2CO2Me
N (+)
Me X
H ?? N
O N E. Knott J. Chem. Soc. 916 (1955)
H
H (–) N
N Me
Me O Me
O Me The General Reaction: Acylation of an Amide C=O
(CH2)2CO2Me
(CH2)2CO2Me O
S
O Base, Thiophile R3
R”
O R N X
MeO2CCH2 R3 R”
R’ R N
O Me
Me O R’
R A. Eschenmoser Helv. Chim. Acta. 54, 710 (1971)
O MeO2CCH2 NH Me Key papers:
N A. Eschenmoser Angew. Chem., Int. Ed. Engl. 6, 866 (1967)
Me A. Eschenmoser Angew. Chem., Int. Ed. Engl. 8, 343 (1969)
R2 Me H
N Review: Trost Comp. Org. Synth. Vol. 2, Ch. 3.7 (1991)
OO MeO2CCH2 N
Me R3 Me
R2 Me Me (–)
NH N Me O (+) O S
Me
Me H (CH2)2CO2Me
(CH2)2CO2Me The Thioamide component: R R” Reagents R”
N N ?? N R N
R2 Me
S S R’ R’
Me Me p-MeOPh P P PhOMe
R3 R3 A. Eschenmoser Science 196, 1410 (1977) S P4S10 JOC 46, 3558 (1981), Synthesis 149 (1973)
S
Lawesson's Reagent Bull. Chim. Soc. Belg. 87, 229 & 293 (1978)
P4S10, Et3N or NaHCO3 Indian J. Chem., Sect. B 14, 999, (1976)
JACS 102, 2392 (1980)
RCS2R' + R2NH ThioamideChem. Ind. (London) 803 (1974)
Imidate +H2S ThioamideAngew. Chem. 79, 865 (1967)
H24-06-Eschenmoser-1 11/5/00 5:27 PM
D. A. Evans and P.H. Carter Intramolecular Enolate Acylation–Dieckmann Condensation Chem 206

The Dieckmann Condensation Regioselectivity: ONa


EtO2C A
CO2Et CO2Et
Reviews: Schaefer, Bloomfield, Organic Reactions 1967, 15, 1. NaH/C6H6
Davis & Garratt, Comprehensive Organic Synthesis 1991, 2, 806-829 Kinetic Control?
CO2Et CO2Et
A
O
CO2RCO2R ONa
CO2R NaOEt/EtOH
( )n ( )n EtO2C CH2CO2Et
EtO2C B
CO2Et Thermodynamic
B1
Control?
Accesible Ring Sizes CO2Et
ONa
O O O O O
CO2R CO2R NaOEt/EtOH EtO2C Not observed
CO2R CO2R
CO2R
Explanation: B2
CO2Et
not viable excellent excellent acceptable situation dependent Enolization at (A) preferred on basis of inductive effects. Hence,
high dilution required
Path A preferred in kinetically controlled situation
The individual steps:
Enolates (B1) and (B2) both more stable than enolate (A)
O O Under equilibrating conditions (B1) appears to be preferred over (B2)
Enolization:
OEt OEt
EtO EtO
O O–
+ base + base-H The effect of beta heteroatoms: classical kinetic vs. thermodynamic control
A variety of bases may be considered for the enolization step.
Either alkoxide or a non-nucleophilic base such as NaH are commonly used.
Choice of base can be important (Vide infra). KOtBu / PhH
N CO2Et
MeO ∆
Ring Closure: Keq (cycliz) Keq (enoliz) N CO2Et OH
O ~1 O ~ 10+4 O– MeO
NaOEt
OEt CO2Et EtOH
CO2Et CO2Et N
EtO NaOEt / EtOH ∆
O– Keq (enoliz) ∆
+ EtO– OH
+ EtOH
R.H. Schlessinger, et al. Heterocycles 1987, 25, 315. CO2Et
Statements claiming that the final enolization step renders the process
irreversible are simply incorrect.

H24-07-Dieckmann-1 11/5/00 5:29 PM


D. H. Ripin, D. A. Evans The Eschenmoser Coupling Reaction-2 Chem 206

Reagents for the Reaction:


CO2Bn
O O
Bases: Me
Inorganic: MHCO3, MOH, MH, MOR TfO Me t-BuO2C N
O O
Organic: R3N, N-methylmorpholine, buffered solutions mix Bn CO2Bn
Et3N, PPh3
t-BuO2C N S
Thiophiles: Ar3P, R3P, (RO)3P
O Bn H. Rapoport J. Org. Chem. 46, 3230 (1981)
PhP NMe2 PhP N
Combination: 2 2
H. Rapoport J. Org. Chem. 46, 3230 (1981)
A. Eschenmoser Helv. Chim. Acta. 54, 710 (1971) CO2Et
OAc CO2Et OMe
MeN CO2Me
MeN OAc Me MeO
A. Eschenmoser Science 196, 1410 (1977) DBU * NaH OAc
O S
O N CuBr
H 79% N Me N
Me
O Me Ar H CO2MeBr 99% NMe
(CH2)2CO2Me O
EtO2C
Me (CH2)2CO2Me Br CO2Me
N (+)
a) 1.05 eqiv. (PhCOO)2 Me
H N T. Kametani J. Chem. Soc., Perkin Trans. I 1607 (1980)
S This center readily
b) P(OEt)3, Xylene, ∆ H epimerizes to a 2:1
H N mix of diaster. in
N Me favor of the shown.
Me 84% O Me
N2
O S S O
Me N2 NaOH
(CH2)2CO2Me ii
(CH2)2CO2Me O
HN N N
O
O CO2t-Bu CO2t-Bu CO2t-Bu
MeO2CCH2
Rh(OAc)2 ∆ Ra-Ni, 68%
O O
Me O
R2 Me R + S
MeO2CCH2 NH Me
OO N S O
Me R3 Me S. Danishefsky
H –O
R2 Me N Tet. Lett. 30, 3625 (1989) N N
NH N MeO2CCH2 N
Me H Br Me
CO2t-Bu CO2t-Bu
N N Me S Me
R2 Me (CH2)2CO2Me
(CH2)2CO2Me
Me Me
t-BuOK
R3 R3 t-BuOH, 25 °C R R R R
64%
C: + S – S+
P(CH2CH2CN)3,
TFA, sulfolane R R R R
carbenes Thioethers S-Ylids

H24-08-Eschenmoser-2 11/5/00 5:27 PM


D. A. Evans and P.H. Carter Intramolecular Enolate Acylation–Dieckmann Condensation Chem 206

Miscellaneous Dieckmann Reactions of Potential Interest Intramolecular Ketone Acylation

O O
O
EtO2C R NaH H
DMSO R CO2Et NaH O

CO2Et Me Me
Me
SEt EtS CO2Et Me Me
O Me Me
R.Danieli, J.Org.Chem. 1983, 48, 123. O
H.-J. Liu and Co-workers, Tet.Lett. 1982, 23, 295.
TMS
(tBOC)HN OTMS
N CO2Et OEt H O
tBOC LDA H
8:1 mixture CO2Et KH, THF
no loss of
S CO2Et S CO2Et stereochemical
N Me N integrity
TsHN TsHN
J.L. Adams, J.Org.Chem. 1985, 50, 2730. O O
X T.M. Harris and Co-workers, J.Org.Chem. 1984, 49, 3681.
X Me
Me
CO2Et
tBuOK / PhH
O O
O
(TMS)2NLi
CO2Et O O Me THF, -78oC O
CO2Et
Peterset, Recl.Trav.Chim.Pays-Bas 1977, 96, 219. Me Me
X HO
Me Me

When X = NR2, this is a good reaction, but when X = OR, it is a poor reaction.

Deduce the mechanism of this multistep process. S.Brandawge and Co-workers, Tet.Lett. 1992, 33, 3025.

O O O
Cl Me O
NaOEt
Me Et2O Cl O Me OMe O
CO2Et LDA
CO2Et R O -78 °C O
H Me Me
G.Stork and Co-workers, J.Am.Chem.Soc. 1960, 82, 4315. OMe

Kocienski and Co-workers, Tet. 1990, 46, 1716

H24-09-Dieckmann-2 11/5/00 5:30 PM


D. A. Evans and P.H. Carter Intramolecular Enolate Acylation–Dieckmann Condensation Chem 206

Kinetically controlled Cyclizations Multistep Condensations


O Cl
CO2tBu Cl
CO2Me CO2Me CO2Me
LDA CO2Me
CO2Me NaH
N CO2Me
N MeOH
Me OH
Me MeO O
Heterocycles, 1987, 25, 315 MeO O OH
MeO2C CO2Me

Me R Me R
NMe2
Li-TMP
OH
JACS, 1979, 101, 5060
Woodward, JACS, 1962, 84, 3222
CO2Me 60-70% CONH2
O OH
OH O OH O
CO2Me CO2Me 6-demethyl-6-deoxy-tetracycline

H H
COSPh O
Li-NTMS2
O Tet. Let, 1981, 22, 3883 DMSO– CO2Me
N N MeO2C O
CO2Me 41%
O O
CO2Bn CO2Bn O
MeO2C
H H MeO2C Danishefsky, JACS, 1973, 95, 2410
PhOCH2CONH Li-NTMS2 H H
PhOCH2CONH
3 equiv
N COSPh
O N
OH O
O
CO2tBu O
CO2tBu CO2Me
Me CO2Me
Hatanaka, Tet. Let, 1983, 24, 4837 EtO–

O OMe EtO2C CO2Et 41%


O
CO2Me C HO
O
H C OCOMe Li-NTMS2 H C O
HO
HO C H 3 equiv LiO C H OLi O O
O
CO2Me CO2Me CO2Me CO2Me
78% Prostaglandin E2

Brandange, JOC, 1984, 49, 927 Sih, JACS, 1975, 97, 865
HO OH
H24-10-Dieckmann-3 11/5/00 5:31 PM
D. A. Evans Synthetic Applications of Metalloenamine Nucleophiles Chem 206
The Rutamycin B Synthesis, H. Ng, Ph. D. Thesis, Harvard University, 1993
JACS 1993, 115, 11446-11459. Me
Me 29
OR OR O O O
H
Me Me Me Me Me
A D 27 E
Li NMe2 NMe2 O 7 11 Me + H 17 21 O O
N I O O N O O 16 Me OMe Me F
Me Me Me Me OR O Me
97% O 1 OH 35
Me Me Me Me OH
26 32 26 32
C1-C16 Subunit C17-C37 Subunit Et
Me Me
Problem is to control C=O reactivity on central D-fragment
TESO O LDA Me
O O Me O H
20 OMe
PMBO 23 N M D
H 17 X
28 29 F
OBn
H O Me
21
Et Me Me 80% O
Me Me OMOP O Me
Me Me Et
H NMe2
TESO O N O O
20 O O MeHO Me
PMBO 23 Me D
HF 26 32
Et Me Me H 21
O 28 30
MeCN-H2O 80% Me O F Me
Me Me Me Me Me OH O
O 5 OH
35 OH
HO 32 Me 33 H 1 Et
H
O
H O O
H Me OH H Me O Me
H O O 25 The Solution: Me Me
H 8
Me OH Et2O, -78 °C Me 29
20
O H Me N H O Me
PMBO Li
H H MeO OMOP N
26 O C20–C21
Et O Me Me CH2 NMe2
Me several steps
20 12
HO Me
Et Me Me
Me Me O Li
Me MOP = C OMe
Evans, Rieger, Jones, Kaldor, JOC, 1990, 55, 6260-6268 Me MeN O
17 tetrahedral intermediate Me
stable for hours at 0°C H H O Me
Me CH2 OMOP N
The X-206 Synthesis, S. L. Bender, Ph. D. Thesis, Harvard University, 1986 NMe2
Me
Me Me Me Me O H
Me 29 LDA, 0 °C
OH 12
A B C 27 E H
Me 15 21 D
O OBn
O 7 O O O O 83% C –C Me
H OH OH Me OH MeO F 28 29
Me OH Me Et
O 1 OH 35
OH Me Me Me
Et O
Me H
OH OH O OH O MeHO Me
A H H O Me O Me
D
Me 27
Me CH2 OMOP NH O OBn
O 7 11 15 21
O 30
F Me NMe2 Me
Me Me Me Me OH Me O O
O 1 OH Et
35 OH Evans, Bender, Morris J. Am. Chem. Soc. 1988, 110, 2506.
Et
H24-11-met-enamine acylation 11/4/01 7:08 PM
D. A. Evans Synthetic Applications of Metalloenamine Nucleophiles Chem 206
■ The Ferensimycin B Synthesis, JACS 1991, 113, 7613-7630 Me Me
Me Me
O Li O Li
Me Me Me Me Me Me
O OH O Et-Li Me–N O B Et2NLi Me–N O
17 B
A B C Et2O 18
Et O THF O
HO O O O H Me H Me
OH H H Me OH Et Me Et N Me Et N
Me Me Me Et OH NMe2 Li NMe2
Me
O Me
Ferensimycin B B 18
MeO
N 11 O
H Me Et
Me Me Me Et N
Me Me O NMe2 21 Et
O O
A B C Et Me
Me 18 O
HO O H O O O O
H Me OH Et
OH H Me Et OH B C
21 Et
Mg
Me Me
11 O O Br
C10-C23 Synthon H Me NH Et
Me Et OH
NMe2
■ The B–C Fragment (C10-C23 Synthon)
NaHSO4
Me Me Me H 2O
18
O O Me Et
B C B 18
Et Et
11 O O 11 O Me Me Me Me
H Me OH Et H Me OH 18 18
Me Et OH Me O OH O O
Et
B C 21 Et B C 21 Et
11 O O 11 O O
H Me OH Et H Me OH Et
Me Et OH Me Et OH
Me
O Me Et
The C-11 ketone must be protected during B 18
the C-18C-19 bond construction Et
(–) PPTS C18 diastereoselection PPTS
11 O O MeOH
H Me MeOH 9 : 1 (48%)
Me Et X OH

Me Me Me Me
■ The In situ protection of the C–11 Carbonyl O
18
O 18

B C B C Et
Et
Me Me O O O O
Me H Me O H Me O
O Me O Li Me Et Me Et
B 18 Me Et Et
MeO R–M Me–N O epi-C18 6
N 11 O B 17
H Me R O
Me Et N H Me
NMe2 Et N
NMe2
carbonyl-protected intermediate

H24-12-Ferensimycin construct 11/4/01 12:32 PM


Quote for the Day

Quote and limrick by J. W. Cornforth

"Nature, it seems, is an organic chemist having some predilection


for the aldol and related condensations."

"That Outpost of Empire, Australia


Produces some Curious Mammalia
The Kangaroo Rat
The Blood-sucking Bat
and Aurthur J. Birch, inter alia."

25-12 Quotes-4 11/12/03 8:17 AM


D. A. Evans The Aldol Reaction–1 Chem 206

■ Suggested Reading
http://www.courses.fas.harvard.edu/~chem206/
Stereoselective Aldol Reactionsw in the Synthesis of Polyketide natural
Products, I. Paterson et al. in Modern Carbonyl Chemistry, pp 249-297, J.
Chemistry 206 Otera, Ed. Wiley VCH, 2000 (handout)

Ager, D. J., I. Prakash, et al. (1997). “Chiral oxazolidinones in asymmetric


synthesis.” Aldrichimica Acta 30(1): 3-12
Advanced Organic Chemistry
■ Other Useful References
Lecture Number 25
Evans, D. A., J. V. Nelson, et al. (1982). “Stereoselective Aldol Condensations.”
Top. Stereochem. 13: 1.
The Aldol Reaction–1
Heathcock, C. H. (1984). The Aldol Addition Reaction. Asymmetric Synthesis.
M Stereodifferentiating Reactions, Part B. J. D. Morrison. New York, AP. 3: 111.
M O O
O O
Oppolzer, W. (1987). “Camphor Derivatives as Chiral Auxiliaries in Asymmetric
Me R R Synthesis.” Tetrahedron 43: 1969.
R H R
Me
Heathcock, C. H. (1991). The Aldol Reaction: Acid and General Base Catalysis.
Comprehensive Organic Synthesis. B. M. Trost and I. Fleming. Oxford,
Pergamon Press. 2: 133.
■ Polyketide Biosynthesis
■ Historical Perspective on the Aldol Reaction Heathcock, C. H. (1991). The Aldol Reaction: Group I and Group II Enolates.
■ Aldol Diastereoselectivity Comprehensive Organic Synthesis. B. M. Trost and I. Fleming. Oxford,
Pergamon Press. 2: 181.
■ Enolate Diastereoface Selectivity
■ Absolute Control in the Aldol Process Kim, B. M., S. F. Williams, et al. (1991). The Aldol Reaction: Group III Enolates.
Comprehensive Organic Synthesis. B. M. Trost and I. Fleming. Oxford,
Pergamon Press. 2: 239.

■ Reading Assignment for this Week: Franklin, A. S. and I. Paterson (1994). “Recent Developments in Asymmetric
Aldol Methodology.” Contemporary Organic Synthesis 1: 317-338.
Carey & Sundberg: Part A; Chapter 7
Carbanions & Other Nucleophilic Carbon Species Cowden, C. J. and I. Paterson (1997). “Asymmetric aldol reactions using boron
enolates.” Org. React. (N.Y.) 51: 1-200.
Carey & Sundberg: Part B; Chapter 2
Reactions of Carbon Nucleophiles with Carbonyl Compounds Nelson, S. G. (1998). “Catalyzed enantioselective aldol additions of latent
enolate equivalents.” Tetrahedron: Asymmetry 9(3): 357-389.
Wednesday, Mahrwald, R. (1999). “Diastereoselection in Lewis-acid-mediated aldol
D. A. Evans November 12, 2003 additions.” Chem. Rev. 99(5): 1095-1120.

25-00-Cover Page 11/12/03 8:56 AM


Aldol Reaction Variants: Each has its Merits & Liabilities

Metal Aldol Process


M M
O O O O
Teruaki Mukaiyama
X H R2 X R2
R1 R1 Chem Lett. 1973, 1011

B–M B–M B–M(H+)

O O OH Mukaiyama Aldol Process


X X R2
TMS
O M TMS M
R1 R1 O O
O
Generally stoichiometric in Metal X
Good diastereomer control now possible H R2 X R2
R1
R1

M M
O TMS
O O

H R2
X R2
R1
Generally catalytic in Metal
Diastereomer control not highly regulated

Clayton Heathcock Satoru Masamune


30-aldol Variants 11/12/03 9:34 AM
D. A. Evans The Aldol Reaction: Polypropionate Biosynthesis Chem 206

"Nature, it seems, is an organic chemist having some predilection Polypropionate Biosynthesis: The Elementary Steps
for the aldol and related condensations."
O
J. W. Cornforth O O OH O
O
J. W. Cornforth Acylation Reduction
Me Me
Me Nobel Prize, 1975 R SR R ✻ SR
R OH R SR – CO2
Me Me Me Me
OH O
O O
Me
Et O O NMe2 Erythromycin A, R = OH HO SR
H Erythromycin B, R = H
OH Me
O O
Me
O OH O O OH OH O
Me H OH O
Acylation Reduction
Me R ✻ SR R ✻ ✻ SR
R ✻ SR – CO2
MeO
OH Me Me Me Me Me
Retro-biosynthesis: Erythromycin A
OH OH O OH OH O
O erythronolide aglycon
Me
O OH
Me Me
Me Me Me Me Me Me Me Me Me
HO OH
Me Me Erythromycin Seco Acid The 7 Propionate Subunits
OH HO OH
O Me Me
Me OH OH OH O OH OH O
Et O O NMe2 Me
H Et O OH OH
OH
O O
Me O OH The Acylation Event
O Me
Me H Decarboxylation-Acylation could either be stepwise (Option A) or concerted
Me (Option B).
MeO O
OH Me The stepwise Option
Me
OH OH O OH OH O H H Me
Me Me
OH C O
Me O O
OH SR RS
Me C
Me Me Me Me Me Me Me Et O OH H
O Me SR R SR
Erythromycin Seco Acid C C
O OH O O– H O– Me
Me
O O O C O The overall acylation
is stereospecific
Recent overview: Staunton, "Polyketide biosynthesis: a millennium review." HO SR
Nat. Prod. Rep. 2001, 18, 380-416.
Me
See Lecture 24; page 24-08 for first laboratory example
25-01-aldol intro 11/12/03 8:32 AM
D. A. Evans Polypropionate Biosynthesis: A Laboratory Simulation Chem 206

Polypropionate & Polyacetate Biosynthesis: Latter Stages of Lonomycin Biosynthesis


Develop a Laboratory Simulation
OMe OMe
O Me Me OMe
OH OH O OH OH O Me Me
HO Me Me
Me Me O
OH HO2C OMe
H OH O O O O
Me Me Me Me Me Me Me Me Me OH H H Me H
OH Me Me O
Me
Me
Et O OR Me OH
OH OH O OH OH O
OMe OMe
O OR OH Me
Me Me Me OMe
Me Me Me Me Me Me Me Me O
OHMe O
The 7 Propionate Subunits HO2C Me
Erythrolide B O O
OH H O Me O
Me Me Me OMe Me
Polypropionate Biosynthesis: The Elementary Steps

O O OH O
O O OMe OH OH OMe O OMe OMe O
Acylation Reduction
R SR R ✻ SR HO2C
R SR – CO2 Me
Me Me
Me Me Me Me Me Me Me Me Me Me
O O

HO SR
Cane, Celmer, Westley JACS 1983, 105, 3594
Me

OH O O OH OH O
OH O Bn O O OH
Acylation Reduction
R ✻ SR R ✻ ✻ SR Sn(OTf)2
R ✻ SR – CO2 R
EtN(iPr)2 N
Me Me Me Me
Me O Me Me
R–CHO
O O O O
The Laboratory Mimic: 95:5 (85% yield)
Dipropionyl Synthon O N
O O Me Me TiCl4 Bn O O
O O O
(–)
Aldol ? O O O
Bn
EtN(iPr)2
R H XC N Me
Me Me N O O Me Me
O O
See Lecture 23; page 23-08: Me Me Me OMe O
with M. Ennis JACS 1984, 106, 1154. Bn 93:7 (86% yield)

with Ratz, Huff, & Sheppard, JACS 1995, 117, 3448


25-02-aldol intro-2 11/12/03 8:34 AM
D. A. Evans The Aldol Reaction: Early Contributions Chem 206

General Reviews of the Aldol Literature: DuBois 1965-67: Rough correlation between enolate stucture & product
stereochemistry for alkali and alkaline earth enolates
Mukaiyama in Organic Reactions, 1982; Vol 28, pp 203-331 M O OH O OH
Evans in Topics in Stereochemistry, 1982; Vol 13, pp 1-115 O O
Heathcock in Asymmetric Synthesis, 1984; Vol 3, pp 111-212 X X R X R
H R
Me Me Me
Comprehensive Organic Synthesis, 1991; Vol 2 (E) Enolate anti diastereomers
Group I & II metal enolates: Heathcock; Chapter 1.6, pp 181
Group III metal enolates: Masamune; Chapter 1.7, pp 239 M O OH O OH
O O
Transition metal enolates: Paterson; Chapter 1.9, pp 301 Me X R X R
X H R
Me Me
Control relative stereochemical relationships (Z) Enolate
syn diastereomers
Zimmerman 1957:
Zimmerman-Traxler Model for (Z) Enolates
Proposed chair-like geometry for the Ivanov Reaction
X L ‡
O OH
OMgX ‡ OMgX ‡ R2CHO H
OMgBr M
Ph H O L
PhCHO H C X R2
MgBr MgBr O
OMgBr H3O+ O Ph O
Ph Me R2 Me
Ph O O M favored
O syn diastereomer
H Ph H H
i-PrMgBr Me ‡
X X L
O OH
R2
OH O OH O M
O R2CHO H O L
C X R2
Ph Ph OH Ph OH O
OH Me H Me
Ph Ph syn:anti anti diastereomer
ratio, 75:25
M = Li > 98 : 2 Heathcock 1977
X = CMe3
Zimmerman recognized that diastereoselection should be a function of M = MgBr > 95 : 5 DuBois 1972
the relative sizes of the substituents on the carbonyl component.
X = C 6H 5 M = Li 80 : 20
He also speculated on the role that the metal center might play in
controlling the process. M
O
M = Li 48 : 52
The only flaw in the study was that he failed to determine whether the M = AlEt2 50 : 50 House 1971
aldol adducts were stable to the reaction conditions.

Zimmerman, J. Am. Chem. Soc 1956, 79, 1920 (handout)


Stereocontrol optimal for "large" X; the reaction is not general.
25-03-aldol-1 11/12/03 8:40 AM
D. A. Evans The Aldol Reaction: Boron Enolates Chem 206

Why Boron? Are (E) enolates intrinsically less diastereoselective?


To tighten up the transition state. M–O B–O M–C B–C
Dialkylboron chlorides (Brown)
Design TS where control can come 1.9-2.2 Å 1.4-1.5 Å 2.0-2.2 Å 1.5-1.6 Å Now that there are good methods for preparing (E)
exclusively from metal center JACS. 1989, 111, 3441-3442.
enolates, it appears that both enolate geometries are J. Org. Chem. 1992, 57, 499-504.
X L ‡ nearly equivalent. J. Org. Chem. 1992, 57, 2716-2721.
O OH J. Org. Chem. 1992, 57, 3767-3772.
R2CHO H
B J. Org. Chem. 1993, 58, 147-153.
H O L JACS. 1989, 111, 3441-3442.
C X R2 OBChx2 O OH
O Chx2BCl
Me R2 Me PhCHO
BL2 favored Et3N Ph Ph Ph
O syn diastereomer
Et2O
‡ Me Me
Me O
X X L ~99% (E) 95% anti
O OH Me
R2 Ph
B O OH
R2CHO H O L
C X R2 9-BBN-Cl OB-9-BBN
O PhCHO
Me H Me DIPEA Me Ph Ph
disfavored anti diastereomer Ph
Et2O Me
syn:anti ~99% (Z) 98% syn
M = Li > 98 : 2 Heathcock 1977 It appears that there is not a great difference in aldol diastereoselectivity
X = CMe3
M = MgBr > 95 : 5 DuBois 1972
M = BBu2 > 97 : 3 DuBois 1972
Dissection of the Aldol Problem: Select for one product diastereomer
X = C 6H 5 M = Li 80 : 20
M O OH O OH
M = BBu2 > 97 : 3 Evans, Masamune O
1979-81 syn
X = Et M = Li 80 : 20 Me X R X R diastereomers
X
M = BBu2 > 97 : 3 O Me Me
M
O H R
M M = Li 48 : 52 House 1971 O OH O OH
O
M = AlEt2 50 : 50 Yamamoto 1977 X anti
X R X R diastereomers
M = BBu2 33 : 67 (ether) Me
Me Me
M = BBu2 17 : 83 (pentane)
M = BCy(thex) 6 : 94 (CH2Cl2) Evans, Masamune Control attack on the two enolate enantiofaces
M
O 1979-81 O
El(+)
Me
(t)BuS M = B(Cyp)2 <5 : 95 (pentane) X
Me O M El
X
Me
H
Evans et al. JACS 1979, 101, 6120-6123; JACS 1981, 103, 3099-3111
O
Relevant stereochemical information
Masamune, Tet. Lett 1979, 1665, 2225, 2229, 3937 Me could be included in either X or M
X El(+)
El
25-04-aldol-boron 11/12/03 8:50 AM
D. A. Evans The Aldol Reaction: Boron Imide Enolates Chem 206

The Alpha substituent, X, plays pivotal role in aldol diastereoselection


Imide Enolates: The problem of enolate face selectivity
R R H
R O OH O O OH
O O Li O B
O O O O R Me2CHCHO
R O
Me LDA Br-CH2R N R + O N R
O N Me O N R X
O N N O X X
Me O R
O
R R R O Ratio
Substituent
Face selectivity predicated on chelate organization X = Me > 300 : 1
Bu2B-OTf Et3N How can we rationalize these data ? X = SMe 60 : 1
R R H 1:1
X=H
B O O OH
R R O O O R
Model for Asymmetric Induction (unpublished)
B RCHO Me
O O O N R
O N L L
Me BL2 O R
Me R O
O N R B
R O N H L R O O
Me H
N favored
R O B
O H L N R
R R H O
O O Me
B R
Chelate organization precluded, O OH Me O
R O O R R
therefore face selectivity uncertain RCHO L L
Me N R H O
N BL2 H B
O Me O O R OL O O O
O N
O O
Me R B
O N H O L disfavored O N R
O Me
The aldol reaction selects for the opposite enolate diastereoface R Me R

∆∆G‡ (273 K) ~ 2.6 kcal mol -1


R R H
B R O OH L L
R O O R
B
Me N R O O
N O
O Me
O O O N R
O
Me
Result discovered but not predicted diastereoselection > 98% R

J. Am. Chem. Soc 1981, 103, 2127-2129 (Handout)

disfavored product diastereomer: The destabilizing interaction?

25-05-aldol-boron/imide-1 11/12/03 8:52 AM


D. A. Evans The Aldol Reaction: Imide Transformations Chem 206

Imide Hydrolysis Trans-esterification


OMe OMe
Imides may suffer attack at either of the two C=O functions (eq 1, eq 2)
O O
O O N3 Ti(OBn)4 N3
Bn Bn
R * – R * O O
R'O
N OR' HN (1)
El O exocyclic El O Bn N O OBn
O O
pKa 20 O 90-93% yield
O Bn O OCMe3
R'O – O OCMe3
R * (2)
endocyclic N
H (OF-4949 Synthesis) JACS1989, 111, 1063
El O–C(O)OR'
Product distribution a function of attacking nucleophile (Tet. Lett. 1987, 28, 6141) Trans-thioesterification:
MeO OMe MeO OMe
Exo:Endo Exo:Endo O Me O
Substrate Reagent Ratio Substrate Reagent Ratio Me Bn-SLi Me
N Ph SBn
Me O Bn Me O Bn THF, 0 °C
LiOH 0 : 100 O
LiOH 06 : 89 90-94%
N Me O
LiOOH 76 : 16 N
O LiOOH 96 : 04 Damon, Tet. Lett. 1990, 31, 2849-2852
Me O PhH2C O
O OTIPS OTIPS

Et OTES Et OTES
OMe OMe Me Me
O O OBn O O OBn H H
O O
O Bn O
H LiSEt H
NH LiOOH NH O O
N THF, 25 °C X
N3 N3 H H H H
BocHN BocHN O 97% X = SEt
O O
Bn O Et3SiH
H H X=H
O O 89% yield
O N O OH
NH2 NH2
OTBS OTBS *5% Pd/CaCO3/PbO
O O 96%
(OF-4949 Synthesis) JACS 1989, 111, 1063 (Lepicidin Synthesis) J. Am. Chem. Soc 1993, 115, 4497-4513

RCOSR RCHO Fukuyama, J. Am. Chem. Soc 1990, 112, 7050-7051

O O O O O O
LiO2H Transamination to Weinreb Amides (see Handout 24A)
MeO N O MeO OH HN O O
HOH/THF OH O OH O OP O
Me Me Me(OMe)NHMe
Bn R N O OMe O-Protect
Bn R N R R
Me3Al R–metal
M. Bilodeau, unpublished results Me
complete hydroytic selectivity possible Me Me Me
Bn
for recent examples see, J. Am. Chem. Soc 1992, 114, 9434-9453
25-06-imide transformations 11/12/03 8:53 AM
D. A. Evans The Aldol Reaction: Syn Aldol Rxns of Chiral Ethyl Ketones Chem 206

General Reaction for Syn Aldols: M = B, Ti


O n-Bu2BOTf
O OH
M R 3N
O O OH Me
RCHO R
RL Me R-CHO
M = B, Ti RL TBSO
R TBSO Me
RM RM Me TBS = SiMe2tBu
R Diastereoselection
The Transition States: Ph 97:3
Me ‡
M R
O O O OH Masamune, JACS 1981, 103, 1566. Et 98:2
RCHO L C H
RL M O RL BnOCH2CH2
Me 96:4
H R
favored RM Me2CH
RM L H RM Me >99:<1
RL
Me ‡
M R TBSO O L2BOTf TBSO O OH
O O O OH i
RCHO L C H Me Pr2NEt
O RL
RL Me M R1 R1 R
H R RCHO
disfavored L RL Me Me Me
RM H RM Me
RM L Diastereoselection
Evans, JACS 1991, 113, 1047.
This system does not give a completely Bu 63:37 - 84:16
clean (Z) enolate
Examples: 9-BBN 83:17 - 85:15
(-)-Ipc 72:28
Bu2BOTf,
O i O OH
t Pr2NEt t
Paterson, McClure, Tet.Lett. 1987, 28, 1229. (+)-Ipc 91:9 - 94:6
BuMe2Si Me BuMe2Si
RCHO R
Me Me Me
TBSO O TiCl4 TBSO O OH
Enders ACIEE 1988, 27, 581. Diastereoselection = 96-98% EtNiPr2
Me Me Me Me
Me2CHCHO
TiCl4 Me Me Me Me Me Me
O O OH
EtNiPr2
Me Me TBSO O TiCl4
Me2CHCHO TBSO O OH
EtNiPr2
TBSO Me Me Me Me
TBSO Me Me Me2CHCHO
Evans, JACS 1991, 113, 1047. Diastereoselection: 99:1 (81%) Me Me Me Me Me Me

Evans, JACS 1991, 113, 1047. Diastereoselection: 95:5 (80-90%)


25-07-Ketone aldols-1 11/11/03 8:27 PM
D. A. Evans The Aldol Reaction: Anti Aldol Rxns of Chiral Ethyl Ketones Chem 206

M Examples: Diastereoselection
O O OH major : Σ others
General Reaction RL Me R-CHO TBSO O (Chx)2BCl TBSO O OH
RL
for R Me Et3N Me Me 94:6 (90%)
Syn Aldols: RM RM Me iPrCHO
Me Me Me Me Me Me Me
M syn-anti diastereomer
O O OH
General Reaction R R-CHO TBSO O (Chx)2BCl TBSO O OH
L RL Et3N
for R Me Me Me
96:4 (75%)
Anti Aldols: RM iPrCHO
Me RM Me
Me Me Me Me Me Me Me

The Transition States: However, the preceding precedent does not extend to these systems:
H ‡
M R
O O O OH
RCHO L C Me
RL O RL O O O O O OH O O OH
M (Chx)2BCl
H R
favored L H Et3N
RM Me RM Me O N Xq R Xq R
RM iPrCHO
RL Me Me Me Me
Me Me
H ‡ Bn
diastereoselection 84:16
M R
O O O OH
L C Me D. A. Evans, H. P. Ng, J. S. Clark, D. L. Rieger Tetrahedron, 1992, 48, 2127-2
RCHO O RL
RL M
H R
disfavored H An analogous case:
RM Me L RL RM Me
RM O O OH O OH
Evans, JACS 1991, 113, 1047. (Chx)2BCl
Et3N
BnO BnO R BnO R
RCHO
(E) Enolate Facial Bias O OH Me Me Me Me Me Me
RCHO RL diastereoselection 95:5
disfavored ? R
Me Me I. Patterson, J. M. Goodman, M. Isaka Tetrahedron Lett. 1989, 30, 7121-71
O M
O H anti-anti diastereomer
RL RL C C O O O
C H Me O
Me Me Me O OH
RCHO O N
RL BnO
favored ? R Me Me Me Me
Bn
Me Me
These enolates do not comply with steric analysis: → electronic effects?
syn-anti diastereomer
Tetrahedron, 1992, 48, 2127-2142.
25-08-Ketone aldols-2 11/11/03 8:28 PM
D. A. Evans The Aldol Reaction: Metal-Centered Chirality Chem 206

Masamune, Sato, Kim, Wollmann J. Am. Chem. Soc. 1986, 108, 8279-8281. Me
H R HO O
S disfavored
Me
B R SCEt3
HR O
O Me
TfO–B Me
(95 % ee) Me
Me H
R Me HO O
BR*2 O
O Me O favored
O B
DIPEA H R SCEt3
Me SCEt3
SCEt3 0 °C, 1 h S Me
Me Me
R
BR*2
O HO O
3 → 36 h
+ RCHO R SCEt3
SCEt3 -78 °C
Me Me
RCHO Yield, % anti/syn ee % (corrected)
n-PrCHO 91 33:1 93 (98)
i-PrCHO 85 30:1 95 (99)
t-BuCHO 95 30:1 96 (99.9)
c-C6H11CHO 82 32:1 93 (98)
PhCHO (71) 33:1 96 (99.8)
Chem 3D
BR*2
O HO O
3 → 10 h
+ RCHO R SCEt3
SCEt3 -78 °C
RCHO Yield, % ee % (corrected)
Analogous Carbonyl Allylation
n-PrCHO 82 87 (91)
i-PrCHO 81 87 (92) Masamune, Sato, Kim, Wollmann J. Org. Chem. 1987, 52, 4831
t-BuCHO 71 94 (98)
c-C6H11CHO 95 86 (90)
PhCHO 78 88 (92) Me HO HO
favored
See analogous study by Reetz Ph Me B R R
Me Me
Me
Reetz Tetrahedron Lett. 1986, 4721 Cl–B + RCHO
syn:anti, 96:4
enantioselection: 95-97%
Ph
25-09-Chiral aldol-metal-1 11/11/03 8:29 PM
D. A. Evans, D. H. Ripin The Aldol Reaction: Metal-Centered Chirality Chem 206

Masamune-Reetz Analogy: ■ Chiral Anti Aldol Reaction: JACS 1990,112, 4977; TL 1991,32, 2857.
R O HO O
enolization
H
R R OtBu RCHO R OtBu
B O HO O
O X
R RCHO Me O B favored
X Ratio
H (R) (X) syn:anti % ee Yield
SCEt3 R SCEt3
S
Me R R Me Ph- Me 2 : 98 94 93 %
chex- Me 6 : 94 75 82 %

■ Metal-Based Chiral Auxilliary:


Ratio
(R) (X) syn:anti % ee Yield
CF3 CF3
Ph Ph References: Ph- Br 2 : 98 96 86 %
(Corey) JACS. 1989,111, 5494 chex- Br 2 : 98 91 65 %
N N (Corey) JACS. 1990,112, 4977
F3C S B S CF3 (Corey) TL. 1991,32, 2857
(Corey) TL. 1993,34, 1737.
O O Br O O ■ Chiral Syn Aldol Reaction JACS 1989,111, 5494.
1 OH O
O enolization
Does this reagent perform in accord with the Masamune-Reetz analogy? Me Me Me
RCHO R
Note: The sulfonamide nitrogens are pseudo-tetrahedral
Me
Ratio
(R) syn:anti % ee Yield
■ Enolization:
Ph- 94 : 6 97 86 %
Either enolate geometry possible with proper choice of base, solvent, and substrate.
Me2CH- 98 : 2 95 91 %
O B * R2
1, Triethylamine O MeCH2- >98 : 2 >98 68 %
t
BuO
PhCH3 / Hexane tBuO
Me -78˚C ■ Chiral Acetate Aldol Reaction JACS 1989,111, 5494.
Me

O OH O
O B * R2 enolization
1, Hunig's Base O
Me Me SPh RCHO R SPh
PhS CH2Cl2 -78˚C Me
PhS
(R) % ee Yield
A mechanistic proposal for enolization control is presented in paper
Ph- 91 84 %
(Corey) JACS. 1989,111, 5494 Me2CH- 83 82 %

25-10-Chiral aldol-metal-2 11/11/03 8:30 PM


D. A. Evans The Aldol Reaction: Chelate vs Steric Control Chem 206

M Complimentary aldol reactions may be obtained by changing


O O OH metal as well as enolate geometry
RL Me R-CHO
Reference Rxn RL
R O O OH
TiCl4
RO RO Me i-Pr2NEt
Xq R
M RCHO Me Me
O O OH syn-syn
Me R-CHO
Chelate-Organized RL RL
Variant R O O O O O OH
Sn(OTf)2
RO RO Me Et3N
O N Xq R
RCHO Me Me
Me Me anti-syn
Nonchelate Reaction Bn

O TiCl4 O OH (Chx)2BCl O O OH
EtNiPr2 Et3N
Me Me
Me2CHCHO Xq R
t RCHO
BuMe2SiO t
BuMe2SiO Me Me Me Me anti-anti
Masamune, JACS 1981, 103, 1566 (boron enolate) JACS, 1990, 112, 866; Tetrahedron, 1992, 48, 2127-2142.
Diastereoselection: 99:1
Evans, JACS 1991, 113, 1047 (titanium enolate)

O O OH chiral reagent (+)(IPC)2-OTf O OH


LDA needed i-Pr2NEt
Me
PhCHO Ph BnO R
RCHO
Me3SiO Me3SiO Me Me Me
anti-syn
LDA Diastereoselection: 90:10
O Sn(OTf)2 O OH
Et3N
L BnO BnO R
Me ‡ RCHO
L Me Me Me Me syn-syn
Li O O
PhCHO O Ph
O Me Li O OH
R (Chx)2BCl
RL H
RL O H Et3N BnO R
Paterson & co-workers
Chelation possible for R = Bn, TMS R RCHO Me Me
Tetrahedron Lett. 1988, 29, 585-588 anti-anti
but marginal for TBS
Tetrahedron Lett. 1989, 30, 7121-7124
Thorton, Tet. Let. 1990, 31, 6001
Tetrahedron Lett. 1992, 33, 4233-4236
25-11-Ketone aldols-3/chelat 11/7/01 7:38 AM
D. A. Evans The Aldol Reaction–2 Chem 206

■ Assigned Reading
http://www.courses.fas.harvard.edu/~chem206/
Lithium Diisopropylamide-Mediated Lithiations of Imines: Insights into highly
Structure -Dependent Rates and Selectivities. D. Colum, JACS 2003, 125,
Chemistry 206 ASAP (handout)

W. R. Roush, J. Org. Chem. 1991, 56, 4151-4157. (handout)


Advanced Organic Chemistry
■ Other Useful References
Lecture Number 26
Evans, D. A., J. V. Nelson, et al. (1982). “Stereoselective Aldol Condensations.”
Top. Stereochem. 13: 1.
The Aldol Reaction–2
Heathcock, C. H. (1984). The Aldol Addition Reaction. Asymmetric Synthesis.
Stereodifferentiating Reactions, Part B. J. D. Morrison. New York, AP. 3: 111.
M
M O O
O O
Oppolzer, W. (1987). “Camphor Derivatives as Chiral Auxiliaries in Asymmetric
Me R R Synthesis.” Tetrahedron 43: 1969.
R H R
Me Heathcock, C. H. (1991). The Aldol Reaction: Acid and General Base Catalysis.
Comprehensive Organic Synthesis. B. M. Trost and I. Fleming. Oxford,
Pergamon Press. 2: 133.
■ (E) & (Z) Enolates: Felkin Selectivity
Heathcock, C. H. (1991). The Aldol Reaction: Group I and Group II Enolates.
■ Double Stereodifferentiating Aldol Reactions Comprehensive Organic Synthesis. B. M. Trost and I. Fleming. Oxford,
■ The Mukaiyama Aldol Reaction Variant Pergamon Press. 2: 181.
■ Allylmetal Nucleophiles as Enolate Synthons Kim, B. M., S. F. Williams, et al. (1991). The Aldol Reaction: Group III Enolates.
Comprehensive Organic Synthesis. B. M. Trost and I. Fleming. Oxford,
Pergamon Press. 2: 239.

■ Reading Assignment for this Week: Franklin, A. S. and I. Paterson (1994). “Recent Developments in Asymmetric
Aldol Methodology.” Contemporary Organic Synthesis 1: 317-338.
Carey & Sundberg: Part A; Chapter 7 Cowden, C. J. and I. Paterson (1997). “Asymmetric aldol reactions using boron
Carbanions & Other Nucleophilic Carbon Species enolates.” Org. React. (N.Y.) 51: 1-200.
Carey & Sundberg: Part B; Chapter 2 Nelson, S. G. (1998). “Catalyzed enantioselective aldol additions of latent
Reactions of Carbon Nucleophiles with Carbonyl Compounds enolate equivalents.” Tetrahedron: Asymmetry 9(3): 357-389.

Mahrwald, R. (1999). “Diastereoselection in Lewis-acid-mediated aldol


Monday, additions.” Chem. Rev. 99(5): 1095-1120.
D. A. Evans November 17, 2002

26-00-Cover Page 11/17/03 8:40 AM


D. A. Evans Carbonyl Addition Reactions: (E)-Enolate Nucleophiles Chem 206

(E) Enolates Exhibit Felkin Aldehyde Diastereoface Selection The Non-Reinforcing syn- RCHO is the most Interesting
Dependence of the Selectivity of Felkin-controlled Reactions on Nu Size
+
+
Me
H
RL O OH
H OTMS
Favored O RL
C Me O OP O OH OP O OH OP
O R
L nM R
H Me Me H i
Pr R i
Pr R i
Pr
MLn R
O O BF3•OEt2
Felkin Me Me Me
–78 °C
RL 20 21
R H + 19 P = PMB
H + Felkin/1,3-syn anti-Felkin/1,3-anti
O OH 6 P = TBS
Me Me H
Me RL RL
O R P = PMB P = TBS
Me C
Disfavored O Me Me
MLn R 20 : 21 20 : 21
H
R Anti-Felkin α-substituent dominates for Large Nu t-Bu 96 : 04 94 : 06
i-Pr 56 : 44 75 : 25
■ The illustrated syn-pentane interaction disfavors the anti-Felkin pathway. β-substituent dominates for small Nu Me 17 : 83 40 : 60
Evans, Nelson, Taber, Topics in Stereochemistry 1982, 13, 1-115.
W. R. Roush, J. Org. Chem. 1991, 56, 4151-4157.

Background Information: The influence of β-OR substituents on RCHO


OB(Chx)2 O OR O OH OR
Evans, JACS 1996, 118, 4322-4343
Me Me Me Me
Nu: O H
‡ OH
Ha Me α R Nu Me Me Me Me Me Me Me Me
α R
H Nu Felkin Selecton
H C O–M Lewis both centers major : Σ minors Felkin
Me acid Me reinforcing
R = TBS 99 : 1 (77% yield)
R R = PMB (84% yield)
93 : 7
Nu: OB(Chx)2
‡ O OR O OH OR
Ha Hb O OR OH OR Me Me Me Me
Nu
H C O–M H
Lewis 1,3-selection
H β R Nu β R Me Me Me Me Me Me Me Me
C acid
PO H major : Σ minors Felkin
R centers
Therefore, one might conclude that: non-reinforcing R = TBS 94 : 6 (79% yield)
R = PMB 74 : 26 (82% yield)
O OR O OR
stereocenters Achiral (E) enolates preferentially add to the Felkin diastereoface
α Me α Me stereocenters
H β reinforcing H β non-reinforcing High anti:syn diastereoselectivity (≥ 97 : 3) is observed in all cases
Me Me Me Me
Evans etal. JACS 1995, 117, 9073
26-01-E enol/RCHO 11/16/03 7:05 PM
D. A. Evans Carbonyl Addition Reactions: (Z)-Enolate Nucleophiles Chem 206
(Z) Enolates Exhibit Anti-Felkin Aldehyde Diastereoface Selection
+
+
Me
Me O OTBS OM O OH OTBS
O OH O OH OTBS
RL H Me
Disfavored O RL R
C H H Me R Me R Me
O R
L nM Me Me Me Me Me
H Me Me
MLn O R Felkin anti-Felkin (Cram-Chelate)
O Felkin
RL
Me H + Felkin : anti-Felkin
R H + OLi
Me Me O OH
RL TMSO Me 27 : 73
Me RL D. W. Brooks & Co-workers
O R Tetrahedron Lett. 1982, 23, 4991-4994.
H C Me Me
Favored O Me Me
MLn 9BBN
H O
R Anti-Felkin 29 : 71
Me
PhS
The illustrated syn-pentane interaction disfavors the Felkin pathway.
■ The bulky OTBS group disfavors chelation. (see Keck, JACS 1986, 108, 3847.)
Evans, Nelson, Taber, Topics in Stereochemistry 1982, 13, 1-115. ■ The boron and lithium enolates display nearly equal levels of anti-Felkin selectivity.
W. R. Roush, J. Org. Chem. 1991, 56, 4151-4157.

An Early study rationalized results through chelated transition states:


Titanium enolates exhibit the same trend

Li O OH OCH2OBn O OH OCH2OBn
O O OH OPMB
TiCln O OPMB
Me R R" R R" O
R iPr iPr
Me Me Me Me Me H iPr
O OCH2OBn iPr Me Me
Felkin Anti-Felkin (Cram Chelate) Me
anti-Felkin : Felkin 77 : 23 (78%)
H R"
Me O OH OPMB
Si-face TiCln O OPMB
O
(R) (R") Felkin : Anti-Felkin iPr iPr
Me H iPr
Et 17 : 83 iPr Me Me
O OCH2OBn Li CH2OBn Me
O O anti-Felkin : Felkin 56 : 44 (84%)
C6H11 H Et 10 : 90 Nu:
Me H R"
Et O OCH2OBn 13 : 87 Evans etal. JACS 1995, 117, 9073
Me
H CHMe2 Masamune
C6H11 8 : 92 JACS 1982, 104, 5526
Me

26-02-Z enol/RCHO 11/16/03 7:07 PM


D. A. Evans Double Stereodifferentiating Bond Constructions-1 Chem 206

Double Stereodifferentiating Aldol Bond Constructions Matched reactant pair: Stereo-induction from both partners reinforcing
O O O OH
The reference reactions:
(–) ❊
H ❊ O OH
OM O
Me Me Me Me Me Me ❊ Me
Me Me
H
Stereochemical Control Elements Me Me Me Me Me
[aldehyde prod ratio] = 10/1
Enolate geometry
O O OH
OM
Me Me
Me H
Product Me Me Me Me
Me
Enolate facial bias Stereochemistry Aldehyde facial bias
[enolate prod ratio] = 10/1

■ The double stereodifferentiating situation: Stereoselectivity?


The Issue: Can one reliably take the diastereoselectivites of the individual
reaction partners and use this information in the illustrated OM O O OH
extrapolation: Me ❊ ❊
H ‡
The model reactions: ∆∆G (rxn)
Me Me Me Me Me
O OH
Nu (–) ■ The assumption: (Masamune, Heathcock)
Nu ∆∆G‡ (aldehyde)
H
It is presumed that useful information can be obtained from related achiral enolate &
Me Me RCHO addition reactions and that the free energy contributions will be additive:
O
OM El (+)

∆∆G‡ (Rxn) ~ ∆∆G‡ (enolate) + ∆∆G‡ (RCHO)
Me El ∆∆G (enolate)
log [Product ratio] ~ log [enolate ratio] + log [aldehyde ratio]
Me Me Me
[Product ratio] ~ [enolate prod ratio] x [aldehyde prod ratio]
The extrapolation:
OM O O OH ■ Hence, for the case at hand: [Product ratio] ~ [10] x [10] ~ 100
Me ❊
H ❊
‡ Mismatched reactant pair: Stereo-induction from partners nonreinforcing
Me Me ∆∆G (rxn) = ? Me Me Me
OM O O OH
Me ❊
H ❊
Masamune, Angew. Chem. Int. Ed. 1985, 24, 1-76 ∆∆G‡ (rxn) ❊
Me Me Me Me Me

∆∆G‡ (Rxn) ~ ∆∆G‡ (enolate) – ∆∆G‡ (RCHO)


26-03-stereodif aldol-1 11/16/03 7:08 PM
D. A. Evans Double Stereodifferentiating Bond Constructions-2 Chem 206

The Masamune-Heathcock generalizations hold to a point:


(Z)-Titanium Enolates: The reference reactions

TBSO O TBSO O OH
(E)-Boron Enolates: The reference reactions
TiCl4, EtN-iPr2 Me Me
Me
B(c-hex)2 R-CHO
O O OTBS O OH OTBS Me Me Me Me
Me Me Me
Me Me Me Me diastereoselection 96 : 4
H
Me Me Me Me Me Me Me Me O OPMB
Me O OH OPMB O OH OPMB
diastereoselection 94 : 6 H
Me Me
Me Me X X
TBSO O TBSO O OH
(c-hex)2BCl, Et3N OM Me Me Me Me Me Me
Me Me Me
R-CHO Me Me M = B(9-BBN) syn: 10: 69 + 21% anti
Me Me Me Me Me Me Me
Me
M = TiCl4 syn: 21: 71 + 8% anti
diastereoselection 96 : 4

(E)-Boron Enolates: The matched cases (Z)-Titanium Enolates: The matched cases
BR2 TiCln
TBSO O O OR TBSO O OH OR TBSO O
Me Me Me Me Me Me TBSO O OH OTBS TBSO O OH OTBS
H
Me Me Me Me Me Me Me Me Me Me Me Me R R R R
O OTBS Me Me Me Me Me Me
diastereoselection: anti : Σ others
Me R = TBS: 87 : 13 (76%)
R = TBS: >99 : 1 (85% yield) H
Me Me
R = PMB: >99 : 1 (84% yield)
(E)-Boron Enolates: The mismatched cases (Z)-Titanium Enolates: The mismatched cases

BR2 TiCln
TBSO O TBSO O
Me Me Me
TBSO O OH OR TBSO O OH OR TBSO O OH OTBS TBSO O OH OTBS

Me Me Me Me Me
R R R R R R R R
Me Me Me Me Me Me O OTBS Me Me Me Me Me Me
O OR
Me
Me R = TBS: 52 : 48 (83% yield) H diastereoselection 62 : 38 (87%)
H
R = PMB: 81 : 19 (79% yield) Me Me
Me Me
"Double Stereodifferentiating Aldol Reactions. The Documentation of "Partially
β-center on RCHO can play a significant role in this marginal situation Matched" Aldol Bond Constructions". Evans, D. A.; Dart, M. J.; Duffy, J. L.;
Rieger, D. L. JACS 1995, 117, 9073-9074.
26-04-stereodif aldol-2 11/16/03 7:09 PM
D. A. Evans Introduction to Complex Aldol Bond Constructions Chem 206

Synthesis of Polyketide chains The Lonomycin Synthesis: An example of polypropionate assembage


Evans, Ratz, Huff, Sheppard JACS 1995, 117, 3448
Given a polyprpionate chain of RO OH OH OH OH
alternating Me & OH substitutents, R
R OMe OMe
select a disconnection point Me
sectioning the fragments into Me Me Me OMe
subunits of comparable complexity Me Me Me Me Me Me O Me Me
O
by adding C=O as illustrated. OMe
HO O O O O
OH H H Me H
OR OR OR OR O OH OR Me Me O
OR OH O Me
R R R α α' R Me OH
β β'
Me Me Me Me Me Me Me Me Me Me Me Me O O OMe OH OH O O O OMe OH OH
T1A T1B
aldol
HO 1 3 5 7 9 11 H HO 1 3 5 7 9 11
Focusing on the =O FG, there are 2 OR OR O O OR Me Me Me Me Me Me Me Me Me Me
1st-order aldol disconnections α' R
R α BH3 Transform: See Lecture No. 8
highlighted. Let's proceed forward with β H β'
T1B. Carry out the dissconnection to Me Me Me Me Me Me
subunits 2K and 2A.
2A
C1–C11 Assemblage
2K
α center important Both centers important O O OH
O O O 1. NaBH(OAc)3
β center ignore
Me
Sn(OTf)2 2. (MeO)2CMe2, H+
For substituted enolate and enolsilane-based processes, there are at least three O N 5 7
XP 5 7 9
identifiable stereochemical determinants that influence reaction diastereoselectivity Et3N
Me Me Me Me
(eq 1). Two of these determinants are associated with the local chirality of the O 93%
individual reaction partners. For example, enolate (enolsilane) chirality influences the Bn (85%) Diastereoselection
A H 95 : 5
absolute stereochemistry of the forming methyl-bearing stereocenter, and in a similar 9
fashion, aldehyde chirality controls the absolute stereochemical outcome of the Me
incipient hydroxyl-bearing stereocenter. The third determinant, the pericyclic transition Me Me
state, imposes a relative stereochemical relationship between the developing Me Me O O O
JACS, 1990, 112, 866
stereocenters. This important control element is present in the aldol reactions of metal O O O LiBH4, EtOH
enolates (M = BR2, TiX3, Li, etc.), but is absent in the Lewis acid catalyzed XP 5 9
(Mukaiyama) enolsilanes aldol variants that proceed via open transition states. H 5 9 Swern Me Me Me
Stereochemically Me Me Me 86%
M Matched aldol addition
O O O OH
R1 α Me α' R2 R1 α α' R2 (1) O O O Me Me
H
Me Sn(OTf)2 O O OH O O
Me Me Me Me Me O N 1 3
Me Et3N XP 1 5 9
Stereochemical Determinants M = BR2 M = SiR3
Bn A Me Me Me Me Me
enolate facial bias ✔ ✔
Anti-Felkin Adduct Diastereoselection >95 : 5 (86%)
aldehyde facial bias ✔ ✔
pericyclic transition state ✔ ✘ The Sn(OTf)2 aldol reaction of A: seethis lecture + JACS, 1990, 112, 866

26-05-PK synth-1 11/16/03 7:10 PM


D. A. Evans Introduction to Complex Aldol Bond Constructions Chem 206

The Altohyrtin Synthesis: An example of polypropionate assembage Model Studies OB(c-Hex)2


Me Me Et
Me Me Me
O Me
Me H Me Et Me
MeO 14
OAc O O O O O
O O O TBSO -78 °C O OH TBSO
HO
H H AcO
H H
O Diastereoselection 97:3
OH H
H Background
H OH H
H Me O
Me X OB(Chx)2 O OH OR major : Σ others
O O OH
O
H H H R = TBS 99 : 1
iPr iPr iPr
HO H R = PMB 93 : 7
H OH OH Me O OR Me Me

Evans, Trotter, Coleman, Côté, Dias, Rajapakse, Tetrahedron 1999, 55, 8671-8726. BR2 H iPr
O Me O OH OR
MeO Me Me Me R = PMB 69:31
H iPr iPr iPr
O O O
H TESO Me
TrO H O Aldol O TESO
OTBS M ? O H
Reaction H The Aldol Fragment Coupling
H
Me Me MeO Me Me Me
O OTBS O O H O
MeO Me
H TESO
O O OH O TrO H O O TESO
OTBS B(cHex)2 O H
H TESO H
TrO H TESO H
OTBS O H
H
H
Me Me OTBS
The stereochemical determinants from
O
each fragment were evaluated OTBS MeO Me
Me O O OH O
Model Studies
O H TESO
Me Me TrO H TESO
MeO MeO Me Me O H
OTBS
H H
O O O O
pentane, -78 °C Me H
H H
TrO H O TrO H O OH
OTBS B(c-Hex) 2 OTBS Diastereoselection 90:10 (70%)
OTBS
H H
2:1 mixture of diastereomers
26-06-PK synth-2 11/16/03 7:11 PM
D. A. Evans Introduction to Complex Aldol Bond Constructions Chem 206

Bafilomycin A1 Synthesis: An example of polypropionate assembage


Enolization Conditions: PhBCl2, i-Pr2NEt, CH2Cl2, -78oC.
Evans, Calter, Tetrahedron Lett. 1993, 34, 6871
tBu tBu
Me Me OMe TBSO O Si
Me Me O O O
Me
HO O H Me
Bafilomycin A1 Me
Me O OH O Me Me
Me Me Me
2
1 OR
OH tBu tBu
Me OMe Me Me Si
TBSO OH O O O
Critical Aldol Disconnection
diastereoselection Me Me
Me Me OMe >99:1
O OR OR Me Me Me Me Me
RO O
O O Me Critical Aldol Disconnection
Me
Me Me Me Me Me OMe
H tBu tBu
Si
Me OMe Me TMSO O O O O
Me O O Me
Required: Syn aldol addition
Aldehyde Fragment: Target contains syn aldol retron wilth anti-Felkin relationship at 1 & 2 H Me Me Me
Enolate Fragment: Can the needed enolate facial bias be built into the reaction?? Me OMe Me

60% diastereoselection
o >95:5
Aldol Model Studies Enolization Conditions: PhBCl2, i-Pr2NEt, CH2Cl2, -78 C.
Me Me OMe

OH O OTBSOTES O OTBSOTES O tBu tBu


Me2CHCHO TMSO Si
Me Me Me OH O O O
Me O
Me Me Me Me
Me Me Me Me diastereoselection
62:38 9 Me OMe Me Me Me Me

The Critical Observation


Me Me OMe
Me Me
tBu tBu 94%
tBu tBu HO O
Si Me
Si O O O OH O
OH O O O Me O HF.pyridine, THF, 25oC.
Me2CHCHO Me
Me Me OH
OH
Me Me Me
Me Me Me Me Me OMe Me Me
diastereoselection
>99:1 Bafilomycin A1
26-07-PK synth-3 11/16/03 7:12 PM
D. A. Evans The Mukayama Aldol Reaction-1 Chem 206

Type I Aldol Reaction: Metal Aldol Process Recent Reviews


This reaction may be run with either a stoichiometric or catalytic amount
of base. R. Mahrwald, Diatereoselection in Lewis Acid Mediated Aldol Additions,
M M Chem. Rev. 1999, 99, 1095-1120
O O O O

X R2
S. G. Nelson, Catalyzed enantioselective aldol additions of latent
X H R2 enolate equivalents Tetrahedron: Asymmetry 1998, 9, 357-389.
R1 R1

slow Mukaiyama Aldol Reaction, E. Carreira In Comprehensive


B–M H–B–M Asymmetric Catalysis, Jacobsen, E. N.; Pfaltz, A.; and Yamamoto, H.
Editors; Springer Verlag: Heidelberg, 1999; Vol III, 998-1059.
O O OH

X X R2
R1 R1 Reaction Mechanism: "Closed" versus "Open" Transition States

Catalytic Version: Slow step in the catalytic variant is protonation of The Mukaiyama aldol reaction proceeds through an "open" transition
the intermediate metal aldolate
state. The two illustrated competing TS orientations do not differ
significantly in energy. For most reactions in this family there is not a
Type II Aldol Reaction: Mukaiyama Aldol Process good understanding of reactans-pair orientation. There is a prevalent
This reaction may be run with either a stoichiometric or catalytic amount view that the anti-periplanar TS is favored on the basis of electrostatic
of Lewis acid. effects.

The minimalist mechanism: MX = Lewis acid M R O M


X L O
TMS TMS M M H R R C R
O M O O O O M
O X O C TMSO C
H L
C H R2 H R2
X R2 O C R
X H R2 X R2 Me R2 R OTMS
R1 R1 R1

TMS–X Metal aldolate TS anti-periplanar TS synclinal TS


M M slow "Open" "Open"
"Closed"
TMS
O O O

H R2 X R2
R1
Other events are also taking place: Silyl transfer is not
Carreira Tet. Lett 1994, 35, 4323 necessarily intramolecular
26-08-Mukaiyama-1 11/16/03 7:12 PM
D. A. Evans The Mukayama Aldol Reaction-2 Chem 206

Denmark has designed a nice substrate to distinguish between Syn-Anti Aldol Diastereoselection
synclinal and anntiperiplanat transition states:
Denmark, J. Org. Chem. 1994, 59, 707-709 Heathcock: J. Org. Chem 1986, 51, 3027

OTMS O OTMS O OTMS


BF3•OEt2
Me Me
‡ PhCHO R Ph R Ph
synclinal Me Me Me
TMS Me
56:44
O
O H HO OTMS O OTMS O OTMS
H O BF3•OEt2
XM Me
Me S‡ S PhCHO R Ph R Ph
Me Me Me Me
CHO 56:44

OTMS
Me OTMS O OTMS
TMS BF3•OEt2
O Me
MX
O Me3C PhCHO R Ph M
H O
H HO O
antiperiplanar Me
H Me
AP‡ AP The effectice size of the enol >95:5 C
substituents are probably dominant. H Ph
C
Lewis Acid syn:anti The transition state? Me3C OTMS

TiCl4 21:79 These reactions "exhibit little simple diastereoselection except in special
cases."....Heathcock
SnCl4 18:82
BF3•OEt2 29:71

TrClO4 27:73
SnCl2 78:22

conclusion: there is a modestpreference for the antiperiplanar TS

26-09-Mukaiyama-2 11/16/03 7:14 PM


D. A. Evans The Mukayama Aldol Reaction-3 Chem 206

Merged Syn-Anti & Felkin Diastereoselection Enolslane Face Selection


Evans: JACS 1995, 117, 9598 TBSO OTMS TBSO O OH TBSO O OH
Me
O OTBS O OH OTBS O OH OTBS R R R R R
iPrCHO
BF3•OEt2 Me Me Me Me Me
H R R R R R
59 : 41 (82%)
Me Me Me Me Me R = iPr Enolsilane Face Selectivity 95 : 5
95 : 5 (95%)
OTMS
R = iPr Felkin : anti-Felkin > 99 : 1
Me TBSO OTMS TBSO O OH TBSO O OH
iPrCHO
Me Me R R R R R
O OH OTBS O OH OTBS Me Me Me Me Me Me
O OTBS
95 : 5 (80%)
BF3•OEt2
H R R R R R favored Enolsilane Face Selectivity 90 : 10

Me Me Me Me Me
70 : 30 (89%) TMS A(1-3) control is good for the
Conclusions: Me
O H (E) enolsilane
Moderate to Good syn diastereoselectlion Felkin : anti-Felkin 99 : 1 C C
Excellent Felkin diastereoselectlion C H Me
RL

O OTBS O OH OTBS O OH OTBS Double Stereodifferentiating Syn Aldol Rxns with Enolsilanes
BF3•OEt2
H R R R R R
Me Me Me Me Me O OTBS TBSO O OH OTBS
87 : 13 (68%)
OTMS
Felkin : anti-Felkin > 99 : 1 H R R R
Me Me R = iPr
Me TBSO OTMS Me Me Me
Me R = iPr 98 : 2 (72%)
O OTBS O OH OTBS O OH OTBS R
BF3•OEt2 O OTBS Me Me TBSO O OH OTBS
H R R R R R
Me Me Me Me Me H R R R
91 : 9 (75%) Me Me Me Me
Felkin : anti-Felkin 87 : 13 98 : 2 (83%)
Conclusions:
Moderate to Good syn diastereoselectlion
Excellent Felkin diastereoselectlion

26-10-Mukaiyama-3 11/16/03 7:15 PM


D. A. Evans, D. M. Barnes Allyl and Crotylmetal Species–1 : Boron Chem 206

■ The Tartrate-derived Allylboronic Esters


■ General Reviews of Allyl Metal Reagents:
Comprehensive Organic Synthesis, 1991;Vol. 2. iPrO2C
O OH
Yield = 72%
iPrO2C B Chex ee = 87%
O
O
■ The General Reactions
iPrO2C H OH
R OH O
RCHO
B B Me Chex Yield = 100%
iPrO2C ee = 91%
R R O Me

R OH iPrO2C
O OH
RCHO
B
R R B Yield = 90%
iPrO2C Chex ee = 83%
O
Me Me
Me Me
OH W. Roush, J. Am. Chem. Soc. 1985, 107, 8186-8190.
R RCHO Tetrahedron Lett. 1988, 29, 5579-5582.
B Me R
R O
Me O
BnN H
O OH
■ The Hoffman Chiral Allylboronic Esters
B Yield = 40%
O Chex ee = 97%
Me Me BnN
OH
1) CH3CHO O
O W. Roush, J. Am. Chem. Soc. 1988, 110, 3979-3982.
B 2) N(CH2CH2OH)3
Me
O
Me Ph R
R R = Me: Yield = 93% ■ A Reagent for the Generation of Anti-1,2-Diols
ee = 60-70% O
R. Hoffman Tetrahedron Lett. 1979, 4653-4656. iPrO2C OH
ACIEE, 1978, 17, 768-769. R = H: Yield = 92% O H
ee = 65%
B SiMe2(OChex) Chex
iPrO2C
O
Me Me (OChex)Me2Si
Ms OH
N EtCHO OH
B Et H2O2, KF, KHCO3 93% Yield
O Chex 72% ee
Me Yield = 92%
ee = 92% OH

W. Roush, Tetrahedron Lett. 1990, 31, 7563-7566.


M. Reetz Chem. Ind. (London) 1988, 663-664.

26-11 - Allyl/Crotyl 1 11/16/03 7:15 PM


D. A. Evans, D. M. Barnes Allyl and Crotylmetal Species–2 : Boron Chem 206

■ Allenylboronic Esters: Tartrate-derived Controllers and Internal Delivery ■ The Masamune Borolane
O Me
OH
iPrO2C
O
H OH Me 96:4
CH2 Yield = 56% B Me 97% ee
B C ee = 92%
iPrO2C Chex O Me Me
O Me
Me 76%
CH2 H
C Me
Ph Me OH
OH O O 96:4
H2C C CHB(OH)2 B Me 93% ee
Me O B OH B
Ph Me HO O O
Me Me
Ph Me Me
H 2C C Me
70%
S. Masamune, J. Org. Chem. 1987, 52, 4831-4832.

H. Yamamoto, J. Am. Chem. Soc. 1982, 104, 7667-7669


■ The Brown IPC Controller
95% Yield
Tetrahedron Lett. 1986, 27, 1175-1178. >99:1
HO Me OH OH
CH3CHO
Me Me
> 99% ee
B Me
H. C. Brown, J. Am. Chem. Soc. 1983, 105, 2092-2093.
J. Org. Chem. 1991, 56, 401-404.
■ The Corey Stein Controller J. Org. Chem. 1992, 57, 6614.

Ph Ts
N OH
OH R1 = Me, R2 = H: ee = 90%
B 98% ee Me Me CH3CHO
Ph
N Me R1 = H, R2 = Me: ee = 90%
R B R1 R1 = H, R2 = OMe: ee = 90%
O R2 R1
Ts
H R2 H. C. Brown, J. Am. Chem. Soc. 1988, 110, 1535-1538.
See also: Tetrahedron Lett. 1990, 31, 455-458.
Ph Ts
N OH
Ph B 98% ee
N R OH
1) THF, RT H
Ts Me Me 2) CH3CHO
+ Me ee = 94%
E. J. Corey, J. Am. Chem. Soc. 1989, 111, 5495-5496. B H N
J. Am. Chem. Soc. 1990, 112, 878-879. H 3) 2 OH

H. C. Brown, J. Chem. Soc., Perkin Trans. 1, 1991, 2633.

26-12 - Allyl/Crotyl 2 11/16/03 7:15 PM


D. A. Evans, D. M. Barnes Allyl and Crotylmetal Species–3 Chem 206

The Allylboron Reagents Add to Carbonyl Compounds via a Another Enantioselective Allyltitanium Reagent
Zimmerman-Traxler Transition State
Ph Ph
Masamune, Sato, Kim, Wollmann J. Org. Chem. 1987, 52, 4831 O Ti OH
O PhCHO
O 95% ee
Ph
Me HO HO O
favored Ph Ph
Me B R R
R. Duthaler, J. Am. Chem. Soc. 1992, 114, 2321-2336.
Me Me
Me
+ RCHO
anti:syn, 96:4
Me
H Chiral α-Substituted Allyl Metal Reagents: Boron
HO
H
B favored Me
Me R O
R OH
Me O
Me PhCHO
Me Me R = H: 92% ee
H B R Ph R = Me: 98% ee
enantioselection: 95-97% O
H Me Cl R
R Me Cl
O disfavored HO
Me
B R
H Chex OH
H Me O PhCHO
Me Me 68% Yield
Chex B Ph 99% ee
O
Me
An Enantioselective Allyltitanium Reagent Me Me

OH
RCHO
ee = 85 - 94% R. Hoffman, Chem. Ber. 1986, 119, 2013-2024.
R*O Ti R = alkyl, aryl Me Chem. Ber. 1988, 121, 1501-1507.
R*O ACIEE, 1986, 25, 1028-1030.
Me O
Me Me Chex
H O Me
O H O H O
R*OH = H O Me
Me B B Chex
Ph O Me Ph O
HO O Me R H O Me O

Cl Me H
M. Riediker, R. Duthaler, ACIEE, 1989, 28, 494-495.
In Organic Synthesis via Organometallics, 1991, 285-309. The favored transition states
J. Am. Chem. Soc. 1992, 114, 2321-2336.
Duthaler Chem. Rev. 1992, 92, 807

26-13- Allyl/Crotyl 3 11/16/03 7:16 PM


D. A. Evans, D. M. Barnes Allyl and Crotylmetal Species–4: Catalytic Systems Chem 206

■ Three Catalytic Asymmetric Allylations of Aldehydes are Known

iPrO O COOH OH Ti(OiPr)4, 4Â sieves


O OiPr
COOH + BH3-THF BLn* Ti
OH O OiPr
OH
OiPr
1
Me OH
Me 20 mol % BLn*
+ PhCHO Ph
Me SiMe3 OH Ti(OiPr)4, 4Â sieves O
O
Me Ti
E/Z = 61/39 OH CF3COOH or TfOH O O
63% yield
90% ee
H. Yamamoto, Synlett 1991, 561-562.
2

OH
1 or 2 (10 mol %), RCHO
SnBu3
R

R Catalyst Yield (%) ee (%)


O Cl Ph
Ti 1 88 95
OH 98 92
O Cl 2
SnBu3 Chex 1 66 94
20 mol % n-C7H15 2 95 92
+ O
81% yield 1 42 89
n-C7H15CHO 97.4% ee H 2 78 77

G. Keck J. Am. Chem. Soc. 1993, 115, 8467-8468.

E. Tagliavini, A. Umani-Ronchi J. Am. Chem. Soc. 1993, 115, 7001-7002.


■ Many Other Metals Have Been Employed in the Allylation Reaction ...
Pb: S. Torii, Chem. Lett. 1986, 1461-1462.
Mo: J. Faller, Tetrahedron Lett. 1991, 32, 1271-1274.
Cr: Y. Kishi, Tetrahedron Lett. 1982, 23, 2343-2346.
P. Knochel, J. Org. Chem. 1992, 57, 6384-6386.
Sb: Y. Butsugan, Tetrahedron Lett. 1987, 28, 3707-3708.
Mn: T. Hiyama, Organometallics, 1982, 1, 1249-1251.
Zn: T. Shono, Chem. Lett. 1990, 449-452.
Ba: H. Yamamoto, J. Am. Chem. Soc. 1991, 113, 8955-8956.

26-14 - Allyl/Crotyl 4 11/9/00 4:33 PM


Chemistry 206

Advanced Organic Chemistry

Handout 26A

The Asymmetric Baylis-Hillman Reaction


An Evans Group Afternoon Seminar
Jake Janey
March 29th, 2001

XH
EWG X
cat. base * EWG
+ R
R R' R'

EWG

J. Janey Monday,
November 17, 2003

26A-00-Cover page 11/17/03 8:28 AM


J. Janey The Asymmetric Baylis-Hillman Reaction Chem 206

An Evans Group Afternoon Seminar


Jake Janey An anti propionate aldol equivalent...
March 29th, 2001 OH OH
Rh(I), H2
EWG EWG
R R
XH
EWG X cat. base
+ * EWG
R
R R' R'

EWG

Early Synthetic Examples

10 years after the Baylis-Hillman German patent...used in a C10 integerrinecic acid synthesis:
p.
p.
Leading References: OH Me Me OH
CO2Et 5% DABCO
Langer, P. Angew. Chem. Int. Ed. Engl. 2000, 39, 3049-3052. + MeCHO CO2Et CO2H
Me
Ciganek, E. Org. React. 1997, 51, 201-350. 25 °C, 7d
HO2C Me
Basavaiah, D.; et. al. Tetrahedron, 1996, 52, 8001-8062. 94% yield
Drewes, S. E.; Roos, G. H. P. Tetrahedron, 1988, 44, 4653-4670.
Drewes, S. E. J. Chem. Soc., Perkin Trans. 1 1982, 2079-2083.

Baylis-Hillman Reaction Scope Shortly thereafter, a more extensive, published study:

X EWG R2 XH OH
cat. base
+ EWG CO2Me 15% DABCO
R1 + RCHO CO2Me
R1 R2 R
25 °C, 0.5 to 7d

X = O, NTs, NCO2R, NPPh2, NSO2Ph EWG = CO2R, CN, POEt2, R = alkyl or aromatic 94% yield
R1 = alkyl, aryl CHO, COR, SO2Ph, SO3Ph
• All reactions run neat in a sealed tube with 1.5-2 equivalents of acrylate.
R2 = H, alkyl, EWG

Hoffmann, H. M. R.; Rabe, J. Angew. Chem. Int. Ed. Engl. 1983, 22, 795-797.
OH N
cat. bases: n-Bu3P:
N N N

3-hydroxyquinuclidine (3-QDL) DABCO quinuclidine

26B-01 11/9/01 1:06 PM


J. Janey The Asymmetric Baylis-Hillman Reaction Chem 206

Hill, J. S.; Isaacs, N. S. J. Phys. Org. Chem. 1990, 3, 285-288.


Kaye, P. T.; Bode, M. L. Tetrahedron Lett. 1991, 32,
Evidence for an Intermediate
5611-5614.
Drewes, S. E.; et. al. Syn. Comm. 1993, 23, 2807-2815.
EWG
Coumarin Salt
OH R 3N + O
+
EWG 1 eq DABCO N Cl –
R' H N
CH2Cl2, r.t., 2.5 h
O O
O O
X-ray
O– 81% yield
O
EWG EWG O
R' –
Drewes:
H "...the counter ion was chloride
+ Cl
R 3N R 3N (presumably originating from the
+ + OH dichloromethane...)."
Or... O
p. O
p. +
E2 elimination...
:B H 1 eq DABCO
N Cl –
R'CHO + OMe N
R' O– CH2Cl2, 0 °C
Hα OH O O
H H H H
O– 40% yield
Hα EWG EWG
NR3 NR3 Effects of Acrylate Ester Substituent
+ +
initially formed eliminates OH O
CO2R 13% DABCO
E1cB is also possible + PhCHO
neat, r.t Ph OR
• rate = Kobs[aldehyde][alkene][amine] 1.3 eq 1.0 eq
• pseudo-second order if [amine]≈constant
• addition to aldehyde is r.d.s. because the dipole is R time (days) yield (%)
increased by further charge seperation Me 6 89
• acrylonitrile and methyl acrylate studied Et 7 79
• For aryl substituted benzyl ethers, no clear
• enolate geometry not considered Bn 2 88
relation between σ values and reactivity was
• ethereal solvent inhibits reaction whereas alcohols n-C10H21 14 75 observed.
(especially diols) accelerate reaction t-Bu 65 65
• huge volume of activation: ∆V‡ of -79 cm3 mol-1 (the 2-adamantyl 62 40 • Trends hold for furfural.
Diels-Alder is -35 cm3 mol-1) found by plotting lnkobs vs. CH2CH2F 3 81
P. 5000 bar increases rate by 1.1 x 106 CH2CH2Br 2 NR • The products undergo retro Baylis-Hillman, i.e.
the reaction is reversible.
• Reaction is reversible (i.e. a Grob type CH2CF3 15 h 58
fragmentation), thus mechanism could be ternary, with CH2CH2OMe 4 89
no discrete enolate intermediate (supported by ∆V‡ and CH2CH2NMe2 8 82
temperature effects). (CH2)6Cl 15 NR
Caubere, P.; et. al. Tetrahedron 1992, 48, 6371-6384.
26B-02 11/9/01 1:07 PM
J. Janey The Asymmetric Baylis-Hillman Reaction Chem 206

Bases for Catalysis • Reaction is accelerated for a wide variety of aldehydes when conducted at 0 °C

• Temperature effect not seen with acrylonitrile (cannot form enolate)


pKa H2O (DMSO)
• Author concludes that one enolate must react faster than another (i.e. a kinetic versus a
thermodynamic enolate).
OH N
> N > N >> – CO2Me +
N + NR3 O R 3N O–
DABCO R 3N +
3-Hydroxyquinuclidine (3-QDL) Quinuclidine OMe
9.5 (~8.5) 2.97, 8.82 (2.97, 8.93) 10.9 (9.80) OMe
E Z
NMe2 NMe2
N Which enolate is more stable and which is more reactive?
OAc
>> or
O
N N N
Leahy, J. W.; Rafel, S. J. Org. Chem. 1997, 62, 1521-1522.
DBU
3-Quinuclidone Proton sponge (~12)
3-Acetoxyquinuclidine 6.9 12.0 (7.50)
p.
p. Sterics also important: Enolate Geometry
Me2NH > Me2NEt > MeNEt2 > NEt3 10.75 (9.00)
– O +
Many, many phosphines screened...the winner: n-Bu3P ~9 + NR3 O O OMe –
R 3N O
+ OMe
• n-Bu3P is only a slightly better catalyst than DABCO. OMe OMe
E + R 3N Z
R 3N
...or could accelerate Kinetic
O H protonation of Thermodynamic
P intermediate, as any • less charge seperation • more charge separation
N O– alcohol additive
P • less reactive • less stable
will accelerate
unreactive OR reaction • enolate twists out of plane by PM3

σ*
Temperature Effects
σ*
MeO
OH –O
CO2Me 0.1 mol% DABCO 4π e-
+ MeCHO CO2Me HOMO –O + NR3
2M in dioxane Me
MeO + NR3
74% yield
25 °C 1 week
0 °C 8 hours! better conjugation into σ*

26B-03 11/9/01 1:07 PM


J. Janey The Asymmetric Baylis-Hillman Reaction Chem 206

Salt Additive Lewis Acid Catalysis

OH
OH CO2t-Bu 1 eq DABCO
CO2Me 5% DABCO + PhCHO CO2t-Bu
CO2Me MeCN, r.t., 1 d Ph
+ PhCHO
Et2O, 0 °C, 20 h Ph
5 mol% ligand, 5 mol% metal
1.2 eq 1.0 eq

LiClO4 (mol%) yield (%) Relative Reaction Rates


0 trace ligand Sc(OTf)3 Yb(OTf)3 Eu(OTf)3 La(OTf)3
5 12 none 3.3 3.6 3.5 4.7
10 40 – + (+)BINOL 9.4 14.4 12.8 14.6
ClO4 R3N O Li
50 63 (+)diethyl tartrate 5.2 9.7 5.5 7.3
70 72 (81)a OMe (+)diisopropyl tartrate 3.5 9.5 4.6 8.1
p.
p. 100 25 Stablize enolate? (+)TMTDA 4.1 8.0 3.6 4.0
200 12 (+)hydrobenzoin 3.5 16.2 5.8 5.3
500 trace (+)triphenylethanediol 3.2 5.2 2.2 5.9
a
15 mol% DABCO was used. (+)TADDOL 2.9 4.5 3.8 4.7
ethylene glycol 3.3
• Ether was found to be optimal from solvent screening.
triethanolamine 4.65 10.8
• General for a variety of alkenes and aldehydes. salen 2.31 6.3 5.2 4.0
box 3.6
Kobayashi, S.; Kawamura, M. Tetrahedron Lett. 1999, 40, 1539-1542.
N-methylephedrine 2.87 5.8 3.2 4.4

• no enantioselectivity observed
• DABCO loading dropped to <10 mol% with (+)-BINOL
• rac -BINOL showed no rate acceleration

Aggarwal, V. K.; et. al. Chem. Commun. 1996, 2713-2714.


Aggarwal, V. K.; et. al. J. Org. Chem. 1998, 63, 7183-7189.

26B-04 11/9/01 1:07 PM


J. Janey The Asymmetric Baylis-Hillman Reaction Chem 206

Possible Stereoisomers
+ + + +
H NR H NR R 3N H R3 N H
O 3 O 3 O O
Hα Hα Hα Hα
H H H H
H R' R' H H R' R' H
Assumptions:
MeO O– MeO O– –O OMe –O OMe

• E enolate formed

+ + + +
– O H NR3 – O H NR3 R 3N H
O–
R3N H
O–
• E2 favored over E1 pathway
Hα Hα Hα Hα
H H H H
• -NR3+ is orthogonal to π face
H R' R' H H R' R' H
CO2Me CO2Me CO2Me CO2Me (stereoelectronics)

p.
p.

+ Hα + Hα Hα + Hα +
R 3N H R3 N H H NR3 H NR3
H OH HO H
MeO2C OH MeO2C H CO2Me CO2Me
H HO
H R' H R' H H
R' R'

120 ° rotation
then E2 elim.

OH OH
CO2Me CO2Me
R' R'

26B-05 11/9/01 1:08 PM


J. Janey The Asymmetric Baylis-Hillman Reaction Chem 206

E/Z Selectivity with Crotononitrile

OH OH
CN
r.t. CN CN
+ PhCHO Ph + Ph
Me
Me Me
1 eq 1 eq
E Z
Solvent E/Z ratio
neat 1.2 : 1 Base E/Z ratio
THF 1.4 : 1 DABCO 1:1
CHCl3 1.5 : 1 3-QDL 2:1
CH3CN 3.1 : 1 NEt3 4:1
MeOH 4:1 10 mol% base, 8 kbar,
17 h, CHCl3 50 vol%
5 mol% DABCO, 8 kbar,
p. 17 h, solvent 50 vol%
p.
• E and Z crotononitrile is easily isomerized under
the reaction conditions.

• Products did not undergo retro-Baylis-Hillman.

Rozendaal, E. L. M.; Voss, B. M. W.; Scheeren, H. W. Tetrahedron 1993, 49, 6931-6936.

5 mol% DABCO,17 h, solvent 50 vol%

26B-06 11/9/01 1:08 PM


J. Janey The Asymmetric Baylis-Hillman Reaction Chem 206

Possible Stereoisomers for Methylcrotonate


+ + + +
H NR Me NR Me NR H NR
O 3 O 3 O 3 O 3
Hα Hα Hα Hα
Me H H Me Assumptions:
R' H R' H H R' H R'
• E enolate formed
MeO O– MeO O– MeO O– MeO O–
• E2 favored over E1 pathway, only
+ + + + after rotation of ammonium to anti
–O
H NR –O
Me NR –O
Me NR –O
H NR conformation
3 3 3 3
Hα Hα Hα Hα
Me H H Me
H H R' R' • -NR3+ is orthogonal to π face
R' R' H H
CO2Me CO2Me CO2Me CO2Me (stereoelectronics)

p. • only one π face of enolate


p.
+ Hα + Hα + Hα + Hα considered, thus there are an
R 3N H R 3N Me R 3N Me R 3N H
OH OH H H additional 4 stereoisomers possible
MeO2C H MeO2C H MeO2C OH MeO2C OH
Me R' H R' H R' Me R' • starting geometry of methylcrotonate
and in situ isomerization not
considered
Hα Hα Hα Hα
H Me Me H Me H H Me
OH OH H H • retro-Baylis-Hillman not considered
MeO2C H MeO2C H MeO2C OH MeO2C OH
NR3 R' NR3 R' NR3 R' NR3 R'
+ + + +

OH OH OH OH
CO2Me CO2Me CO2Me CO2Me
R' R' R' R'

Me Me Me Me

26B-07 11/9/01 1:09 PM


J. Janey The Asymmetric Baylis-Hillman Reaction Chem 206

Camphorsultam Acrylate Baylis-Hillman α -Branched Aldehydes: Modest Felkin-Anh Selection


R1 R1
O O R = Et CO2Me r.t
RCHO, 10% DABCO MeOH, CSA +
R2 CO2Me R2 CO2Me
N O +
CH2Cl2, 0 °C, 12 h (85%) OH OH
S O R1R2CHCHO
R O R anti syn
O
R yield (%) ee (%) R1 R2 Conditions yield (%) anti:syn
Me 85 >99 MeOCH2O Me DABCO, 4 d 55 70:30
Et 98 >99 MeOCH2O Me 3-QDL, 1.5 d 60 72:28
MeO2C Me
n-Pr 70 >99 BnOCH2O Me DABCO, 6 d 42 70:30
OH
i-Pr 33 >99 MeOCH2O Ph DABCO, 10 d 42 37:63
Rh(I), H2 Me n-Pr 3-QDL, 60 d 30 35:65
PhCH2CH2 68 >99 (85%)
-OC(Me)2OCH2- DABCO, 55 d 62 69:31
AcOCH2 68 >99
p.
p. NHCO2t-Bu Me DABCO, 7 d 80 26:74
(CH3)2CHCH2 67 >99
Me
Ph 0 N-Phthalimidyl Me DABCO, 3.5 d 28 46:54
MeO2C Me -N(CO2t-Bu)C(Me)2OCH2- DABCO, 11 d 43 89:11
Leahy, J. W.; et. al. J. Am. Chem Soc. 1997, 119, 4317-4318.
OH
• Varying the amount of catalyst only affects the rate, not selectivity.

Camphorsultam Acrylate Mechanism • Anti and syn drawn incorrectly in review, should be reversed.

Author's model: RCHO


+ +
Dipole minimized –O NR3
NR3 + H t-BuO2C N
O– H
N –O NR3 Me
XcN
R O Ciganek, E. Org. React. 1997, 51, 217-218. O
S O H H
N H
O
syn selective
S O
O
O
O O–
O O +
XcN NR3
+ X cN R
O O NR3
O R
R3 N
R O R R O R +
–O R

26B-08 11/9/01 1:09 PM


J. Janey The Asymmetric Baylis-Hillman Reaction Chem 206

Chiral Aldehydes: Chromium Auxiliary OH


N CHO CO2Me 10 mol% cat. CO2Me
+
O MeO2C neat, r.t. 9-94 h
N
R OH 18-83% yield
H + R 2-19% ee
R 50% DABCO OH hν, air CO2Me R'O
Cr(CO)3 R H
neat, r.t. CH3CN cat. = P Ph R = Me and R' = H gave highest reactivity
Cr(CO)3
CO2Me R'O
R
aldehyde R time (h) yield (%) dr
excess rac OMe 93 87 >98:2 Zhang, X.; et. al. J. Org. Chem. 2000, 65, 3489-3496.
rac Cl 6 89 >98:2
rac F 7 92 92:8
rac Me 58 90 84:16
S-(+) OMe 93 85 >98:2
S-(+) Cl 8 97 >98:2
p.
p. The High Point of Chiral Phosphine Catalysts
1
• dr determined by 200 MHz H NMR
OH
• N-Tosyl arylimine chromium complex also reacts CHO CO2R2
N 20 mol% (S)-BINAP CO2R2
Kundig, P. E.; et. al. Tetrahedron Lett. 1993, 34, 7049-7052. + N
CHCl3, r.t. 3-14 d
R1 N
2.4 eq R1 N

Chiral Phosphine Catalysts R1 R2 time (d) yield (%) ee (%)

H i-Pr 4 8 9
18 mol% (-)-CAMP OH
O
CO2Et H Et 3 12 25
CO2Et neat, r.t., 10 d
H Me 4 24 44
75% yield (40% isolated)
P 14% ee Me Me 14 18 37
(-)-CAMP = Me
Me Me 3 26 30a
OMe 62% ee
a
Tol-BINAP was used
Frater, G.; et. al. Tetrahedron Lett. 1992, 33, 1045-1048. • other phosphines screened gave ~racemic products:
DIOP, NORPHOS, BPPFOH, and MOP

Soai, K.; et. al. Chem. Commun. 1998, 1271-1272.

26B-09 11/9/01 1:09 PM


J. Janey The Asymmetric Baylis-Hillman Reaction Chem 206

Naturally Occurring Alkaloids as Chiral Catalysts Model For Quinidine Catalyst


Author's model:

OH N N
CN ? mol% cat. H α H H α H
CN 0-81% yield
+ MeCHO 3-17% ee
Me
9 kbar, 25-60 °C – –
N+ O N+ O
• (-)-quinine, (1R,2S) N-methylephedrine, S-(-)-nicotine, S-(-)-N-methylprolinol screened H R H H
MeO O H MeO O H
Me Me
• (-)-menthyl acrylate ester gave 100% de with aromatic aldehydes and DABCO under high P H H
O H O R

Isaacs, N. S.; et. al. Tetrahedron: Asymm. 1991, 2, 969-972.

O OH O OH O OH O
10 mol% quinidine
+ RCHO R Me R Me
Me R Me
CH2Cl2, r.t. 20 h
p.
p.
40-50% yield minor major

R Pressure ee (%) • Cα hydrogens control π face of the aldehyde


n-Pr 3 kbar 18 • bulky R should enhance selectivity, a trend that they say is "...clearly visible."
• H-bonding plays a "clear role" as O-acyl quinidine gives no enantioselectivity
n-C9H19 10 kbar 31
i-Pr 3 kbar 37
c-hex 3 kbar 45 Alternative: N
H α H

major N+
• 3-QDL, quinine, cinchonine, cinchonidine, O-acetyl quinidine, N-methylprolinol, H
N-methylephedrine also screened O H H
MeO
H R
–O O
• ee is highly pressure dependent, optimized pressure is shown in table
H
Me
Marko, I. E.; Giles, P. R.; Hindley, N. J. Tetrahedron 1997, 53, 1015-1024.
Marko, I. E.; Giles, P. R.; Hindley, N. J. Tetrahedron 1997, 53, 1015-1024.

26B-10 11/9/01 1:10 PM


J. Janey The Asymmetric Baylis-Hillman Reaction Chem 206

C2 Symmetric DABCO Catalyst Chiral Pyrrolizidine Catalyst


O OH O
OH O 10 mol% cat.
CHO O ArCHO +
15 mol% cat. Me 1 eq NaBF4 Ar Me
+ Me Me
1% Hydroquinone CH3CN, -40 °C, 0.5-3 d
O2N
5-10 kbar, THF, 30 °C O2N
Ar yield (%) ee (%)
2-NO2 71 67
R time (h) yield (%) ee (%)
2-F 31 63 O2N
Bn 12 45 47
2-Cl 58 72
TBDPS 12 23 34 H
2-Br 63 71
TIPS 28 33 19 H
3-NO2 51 37 cat. =
N H Ph 16 60 35
cat. = 2-pyridyl 83 21 N
N HO
OR Mesityl 28 67 16
3-pyridyl 93 49
p.
p. H OR 1-naphthyl 16 66 42
4-quinolinyl 63 70
1-anthranyl 24 9 11
4-NO2 17 39
1-napththoyl 17 68 15
N-Cbz-Gly 24 63 21

Author's model:
• racemic alcohol product can be easily resolved by kinetic
resolution with Sharpless asymmetric epoxidation

• other chiral DABCO's made, but not tested... + +


O– N O– N
+ O + O
H Na O Na
H NO2 H NO2
N Ph N H Ph N Me Me H
H R O
N H N N Ph R
H Ph H favored disfavored
Ph Ph H
Barrett, A. G. M.; et. al. Chem. Commun. 1998, 2533-2534.
Hirama, M.; et. al. Tetrahedron: Asymm. 1995, 6, 1241-1244.

26B-11 11/9/01 1:10 PM


J. Janey The Asymmetric Baylis-Hillman Reaction Chem 206

C2 Symmetric DABCO Catalyst Chiral Pyrrolizidine Catalyst


O OH O
OH O 10 mol% cat.
CHO O ArCHO +
15 mol% cat. Me 1 eq NaBF4 Ar Me
+ Me Me
1% Hydroquinone CH3CN, -40 °C, 0.5-3 d
O2N
5-10 kbar, THF, 30 °C O2N
Ar yield (%) ee (%)
2-NO2 71 67
R time (h) yield (%) ee (%)
2-F 31 63 O2N
Bn 12 45 47
2-Cl 58 72
TBDPS 12 23 34 H
2-Br 63 71
TIPS 28 33 19 H
3-NO2 51 37 cat. =
N H Ph 16 60 35
cat. = 2-pyridyl 83 21 N
N HO
OR Mesityl 28 67 16
3-pyridyl 93 49
p.
p. H OR 1-naphthyl 16 66 42
4-quinolinyl 63 70
1-anthranyl 24 9 11
4-NO2 17 39
1-napththoyl 17 68 15
N-Cbz-Gly 24 63 21

Author's model:
• racemic alcohol product can be easily resolved by kinetic
resolution with Sharpless asymmetric epoxidation

• other chiral DABCO's made, but not tested... + +


O– N O– N
+ O + O
H Na O Na
H NO2 H NO2
N Ph N H Ph N Me Me H
H R O
N H N N Ph R
H Ph H favored disfavored
Ph Ph H
Barrett, A. G. M.; et. al. Chem. Commun. 1998, 2533-2534.
Hirama, M.; et. al. Tetrahedron: Asymm. 1995, 6, 1241-1244.

26B-12 11/9/01 1:10 PM


J. Janey The Asymmetric Baylis-Hillman Reaction Chem 206

Quinidine Ether Catalyst

O CF3 OH O CF3
Proposed Mechanism: Partial Kinetic Resolution
10 mol% cat.
RCHO + O CF3 DMF, -55 °C, 0.5-3 d R R O CF3 Et
H O
ester R
• obscures inherent facial N+
selectivity of the catalyst N –
O O O
+
R S O OR'
RCHO RCHO
ee (%), ee (%), OH
R yield (%) (config) yield (%) (config) dioxanone
p-NO2 58 91 (R) 11 4 (R) Et Hα Et
H H Y=R
Ph 57 95 (R) -- -- H O X=R –O H O
CO2R'
(E)-PhCH=CH 50 92 (R) -- -- N+
p. N Y N+
p. Et 40 97 (R) 22 27 (S) H X
CO2R' CO2R'
O N
i-Bu 51 99 (R) 18 18 (S) Hα H O Et H Hα
H
O H O H
i-Pr 36 99 (R) 25 25 (S) R
–O R
c-Hex 31 99 (R) 23 23 (S) –O
t-Bu -- -- -- -- N
RCHO RCHO
• Quinidine and other acyclic derivatives showed no enantioselection and very
low reactivity.
R R
• Free hydroxyl on quinoline is essential for enantioselectivity.
OH O OH O
• Reactions conducted at room temperature showed lower enantioselection. O O O O
• Racemic ester does not react to give dioxanone under the reaction conditions. R OR' R OR'
R R O R O S
R S
Hatakeyama, S.; et. al J. Am. Chem. Soc. 1999, 121, 10219-10220.

cat. = N
N
prepared in 65% yield from
quinidine in 85% phosphoric
OH acid and KBr (100 °C, 5 d).

26B-13 11/9/01 1:10 PM


J. Janey The Asymmetric Baylis-Hillman Reaction Chem 206

A Model for Facial Selectivity BINOL as an Additive or Ligand

O 20 mol% n-Bu3P: O OH
H 10 mol% BINOL
+ RCHO
THF, r.t. 2-24 h R
O
H
R
R yield (%)
n-C7H15 quant.
Ph 92 • ee were all <10%
MEMO(CH2)3 98 • phenol also accelerates reaction
Et 91 • other acrylates also tolerated
PhCH2CH2 quant.
p.
p.
Favored

PM3 minimized: C-N bond to enolate constrained to 1.6 Å


O
16 mol% Ca
• Catalyst orthogonal to opposite π face of the O
enolate leads to same major enantiomer after
elimination.
H O O OH
10 mol% n-Bu3P:
+ CHO
O Ph Ph
THF, r.t. 7 h
R
H 62% yield, 56% ee

Ikegami, S.; Yamada, Y. M. A. Tetrahedron Lett. 2000, 41, 2165-2169.

Disfavored

26B-14 11/9/01 1:11 PM


J. Janey The Asymmetric Baylis-Hillman Reaction Chem 206

A Related Phosphine Catalyzed Reaction 88% yield


100:0 A:B Phosphine Catalyzed Addition
CO2i-Bu 93% ee

EtO2C CO2i-Bu 10 mol% cat. RO2C 10 mol% cat.


+ + CO2i-Bu + NuH Nu CO2R
• •
PhCH3, 0 °C NaOAc/HOAc
CO2Et CO2Et r.t. PhCH3
10 eq
:PR3 A B NuH R yield (%) ee (%) product

O Me 80 73 O
Ph
P :PR3 :PR3 CO2Me
– cat. = i-Pr CO2Me Et 76 74
CO2R CO2R
i-Pr CO2i-Bu t-Bu 74 75
+ PR3
+ CO2i-Bu
– – O O
R3 P CO2Et R3 P CO2Et
CO2i-Bu + + COMe COMe
p. Et 67 56
p. CO2Et

CO2i-Bu O
O
COMe COMe
+ CO2i-Bu O Et 83 48
i-Pr i-BuO2C O CO2Et
– –
Ph
R3 P CO2Et R3 P CO2Et O
+ + P
– O
O2N NO2
• other catalyst and conditions give lower regio- and enantio- selection i-Pr Et 31 41 CO2Et
EtO2C

Zhang, X.; et. al. J. Am. Chem. Soc. 1997, 119, 3836-3837.
Lu, X.; et. al. J. Org. Chem. 1995, 60, 2906-2908. NC CO2Et NC CO2Et
Me 47 45 CO2Me

Zhang, X.; et. al. J. Org. Chem. 1998, 63, 5631-5635.

Ph
cat. = P
Me
Me

26B-15 11/9/01 1:11 PM


J. Janey The Asymmetric Baylis-Hillman Reaction Chem 206

Addition Mechanism Conclusions


Author's proposal:

CO2R cat. :PR3 • The Baylis-Hillman reaction provides convenient access to


• HOAc/NaOAc Nu CO2R valuable allylic alcohol building blocks which may serve as
PhCH3 :PR3 synthetic equivalents to anti-propionate aldol addition products.
:PR3
H+ shift
• The basics of the reaction mechanism are understood, but the
mechanistic details still remain elusive at best.
– –
CO2R NuH CO2R Nu CO2R • Few examples of a general, diastereoselective Baylis-Hillman
+ PR3 + PR3 – + PR3 have been reported and the successful ones are rather limited in
+ Nu scope.

• Only one synthetically useful enantioselective, base catalyzed


Recipe for a Good Catalyst? Baylis-Hillman reaction exists. There is no rational design, nor
p.
p. models for asymmetric catalysis.
O– OR • The asymmetric, catalytic Baylis-Hillman reaction is very
Control enolate geometry... vs.
+ NR3 OR + NR3 O– promising and attractive methodology, but remains an elusive
goal of chiral Lewis base catalysis.
E Z

H R
O O
Control aldehyde π face... R H
O– vs. O–
NR2 OR NR2 OR
+ +

• for substituted acrylates, must control enolate π facial selectivity


• chirality on catalyst may also gear ester substituent to influence aldehyde approach

or
R R2 X
O
R1 N O–
H
X O–
H OR
NR2 OR
+ O R

26B-16 11/9/01 1:11 PM


Chemistry 206

Advanced Organic Chemistry

Handout 26B

Asymmetric Carbonyl Ene Reactions

Evans Group Seminar


by
Steven Tregay, December 12, 1997


H H H
O O O

R H R R R R R

R R R

D. A. Evans Monday,
November 17, 2003

H26B-00 11/16/03 6:46 PM


Chem 206 Problems Containing the Ene Reaction

The problems provided on this and the following page deal with the ene reaction either directly or indirectly. In the latter
cases, this reaction is imbedded within a multistep rearrangement sequence. Answers to these questions may be obtained
by entering the descriptors "Rearrangement" and "Ene" into the problems database: http://evans.harvard.edu/problems/

Problem 210. The carbonyl ene reaction is illustrated below. Using FMO analysis, evaluate the transition state of this
reaction. Your answer should include: a transition state drawing; clear orbital depictions and HOMO-LUMO assignments; an
indication of the number of electrons from each segment; and indication of whether the reaction is thermally allowed.

O CH3 H
O
+
Ra H Rb
Ra Rb

Problem 19. The following transformation was recently reported by Barriault and Deon in conjunction with their synthesis
of arteanniun M (Org. Lett. 2001, 3, 1925-1927). Provide a mechanism for the illustrated thermal rearrangement(s) of A
to B. Where stereochemical issues are at stake, provide clear three dimensional drawings to support your answer.

Me
H H
DBU
toluene, 120 °C Single product
OR diastereomer
Me Me
OH OH
A B OR

N N
DBU is a useful amidine base; pKa ~ 12

Problem 83. Chiral methyl groups are commonly used to probe the stereochemical outcome of
biological reaction mechanisms. Many interesting strategies have been developed to synthesize
chiral methyl groups in high enantiomeric excess. The first approach, designed by Arigoni HO2C D
(Chem. Commun. 1975, 921), is illustrated below.
H T
D D

O OMe

D
*
260 °C T
T H

Provide a mechanism for the following transformation that accounts for the (H,D,T) stereochemistry of the chiral
methyl group. You do not need to account for the stereochemistry at the starred carbon (it was not determined by
the investigators).

H26B-01 11/16/03 6:26 PM


Chem 206 Problems Containing the Ene Reaction
Problem 177. Provide a mechanism that predicts the observed stereochemistry at the starred (*) carbon atoms
(Rajagopalan, Tetrahedron Lett. 1998, 39, 4133). Draw the starting material, each intermediate, and the product clearly in
3D.
Me Me
Me H CO2Me
CO2Me

OH **
*
OH
Me Me Me Me

Problem 184. The key step in Kim's synthesis of perhydrohistrionicotoxin, 3, was the conversion of intermediate 1
to ketone 2 in a single acid-catalyzed transformation (Chem. Commun., 1997, 2263). Provide a mechanism for the
conversion of 1 to 2 that accounts for the observed stereochemistry.
OH C5H11
H
OMe O
Me HN
p-TsOH/H2O O steps HO
O
H ∆ H
OH
n-Bu n-Bu n-Bu

1 2 3

Problem 203. Provide mechanisms that account for the stereoselective formation of the products obtained by treatment of
aldehyde A to the conditions shown below. Briefly comment on the difference in reactivity under the two sets of conditions
(JOC, 1998, 7586).
O O
O O

OH Ts O 10 mol% O O

SnCl4 Ts
N N
240 °C H 0 °C O H
B
N C
H Ts A

Problem 233. Snapper and co-workers have reported an approach to the [5.3.0] ring system that is
commmon to a number of sesquiterpenes(JACS 2001, 123, 5152). One cited example is alismol
H
whose structure is provided for reference. Upon thermolysis, the illustrated tetracyclic ester is
transformed into the illustrated bicyclic ring system in 64% yield (eq 1).

H CO2Me Me
CO2Me H
H HO Me
>130° C Me
H (1) Alismol
TBSO
OTBS
Provide a plausible mechanism for this transformation. Your answer should include an explanatin of the somewhat
unusual stereochemical inversion of the center carrying the flagged "red" hydrogen.

H26B-02 11/14/03 5:48 PM


Seminar Topics

Covered in this Seminar:


H OH
O Asymmetric Ene
(Metallo-Ene not covered)
R
R
Chiral Chiral Auxilaries

Catalytic/Promoted

R O
+
H R

R = Si, Sn
Reviews:

Snider in Comprehensive Organic


Chemistry, 1991, vol 2, 527.
Mikami, Chem. Rev. 1992, 92, 1021.
History: Mikami, Advances in Asymmetric
Synthesis, 1995, 1.
Alder, Ber., 1943, 76, 27. Bolm, ACIEE, 1995, 34, 1717.
Alder, Ann., 1962, 651, 141. Mikami, Advances in Catalytic
Processes, 1995, 1, 123.
Mikami, Pure & Applied Chem.,
1996, 68, 639.

Early Work on Chiral Glyoxylates

Me O
O
Me
2 equiv.
O *RO
O
SnCl4 (1.0 equiv)
Me OH
H 0 oC, CH3NO2
31 % ee, 87 % yield

Thermal reaction (160 oC) gave no induction

Achmatzowicz, JOC, 1972, 37, 964.

H26B-03 12/12/97 11:24 AM


Ene reactions of 8-Phenylmenthol Glyoxylate Ester
O
R O
H
*RO 2 equiv.
*RO R
O SnCl4, 0 oC, 3 hrs
OH
*R = 8-Phenylmenthol
>97 % de
< 6 % cis olefin observed
Me
Me O SnCl4 Alkene Product Yield
O
O
O
99 %
Me *RO
H
OH
O

*RO 92 %

OH

O R=H 100 %
(2:1 cis : trans)
OR
*RO OR R=OAc 59 %
OH
R=Bn 99 %
Whitesell, JCS CC, 1982, 989. R=TBS 89 %
Whitesell, Tetrahedron, 1986, 42, 2993.

Mechanism for 1-Substituted Olefins

Olefin Geometry

Absolute Configuration SnLn R


O S O
n
L O O
n H R H
Me *RO *RO
Me O SnLn
H H
O
O
Me
H
O O R

*RO R *RO

OH OH

Competing Pathway for Free Hydroxyl

O OH
O
O SnLn
O
H *RO
*RO
OH
H

H26B-04 12/12/97 11:24 AM


Mechanism for 1-Substituted Olefins

Olefin Geometry

Absolute Configuration SnLn R


O SnLn O
O O
H R H
Me *RO *RO
Me O SnLn
H H
O
O
Me
H
O O R

*RO R *RO

OH OH

Competing Pathway for Free Hydroxyl

O OH
O
O SnLn
O
H *RO
*RO
OH
H

1,1 Disubstituted Olefins


R
O
O
H R'
*RO 2 equiv.
*RO R'
O SnCl4, 0 oC, 3 hrs
OH R
*R = 8-Phenylmenthol
>97 % de

Alkene Product Yield

*RO 94 %

OH

O
84%
*RO 3:1
H
OH

O
OR R=H "Dominant product"
OR
*RO
R=OAc 42 %
OH 24 % trans olefin
Whitesell, JCS CC, 1982, 989. 20 % cis olefin
Whitesell, Tetrahedron, 1986, 42, 2993.

H26B-05 12/12/97 11:24 AM


1,2 Disubstituted Olefins
R
O
O R
H R'
*RO 2 equiv.
2
3
*RO R'
O SnCl4, -78 oC, 3 hrs
OH
*R = 8-Phenylmenthol
C-2 >97 % de

Alkene Product Yield

O C-3 92 % de 90 % yield
3 "Exo TS"
2
*RO
H
OH

O Me C-3 15 : 1

2
85 % yield
*RO 3

C-3 8 : 1
OH

2
one compound 86 % yield
*RO 3

OH

Whitesell, JCS CC, 1982, 989.


Whitesell, Tetrahedron, 1986, 42, 2993.

1,2 Disubstituted Olefin Mechanism


O

H Me
*RO +
R
*R = 8-Phenylmenthol O

O SnLn SnLn
H O
R
O O
H H
*RO *RO
R H
H H

O R O R

*RO *RO

OH OH

R = Me
trans-butene 15 : 1 Note: cis-butene does not isomerize in presence
cis-butene 8:1 of SnCl4 or SnCl4 / isopropyl alcohol at -78 oC
Isomerization does occur in the presence of gyloxylate
R = i-Pr
4-methyl-cis-2-pentene one compound

H26B-06 12/12/97 11:28 AM


Asymmetric Desymmetrization using 8-Phenylmenthol Glyoxylate Ester

OH OH
H H
H H
H Me SnCl4
Me O COOR* COOR*
(1.25 equiv.)
Ph
+ O
O -78 oC, 3.5 h
Me H H
H H
Excess 1 8 : 1 (72 % yield)

OH
H SnCl4 H
O H
Ph (1.5 equiv.)
COOR*
+ O
O o
-78 to -30 C, 3.5 h
Me
H H
H

one diastereomer

Whitesell, JACS, 1988, 110, 3585. (81 % yield)


Whitesell, JACS, 1986, 108, 6802.
Whitesell, JOC, 1985, 50, 3025. Note: 1 gives opposite bridgehead selectivity

Phenylmenthol Imine-Ene Reaction

Me
O
O
H Me
*RO 2 equiv.
*RO
N SnCl4, 20 oC
Bn NBn Me
*R = 8-Phenylmenthol
97 % de
76 % yield

O
O
H
*RO 2 equiv.
*RO
N SnCl4, 0 oC
Ts NTs
*R = 8-Phenylmenthol
> 98 % de
60 % yield

Use of Ts rather than Bn


was not discussed

Mikami, TL, 1993, 34, 4841.

H26B-07 12/12/97 11:28 AM


Ene reaction of (S)-2-(Ethylthio)-3-siloxy-1-butene

OMe OMe
SMe
Me2AlCl
+ Me
H
OTBS
OTBS
OTBS O OTBS OH SMe

99 % ee
100 % yield

Referenced in Mikami, Chem. Rev., 1992, 92, 1021

Kuwajima, Annual Meeting of the Chemical Society of


Japan, 1991.

Ene Reactions of N-Glyoxyloyl -(2R)-bornane-10,2-Sultam

O
R O
H
N 2 equiv.
*RO R
S O Cat (1.1 equiv.)
O OH
O

R= Catalyst Temp (oC) % de % Yield

SnCl4 -78 84 : 16 78
Et
ZnBr2 5 90 : 10 50
H
SnCl4 -78 75 : 25 93
N
O n-Pr
S ZnBr2 5 89 : 11 43
O O
O

Most Reactive Conformation according to


PM3 and Ab initio calculations
Chapuis, Helv. Chim. Acta, in preparation Jurczak, Tet.: Asymm., 1997, 8, 1741.

H26B-08 12/12/97 11:30 AM


Zn BINOL Promoted Intramolecular Ene Cyclizations

Me Me Me Me
3 equiv. Zn BINOL
88 % ee Note: using 1.5 equiv.
CH2Cl2, -78 to 0 oC 86 % yield reagent lowers % ee
CHO
OH

Me Me Me

O
Zn
Me Me O
Me Me

CHO
OH 88 % ee
3 equiv. Zn BINOL 89 % yield
Me
CH2Cl2, -78 to 0 oC (E)-methylfarnesal afforded other
regiochemistry in 20 % ee

(Z)- methylfarnesal
Yamamoto, Tetrahedron, 1986, 42, 2203.

Zn BINOL Promoted Intramolecular Ene Cyclizations

3 equiv. Zn BINOL 0 % ee
31 % yield
CHO
OH
CH2Cl2, -78 to 0 oC

Me Me Me

Me Me

3 equiv. Zn BINOL one compound


CHO
OH
CH2Cl2, -78 to 0 oC

Me Me Me
(R)-isopulegol Note : ZnBr2 gives 95:5 ratio of products
(R)- citronellal of idenical configuration
Me Me See: Nakatani, Syn. Comm., 1978, 147.

CHO
3 equiv. Zn BINOL one compound
OH
o
CH2Cl2, -78 to 0 C
Me Me Me

(S)- citronellal (S)-isopulegol

Yamamoto, Tetrahedron, 1986, 42, 2203.

H26B-09 12/12/97 11:32 AM


TADDOL Promoted Intramolecular Ene

O O R R R R
R R TADDOL TiCl2
(1.1 equiv) O O
N O
Solvent, 0 oC +
MS, 2-4 days N O

Me
Me O O
Me Me Me
1
2

R= Solvent 1 % ee (% yield) 2 % ee (% yield)


H Toluene (20 days) ND (17) ND (37)
Ph Ph
Toluene 82 (39) 92 (36)
Ph OH
Me 1,3,5 Trimethylbenzene 86 (32) >98 (37)
97 (47) Me OH
CFCl2CF2Cl/ CH2Cl2 ND (16)

-SCH2CH2S- CFCl2CF2Cl/ CH2Cl2 84 (ND) >98 (ND) Ph Ph

Narasaka, Chem. Lett., 1988, 1609.

3-3'-bis(triphenylsilyl)BINOL Aluminium Catalyst

O Me Cat (20 mol %) OH SMe


+
C6F6 H SMe CH2Cl2, -78 oC C6F6
4 A MS
88 % ee
88% yield

Note: Use of less reactive aldehydes (ie Chloral)


SiPh3 afforded lower % ee and stoic. LA were required

Use of MS is required for catalytic reaction


O Use of 3-3'-diphenylbinaphthol complex gave 0 % ee
Al Me
O

SiPh3

Yamamoto, TL, 1988, 29, 3967.

H26B-10 12/12/97 11:33 AM


(i-PrO)2TiCl2 (R)-BINOL Catalyzed Ene Reaction

Preliminary Result:

MS 4A
OH
+ (i-PrO)2TiCl2 Cat.
OH
CH2Cl2

O
O
Cat. (10 mol %) (R)
OEt OEt
+ H
CH2Cl2, -30 oC OH
O

86 % ee
82 % yield

Nakai, JACS, 1989, 111, 1940.


Nakai; Mikami, JACS, 1990, 112, 3949.

(i-PrO)2TiBr2 (R)-BINOL Catalyzed Ene Reaction : 1,1 Disubstituted

Olefin Product Cat. Mol % % yield % ee

(R)
OEt 5 73 98
OH

Me Me 10 87 94
OEt

Me OH

Ph Ph
OEt 1.0 98 94
Me OH

OEt 5 92 89
OH

Reaction conditions: Ethyl glyoxylate, -30 oC, 3 hr, MS, CH2Cl2


Nakai, JACS, 1989, 111, 1940.
Nakai; Mikami, JACS, 1990, 112, 3949.
Nakai, Org. Syn., 1993, 14.

H26B-11 12/12/97 11:35 AM


(i-PrO)2TiBr2 (R)-BINOL Catalyzed Ene Reaction : More 1,1 Disubstituted

Olefin Products

O
O
E Z
OEt
OEt 91 % yeild 9 % yeild
98 % ee OH >90 % ee
OH

O
O
E Z
39 % yeild OEt
OEt 91% ee 57% yeild 4% yeild
OH >98 % ee
OH >90 % ee

O
O
17% yeild
83 % yeild OEt >98 % ee
OEt 92% ee
OH
OH

Reaction conditions: Ethyl glyoxylate, 5 -10 mol % cat., -30 oC, 3 hr, MS, CH2Cl2

Nakai, JACS, 1989, 111, 1940.


Nakai; Mikami, JACS, 1990, 112, 3949.

Importance of Molecular Sieves

Additive
BINOL-(OH)2 + (i-PrO)2TiCl2 Method A
CH2Cl2

Additive ref: Reetz, Chem. Ind.


+ TiCl4 Method B
BINOL-(OLi)2 (London), 1986, 824.
CH2Cl2

For reaction of α-methyl Styrene and ethyl gloxylate (CH2Cl2, -30 oC)

Additive Additive
4 A MS Additive 4 A MS
(g/mmol) Yield % ee i-PrOH (g/mmol) Yield % ee
5 100 97 0 0 95 93
Method A 0 81 10 Method B 10 mol % 0 90 95
5 then filter 96 97 0 5 100 95
10 mol % 5 98 96
Note : By 13C NMR no Ti BINOL complexation Note : MeOH, t-BuOH give similar results
occurs until MS are added.
(i-PrO)2TiCl2 is a viable catalyst for the reaction.

Nakai; Mikami, JACS, 1990, 112, 3949.

H26B-12 12/15/97 7:22 PM


NonLinear Effect in the(i-PrO)2TiX2 BINOL Catalyzed Ene Reaction

X X
O Ti
O O
Ti O
X X

L*Ti
RCHO O RCHO
(R)(S)-(BINOL Ti X2)2
(R)(R)-(BINOL Ti X2)2 Slow
Fast
H COOEt

Note : For X-ray crystal structure of dimeric ((PhO)2TiCl2)2


For chiral poisoning of racemic See: Watenpaugh, Inorg. Chem. 1966, 5, 1782.
BINOL Complxes see:

Faller, TL, 1996, 37, 3449. Mikami, Tetrahedron, 1992, 48, 5671.
Mikami, Nature, 1997, 385, 613. Mikami, JACS, 1994, 116, 2812.

(i-PrO)2TiBr2 (R)-BINOL Catalyzed Ene Reaction : Other Enophiles

O
O
(R)
(i-PrO)2TiBr2 (R)-BINOL R
+ H R
20 mol %, RT, CH2Cl2, MS OH

Enophile n= % Yield % ee

O 87
0 85

1 70 94
H

COOMe
O
0 80 72

H COOEt 1 60 86

Mikami, TL, 1996, 47, 8515.

H26B-13 12/12/97 11:38 AM


Asymmetric Desymmetrization using (i-PrO)2TiBr2 (R)-BINOL
OR

COOMe
O
(i-PrO)2TiBr2 (R)-BINOL
+ 1
H
o (89 % ee)
20 mol %, -30 C
COOMe CH2Cl2, MS
OTBDMS
OTBDMS

OR

COOH

COOMe

2
n-C5H11

OH OH OTBDMS

isocarbacyclin
1 : 2 ratio 92 : 8

Mikami, TL, 1996, 47, 8515.


Mikami, Synlett, 1995, 29.

Asymmetric Desymmetrization / Resolution using (i-PrO)2TiCl2 (R)-BINOL

OR O OR O
(i-PrO)2TiCl2 (R)-BINOL
OEt Me
+ H 10 mol %, RT, OEt
CH2Cl2, MS
Me O OH
Me

R = Dimethylthexylsilyl > 99 % syn


(97 % ee)

OR O OR O
(i-PrO)2TiCl2 (R)-BINOL
OEt 2
+ H 10 mol %, RT, Me OEt
Me
CH2Cl2, MS
O OH
Me
1
> 97% syn
(>95 % ee)

Catalyst Product
OR
(i-PrO)2TiCl2 (S)-BINOL ent-1 (>99 % syn, 71 % yield)
Me
(i-PrO)2TiCl2 (R)-BINOL ent-1 (50 : 50 at C-2, 31 % yield)
Me

Mikami, Annual Meeting of the Chemical


Scoiety of Japan, 1990 and 1991.
See : Mikami, Synlett, 1992, 255.

H26B-14 12/15/97 7:18 PM


Synthesis of C10 - C15 and C30- C35 fragments of Rapamycin

RO (i-PrO)2TiCl2 BINOL Me O
Me O
ethyl glyoxylate
OEt
OEt
10 mol %, -30 oC
Me OR OH
CH2Cl2, MS OR OH

R= Dimethylthexylsilyl

(R)-BINOL (83 % yield) >99 <1

(S)-BINOL (61 % yield) 3 97

Mikami, TL, 1994, 35, 7793.


Me O Me O

OR OEt

OR OH OR O OH

C10 - C15 fragment C30- C35 fragment

(i-PrO)2Ti(ClO4)2 (R)-BINOL Catalyzed


{3, 4} exo, exo Intramolecular Ene Reaction

O
(i-PrO)2TiCl2 OH OH
X (R)-BINOL
H
Additive Me Me

X X
20 mol %, 0 oC
trans cis
Me Me CH2Cl2, MS,

Ratio trans
4 5 X= Additive time % Yield trans : cis % ee

H O none 24 h 73 47 : 53 70
3
2 1 O AgClO4 24 h 50 80 : 20 84
Type I -CH2- AgClO4 48 h 66 69 : 31 55

Mikami, TL, 1991, 32, 6571.


Mikami, Tet. :Asymm., 1991, 2, 1403.

H26B-15 12/12/97 11:41 AM


(i-PrO)2Ti(ClO4)2 (R)-BINOL Catalyzed
{2, 4} exo, exo Intramolecular Ene Reaction

O
(i-PrO)2TiCl2 OH
R O
H
(R)-BINOL
R AgClO4
Me
n O
20 mol %, 0 oC
CH2Cl2, MS, n
R
R

n= R= % yield % ee

0 H or Me --
4 5 NR
1 H 91
43
3 H
1 Me 82
40
1
2

Type II

Mikami, TL, 1991, 32, 6571.


Mikami, Tet. :Asymm., 1991, 2, 1403.

(i-PrO)2Ti(Cl)2 (S)-BINOL Catalyzed


{2, 4} exo, exo Intramolecular Ene Reaction

Me OAc
1) (i-PrO)2Ti(Cl)2 (S)-BINOL OAc
OHC CHO (25 mol %), CH2Cl2, 12h Me
Me +
2) Acylation
CHO CHO
Me

4.5 (38 % ee) : 1

H H
Me OH
H M
O H M O
O
Me
CHO
AcO Me OAc
Me
Trichothecene Anguidine CHO

Ziegler, JACS, 1990, 112, 2749.

H26B-16 12/12/97 12:10 PM


Approaches toward Ipsdienol

O O
+ (i-PrO)2TiCl2 (R)-BINOL
OEt PhX
XPh
H OEt
0.5 mol %, -30 oC
O CH2Cl2, MS OH

X= S X= Se

94 % yield 95 % yield
>99 % ee >99 % ee

O O
(i-PrO)2TiBr2 Ligand O
+ OEt
H OEt
2 mol %, -30 oC COOEt
O CH2Cl2, MS OH
Ene Hetero D.A.

OH
Product Ratio
Ligand Yield Ene Hetero D. A.

BINOL 94 79 (97 % ee) 21


Br
(R)-Ipsdienol
OH 84 92 (>99 % ee) 8
OH

Mikami, JCS CC, 1995, 2391. Br


Mikami, JCS CC, 1993, 327.

Corey's Model for Ti BINOL Ene Reactions


H
X H
H
H
X O
BINOL Ti X2
R
Aldehyde Complex O O H

(R)
OEt

OH
X

X O RO
R
O O H X HH H
OR O CH3
H
Me X
OEt O
R
OH O O H

OR

Corey, TL, 1997, 38, 6513.


Me Me
R= Dimethylthexylsilyl

H26B-17 12/12/97 11:45 AM


BF3 Menthylethyl Etherate catalyzed Ene reaction

O Me OH Me
Cat (20 mol %)
+
Cl3C H Me -70 oC, CH2Cl2 Cl3C

2 equiv.
22 % ee
55 % yield

Me

Demir, Syn. Comm. 1994, 24, 137.


BF3 Complex

OEt

Carreira's Catalytic Ene

O OMe Cat (2-10 mol %) OH OMe OH O


+ 2 N HCl
R H Me 0- 23 oC R Et2O R Me
used as solvent
~ $50 per L O3, Ph3P OH O
b.p. 34- 36 oC
CH2Cl2 R OMe
Aldehyde % eea % yield

99
Ph(CH2)3 CHO 98 OH O
OsO4, NMO
OH
Acetone/H2O R
TBSOCH2 CHO 93 85

t-Bu
Ph CHO 91 99

CHO 90 98
Ph N
Ti O
66 83 O Br
PhCHO
i-Pr-O O-i-Pr
Only α-
branched
aldehyde
75 79
which reacts CHO Carreira, JACS, 1995, 117, 3649.

a
% ee detrmind by NMR analysis of (S)-MPTA ester of
methyl ketone

H26B-18 12/12/97 11:46 AM


Yb(OTf)3 BINOL Catalyzed Ene Reaction

O Ph Cat (20 mol %) OH


+
MeO
H Me CH2Cl2, 0 oC Ph COOMe
24 h
O yields 78 - 82 %

Ligand R= % ee

H 12
R
Br 38

OH Ph 25
+ Yb(OTf)3
OH 29
TMS

Qain, TL, 1997, 38, 6721.

Jorgensen's Ene byproducts

O OH
10 mol % cat. O
OET + +
H RT, 12 hr COOEt
COOET
O

Diels-Alder Ene Product DA :Ene


Solvent % ee % ee Ratio
+
Me Me 2
CH2Cl2 85 83 1 : 1.8
O O 1 : 0.8
CH3NO2 90 78
N N
Cu
Ph Ph
– Jorgensen, Tetrahedron, 1996, 52, 7321.
2 TfO
For optimization of hetero Diels-Alder reaction products
See: Jorgensen, JCS PT 2, 1997, 1183.

H26B-19 12/12/97 11:48 AM


Work from the Evans Groups

"C2-Symmetric Copper(II) Complexes as Chiral Lewis Acids. Catalytic Enantioselective


Carbonyl-Ene Reactions with Glyoxylate and Pyruvate Esters". Evans, D. A.; Tregay, S. W.;
Burgey, C. S.; Paras, N. A.; Vojkovsky, T. J. Am. Chem. Soc. 2000, 122, 7936-7943.

Me Me 2+ Me Me 2+ Me Me 2+
O O O O O O

N N N N N N
Cu Cu Cu
Me3C CMe3 Me3C H O OH2 CMe3 Ph Ph
2
– – –
2 SbF6 2 SbF6 2 TfO

olefin producta catalyst mol% % yield % ee


O
(S) 2 1 90 97 (S)
OEt
3 10 99 87 (R)
OH

O
Me
Me 2 1 83 96 (S)
OEt 92 (R)
Me
3 10 92
OH

O
Ph Ph 2 1 97 93 (S)
OEt
3 10 99 89 (R)
Me OH

O
2 1 95 96 (S)
OEt
3 10 97 76 (R)
OH

C4H9 C4H9 O
2 Regiochemistry
1 89 96 (S) 75 : 25
OEt 3 10 81 92 (R) 90 : 10
Me OH

OTBDPS OTBDPS O
2 1 72 96 (S)
3 one regioisomer
OEt 10 85 91 (R)
Me OH

OBn OBn O
2 10 62 98 (S)
OEt 3 2 88 92 (R) one regioisomer
Me OH

O exo:endo
1 10 95 98 (S) 86:14
OEt 3 10 70 94 (R) 95:5
OH

O
C3H7 E: Z
H 7 C3 OEt 1 10 96 98 (S) 96 : 4
OH

H26B-20 11/16/03 6:30 PM


Origins of Enantioselectivity in Cu Box Ene Reactions

(S,S )-t-Bu Box Cu Glyoxylate (PM3tm) However, (S,S)-Ph-Box Cu (OTf)2 gives


(R) configured alcohols:

Tetrahedral Cu center??
Jorgensen, JOC, 1995, 60, 5757.

000
000
000
000
000
000
000
000
000
000
000
000
000
O

0000 00
0000
0000 00
0000
(S)

000
000
OEt

OH

00
(S,S)-Ph Box Cu (OTf)2(H2O)2 X-ray
H26B-21 12/12/97 11:51 AM
Quotes for the Day

"In the last third of his life, there came over Laslo Jamf-so it seemed to those
who from out of the wood lecture halls watched his eyelids slowly granulate,
spots and wrinkles grow across his image, disintegrating it towards old-age
hostility, a strangely personal hatred, for the covalent bond."

Thomas Pynchon, "Gravity's Rainbow"

"Faced with the choice between changing one's mind


and proving that there is no need to do so, almost everyone gets busy with the proof. "

John Kenneth Galbraith

Quotes-5 11/19/03 9:36 AM


Chemistry 206

Advanced Organic Chemistry

Handout–27A

An Organizational Format for the Classification of


Functional Groups. Applications to the
Construction of Difunctional Relationships

D. A. Evans Wednesday ,
November 19, 2003
27A-00-Cover page 11/17/03 1:11 PM
D. A. Evans Functional Group Classification Scheme for Polar Bond Constructions Chem 206

Papers of Historical Interest:


http://www.courses.fas.harvard.edu/~chem206/
"Arthur Lapworth: The Genesis of Reaction Mechanism."
M. Saltzman J. Chem. Ed. 1972, 49, 750. (Handout)
Chemistry 206
"A Theoretical Derivation of the Principle of Induced Alternate Polarities."
A. Lapworth J. Chem. Soc. 1922, 121, 416.
Advanced Organic Chemistry "The Electron Theory of Valence as Applied to Organic Compounds."
J. Steiglitz J. Am. Chem. Soc. 1922, 44, 1293.
Lecture Number 27 Monographs:
Hase, T. A. "Umpoled Synthons. A Survey of Sources and Uses in Synthesis".;
Functional Group Classification Scheme John Wiley & Sons, Inc.: New York, 1987.

for Polar Bond Constructions Ho, T.-L. "Polarity Control for Synthesis"; John Wiley & Sons, Inc.: NY, 1991.

■ Historical Perspective Ono, N., "The Nitro Group in Organic Synthesis", Wiley-VCH, 2001

■ Charge Affinity Patterns Several Interesting Problems O R R


■ Functional Group Classification Scheme 1) HO –
Provide a mechanism for the Nef reaction N H O
■ The Chemistry of the –NO2 Group O R 2) H3O +
R
■ The Chemistry of the –N2 Group The von Richter reaction is illustrated in the accompanying equation. Please provide a
plausible mechanism for this transformation taking into account the following observations.
Reading Assignment for this Week: (a) If 15N-labeled KCN is used, the N2 formed is half labeled; (b) 3-bromo-benzonitrile does
not form 3-bromo-benzoic acid under the reaction conditions.
"An Organizational Scheme for the Classification of Functional Groups.
Applications to the Construction of Difunctional Relationships." Br Br
D. A. Evans Unpublished manuscript. (Handout)
KCN
"Methods of Reactivity Umpolung." + N2
heat,
D. Seebach Angew. Chem. Int. Ed. Engl. 1979, 18, 239. (Handout) aqueous EtOH CO2H
NO2
"Nitroaliphatic Compounds–Ideal Intermediates in Organic Synthesis"' Seebach,
D. et. al, Chimia, 1979, 33, 1-18. (Handout) Stoltz and co-workers recently reported the interesting rearrangement illustrated below
(JACS 2003, 125, 13624). Please provide a mechanism for the illustrated transformation.
Your answer should include clear 3-D drawings where relevant. the answer may be found in
the database.

MeO O MeO O
Wednesday, N2 AgOBz, Et3N
D. A. Evans November 19, 2003 Me 95% yield
H THF, 45 °
Me
27-00-Cover Page 11/18/03 4:53 PM
D. A. Evans Ambiphilic Functional Groups Chem 206

Required Reading: Arthur Lapworth (1872–1941)


"An Organizational Scheme for the Classification of Functional Groups. Lapworth was among the first to understand and conceptualize the effect
Applications to the Construction of Difunctional Relationships." of heteroatomic substituents on the reactivity of individual carbon centers,
D. A. Evans Unpublished manuscript. and how this effect is propagated through the carbon framework of organic
molecules.
"Methods of Reactivity Umpolung."
D. Seebach Angew. Chem. Int. Ed. Engl. 1979, 18, 239. Lapworth's Theory of Alternating Polarities:

"Nitroaliphatic Compounds–Ideal Intermediates in Organic Synthesis"' "Latent Polarities of Atoms and Mechanism of Reaction, with Special
Seebach, D. et. al, Chimia, 1979, 33, 1-18. Reference to Carbonyl Compounds."
A. Lapworth Mem. Manchester. Lit. Phil. Soc. 1920, 64 (3), 1.
Papers of Historical Interest:

"Arthur Lapworth: The Genesis of Reaction Mechanism." "The addition of electrolytes to the carbonyl compound invariably
M. Saltzman J. Chem. Ed. 1972, 49, 750. proceeded as if the carbon were more positive than the oxygen atom,
and invariably selected the negative ion; for example:"
"A Theoretical Derivation of the Principle of Induced Alternate Polarities."
A. Lapworth J. Chem. Soc. 1922, 121, 416.
C O
"The Electron Theory of Valence as Applied to Organic Compounds." C O
J. Steiglitz J. Am. Chem. Soc. 1922, 44, 1293.
NC H NC H
––––––––––––––––––––– "The extension of the influence of the directing, or "key atom," over a
long range seems to require for its fullest display the presence of double
"Displacement of Aliphatic Nitro Groups by Carbon & Heteroatom bonds, and usually in conjugated positions...."
Nucleophiles." R. Tamura, A. Kamimura, N. Ono Synthesis 1991, 423.

"Functionalized Nitroalkanes as Useful Reagents for Alkyl Anion C C C O C C C O


Synthons." G. Rosini, R. Ballini Synthesis 1988, 833.
NC H
NC H
"Conjugated Nitroalkenes: Versatile Intermediates in Organic
Synthesis."
A. G. M. Barrett, G. G. Graboski Chem. Rev. 1986, 86, 751. The "key atom" is the one with the most electronegative
character, in this case the carbonyl oxygen.
Monographs:

Hase, T. A. "Umpoled Synthons. A Survey of Sources and Uses in anionoid/cationoid nucleophilic/electrophilic


Synthesis".; John Wiley & Sons, Inc.: New York, 1987.
The Lapworth polarity designations can be used to form the basis
Ho, T.-L. "Polarity Control for Synthesis"; John Wiley & Sons, Inc.: New of a functional group classification scheme.
York, 1991.
27-01-Historical 11/17/03 12:57 PM
D. A. Evans Reactivity Patterns of Functional Groups: Charge Affinity Patterns Chem 206

■ Polar rxns form the basis set of bond constructions in synthesis ■ The actual reaction associated with this transform is the addition of
organometals to carbonyl substrates.
■ Generalizations on conferred site reactivity will therefore be important
O M OM
O
CH3 CH CH3 CH3 CH CH3
Given this target and the desire to form this bond
R R

The functional group =O "dictates" the following bond construction When one considers the polar resonance
O– +M structure for the C=O group it is clear that an
O atom is very good at stabilizing an adjacent
CH3 CH – CH3
M + (+) charge through resonance.
O O O
M R
R R R R R
■ Consider polar disconnections of the illustrated β-hydroxy ketone 1:
O
O (–)
■ Conferred site reactivity of =O (–) TA
R (+) (+) R
R C CH3 CH2 O
O (–)
(+) (–) (+) (–)
Charge Affinity Patterns R C CH2 CH2 OH
(+) (–) (+) (–) TB O (–)
■ Use the descriptors (+) and (–) to denote the polar disconnections shown. 1 R C CH CH2 OH2
(+) (–) (+) (–)
(–) (+)
A B A: – B+ It is evident that the heteroatom functional groups, =O and -OH, strongly bias
(–) (+) the indicated polar disconnections.
A B A: + B:–

■ In the transforms illustrated above, symbols (+) & (–) are used to denote the
Charge Affinity Patterns of Common Functional Groups
particular polar transform illustrated.
In the present case there is NO INTRINSIC BIAS in favoring one transform over (+)
the other. Me CH2 CH2 Br C C C E1
H
Let's now add an OH functional group (FG) to propane at C-2 and see (–) (+)
whether one creates a bias in the favoring of one or the other transforms: Me CH2 C O C C C E2
OH OH (+) (+)
TA H 2C CH CH2 OH C C C E3
CH3 CH CH3 CH3 CH: – + CH3 disfavored
(–) (+) OR
(+) (–) (+)
OH H 2C CH C O C C C E4
OH
TB
CH3 CH CH3 CH3 CH + – : CH3 favored
(+) (–)

27-02-Chg Affinity 11/18/03 9:21 PM


D. A. Evans Classification of Functional Groups Chem 206

Functional groups activate the carbon skeleton at the point of attachment A-Functions:
by either induction & resonance.
A 3rd hypothetical FG, designated as A, may be defined that has an
Induction (+) (+) (–) (–) unbiased charge affinity pattern as in 1. Such an idealized FG's activates all
C F1 C F2 C F3 C F4 sites to both nucleophilic and electrophilic reactions, and as such include
(+) (–) (+) (–) those functions classifies as either E– or G–. The importance of introducing
Resonance
this third class designation is that it includes those functional groups having
non-alternate charge affinity patterns such as 2–4.
(+) (±) (–)
Symbol C E C A C G (+) (+)
Hypothetical A-function C C A 2
E = electrophilic at the point of attachment
A = ambiphilic at the pont of attachment (+–) (+–) (+–) (–) (–)
G = nucleophilic at the point of attachment C C C A C C A 3
1 (+–)
For simplicity, we will designate three FG classes according to the
C A 4
designations provided above.

E & G-Functions:
FG-Classification Rules
To organize activating functions into common categories it is worthwhile to
In the proposed classification scheme the following rules followed in the
define "hypothetical" functional groups E, and G, having the charge affinity
assignment of class designation of a given FG.
patterns denoted below.
■ Activating functions are to be considered as heteroatoms appended to or
Hypothetical E-function Hypothetical G-function included within the carbon skeleton.
(+) (–) (+) (–) (+) (–)
C C C E C C C G ■ Activating functions are inspected and classified according to their
observed polar site reactivities.
Given the appropriate oxidation state of the carbon skeleton, such functional
groups confer the indicated polar site reactivity patterns toward both
■ Since proton removal and addition processes are frequently an integral
electrophiles and nucleophiles.
aspect of FG activation, the FG, its conjugate acid or base, and its proton
Any FG that conforms either to the ideal charge affinity parrern or a tautomers are considered together in determining its class designation.
sub-pattern thereof will thus be classified as either an E- or G-function.
■ The oxidation state of the FG is deemphasized since this is a subordinate
strategic consideration.
Representative E-functions:

Me CH2 CH2 Br
(+) Common E-Functions: Symbol: (+) C E
C C C E1
H
(–) (+)
Me CH2 C O C C C E2 OR O exception: O
(+) (–) (+)
(+) (+) C C C E NR2 NR exception: N
H 2C CH CH2 OH C C C E3
OR X, X = halogen
(+) (–) (+)
H 2C CH C O C C C E4
Also consider all combinations of of above FGs; e.g =O + OR
27-03-FG Classification-1 11/18/03 9:23 PM
D. A. Evans Classification of Functional Groups Chem 206

Common G-Functions: Symbol: (–) C G H-tautomer conjugate base conjugate acid


O HO O HO
Typical G-class functions are the Group I-IV metals whose reactivity patterns,
falls into a subset of the idealized G-FG 5. N CH2R N CHR N CHR N CHR
O O O HO
(–) (–) (–) (+) (–)
H 2C CH CH2 Li C C C G FG C (–) FG C (+)
(–)
5
CH3 CH2 MgBr
–O
The Reaction: + N CH–R
Common A-Functions: Symbol: (±) C A O –O O R
base El(+)
+ N CH2R +N
A-functions are usually more structurally complex FGs composed of
–O –O El
polyatomic assemblages of nitrogen, oxygen and their heavier Group V and VI
relatives (P, As, S, Se). pKa ~ 10 O –
●●

+ N CH–R
O (–)
Typical A-functions, classified by inspection, are provided below –O
+ N CH2–R
The charge affinity pattern: –O
NO2 NOR NNR2 N(O)R N2 N
+
SR S(O)R SO2R SR2
+ ■ This reactivity pattern may be extended via conjugation:
PR2 P(O)R2 PR3
O
■ These FG's are capable of conferring both (+) and (–) at point of attachment. The Reaction: + N CH CH R
–O O (–) (+)
(+) Nu(–)
+ N CH2 CH Nu
X: X: X:
(–) N N N –O R
–O
+
X = OR, NR2 + N CH CH R
R (+) H R H R (–) H
–O

Remarkably, the dual electronic properties of oximes were first discussed by O (–) (+)
Lapworth in 1924 before the modern concepts of valence bond resonance Charge affinity pattern: + N CH CH R
were developed. –O
Lapworth, A. Chemistry and Industry 1924, 43, 1294-1295.

The Nitro Functional Group ■ The resonance feature which has been exploited:

As an example, the class designation of the nitro function is determined by an (+)


X: X: X:
evaluation of the parent function, its nitronic acid tautomer, as well as (–)N N ✔✔ N
conjugate acid and base. X = OR, NR2
R (+) H R H R (–) H

27-04-FG Classification-2 11/18/03 9:24 PM


D. A. Evans Charge Affinity Patterns & the Nitro Functional Group Chem 206

Some Reactions of the Nitro Functional Group The Nef Reaction

O OH O R R
R 3N 1) HO –
O
R NO2
R ■ Overall Transformation: +N H O
(–) (+) (–) H (+) R R 2) H3O +
EtOH –O R R
+ N CH CH R
NO2
–O
O O
■ Mechanism O R
R 3N HO – –O R HO R
R NO2 H+
R +N H +N +N
(–) (+) EtOH –O R –O R –O R
NO2 nitronate anion nitronic acid
O O O2N O2N
R 3N O H+
MeO Me (+) EtOH
Me
HO R HO R HO R
CO2Me + H 2O
N H O N +N
R -H +
O2N O2N HO H R HO HO
Et–NH2 OH R

(+) EtOH
Me N The charge affinity patterns represented
H

O R –O R HO R
Important Transformations of the –NO2 Functional Group HO – H+
+N H +N (±) +N (±)
–O R –O R –O R
O O
(–) nitronate anion nitronic acid
H2
■ Reduction: rxn is quite facile O2N C(–)
R Pd, Ni, Pt etc R H+

NO2 NH2 R
O HO R H 2O HO R
+ (+)
O N + +N (+)
R3SnH R -H
Ono, N.; Kaji, A. H R HO HO
OH R
∆ R Synthesis 1986, 693.
O2N C(+)
O O ■ The resonance features which have been exploited:
■ Nef Reaction: –
HO (+)
R H 3O + X: X: X:
R (–) N ✔✔ N ✔✔ N
X = OR, NR2
NO2 O Pinnick, R (+) H R H R (–) H
Org. Reactions 1990, 38, 655
O2N C(+) O2N C(–)
27-05 NO2 Chem-1 11/18/03 9:25 PM
D. A. Evans Charge Affinity Patterns & the Nitro Functional Group Chem 206

Other Nonalternate Behavior of –NO2 FG (–) (–)



Representative examples: O2N C C
H ●●

O CH2R –O CHR –O CH2


base base Bn
+N +N +N –O Ph
O Ph PhCH2Br O Ph
–O –O H –O H 2 LDA
+N +N +N
–O –O
–O
40% yield
base

CH2 Bn
–O CH2R –O
CO2Et –O CO2Et PhCH2Br
O CO2Et
N O 2 LDA
nitronate dianion +N ●●
nitronate dianion +N +N
– –O H +N
–O –O –O
–O
Seebach et. al. Tetrahedron Lett. 1977, 1161-1164 80% yield
2(–) (–) (–)
O2N C C O2N C C
Reactivity Patterns
–NO2 As a Leaving Group
Seebach et. al. Tetrahedron Lett. 1977, 1161-1164 Review: Tamura et. al. Synthesis 1991, 423-434.
"Nitroaliphatic Compounds–Ideal Intermediates in Organic Synthesis"'
Seebach, D. etal, Chimia, 1979, 33, 1-18
2(–)
Representative examples: O2N C C Representative examples: O2N C(+)

O CH2Me O R R
O CH2Me 2 BuLi –O Et PhCH2Br Nu(–)
+N +N +N +N CH Nu CH + NO2–
●●

–O –O – –O Bn –O R R
51% yield

SnCl4
Ph
74%
O Ph –O Ph O Ph
2 BuLi Et–I
+N +N ●●
+N NO2 Ph
–O –O – –O Et
80% yield SiMe3
NO2
Me Me
Me TiCl4 Me
SPh 65% SPh
27-06 NO2 Chem-2 11/18/03 9:26 PM
D. A. Evans The Nitro Function as a Leaving Group Chem 206

–NO2 As a Leaving Group O O

CO2Et
N N CO2Et
Representative examples: O2N C (+) + (+) (+)
O2N C C
NO2
NO2
O R R
Nu(–)
+N CH Nu CH + NO2–
H3O+
–O R R
O CO2Et O CO2Et
(+) R2NH
(+)
SnCl4 O2N C C
–NO2–
Ph NO2
74%
(+) (+)
NO2 Ph
McMurry etal. Chem Comm. 1971 488-489. O2N C C

NO2 SiMe3

Me Me O
Me TiCl4 Me O O
65% R 3N R2NH
SPh SPh
(+) (–) CO2Et CO2Et CO2Et
NO2 N(CH2)5 NO2 NO2
(–)
Pd(PPh3)3 Bakuzis etal. Tetrahedron Lett. 1978 2371. O2N C C
(+)
N
H
CH(CO2Me)2
CO2Et (+) NO
Pd(PPh3)3 2
NaCH(CO2Me)2 + (+)

MeO NO2 MeO CO2Et


SO2Ph
DBU –NO2–
Pd(PPh3)3
(+) (+)
NaO2SPh O2N C C
Danishefsky etal. JACS 1978, 100, 2918.
MeO CO2Et

27-07 NO2 Chem-3 11/18/03 9:26 PM


D. A. Evans Nonalternate Reactivity Patterns of Diazo Functional Groups Chem 206

The Diazo Functional Group


Acid Catalyzed Reactions of Diazo Compounds
H H H
– + – +
+C N N C N N –C N N Review: Smith, Tet. 1981 2407
R R R

■ Both (+) and (–) reactivity patterns suggested by resonance structures O O


H+
CH3 CH3 N2 C R
H 3C H3 C
(–)
■ Rxns with acids: N N
H +H – H N N
+ X
H X –C N N H C N N H C X N N Diazocarbonyl Diazonium
R R R
Common acids include BF3•OEt2, HBF4, TFA, etc.

N2 C R N2 C R
(–) (+) Mechanism of activation is unclear for both Lewis and protic acids; activation
may occur by protonation on C or O
■ Initiating reactivity is (–); subsequent reactivity is (+)

■ Ring expansion reactions: O O

CH2N2 Acid-Catalyzed Reactions


EtOH O N2 C R
N2 (–)
TFA, -20°C O
+
N2 C R HO CH2–N2 N2 C R (96%)
(–) (+)
O N2 C R
HO Mander, Chem. Comm. 1971 773 (+)
Tet., 1991 134
Restriction: Starting ketone must be more reactive than product ketone
"Having become familiar with the peculiarities of diazoketone chemistry while preparing
(–) [other compounds] (and, I might add, inured to handling uncomfortably large quantites of
■ Precursors to Carbenes: diazomethane), it occurred to us that we might be able to substitute a diazo group for
N2 C bromine."
(+) empty (+) Lewis Mander
H + H O
– ∆
C N N C: H Cl3COCO OCOCCl3
N2
R –N N R C filled (–)
R TFA O
-25°C, 2 min
Gibberrellic Acid
H E –E,G H (82%)
(+, –) C C: OMe O Mander, JACS 1980 6626
R G R

27-08 diazo chem 11/19/03 8:47 AM


D. A. Evans Nonalternate Reactivity Patterns of Diazo Functional Groups Chem 206

Diazo–Carbonyl Insertations: Web Problem 109. The following is a general reaction for the formation of pyrroles.
In this condensation, any of the three reaction constituents may be widely varied.
O (Ono, "The Nitro Group in Organic Synthesis" Wiley-VCH, 2001. Chapter 10, pp
N2 326-328). Siince it is not clear what the "inorganic" reaction product is, provide us
O
OO BF3•OEt2 N2 –N2 with anything that is mechanistically sound using the reagents illustrated. Key
descriptor for answer, "Nitro". O
OBF3 O O Me Ph
R R
R O NO2
Ph NH3
Me Me
Mander, Aust. J. Chem. 1979 1975 Me
N
H
In the space below provide a plausible mechanism for this transformation.
Wolff Rearrangements
Web Problem 332. Stoltz and co-workers recently reported the interesting
Web Problem 188. Provide a mechanism for the following reaction that predicts the
rearrangement illustrated below (JACS 2003, 125, 13624).
stereochemistry at the starred (*) carbon atoms (Valentin, TL, 1983, 1621). Key
MeO O MeO O descriptor for answer, "Nitro".
H i-Pr
N2 AgOBz, Et3N O H NO2
95% yield NO2
H THF, 45 °
Me
N * **
Me *
i-Pr Me Me
N O
AgOBz, Et3N –N2 Cope Rearr
O
O Web Problem 150. Provide a concise mechanism for the indicated reaction in the space
C
H H Me below. Key descriptor for answer, "Carbene".
Wolff Rearr
Me H H N2
H O
H
H CF3COOH
cis olefin O
Diazo-mediated Ring Construction:
O
Evans, Mitch, JACS 1980, 102, 5956
Morphine alkaloid MeO
Ph skeleton
Web Problem 13. The following transformation was recently reported by Stoltz (J. Am.
1. HClO4
CH2 30% yield Chem. Soc 2002, 124, 12426). In addition to the illustrated product, styrene and
N 2. CH2N2 dinitrogen are produced as by-products in this transformation. Key descriptor for
Me N
Me answer, "Carbene".
HClO4 –N2 Ph O R
A C (–) A C (+) cat Rh2(OAc)4
N 130 °C
Intermediate
Ph N – styrene 1 –N2 H
N Me
CH2N2 O R Ph
H Ph
N H anti diastereoselection > 20:1, 80% yield
Me CH2–N2
H Provide a plausible mechanism for this transformation and identify intermediate 1. Your
mechanism should provide a rationalization for the product stereochemical relationship.
A
27-09 diazo chem-2 11/19/03 9:18 AM
D. A. Evans, N. Finney Hydrazone Transformations-1 Chem 206

Hydrazone Anions: A useful Reversed Polarity Equivalent O O Me2N


N N NO2
R R R Me2N RT, ca. 24h
N
N:– N:– N: + R' R'
CH2Cl2
(–) N N ✔✔ N CH2
R
R
R (+) H R H R (–) H 1. O3; 2. DMS
A C(+)
A C(–)
A C(+) A C(–) (40-92%)
O NO2
R=alkyl or aryl
R'=H or alkyl H R'
t i t i t i
Bu N Pr n-BuLi, 0°C Bu N Pr Bu N Pr
N N N R
THF Lassaletta, J-M, et al.Tet. Lett. 1992, 33, 3691.
H H H H

J. E. Baldwin, et al. JCS Chem. Comm. 1983, 1040. PhCHO Wolff-Kishner Reduction Procedures

Me R Me R
t i O
O i
Pr Bu N Pr N 2H 4 ,
1. n-BuLi N Me NaOCH2CH2OCH2CH2OH, Me
95% H
(HOCH2CH2)2O
HO Ph 2. H+/H2O LiO Ph H H H H
H H
CH3CO2 O reflux and then
heat to 210°C HO
H H
Me Me Me Me
tBu
tBu Barton, D. H. R., Ives, D. A. J., and Thomas, B. R. J. Chem. Soc. 1955, 2056.
NH 1. n-BuLi, -78°C N
N N
Me OCH3 OCH3 For particulary hindered ketones: anhydrous hydrazine or formation of hydrazone
Me H Me under acid catalysis (hydrazine/hydrazine dihydrochloride), then basify.
H
O Me O
Under these forcing conditions, saponification, epimerization, and methyl ether
cleavage can occur.
A C(–) H+, H2O
Mechanism

A C(+) R 2C N NH2 + OH R2C N NH R 2C N NH


tBu
O N A C(–) H
hydrolysis N H RO–H
OCH3
Me OCH3
58% Me
Me O
Me O OH –N2
R 2C N N R2 C R 2C H
(RDS)*
H H H
J. E. Baldwin, et al. JCS Chem. Comm. 1984, 1095.
A C(–) RO–H RO
27-10 Hydrazones-1 11/18/03 9:28 PM
Chemistry 206

Advanced Organic Chemistry

Handout–27A

An Organizational Format for the Classification of


Functional Groups. Applications to the
Construction of Difunctional Relationships

D. A. Evans Wednesday ,
November 19, 2003
27A-00-Cover page 11/17/03 1:11 PM
Functional Group Classification Chemistry 206, 2001

An Organizational Format for the Classification of Functional Groups.


Applications to the Construction of Difunctional Relationships
D. A. Evans
Department of Chemistry & Chemical Biology, Harvard University, Cambridge, MA, 02318
Introduction
Among the subdisciplines of chemistry the area of organic synthesis is probably the least organized
in terms of unifying concepts and general methodology. This conclusion has been made quite obvious by
the relative scarcity of critical monographs covering this important topic.1 The wide structural diversity of
organic molecules, the vast abundance of organic reactions, and the restrictions imposed upon these
reactions when applied to the synthesis of a complex structure all contribute to the magnitude of the
problem of making generalizations in this area.
However difficult the overall task of explicitly defining a priori a total synthesis of an organic
structure may be, there are certain simplifying features which can be developed to generate logical sets of
potential synthetic pathways to a given molecular target . Some of the general guidelines which help to de-
fine this task have been outlined.2 Recently, some of the problems associated with reducing synthetic de-
sign to a mathematical basis and the application of machine computation to synthetic analysis have been re-
ported.3,4
Difunctional Relationships. One aspect of the synthesis of any polyfunctional target structure
deals with strategies associated with the construction of arrays of relationships between heteroatom func-
tional groups which may be denoted as F1, F 2 , etc. The general reactions illustrated below simply repre-
sent the union of two monofunctional organic fragments where the functional groups F 1 , F 2 provide the
necessary activation for the coupling process. In these reactions, the oxidation states of the associated car-
bon fragments are purposely left undefined. In relating the generalized notation below to a real situation, if
F1-C-C were an enolate, Equation 1 might be used to represent a generalized aldol or Mannich reaction
while equation 3 might represent a Michael reaction.
F1 F2 F1 F2

(1)
C C + C C C C

F1 F2 F1 F2

(2)
C C + C C C C C C

F1 F2 F1 F2

C C + C C C C C C C C
(3)

Henrickson has provided some useful generalizations on the construction of difunctional relation-
ships which are worth summarizing. For example, he defines the construction span as the number of
carbons linking F1 and F2. In the cases illustrated above, the product of the reaction illustrated in Equation
1 has a construction span of three. The construction fragments are then defined as the monofunctional
reactants, such as F1-C-C and F1-C. In general, construction spans are limited to six or less. This is a
consequence of the fact that the operational utility of a given functional group diminishes as it is removed

1) (a) Corey, E. J.; Cheng, X.-M. The Logic of Chemical Synthesis; Wiley, New York, 1979. (b) Fuhrhop, J.; Penzlin,
G. Organic Synthesis: Concepts, Methods, Startimg Materials; Verlag Chemie, Weinheim, 1983. (c) Carruthers, W.
Some Modern Methods of Organic Synthesis, 3nd ed.; Cambridge Univ. Press, Cambridge, 1987. (d) Organic Synthesis,
The Disconnection Approach; Wiley, New York, 1982. (e) Payne, C. A.; Payne, L.B. How To Do An Organic
Synthesis; Allyn and Bacon., Boston, 1969. (f) Ireland, R. E. Organic Synthesis, Prentice-Hall, Inc., Englewood Cliffs,
1969.
2) (a) Corey, E. J. Pure Appl. Chem. 1967, 14, 19. (b) Corey, E. J. Quart. Rev. 1971, 25, 455.
3) (a) Hendrickson, J. B. J. Am. Chem. Soc. 1971, 93, 6487. (b) Ugi, I; Gillespie, P. Angew. Chem. Int. Ed. 1971, 10,
914. (c) Corey, E. J.; Wipke, W. T.; Cramer, III, R. D.; Howe, W. J. J. Am. Chem. Soc. 1972, 94, 421. (d) Corey,
E. J.; Cramer, III, R. D.; Howe, W. J. ibid. 1972, 94, 440, and earlier references cited therein. (e) Corey, E. J.; Howe,
W. J.; Pensak, D. A. ibid. 1974, 96, 7724. (f) Blair, J.; Gasteiger, J.; Gillespie, C.; Gillespie, P. D.; Ugi, I.
Tetrahedron 1974, 30, 1845. (g) Bersohn, M. J. Chem. Soc., Perkin I 1973, 1239.
4) (a) Thakkar, A. J. Fortschritte Chem. Forschung 1973, 39, 3. (b) Dungundji, J.; Ugi, I. ibid. 1973, 39, 19. (c)
Gelernter, H.; Sridharan, N. S.; Hart, A. J.; Yen, S. C.; Fowler, F. W.; Shue, J.-J. ibid. 1973, 41, 113.
Functional Group Classification page 2
from the C-C bond being formed. The problem of site or ambident reactivity in systems possessing ex-
tended conjugation is the principal liability in the extension of the construction span. This point is illus-
trated below for both conjugate addition and enolate alkylation (Scheme I).
Scheme I The Problem of Ambident Reactivity
Nu O
Nu(-)
1,6 Addition MeO2 C α-alkylation El(+)
H+ R MeO R
+
OM H
4 6 El
MeO2C
R MeO R O
Nu α γ
Nu(-) El(+)
1,4 Addition MeO2 C γ-alkylation + MeO R
H+ R H
El

The objectives of the present discourse are to present an organizational format which can serve to
correlate strategies for the construction simple pairwise functional group relationships. As a result of the
overwhelming predisposition of nature to employ polar rather than free radical processes in the
biosynthesis of organic compounds the chosen organizational format reflects this bias in reaction type.
The designation of reactions as polar is recognized to be rather arbitrary since known reactions vary widely
in their polar character, ranging from essentially nonpolar radical reactions and weakly polar electrocyclic
reactions to strongly polar ionic processes. Of primary concern in this discussion will be those reactions
that involve charged species at some point along the reaction coordinate.
Charge Affinity Patterns.
In order to describe an organizational model for the
classification and synthesis of heteroatom-heteroatom A B A: – B+ (4)
(difunctional) relationships in organic molecules, two familiar
ideas will be employed. The first is that in a given target molecule A B A: + B:– (5)
the various bonds can be ionically "disconnected" (eq 4, 5). That
is, if the A-B bond could be cleaved heterolytically, the indicated set of polar fragments would result.
This antithetic process suggests ionic precursors suitable for the construction of the target molecule
via polar coupling processes. The second well accepted idea is that functional groups determine site
reactivities on a carbon skeleton based upon known reactions. That is, the oxygen atom in both acetone
and anisole dictates the site reactivities that are displayed for each molecule with nucleophilic and
electrophilic reagents. Thus, if the molecule A-B contained one or
more functional groups proximal to the bond to be disconnected, (–) A B
(+)
A: – B+ (6)
one pair of ionic precursors, eq 6 or 7, would be strongly favored (–) (+)
as plausible precursors. In such a case the favored ionic
precursors to A-B could be symbolized with either (+) or (-) in the A B A: + B:– (7)
target molecule, e.g.5
As an example, two possible polar disconnections for ketone 1 are illustrated below. The parity
labels in the target structure suggest plausible monofunctional precursors from which the target structure
can be assembled by polar processes. It is also evident that the heteroatom functional groups, =O and -
OH, strongly bias the indicated polar disconnections.
cheme II Polar Disconnections and Charge Affinity Pattterns
O (–)
TA
R C CH3 CH2 O
O(–)
(+) (–) (+) (–)
R C CH2 CH2 OH
(+) (–) (+) (–) O(–)
TB
1 R C CH CH2 OH 2
(+) (–) (+) (–)

5) The use of the symbols, (+) and (-), in no way represents formal positive or negative charges and will always be
bracketed to denote this distinction. Other forms of notation have been considered such as (0) and (1) to denote a
potential site of electrophilicity or nucleophilicity; however, the chosen symbols convey more direct information to the
organic chemist.
Functional Group Classification page 3
For any given atom or heteroatom assemblage which is defined as a functional group linked to a
carbon skeleton, the parity labels, (+) and (-), may be employed to denote the positional polar site
reactivity, or charge affinity pattern which the functional group confers upon the carbon framework.
For the simple molecules shown below (Scheme III) containing a homogeneous set of activating
functions, E, there are associated charge affinity patterns 2 - 5 of which each is a sub-pattern of the gen-
eralized structure 6. Note that the carbonyl function is defined as =O rather that C=O in this discussion.
You might contemplate why this functional group is defined in this fashion.
Scheme III Charge Affinity Patterns of Common Functional Groups
(+)
H3 C CH 2 CH 2 Br C C C E1 2
H
(–) (+)
H3C CH 2 C O C C C E2 3 (+) (–) (+)
(+) (+) C C C E
H 2C CH CH 2 OH C C C E3 4 6
OR
(+) (–) (+)
H2C CH C O C C C E4 5

The notion that an organic structure can be viewed as an "ion assemblage" has an interesting his-
tory originating with the work of Lapworth and others.6, 7 Although the ion assemblage viewpoint was
developed historically to predict site reactivity in both aliphatic and aromatic systems, this description of an
organic structure is equally instructive in defining rational sets of synthetic pathways for a given target
structure employing heterolytic processes as the primary set of coupling reactions. Indeed, the thought
processes associated with the construction of organic molecules operate intuitively to recognize many sub-
units of a given structure in terms of polar fragments. The present use of parity labels to denote viable
polar fragments simply formalizes this intuition.
Classification of Functional Groups (FG).
In order to organize general strategies that have been developed to construct heteroatom-heteroatom
relationships from monofunctional precursors it is useful to develop a self-consistent classification scheme
for single functional groups (FG) based on the concepts of polar disconnection and conferred site reactiv-
ity towards nucleophiles and electrophiles. The proposed scheme recognizes the dominate inductive and
resonance components of various substituents and establishes8 broad categories for activating functions
which correlate similar conferred chemical properties to carbon.9 Four possible functional group cate-
gories (F1-F4) are shown below. Those FGs which are more electronegative than carbon provide in-
ductive activation defining the electrophilic potential at the point of attachment denoted as (+). In a com-
plementary fashion, FGs which are less electronegative than carbon provide inductive activation creating
nucleophilic potential at the point of attachment denoted as (–). Since FG activation through induction and
resonance are independent variables which contribute to the overall FG reactivity pattern, four possible
classes of functional groups can be defined (Scheme IV). This discussion is reminiscent of the classifica-
tion of FGs according to their impact on electrophilic aromatic substitution.10
Scheme IV Classification of Functional Groups
Induction (+) (+) (–) (–)
C F1 C F2 C F3 C F4
Resonance (+) (–) (+) (–)

(+) (±) (–)


Symbol C E C A C G

6) (a) Lapworth, A. Mem. Proc. Manchester Lit. Phil. Soc. 1920, 64, 1. (b) Lapworth, A. J. Chem. Soc. 1922, 121,
416. (c) Lapworth, A. Chem. Ind. 1924, 43, 1294. (d) Lapworth, A. ibid. 1925, 44, 397. For an excellent review of
Arthur Lapworth's contributions to chemistry see: Saltzman, M. J. Chem. Ed. 1972, 49, 750-753.
7) (a) Vorländer, D. Chem. Ber. 1919, 52B, 263. (b) Stieglitz, J. J. Am. Chem. Soc. 1922, 44, 1293.
8) See reference 3c for an alternate classification scheme for functional groups.
9) For an analysis of the relative importance of field and resonance components of substitutent effects see: Swain, C. G.;
Lupton, Jr., E. C. J. Am. Chem. Soc. 1968, 90, 4328.
10) March, J. Advanced Organic Chemistry, 4th ed.; Wiley-Interscience: New York, 1992; pp 507-512.
Functional Group Classification page 4
E & G-Functions. From the preceding discussion, one might opt for
the creation of four classes of functional groups; however, for the sake of ypothetical E-function
simplicity, three FG class designations will be chosen. To organize activating (+) (–) (+)
functions into common categories it is worthwhile to define "hypothetical" C C C E
11
functional groups E, and G , having the charge affinity patterns denoted in 6 6
and 7 respectively. Given the appropriate oxidation state of the carbon skeleton,
such functional groups confer the indicated potential site reactivity patterns ypothetical G-function
towards both electrophilic and nucleophilic reagents. Any functional groups (–) (+) (–)
whose reactivity pattern conforms to the ideal pattern or to a sub-pattern of these C C C G
hypothetical functions will be thus classified as an E- or G-function respectively. 7
For example, the halogen and oxygen-based functional groups in four molecules
illustrated in Scheme III may be classified as E-functions since their respective charge affinity patterns
conform to a subset of the charge affinity pattern of the hypothetical E-function.

A-Functions. A third hypothetical function, A, (A for (+) (+)


amphoteric!) can be defined which has an unbiased charge ypothetical A-function C C A 9
affinity pattern as in 8. Such an idealized functional group (+–) (+–) (+–) (–) (–)
activates all sites to both nucleophilic and electrophilic reactions C C C A C C A 10
and, as such, include those functions classified as either E or G . 8 (+–)
The importance of introducing this third class designation is that C A 11
it includes those functional groups having non-alternate charge
affinity patterns as in 9, 10 and 11.
The differentiation of polar reactivity patterns can be described in an alternative manner. Starting
with an ideal A-function, one could imagine a process in which the reactivity pattern is gradually polarized
towards E- or G-behavior (Scheme V). Since site reactivity is not an on-off property but varies continu-
ously over a wide range, one could further subdivide A-class functions into those functions with a bias
towards E-class or G-class properties. Such a bias could be denoted by the dominant subordinate charge
affinity notation in 12 and 13; however, for the concepts to be presented in this discourse, such A-func-
tion subclasses are nonessential. It should be emphasized that the purpose of the E- and G-classification is
not to rigidly pigeon-hole functional groups based on site reactivity, but only to separate those which are
strongly polarized toward E or G behavior. The decision has been made to avoid the pursuit of an overly
detailed FG classification scheme since such attempts will dangerously oversimplify problems since an es-
sentially contiguous function cannot be segmented in to discrete parts.
Scheme V Alternate vs Nonalternate Reactivity Patterns

Hypothetical A-function
(+–) (+–) (+–)
C C C A
(±) (±) (±) (±) (±) (±)
C C C A C C C A
12 13

(–) (+) (–) (+) (–) (+)


C C C G C C C E

Hypothetical E-function Hypothetical G-function

11) The symbol E was selected to denote electrophilic at the point of attachment to the carbon skeleton Unfortunately, the
symbol N cannot be used to represent those FGs which are nucleophilic at the point of attachment since this is also the
symbol for nitrogen. To avoid this conflict, the symbol G was chosen for this FG class designation.
Functional Group Classification page 5
FG Classification Rules. In the proposed classification scheme the following rules are
followed in the assignment of class designations to functional groups.
■ Activating functions are to be considered as heteroatoms appended to or included within the
carbon skeleton.
■ Activating functions are inspected and classified according to their observed polar site reactivi-
ties.
■ Since both proton removals and addition processes are frequently an integral component in
functional group activation, the function, its conjugate acid or base, and its possible proton
tautomers are considered together in determining its class designation.
■ The oxidation state of the FG is de-emphasized since this is a subordinate strategic considera-
tion.
E-Functions. For example, carbonyls and carbonyl derivatives will be represented as =X where
X may be either oxygen or substituted nitrogen. Well recognized exceptions to the polar class designa-
tions illustrated in Scheme I may be found in the chemistry of CO and HCN. In these instances the carbon
bearing the heteroatom exhibits well-defined nucleophilic properties. Accordingly these two functional
groups will be classified as A-functions by inspection (vide infra).
Table I. Common E-Functions: Symbol:(+)C E

OR O exception: O
NR2 NR exception: N

X, X = halogen
Also consider all combinations of of above FGs; e.g =O + OR
G-Functions. Typical G-class functions are the Group I-IV metals whose reactivity pattern,
falls into a subset of 7.
(–) (–) (–) (+) (–)
2C CH CH2 Li C C C G
(–) 7
CH3 CH2 MgBr

A-Functions. A-functions are usually more structurally complex FGs composed of polyatomic
assemblages of nitrogen, oxygen and their heavier Group V and VI relatives (P, As, S, Se). Typical A-
functions, classified by inspection, are provided in Table II.
Table II. Common A-Functions: Symbol:(±)C A
NO 2 NOR NNR2 N(O)R N2 N
+
SR S(O)R SO2R SR2
+
PR2 P(O)R2 PR 3

Functional groups possessing the following general structure, =N-X where X is a hetroatom
bearing a nonbonding electron pair, have an expanded set of resonance options which create either an
electrophilic or nucleophilic potential at the point of attachment. Remarkably, the dual electronic properties
of oximes were first discussed by Lapworth12 in 1924 before the modern concepts of valence bond reso-
nance was developed.
■ These FG's are capable of conferring both (+) and (–) at the point of attachment.
(+)
X: X: X:
(–) N N N
X = OR, NR 2
R (+) H R H R (–) H

12) Lapworth, A. Chemistry and Industry 1924, 43, 1294-1295.


Functional Group Classification page 6
A Case Study: The Nitro Group. As an example, the class designation of the nitro function
is determined by an evaluation of the parent function, its nitronic acid tautomer, as well as conjugate acid
and base 14 and 15.
H-tautomer conjugate base conjugate base

O HO
HO –O
+N CH2R +N CHR +N CHR +N CHR
–O –O –O HO

nitronic acid nitronate anion, 14 15

From the collection of transformations of the nitro group one finds that the dominate mode of reac-
tivity of the nitronate anion 14 is that of a G-function while the protonated nitronic acid 15 mirrors the re-
activity of an E-function.
–O (–)
14 + N CHR FG C
–O
The dominate polar site reactivity
HO
(+)
15 +N CHR FG C
HO

The typical behavior of nitronate anions 14 is summarized in the representative transformations


provided in Scheme VI. These moderately nucleophilic species, although they are not readily alkylated,
readily undergo aldol and conjugate addition reactions.
cheme VI Selected Reactions of the Nitronate Anion –O
+N CH–R
The Reaction:
–O
O O R
base El(+)
+N CH2–R +N
–O –O El
pKa ~ 10 O –
●●
+N CH–R
O
(–) –O
The charge affinity pattern: + N CH2 –R
–O

■ This reactivity pattern may be extended via conjugation:

It is no surprise that the charge affinity pattern of this FG may be extended by conjugation, and
α,β-unsaturated nitro compounds readily participate in conjugate addition reactions (Scheme VII).
Scheme VII Selected Reactions of the Nitronate Anion

O –O O (–) (+)
+ Nu(–)
The Reaction: +N CH CH R +N CH CH R +N CH2 CH Nu
–O –O –O R
(+)
O –O
(–) (+)
The charge affinity pattern: +N CH CH R Nitro aromatics: +N (+)
–O O
(+)
(+)
X: X: X:
■ The resonance feature (–) N N ✔✔ N
which has been exploited: X = OR, NR2
R (+) H R H R (–) H
Functional Group Classification page 7
The non-alternate behavior of the nitro functional group is dramatically illustrated in the transfor-
mations provided in Scheme VIII. In both instances the derived anions 16 and 17 are highly nucle-
ophilic.13 The non-alternate charge affinity patterns of these nucleophiles is provided.
Scheme VIII Deprotonated Nitronate Anions
–O CH3 –O CH3
LDA (–)
+N C +N C FG C (8)
-78 °C (–)
–O H –O Li
16
–O CH3
–O CH2Li
(–) (–)
+N n-BuLi +N C (9)
C FG C C
CH3
-78 °C –O CH3
–O
17
The nitro group also exhibits the potential of undergoing direct displacement under specific condi-
tions, a general transformation characteristic of E-functions. A recent review by Tamura provides numer-
ous literature precedents for this general class of reactions.14 while table III provides some of the cited re-
actions. Although the NO2 group cannot be considered as a general leaving group, there are a number of
conditions under which this moiety can be exploited, particularly when it is either allylic or tertiary.
O R R
Nu(–) (+)
+ N CH + NO2– (10)
Nu CH FG C
–O R R

Table III. Representative Substitution Reactions of the Nitro Group (eq 10).
NO2 N(CH2)5 Ph SnCl4
74%
Pd(PPh3)3
NO2 Ph
Me
N Anisole
H Me NO2 t-Bu OMe
SnCl4
CH(CO2Me)2 Me 94%
Pd(PPh3)3 NO 2
NaCH(CO2 Me)2 SiMe3
Me Me
Me Me
TiCl 4
SO2Ph SPh 65% SPh
NO2 CN
Pd(PPh3)3 Me Me 3 SiCN Me
NaO2SPh Ph Ph
TiCl4
SPh 73% SPh

A particularly useful transformation of the nitro group is the Nef Reaction, a process which trans-
forms NO2 into =O (Scheme IX). A recent comprehensive review of this transformation provides a de-
tailed discussion of this process.15 In addition to the Pinnick review, Seebach has also written a compre-
hensive review of the diverse chemistry of the nitro functional group.16

13) (a) Henning, R.; Lehr, F.; Seebach, D. Helv. Chim. Acta 1976, 59, 2213-2217; (b) Seebach, D.; Henning, R.; Lehr,
F.; Gonnermann J. Tetrahedron Lett. 1977, 1161-1164.
14) Tamura, R.; Kamimura, A.; Ono, N. Synthesis 1991, 423-434.
15) Pinnick, H. W.; Org. Reactions 1990, 38, 655-792.
16) Seebach, D.; Colvin, E. W.; Lehr, F.; Weller, T.; Chimia 1979, 33, 1-18.
Functional Group Classification page 8
Scheme IX The Nef Reaction
O R R
1) HO –
■ Overall Transformation: +N H O
–O R R

■ Mechanism O R – –O R + HO R
HO H
+N H +N +N
–O R G-Property
–O R –O R
nitronate anion nitronic acid

H+

HO R HO R
H 2O HO R
N H +
O N +N
R -H +
HO H R HO HO R
OH
E-Property

The Diazo Functional Group. This functional group provides one of the best illustrations of
an A-function. As illustrated in Scheme X, both (–) and (+) polar site reactivity is observed in is reactions
with carboxylic acids.
Scheme IX The Nef Reaction
O R R
1) HO –
■ Overall Transformation: +N H O
–O R R

■ Mechanism O R – –O R + HO R
HO H
+N H +N +N
–O R G-Property
–O R –O R
nitronate anion nitronic acid

H+

HO R HO R
H2 O HO R
N H +
O N +N
R -H +
HO H R HO HO R
OH
E-Property

The same overall reactivity pattern is expressed by the diazo functional group in the Tiffeneau-
Demjanov ring expansion reaction17 wherein diazomethane functions as the nucleophilic agent in the first
step and the functional group is lost as a leaving group in the subsequent step (Scheme XI).
Scheme XI The Tiffeneau-Demyanov Ring Expansion
O + O
HO CH2 –N2
CH 2 N2 -N2
EtOH

N2 C R N2 C R
(–) (+)

Restriction: Starting ketone must be more reactive than product ketone

17) For a monograph on ring expansion reactions see: Hesse, M. Ring Enlargement in Organic Chemistry; VCH: New
York, 1991.
Functional Group Classification page 9

Sulfur-Based Functional Groups


Sulfonium Salts. The dual electronic behavior of sulfur functions may be illustrated in the reac-
tions of sulfur ylids which are excellent examples of A-functions. As illustrated in Scheme XII, sulfonium
salts are effective in carbanion stabilization, a characteristic of G-functions, and sulfonium salts are effec-
tive leaving groups, a characteristic of E-functions.
Scheme XII. Sulfonium Salts: Modes of Reactivity
R +
R
+ ●●

■ Carbanion Stablization: S CH3 S CH2 + H
+

R R
(–) pKa (DMSO) ~ 18
R 2S C

R + S N2 R
■ Leaving Group Potential: Good S CH3 + Nu: S + Me Nu

●●
R R
(+)
R 2S C

The non-alternate reactivity pattern of trimethylsulfonium ylids is revealed in the cyclopropanation


of unsaturated ketones as illustrated in the case below (Scheme XIII).18
Scheme XIII. Reactions of Sulfonium Ylids: Conjugate Addition
Me – (+) O
+ ●●
Me + O–
S CH2 S
Me
Me
(+)
R 2S C
(–)
R 2S C Me + O–
S O
Me

■ Nonalternate reactivity pattern revealed in consecutive reactions

Sulfones. Other types of sulfur-derived functional groups exhibit reactivity profiles similar to
that exhibited by sulfonium salts. A number of excellent applications of the arylsulfonyl functional group
illustrate this point. Two applications utilizing the sulfone functional groups are presented below.
The phenylsufonyl moiety strongly stabilizes carbanions and may be equated with the –CN FG in
its potential for hydrocarbon acidification.19 In addition, this FG is a respectable leaving group in selected
situations. In comparisons with sulfonium ions (Scheme XV), arylsulfonyl-stabilized carbanions are more
nucleophilic than sulfonium ylids (G-property), while ArSO2- is a poorer leaving group than Me2S- (E-
Property).
Scheme XV. Sulfones: Modes of Reactivity
Me Me Li
Ph BuLi R + ●●

Ph
Me S Me S more nucleophilic S CH2
O O O than: R
O
pKa ~ 25 Me
R +
Ph poorer leaving S CH3
Me S
group than: R
O O

18) Corey, E. J.; Chaykovsky, M. J. Am. Chem. Soc. 1965, 87, 1352-1364.
19) For an excellent compilation of pKa data for organic functional groups in DMSO see: Bordwell, F. G. Acc. Chem. Res.
1988, 21, 456-463.
Functional Group Classification page 10
Julia's use of phenylsulfonyl carbanions in the synthesis of trans-chrysanthemic acid provides the
justification for defining this functional group as an A-function (Scheme XVI).20
Scheme XVI. The Julia Chrysanthemic Acid Synthesis

Me O
Me SO2Ph
trans chrysanthemic acid
Me Li
Me (+) OEt
Ph Me Me H CO2Et
Me S
O OEt
O Me H
OLi
(–) (+)
R2SO2 C R2SO 2 C

The dual electronic properties of the sulfone functional group are illustrated in the Julia synthesis of
vitamin A (Scheme XVII).21 In this application, the E-property of the FG is exploited in the base-induced
elimination reaction to generate the fully conjugated polyene.
Scheme XVII. The Julia Vitamin A Synthesis
Me SO2Ph SO2 Ph
Me Me Me Me Me Me
CO2R
(–) Li Me
CO 2R
Me (–) Me
R2SO2 C KOH/MeOH
Br

Me Me Me Me
- PhSO 2 –
CO 2R SO2Ph
Me O–

Me (+) R (+) OR
R2SO2 C

Julia & Co-workers, Bull. Soc. Chim. Fr. 1985, 130

For additional reading on the utility of the utility of sulfones in organic synthesis a monograph on
this subject has recently appeared.22 Several other reviews providing extensive literature coverage are
worth reading.23
Organoboranes. The boron atom exhibits many of the common reactions normally attributed to
metals, and when bound to carbon, serves as an excellent source of nucleophilic carbon.24 The transfor-
mations provided in (Table IV) represent but a few cases which demonstrate the G-properties of this acti-
vating function.25,26,27,28,29,30,31

20) (a) Julia, M.; Guy-Rouault, Bull. Soc. Chim. Fr. 1967, 1141. (b) Campbell, R. V. M.; Crombie, L.; Findley, D. A.
R.; King, R. W.; Pattenden, G.; Whiting, J. J. Chem. Soc., Perkin Trans. I 1975, 897.
21) Arnould, D.; Chabardes, P. Farge, G.; Julia, M. Bull. Soc. Chim. Fr. 1985, 130.
22) Simpkins, N. S. Sulfones in Organic Synthesis, Pergamon Press, New York 1993.
23) (a) Trost, B. M. Bull. Chem. Soc. Jpn. 1988, 61, 107-124. (b) Magnus, P. D. Tetrahedron, 1977, 33, 2019-2045.
24) (a) Brown, H. C. Boranes in Organic Chemistry, Cornell University Press, New York 1973. (b) Cragg, G. M. L.
Organoboranes in Organic Synthesis, Marcel Dekker, New York, 1973
25) (a) Kow, R.; Rathke, M. J. Am. Chem. Soc. 1973, 95, 2715. (b) Zweifel, G.; Fisher, R. P.; Horng, A. Synthesis
1973, 37. (c) Matteson, D. S. ibid. 1975, 147.
26) Negishi, E.; Abramovitch, A.; Merrill, R. E. J. Chem. Soc., Chem. Commun. 1975, 138.
27 For a recent citation on allylboron-based nucleophiles see: Wang, Z.; meng, X. J.; Kablaka, G. W. Tetrahedron Lett.
1991, 32, 5677-5680 and references cited therein.
28) Marshall, J. A. Synthesis 1971, 229.
29) (a) Brown, H. C.; Rhodes, S. P. J. Am. Chem. Soc. 1969, 2149, 2149. (b) Hawthorne, M. F.; Dupont, J. A. J. A m .
Chem. Soc. 1958, 80, 5830.
30) (a) Pelter, A.; Subrahmanyan, C.; Laub, R. J.; Gould, K. J.; Harrison, C. R. Tetrahedron Lett. 1975, 1633. (b) Pelter,
A.; Harrison, C. R.; Kirkpatrick, D. ibid. 1973, 4491. (c) Pelter, A.; Harrison, C. R. J. Chem. Soc., Chem. Comm.
1974, 828. (d) Naruse, M.; Utimoto, K.; Nozaki, H. Tetrahedron 1974, 30, 3037.
Functional Group Classification page 11
Table IV. Reactivity Patterns for Organoboranes
Entry Reaction Charge Affinity Pattern

25 R R R
A base – El(+) (–)
B R B CH2 B CH2 R C A
R R R

R O R
26 O (–)
B – +
R B Ph + Cl C Me B R Ph C Me C A
R
R
OBR2
O
(–) (–)
27 R 2B +
C H C R R C C C A

OTs OTs
Me Me Me
28 (–)
D HO– C A

B(OH)2 – B(OH)3

Ph Ph
Ph
29 (–)
E R 2B Cl
HO– C A

(HO)3B Cl

El δ+
R R R El
30 (–) (+–)
F El(+)
R B R' R C C A
– B
R δ– R' R2B R'
R

El(+) = n-C 6H13I, C3H5Br, ethylene oxide, CH2I2, MeSO3 H


OH
R O
31 R (–) (+–)
G 1) addition
R B CH CH2 C C A
– 2) H 2O2 OH
R

stereochemical aspects of this reaction not determined

The potential for non-alternate charge affinity patterns for boron have been revealed in the reactions
of acetylenic and vinylic boron ate complexes (Table IV, entries F, G).30,31 These compounds exhibit
high nucleophilicity towards a variety of electrophiles β to the boron atom. The origin of such β-nu-
cleophilicity could be a consequence of σ−π conjugation32 (e.g., 19) not observed with the heavier metal-
lic elements which are attacked by electrophiles α to the metal where the alternate mode of conjugation 1 8
is possible.33 In principle, both types of conjugative stabilization are possible with a range of
organometaloids; however, in practice this is not the case. It would be expected that the effects of
31) (a) Utimoto, K.; Uchida, K.; Nozaki, H. Tetrahedron 1973, 30, 4527. (b) Utimoto, K.; Uchida, K.; Nozaki, H. Chem
Lett. 1974, 1493.
32) (a) Harmon, G. D.; Traylor, T. G. Tetrahedron Lett. 1975, 939, and reference cited therein. (b) for example of σ−π
delocalization of type 25 involving R3B— see Hanstein W.; Traylor, T. G. ibid. 1967, 4451; (c) for the reaction of
vinylsilanes electrophiles see Miller, R. B.; Reichenbach, T. ibid. 1974, 543, and references cited therein.
33) Kitching, W. in "Organometallic Reactions," Vol. 3, E. I. Becker and M. Tsutsui, Ed., Wiley-Interscience, New York
1972, pp. 319-398.
Functional Group Classification page 12
σ−π conjugation, such as that illustrated in 24, would be more important in those systems having shorter
C-M bonds, a situation which may be unique to boron. It is noteworthy that the other group III and IV
organometallic compounds, R3M—CH=CH2 (M = Al, Si, Ge, Sn) react with electrophilic reagents α to
the metal. These elements all exhibit polar reactivity patterns common to G-class functions.
R R
M +
H +
α-attack R M H β-attack R M H
H C C C C (11)
H R C C
H α β H R
H H
El
El
El(+)
18 19
Metals. In deriving a class designation for metals, M, bound to carbon, two reaction types are
considered. Metals undergoing exclusive substitution at the metal-carbon bond by electrophiles, El+, are
classified as G-functions (eq 12), while metals which are involved in redox processes (eq 13) are classi-
fied as A-functions since such organometallic compounds also exhibit G-type behavior.
(–) (+)
R M + El(+) R El + M(+) (12)
(+) (–)
R M + Nu(–) R Nu + M(–) (13)

The organic chemistry of Tl(III),34and TlX2


2 KI
I
Pd(II)35 (eq 14-16) illustrate the role of metals as (14)
- TlX
leaving groups (reductive elimination). Oxidative
addition reactions of metal carbonyl anions and alkyl OMe
OMe
halides provide examples of the reverse process.36 MeOH C A (15)
In general, transition metal-mediated cross-coupling TlX2
- TlX OMe
reactions provide a useful illustration of the A- PdX
classification of redox metals (eq 17).37 The NaCH(CO2 Me)2 CH(CO 2 Me)2
(16)
assignment of charge affinity labels to R1 and R2 in - Pd(0)
this case is arbitrary.
Employing polar processes as the basis set of
synthetic reactions, existing functional groups may R (+) reductive
be organized according to their known chemical L M elimination (17)
n R R + LnM
properties. Any number of positions may be taken
R (–)
relative to the classification of atom reactivity. The M = Pd, Fe, Cu, Ni
goal of this section has been to define a general
classification scheme which may be used to organize the multitude of different strategies which have been
developed to construct pairwise functional group relationships in organic molecules.

34) Taylor, E. C.; McKillop, A. Acc. Chem. Res. 1970, 3, 338.


35) Trost, B. M. Acc. Chem. Res. 1980, 13, 385-393.
36) Ellis, J. E. J. Organomet. Chem. 1975, 86, 1.
37) (a) Neuman, S. M.; Kochi, J. K. J. Org. Chem. 1975, 40, 599. (b) Normant, J. F. Synthesis 1972, 63. (c) Tamao,
K.; Kiso, Y.; Sumitani, K.; Kumada, M. J. Am. Chem. Soc. 1972, 94, 9268. (d) Tamaki, A.; Magennis, S. A.;
Kochi, J. K. ibid. 1973, 95, 6487.
Functional Group Classification page 13

Classification of Difunctional Relationships.


One of the basic assumptions employed in synthetic design involves the maximum utilization of
existing functionality at all intermediate points in the construction of a polyfunctional molecule. Such
guidelines aid in minimizing the number of side reactions and protection-deprotection steps during the
assemblage operation. In the synthesis of even simple difunctional organic molecules, the relative
positioning of the two activating functions on the carbon framework strongly influences the reaction types
that will usually be employed to establish the difunctional relationship. Using the general notation devel-
oped in the previous section for activating functions, two distinct classes of difunctional relationships
which may be defined between ideal E- and G-functions which may be defined are illustrated in Table V.

Paths. Difunctional relationships between heteroatoms having "matched" charge affinity patterns
will be defined as consonant while unmatched relationships will be labeled dissonant.
It should be pointed out that the charge affinity notation is unnecessary to define the appropriate
relation; other parity labels could serve equally well. For example, the number of bonds between E- and
G-functions could be used to define the appropriate relationship. Employing E-functions for the purpose
of illustration, the two carbonyl groups in 20a have a matched charge affinity pattern along the potential
construction path. Since they are separated by three atoms they can be defined as 1,3-consonant (1,3-C).
The symbol notation 20b transmits information relative to the E—E' positioning along the construction
path and since the E-symbol represents a homogeneous class of electronically equivalent functional
groups, a common symbol is employed. In those cases where it is necessary to recognize oxidation states
of carbon to derive a symbolic structural notation, one may easily do so.
Table V. Consonant & Dissonant Difunctional Pairwise Functional Group Relationships
Consonant Relationships Symbol Notation Dissonant Relationships Symbol Notation

O O G
Li CH2 Cl C E 1,1–D
Me OMe
O E2
O E1 E2
1,3–C OR 1,2–D
Me NMe2 NH 2 E1

Cl OMe O E1
1,4–D
Me OMe Me CO2 R Me
E2

O O E1 E2

H OMe 1,5–C E
O
OTHP E

1,4–C O D-cycles
Li G O E
O E

O E

O C-cycles

O E2
1,2–D, 1,4–D
NH E
O E1

(+)
O O (–)E 1 E 2 (–)

O (+) (+) (-) symbol deleted for


Me OMe (+) (–) (+) E2 convenience

20a 20b (+) 21b


O 21a
E1
Functional Group Classification page 14
Cycles. In cyclic structures, a heteroatom attached to or contained within the cycle creates a
relationship with itself. For non-arbitrary mathematical considerations it is convenient to define an even-
membered ring with or without a single functional group as consonant and corresponding odd-membered
rings as dissonant. For the bicyclic ketone 21a, both of the oxygen heteroatoms, denoted as E1 and E2,
establish consonant relationships with each other via all bond paths and individually by virtue of their
position either attached to or contained within an even-membered ring.
Consonant and Dissonant Bond Paths. In contrast to the uniformity with which consonant
relationships may be established through common classes of polar processes, the synthetic methods and
functional groups required for the construction of the bonds define a D-relationship are quite varied and
involve either more steps, more functional groups or more reactive intermediates than reactions leading to
C-paths. This statement will be reinforced in a series of case studies (vide infra); however a single case is
presented to reinforce this assertion. Consider the Michael transform executed on the 1,5- and 1,4-dike-
tones shown below (eq 18, 19). In the first instance, the transform may be executed using only the func-
tional groups illustrated; however, this is not possible with the dissonant dicarbonyl relationship since one
of the resulting polar fragments will be electronically mismatched with its associated FG. In the illustrated
disconnection (eq 19), the electronically mismatched fragment is the carbonyl anion.
O O O O
(+) Michael
,5-Consonant (+) (18)
Relationship R 1 (+) (–) (+) R 2 R 1 (+) (+) R2
(–) (–) (–)


O O
(+) Michael R1
1,4-Dissonant R 1 (–) (–) (+) (19)
Relationship (+) R 2 (+) R 2
(–) (–)
O
O
Acyl anion
Nef

O✻ O
(±) (+) Michael R1
R1 (±) (+) (20)
(+) R 2 (+) R2
(–) (–)
NO2
NO 2
Acyl anion
equivalent

One possible solution to the construction of this dissonant relationship is through FG manipula-
tion. In the present instance the application of the Nef transform (vide supra) provides the opportunity to
match the charge affinity patterns so that the Michael transform may be properly executed. The use of A-
functions in this fashion is just one of a number of strategies which may be employed to construct disso-
nant difunctional relationships.
In conclusion, dissonant pairwise relationships, either identified in simple acyclic molecules or
within complex cyclic structures, generally pose a greater synthetic challenge and represent seams of lower
flexibility within the carbon framework. At this point, it may be instructive to the reader to contemplate a
synthesis strategy based on how and when D-relationships are incorporated into target structures. This
point will be addressed later in the discussion.
Synthesis of Consonant Difunctional Relationships.
Every complex polyfunctional molecule may be analyzed structurally CH2OH (+)
E2
in terms of its individual consonant or dissonant construction paths or
(+)
cycles. For example, in the alkaloid lupinine (22) all possible construction (+) (+)
paths interconnecting E1 and E2 are consonant. On the other hand, N
E1
mesembrine (23) 38 contains the potential dissonant paths and cycles (+) (+)
illustrated in heavy lines. Consonant paths within the polyatomic 22 (lupinine)
framework define seams in the structure that may be constructed using aldol and related condensation
processes.

38) (a) Curphey, T. J.; Kim, H. L. Tetrahedron Lett. 1968, 1441. (b) Keely, S. L.; Tahk, F. C. J. Am. Chem. Soc. 1968,
90, 5584. (c) Stevens, R. V.; Wentland, M. P. ibid. 1968, 90, 5580; (d) Shamma, M.; Rodrigues, H. R. Tetrahedron
1968, 24, 6583.
Functional Group Classification page 15
■ Regarding the number of different possibilities available for the synthesis of a consonant di-
functional relationship interconnected by n bonds, there exists a set of n different connective operations
that may be employed to establish any bond along the construction path from monofunctional or consonant
polyfunctional precursors.39
OMe Ar Ar

OMe
Ar
E2 E2
E1 E1

dissonant bond paths (cycles)


O N
Ar Ar
N Me
O 23
Me Shortest consonant
23 (lupinine) E2 E2 bond path
E1 E1

In the analysis of potential routes to structures like lupinine, identify the shortest consonant bond
path and then proceed to carry out all polar disconnections along that bond path (Scheme XVIII). Since
there four bonds interconnecting =O and N (E1 and E2), there will be four associated transforms which
one may execute using the illustrated functional groups. In each set of precursors the intrinsic polar reac-
tivity patterns of the heteroatoms are accommodated in the coupling process. The resulting adducts con-
taining the requisite nitrogen-oxygen relationship may then be ranked in order of desirability by consider-
ing criteria such as chemical feasibility of the coupling step, ease of subsequent transformation to the target
structure, and availability of precursor fragments. In the present example, transforms A and B might be
more highly ranked that transform C while transform D might be discarded since it does not lead to struc-
tural simplification.
Ar
Scheme XVIII Ar
A
equivalent to:
E1 (+) (+) E2 O HN
(–) (–)
Me
Ar Ar
B
Ar equivalent to:
E N+
E 1 (+) (–) (+) 2(–) HO
Me
(–) E1 (+) (+) E2 Ar
(–) (–) Ar
Shortest consonant C
bond path equivalent to:
E RO2C
E 1 (+) (–) (+) 2(–) N
Me

Ar Ar
D
equivalent to:
(+) (+) E2 HO– N+
(–) E1 (–) (–)
Me

In those cases when a given consonant or dissonant relationship is (+)


separated by a significant number of bonds, it is strategically worthwhile to 6 4

consider the option of incorporating additional functions to aid in the 2


construction of the desired target molecule. The relative placement of such a O R E1 (+) (+) R
24
functional group is of prime importance in dictating the subsequent polar
disconnections that are perceived in generating a plausible synthetic tree. This point is illustrated when
considering plausible precursors to ketone 24 (Scheme XIX). In this structure, the =O FG establishes a
1,5-relationship with itself on the six-membered ring. Through the addition of an appropriate second

39) The presence of a quaternary or bridgehead center along the construction path limits bond construction to those adjacent to
the center.
Functional Group Classification page 16
activating function to the target molecule 24, an expanded set of potential disconnections is created. In the
placement of the second FG, the charge affinity pattern of the resident FG should be used. For example,
consider the installation of a second FG, E2, at the (+) sites on the ring to set up aldol or Claisen
transforms. In a complementary fashion the addition of C-E2 fragments to the (–) sites will open up the
execution of the two possible Dieckmann transforms.40 The preceding analysis leads to the three
precursors 26a-26c. Each of which contains a 1,5-consonant difunctional relationship between the
carbonyl functions. These subgoals now become the focus of the next level of analysis wherein the
preceding logic is again applied. It should be emphasized that the precursors illustrated in Scheme XIX
are not inclusive but represent one set which leads to the generation of a synthetic tree based upon aldol
and related reactions. The point to be emphasized is that in the first stage of the analysis where
functionality is being added to the target structure, consonant, rather than dissonant relationships should be
created.
Scheme XIX Aldol O
A equivalent to:
(+) E1 R R
O R
E2 OH
25a O Me 26a
E1 (+) (+) R E2 OH O
B
Where (+) add E2 equivalent to: Aldol
Me H

E1 R O R O R
25b
26b
E2
RO 2C
C Dieckmann
equivalent to:
(–) (–) RO 2C
E1 R O R
E1 R 25c
(–) R
Where (-) add C-E2 D Dieckmann 26c CO 2R
equivalent to:
E1 R
O R

E2 25d CO2R

40) To be completely rigorous with regard to this analysis, the addition of C-E2 to the 4-position should also be considered;
however, the E1-E2 construction span from such a precursor is sufficiently large as to render this precursor less attractive
than the other precursors 25a-25d.
Chemistry 206

Advanced Organic Chemistry

Handout 27B

Synthetic Applications of α-Diazocarbonyl


Compounds

An Evans Group Afternoon Seminar

Krista B. Goodman

January 15, 1999

D. A. Evans Monday,
November 17, 2003

27B-00-Cover page 11/17/03 1:16 PM


Krista Beaver Synthetic Applications of α-Diazocarbonyl Derivatives Chem 206

Synthetic Applications of α-Diazocarbonyl Compounds Synthesis of α-Diazocarbonyl Compounds

An Evans Group Afternoon Seminar • First synthesized by Curtius in 1883 by diazotization of α-amino acids
Krista Beaver
• Arndt-Eistert synthesis (1927)
January 15, 1999
1. (ClCO)2, DMF
Leading References: O O
2. CH2N2
N2
McKervey and Ye, Chem. Rev. 1994 1091 R OH R

Arndt and Eistert, Ber. Dtsch. Chem.


Doyle, McKervey and Ye, Modern Methods for Organic Synthesis with Ges. 1927 60B 1122
Diazo Compounds, Wiley, 1998 Pettit, JOC 1986 1282

• Diazo Transfer

O O
R' RSO2N3, base R'
R R
Diazocarbonyl Compounds: Structure and Nomenclature R=Me, Ts, etc.
N2
Regitz, ACIEE 1967 733
O O O
∆, hv or M acid For temporary activation of carbonyl
R R R
R' R' R' compounds prior to diazo transfer,
see Danheiser, JOC 1990 1960
N N
N N
Diazocarbonyl Diazonium

• Acyl Transfer

O
cyclopropanation
O O
insertion
N N2 R N2
rearrangement O ROH
O
ylide formation displacement O Badet, JOC 1993 1641
rearrangement
solvolysis

27B-01 11/11/01 7:59 PM


Krista Beaver Synthetic Applications of α-Diazocarbonyl Derivatives Chem 206

Some Reactions of α−Diazocarbonyl Compounds


Acid Catalyzed Reactions of Diazo Compounds
O
O
O
R1
R1 R Review: Smith, Tet. 1981 2407
O R R1
X X R
R1 H BR2 O
R H X
R1 O O
O H SiR3 R acid
CH3 CH3
R1 H OH O H 3C H 3C
R R1 N N
H NR2R3 R
H OR N N
O
O Diazocarbonyl Diazonium
R1
R R1
R
HO OH Common acids include BF3•OEt2, HBF4, TFA, etc.
R2 OR
O O O
R1 R1 R1
R R
Mechanism of activation is unclear for both Lewis and protic acids;
R
CNS SeR
activation may occur by protonation on C or O
N2 H OTs
O
R1 O
R R1
AcO SeR R
O H OCOR2
Acid-Catalyzed Reactions
R1 O O
R
R1 N2
R2 H R
O TFA, -20°C O
O H O2P(OR2)2
R1 (96%)
R O R1
O R
H SeR R1 O O
HO
R R1 H PO(OR)2
R1 R
R2 R3 R Mander, Chem. Comm. 1971 773
H SR Tet., 1991 134
RS SR
"Having become familiar with the peculiarities of diazoketone chemistry while preparing [other
Adapted from McKervey, Chem. Rev. 1994 1090 compounds] (and, I might add, inured to handling uncomfortably large quantites of
diazomethane), it occurred to us that we might be able to substitute a diazo group for bromine."

O Lewis Mander
Cl3COCO OCOCCl3
N2
O
TFA
-25°C, 2 min Gibberrellic Acid
(82%)
OMe O Mander, JACS 1980 6626

27B-02 12/20/99 4:05 PM


Krista Beaver Synthetic Applications of α-Diazocarbonyl Derivatives Chem 206

More Acid Catalysis β-Ketoester synthesis:


Olefins as nucleophiles: O
O O
O SnCl2
O Me H
Me +
HCl EtO EtO R'
R H (50 - 90%)
O
Cl N2
(100%)
N2 Me
Me Roskamp, JOC 1989 3258 Yields are good when R is aliphatic;
moderate when aromatic
Mander
Smith's cyclopentenone annulation: O
O Ring expansion:
N2 BF3•OEt2 Me Me CO2Et
(40 - 65%) Me
Me
Me O BF3•OEt2
Me
EtO2CCHN2 O
Lindlar's cat. (100%) Me O Me Me
(81%) Me O
O

Me
Smith, TL 1975 4225
Me
Jasmone Me Br
Ghosh, Chem. Comm. 1988 1421
Aplysin
Me
O Me
Me
Rearrangement:
O Substitution:
N2
O
O H O O
O O BF3•OEt2 N2 H O RO
N2 S
S Me
Me ROH, BF3•OEt2
OBF3 ">60%" N Me
R N Me
R O
R O O CO2CH2CCl3
CO2CH2CCl3

Mander, Aust. J. Chem. 1979 1975


Thiols also work well John and ThomasTL 1978 995
Polyene cyclizations:

O Me O Me O Tetrahydrofuran Synthesis:
R R OH
BF3•OEt2 + BF3•OEt2
N2 TESO O
Me Me BnO2CCHN2 CO2Et
O
H (53 - 87%)
diastereoselection 3:1 - 20:1
46% 12%
Smith, JACS 1981 2009 Diastereoselectivity increases with size of R; Angle, TL 1998 3119
independent of Lewis acid or protecting group TL 1998 8195
27B-03 12/20/99 4:14 PM
Krista Beaver Synthetic Applications of α-Diazocarbonyl Derivatives Chem 206

Substitution Reactions Base-Induced Reactions


Deamination:

O O Aldol-type reactions:
H 2N
H O Br H O O H O
S S S O OH
Me NaNO2, Br2 Br Mg Me O
Me O
N Me (90%) N N Me Li R1
Me R + R R2
O O O
COOH COOH R1 R2 N2
COOH N2
Kapur and Fasel, TL 1985 3875 LDA is the optimal base for lithiation Pellicciari, JCS Perkins I 1985 493

Synthesis of α-substituted chiral acids:

OH
Me COOH O
Ester alkylation:
Me COOH
Me H
O O O O O
H2N H Nu H Li CO2Et
N2 H Me Rh2(OAc)4
+ CO2Et
HO CO2Et
-
Nu = Br, Cl, F O N2 100%
Nu N2 O
Displacement occurs with Ingold, Nature 1950 179
retention of stereochemistry Rapoport, JOC 1985 5223
For other examples, see McKervey,
Chem. Rev. 1994 1091
Gilbert-Seyferth Reagent:
Reaction with Boranes (MeO)2OP H O
KOt-Bu
+ R1 R2
N2 R1 R2
D H Seyferth, JOC 1971 1379
BH3 EtO2CCHN2, then D2O Gilbert, JOC 1982 1837
B CO2Et
(97%, 100% d1)
3

Hooz, JACS 1969 6195


N2
O
OBBu2 O Mechanism? O
N2
Bu3B Bu n-BuLi, then MeI Bu
Me
(61%) CF3COOH O

MeO O
Wojtkowski, JOC 1971 1790

27B-04 12/20/99 4:24 PM


Krista Beaver Synthetic Applications of α-Diazocarbonyl Derivatives Chem 206

Carbenoid Reactions: The Catalysts Ligand Effects: Selectivity


Review: Padwa, ACIEE 1994 1797 Methine versus methyl:
O
O
Decomposition can be catalyzed by: O O H3COC
H3COC O
Rh(II) O
Heat or light +
H 3C O
Transition metals, including CuII, RhII, MnII, FeII, CoII, Ni0, NiII, ZnII, MoII, RuII, RuIII, PdIII N2

Most common catalysts: Doyle, JACS 1993 958 Rh2(OAc)4 90:10


Copper (I): CuOTf, Cu(OTf)2, CuSO4, CuX, Cu(acac)2 Rh2(pfb)4 38:61
Cyclopropanation versus C-H Insertion:
Rh2(acam)4 100:0
Rhodium (II): Much milder catalyst than Cu (introduced in 1973 by Tessié)

Structures generally contain bridging ligands and contain a Rh-Rh single bond CH3 Rh(II) H 3C CH3

Reaction pathways are highly sensitive to steric and electronic effects O +


H 3C N2
O H 3C
Me Rh2(OAc)4, Rh2(tfa)4, Rh2(oct)4, O
Rhodium Carboxylates:
Rh2(tpa)4, Rh2(pfb)4 CH3
O
O Me Padwa and Doyle, JACS 1993 8669 Rh2(OAc)4 44:56
L O Rh2(acm)4, Rh2(cap)4,
Rh
Rhodium Carboxamidates: Rh2(pfb)4 0:100
O Rh2(CF3CF2CF2CONH)4
O Rh
O L Rh2(cap)4 100:0
Me O Rhodium Acetate
O More Competition Experiments
Me
Dipolar Cycloaddition versus C-H insertion:
Transition Metal Catalyzed Diazo Decomposition
SCR2 O Ar O
S:
O CH3
H CH3 O
Rh(II)
N2 O +
E E O CH3

E E
B
MLn B L nM LnM CR2
Rh2(OAc)4 75:25
Padwa and Moody, Tet. 1993 5109 Rh2(pfb)4 0:100
Rh2(cap)4 100:0
LnM CR2
R 2C N 2 N2 N2 Conclusions:
These results imply that the metal is involved in the transition state
Doyle, Chem. Rev. 1986 919
Reaction pathways can be controlled by tuning the ligands on the metal
27B-05 12/20/99 4:33 PM
Krista Beaver Synthetic Applications of α-Diazocarbonyl Derivatives Chem 206

Generalizations: Sigma Bond Insertion Tandem O-H Insertion/Claisen Rearrangement


Me

O O O O
X-H OH
R R + Rh2(OAc)4
OEt catalyst OEt Me OMe HO O
Me
N2 X H PhH, ∆, 20 min
N2
(66%) Me OCH3

98% ee O
95% ee
Reviews O-H Insertion: Moody Tet. 1995 10811
C-H Insertion: Sulikowski Tet. Asymm. 1998 3145

• When X is a heteroatom, insertion is facile


O H H
O
[3,3]
• When X is carbon: Me O O
Me
Me fast Me
Only intramolecular processes are generally useful CO2Me CO2Me
5 - membered ring formation is favored in general Z-Enol Transition State
Wood, JACS 1997 9641
Order of selectivity: methine > methylene > methyl Wood, JACS 1999, in press
slow

Me The opposite
O-H Insertion Reactions O O enantiomer is
Me observed!
CO2Me Me OMe
CO2Me COOH PhH, ∆, 18 hrs O O
N2 HO HO
Me O Me
MeO2C CO2Me CO2Me (75%) Me OCH3
CO2Me
OH Rh2(OAc)4 O
O CO2Me O COOH
(75%) 47% ee
OMEM E-Enol Transition State
OMEM OH

Ganem, JACS 1982 6787 Chorismic Acid


Merck Thienamycin Process
O Me OH OH
O TMSO O TMSO CO2Et H H H H
N2 O NO2
Me Me Rh2(oct)4 Me
EtO2C PO(OEt)2 O OH
Me H OH Me H NH O N
PhH, 80 °C
O N2 O
Rh2(OAc)4, then NaH 100%
H H O CO2p-NB
(75%, 2 steps)
AcO AcO OH
H H H H NH3

"the most complex alkoxyphosphonate yet described" Fuchs, TL 1994 7163 S Thienamycin
N
O Salzmann, JACS 1980 6161
CO2

27B-06 12/20/99 4:39 PM


Krista Beaver Synthetic Applications of α-Diazocarbonyl Derivatives Chem 206

Generalizations: Cyclopropanation
C-H Insertion: Reactions
O O
R1 catalyst
R R
+ OEt
H OEt
N2 S H R2
Me Ph S N2 R1 R2
N hv Me
Ph Me N Me
O CO2Me O Reviews: Davies, Ald. Acta. 1997 107
CO2Me Davies, Tet. 1993 5203

Corey, JACS 1965 2518 For subsequent reactions: Calter, Evening Seminar 1992
Me Electron rich olefins work best
O Me
Me Both concerted asynchronous and stepwise mechanisms have been proposed
Rh2(OAc)4 Me O
N2 Cyclopropanes can participate in tandem reactions
H (59%)
H
AcO
H
Me
AcO
H
Cyclopropanation Followed by Rearrangement
Me
OAc
Wenkert, JOC 1982 3243 OAc OEt OEt CO2Me
N2
Rh2(oct)4 Et2AlCl
H (88%)
N2 MeO2C
O O (87%) EtO
CO2Me
O
Rh2(OAc)4
+ Davies, JOC 1992 4309; TL 1992 453
(65%) AcO AcO
AcO
H3CO H3CO
H3CO
Diastereoselection > 99:1
Adams, JACS 1994 3296
Cu(TBS)2
TBSO (84%) H H
Me O TBSO TBSO
O O Me Me
O
BnO O O O O
O O O
+
Rh2(5R-MEPY)4 N2
BnO N2 Et2AlCl
O (80%)
BnOH2C OBn BnOH2C OBn
97% ee 50% ee OH
Doyle, JACS 1994 4507
Diastereoselection 93:7
O TBSO
For a review of catalytic enantioselective carbene reactions, see: HO Me
Doyle, Chem. Rev. 1998 911 Me H CO2Me O O
O Corey, JACS 1985 5574
Antheridic Acid
27B-07 12/20/99 4:46 PM
Krista Beaver Synthetic Applications of α-Diazocarbonyl Derivatives Chem 206

More Cyclopropanation Reaction with Aromatic Rings


• Discovered by Büchner (1893)
Me E
H CO2Me
O hv
N2 CO2Me (79%) E
N2
OEt
Ph CO2Me 90% ee
Rh2(TBSP)4 Me
Ph Büchner, Liebigs Ann. Chem. 1893 214
Ph Me Doering, JACS 1956 5448
Davies, TL 1994 8939
• Initial experiments gave poor selectivity, but transition metals help...
DMB
DMB H3CO
N N
O Rh2(OAc)4 O ∆
pinacolone O + 6 other products
120°C N2
O (35%)
O N2 OEt
+ (62%) OCH3
E
+
N N H3CO
H H OCH3
N N
H H DMB O
Rh (II) +
N2
N (73%)
Wood, JACS 1997 9461 O OEt
E E
Tessié, Chem. Comm. 1980 765
JOC 1981 873
Staurosporine
N N Büchner Reaction: Confertin Synthesis
H H

Me Me
AcO
O Ph
Rh2(mandalate)4
Ph Rh2(S-TBSP)4 Ph
+ OMe H
(79%) AcO N2 O
N2 CO2Me Me
94% ee O
Me
Davies, TL 1993 7243 1. KMnO4, NaIO4
2. Me2SO4, K2CO3,
O acetone McKervey, Chem. Comm. 1988 1028
(97%, two steps) JCS Perkins I 1991 2565
CO2Me Ph
1. 6N HCl, ∆
Li2CuCnAr2 Me Me
2. ClSO3H CO2Me H
CO2Me H O
(84%) (82%) H H
CO2Me O
Cl Confertin
Cl H
Cl Me Me Me
Corey, TL 1994 5373
Cl O OTBS

27B-08 12/20/99 4:59 PM


Krista Beaver Synthetic Applications of α-Diazocarbonyl Derivatives Chem 206

Dipolar Cycloadditions: Carbonyl Ylides


Ylide Formation
TMSO
TMSO TMSO
O O
R 2X Rh2(OAc)4
R R N2 (66%)
OEt catalyst OEt MeO2C R MeO2C O R
O MeO2C O R
N2 X
R R O
O O
Merck, TL 1994 9185 Zaragozic Acid Skeleton
Reviews: Padwa, Chem. Rev. 1991 263
Me Me
Padwa, Chem. Rev. 1996 223 OAc Me
Me Me AcO
Barnes, Evening Seminar, March 16, 1993 H AcO Me
H H
H
O H
X is generally S, O or N and can be sp2 or sp3 hybridized N2 Rh2(OAc)4 H
H (86%) H
Ylides often undergo sigmatropic rearrangements or cycloadditions O
CO2Et O
H
O CO2Et
H H CO2Et
[2,3]-Sigmatropic rearrangement: O O

Ph Dauben, JOC 1993 7635 Tigilane Skeleton


SPh S
O O O Me O
Rh2(OAc)4 E O O
N O MeO2C MeO2C Me
OMe
N
N2 N Me
O N2 OMe O O
(93%)
Rh2(pfb)4 H
Kido and Kato, JCS Perkins 1 1992 229 H
N
SPh Bz
E N
Acorenone B O N
Padwa, JOC 1995 2704 Bz
Bz
Lysergic Acid Skeleton

Stevens Rearrangement ([1,2] alkyl shift):


O O O
O O N CO2Me N
R2 Rh2(OAc)4 Me N
N N2 Et
N2
N N O Et
R1 O N R1
R1 R 2 N O
R2 Rh2(OAc)4 (95%)
O Et Me CO2Me
West, JACS 1993 1177 N O Vindoline Skeleton
Padwa, JOC 1995 6258 Me CO2Me

27B-09 12/20/99 5:14 PM


Krista Beaver Synthetic Applications of α-Diazocarbonyl Derivatives Chem 206

Wolff Rearrangement Catalyst Glossary Ac


N
Cu(TBS)2 Copper t-Butylsalicylaldimine t-Bu Rh2(MACIM)4
O O N O CO2Me
O N
H R1 H
H
R1 R1 OH
N2 H O
Rh2(OAc)4 Rhodium Acetate CH3CO2 Rh2(MEOX)4 O CO2Me
N
H
Arndt-Eistert Homologation:
Rh2(acam)4 or Rhodium Acetamidate CH3CONH
Rh2(acm)4
OMe OMe O
OMe OMe Rh2(MEPY)4 O
O 2N N2 CO2Me
Rh2(cap)4 Rhodium Caprolactamate O M N
+
O 2N H
Ag , H2O COOH
Me M O
Me N
Ph
OMe Rh2(MPPIM)4 N
OMe Evans, JOC 1993 471 Rh2(oct)4 Rhodium Octanoate CH3(CH2)6CO2
O N CO2Me
H
Rh2(pfb)4 Rhodium Perfluorobutyrate CF3CF2CF2CO2

Wolff Rearrangement - [2+2] Cycloaddition Rh2(tfa)4 Rhodium Trifluoroacetate CF3CO2 Rh2(S-TBSP)4 N CO2

TMS TMS Rh2(tfm)4 Ts


Rhodium Trifluoroacetamidate CF3CONH
Me
R
Rh2(tpa)4 Rhodium Triphenylacetate Ph3CCO2
[2+2] Rh2(S-DOSP)4 CO2
N
O O O
Me Me R
TMS
O
N2 O All ligands on Rhodium are bridging
O
O R through the carboxylate or the amide
O H O H SiO2 O
Rh O (CH2)11CH3
(60%) O Rh
Ireland, JACS 1981 2446; JOC 1984 1001 O
R O
O
Me
R
Me R
Aphidicolin
O
O

O H OE

27B-10 12/20/99 5:22 PM


D. A. Evans Ambiphilic Functional Groups–2: Hydrazone-Based Transformations Chem 206

Relevant Background Reading


http://www.courses.fas.harvard.edu/~chem206/
Hutchins, R. O. (1991). "Reduction of C=X to CH2 by Wolff-Kishner and Other
Hydrazone Methods". Comprehensive Organic Synthesis. Trost and Fleming.
Chemistry 206 Oxford, Pergamon Press. 8: 327.

Shapiro, R. H. (1976). “Alkenes from Tosylhydrazones.” Org. React. (N.Y.) 23:


Advanced Organic Chemistry 405.

Addlington, R. M. and A. G. M. Barrett (1983). “Recent Applications of the


Shapiro Reaction.” Acc. Chem. Res. 16: 55.
Lecture Number 28
Chamberlin, and Bloom (1990). “Lithioalkenes from arylsulphonyl-hydrazones.”
Org. React. (N.Y.) 39: 1.
Ambiphilic Functional Groups–2
Hydrazone-Based Transformations Bergbreiter, and Momongan (1991). "Hydrazone Anions". Comprehensive
Organic Synthesis. Trost and Fleming. Oxford, Pergamon Press. 2: 503.

■ Wolff-Kischner Reduction
Cume Question, November, 2000
■ Wharton Rearrangement
Sorensen and coworkers recently reported the synthesis of (–)-hispidospermidin (Sorensen
■ Eschenmoser-Tanabe Fragmentation JACS. 2000, 122, 9556). The Shapiro Reaction, along with methodology developed by
Whitesell, was use in the construction of intermediate 3 from the indicated building blocks 1
■ Reduction of Tosyl Hydrazones: "The Alkene Walk" and 2 (eq 1).
R
■ Tosyl Hydrazone-Based Fragment Coupling O O H HN
H Me
Me H
■ The Shapiro Reaction Me
O Me H
■ Bamford-Stevens Reaction Me
O
Ph
Me O
Me
Me Me
1
2
Reading Assignment for this Week: (–)-hispidospermidin
2,4,6-triisoproylbenzene-
sulfonyl hydrazine,
Shapiro Reaction: Chamberlin, and Bloom. “Lithioalkenes from HCl, CH3CN, 75% Intermediate n-BuLi (2.05 equiv) Intermediate
arylsulphonyl-hydrazones.” Org. Reactions 1990, 39: 1. (handout) 1 A B

Wolff-Kishner & Related Reactions: Hutchins, (1991). "Reduction of C=X to MgBr2, -78 °C
55% then add 2
CH2 by Wolff-Kishner and Other Hydrazone Methods". Comprehensive Organic
Synthesis. Trost and Fleming. Oxford, Pergamon Press. 8: 327. (in library)
O
Me Me
O Me
Ph
Friday, HO Me
Me (eq 1)
D. A. Evans November 21, 2003
Me 3

28-00-Cover Page 11/19/03 2:02 PM


D. A. Evans, N. Finney Hydrazone Transformations-1 Chem 206

Hydrazone Anions: A useful Reversed Polarity Equivalent O O Me2N


N N NO2
R R R Me2N RT, ca. 24h
N
N:– N:– N: + R' R'
CH2Cl2
(–) N N ✔✔ N CH2
R
R
R (+) H R H R (–) H 1. O3; 2. DMS
A C(+)
A C(–)
A C(+) A C(–) (40-92%)
O NO2
R=alkyl or aryl
R'=H or alkyl H R'
t i t i t i
Bu N Pr n-BuLi, 0°C Bu N Pr Bu N Pr
N N N R
THF Lassaletta, J-M, et al.Tet. Lett. 1992, 33, 3691.
H H H H
Chiral Hydrazones as Carbonyl Anion Equivalents
Ph Ph
J. E. Baldwin, et al. JCS Chem. Comm. 1983, 1040. PhCHO Ph Ph ‡ OBn
OMe MgI2
N
i t i N N CO2Me
O Pr Bu N Pr CH2Cl2, –78°
N N
1. n-BuLi H
95% H CO2Me
HO Ph 2. H+/H2O LiO Ph H H R
H H
Ph R = Me; 89:11
CO2Me
R = Ph; >98:2
tBu N O Me
tBu N CO2Me
Ph
NH 1. n-BuLi, -78°C N H X I
N H R H O
N Mg
H 3 CO2Me
Me OCH3 OCH R O I
Me H Me X
H R CO2Me
O Me O

A C(–) H+, H2O Lassaletta, J-M, et al. Chem Commun 2002, 498-499
‡ OMe
N
A C(+) OMe TBSOTf N
tBu
N N O CH2Cl2, –78°
O N H
hydrolysis N H
OCH3 O
Me OCH3 H H O
58% Me
Me O dr = 90:10 OTBS
Me O
D. Enders et al. Syn Lett 1999, 629-631
O
J. E. Baldwin, et al. JCS Chem. Comm. 1984, 1095. O TBS

28-01 Hydrazones-1 11/24/03 2:23 PM


D. A. Evans, N. Finney Hydrazone Transformations-2 Chem 206

Wolff-Kishner Reduction The Wharton Rearrangement


Me R Me R OH
O N 2H 4 , R R N 2H 4 R R
NaOCH2CH2OCH2CH2OH, R
Me Me R R
(HOCH2CH2)2O R
O
H H H H NNH H
reflux and then
CH3CO2 O heat to 210°C HO
H H
Me Me Me Me
This example illustrates the 2 possible modes for the decomposition of A.
Barton, D. H. R., Ives, D. A. J., and Thomas, B. R. J. Chem. Soc. 1955, 2056.

H
O N NH
Mechanism Me Me O N H
Me Me O Me Me OH N
R 2C N NH2 + OH R 2C N NH R 2C N NH Me :B
Me Me
A C(–) H
Me N 2H 4
RO–H CH3OH, ∆ Me Me A
OH –N2
R 2C N N R 2C R 2C H A C(+)
(RDS)*
H H H
–N2, H•
radical polar
A C(–) RO–H RO pathway –N2
pathway

Elimination of α-Leaving Groups Me Me OH Me Me OH Me Me OH



BrCH2 Me Me Me Me
BrCH2 Wolff-
Me Me
N2H4, H+ Kishner Me Me
O N–NH2 Me Me
30% 20%
A C(–)
G. Stork et al. JACS 1977, 99, 7067.

Me Some procedural improvements:


Br Me
Me
H Me Me O Me Me OH
–N2 O
Me NH2NH2•H2O Me
N N –HBr
-B KDA, KO Bu, etc.t
Me Me 76%
H
Me CH2
For stable hydrazones, strongly basic conditions favor the ionic pathway.

D. H. Gusyafson, W. F. Erman J. Org. Chem. 1965, 30, 1665. C. Dupuy, J. L. Luche Tetrahedron Lett. 1989, 44, 3437.

28-02 Hydrazones-2 11/19/03 4:05 PM


D. A. Evans, N. Finney Hydrazone Transformations-3 Chem 206

Tosylhydrazones – Better Than Hydrazones R O R Ts NaOAc•3H2O


C NNHTs + BH R C N NH
Tosylhydrazones are isolable, stable, and easily prepared.
R O H B
The presence of the tosyl leaving group strongly biases the system towards RO OR
polar reaction pathways under hydridic reducing conditions.

H R Ts
CHO TsNHNH2 LAH (xs) R C N NH R R
NHTs
N H– R C N NH R C H
THF, ∆ THF, ∆ B OAc
H H
RO OR

G. W. Kabalka, et al. J. Org. Chem. 1975, 40, 1834.


H H – H H
– –
N –Ts N
N Ts N

Another Interesting Leaving Group
Ph

CH3 N Ph
H+/H2O N Ph Ph

LAH N N
Me N Ph N
H– Ph
L. Caglioti, M. Magi Tetrahedron 1963, 19, 1127. R Me R Me

-stilbene
Further Refinements
N
Very mild reduction with NaBH3CN under slightly acidic conditions (pH 4-5). –N2 N
Me R–H Me
No reduction in the absence of acid; carbonyl, nitro, nitrile FGs unaffected.
Aromatic, sterically hindered carbonyls very poor substrates. R Me
59%

A. R. Chamberlin, et al. Tetrahedron Lett. 1991, 32, 1691.

N NaBH3CN, CH3CO2H
NHTs 94%

R. O. Hutchins, et al. JACS 1973, 95, 3662.

28-03 Hydrazones-3 11/19/03 1:49 PM


D. A. Evans Hydrazone Transformations-4 Chem 206

The Eschenmoser–Tanabe Fragmentation Tosylhydrazone Reductions: The Alkene Walk


CH3 CH3
TsNHNH2, AcOH Me Me
H NaBH3CN
O CH2Cl2, RT N N
O O+ Ts TsNHN
H H

Me Me
CH3 CH3 NaBD3CN
–N2
O
N N Ts O
O S Ar NNHTs
O 68% H D
base H
C. Djerassi, et al. JACS 1976, 98, 2275.

A. Eschenmoser, et al. Helv. Chem. Acta 1967, 50, 708.

Ph H-
N H N H N H N
O Ph N : N + H N H N
HOAc ∆ Ts Ts
CH3 + N CH3
O Et2O, 0°C O
NH2 +
H This has been developed into a reliable reduction

O
84% Me
Me
Ph
N CHO
N + H 3C
Me O Me
+ 94%
CH3 81%
Me Me
Ph CH2 Me Me
O H
Me Me
A. Eschenmoser, et al. Helv. Chem. Acta 1967, 50, 2108.
16 cases reported: Hutchins, et al. JOC 1975, 40, 923

28-04 Hydrazones-4 11/19/03 1:50 PM


D. A. Evans, N. Finney Hydrazone Transformations-5 Chem 206

Alkene Walk: Syntheses Sulfonylhydrazone Reductions: Alcohol Deoxygenation

EtO2CN NCO2Et O
MeO MeO RCH2 OH RCH2 N S Ar
H H Ph3P, –30 °C
Me 1. NH2NHTs, THF, 25°C Me ~0 °C
NH2 O
H SO2–NHNH2 O
2. Catecholborane
3. NaOAc•3H2O RCH2 N N–H S Ar
H H (Mitsunobo Reaction)
AcO O AcO NO2 Org Rxns Volume 29 O–
In●
[H]
RCH2 H RCH2 ●

MeO
H MeO
Me H OH H
Me
–OAc 80%
Bu Bu
–HSO2Ts
H
AcO N H H
H N AcO N H
N OH
Ts Me
86%
Compactin,
Wendler, N. L., et al. Tet. Lett. 1982, 23, 5501. MeO MeO
OMe OMe

The intervention of radicals has been implicated (again):


The stereochemical course of the hydrazone reduction may be
stereospecifically transferred via the 1, 3-rearrangement

86% Me
OH

CO2Et CO2Et
1. NH2NHTs, THF, 25°C

2. CB O O
O 3. NaOAc•3H2O N N
H Me Me
(60%) OH
Topiramate,
Maryanoff, B. E., et al. Tet. Lett. 1992, 33, 5009.

10 cases reported: A. Myers, et al. JACS 1997, 119, 8572.

28-05 Hydrazones-5 11/19/03 2:26 PM


D. A. Evans, N. Finney Hydrazone Transformations-5 Chem 206

Tosylhydrazone-Based Fragment Coupling Stereoselective Construction of Trisubstituted Olefins


A. G. Myers, P. J. Kukkola JACS, 1990, 112, 8208.
O
R' Li R R'
R H Me Me
Me O Me
O 1) H2NNHTs
O O
Ts Ts 2) Et3N, TBSOTf Me O
Me O Et
N 3) RLi (Z)
N NH Me
N TBS R' Li LiN TBS N Me O
AcOH O 4) AcOH, F3CCH2OH O Me
O CHO
–78 °C A
R H CF3CH2OH R R' H
R R'
–78 → rt
TBS = t-BuMe2Si– RLi Ratio Z:E Yield
H Me
The monoalkyl azene A decomposes via a radical pathway
O Me
Et
NHSO2AR Me Me H 50:50 79%
N O
Me O
Ph 95% Ph Li H Me
RO (E)
RO H Me Me
LI Me Me <5:95 81% O O Et
Me
Li H
16 cases reported: A. G. Myers etal. JACS, 1998, 120, 8891.

A Complex Application: A. Smith etal. JACS 1999, 121, 7423 Me Me


O Me Me
O
Me
Ts Bu O H O
N Me O Me O
N Me
I Me
TBS MeO OMe t-BuLi (1.8 eq.) Me Me Me H
O O O O
ether, -78 °C Li
TBS N N Me
(73%) N N
MeO OMe
RO SO2Ar H
Bu H
Me
Me Et
Bu H
Me Me Me Me Me H
Li
Bu MeO OMe CHO Me
Me (as above) Me
HO OH H H Et
MeO OMe 90%, ca. 2:1
H
RO
HO OH Bu Me
Me Me Me H
Me Me Me
Bu Li Et
CHO
Me Cylindrocyclophane-F Me (as above) Me
H H CH3
82%, >20:1
28-06 Hydrazones-6 11/19/03 1:52 PM
D. A. Evans Hydrazone Transformations-7 Chem 206

O Cycloadditions
α-Keto Carbonium Ion Equivalents
C C (+) OH O
base N N
90% yield
O Br Na2CO3
HO H–O N F3C F3C
N base N
Br Br R
R R Ressig, J. Org. Chem. 1992, 57, 339
very electrophilic
Nu: acceptor there is the expected
Review: T. L. Gilchrist, Chem. Soc. Rev. 1983, 12, 53-72 Stereoelectronic Issues Nu
strong chair-axial bias
Reactions with Nucleophiles H Br H
base PhMgBr
H

OH O H N NHPh H N NPh
O HO
N base N N O H Ph
Br –HBr N
Ph N Ph
H Ph H
rate-det
step R2NH anti oxime is H N NHPh
Stereoelectronic vs Steric Issues
kinetic product
T. Kaiser, JACS. 1972, 94, 9276-9277 Me Br Me H
PhMgBr
H Ph
OH O OH
N N LiCuMe2 N N NHPh
H H N
Me PhHN
LiCuMe2
O Bozzini, Tetrahedron 1982, 38, 1459
OLi OH

H H
Corey, Tetradedron Lett. 1975, 3117 SPh there is the expected
NaSPh
t-Bu t-Bu strong chair-axial bias
H

OH HO N NPh N
N N PhHN
Br Me Bozzini, Tetrahedron 1982, 39, 3413
R R
OLi Me
O
R = (CH2)3CH=CH2 2 equiv 92% yield

Corey, Helv. Chem. Acta. 1977, 60, 2964

28-07 Hydrazones-7 11/20/03 11:35 AM


D. A. Evans The Shapiro Reaction-1 Chem 206

Deprotonation of the monoanion occurs predominantly at the kinetically


NHTs
more acidic site giving after elimination the less substituted alkene product.
1. Strong Base
N R’ + N2 Li
R BuLi
R’ 2. Quench C5H11
R
TrisHN TMEDA/hexane
N

General Reviews: C5H11


Me Li Li
Trost, Ed., Comprehensive Organic Synthesis 1992, Vol 6, Chapters 4.3 . BuLi
(E):(Z) = 4:1 C5H11 + C5H11
Trost, Ed., Comprehensive Organic Synthesis 1992, Vol 6, Chapters 4.6. Me
THF
Shapiro, Organic Reactions 1976, Vol 23, pp 405-507. 4:1
In THF solution regiochemical ratios generally reflect the starting
Mechanism: hydrazone geometries
SO2Ar SO2Ar Bond J. Org. Chem. 1978, 43, 154.
NHTs N N N
Li Li N Li Me
N nBuLi N nBuLi N Li Me
1. TsNHNH2
–TsLi H H
O Me 2. LDA
p.
p. H Me
Li Me O 65 % Me H O
O O
O O Grieco J. Org. Chem. 1977, 42, 1717.
nBuLi
S NR S NR
O O Me Me
H Li
Side Reaction nBuLi, THF
Me MeO MeO
NNHTs
Me H 80 % H
Me
the Triisopropylsulfonyl (Trisyl) group is used
(Roberts Tet. Let. 1981, 22, 4895). Trisyl = SO2
Nemoto et. al. JCS, Perkin Trans. 1 1985, 927.
Me
Me
Me 1. TsNHNH2
O via dianion
Me Me Me 2. K2CO3, ∆ Me
O
O 1. TsNHNH2 Me
Regiochemistry + Me
2. MeLi, Et2O O 1. TsNHNH2
Me via trianion
Me
(98 %) 2. LDA O
(2 %) Me
O
1. TsNHNH2
Me Me
Me O 1. TsNHNH2 Me CO2Et
2. MeLi, Et2O Me
Me Me CO2Et EtO2C
Me 2. LDA
(100 %) (95 %) (5 %)
28-08-Shapiro Reaction-1 11/19/03 1:54 PM
D. A. Evans The Shapiro Reaction-2 Chem 206
Trapping of the intermediate alkenyllithium Ts Ts H
N TBS H N TBS
N N Ph
CH2R 1. nBuLi N N
R’ OH -"TsTBS"
Ph H 2. HOAc Ph C 4H 9
R C6H13 CO2H C 4H 9
H Ph C 4H 9 Me
Me Me H
C6H13 C6H13 Me (E):(Z) = 12:1
RCH2Br
O CO2
Li O Me O
O O
R’ R O H O
C6H13 + Li Me
Me2NCHO N 81 %
O O N(TBS)Ts Et
Br+ >20:1 (E):(Z) O O Et
SiMe3 Me3SiCl CHO
H
Br Applications Myers, J. Am. Chem. Soc. 1990, 112, 8208
C6H13 C6H13
O
C6H13

Me H Me H
NHTris
p.
p. 1. TsNHNH2, TsOH
N Cl BusLi Li Cl
0°C
Me Me
TMEDA/hexane O 2. nBuLi, TMEDA O
82 % H H Aphidicolin
-78°C 82 %
Bloom Tet Lett. 1984, 25, 4901 O O

van Tamelen J. Am. Chem. Soc. 1983, 105, 142.


Carbonyl Transposition Me Me Me
Me Me
1. 2.1 equiv. nBuLi NNHTris BuLi
NNHTs Li
TMEDA/THF 2 eq. HgCl2 TMEDA/hexane
HO Me HO Me
2. MeSSMe 82 %
Me Me SMe Me O
3. 1 3quiv BuLi RCHO
warm Nakai Chem. Lett. 1980, 1099 75 % Me

Me
Shapiro alternatives Me Me
Me Me Me Me
CO2Me CO2Me CO2Me
O N N 2.7 eq. LDA OH
H HO Me
H H 0°C THF Me
H 2N N Ph OH
- styrene Me
Ph Me Me
-N2
Me OMe Me OMe Me OMe
Me Me Me
Me
(Juvabione) Evans J. Am. Chem. Soc. 1980, 102, 774 HO Me
loroxanthin Baütikofer Helv. Chim. Acta. 1983, 66, 1148

28-09-Shapiro Reaction-2 11/19/03 1:55 PM


D. A. Evans The Shapiro Reaction-Applications Chem 206

A Recent Aplication of the Shapiro Reaction Part B (7 points). Provide a mechanism for the transformation of intermediate B to the illustrated
product 3. Use 3-dimensional representations to illustrate the stereochemical aspects of this
individual step.
Web Problem 24. Sorensen and coworkers recently reported the synthesis of (–)-hispidospermidin X
(Sorensen JACS. 2000, 122, 9556). The Shapiro Reaction, along with methodology developed by X
Whitesell, was use in the construction of intermediate 3 from the indicated building blocks 1 and 2 Mg O
MgBr2, -78 °C
(eq 1). O Me
Li then add 2
MgBr Back(Si) face of C=O O Me
R attacked by Nu
Ph
O O H HN Me Me Me Me
H Me Me Me 2
Me H
Me
O Me H Front (Re) face of C=O
Ph blocked by Aryl moiety
Me Me O
O Me
Me Me
1 O
2 Me
(–)-hispidospermidin Me
2,4,6-triisoproylbenzene- O Me
sulfonyl hydrazine,
n-BuLi (2.05 equiv) Ph
HCl, CH3CN, 75% Intermediate Intermediate HO Me
1 A B Me (eq 1)

Me 3
MgBr2, -78 °C
55% then add 2

p.
p. O
Me Mattox-Kendall Dehydrohalogenation (Paquette, Reagents, Vol 5, p 3509)
Me
O Me
Ph Me Me
HO Me
Me (eq 1) H2N–NHCO2Et
H3O+
Me 3
AOAc, heat
O N
H
Part A (8 points). Provide a mechanism for the Shapiro Reaction of 1 to intermediate B in the space Br NH Me
EtO2C
below. Feel free to use a simplified analog of 1 such as 2,2-dimethylcyclopentanone to answer this
question. Me

H O H O
Li
O N SO2Ar N S Ar Problem: The syn relationship between Br and H
N N H renders the direct dehydrohalogenation with base
O unfavorable (relative to other potential reactions.
Me Me n-BuLi (2.05 equiv) Li O Br Solution; proceed via the hydrazone.
Me Me Me
SO2Ar Me
H 2N N O
H S Ar Me
Me
Me
O
N Li AOAc, heat
Li N N
N N
H N
Me Me N Br H N
Me Me EtO2C H N
EtO2C H NH
Br EtO2C
tautomerization

28-10-Shapiro Reaction-3 11/19/03 2:13 PM


D. A. Evans The Bamford-Stevens Reaction Chem 206

An Alternate Decomposition Pathway for Tosyl Hydrazones


Me Me Me Me
NaOMe, diglyme CH2
ca. 180 °C Me
R R R Me
R' base R' ∆ R' N NHTs Me Me
N+ 45% 55%
N : N –
NHTs NTs
N–

Me Me Me Me
R products
R' CH3Li, Et2O, 0 °C
:

quantitative
Me N NHTs Me

p.
p.

Bamford-Stevens vs. Shapiro


Directed Bamford-Stevens Hb
CH3 CH3
R R'
R R' Ha Hb 1,2 Ha Ha
CH3 63% 27%
NaOCH3, NMP, ∆ 1,2 Hb
NNHTs N R R' Ha
NNHX
CH3 R R'
4% Hb
CH3 trace

PhCH3, 145°C
TMS Bn
CH3 Ph
CH3 66% (4:1 E:Z)
2 eq. BuLi, 0 °C
N
NNHTs N
98% PhCH3, 145°C
TMS Bn Rh2(OAc)4 TMS Bn
66% (14:86 E:Z)
R. H. Shapiro Org. React. 1976, 23, 405.
T. K. Sarkar, et al. JCS Chem. Comm. 1992, 1184.

28-11-Bamford-Stevens 11/19/03 1:58 PM


Chemistry 206

Advanced Organic Chemistry

Handout–28A

Oxidative Coupling of Enolates & Enol Derivatives

OM O
R –2 electrons R
R R
OM O

Chuck Scales

Evans Group Seminar, March, 1995

D. A. Evans Friday ,
November 21, 2003
28A-00-Cover page 11/20/03 5:52 PM
Reaction Background

The Oxidative Coupling of Enolates Synthesis of 3,4-diphenyl succinic acid


and Enol Derivatives O
1) i-PrMgBr / Et2O OH
ONa
Evans Research Seminars 2) O2 OH + HO
March 14, 1995 OH
O
3) H3O+ O
O

38% 12%
O O
[O]
O
- 1) i-PrMgBr/Et2O
O ONa
2) Br2/CCl4 HO
An underappreciated umpolung of enolates O OH
3) H3O+
O

1. Ketone Enolates 22%


2. Ester Enolates
3. Carboxylic Acid Dianions • Also obtained a "high" yield of a-bromophenylacetic acid
4. Silyl Enolate Derivates • Proposed radical dimerization as mechanism for production of
5. Applications to Organic Synthesis β-diphenylsuccinic acid.

Ivanoff, Bull. Soc. Chem. Fr., 1935, 2, 76


Leading Papers:
Saegusa, T. JACS, 1977, 99, 1487 Synthesis of 1,4 Diketones
Fox, M.A. JOC, 1988, 53, 3745
Narasaka, K. Chem. Letters, 1992, 2099

O O
(AcO)2, 75° C
Chuck Scales 52%
O

Kharasch, M.S., McBay, H.C., and Urry, W.H., JACS, 1948, 70, 1269

28A-01.handouts 1/11/00 11:21 AM


Reaction Background Synthesis of 1,4-Diketones

Coupling of Stabilized Anions


Methyl Ketone Dimerization
X X
1) n-BuLi
M M (R)n O 1) LDA, THF, -78°C O
R)n R)n M 2) CuCl2, DMF, -78°C
II R
2) Cu R R
X
M = C, P, S O
X = O, BH3, S
R Yield

phenyl 95%
ferrocenyl 78%
Mislow, K. JACS, 1973, 95, 5839 furanyl
Tamaru, Y. JACS, 1978, 100, 1923 41%
cyclopropyl
A. Muci and K. Campos, unpublished results
-C(CH2)2CO2Et 46%
64%
CH CH 82%
Anion Coupling Models

• Type 1 Oxidants
α-Substituted Ketone Dimerization
OM -1e - O dimerization O
R
R R • R
LDA, THF
CuCl2, DMF
O 73%
-78° C
O O
2

• Type 2 Oxidants

OM LDA, THF
OM O O
CuCl2, DMF 60%
E+ E R R -78° C
R R R
O O
2
O

• Product mixture of exo-exo, exo-endo, and endo-endo isomers

Saegusa, JACS 1977, 99, 1487

28A-02.handouts 3/14/95 1:03 PM


Synthesis of 1,4-Diketones 1,4-Diketones with Copper (II) Triflate

From enolates
Cross-Coupling of Methyl Ketones
3 eq acetone
O 4.5 eq LDA O O LDA, THF, -78 °C O
4.5 eq CuCl2 R Cu(OTf)2, i-PrCN Ph
R Ph Ph 83%
THF/DMF, -78° C O O

Yield
R Cross-couple Dimer O LDA, THF, -78 °C O
Cu(OTf)2, i-PrCN Ph
Et 80%
68% 1% Ph Ph
O

53% 1%
O LDA, THF, -78 °C O
Cu(OTf)2, i-PrCN Et
Et 63%
59% 4% O
O

65% 2%
Anions Make a Difference

O O
CuCl2
Ph
Ph Ph 28%
γ Coupling of α,β
β-unsaturated ketones O
O
O LDA, THF/HMPA (4:1)
O
O
O CuCl2, DMF
-78°C O

33% γ,γ coupling product • Yield difference may be due to lability of triflate versus chloride ions

• No α,α coupling product seen O


O
• γ,γ couple not produced from thermal O
rearrangment of α,α product
O Kobayashi, et al. TL, 18, 3741 (1977)
Kobayashi, et al. Chem. Pharm. Bull., 28, 262 (1980)
32% α,γ coupling product
Saegusa, T. et al. JACS 1977, 99, 1487

28A-03.handouts 3/14/95 1:05 PM


Coupling of Ketone Enolates Intramolecular Coupling

Ketone cyclization
Oxidative Coupling with Ferric Chloride

O O O O
1) LDA, THF, -78 °C 1) LDA (2 eq), THF, -78 °C
52% Ph Ph 38%
2) FeCl3, DMF, RT 2) Cu(OTF)2, i-PrCN
O
O O

• Dimers obtained from hindered enolates in moderate yields (40-60%)


• Generally poor yields; spirocyclic examples included.
• Prepared from kinetic and thermodynamic enolates

Frazier, R.H. et al. JOC, 1980, 45, 5408 Kobayashi, et al. TL, 1978, 19, 3555

Oxidation with Manganic Acetate


Tetraketone Synthesis
O O
Mn(OAc)3 O O
OAc
+ AcOH 22% KOH (10 eq.)
O O
80% aq. MeOH Ph
70 °C O 62%
Ph
0 °C Ph
e- , 30 min.
O O
• Proposed radical coupling mechanism for transformation

Dessau, R.M. et al. JOC, 1974, 39, 3457


• Synthesized various aryl substituted 3,4-aroyl-2,5-hexanediones

Oxidative Dimerization of Aldehydes • Observed that EDG favor oxidation and EWG disfavor oxidation
(correlated to Hammett plot)
O O O
MnO2, reflux
H + O
H H H
48 hours
O

Hammett Plot Here


• All examples produced ca 80% yield of dimer in 45:55 ratio (C-C:C-O).

Leffingwell, J.C. JCS Chem. Comm., 1970, 357

Lacan, M. et al. Croat. Chem. Acta, 1973, 45, 465

28A-04.handouts 3/14/95 1:07 PM


Ester Enolate Coupling Oxidative Coupling Mechanism

Proposed Mechanism

Synthesis of Succinate Esters OLi O


Radical
O
SET
R
R R • Recombination R
O
1) LiN(Cy)iPr, THF O R
O -78° C OR' • Radical may be associated with oxidizing agent
R'O
RCH2 OR' 2) 1 eq CuX2, -78° C
R O
15 min. Evidence for Radical Mechanism
O O O
1) LiN(Cy)i-Pr, THF, -78°C
Br
O O + O
2) CuBr2
2
Yield
85% "minor side product"
R R' CuBr2 Cu(O2C5H9)2

-H 85 • Investigation of possible SN2 mechanism


-t-Bu 95
-Me -Et 81 50
-n-Bu 63 ----
-Me2 -Et OLi O O
25 20
-i-Pr -Et 20 THF
20 + Br
-Ph -Et 75 O O O 10%
60 -78 °C
-Et
2

• Exclusion of bromide source also leads to product


• Increasing alkyl substitution decreases yield of dimer

• Yield of a-bromoester increased with increasing alkyl substitution O


OLi Cu(O2C5H9)2
when copper (II) bromide used as oxidizing agent
O 95%
• Yield of dimer not increased with copper (II) valerate O THF, -78 °C
2
• Product obtained as an unspecified mixture of stereoisomers

Rathke, J. Am. Chem. Soc. 1971, 93, 4605


Rathke, J. Am. Chem. Soc. 1971, 93, 4605

28A-05.handouts 3/14/95 1:09 PM


Succinate Esters with Other Oxidants Intramolecular Coupling

Oxidation with Iodine Ester cyclization

COOR LDA (2 eq) COOR


O O (CH2)n
THF, -78 °C (CH2)n-2
1) LDA, THF, -78° C R1 R2
R1 R3 2) I2 (0.5eq), warm to RT R3 O COOR CuCl2 COOR
O O R3
R2 R2 R1 O
n R Yield
R1 R2 R3 Yield

H H t-Bu 97% 3 Me 88% (3:1 cis:trans)a


Me H t-Bu 80% 4 t-Bu 20% (undetermined mixture of stereoisomers)b
Me Me Me
Me Me Et
90% 5 t-Bu >50% (undetermined mixture of stereoisomers)b
85% 6 Me 93% (0.6:1 cis:trans)a
• Equilibration between cis and trans isomers noted for all reactions.
• Authors propose an SN2 mechanism for this transformation
• Dimethyl adipate and dimethyl pimelate gave exclusively Dieckmann
cyclization under the reaction conditions.
Brocksom, T.J., et al. Synthesis 1987, 396
a
Chung, S.K. et al. JOC, 1983, 48, 1125
b
Babler, J.H. et al. JOC, 1987, 52, 3462
Oxidation with TiCl4

Oxidation of dimethyl β,β


β-dimethylglutarate
O Me CO2Me
1) LDA, THF, -78° C Me O O 1) LDA (2 eq)
OMe
2) TiCl4, CH2Cl2, -78 ° C 80 % THF, -78 °C H CO2Me
Me
Me CO2Me MeO OMe
2) [O], DMF MeO2C H

Oxidant Temp. Yield cis:trans


• Excellent yields with α-substituted esters (i-Pr, BnO)
• Other Lewis acids (BF3.OEt2, SnCl4) do not promote oxidation CuCl2 -78 71% 57:43
• ZrCl4 resulted in Claisen condensation products CuCl2 0 83% 68:32
• Not applicable for ketones or amides I2
-78 76% 19:81
AgCl
-78 81% 80:20
Ojima, et al. Chem. Letters 1992, 1591

• No attempt to rationalize stereochemical outcome of reaction

Babler, J.H. et al. Synth. Comm., 1983, 13, 905

28A-06.handouts 3/14/95 1:12 PM


Carboxylic Acid Dianion Coupling Iodine Oxidation Mechanism

Coupling of Acid Dianions Mechanistic Investigations

O O Ph
1) n-BuLi (2 eq), THF, -78° C
1) 2 eq LDA COOH
Ph 2) I2 (O.5 eq), -65° C OH
+
OH HO COOH
2) 1.1 eq I2 I
Ph O COOH HOOC
THF, -78° C
2 min. 72%
16%
90%, 11:1 dl:meso
• Yields increased with dianion salt isolation

Stereochemical Model THF/HMPA COOH


+ -78 °C
<10%
Li COOLi COOLi
10 min. HOOC
O O Li I
O O

- I
S L L S • "Dimerization reaction is much faster than nucleophilic substitution
under the reaction conditions."
Belletire, J.L. et al. TL, 1984, 25, 5969

γ Coupling of α,β
β-Unsaturated Carboxylic Acids

R' O 1) 2 Et2NLi, THF O R' R


-70° C OH
1) 2 LDA, THF, -78 °C
Ph COOH
OH HO HOOC
COOH +
R 2) AgNO3 R R' O 2) [O] Ph COOH
1
R R' Yield E,E:E,Z:Z,Z
[O] 1 2
H H 49 1:0:0
COOH
Me H 63% 1:0:0 e- 16 40
H Me 38% 1:1:1 2
Me Me 61% 7:3:0 I2 23 38
O
O 1) 2 Et2NLi, THF • Formation of α,para coupling product 2 supports radical anion intermediate
-70° C OH
31%, E,E only
OH OH
2) AgNO3

O
• No rational for observed stereochemistry • COOLi COOLi

• Also observed unspecified yields of γ,α coupled product


Mestres, R. TL, 1988, 29, 6181 Fox, M.A. et al. JACS 1988, 53, 3745

28A-07.handouts 3/14/95 1:14 PM


Iodine Oxidation Mechanism Mechanism for Iodine Promoted Coupling

5-Hexenyl Radical Trap


R OLi

R' OLi
I A
1) 2 eq LDA
2) I2 (1 eq) 70%
COOH COOH
+
R COOLi R
COOH • COOLi
12% R' I R'
COOH C D

R
Ph O COOLi
1) 2 eq LDA R'
Ph
2) [O] O 1 R'
COOLi
COOH R
Ph O B
+

Ph
[O] 1 2 COOH

I2 26% 32%
e- 2 • Proposed initial step is SET to form radical anion (D).
8% 36%
• Radical anion (D) may iodinate, then form dimer (B) via SN2 reaction

COOH • Radical anion may form dimer directly, especially if R and R' are large (>H)

• Direct iodination of dianion neither supported or excluded by experiments

No cyclopentylmethyl products seen!

Fox, M.A. et al. JACS 1988, 53, 3745


Fox, M.A. et al. JACS 1988, 53, 3745

28A-08.handouts 3/14/95 1:16 PM


Silyl Ketene Acetal Dimerization Silyl Ketene Acetal Coupling

Carbocycle Synthesis
Silyl Ketene Acetals

MeO OMe TiCl4, CH2Cl2


R R' Yield H CO2Me
69% Yield
TMSO OTMS RT 100% trans
TiCl4 (1 eq) R' CO2Me MeO2C H
R OTMS
CH2Cl2, RT R H Me 79%a
R
H i-Pr 62%b • Stereochemical course under thermodynamic control
R' OMe H
R' CO2Me t-Bu 72%b
H Cy 57%b • No coupling from ketone-derived enol silyl ethers
H Ph 73%a
Me Me • Generally poor yields; exclusively 1,3-trans product.
80%a, 93%b
Et Et
67%b Chan, T.H. Tetrahedron, 1983, 39, 847
• No Claisen type products observed
Acrylonitrile Trapping
• SET (TiIV --> TiIII) followed by radical coupling mechanism proposed by both authors

• Other reagents (CuII salts, FeCl3) ineffective for coupling reaction. CN CN


Me OMe
CO2Me 63% Yield
a
Ojima, I. et al. TL 1977, 18, 2009 MeO2C 1:1 diast. ratio
b Me OTMS TiCl4, CH2Cl2
Rhodes, Y.E. et al. Synth. Comm. 1985, 15, 301 CN
-78° C

Vinyl Ketene Silyl Acetal Coupling


OMe CN CO2Me
NC
OTMS
52% Yield
TMSO R' TiCl4 R R' (CH2)3 Mixture of all diastereomers
OMe
CH2Cl2 CO2Me γ,γ coupling TiCl4, CH2Cl2 NC
MeO MeO2C OTMS -78° C CO2Me
R 0° C R' R
+
R R' • Authors propose enoxyradical trapping by acrylonitrile,
followed by dimerization
R R' Yield γ,γ : γ,α
MeO2C γ,α coupling
83:17 • Reactions with methacrylonitrile gave poor yields.
H H 98% R' R CO2Me
H Me 76% 64:36
Me H 98% 96:4 • Attempts to trap putative radical intermediate with FeCl3, CCl4, CBr4, and
tributyltin hydride failed.
Rousseau, G. Tetrahedron, 1990, 46, 7011
Ojima, I. et al. TL 1983, 24, 785

28A-09.handouts 3/14/95 1:19 PM


Silyl Enol Ether Dimerization Silyl Enol Ether Dimerization

Hypervalent Iodine Oxidants Metal Oxidants

Cu(OTf)2, Cu2O O
O O
1) Et3N, TMSCl OTMS i-PrCN, 0 °C Ph
Ar Ph 55%
Ar Ar 43-62%
2) (PhIO)n-BF3•Et2O Ph
O
CH2Cl2, -40 °C, then RT O
Oxidant # eq Solvent Temp.(°C) Time (hr) Yield Ref.
Ar = Ph, p-HOC6H4, p-MeOC6H4, p-ClC6H4, p-MeC6H4
Cu(OTf)2/Cu2O 1,4 i-PrCN 0 2 55% a
Ag2O 1 DMSO 65 2 73% b
O O Pb(OAc)4 2:1 CH2Cl2:THF
1) Et3N, TMSCl 0.5 -78, then 23 1.5;1 45% c
VO(OEt)Cl2 CH2Cl2
t-Bu 2 -75, then -40 3;4 30% d
t-Bu t-Bu 57%
2) (PhIO)n-BF3•Et2O
CH2Cl2, -40 °C, then RT O
• All authors also reported yields for non-styrenyl silyl enol ethers.
• Yields are extremely substrate dependent.
Proposed Mechanism • Generally, increasing steric hindrance decreases yields.
• All authors propose oxidation to cation radical, followed by loss of trimethylsilyl
OTMS cation and radical coupling.
a
OTMS + - O O Kobayashi, Y. et al. Chem. Pharm. Bull., 1980, 28, 262
(PhI -OBF3 ) R
b
Saegusa, T. et al. JACS 1975, 97, 649
I(Ph)OBF2 R c
Moriarty, R.M. et al. TL 1987, 28, 873
R R R d
Ohshiro, Y. et al. TL 1992, 33, 5823
O
Cross Coupling Experiments
NuH
OTMS OTMS VO(OEt)Cl2 (3 eq) O
Nucleophile Trapping
+ CH2Cl2, -75 °C, 3h, Ph
Ph 93%
O H20, MeOH, EtOH
then -30 °C, 4h. O
Nu
R 2 eq 1 eq

OTMS O
OTMS
• Nucleophile trapping accomplished in "good yields." VO(OEt)Cl2 (3 eq) Ph
+ CH2Cl2, -75 °C, 3h, 77%
Ph
O
then -30 °C, 4h.
2 eq 1 eq
Moriarty, R., et al. JCS Chem. Comm., 420 (1985)
Moriarty, R., et al. J. Chem. Soc. Perk. Trans. I, 559 (1987)
Caple, R., et al. JOC, 54, 2609 (1989)
• Less reactive substrate added first, followed by more reactive substrate.
• In all cases, trace amounts of dimers isolated.

Ohshiro, Y. et al. ibid.

28A-10.handouts 3/14/95 1:21 PM


Silyl Enol Ether Coupling Application: Ketone Enolates

Alkene Trapping Cerorubenic Acid III: Construction of the Tetracyclic Core

OTBDMS O
H H •
Cu(OTf)2, Cu2O
H 90%, 20:1 •
CH3CN, 0° C H
H
H COOH
H 3C H
CH 3

Cerorubenic Acid III


• Also observed for δ,ε olefins.

• Kinetic product can be isomerized in KOH/MeOH. O O


1) LDA (2 eq), H •
Snider, et al. JOC, 1990, 55, 4786 THF, -78 °C
O O
2) FeCl3, DMF, H
6-Oxo-α
α,β-Unsaturated Carbonyl Compounds -78 °C, 2 hr. CH3
O
CH3
46-55% yield
CAN (2 eq) O
OTMS 0 °C H
OTMS + 77%
• Other oxidants (CuCl2, Cu(OTf)2) coupled with poor yields
CH3CN O
10 eq
Paquette, L.A. JOC, 1993, 58, 4245
Proposed Mechanism
+. C16 Hexaquinacene
OTMS OTMS O
R CAN -TMS+
R R
R' R' . R' 1) LDA, THF
-78 °C O
O OTMS TMSO R COCH3
2) CuCl2, -78 °C
COCH3
R . + R'' . R'' DMF:THF 7:1 O
R' R''' R'''
R'' O 58%
O R
CAN
R''
R'''
-TMS+
R'' O
Paquette, L.A. et al. JACS, 1978, 100, 1600

Ruzziconi, R. et al. TL 1993, 34, 721

28A-11.handouts 3/14/95 1:24 PM


Ketone Enolate Applications II Application: Carboxylic Acid Dianions

Total Synthesis of Racemic Wikstromol


Coupling of 2-acetylthiophene
O
1) LDA, THF, -78 °C BnO 1) LDA (2 eq) BnO OBn
S 2) I2 (0.5 eq)
S S 85%
O
2) CuCl2, DMF, -40 °C MeO OMe
O MeO CO2H 3)Ac2O O O O
A

1) MeOH BnO OBn


HCl, Ac2O 87% 61% from
S O S 2) BH3-THF
acid
A 3) H+ MeO OMe
O O

H2S, HCl 73%


S S S

HO HO OH

Kagan, J. et al Heterocycles, 1983, 20, 1941


MeO OMe
O O
γ-Coupling of Ketone Enolates
O O O
Belletire, J.L. et al, JOC, 1988, 53, 4724
1) KH, THF, 20 °C
90% • Anhydride "obtained as a single diastereomer by NMR analysis."
2) Br2, CH2Cl2, 0 °C
Total Synthesis of Racemic Hinokinin

1) CH3Li BnO
O O
61%
90%
2) TsOH, pyr., MeO CO2H O O
reflux O O

• Mixture of erythro and threo acids; anhydride exclusively threo.


Hart, H. et al. TL, 1977, 18, 2307

Belletire, J.L. et al, JOC, 1987, 52, 2549

28A-12.handouts 3/14/95 1:27 PM


Applications: Ketone Enolates and
Silyl Enol Ethers
Synthesis of Racemic Hirsutene

O OLi O
1)LiC(SPh)3 -78 ° C
+
2) s-BuLi
-78 °C
CLi(SPh)2

OLi OLi O O
H
FeCl3, DMF
64%
-78° to -40°, 30 m •
-40° to r.t., 10 h H
PhS SPh PhS SPh

• Isolated as a single diastereomer; proof by conversion to hirsutene.

Cohen, T. JOC, 1992, 57, 1968


Studies Toward the Synthesis of Brackenin

HO OH

O OH

HO
OH

HO O

HO OH

OTMS
2
Ag2O 38%
MeO O 1:1 dl:meso
MeO OMe OMe
MeO OMe

• "Use of Li enolates [for coupling] proved to be unsatisfactory."


Drewes, S.E. JCS Perk. Trans 1, 1989, 1585

28A-13.handouts 3/14/95 1:28 PM


Stereoselective Synthesis of Succinamides Oxidative Coupling of Enamines

Silyl Enol Ether Trapping


Oxazolidine Auxiliary Experiments
O
OTBDMS O
O Et O CAN (2 eq) Ph
1) LDA, THF, -78 °C N +2 Ph t-Bu 63%
Nc MeCN, rt
O N Nc O
2) [O] t-Bu 30 min
O Et

1
O
OTBDMS
Oxidant Yield 1: Σ others 2 TBCAN Ph
N 91%
CO2Me +2 Ph
I2 40-50% 92:8 EtCN, -78 °C N
50-55% CO2Me
CuCl2 98:2 K2CO3

• No model for induction proposed Proposed Mechanism


OTBS O
Porter, N.A. et al. TL, 1993, 34, 4457 O O
CAN Ph +
Oxazolidinone Auxiliary Experiments + N
N N
-e- • Ph
t-Bu
O O t-Bu t-Bu •
O OTBS
1) LDA, THF, -78 °C
O N Xp 50% yield
Xp 5:1 dl mixture
2) Cu(OTf)2, MeCN
O
Bn O
O
CAN + -TBS+
N Ph
t-Bu
-e- + Ph H 3O +
• Also obtained ca. 30% yield of meso dimer t-Bu O
OTBS

J. Ellman and M. Dart, unpublished results


Narasaka, K. et al. Chem. Letters 1992, 2099

28A-14.handouts 3/14/95 1:30 PM


Chemistry 206

Advanced Organic Chemistry

Handout–28A

Oxidative Coupling of Enolates & Enol Derivatives

Chuck Scales

Evans Group Seminar, March, 1995

D. A. Evans Friday ,
November 21, 2003
28A-00-Cover page 11/20/03 3:14 PM
Chemistry 206

Advanced Organic Chemistry

Handout–28B

Asymmetric Organocatalysis Using Chiral Amines

O Me O
Me O 3 mol% L-proline
Me
DMF, 20h, rt O
O OH
93% ee

An Evans Group Friday Seminar


Jonathan Lawrence
November 14th 2003

D. A. Evans Friday ,
November 21, 2003
00-00-Cover page 11/20/03 5:29 PM
Asymmetric Organocatalysis Using Chiral Amines

Me Me
Contents:
S
Background CO2H CO2H
Aldol reactions N N N NR2
H H H
Mannich reactions
Amination/Oxidation reactions
Michael reactions O Me O Me
Cycloaddition reactions N N
Me Me
Alkylation reactions Me
N Me N
Ph H Ph H Me

An Evans Group Friday Seminar


Jonathan Lawrence
November 14th 2003

Revent Reviews:
List, B. "Proline Catalyzed Asymmetric Reactions", Tet. 2002, 58, 5573-5590
Miller, S. "Amino Acids and Peptides as Asymmetric Organocatalysts", Tet. 2002, 58, 2481-2495
List, B. "Asymmetric Aminocatalysis", Synlett 2001, 11, 1675-1686
Dalko, P. "Enantioselective Organocatalysis", ACIEE 2001, 40, 3726-3748

Other Chiral Amines

Cinchona alkaloids: The "nucleophilic" catalysts

H NMe2
N
H H OH H
N
X R R
R2 N
H N R
N R1
N
R = OMe [(+)-quinidine] R
R = OMe, X = OH [(-)-quinine]
R = H [(-)-cinchonine] N
R = H, X = OH [(-)-cinchonidine]
N HH
R
Reviews:
Pracejus, H. Fortschr. Chem. Forsch, 1967, 8, 493.
Morrison, J., Mosher, H. Asymmetric Organic Reactions; Prentice-Hall: Englewood Cliffs, 1971.
Wynberg, H. Top. Stereochem. 1986, 16, 87.

Relevant Group Seminars:


Karl Scheidt, Asymmetric Catalysis with Chiral Lewis Bases (Part I), March 2001
Hemaka Rajapakse, Nonmetal-Based Asymmertic Catalysis (Part II), March 2001
Essa Hu, Asymmetric Catalysis with Chiral Lewis Bases (Part III), March 2001
Jake Janey, Asymmetric Catalysis with Chiral Lewis Bases (Part IV), March 2001

01-02 11/20/03 5:26 PM


Preliminary Findings

Yamada, 1969:

Me H ∆ CONR2
N
N
+ N Me
Ph CHO H preformed enamine
O
Ph
O

Me

MeOH/benzene 1:9

Me Me CHO
AcOH, H2O
Ph Ph Me

O O

49% ee
Yamada, S. TL 1969, 10, 4237.

The Seminal Experiments


O O O
R R
Me
3 mol% L-proline

O DMF, 20h, rt O
OH
R = Me 100%, 93% ee
R = Et 71%, 99% ee

O O O
Me Me
3 mol% L-proline
Me
DMF, 72h, rt
O O
52% OH
74% ee

the use of protic solvents severly diminishes enantioselectivity


other amino acids as catalysts lead to decreased chemical yield and enantioselectivity
Eder, Sauer, and Weichert obtained the corresponding aldol condensation products
in similar optical purity using 47 mol% L-proline and 1N HClO4

Hajos, J., Parrish, D. JOC 1974, 39, 1615.


Eder, U., Sauer, G., Weichert, R. ACIEE 1971, 10, 496.

03-04 11/20/03 3:24 PM


Effect of the Catalyst

O CO2H
Me N Me Me O
Me Me racemic
H
DMF HO
O O O
"major product"

CO2Me Me O
Me O
N
Me H
racemic
O DMF O
O OH O

"major product"

Me O
N CO2H
Me H no reaction
O DMF
O

Hajos, J., Parrish, D. JOC 1974, 39, 1615.

Transition States
Agami, 1984-1986 Houk, 2001-2003

H O
H
-
O 2C
N H N O
H O O H general hydrogen
NH bond energies
Me Me -OH - - O 3.0-8.0 kcal
-CH - - O 0.5-3.8 kcal
CO2-
O O

favorable (enamine) N-H---O hydrogen bond N-H---O hydrogen bond does not lower energy
N-H anti to carboxylate electrostatically favored of transition state
favorable O-H---O hydrogen bond
reaction is second-order in proline (non-linear
effect observed) additional NC-H---O hydrogen bond further
second proline acts as a proton shuttle, allowing stabilizes system
enamine to be nucleophilic reaction is first order in proline (supported by
kinetic data) and no non-linear effect observed
Agami, C. TL 1986, 13, 1501.
Houk, K. JACS 2001, 123, 12911.
Houk, K., List, B. JACS 2003, 125, 16. for a discussion on R3N+-C-H---O=C bonds, see:
Houk, K. JACS, 2002, 124, 7163.
05-06 11/20/03 3:26 PM
Direct Aldol Addition 1
The initial reaction:
O O OH
O 30 mol% L-proline
+ H Me
Me Me DMSO, 4hr, rt
20 vol% NO2 68% NO2
76% ee

Catalyst screen:
(selected examples)

compound % yield % ee compound % yield % ee

(L)-His, (L)-Val < 10 -- S


CO2H 67 73
(L)-Tyr, (L)-Phe N
H
HO
CO2H 55 40 CO2H 85 78
N N
H H
HO
CO2H > 50 - 62
CO2H < 10 -- N
NH H
Me Me

List, B., Barbas, C. JACS, 2000, 122, 2395-2396


S
CO2H
66 86 *
*Barbas, C. JACS, 2001, 123, 5260-5267 N
H

Direct Aldol Addition 2 Me Me


Substrate scope: S
variation of the aldehyde O OH CO2H CO2H
N N
Me R H H
1 2

product R = cat. % yield % ee product R = cat. % yield % ee


Cl
1 68 76 1 94 69
2 60 86 2 71 74
NO2

1 62 60 1 54 77
2 60 89 2 60 88

1 74 65 1 97 96
2 65 67 2 61 94
Br

DMTC 2 is catalyst of choice for aromatic aldehydes, although chemical yield decreases
due to slower rate of reaction
α−unbranched aldehydes yield no appreciable amount of product with proline catalyst 1
due to enolization and self-aldolization under reaction conditions (DMSO/acetone = 4:1)

List, B. JACS, 2000, 122, 2395.


Barbas, C. JACS, 2001, 123, 5260.

07-08 11/20/03 3:28 PM


Direct Aldol Addition 3
Me Me
Substrate scope: S
variation of the ketone donor CO2H CO2H
N N
H H
1 2
O O 20 mol % catalyst O OH
+
R1 Me H R2 DMSO, rt, 24-48hr R1 R2

product R = cat. % yield % ee

O OH
Me
1 65 77
2 57 74
NO2
O OH
Me Me 1 60 80
2 <5 ---
Me
O OH
1 syn 24 67
anti 35 90
2 syn 21 69
NO2 anti 41 89
Barbas, C. JACS, 2001, 123, 5260.

Direct Aldol Addition 4


Substrate scope:
use of α-unbranched aldehydes

O O 20 mol% L-proline O OH O
+ +
Me Me H R CHCl3, rt, 3-7 d Me R Me R
20 mol% 1 2

product R= % yield 1 % yield 2 % ee product R= % yield 1 % yield 2 % ee

Me
Me 29 0 70 23 46 61
31 38 67 * Me 34 42 73 *

Me Me
Me 35 40 73 * 22 50 36
Me
34 35 72
* reaction performed neat in acetone

use of cyclic ketones (cyclopentanone, cyclohexanone) result in moderate yield and


diastereoselectivity, and up to 95% ee
enone products arise from a Mannich addition-elimination sequence
List, B. OL 2001, 3, 573.

09-10 11/20/03 3:29 PM


Direct Aldol Reaction Mechanism
Me
HN
+H2O N N
R2 + H H H
R2
O -H2O O
O Me Me O
HO O HO

R1CHO

H O
H
R2 R2
+H2O N O
HN R1 Me R1 N H
H + H O H
O -H2O O Me
OH O OH Me R1
HO O H
R2

previously proposed T.S.:


metal-free Zimmerman-Traxler model Houk's calculated T.S.

H
synclinal approach of aldehyde
N R1 in pseudo-eqitorial position
R1 R2 H
O H
Me O O C-H - - O distance ~ 2.4 Α
H DFT calculations in DMSO

List, B. JACS 2000, 122, 2395.


List, B., Houk, K. JACS 2003, 125, 2475.

Me Me
Synthesis of Anti-1,2-Diols
S
CO2H CO2H
N N
O OH H H
O O 20 mol% catalyst
1 2
Me R
Me H R DMSO/acetone 4:1 OH
OH 24-72 hr, rt

product R = cat. dr % yield % ee product R = cat. dr % yield % ee

1 >20:1 60 >99 1 1:1 83 80 (n.d.)


2 >20:1 45 95 2 1:1 52 95 (50 )

Me
1 >20:1 62 >99 1 3:1 62 79 (33)
Me 2 --- <5 --- 2 --- <5 ---
Me
1 >20:1 51 >99 Me 1 1.7:1 38 >97 (84 )
2 --- <5 --- Me 2 --- <5 ---
*
Me 2:1
Cl
1 3:2 95 67 (32) O 1 2:1 40 >97 (97 )
O 2 --- <5 ---
2 3:2 60 92 (78)
Me Me

more substituted enamine formed due to:


List, B. JACS, 2000, 122, 2395.
increased acidity of proton removed
Barbas, C. JACS, 2001, 123, 5260.
increased stability of enamine due to On.b. --> π* C=C

11-12 11/20/03 3:30 PM


Use of Aldehydes as Donors in Direct Aldol

O OH
O O 10 mol% L-proline
R1 H R
H H R2 DMF, 11-26 hr, 4ºC
OH
2 equiv.

product R = dr % yield % ee product R = dr % yield % ee


O OH O OH
Me 4:1 80 99 Me 24:1 82 >99
H H
Me Me Me
O OH Me O OH
H Me 3:1 88 97 Me 24:1 80 98
H
Me Bu Me
O OH
O OH
H 14:1 87 99 Me 19:1 75 91
Me H
Bn Me
O OH

H 3:1 81 99 reaction requires lower catalyst loading, shorter


Me times, and only 2 equivalents of aldehyde donor

MacMillan, D. JACS, 2002, 124, 6798.

Trimerization of Acetaldehyde

O 0.5 mol% L-proline O OH


3
Me H THF, 0ºC, 5hr H Me
10% 90% ee

THF at 0ºC was found to be the optimal conditions


for yield and ee (DMSO @ rt = 13% y, 57% ee,
CHCl3 @ rt = 2% y, 68% ee)

Mechanism:

O -H2O
HN Me N N
H Me +H2O CO2
CO2H H CO2H

Me H
O
O OH H Me
HN Me N
H Me Mannich condensation CO2
CO2H OH

Barbas, C. JOC, 2002, 67, 301.

13-14 11/20/03 3:31 PM


Propionaldehyde Trimerization
A method for carbohydrate assembly
Me
O OH OH HO O
Me
H
Me Me
Me Me
O OH
10 mol% L-proline
3 Me 6% 1:2 α:β,
H 11% ee
DMF, 4ºC, 3d Me
O OH OH HO O
Me
H
Me Me
Me Me
OH
47%

reaction analygous to an aldolase enzyme that furnishes the minor product shown above
propionaldehyde added slowly dropwise in order to obtain trimer over dimer products
enantioselectivity erodes with longer reaction times (after 10 hr product ee = 47%)
substituent at C-6 variable by using 1 eq. of corresponding aldehyde and 2 eq. of propionaldehyde

Barbas, C. TL, 2002, 43, 9591.

Mechanism of Propionaldehyde Trimerization

O OH O OH
O O L-proline L-proline
Me Me
Me Me H H
H H
Me Me

1 2

O O
L-proline Me L-proline Me
H H

O OH OH O OH OH
Me Me
H H
Me Me Me Me

incubating isolated 1 with L-proline led to formation of 2 through


epimerization (1:1 ratio of 1:2 after 96 hr)

Barbas, C. TL 2002, 43, 9591.

15-16 11/20/03 3:34 PM


Aldehyde Aldol Addition to Activated Carbonyl Compounds

O O OH
O 20 mol% L-proline
CO2Et
H H
EtO2C CO2Et CH2Cl2, 1-3 hr, rt CO2Et
R R

product R = % yield % ee
H O
H
Me 90 90
H N O
Et 91 85 O H
H
EtO2C CO2Et
i-Pr 88 85
R
94 88

n-Hex 91 84

Ph 97 0
protection of the aldehyde as the dioxolane
Jorgensen, K. Chem. Comm. 2002, 620. prevents epimerization of the α center
during column chromatography

Other Chiral Amine Catalysts for the Direct Aldol Addition

Chiral diamines: N N

N N
H H
1 2

O O catalyst O OH O

Me Me H R 30ºC, 2hr Me R Me R
3 4

product R = cat. % yield 3 % ee % yield 4

1.5 mol% 1 2TfOH + 60 88 7


1.5 mol% 1
NO2
3 mol% 2 72 93 7

1.5 mol% 1 2TfOH + 37 83 32


1.5 mol% 1
3 mol% 2 13 91 25

proposed mechanism similar to that of proline catalyzed reactions, with


proton transfer from protonated tertiary N to O
Yamamoto, Tet. 2002, 58, 8167.
17-18 11/20/03 3:35 PM
Mannich Reaction: First Report
Required Conditions:
enamime addition must be faster to the imine than to the corresponding aldehyde
formation of the aldimine from a primary amine must be faster than the aldol addition
NMR studies show that Keq(aldehyde imine) = 1

O O
O OH O R'NH2 NR' O NHR'

R kAldol H R Keq=1 H R kMannich R

3-component reaction verifies hypotheses:


OMe
CHO OMe
O 35 mol% L-proline O HN (+ <20% aldol
product)
Me Me acetone/DMSO 1:4 Me
NO2 NH2 12 hr
50% NO2
20 vol% 1 equiv. 1.1 equiv. 94% ee

10 mol% proline and 1.3 eq ketone used without loss of efficiency


List, B. JACS, 2000, 122, 9336.

Mannich Reaction: Scope


Variation of the ketone donor:
CHO PMP
35 mol% L-proline O
O HN
1.1 eq. p-anisidine
Me Me
R
acetone/DMSO 1:4 R
NO2 12 hr, rt NO2

R= product % yield dr % ee

PMP
O HN
Me 2.5 95 >20:1 99
Me Ar
Me
PMP
O HN
Me 1 --- >20:1 94
Ar
PMP
O HN
OMe 93 >20:1 98
Me Ar
OMe
PMP
O HN
OH 92 >20:1 >99
Me Ar
List, B. JACS, 2000, 122, 9336. OMe

19-20 11/20/03 3:36 PM


Mannich Reaction: Transition States

O N CO2H Aldol
Mannich
R H Me
X

H O
H O
MeO
non-bonding interactions govern N O
N O which diastereoface of H
O
N H electrophile is favored Me
Me
R H
H R X
X

ArHN O OH O

R Me R Me
X X
List, B. JACS 2002, 124, 827.

Mannich Reaction: Scope 2


Variation of the aldehyde:

aliphatic aldehydes, including α-unbranched are good substrates (60-90% yield, 73-93% ee)
aromatic aldehydes are excellent substrates, (79-92% yield, 61-99% ee)

Effect of electron donation from the aldehyde:

O NHPMP R= % yield dr % ee
CN 88 15:1 99
Me H 83 9:1 93
OH Me 85 5:1 86
R OMe 88 3:1 61
Variation of the catalyst:

proline proves to be the best catalyst, with other catalysts affording reduced yield and optical purity.

Reaction of acetone with isovaleraldehyde:

S N
CO2H N
H
N
H
List, B. JACS 2002, 124, 827. 60% y., 16% ee 26% y., 0% ee

21-22 11/20/03 3:37 PM


α -Imino Ethyl Glyoxylate as Mannich Acceptor 1
An entry to α-amino acids
Addition of ketones:
O PMP O NHPMP
N 20 mol % L-proline
R1 OEt R1 CO2Et
H DMSO, 2hr, rt
R2 O R2
20 vol%

product % yield dr % ee

O NHPMP
86 --- 99
Me CO2Et
R2 =
O NHPMP
Me 72 >19:1 >99
Me CO2Et
R2 allyl 79 >19:1 >99

OH 62 >19:1 99
O NHPMP
Me 47 >19:1 >99
CO2Et
Me
O NHPMP
81 >19:1 >99
CO2Et

Barbas, C. JACS 2002, 124, 1842

α -Imino Ethyl Glyoxylate as Mannich Acceptor 1


An entry to α-amino acids
Addition of aldehydes:
O PMP O NHPMP
N 5 mol % L-proline
H OEt H CO2Et
H dioxane, 2-24hr, rt
R O R
20 vol%

R= % yield dr % ee

Me 72 1.1:1 99

Et 57 1.5:1 99

i-Pr 81 >10:1 93

n-Bu 81 3:1 99

n-Pent 89 >19:1 >99

71 >19:1 >99

aqueous workup or column chromatography may lead to decreased diastereoselectivities


reaction has been performed in aqueous media (Barbas, TL 2003, 44, 1923)

Barbas, C. JACS 2002, 124, 1866.

23-24 11/20/03 3:38 PM


Anti-Selective Mannich Reaction
Addition of aldehydes:

O PMP 20 mol% O NHPMP


N N OMe
H
H OEt H CO2Et
H
DMSO, 24-48h, rt
R O R
20 vol%

proposed transition state


R= % yield dr % ee
MeO H
Et 44 1:1 75
N
N OMe
i-Pr 52 10:1 82
H
n-Bu 54 10:1 74 H CO2Et
R

t-Bu 57 >10:1 92 bettter?


OMe
n-Pent 78 >10:1 76
MeO H
n-Hex 68 >19:1 76 N
H
For a review of SMP use in asymmetric synthesis, see: N
Enders, D. Synthesis 1996, 1403. H
Barbas, C. TL 2002, 43, 7749. H CO2Et
R

Direct α− Amination 1
Addition of aldehydes:

O Cbz Cbz
N 10 mol% L-proline
N N Cbz
H Cbz HO N
CH3CN, 0ºC-rt, 3hr H
R R
then NaBH4, EtOH
1.5 equiv. 1 equiv.

R= % yield % ee

Me 97 >95
H O
n-Pr 93 >95
BnO2C N O
N H
n-Bu 94 97
N Me
i-Pr 99 96 CO2Bn
X
Bn 95 >95

longer reaction time leads to epimerization, so


aldehyde is reduced in situ
List, B. JACS 2002, 124, 5656.

25-26 11/20/03 3:39 PM


Direct α− Amination 2
Addition of ketones:

O EtO2C O CO2Et O CO2Et


N 10 mol% L-proline
N CO2Et N CO2Et
R1 N R1 N N
CO2Et H
CH3CN, 1-4d, rt H
R2
R2 R2

1 2

product 1 ratio 1:2 % yield (1+2) % ee

O CO2Et
N CO2Et 10:1 80 95 (93)
Me N
H
Me
O CO2Et
N CO2Et 4.5:1 92 98 (94)
Me N
H
Bn
O CO2Et
N CO2Et 3:1 99 99 (99)
Me N
H
i-Pr
O CO2Et
N CO2Et --- 79 94 (93)
Et N
Jorgensen, JACS 2002, 124, 6254. H
Me

α−Oxidation of Aldehydes with Nitrosobenzene 1

The choice of reaction conditions determine N or O selelctive addition:

O
O OH uncatalyzed OX O Lewis acid
N N O Ph
Ph Ph N
X=Li, SnBu3, X=SiMe3 H
SiMe3
Yamamoto, H. OL, 2002, 4, 3579.

Larger basicity of nitrogen allows proline to catalyze O-nucleophilic addition:

O O
O 5 mol% L-proline
O Ph
H N N
Ph CHCl3, 4ºC, 4 h H
R R

H O

N O
N H
H
O
X

a possible transition state


MacMillan, D. JACS 2003, 125, 10808.
27-28 11/20/03 3:40 PM
α−Oxidation of Aldehydes with Nitrosobenzene 2
Aldehyde scope:

O O
O 5 mol% L-proline
O Ph
H N H N
Ph CHCl3, 4ºC, 4 h H
R R
3 equiv. 1 equiv.

R= % yield % ee R= % yield % ee

Me 88 97 Bn 95 97

n-Bu 79 98 Ph 60 99

i-Pr 85 99 (CH2)3OTIPS 76 98

CH2CH=CH2 99 96
CH2-(3'-N- 83 98
methyl indole)

product most easily isolated as the primary alcohol (NaBH4 reduction)

MacMillan, D. JACS 2003, 125, 10808.

Asymmetric Organocatalysis of the Michael Reaction

Two mechanistic possibilities exist:

N N
EWG or :Nu

enamine imminium

Examples include:

additions to: additions of:


alkylidene malonates malonate esters
α,β−unsaturated nitroalkenes nitroalkanes
aromatics (Friedel-Crafts reactions)
silyloxy furans
Diels-Alder reaction
Dipolar cycloaddition

29-30 11/20/03 3:41 PM


Michael Additions using Enamine Catalysis:
Moderate Success has been Achieved

O O
H
S 100 mol% L-proline S

DMF, -15ºC, 7d
O Me O
81% H
28% ee

Kozikowski, A. JOC, 1989, 54, 2275.

O O
100 mol% L-proline
Me Me
BnN BnN
CO2Et DMF, rt, 7d CO2Et
45%
34% ee

Momose, T. J.Chem.Soc., Perkin Trans.,


1992, 509.

Enamine Catalysis: Examples 2


Recent examples:
List: NO2
O 15 mol% L-proline O
NO2
Ph
Me Me DMSO, 24 hr, rt Me Ph
97% 7% ee
List, B. OL 2001, 3, 2423.
Enders:
NO2
O 20 mol% L-proline O
NO2
Me Me Ph
MeOH, 24 hr, rt Et Ph
74% Me

73% ee, dr=7.3:1


use of MeOH as solvent increases ee

H O
O

N
N O
O H
Me
Enders, Synlett 2002, 26. Ph H
X

31-32 11/20/03 3:42 PM


Enamine Catalysis: Examples 3
Recent examples:
Barbas:
O O O 20 mol% N EtO2C CO2Et
N O
H
Me Me EtO OEt Me Ph
THF, rt
Ph 59% 47% ee
Barbas, C. TL, 2001, 42, 4441.

N
20 mol% N NO2
O H O O
NO2
R Ph
H Et Ph
THF, rt
85% R
R = Me 56% ee, dr=9:1
proposed transition state: R = i-Pr 72% ee, dr=11:1

R2N

O2N N

H
H Ph
R

Barbas, C. OL 2001, 3, 3737.

A Highly Enantioselective Michael Addition Using Enamines


A New Chiral Diamine Catalyst

N N syn/anti
15 mol% H
O NO2 R yield dr ee
Me O
OR Ph Me
Me NO2 Ph Me 75 1:5 69
CHCl3, rt, 7d H 79 5:1 98
10 equiv. OR

HH

N H N NO2
O
Me Me Me
Ph
H

with variation of aromatic group on nitroolefin:


ee = 96-98%
dr = 3.5:1 - 19:1
selection of aromatic groups used:
tolyl, p-methoxyphenyl, p-chlorophenyl, 2-thienyl
Alexakis, A. OL, 2003, 5, 2559.

33-34 11/20/03 3:43 PM


Imminium Catalysis of Conjugate Additions 1

Proline has been used with only mild success:

O
CO2Rb
O
O O 5 mol% N
H
CHCl3, rt CO2i-Pr
i-PrO Oi-Pr
91% CO2i-Pr

58% ee

Proline rubidium salt gives lower ee in the Hajos-Parrish-Weichert reaction

CO2Rb
N O Me
O 5 mol% H NO2
Me
Me Me NO2 CHCl3, rt
Me Me
74%
68% ee
Yamaguchi, JOC 1996, 61, 3520.

MacMillan Introduces A New Catalyst


Imminium ion formation lowers the LUMO of the system and allows catalysis to occur:
LUMO LUMO
versus

E
HOMO HOMO

L.A.
O O N O

Consensation of an aldehyde with the catalyst produces an imminium complex:


O Me

0000
00
00
N

N Me
Me
O Me 00
Ph H N
Me
N Me
Ph
O H

H :Nu

PM3 minimized structure


35-36 11/20/03 3:44 PM
Diels-Alder Cycloaddition 1

H 5 mol% 1
R CHO
R O MeOH-H2O, rt
CHO R
12-24 hr endo exo

Dienophile scope:

R= % yield endo:exo % ee(endo) % ee(exo)

Me 75 1:1 86 90 O Me
N
n-Pr 92 1:1 86 90 Me
N Me
Ph H
i-Pr 81 1:1 84 93 HCl
1
Ph 99 1.3:1 93 93

Furyl 89 1:1 91 93

MacMillan, D. JACS 2000, 122, 4243.

Diels-Alder Cycloaddition 2
R
H 20 mol% 1 CHO endo
R O X MeOH-H2O, rt adduct
X
12-24 hr
Diene scope:

diene R product % yield exo:endo % ee


Ph O Ph

O Me CHO 75 35:1 96
Ph Me
Ph

H 82 1:14 94
CHO O Me
Me N
Me
84 --- 89 Me
H
CHO N Me
Ph H
Ph R 90 --- 83 HCl
Ph H 1
Me 75 --- 90
CHO

Me Me Me Me
H 75 1:5 90
CHO
OAc OAc
H 72 1:11 85
CHO
MacMillan, D. JACS 2000, 122, 4243.
37-38 11/20/03 3:45 PM
Application to Complex Synthesis
Me
CHO
O Me
N Me
CHO Me
Me N Me N
Ph H Ts
HCl
exo (92% ee)
N Me
40 mol%
Ts DMF/MeOH (1:1)
5% H2O, 36 h
35% yield,
Me
70 de (endo)
%
NTs

Me CHO
Me
SCN Me endo (93% ee)

NTs

Kerr, M. JACS 2003, ASAP


(+)-Hapalindole Q

Nitrone Cycloaddition
Z Z
Z H 20 mol% catalyst N O N O
N R
R R1 R R1
O R1 O MeNO2-H2O
-20ºC CHO CHO
endo exo

Z R R1 endo:exo yield ee (endo) O Me


N
Bn Ph Me 94:6 98 94 Me
allyl Ph Me 93:7 73 98 N
Me Ph Me 95:5 66 99 Ph H Me
HClO4
Bn C6H4Cl-4 Me 92:8 78 95
Me C6H4Cl-4 Me catalyst
93:7 76 94
Bn C6H4OMe-4 Me 98:2 93 91
Me C6H4Me-4 Me 93:7 82 97
Bn 2-napth Me 95:5 98 93 HClO4 proved to be the best Bronsted acid
Bn c-Hex Me 99:1 70 99 cocatalyst to promote only enantioselective
Bn Ph H 81:19 72 90 catalysis
Bn Ph H 86:14 80 92 high endo selectivity attributed to to
Bn C6H4Cl-4 H 85:15 80 90 favorable placement of R group away from
Bn C6H4Cl-4 H 80:20 80 91 geminal dimethyl substituents on catalyst
Bn 2-napth H 81:19 82 90
Bn C6H4OMe-4 H 91:9 83 90

MacMillan, D. JACS 2000, 122, 9874


38b-39 11/20/03 3:46 PM
Friedel-Crafts Alkylation 1: Pyrroles

H
20 mol% catalyst
O
R O N
N THF-H2O, 3-5 d
Me Me
R H

O Me
R temp(ºC) yield ee N
Me -60 83 91 Me
N
n-Pr -50 81 90 Ph H Me
i-Pr -50 80 91 TFA
Ph -30 87 93 catalyst
PMP -30 79 91
CH2OBn -60 90 87 use of N-benzyl pyrrole and N-allyl pyrrole give
CO2Me -50 72 90 similar results
Substitution on the pyrrole is also possible:
H
20 mol% catalyst
O
Bu N Ph O Bu N 87% y, 90% ee
Me THF-H2O, 3-5 d Me
Ph H
Pr Pr
H
20 mol% catalyst
Ph O O 68% y, 97% ee
N THF-H2O, 3-5 d N
Me Me
MacMillan, D. JACS 2001, 123, 4371. Ph H

Alkylation of Indoles 1
O Me
The need for a new amine catalyst: N Me H
20 mol% Me
N Me O
H Ph H
TFA
N
N Me O CH2Cl2, -40ºC, 2d Me
Me
83% 58% ee

Indole is less electron-rich than pyrrole, so is less nucleophilic toward conjugate addition

Second generation catalyst: a more reactive variant


lone pair exposed
CH3 - lone pair interaction

O Me O Me
N N
Me
Me

Ph
N
H Me Ph
N
H
Me
Me 00 000
00
0000 000
00 00 000
000000
00 Kinetic studies indicate rate of reaction influenced by imminium
formation as well as carbon-carbon bond forming event
MacMillan, D. JACS 2002, 124, 1172.
40-41 11/20/03 3:47 PM
Alkylation of Indoles 2
R H

H 20 mol% catalyst O

N R O N
Me THF-H2O, 3-5 d Me

O Me
N
Me An increase in rate of reaction and enantioselectivity:
N Me
Ph H Me
TFA
catalyst

000
O Me

0000
00
N
Me
R temp(ºC) yield ee 000 N Me
Me -83 82 92 Ph Me
n-Pr -60 80 93
i-Pr -50 74 93
Ph -55 84 90
CH2OBz -83 :Nu
84 96
CO2Me -83 89 91 increased top-face coverage
nucleophile-geminal dimethyl interation removed
MacMillan, D. JACS 2002, 124, 1172.

Z
Alkylation of Indoles 3 R H
Z
H 20 mol% catalyst O

Y N R1 O N
CH2Cl2-i-PrOH, 3-24 hr Y
R R

Indole Scope:

R Y Z R1 temp(ºC) % yield % ee
Me H H Me -87 82 92
H H H Me -60 72 91
allyl H H Me -72 70 92
Bn H H Me -60 80 89
H H Me -CH2OBz -60 94 94
Me H OMe -CH2OBz -87 90 96
H Cl H -CH2OBz -60 73 97

Application to Simple Synthesis: Me


MeO CO2H
MeO
Me 1. 20 mol% catalyst Me
N
N
H
87% ee, 82% y.
Me O
2. AgNO3, NaOH COX-2
Br inhibitor
Br
MacMillan, D. JACS 2002, 124, 1172.
42-43 11/20/03 3:48 PM
Alkylation of Benzenes
Z Z X H
H 10 mol% catalyst
O
X O CH2Cl2, 2-4 d
R2N R2N

OMe
OMe

N N
Me2N
1 2 3 O Me
N
aniline X temp(ºC) % yield % ee Me
N Me
1 Me -40 86 89 Ph H Me
2 Me -20 70 87 HCl
1 Et -50 68 88 catalyst
1 CH2OBz -20 89 92
2 CH2OBz +20 73 90
1 CO2Me -20 90 92 other substituted anilines used with
2 CO2Me -20 97 97 similar results
3 Ph -50 82 84 catalyst loading can be lowered to 1%
3 p-ClPh -10 87 92 without significant loss of yield and
1 p-NO2-Ph -10 87 92 enantioselectivity
2 p-NO2-Ph -20 82 90
MacMillan, D. JACS 2002, 124, 7894.

Mukaiyama-Michael Reaction 1
Previous Michael additions with silyloxy furans:
O
O O
O O H
TMSO Chiral Lewis Acid
Me N O N O
(bisoxazoline or
Me O O
pyridyl bisoxazoline
ligands) Katsuki, Tet. 1997, 53, 17015
Desimoni, G. Tet. 2001, 57, 10203

note that Lewis acids promote 1,2-addition products when possible, such as α,β−unsaturated enals

Optimized reaction conditions:


O

TMSO O Me
H 20 mol% catalyst O Me
O
R O CH2Cl2, 2 equiv H2O
R H
R temp (ºC) % yield syn:anti % ee O Me
Me -70 81 22:1 92 N
Me
n-Pr -50 87 31:1 84 Me
N
i-Pr -20 80 7:1 98 Ph H Me
Ph -40 77 1:6 99 DNBA
CH2OBz -70 86 20:1 90 catalyst
CO2Me -60 84 11:1 99
MacMillan, D. JACS 2003, 125, 1192.
44-45 11/20/03 3:49 PM
Mukaiyama-Michael Reaction 2
Variation of the silyloxy furan
O

TMSO O R
H 20 mol% catalyst O R
X O
Me O CH2Cl2, 2 equiv H2O
X Me H

R X % yield syn:anti % ee

H H 87 8:1 90

Me H 80 22:1 92

Et H 83 16:1 90

CO2Me H 86 6:1 98 TFA as cocatalyst


CO2Me H 83 1:7 98 TfOH as cocatalyst

Me Me 73 24:1 90

MacMillan, D. JACS 2003, 125, 1192.

Another Chiral Amine Catalyst


Asymmetric Michael Additions
NMe
10 mol% BnO2C CO2Bn
O Bn CO2H O
N
BnO2C CO2Bn H
R1 R2 R1 R2
neat, rt

R1 R2 % yield % ee
NMe

p-NO2Ph Me 84 89 N CO2H
p-NMe2Ph Me 58 77
2-furyl Me 75 92 Me
2 -pyridyl Me 95 88
n-Bu Me 61 91
i-Pr Me 33 84 :Nu
MeO2C Me 59 59
Ph Me 86 99
Ph Et 66 95
Ph i-Pr 2 94

Nitroalkane additions to α,β−unsaturated ketones has also been performed


in good to excellent selectivity (Jorgensen, K. JOC 2002, 67, 8331.)

Jorgensen, K. ACIEE 2003, 42, 661.

46-47 11/20/03 3:50 PM


A Listing of Other Asymmetric Organocatalytic Reactions
O Me
[4+3] cycloadditions:
N O
20 mol% Me
N Me
OTMS O CHO
Me Me Ph H Me O
TFA Me Me
CHO
CH2Cl2, -78ºC, 4d
endo only (89% ee)
64%

Harmata, M. JACS, 2003, 125, 2058.


Michael Reactions:
Ph H
N OH R O
OH
O
10 mol% CO2H
Ph N Me
H
R Me
O O CH2Cl2, rt O O
75 - 88% ee
Jorgensen, K. ACIEE 2003, 42, 4955
Tandem Knoevenegel-Diels-Alder Reactions:
Me Me
S Me
Me Me CO2H
O O p-NO2Ph O O Me
N O
O O H
Me Ph H
O O
MeOH, rt, 4 d O
O2N 88% O Ph
86% ee
Barbas, C. ACIEE 2003, 42, 4233

Summary
Reactions are direct:
Donors can be used without modification -- no need to deprotonate or silylate prior to reaction
Electrophiles can be generated in situ (Mannich reaction) most of the time

Catalysts are:
inexpensive
commerially available or easily prepared in both enantiomeric forms
non-toxic
recoverable

Many reactions can be run at room temperature, under an aerobic atmosphere, with wet solvents

Many types of reactions can be catalyzed; for some reactions, organocatalysis is the only highly
efficient way known (Mannich and Mukaiyama-Michael additions)

Reaction yield and enantioselectivity is highly dependent on solvent system so require


"fine tuning"

Only reactions that use ketones or aldehydes as donors (electrophiles for Michael additions)
can be catalyzed

Organocatalysis using small molecules is a field that has emerged


only within the past decade. It is bound to receive increasing
attention in the future; as a result, new catalysts will emerge which will allow
for the catalysis of reactions previously unutilized in the realm of organocatalysis.

48-49 11/20/03 3:51 PM


D. A. Evans Ambiphilic Functional Groups–3: Sulfur-Based Activating Groups Chem 206

Relevant Background Reading


http://www.courses.fas.harvard.edu/~chem206/
General: Simpkins, N.S. Sulphones in Organic Synthesis,
Pergamon Press, New York, 1993.
Chemistry 206 General: Magnus, P.D. Tetrahedron 1977, 33, 2019.

Julia: Blakemore, J. Chem. Soc. Perkin Trans I. 2002, 2563.


Advanced Organic Chemistry
Electrophilic Properties: Trost, B.M. Bull.Chem. Soc. Jpn. 1988, 61, 107.
Lecture Number 29 SO2 Extrusion: Vogtle, F.; Rossa, L. ACIEE 1979, 18, 515.

Ramberg-Bäcklund Rxn: Paquette, L.A. Org. Reactions 1977, 25, 1.


Ambiphilic Functional Groups–3
Sulfur-Based Activating Groups Triflones: Hendrickson, J.B. Org. Prep. Proc. Int. 1977, 175.

■ Sulfur-Ylides Sulfoximides: Johnson, C.R. Tetrahedron 1984, 40, 1225

■ Sulfur-Stabilized Carbanions: Structure


Cum Question, 1998: The stereoselective construction of trans olefins through
carbanion-mediated condensation processes has still not been rendered general. One
■ Sulfone-Based Transformations transformation that may be used in certain circumstances is the "one-step" Julia
transformation illustrated below. PPProvide a mechanism for this transformation.
■ Pummerer Rearrangement RA
O
LiN(iPr)2 S
S RS RACHO
RS
+ O
S +
Reading Assignment for this Week: THF N
N O H –78 → 25 °C H
1
RS
Carey & Sundberg: Part A; Chapter 7 + SO2
RA
Carbanions & Other Nucleophilic Carbon Species
Carey & Sundberg: Part B; Chapter 2 The cruel mechanistic problems that you should be prepared for in Chem 206
Reactions of Carbon Nucleophiles with Carbonyl Compounds O
O
MeO S Ac2O/HOAc
"Chemical Chameleons: Organosulfones as Synthetic Building Blocks" O Et CO2Me
B. M. Trost, Bull. chem. Soc. Japan, 1988, 61, 107-124 (handout) 70%
MeO N MeO
N
O
O
Monday, MeO
D. A. Evans November 24, 2003 CO2Me
Padwa et al. JOC 1996, 61, 4888
29-00-Cover Page 11/24/03 8:32 AM
D. A. Evans Sulfur-Based Functional Groups-1 Chem 206

Relevant Background Reading Reactions of Sulfonium Ylids


R S N2 R +
General: Simpkins, N.S. Sulphones in Organic Synthesis, ■ Synthesis: S Me I S CH3 I –
+

●●
Pergamon Press, New York, 1993. R
R
Sulfonium Salt: pKa ~ 18
General: Magnus, P.D. Tetrahedron 1977, 33, 2019. (+)
O Me R2S C
Julia: Blakemore, J. Chem. Soc. Perkin Trans I. 2002, 2563. NH2
S
HO
Electrophilic Properties: Trost, B.M. Bull.Chem. Soc. Jpn. 1988, 61, 107. O Me N N
R NH2 S N2
O NH2 S
SO2 Extrusion: Vogtle, F.; Rossa, L. ACIEE 1979, 18, 515. HO + N N
–O P O N N O
NH2
Ramberg-Bäcklund Rxn: Paquette, L.A. Org. Reactions 1977, 25, 1. O
N N
O OH OH
Triflones: Hendrickson, J.B. Org. Prep. Proc. Int. 1977, 175.
(–)
S-Adenosylmethionine
Sulfoximides: Johnson, C.R. Tetrahedron 1984, 40, 1225 R2S C
OH OH
R Na H R
■ Deprotonation: S CH3 S CH2 H H
Acidities of Sulfur-based Functional Groups R R pKa ~ 38
Bordwell, F. G.; Zhang, X.-M. Acc. Chem. Res. 1993, 2 6, 510-17. pKa ~ 18

pKa (DMSO) O O
Na H
CH3 S CH3 CH3 S CH2
CH4
pKa (~56) CH3 CH3
CH3 S CH3 Sulfide (45)
NH3
O pKa (~41) ■ Leaving Group Potential: R2S C(+)
CH3 S CH3 Sulfoxide ~35
L L
S N2
O L S CH3 + Nu: L S: + Me Nu
CH3 S CH3 Sulfone ~ 31 HOH L (+) L
O pKa 31
CH3
S CH3 Sulfonium Salt 18.2 R S N2 R
CH3 Excellent LG S CH3 + Nu: S + Me Nu

●●
R R
O
CH3 S CH3 Oxo-Sulfonium Salt
O O
CH3 (–) S N2
Reactivity Pattern Good LG R S CH3 + Nu: R S + Me Nu

●●
R2S C Nonalternate
(+) O O
29-01 Sulfur chem-1 11/23/03 6:07 PM
D. A. Evans, K. Scheidt X-ray Structures of Metallated Sulfones & Sulfoxides Chem 206

Sulfone- & Sulfoxide Based Carbanions: Structure O Me Boche, etal. Angew. Chem. Int. Ed. 1986, 25, 1101

■ Sulfone- and sulfoxide-stabilized carbanions are extremely useful carbon S C–Li


nucleophiles in organic synthesis.

O O O + TMEDA
LDA El(+)
Ar S CH3 Ar S CH2–Li Ar S CH2
O O O El
O O O
LDA El(+) Li
Ar S CH3 Ar S CH2–Li Ar S CH2
El

However, until recently little information was available on the solid state Li
structures of these species:
"The Structure of Lithium Coumpounds of Sulfones, Sulfoximides, Sulfoxides,
Thioethers, 1,3 Dithianes, Nitriles, Nitro Compounds, and Hydrazones."
Boche, G. Angew. Chem., Int. Ed. Engl. 1 9 8 9, 2 8, 277.

Here are several examples taken from the Boche review:

O
S CH2–Li–[TMEDA]
O

Li

Gais, etal.
Angew. Chem. Int. Ed. 1985, 24, 859 ■ The Li counterions are not associated with the charged carbon.
■ The carbanions are largely trigonal.

29-02 Sulfone anion-Xrays 11/23/03 5:16 PM


D. A. Evans, K. Scheidt X-ray Structures of Phenylsulfinyl Carbanions Chem 206

O Me O

S C–Li S CH–Li
O

+ TMEDA + TMEDA

Li

Li

Li

Li

Li

Boche, etal. Angew. Chem. Int. Ed. 1986, 25, 1101 Boche, etal. Angew. Chem. Int. Ed. 1985, 24, 573

29-02a Sulfone anions-Xrays-2 11/23/03 5:16 PM


D. A. Evans Sulfur-Based Functional Groups-2 Chem 206
(±)
■ Reactions with ketones: Reactivity Pattern: Nonalternate Reactions of Sulfones
R2S C
Synthesis:
Me Me
O PhS: – HIO4
O Me H Me Ph
S C CH2 S Me Br Me S
Me H Me
H2O2 Me
Ph
Corey & Chaycovski, JACS 1965, 87, 1353-1364. (Handout) Me S
Me O
O
O Me H Me Ph H2O2
(–) Me S
S C CH2 S R2S C O O
Me H Me
A
Me O Me O
O O (+) :S Ph
Me Me Br S
R2S C Me O Ph
CH2 O–
CH2 S
(Sulfinate anion) Sulfinate ester
Me not observed
"Twenty-five Years of Dimethylsulfoxonium Methylide (Corey's Reagent).", Good review article: Magnus, Tetrahedron 1977, 33, 2019-2045.
Gololobov, Y. G.; Lysenko, V. P.; Boldeskul, I. E. Tetrahedron 1987, 43, 2609.
(electronic handout)

Reactions of OxoSulfonium Ylids: Conjugate Addition Reactions of Sulfones


Me
Me Li
O O Ph BuLi R +
Me (+) O Me S Ph –
●●

S CH2
Me O Me S more nucleophilic S CH2
S O O than:
Me O O R
Me pKa ~ 25
(+)
B
R2S C
Me R +
O (–)
(–) Ph poorer leaving S CH3
R2S C
Me O R2SO2 C Me S R
S O group than:
O O
Me

(+)
Nonalternate reactivity pattern revealed in consecutive reactions
R2SO2 C
O O
A O
CH2 B Me Li
(+) O 1,2- vs 1,4-addition ??
Ph ??
Me S + (+)
Will function as LG ??
O O
29-03 Sulfur chem-2 11/23/03 6:32 PM
D. A. Evans, P. Carter Sulfur-Based Functional Groups-3 Chem 206

Me Li Me SO2Ph The Sulfone group may also be readily removed reductively:


(+) O
Ph (+)
Me S Me
O BuLi R SO2Ph Na(Hg) R H
O –O R SO2Ph
El(+) MeOH
1,2-addition El El
(–)
R2SO2 C –PhSO2 – Fragment Coupling with Sulfonyl Carbanions
Me OH
Alkoxide not sufficiently nucleophilic to
MEMO
displace PhSO2– anion. Me Heathcock, C.H.; et al.
O
Me Me J.Org.Chem. 1988, 53, 1922.
Not observed
However!! 1. MsCl, TEA
2. PhSLi, THF, RT
Me Li Me SO2Ph 3. mCPBA SO2Ph 2 eq. nBuLi
Me O
Ph Me THF/HMPA;
MEMO
Me S Me OTBS
Me OEt Me then add iodide
O O OEt Me Me I Me Me

Industrial synthesis developed by M. Julia OLi


X OTBS
Me H CO2Et MEMO 65 - 85% yield
(–) (+) Me H Me Me Me Me
R2SO2 C R2SO2 C
Me Me
X= SO2Ph
trans chrysanthemic acid Li wire, Na2HPO4 50% yield
THF/HMPA/tBuOH
X=H

Synthesis of Vitamin A: Julia & Co-workers, Bull. Soc. Chim. Fr. 1985, 130
H Me
TBDPSO S
Me SO2Ph SO2Ph TBDPSO
Me Me Me Me Me Me O
CO2R MeO PMBO S SO2Ph
Li (+) MeO Me
Me PhO2S
Me H S
Me HO
CO2R Smith, A.B. III; et al.
(–) Tet.Lett. 1989, 30, 6963. PMBO S
Br KOH/MeOH
R2SO2 C
77% yield TFAA, DMSO; NEt3
TBDPSO CH2Cl2, -78 oC
TBDPSO
Me Me Me Me
Al-Hg, aq. THF SO2Ph
CO2H MeO Me Reflux
(+) H MeO Me
R2SO2 C S S
O 90% yield O
Me
PMBO S PMBO S
29-04 Sulfur chem-3 11/23/03 6:03 PM
D. A. Evans, T. Dunn Sulfur-Based Functional Groups-4 Chem 206

Functionalization of cyclic Ethers Total synthesis of Okadaic Acid


Me Me

BuLi El(+) n-BuLi, DMPU


El I
THF, -78o C +
PhO2S O O O PhO2S O PhO2S O O O
SO2Ph SO2Ph
OH
(–) –SO2Ph Lewis acid H
R2SO2 C Me O 1.) Addition of
O iodide,
H H OH -78o C to RT
OH O
ROH O H 2.) CSA, MeOH
Me
(+) El O El O
Me H
R2SO2 C OR O Me Me
O H
H O HO O
COOH
Ley et al, Synlett, 1992, 395; Ley et. al, Tetrahedron, 1992, 48, 7899 O
Me OH Me 90% yield

Total Synthesis of Routiennocin (CP-61,405) Ley et al, J. Chem. Soc., Perkin Trans. 1, 1998, 3907.

Me Me Total synthesis of Bryostatin 2


OBn n-BuLi,
I OTBDMS OPMB O
DMPU Me
+
PhO2S O O R O O Me
H THF, -78oC PhO2S H N–iPh
OTf Li
C Ring O O OTBDMS 87% yield
H H PhO2S H
1.) Addition of iodide,
Me -78o C → RT OTBDMS OPMB O
H 2.) H+, H2O Me
O
N Me Me Me NHPh
O HO
OH
H O H MeO2C
Me O C Ring O O OTBDMS
N Me O O H H OH H
H OBn
Routiennocin HO O
O
O
HOOC H OH H OH Evans et al, JACS. 1999, 121, 7540-7552.
Me Me O O
HO
Me
68% yield Bryostatin 2
O Me OH

n-Pr O CO2Me
29-05 Sulfur chem-4 11/23/03 6:06 PM
D. A. Evans, P. Carter Sulfur-Based Functional Groups-5: Julia Olefin Synthesis Chem 206

First Generation Julia Trans Olefin Synthesis: Julia Olefination - Ionomycin


1. add RCHO, -78 oC;
OH
BuLi add Ac2O, -78 oC → R.T.
R SO2Ph R
R'CHO R' Elimination is stepwise; PhO2S Li 2. Na/Hg, EtOAc/MeOH, –30oC
therefore, not stereospecific
SO2Ph
Ac2O OTBDPS 70% yield
Problem: Work out the
mechanism of reduction step. Me Me 86:14 olefin mixture
Me
OAc O O
Na(Hg) R' H Me H Me
R R Me Me OTBS
R' MeOH R' R O O O
H Me H Me
SO2Ph major Me OTBS Me Me
O O
Me Me
Good sulfone review: Trost, Bull Chem. Soc. Japan, 1988, 61, 107-124. Me
O
Julia Review, Me
CHO
Blakemore, J. Chem. Soc. Perkin Trans I. 2002, 2563. (electronic handout) Evans, et al. OTBDPS
JACS 1990, 112, 5290.
Me Me
The reduction step is not stereosecific Cytovaricin Synthesis: JACS 1990, 112, 7001
Me

R2 H
R2 R2 Me O
+ e- + e- H OCH2OCH2CCl3 TESO Me t-Bu
X X + DEIPSO H
SO2R R2 O 21 HO O
H Si t-Bu
R1 R1 R1 Me CHO Me
R1 H
TBSO + O O
Me H H
SO2Ar TESO MeO
OPMB O O Me
Me OTES
2 LiNEt2, THF H
OAc Ac2O, pyr PhSO2 H
OTES
6% Na(Hg)
-40 °C
Me
Na(Hg) O Reactions accomplished:
H
Me O C=C construction
OH HO
OAc 1
SO2Ar H OTES
DEIPSO H 21 C21 OH deprotection
O 3
H Me
Me H Me Free acid must be used to prevent
H Me
OAc O loss of C4 OH in 2nd step
Kochenski, J. Chem. Soc Perkin Trans I, 1978, 834 TBSO
TESO t-Bu
Si
OPMB O overall yield, 66%
t-Bu
Me H H
MeO
O O Me
OTES
OTES
H
29-06 Julia-1 11/23/03 6:14 PM H
D. A. Evans, P. Carter Sulfur-Based Functional Groups-5: Julia Olefin Synthesis-2 Chem 206

HO The Mechanism:
Phorboxazole B Synthesis 13

O O
9
N H O
H H S
Br 19 H S
H H
46 H 4 CH2 N S N N
H O O
MeO O–Na
O S R
Me O MeO 1 O S I I
Me I O
H
OH A O R CH3 CH3
disconnection 38 H N Me H O R CH3 O S
O Olefin stereochemistry could be
HO
33
O O
MeO Evans, Smith, Fitch, Cee established in the formation of A.
JACS 2000, 122, 10033-10046.

NaO N R
OMe I
Br S CH3
I C39–C46 Synthon
SO2
CH3
Julia Construction

OMe Recent Modifications of the Julia Process:


OMeO O Kocienski, SynLett 2000, 3, 365-366.
Br
Br S NaHMDS I Ph
N O2
CH3 N S C4H9
O KHMDS
S N C
E/Z = >95:5 E/Z: 99:1 (75%)
N N C 4H 9 –60 °C→rt
H I 75%
OHC
THF, -78˚C - rt CH3
NaHMDS
RCHO R
O2
N S KHMDS
N C Ph
C9H19
OMe O–Na OMe N N Ph –60 °C→rt
Br Br OHC C9H19
I I R = Ph: E/Z: 29:71 (70%)
CH3 Ph
S S O CH3 O2 R = tBu: E/Z: <1:99 (95%)
O
Mechanism?? SO2 N S
N C KHMDS
N
NaO N N N –60 °C→rt Me
R O Me R E/Z: >10:1 (64%)
R'
S
R'
H RO
RO Metternich, JOC 1999, 54, 9632
29-07 Julia-2 11/23/03 6:15 PM
D. A. Evans, N. Finney Sulfur-Based Functional Groups-5 Chem 206

Carbonyl Anions: A useful Reversed Polarity Equivalent The overall set of reactions which establishes the equivalency of the
hypothetical carbonyl anion 1 and its equivalent synthon 2 is shown below:
Consider the two possible polar disconnections of the C–R2 bond of the ketone
shown below:
Construction
S Li S El
O Step
T1 El
R1 C + R 2: R + R
O S S
R1 C R2 1 (–) C–G
T2 O
R1 C + R 2: Equivalent H 3O +
Synthons (+) C–E
carbonyl anion
O
Carbonyl anions are not normally accessible via aldehyde deprotonation El
R1 C: T2
+ El O C
O O 2
Mg0 R
R1 C Cl R1 C MgCl not feasible !! Why??

O Nitronate Anions are also useful Carbonyl Anions


Operational equivalents to the carbonyl anion
are useful in synthesis R1 C: R
O –O R
HO –
pKa 18 +N H +N (±)
–O R –O R
1,3-Dithianes as Carbonyl Anion Equivalents (–)
nitronate anion highly stablized
SH –H2O S H R–Li S Li
RCHO Construction El
R R –O H Step
SH +H2O S S
+N + El O2N C
pKa ~ 39 –O R
R (–) C–G
Reactivity Patterns: (RS)2 C(+) (RS)2 C(–)
Equivalent
Synthons Nef Reaction (+) C–E
Latest Innovations: A. B. Smith, JACS 2003, 125, 14435-14445 (Handout)

O
O El
R1 T2
HMPA El(+) + El
S S S S S S S S R C: – O C
R3Si H R3Si Li El R

tBuLi LiO R1 R3SiO R1 R3SiO R1 Dithianes anions highly nucleophilic (indiscriminate):


Nitonate anions higly discriminating
29-08 carbonyl anions 11/24/03 8:44 AM
D. A. Evans Charge Affinity Inversion Operators Chem 30
■ Introduction. As you know, transform T1 conforms to the polar bias
mapped on to the carbon skeleton by =O, while transform T2 does not. Nature's Inversion Operators
Although T1 is the more common transform, sometimes, because of the There is a clear need in nature to have both types of polar bond constructions
presence other functionality in either R1 or R2, the "reversed-polarity" exemplified by Transforms T1 and T2 (Eq 1-2). One such reaction is shown below.
transform is more suitable for the particular synthesis at hand.
T1 O ■ This reaction, which is enzyme-catalyzed, requires the cofactor thiamine which
O R1 C+ + R 2: – Eq 1 functions as the inversion operator in these biological processes.
O O OH O
T2
R1 C R2
R 2: + Eq 2 ■ α-Ketol Transferases: H 2C C H
R1 C: – +
R1
Reversed Polarity Synthon + thiamine (Q:–) OH R1

Ideally, one might visualize a catalytic agent (Q) which might react reversibly This unit is O
OH
with an aldehyde in conjunction with inverting its charge affinity pattern. transferred
O H 2C C
Nature has designed such reagents. H
OH R2
General Scheme R2

■ Charge Affininty Inversion Step: The structural constraints on (Q) are OH O: – OH


O
that it must be nucleophilic, add reversibly to aldehydes, and stabilize an
■ Crucial bond H 2C C: – + H H 2C C
adjacent carbanionionic center.
construction: R2 OH R2
O O– OH OH Q Q
R1 C H + Q: – R1 C H R1 C ●●

Equivalent
Q Q synthons + Q:–
Lets call (Q) a charge affininty inversion operator since in operates on RCHO
and reverses the intrinsic polar reactivity of the RCHO carbon from (+) to (–).
O O O OH
■ Overall Process: O OH ■ Related transform: H 2C C: – + H H 2C C
R1 C H + Q: – R1 C ●●
– OH R2 OH R2

Q ■ The Thiamine Coenzyme (Virtamin B1) & how it functions


The Inversion El(+)
Operator H 2N H acidic proton
●●
– ●●
O OH + R + R
N N S N S N S
R1 C El + Q: –
R1 C El
Me N Me O O Me R Me R
Q
O P O P O– thiamine (Q:–)
■ Benzoin Condensation: O– O– O OH
O OH O H
HOCH2 C
2 C H Ar C ●●
– C C OH H
R1 O: –
OH OH O
Q H C Ar OH H 2C C: –
– HOCH2 C HOCH2 C ●●
– HOCH2 C
:CN OH Q
O :CN – O R1
R R + + H
R +
Cyanide ion is the best example of a reagent which functions as an inversion Equivalent N S N S N S
operator. The benzoin reaction is restricted to aromatic aldehydes. Why? synthons R1
29-09 Inversion Operators 11/24/03 8:50 AM Me R Me R Me R
D. A. Evans The Pummerer Rearrangement Chem 206

Basic Transformation:
Transformations Mediated by the Pummer Rearrangement

O OH
H+ –H2O S R +H2O O R
S R S R
Ar Ar Ar
–ArSH TsOH 86%
H H H S S S
O ∆

Ar O Ar O Ar O
The Pummerer Rearrangement facilitates the transformation of a sulfinyl → aldehyde
transformation. The rearrangement may be initiated by either a Bronsted acid or an
anhydride such as trifluoroacetic anhydride (TFAA). With the latter reagent, the
transformation occurs at room temperature.
O O O
OTFA S S
O – Ph N Ph N
TFAA –TFAOH S R +TFAO S R
S R S R
Ar Ar Ar Ar TFAA
H H OTFA H Et H Et
N N
Leading References R R
81%
O
De Lucchi, Miotti, et al. (1991). “The Pummerer reaction of sulfinyl compounds.” Organic
Reactions 1991, 40: 157. P. Magnus et al. Accts Chem Res. 1984, 17, 35 N
Grierson, and Husson (1991). Polonovski- and Pummerer-type Reactions and the Nef PhS
Reaction. Comprehensive Organic Synthesis. Trost and Fleming. Oxford, Pergamon
Press. 6: 909. H Et
Padwa, A., D. E. Gunn, et al. “Application of the Pummerer reaction toward the N
synthesis of complex carbocycles and heterocycles.” Synthesis 1997 1353-1377. O R
Carreno, “Applications of sulfoxides to asymmetric synthesis of biologically active RHN RHN S
S
compounds.” Chem. Reviews 1995 95, 1717-1760. Ac2O/HOAc
N N OAc
Kita, Y. and N. Shibata (1996). “Asymmetric pummerer-type reactions induced by O >80% O
O-silylated ketene acetals.” Synlett(4): 289-296. CO2Bn
CO2Bn

The Related Polonovski Reaction: The cruel mechanistic problems that you should be prepared for in Chem 206
O O
O OH R
–H2O +H2O O R
H+ R N N R
MeO
O
S
R N R R Et CO2Me
R –R2NH
R R H Ac2O/HOAc
H H MeO N
70% MeO
N
O O
Regioselectivfity: Depends on the relative kinetic acidy of the α protons MeO
CO2Me
Padwa et al. JOC 1996, 61, 4888
29-10-Pummerer 11/20/03 3:06 PM
D. A. Evans The Pummerer Rearrangement Chem 206

Mechanism?? O Exam 3, 2000: Question 5 (11 points). An interesting rearrangement which also
results in the construction of this same ring system (Question 4) has been reported by
O Langlois & coworkers (J.Org. Chem. 1985, 50, 961). This rearrangement is illustrated
MeO S CO2Me below. Provide a mechanism for this transformation.
O Et Ac2O/HOAc
MeO
MeO N 70% N N
O H
N TsOH, THF
O MeO N
Me 78 %
H Et
Padwa et al. JOC 1996, 61, 4888 Me
CO2Me S N O
Et
O O Me SMe
Pummerer

Et Pummerer
O Et S
Ac S
OAc
H Pummerer H H
O N Pummerer N
O N N
Me Me
N Me S
N S
Et Et
O O Me O
O CO2Me O CO2Me H
MeO MeO
OMe OMe
SEt
N
OAc
O H
H SEt H Et N
Product N
Endo [4+2] N O Me
N H
O SMe S
Me
O CO2Me Me O Et
MeO
N
OMe
Endo [4+2] O CO2Me
MeO
O SEt OMe

O O
N
COOCH3
COOCH3
MeO Product
N
OMe MeO O
OMe

29-11-Pummerer-2 11/20/03 3:06 PM


Chemistry 206

Advanced Organic Chemistry

Handout 29A

Overview of the Julia–Kocienski Olefination

Evans Group Seminar


by
Scott Peterson, September 26, 2003

S N
Base S N
R1 S O Li R1 R2
O
O O S O
R1
+ H
H H R2
R2
O

D. A. Evans Friday,
November 21, 2003

H29A-00 11/20/03 2:51 PM


Examples of Direct Olefination from Carbonyl Compounds

X H H
+ R2
R1 H R2 O R1
H

X Reaction

R3P+ Wittig
B.E Maryanoff, A.B. Reitz,
Chem. Rev., 1989, 89, 863 R2P(=O) Horner-Wittig

(RO)2P(=O) Horner-Wadsworth-Emmons
L.F. van Staden, D Gravstock,
D.J. Ager,Chem. Soc. Rev., R3Si Peterson
2002, 31, 195
ArS(=O)(=NMe) Johnson
P.R. Blakemore, J. Chem. Soc., ArSO2 classical Julia
Perkin Trans. 1, 2002, 2563
HetSO2 modified Julia

Classical Julia Olefination

O
SO2Ph SO2Ph single electron H
SO2Ph n-BuLi H R2
R2 donor R2
R1 R1 R1
R1 -78 oC Li 2) Ac2O Na/Hg
OAc SmI2 H
THF

C5H11 C5H11
C5H11

E:Z = 80:20 E:Z = 90:10 E:Z = >99:1

H29A-01 11/24/03 9:03 AM


Mechanism of Olefin Formation

Ph Ph
O S O M O S OM R2
R2 R2 H H
R1 R1 R1
OAc
OAc OAc

-Originally proposed mechanism H


for Na/Hg elimination, though R2
Keck has shown this is not the R2 H H
R1
case R1
H OAc
-Believed to be mechanism for
SmI2 elimination

G.E. Keck et al., J. Org. Chem., 1995, 60, 3194

Mechanism of Olefin Formation


Na/Hg + MeOH

Ph
H O S O Na/Hg 1e-
OMe
PhSO2 R2 R2 R2
R1 -NaOAc R1 -PhSO2Na R1
OAc H H

Na/Hg
1e-

-Using MeOD results in


>90% Deuterium incorpration H Na
quench
-After initial elimination, there R2 R2
is no equilibration, explaining
R1 R1
why Na/Hg and SmI2 can give H H
different results

G.E. Keck et al., J. Org. Chem., 1995, 60, 3194

H29A-02 11/24/03 9:05 AM


Synthesis of Bryostatin 2

OTBS
TBSO OTBS
TBSO
O o
1) n-BuLi, THF, -78 C
O
H O 2) Ac2O, DMAP
3) Mg, 20mol% HgCl2
H
SO2Ph Single Olefin Product (E) O OTBS
64% over 3 steps Me
H Me
O OTBS
Me Me OPMB
Me
Me OPMB

D.A.Evans, P.H. Carter, et al., J. Am. Chem. Soc., 1999, 121, 7540

Me
HO Me
TBSO OPMB
Me Me
PhSO2 OPMB O O

n-BuLi (2eq) NHPh


O

OTf NHPh THF, HMPA H OTBS O


O OTBS
TBSO Me
TBSO O Me
O Me OPMB

H
O OTBS
Me
Me Me Me
HO OH
Me OPMB
MeO2C
O O

O
OH H OH
D.A.Evans, P.H. Carter, et al., J. Am. Chem. Soc., 1999, 121, 7540 O O
Me
Me
O Me OH

n-Pr O CO2Me
H29A-03 11/24/03 9:08 AM
Modified Julia Olefination

N N N N N N
SO2R SO2R SO2R SO2R
S N N N N

BT PYR PT TBT

R
N
Bt = N
N
(benzotriazole)

Modified Julia Olefination - Smiles Rearrangement

S N Li
S N S N
R2CHO
O2S OLi O2S O
O S O
R1 Li R1 R2 R1 R2

O
N R2 LiO S R2
OH + SO2 +
S R1 R1 OBT

H29A-04 11/24/03 9:10 AM


Diastereoselectivity of BT-Sulfones

OLi
S N S H
R1 slow N
R2 O Li R1 SO2Li
O Li
O S R1 SO2
SO2BT R1 O BTO R2
H H
anti R R2 H
H 2 H
R2

R1

Li O E
BT +
S R1 H R2 R1 R2
O2

OLi S N S
N R2
R1 O Li O Li
fast R1 SO2Li
R2 O S O R1 SO2
R1
SO2BT R2 BTO H
H R2
syn HH H H

J.B. Baudin, Bull. Soc. Chim. Fr., 1993, 130, 856

H29A-05 11/24/03 9:12 AM


Effects of Solvent and Counterion with BT-Sulfone
CHO

BTSO2
(Me3Si)2N-M
-78 oC to RT

M Toluene Et2O THF DME

Li 50:50 49:51 66:34 70:30


Na 54:46 50:50 62:38 75:25
K 54:46 51:49 54:46 76:24

Solvent Screen in U-106305 Synthesis


Me NH

solvent E:Z

Me SO2-BT toluene 9:91


NaHMDS
Me OTIPS CH2Cl2 9:91
o
-78 C to RT
Et2O 11:89
OHC OTIPS
THF 52:48

DME 71:29

Me SO2-BT DMF 78:22

OHC OTIPS NaHMDS, THF-DMF


92% 81:19
o
-60 C to RT

A.B. Charette, et al., J. Am. Chem. Soc., 1996, 118, 10327

H29A-06 11/24/03 9:15 AM


Retroaddition - Addition with BT-Sulfone

O n-C8H17CHO
LDA, THF C8H17
BT S
o
O -78 C to RT

E:Z = 23:77

OLi OLi
slow Ph RCHO Ph RCHO Ph fast C8H17
Ph C8H17
C8H17 C8H17
BTSO2 Ph
SO2BT SO2BT
anti syn

CHO
OH
Ph O2N NO2

BTSO2 LDA, THF Ph Ph


-78 oC to RT 40% E:Z = 98:2 60% E:Z = 92:8

P.R. Blakemore, Ph.D. Thesis, University of Glasgow, Glasgow, 1999

Aromatic Aldehydes with BT-Sulfones

S
OLi N O O
O Li -BTOLi S -SO2
R1 R1 SO2 R1
Ar Ar C H
Ar H R1 H Ar
SO2BT
H cis

OLi S
N O O
R1 O Li -BTOLi S -SO2 Ar
Ar R1 SO2 R1
SO2BT H C Ar
H Ar R1 H trans
H

CHO
R R yield E:Z
R OMe 95% 99:1
BTO2S LDA, THF H 68% 94:6
-78 oC to RT Cl 51% 77:23
J.B. Baudin, et al. Bull. Soc. Chim. Fr., 1993, 130, 856

H29A-07 11/24/03 9:16 AM


Reversibility in Rapamycin Synthesis
P. Kocienski, et al., Synthesis, 1996, 285
S S
Me
O OMe
BTSO2 OTBS
CHO Me Me
H
Me
MN(SiMe3)2, THF M yield E:Z
S S -78 oC to RT
Li 75% 29:71
Me
O OMe
Na 79% 43:57
OTBS K -- 18:82
H
Me Me Me

MN(SiMe3)2, THF
M yield E:Z
-78 oC to RT
S S
Me Li 68% 95:5
O OMe
BTSO2 OTBS Na 21% 78:22
CHO Me Me
H
Me

Possible Explanation for Diastereoselctivity

O SO2BT
S N H R1
S Li R2
or BT
R1 O R1 R2
H O
O S Li R2 OLi
O
R1 O H
R2
H

Chelate (closed) Transition State favored for non-polar solvents, small counter-ions (Li)

Non-chelate (opened) Transition State favored for polar solvents, large counter-ions (K)

BT O
LiO O H SO2BT
S
H R2 = BT S Li R2 R2
O R1
R1 R1
R2
R1 H O
H OLi
O

S. Peterson, Meandering Thoughts, 2003


H29A-08 11/24/03 9:21 AM
Ipso Substitution with BT-Sulfones

S
N O LDA
N O N
S CH3 o S
S THF, -78 C
O 3 hr O O
52%

S
N O N
S SO2Me
S O

J.B.Baudin, et al., Bull. Soc. Chim. Fr., 1993, 130, 856

Synthesis of ent-Bengamide E
TBSO OCH3
OHC OCH3
LiHMDS TBSO OCH3
TBSO OTBS O
OCH3
DME
O -78 oC to RT TBSO OTBS O
N 62%
S
S O

OH OCH3 O
H
N
NH
OH OH O
K.J.McRae, PhD Thesis, Research School of Chemistry, Canberra, 2001
J.B.Baudin, et al., Bull. Soc. Chim. Fr., 1993, 130, 856 ent-Bengamide E

H29A-09 11/24/03 9:23 AM


Synthesis of LAF389
O
O
S N O 1) LiHMDS, -78 oC, THF
CH3 O
OHC OCH3 2) TMSCl H3C
O S O + OCH3
3) aldehyde H3C
H3C O O
4) -78 oC: 1 hour O O
H3C H3C CH3 0 oC to RT: 1 hour
CH3 H3C CH3
50 oC: 1 hour

45% single isomer,


white crystalline solid

CH3 OH OCH3 O
H
N
H3C NH
CH3
OH OH O
O
L. Waykole, et al., Organic Process Research and Development, 2003, ASAP O
(Novartis Process Group)
LAF389

Synthesis of LAF389

S N O
1) LiHMDS, -78 oC TMS O O
O S O H 2) TMSCl N S t-Bu
+
H3C
3) aldehyde S O Ph
H3C
CH3 4) -78 oC to 50 oC
Ph

H3C Ph
H3C TMS O O
CH3 Ph
TBAF N S t-Bu MeSO3H H3C Ph

S H3C
H3C E:Z = 1:1 O Ph E only CH3 Ph
H3C Ph
CH3 Ph

Ph L. Waykole, et al., Organic Process Research and Development, 2003, ASAP


(Novartis Process Group)

H29A-10 11/24/03 9:26 AM


Synthesis of LAF389
O
O
S N O 1) LiHMDS, -78 oC, THF
CH3 O
OHC OCH3 2) TMSCl H3C
O S O + OCH3
3) aldehyde H3C
H3C O O
4) -78 oC: 1 hour O O
H3C H3C CH3 0 oC to RT: 1 hour
CH3 H3C CH3
50 oC: 1 hour

45% single isomer,


white crystalline solid

CH3 OH OCH3 O
H
N
H3C NH
CH3
OH OH O
O
L. Waykole, et al., Organic Process Research and Development, 2003, ASAP O
(Novartis Process Group)
LAF389

Synthesis of LAF389

S N O
1) LiHMDS, -78 oC TMS O O
O S O H 2) TMSCl N S t-Bu
+
H3C
3) aldehyde S O Ph
H3C
CH3 4) -78 oC to 50 oC
Ph

H3C Ph
H3C TMS O O
CH3 Ph
TBAF N S t-Bu MeSO3H H3C Ph

S H3C
H3C E:Z = 1:1 O Ph E only CH3 Ph
H3C Ph
CH3 Ph

Ph L. Waykole, et al., Organic Process Research and Development, 2003, ASAP


(Novartis Process Group)

H29A-11 11/24/03 9:26 AM


Pyridinyl (PYR) Sulfones Examples

Me

KHMDS OTIPS
CHO + OTIPS E,Z
Me PYR-SO2 toluene +

Me OTIPS
E,E

temp yield Ratio E,Z : E,E

-78 oC 35% 84:16

0 oC 53% 90:10
25 oC 67% 91:9 Potassium metallate is
stable at RT for 5 min!

A. B. Charette, et al., Tet Lett, 2001, 42, 5149

Pyridinyl (PYR) Sulfones Examples


Me

OMe Curacin B
N Me
S
Me Me
CHO Me Me
Me KHMDS, toluene

+ 25 oC Me

OTIPS 64%, 88:12


PYR-SO2 OTIPS

Me Me
Me Me
Me SO2-BT LiHMDS, CH2Cl2
+ -78 oC to RT Me
CHO 75%, 6.5:1
( 87:13 )

A. B. Charette, et al., Tet Lett, 2001, 42, 5149


H29A-12 11/24/03 1:20 PM
1-Phenyl-1H-tetrazol-5-yl Sulfones
O O O
N S MN(SiMe3)2
N Me Me
H
N N o
-78 C to RT
Ph

M toluene Et2O THF DME

Li 51:49 61:39 69:31 72:28


Na 65:35 65:35 73:27 89:11
K 77:23 89:11 97:3 99:1

n-C4H9 n-C5H11 n-C5H11 n-C5H11


n-C5H11
Me Me
E:Z = 94:6 E:Z = 96:4 E:Z = 99:1

1.5eq aldehyde, KHMDS, DME -78 oC to RT

P.J. Kocienski, et al., Synlett, 1998, 26

Kinetically Controlled Diastereoselectivity - Irreversible


HO Me KHMDS, DME

-60 oC to rt, 1.5hr


Me O2S Me
PT
91%, E:Z = 98:2
TESO Me
TBAF, DME
Me
Me
Me O2S -60 oC to rt, 1.5hr
PT
91%, E:Z = 2:98

CHO

OH NO2
O2N
Ph
Ph Ph
PTSO2 LDA, THF
-78 oC to RT 100% E:Z = 98:2 0%

P.R. Blakemore, Ph.D. Thesis, University of Glasgow, Glasglow, 1999


H29A-13 11/24/03 1:23 PM
Synthesis of Herboxidine
SO2Et
N
O KHMDS O
+ N
CH3 N CH3
N DME
O o
O
-60 C, 45min
CHO
93%, 93:7 dr

1) AD-mix α
2) TsOH, MeOH

CH3
10 steps
CH3
CH3
O
H CH3
CO2All H H3CO O
H
OH
O
P.J.Kocienski, et al., J. Chem. Soc., Perkin Trans. 1, 1999, 955

CH3

CH3
Synthesis of Herboxidine
O
H CH3
CO2All H
CH3
O 1) LDA, THF O
H
CO2H
S N o o
-78 C to -20 C
81%, 91:9 dr Me
SO2 2) K2CO3 OMe
O
Me Me
OMe
O
Me OH Me Me

PCBO Me Me

P.J.Kocienski, et al., J. Chem. Soc., Perkin Trans. 1, 1999, 955

H29A-14 11/24/03 1:25 PM


Synthesis of (+)-Ambruticin
OTBS
Me
OTBS
PT
+ S
H O O O
TBDPSO O
Me Me Me
O Me

M-HMDS solvent temp E:Z

Na THF -78 oC 1:8

Na THF -35 oC 1:6


OTBS
OTBS K DMF -60 oC 1:1
Me
K DME/18-c-6-60 oC 1:3
TBDPSO O
O Li THF/HMPA -60 oC 3:1
Me Me Me
Me Li DMF/HMPA-35 oC >30:1

Li DMF/HMPU-35 oC >30:1
E.N. Jacobsen, P. Liu, J. Am. Chem. Soc., 2001, 123, 10772

tert-Butyl-1H-tetrazol-5-yl Sulfones

1) KHMDS, DME Het yield


SO2Het -60 oC, 2 hr SO2Het
0%
BT
Me Me PT 20%
2) H2O
TBT 91%

O O 1) KHMDS, DME
N S -60 oC, 30min
N n-C9H19
n-C9H19
N N
2) n-C9H19CHO
-60 oC to RT
39% 67:33
O O 1) KHMDS, DME
N S
-60 oC, 30min
N n-C9H19
n-C9H19
N N
2) n-C9H19CHO
-60 oC to RT 60% 4:96

H29A-15 11/24/03 1:29 PM


Diastereoselectivity of TBT-Sulfones
N N
N
t-Bu N N N
OLi t-Bu N
O Li N
S slow H
O O Li SO2Li
R2 O SO2
H
SO2TBT PYRO R2
HR2 R2 H H
anti H

R2
N N Li O
+ E
N
S H R2
N
O2 R2
t-Bu Z

N N N
OLi N
t-Bu N N t-Bu N
N
O Li fast R2
R2 O Li SO2Li
O S SO2
SO2TBT O
R2 PYRO H
H R2 H
syn HH H

Sulfone Synthesis
RX, B-

X = halide, tosylate, triflate


N N
N N
SH S
N N N N R

ROH

R'O2CN=NCO2R'
PPh3 N
N
SH
N N N
N
SH
N N
N
SH
S t-Bu N C S + NaN3

2-mercaptobenzothiazole
1-phenyl-1-H-tetrazole-5-thiol tert-butyl isothiacyanate; 25g = $57.80
100g = $18.00 Sodium azide; 25g = $51.90
25g = $22.60

H29A-16 11/24/03 1:33 PM


Sulfone Synthesis
O
N N [O] N
N
O
S S
N N N
N R R
Ph Ph

MCPBA

(NH4)6Mo7O24-4H2O / H2O2 Mo(VI)

Na2WO4-2H2O / H2O2 W(VI)

Oxone

CH3CO3H

KMnO4

P.R. Blakemore, J. Chem. Soc., Perkin Trans. 1, 2002, 2563

Oxidation Problems - Allylic Sulfones

C(CF3)2OTES
C(CF3)2OTES

C(CF3)2OTES

N Mo(VI), H2O2 O2
S N
N S
EtOH - H2O, r.t. N
N N HO
N N

11%
38%

R [2,3] R
O O
S S
PT PT

H. Hilpert, B. Wirz, Tetrahedron, 2001, 57, 681


D.A. Evans, G.C. Andrews, Acc. Chem. Res., 1974, 7, 147

H29A-17 11/24/03 1:34 PM


Synthesis of the Proposed Structure of Amphidinolide-A

AcO
H
KHMDS AcO
Me O Me
OTBS THF
-78 oC to RT Me Me
O O Me
S 78% 4:1 dr OTBS
N
Me inseparable
S 1) HF-pyr (separate isomers)
2) t-BuOOH, Ti(OiPr)4
3) EDC, DMAP, CH2Cl2,
(E)-iodobut-2-enoic acid
OTBS
O
Me OTBS O
+ KHMDS
H
Me O
O DME AcO
-55 oC to RT Me
PT 92% 95:5 dr
S O 40g scale
O2 Me Me
O
I O
G.Pattenden, H.W.Lam, Angew. Chem. Int. Ed., 2002, 41, 508
M. Hirama, Tet. Lett., 1999, 40, 4897 Me O

Synthesis of Proposed Structure of Amphidinolide-A


OTES
TESO AcO
SnBu3 AcO Me
Me
TESO Me n-Pr
Me n-Pr O
O I O
TESO SnBu3 I O
Me O
Me O

1) Pd2(dba)3, Ph3As
2) PPTS
3) Pd2(dba)3, Ph3As, LiCl
OH OH
HO HO
Me Me
HO HO
Me n-Pr Me n-Pr
O O
O O
HO HO
Me O Me O

NOT Amphidinolide-A NOT Amphidinolide-A

G.Pattenden, H.W.Lam, Angew. Chem. Int. Ed., 2002, 41, 508


H29A-18 11/24/03 1:38 PM
Synthesis of Okadaic Acid
Ph
OTBDPS
H
O H H O
O
O SO2-BT OHC O
O O H OTIPS
H Me O
O OBn H H
O Me
NaHMDS H
Ph DMF-THF Me
-60 oC to RT O
Ph 66%
OTBDPS O
H
O H H O
O
O O
O O H OTIPS
H Me O
O OBn H H
O Me
H
Ph Me
O
"small quantity of the corresponding (Z)-isomer was also
detected by 600 MHz 1H NMR" O

S.V. Ley, et al., J. Chem. Soc., Perkin Trans. 1, 1998, 3907

Synthesis of Vinylsilanes

O SiMe3
O O
+ Ph
Ph SiMe3 S Ph Ph
PT

O O
S Ph M-HMDS temp yield E:Z
PT Smiles SiMe3
Ph Li -78 oC 93% 64:36
LiO R Ph
SiMe3 Li -85 oC 89% 74:26

Brook Li -95 oC 84% 75:25

Na -78 oC 50% 59:41


O O
S Ph K -78 oC NA NA
PT OTMS
Ph J. Wicha, et al., Org. Lett., 2003, 5, 2789
TMSO R Ph

H29A-19 11/24/03 1:42 PM


Conclusions

BT and PT sulfones have become useful funtional groups for the synthesis of olefins from aldehydes
inexpensive starting materials
sulfones can be made in high yield from alcohols
olefination reactions occur under mild conditiions, and are typically high yielding and selective

Stereochemical outcome is kinetically controlled in most cases. Though reaction conditions can often influence selectivities
Polar solvents with soft counter ions often favor E olefins
Non-polar solvents with hard counter ions often favor Z olefins

BT sulfones are most useful for the synthesis of conjugated dienes through reaction with α,β−unsaturated aldehydes

R SO2BT + OHC R2 R R2

PT sulfones are most useful for the synthesis of non-conjugated (E) olefins

R + R
SO2PT OHC R2 R2

PYR and TBT sulfones both produce high levels of cis selectivity, though yields are typically lower than BT reactions

Definitely more to come in the future

H29A-20 9/25/03 2:45 PM


D. A. Evans Pairwise Functional Group Relationships Chem 206

Relevant Questions
http://www.courses.fas.harvard.edu/~chem206/
The pyridoxal co-factor (Vitamin B6) 1, facilitates the decarboxylation of α-amino
acids. Provide a mechanism by which 1 carries out this transformation.
H O
Chemistry 206 O
-CO2
H2 N
O
OH H2 N
Phosphate–O C
OH 1
Advanced Organic Chemistry 1 N Me
R R O

Lecture Number 30
Cume Question, Fall 2001. The reaction illustrated below was recently
reported by Murry and co-workers from the Merck Process Group (JACS 2001,
123, 9696-9697). Provide a mechanism for this transformation.
Ambiphilic Functional Groups–4
Tol Ar2 O
■ Construction of Consonant & Dissonant FG Relationships O SO2 O
10 mol% 1
+ Ar1
N OCMe3
■ Charge Affinity Inversion Operators Ar1 H Ar2 N
H
OCMe3 Et3 N, CH2 Cl2 H
35 °C O

Lecture 27 and handout 27A S


N
Me CH2 CH2 OH
Handout 30A: Homoenolates: Synthesis & Applications 1 (used as catalyst)

Mesembrine Syntheses: Keely, S. L.; Tahk, F. C. JACS. 1968, 90, 5584.


Stevens, R. V.; Wentland, M. P. JACS 1968, 90, 5580 (handouts) The nucleophilic addition of aldehydes to electrophilic double bonds catalyzed by
thiazolium salt 1 is referred to as the Stetter Reaction (Stetter, H.; Kuhlmann, H.
Stetter, The catalyzed nucleophilic addition of aldehydes to electrophilic Org. Reactions 1991, 40, 407). Provide a mechanism for this transformation.
double bonds. Org. React. (N.Y.) 1991, 40, 407.
O O
Ahlbrecht, "Stereoselectivity of chiral homoenolate equivalents." Synthesis Me 10 mol% 1
Me
1999, 365-390. Me H Me
Et3 N, EtOH
O 60 °C O

Monday,
D. A. Evans December 1, 2003

30-00-Cover Page 12/1/03 9:07 AM


D. A. Evans Summary of Functional Group Classification Scheme Chem 206

Classification of Functional Groups (–) (+) (–)


G-Functions
G C C C
Each substituent attached to carbon activates that carbon toward a polar ■ Those ideal FGs which create nucleophilic carbon at point of attachment.
reaction by either resonance or induction or both.
■ Exhibit strictly alternate charge affinity patterns.
Induction (+) (+) (–) (–)
F1 C F2 C F3 C F4 C These are your metallic FGs such as Li, Mg, etc.
Resonance (+) (–) (+) (–)
CH3 CH2 Li CH2 CH CH2 MgBr
(–) (–) (–)

Symbol E C (+) A C (±) G C (–) ■ Note that a 2-electron reduction (or oxidation) will transform an E-Class
FG to a G-Class FG.
Charge Affinity Patterns
(+) +2 e – (–)
(+) (–) (+)
(±)
(±) (–) (+) (–) E C G C
E C C C A C C G C C C
(–) (–) (+) (+) CH3 CH2 Br CH3 CH2 MgBr
(+) (–)
A C C A C C
(±) (±) (–) (–) (+) (+)
Real functional groups are assigned to a class designation by inspection of the A-Functions
A C C A C C A C C
chemistry of that FG, along with that of its conjugate acid and conjugate base
■ All sites activated equally for electrophilic & nucleophilic reactivity.
Charge affinities of real functional groups form a subset of the ideal FG classes. ■ Those ideal FGs which exhibit nonalternate polar site reactivity are included.

(+) (–) (+)


E-Functions E C C C One might visualize a process wherein A-functions are gradually polarized
OR towards either E– or G– behavior in response to changes in inductive and
resonance effects.
CH3 CH2 OR CH3 CH O CH3 C O
(+) (–) (–) (+) (–) (–) (+) (–)

(±) (±)
(±) (+) (–) (+)
CH2 CH CH2 Br CH3 CH2 Br CH3 CH2 NR2 A C C C E C C C
(+) (+) (–) (+) (–) (+) (–)
(+-) (+-) (+-)
■ Note that the issue of oxidation state in not explicitly incorporated. A C C C
This issue is subordinate to that of defining site reactivity. (±) (±) (±)
(–) (+) (–)
For example, A C C C G C C C
OR
(–) (+) (–) (+)
CH3 C O is represented as: C C E and not: C C E2
(–) (+) (–) A-functions are some of the most useful FGs in organic synthesis because of
E1 the unique reactivity provided.

30-01 Pairwise FGs-1 11/30/03 6:09 PM


D. A. Evans Pairwise Functional Group Relationships Chem 206

A-Functions: Real Examples Classification of Pairwise Difunctional Relationships


(±) (±) (+) (+) (–) (–)
A-Functions Consider the paired relationships of E-functions. There are two relationships.
A C C A C C A C C
E# E# E
■ A-functions are composed of polyatomic arrangements of N & O. add E
C C C C C C C C Charge affinity patterns
(+) (–) (+) (–) (+) (–) (+) (–) are "matched."
NO2 NOH NNR2 N NR2 N(O)R N N N
Pairwise relationship is
"consonant".
■ A-functions are composed of second-row elements such S and P.
O O R E# E# E
add E
S R S R S R S C C C C C C C C Charge affinity patterns
R (+) (–) (+) (–) (+) (–) (+) (–) are "unmatched."
O
R O R E# E
P P R P R Pairwise relationship is
C C C C
R (+) (–) (+) (–) "dissonant".
R R
■ Functional groups derived from many of the transition elements Consonant & dissonant relationships may be established with E-E, E-G, or G-G
pairings.
Synthesis of Targets containing E-Functions
Most target structures are composed of E-functions.
Transforms utilizing target E-function in synthesis plan given highest priority.

E E G
G–FG lost
Representative difunctional relationships
C C C C C C C C
(+) (–) (+) (–) (+) (–) (+) (–) in construction O O
E E'
Me OH Classification: 1,3-C
E E E' O
E'–FG lost symbolic representation
C C C C C C C C in construction
(+) (–) (+) (–) (+) (–) (+) (–) Me NR2

E
E E G O Cl
E'
C C C C C C C C G–FG lost HO Classification: 1,2-D
(+) (–) (+) (–) (+) (–) (+) (–) in construction symbolic representation
NH2 OH

Given the resident E-function, the charge affinity pattern dictates the nature of the polar G
coupling process and thus functional groups to be employed in synthesis. Li
E
Cl Classification: 1,1-D
symbolic representation
30-02 Pairwise FGs-2 11/30/03 6:11 PM
D. A. Evans Pairwise Functional Group Relationships–2 Chem 206

Classification of Pairwise Difunctional Relationships Pairwise Relationships: Path-Path Interconversions via


Sigmatropic Rearrangements
■ A single FG residing either in or appended to a cycle may establish a FG [3,3] Sigmatropic Rearrangements:
relationship with itself.
R R
E
E O O
E E 1,2–D 1,4–D

OMe OMe
Consonant cycles Dissonant cycles
R R
HO O
1,2–C 1,5–C
Consonant & Dissonant Relationships: Path-Cycle Interconversions
OMe OMe

Linear molecules may be transformed into cycles & vice-versa: ■ For these rearrangements, C→C', D→D' but C→D not possible

O O

Relationship: MeO HN [1,2] Sigmatropic Rearrangements:


(+) (+) (+) NH2
1,5-C
O
Consonant Cycles CH2N2
C– cycle O D– cycle
Relationship: Cl OH
1,5-C (+) (+) (+) O
[2,3] Sigmatropic Rearrangements:
O
O
Me R
Me Me – –O R
O
O Dissonant Cycles 1,2–D 1,3–C
Me
Relationship:
1,4-D O O
OMe OMe
Me Me Me Me
O OR General Rule For [m,n] Sigmatropic Rearrangements:

Path-cycle interconversions such as those illustrated permute, but do not When the sum of m+n is even, the FG relationship is maintained, e.g. C→C'
eliminate the relationship. i.e. D-bond paths are transformed into D-cycles. When the sum of integers is odd, the FG relationship is changed, e.g. C→D

30-03 FG Relationships-1 11/30/03 6:12 PM


D. A. Evans Pairwise Functional Group Relationships–3 Chem 206

Pairwise Relationships in Inorganic Reagents A Specific Case


O OH
Target structure: Step I: There are 4 bonds interconnecting E & E'.
■ E-functions in their most stable oxidation states (HO–, NH3, Cl –) are Hence generate the 4 transforms leading to mono-
represented as E(–). RO Me functional precursors:

There exist an important family of reagents which have E-FGs directly coupled: O
E# E
OLi O O [H] O OH
E–E Reagents C C C C RO X
(+) (–) (+) (–) Me RO Me RO Me
Br Br HO NH2 In each of these reagents
HO OH H2N NH2 there is a 0,0-D relationship
E# E OLi O OH
O
C C C C
These reagents are used to construct D-Relationships: (+) (–) (+) (–) RO H Me RO Me

O O Br
Br2
R Br2
R E# E O O OH
Br HO –
R Me -HBr R R R
H 2O C C C C RO Me RO Me
OH (+) (–) (+) (–) [H]
Synthesis of Targets containing Consonant Pairwise Relationships
E# E
E# E E# E O
HO –
O
[Ox] O O
C C C C
C C C C C C C C (+) (–) (+) (–) Me HO Me RO Me
(+) (–) (+) (–) (+) (–) (+) (–)
Aldol, Claisen Step II: Evaluate the efficiency of the 4 plausible routes to the target from available
Mannich Rxns precursors.
E# E E# E
Given the oxidation state in the target, the second synthesis looks the best and the
C C C C C C C C fourth looks the worst.
(+) (–) (+) (–) (+) (–) (+) (–)
The Constraint of Quaternary Centers
# #
E E E
If a quaternary center occurs along the consonant bond path, one is limited to bond
C C C C C C C C E (–) constructions on either side of that restriction
(+) (–) (+) (–) (+) (–) (+) (–)
O Me Me O O Me OLi
# Conjugate
E E E Addition
RO Me RO Me Me
C C C C E #(–) C C C C
(+) (–) (+) (–) (+) (–) (+) (–) OLi
O Me Me O Me O

Consonant difunctional relationships can be constructed from just the functions illustrated RO Me Me
& polar bond constructions. RO Me

30-04 FG Relationships-2 11/30/03 6:13 PM


D. A. Evans Pairwise Functional Group Relationships–4 Chem 206

Quaternary Centers & Bridgehead Restrictions Synthesis of Dissonant Pairwise Relationlships

Me The pairwise relationship is "unmatched"; hence, the


N
Lucidulene Synthesis: JACS 94, 4779 (1972) illustrated E-functions cannot be used exclusively to E# E
construct the bond path. Let's consider the simplest case:
H Me a 1,2-D relationship. C C
Me # (+) (–)
H H E E
N
O
(+)
Me C C Resident E-functions do
H H (+) (–) not provide required
(–) H charge affininty pattern for
Focus on the shortest consonant bond path: (+)
E# E E#
coupling
O
H C C C C E (+)
(+) (–) (+) (–)
The two permitted bond constructions along illustrated bond path flank
the bridgehead carbon
E This transform defined a path-cycle
Me Me permutation of the D-relationship
C C
(+)
N N+ (+) (+)
Me H2C Me
H H In the illustrated polar disconnections, one of the fragments may exploit the charge
(–) H H affinity pattern of the resident FG while the other may not.
Hence dissonant pairwise relationships may not be constructed via just the functions
O (+) HO
H H present in the target.
Mannich Transform Dissonant Pairwise Relationlships via A-Functions

Me
E# E E# A E# A
Me
+ N C C C C C C
N Me (+) (–) (+) (±) (+) (±)
Me H
H
X H
H In implementing this strategy you must know all important 1,1-A↔
↔E FG transformations

O O H C A C E
H
Enamine Acylation
Me O
Me S S
R R
H N N R R
H Me Me
H HO –
Option Selected: (CH2O)n
H H O Ph trifluoroacetic
∆, isoamyl alcohol H S CF3CO2 SPh The Pummerer Rearrangement
anhydride
O R R
H O R R "The Pummerer reaction of sulfinyl
H compounds.", De Lucchi, etal.
■ Corrolary: π-conjugation cannot be extended through bridgehead or Org. Reactions 1991, 40, 157.
O O– 1) OH O
quaternary centers N 2) H3O +
R R The Nef Reaction
R R
30-05 Synth Consonant-1 11/30/03 6:15 PM
D. A. Evans Pairwise Functional Group Relationships–5 Chem 206

Bond path analysis of simple alkaloids OMe


OMe Mesembrine
CH2OH (+) E2
Curphey, T. J.; Kim, H. L. Tetrahedron Lett. 1968, 1441.
(+)
(+) (+)
lupinine Keely, S. L.; Tahk, F. C. JACS. 1968, 90, 5584.
N E1 Stevens, R. V.; Wentland, M. P. JACS 1968, 90, 5580
(+) (+) N Shamma, M.; Rodrigues, H. R. Tetrahedron 1968, 24, 6583
O
Every complex polyfunctional molecule may be analyzed structurally in terms of its Me
individual consonant or dissonant construction paths or cycles. For example, in the In the analysis of potential routes to structures like mesembrine, identify the shortest
alkaloid lupinine all possible construction paths interconnecting E1 and E2 are consonant bond path and then proceed to carry out all polar disconnections along that
consonant. Consonant paths within the polyatomic framework define seams in the bond path. Since there four bonds interconnecting =O and N (E1 and E2), there will be
structure that may be constructed using aldol and related processes. four associated transforms which one may execute using the illustrated functional
Begin the disconnection process by focusing on the shortest consonant groups. Ar Ar
bond path. In this case, there are 4 bonds, hence 4 disconnections. C–E2
E2 E2 equivalent to:
(+) (+) C CH2=O
T1 E1 (+) (–) (+) E2(–) O HN
(+) Me
(+) (+) (+)
C–C (+) (+)
E1 equivalent to: Ar Ar
E1 N
(+) (+)
(+) (+)
C–C
Ar equivalent to:

E2 X OH
T1 E1 (+) (–) (+) E2(–) HO N
(+)
C Me
(+) (–)E1 (+) (+) E2(–) Ar
C–C (+) (+) Ar
equivalent to: Shortest consonant C–C
E1 N bond path equivalent to:
(+) (+) RO2C
T1 E1 (+) (–) (+) E2(–) N
Me
(+) E2
CHO Ar Ar
(+) C–E1
C–E1 (+) (+)
equivalent to: equivalent to:
E1 N T1 (+) (+) E2 N
(–) (–) HO–
(+) (+) H (–) E1
Me
OH2 Now consider further analysis of T1: Again, select the shortest E1-E2 bond path and
(+) E2 (–) CH2 disconnect next to quaternary center. Dissonant element is localized in 5-membered
enamine
(+) Ar
C–E2 (+) (+)
(+) Ar
equivalent to: (+) Ar
E1 N (–) (–) equivalent to:
(+) (+) N
O Me
(+) E2
Note that oxidation states of precursors is not yet considered. E1 (+) (–) (+) E2(–) E1 (+) (–) (–) Me

Keely, S. L.; Tahk, F. C. JACS. 1968, 90, 5584.


Handouts Stevens, R. V.; Wentland, M. P. JACS 1968, 90, 5580
30-06 Synth Consonant-2 11/30/03 6:16 PM
D. A. Evans Inversion Operators-1: The Electronic Characteristics of Cyanide Ion Chem 206

The Benzoin Condensation


Inaccessible Reactivity Modes in Carbonyl Deprotonation
O
O
Example: H
– :CN 2 – :CN
O OH
Me ■ Cyanide ion is such a "catalyst"
H
O O
O O
Me
H X C Me carbonyl anion inaccessible Ph H
Ph H CN
base
O
X
CH2 homoenolate anion inaccessible
H
OH
Can one design "catalysts" which will provide access to carbonyl anion CN Equivalent to:
equivalents in situ?? Ph CN
O
C
Ph
O
Let Q – be such a catalyst, we will call it an "inversion operator" OH O
Ph
Ph
Ph El Ph H
O OH CN
O
R H
R H Q How do we classify the N functional group?

■ Hydrogen cyanide is a fairly good Bronsted acid (pKaHOH 9.5)


OH
Q Equivalent to: (–)
R Q H C N H+ + – :C N C G

O
C ■ Acetonitrile can be attacked be nucleophiles:
R N– (+)
O OH Nu: –
Me C N Me C Nu C E
R El R El
Q
■ Acetonitrile can be deprotonated by strong bases (pKa DMSO ~ 30)

– (–) (+)
Me C N H+ + H 2C C N C C E
30-07 Inversion Operators-1 11/30/03 6:18 PM
D. A. Evans Thiazolium Salts as Inversion Operators Chem 206

Cyanide-based Carbonyl Anion Equivalents Thiazolium Salts: Nature's Inversion Operators


Reactions equivalent to the benzoin are catalyzed by biological co-factors to make
■ Extensions of the Benzoin condensation concept are possible in some instances: (and break) dissonant difunctional heteroatom-heteroaton relationships

NH2 H The pka of this proton has been the subject of


O O considerable study. The current estimates are
Na–CN N N S that the value falls in the range of 16-20 but this
Ar H CO2Et OEt 44% yield number is not firm.
DMF Ar OH
Me N Me
O F. G. Bordwell JACS 113, 985, (1991)
The thiamine cofactor
The C–C Bond Construction Stetter, Org. Reactions 1991, 40, 407.
H
●● ●●
OH OH Me Me Me
OEt N S base N S N S
R C OEt 1
Ar
CN O CN Me Me
O Me

■ The in situ use of cyanide ion as an inversion operator is limited. Greater O O


generality may be achieved by multistep alternatives: Carbonyl anions might be C ●● C ●● 2
similarly stabilized R R
Aldehyde Derivatization Step
In the absence of electrophiles 1 & 2 dimerize as would be expected for carbene
O reactivity.
ZnI2 O–SiMe3
Me3Si–CN Me Me Me
R H R CN Me N Me Me
H base N N
Et H
OH S S S
O O OEt
– :C N OEt
R H R CN OLi
H R CN O O R Li O OLi

+
H R
C –C C C ●● R
●● ●● R Li R
Substrate Deprotonation Step OLi
O–SiMe3 O–SiMe3
LiNR2 Deprotonation possible only for R = Ar Reactions catalyzed by thiamine
R CN R CN due to Si migration.
O O
H
Me
Et Me
Et Me H
OH
O OEt LiNR2 Deprotonation possible for All R groups
O OEt O O O
R CN G. Stork JACS 93, 5286 (1971)
R CN OH + C
H Me Me H
O
O
30-08 Inversion Operators-2 11/30/03 6:19 PM
D. A. Evans Thiazolium Salts as Inversion Operators–2 Chem 206

Aldehyde dimerization by Thiazolium Salts Cataylzed Michael Reactions byThiazolium Salts

O O
O Thiazolium ion O
The Reaction R' R"" R''
The Reaction catalysis Me R 1,4–D
Me 1,2–D R H
Me H base O R' O
OH
CH2Ph
The Catalytic Cycle Me N The Conditions:
The Catalyst: H 0.1 equiv catalyst, Et3N or NaOAc,
R
R HO S EtOH or DMF at 60-80 °C
O N O
H
Me H S Me
Examples: "The catalyzed nucleophilic addition of aldehydes to electrophilic double
bonds.", Stetter, H.; Kuhlmann, H. Org. Reactions 1991, 40, 407.
O O
Me Me 61% yield
R R Me
R Me H
R N N OH O O
Equivalent to:
S Me O O
S
Me Me Me Me 41% yield
O
H
C– O Me O
R Me Me

O O O
S N O Ph Me
Me R Me 21% yield
Me Me H Me
Me OH Me H O Ph O
OH
H O
1,2–D O O
Me Ph
Ph Ph 70% yield
n-Pr H n-Pr
■ Hence dissonant relationships may made from E-functions if "inversion
O Ph O
operator" is employed
■ 1,4-D relationships may also be made from E-functions if "inversion operator" is
(–) (+) (+) (–) inversion operator employed.
C C + C C C C C C 1,2–D
E E E E
(+) (+) (–) (+) inversion operator
■ The is a fundamental strategy for handling the formation and cleavage of C + C C C C C C C 1,4–D
D-relationships in nature. E E E E

■ There is no analogue to this reaction in nature.


30-09 Inversion Operators-3 11/30/03 6:21 PM
D. A. Evans Thiazolium Salts as Inversion Operators–3 Chem 206

Decarboxylation Cataylzed by Thiazolium Salts Design Attributes of Inversion Operators


■ Background: Decarboxylation from consonant difunctional relationships is facile: Inversion operators are constructed from A-functions or molecules
containing D-relationships.

O O H H
∆ O O O O The pyridoxal Co-factor (Vitamin B6)
(–) (+) C
R (+) (+) OH R O H O
R O
(–) The critical difunctional relationlship is
OH
Phosphate–O that between =O & =N. This is a 1,4-D
■ The reverse processe can be achieved under basic conditions: relationship
N Me
M M
O O O O
O O H+ H O❋ ❋
(+)C (–) O
R R O R (+) (+) OH ❋ -CO2 O
(–) O R OH H2 N
The Reaction H2N
OH C
■ Such consonant relationships may be readily made (and broken) via the resident R R O
functional groups. The analog reactions for dissonant relationships not possible. N Me

O O O 1,4-D 1,2-D
For example: OH C relationship relationship
Me X
Me H
+
O
O O❋
■ Nature uses inversion operators to break such 1,2-D relationships
The Mechanism
R
R (+) OH
O O
H N (+)
O O H 2N - H2 O C
S N OH
OH R C O
Me R (+)
Me O O
O
H N (+) R H
O O– ❋
H O 1,6-C H N
R relationship
R
R N
S N
R
S N N
Me OH H
R R

S N H N Tautomerization
O R H 2N
+ H2O
Me H R
Me O–
H
30-10 Inversion Operators-4 11/30/03 6:37 PM N
Chemistry 206

Advanced Organic Chemistry

Handout 30A

Homoenolates: Synthesis & Applications

Evans Group Seminar


by
Jason Burch, March 24, 2000

O MLn E O E

X X

D. A. Evans Monday,
December 1, 2003

30A-01 12/1/03 11:16 AM


Enolates and Homoenolates
The Tautomerism Problem

• Enolates
M
O O • tautomerism is generally not a problem
M because oxyanionic tautomer still acts as
carbon nucleophile

• Homoenolates
M
O M O • tautomerism is a much larger problem
because it is often irreversible and
oxyanioic tautomer rarely acts as a
carbon nucleophile

Nakamura in Comp.Org.Synth., 1991, 2, 441

Homoenolate Equivalents
Definition: species containing an ionic carbon β to a moeity
which can be converted into a carbonyl group

Examples:

O O

O O

in situ protection

Y O

X NR
X = OR, NR2, etc.
Y = H, R, OR, NR2, etc.
Werstiuk in "Umpoled Synthons", Hase, Ed.;
Wiley: New York, 1987, Chap. 6
Ahlbrecht, Synthesis, 1999, 365 (chiral examples)

30A-02 12/1/03 11:18 AM


The First "Homoenolate"

Me Me
KOtBu, HOtBu
Me 250 °C, 4h Me
O O
(+)-camphenilone (±)-camphenilone

• no racemization occurred in >4 days at 250 °C in the absence of base


• proposed to proceed via a "homoenolate anion"

Me Me
Me Me Me
OtBu
Me Me Me
O O O O

Nickon, J.Am.Chem.Soc., 1962, 84, 4604

Cyclopropane Ring Opening


Synthesis of Titanium Homoenolates

O TiCl3 R = Et
OTMS TiCl4
Me
OR RO i
Pr
• if conducted in CDCl3 leads to a deep wine-red color; precipitates as purple needles in hexanes
• IR spectrum strongly supports coordinated carbonyl (νC=O = 1603 for R = iPr in benzene)
• molecular weight by cryoscopy is 560-620 indicating dimeric structure
→ later verified in solid state by x-ray crystal structure (Floriani)

°
Relevant bond lengths (A):
Ti-C 2.081
Ti-O 2.072
C=O 1.235

Nakamura, J.Am.Chem.Soc. 1983, 105, 651


Floriani, Organometallics, 1993, 12, 2845

30A-03 12/1/03 11:19 AM


Cyclopropane Ring Opening
Regioselectivity of Ring Cleavage - Titanium
• in general, cleavage occurs selectively at the least substituted cyclopropane bond

O TiCl3
R R' A:B
RO
i
OTMS TiCl A Pr Me >95 : 5
4 R'
OR
Me Me 60 : 40
R' O TiCl3
Et Ph 78 : 22
RO R'
B
• A can be isolated, but B is too unstable; only detected by in situ
quench with electrophiles (i.e. Br2, RCHO)
• if non-racemic starting material is used, quench with electrophiles
indicates non-racemized A and totally racemic B
i.e.
O E O E
OTMS 1. TiCl4
+
OMe 2. E+ MeO MeO Me
>99:1 Me 1:1 60 : 40
Me
> 99% ee racemic

Nakamura, J.Am.Chem.Soc., 1986, 108, 3749

Cyclopropane Synthesis
• most common method

Cl OEt 2 Na OTMS
TMSCl OEt
O
- used to prepare substituted cyclopropanes
Me Me
Cl OEt 2 Na OTMS
TMSCl OEt
O
• use Simmons-Smith for ketone-derived substrates

OTMS OTMS

Et2Zn
CH2I2
Ruhlmann, Synthesis , 1971, 236.
Salaun, Org. Synth., 1985, 63, 147
Murai, J.Org.Chem., 1973, 38, 4354

30A-04 12/1/03 11:20 AM


Alkoxide-Modified Homoenolates
Tuning Titanium Homoenolate Reactivity

• Problem: trichlorotitanium homoenolates are not reactive enough for some applications
can also lead to chlorinated byproducts
• Idea: replace 1 or more chlorides with alkoxides to increase nucleophilicity

O TiCl3 O TiCl2OiPr
i
+ 1/2 equiv. Ti(O Pr)4 + 1/2 TiCl2(OiPr)2
i i
PrO PrO

O TiCl3 O TiCl2OtBu
t
+ 1/2 equiv. Ti(O Bu)4 + 1/2 TiCl2(OtBu)2
i i
PrO PrO

• Have not been characterized to the detail of the trichlorohomoenolates


• Appear to have "significant contribution from monomeric forms" (from molecular weight data)
• Are more reactive than trichloro homoenolates towards homoaldolisation

Nakamura, J.Am.Chem.Soc., 1986, 108, 3745

Cyclopropane Ring Opening


Zinc Homoenolates
• Zinc homoenolates can be prepared in a similar method to titanium

O OR
OR
OTMS ZnCl2 vacuum
2 + 2 TMSCl O Zn O
OR Et2O O Zn OEt2
RO
RO
νC=O = 1662 cm-1 νC=O = 1645 cm-1
1740 cm-1
4 inequivalent 2 inequivalent
methylene groups methylene groups
in 1H-NMR in 1H-NMR

Nakamura, Organometallics, 1985, 4, 641

30A-05 12/1/03 11:21 AM


Direct Oxidative Addition
Zinc and Lanthanide Homoenolates

O I O ZnI
Zn•Cu
Quantitative
Benzene / DMA
EtO 60 °C, 3-4 h EtO

O I O ZnI
Zn•Cu
Poor yield (~50%)
Benzene / HMPA
Et 25 °C, 1 h Et

O Br O LnBr Poor yields of


Ln, "pinch" I2 homoaldolate (50-60%)
THF due to competition with
MeO RT, 5 min MeO McMurray coupling of
Ln = La, Ce, Nd and Sm aldehyde

Yoshida, Tetrahedron Lett., 1985, 26, 5559


Yoshida, Angew.Chem.,Int.Ed.Engl., 1987, 26, 1157
Fukuzawa, Chem. Commun., 1986, 475

Enolate Homologation
Synthesis of Zinc Homoenolates

OTMS O ZnX
1. MeLi, 25 °C

2. ZnEt2, CH2I2

• less reactive than most zinc homoenolates


• can also be used to form aldehyde homoenolates

H OTMS
H O
1. MeLi, 25 °C
ZnX
2. ZnEt2, CH2I2

Knochel, J.Org.Chem., 1993, 58, 2694

30A-06 12/1/03 11:23 AM


Direct Tin-Titanium Exchange
New Route to Titanium Homoenolates
• Treatment of β−tri-n-butylstannyl esters with TiCl4 directly forms titanium homoenolate

O SnBu3 O TiCl3
TiCl 4 R = Me, iPr
+ Bu3SnCl
RO CH2Cl2 RO

• Isotope labelling studies showed rxn does not proceed via cyclopropane

• Substrates can easily be prepared by two methods:

O O SnBu3
Bu3SnH, 80 °C, 4 h
MeO MeO

O O SnBu3
1. LiHMDS, -70 °C, 15 min
i i
PrO 2. ICH2SnBu3 PrO
Goswami, J.Org.Chem., 1985, 50, 5907
van der Kirk, J. Appl. Chem., 1957, 7, 356
Still, J.Am.Chem.Soc., 1978, 100, 1481

The First Homoaldol Reaction


Synthesis of γ-hydroxyesters and γ-lactones
O
O TiCl3 O O
R'CHO + +
CH2Cl2 R' O R'
RO RO RO
OH R' Cl

Aldehyde R Temp (°C) Time (h) Workup Product Yield


O
O
Et 0 1 Acidic O 81
H n-Oct
n-Oct
O O Me
Ph i 67
Pr 0 2.5 Neutral
H i
PrO Ph (85:15)
Me OH
O
O
Ph
Et 0 1.5 Neutral EtO 90
H Ph
Cl

Nakamura, J.Am.Chem.Soc., 1977, 99, 7360


30A-07 12/1/03 11:24 AM
Homoaldol Reactions with Alkoxide-modified Homoenolates
Homoaldol Reactions with Aromatic Aldehydes and Ketones
O
O
O TiCl3 1 2
O 2
R R R +
+ 1/2 Ti(OR)4 R1 O
i CH2Cl2 i
PrO
PrO 1 hour
OH R1
R2
A B

Electrophile R Temp(°C) Yield (A or B)

i 90(A)
Benzaldehyde Pr 0

i
Crotonaldehyde Pr 0 88(A)

i
Acetophenone Pr 20 66(A), 12(B)
t
Bu 20 93(B)

Cyclohexanone i 20 62(B)
Pr
t
Bu 20 91(B)

t 91(B)
Bu 20
dr = 88 : 12
Me O equatorial attack

Nakamura, J.Am.Chem.Soc., 1986, 108, 3745

Homoaldol Reactions of Zinc Homoenolates


First Catalytic Homoaldol Reactions

OTMS cat. ZnX2 O


1.2 eq + RCHO
OEt CH2Cl2, RT EtO R

OTMS
• TMSCl generated is essential (i.e. no reaction if removed in vacuo for stoichiometric case)

Catalyst, yield
Aldehyde ZnCl2 (30-50 mol%) ZnI2 (0.1-1 mol%)

PhCHO 84 89

Ph 94 84
CHO

O CHO
91 95
O

O2N CHO -- 84

n-Pent CHO 79 --
93:7 syn : anti
OBn chelation product

Nakamura, J.Am.Chem.Soc., 1987, 109, 8056


30A-08 12/1/03 11:27 AM
Homoaldol Reactions of Zinc Homoenolates
Proposed Catalytic Cycle

EtO OTMS

O ZnX
+ Me3SiX
ZnX2
EtO

X
O RCHO
O ZnX SiMe3
R O
EtO +
EtO R H
OTMS

• with ZnI2, the homoenolate is reactive enough to add to ketones:


OTMS
O OEt 1 mol% ZnI2
+ 1.2 eq OEt
Ph Me OTMS CH2Cl2, RT Ph
Me
77%
O
- no reaction even with stoichiometric ZnCl2

Nakamura, J.Am.Chem.Soc., 1987, 109, 8056

Gleason's Homoaldol reaction


First Catalytic Titanium Homoaldol Reaction
1. 0.1 equiv. TMSOTf
0.1 equiv. (R)-Binol-Ti(OiPr)2
EtO OTMS O O R2
+ 3:1 CD3CN / CDCl3 O
1.5 eq
2. pTsOH R1
R1 R2

R1 R2 Conditions Yield

Ph H 0 °C, 24 h 99

Ph H 0 °C, 36 h 76

TMS H 0 °C, 36 h 82

Cl H 0 °C, 80 h 84

t
Bu H 45-50 °C, 54 ha 52

Ph Me 45-50 °C, 60 ha 78
a
2 equiv. of cyclopropane used

Gleason, Org. Lett., 1999, 1, 1643

30A-09 12/1/03 11:28 AM


Gleason's Homoaldol reaction
Proposed Catalyst and Catalytic Cycle
Catalyst:

O OiPr TMSOTf O OiPr


Ti Ti + TMSOiPr
O OiPr O OTf
observed in
1
H-NMR
• catalyst appears as
mixture of interconverting
species in NMR at RT
Catalytic cycle:
EtO OTMS OiPr
O
i
Ti
O O Pr O + TMSOTf
Ti
O OTf O
OEt

OiPr OTf RCHO


O O TMS
Ti O
Ph O +
EtO
OTMS O R H
Gleason, Org. Lett., 1999, 1, 1643
OEt

Diastereoselective Homoaldol Reactions of Amide-homoenolates


Synthesis of syn- or anti-β-methyl-γ-hydroxyamides

1. Zn•Cu, benzene/DMA O Me
2. (iPrO)3TiCl, THF, 0°C
i R
3. RCHO, THF, 0 °C → RT Pr2N
O I Method A syn
OH
i +
Pr2N Me O Me
1. Activated Zn, 0.1 equiv. TMSCl
CH2Cl2, RT, 1 hour i R
Pr2N
2. RCHO, 2 equiv. TMSCl anti
CH2Cl2, RT OH
Method B

R Method Time (h) Yield syn:anti

Ph A 3 79 94 : 6
B 2.5 82 13 : 87
o-MeOPh A 3 87 85 : 15
B 0.6 95 25 : 75
2-furyl A 0.5 61 96 : 4
B 0.6 60 38 : 62

Asaoka, J.Chem.Soc. Perkin Trans. 1, 1995, 285


30A-10 12/1/03 11:29 AM
Tandem Asymmetric Enolate Homologation - Homoaldol Reaction
Asymmetric Synthesis of α-alkyl, γ-hydroxy Carbonyl Compounds

• idea: O ICH2Zn O O
1. nBuLi 1. Cl3TiOiPr
R R'
Xc 2. Zn(CH2I)2 Xc 2. R'CHO Xc
R OH R
2 new stereocenters
generated

O
Xc = Me
Me N

• initial results: 1. nBuLi O


O
2. Zn(CH2I)2
Bn Me But ReactIR showed disappearance of
Xc 3. H + Xc enolate and appearence of new species
Bn 30-40%

McWilliams, J.Am.Chem.Soc., 1996, 118, 11970

Tandem Asymmetric Enolate Homologation - Homoaldol Reaction


Asymmetric Synthesis of α-alkyl, γ-hydroxy Carbonyl Compounds
• proposal: zinc enolate unreactive towards homolagation; higher order zincate (zincate + extra enolate)
active species in migration
Bn
OLi
OLi O
Bn Xc O O
Bn Zn(CH2I)2 (ICH2)2Zn Xc
Xc Xc (ICH2)2Zn
Xc
Bn
Bn
O OR' • involvement of higher
+
• 2 eq. enolate Me H ICH2Zn O order zincates in 1,2-
needed so max Xc migrations is known
yield is 50% (Harada)
Bn Xc
Bn
• idea: add an equivalent of alkoxide to take the place of the enolate in the higher order zincate

1. nBuLi O ROLi Conversion


O
2. ROLi EtOLi 35
Bn Me
3. Zn(CH2I)2 Xc nPrOLi 74
Xc BnOLi 82 (78% isolated)
4. H +
Bn LiO(CH2)2OLi 31

McWilliams, J.Am.Chem.Soc., 1996, 118, 11970


Harada, J.Org.Chem., 1993, 113, 2958
30A-11 12/1/03 11:30 AM
Tandem Asymmetric Enolate Homologation - Homoaldol Reaction
Asymmetric Synthesis of α-alkyl, γ-hydroxy Carbonyl Compounds

• How about the homoaldol reaction? BnO


1. 2 equiv. Cl3TiOiPr
O ICH2Zn O O
1. nBuLi, <-65 °C -80 °C → T °C
R R'
Xc 2. Zn(CH2I)2, Xc 2. R'CHO, T °C Xc
3 equiv. BnOLi
-70 °C R OH R

Aldehyde (R') R T (°C) de (%) Yield

Bn Bn -20 ≥99 59

BocHN Me -20 82 58

phenyl Bn -40 82 50

phenyl Me -40 80 44

iso-propyl Bn -50 76 53

n-butyl Bn -20 64 53

McWilliams, J.Am.Chem.Soc., 1996, 118, 11970

Reactive Homoenolates
First Synthesis of a Metal-free Homoenolate

TMS O Bu4N O E E O
TBAT
OMe OMe OMe

• reactive enough to add to imines, as well as aldehydes and ketones

e.g.: Ph Ph
TMS O 1. TBAT
Ph NHPh N
O
+ O
2.
OMe Ph
Ph N Ph OMe Ph
single diast. 1:1 dr
22% 60%

• substrate made easily using Nishiguchi method


TMS O
O
-
Mg anode, 2 e
OMe
OMe TMSCl, DMF

Fry, Tetrahedron Lett., 1999, 40, 7945


Nishiguchi, Tetrahedron Lett., 1992, 33, 5515

30A-12 12/1/03 11:31 AM


Conjugate Addition of Zinc Homoenolates
Synthesis of δ,ε-(silylenolether)-esters

O OTMS
2
Zn(CH2CH2CO2Et)2 (1.2 equiv.)
1 R CuBr•DMS (2 mol%) 1 R2
+ + R R
Et2O / HMPA
TMSCl (2.4 equiv.) R3 OEt
Important!! R3 R4
R4
O
Enone Product Yield Enone Product Yield

O OTMS O OTMS

93 H Me H Me 75

91:9
(CH2)2CO2Et Me Me (CH2)2CO2Et
O OTMS
72:28 Me OTMS
O
78
73
Me
(CH2)2CO2Et Me
Et (CH2)2CO2Et
Me Me

Nakamura, J.Am.Chem.Soc., 1987, 109, 8056


Nakamura, Org. Synth., 1987, 66, 43

Acylation of Zinc Homoenolates


Synthesis of γ-ketoesters - Yoshida

O
O ZnI + O 4 mol% Pd(PPh3)4
1.5 equiv. R
Benzene / DMA EtO
EtO R Cl RT, 30 min O

R Yield

Ph 100

90
Me

Ph 92

Cl 100

Yoshida, Tetrahedron Lett., 1985, 26, 5559

30A-13 12/1/03 11:33 AM


Acylation of Zinc Homoenolates
Synthesis of γ-ketoesters - Nakamura

OR1 O
0.5 equiv. + O 5 mol% PdCl2(PPh3)2 R2
O Zn O R 1O
Et2O, RT
R2 Cl O
R 1O
• note: only 0.5 equiv. of Zn species needed so both homoenolates are transferred

R1 R2 Yield

93 • when carried out in CDCl3 got


Et Ph quantitative O-acylation (with or
Me without Pd)
i 81
Pr i.e. OR1
Me
OCOR2
Et Ph 83

i t
Pr Bu 50

Nakamura, J.Org.Chem., 1987, 26, 8056

Application of Homoenolate Acylation


Synthesis of α,β-disubstituted γ-butyrolactones by Diastereoselective Reduction

Me O Me O
DIBAL
R R
NiPr2 THF, -78 °C, 10 min NiPr2
O OH
Prepared by dr 100:0 for all substrates
Yoshida method MsCl / Et3N
CH2Cl2
0 °C, 1 h
Me Me O
MsO R
NiPr2
Silica Gel R NiPr2
O
1-3 days OMs

R Yields
Reduction Cyclization
Me
Ph 87 82

R O p-ClPh 93 95
O
p-MeOPh 90 92

Bn 62 83

Asaoka, Heterocycles, 2000, 52, 227


30A-14 12/1/03 11:34 AM
Arylation and Vinylation of Zinc Homoenolates
Synthesis of β-vinyl and β-aryl esters - Yoshida

O
O ZnI X
+ 1-4 mol% PdCl2(P(o-Tol)3)2
Benzene-DMA EtO
EtO
60 °C
X = I, OTf 0.5-1 h

Coupling Partner Yield

t 74 Only vinylation example given


Bu OTf

MeO I 96

Br I 67

O2N I 80

Yoshida, Tetrahedron Lett., 1986, 27, 955

Arylation and Vinylation of Zinc Homoenolates


Synthesis of β-vinyl and β-aryl esters - Nakamura

OR O
X
1-2 mol% PdCl2(P(o-Tol)3)2
1.5 equiv. O +
Zn O RO
THF, 0-20 °C
RO
R = Et, iPr X = I, Br, OTf

Coupling Partner Yield

Bu 90
I
SiMe3
Me 87
Br
X = OTf 0
X Br 67
I 79

Me
Br 49
O

Nakamura, J.Org.Chem., 1987, 26, 8056

30A-15 12/1/03 11:35 AM


Arylation of Palladium Homoenolates
Catalytic Formation of β−aryl Ketones

R1 R2 O
[PdCl(C3H5)]2 (5 mol%)
1.5 equiv. OTMS+ ArOTf
Ph3P (20 mol%) R2 Ar
HMPA, 100 °C, 12 h
R1
• proposed to proceed via O PdArLn

R2
R1

R1 R2 Ar Yield

-CH2CH2CH2CH2- 1-napthyl 84

-CH2CH2CH2CH2- p-NO2Ph 68

H p-OMePh Phenyl 65

n-Heptyl H 1-napthyl 58

Nakamura, J.Am.Chem.Soc., 1988, 110, 3296

Allylation of Zinc Homoenolates


Synthesis of δ,ε-unsaturated Esters - Yoshida
O

EtO SN2'
O ZnI
R1 X 20 mol% CuCN 1 R2
+ R
+
EtO THF / DMA O
R2 RT, 24 h
R1
EtO SN2
2
R

R1 R2 X Yield SN2' : SN2

H H OTs 89 --

Ph H OTs 80 87 : 13
Br 93 88 : 12
Cl 99 87 : 13

CO2Me H Br 80 100 : 0

Yoshida, J.Org.Chem., 1987, 52, 4418

30A-16 12/1/03 11:35 AM


Allylation of Zinc Homoenolates
Synthesis of δ,ε-unsaturated Esters - Nakamura

OiPr O
i SN2'
PrO
O Zn O R1 Cl 1 R2
+ 5 mol% CuBr•Me2S R
i
+
PrO 2 THF / DMF O
R RT, 24 h
R = Et, iPr
i R1
PrO SN2
2
R
Allyl chloride Yield SN2' : SN2

97 96 : 4
Ph Cl
65 1 : 99 Catalyst changed to
5 mol% NiCl2•dppe
Me Me
81 88 : 12
Me Cl
Me Me
72 100 : 0
AcO
Cl

Nakamura, J.Am.Chem.Soc., 1987, 109, 8056

Carbonylative Symmetrical Coupling of Palladium Homoenolates


Catalytic Synthesis of 4-keto Pimelates

CO (1 atm)
O O
OTMS PdCl2(PPh3) (2 x 2.5 mol%)
OR CDCl3, 60 °C, 32 h RO OR
i
R = Et, Pr, n-Hex O Yields: 65-80%

• proposed to proceed via O PdArLn

RO

• evidence
O O
OTMS PdCl2(PPh3) (2 x 2.5 mol%) +

OR CDCl3, 60 °C, 32 h RO RO

Nakamura, Tetrahedron Lett., 1988, 29, 1541

30A-17 12/1/03 11:36 AM


Crimmins' Cyclopentenone Synthesis
Introduction and Generality

X Yields
O O O
1.2 equiv. O
OR 50-60%
X EtO 2
Zn X
R TMSCl, 2.4 equiv. N 80-90%
CuBr•Me2S, 0.15 equiv. R
HMPA, 2.4 equiv.

O O O
2.4 equiv. O OR 50-80%

X EtO 2 Zn X
R N 80-90%
TMSCl, 4.8 equiv.
R
CuBr•Me2S, 0.3 equiv.
OH HMPA, 4.8 equiv.
HO

Functionality supported in R: ethers, epoxides, furans, α,β unsaturated esters

Crimmins, J.Org.Chem., 1993, 58, 1038

Crimmins' Cyclopentenone Synthesis


Mechanistic Considerations
Cl
Si TMSO OEt
M O
O
H CO2Et
EtO
OEt H+
M = Cu or Zn R
R EtO2C R
EtO
main side product
O
ZnX
• problem: two steps have opposite
electronic requirements
• appears amide and ester have right balance;
ketone too electron poor to cyclize O
O
i.e. TMSO Me
O OEt
O
EtO 2
Zn R
Me
TMSCl
R CuBr•DMS
HMPA EtO
80%
O
Crimmins, J.Org.Chem., 1993, 58, 1038

30A-18 12/1/03 11:37 AM


Leahy Cyclopentannulation
Synthesis of 3,3-disubstituted cyclopentanones

O
1.2 equiv. EtO2C
1
R
EtO 2 Zn R2
TMSCl, 2.4 equiv. R1
OHC R2 CuBr•DMS, 0.5 equiv.
HMPA, 2.4 equiv. TMSO

H2SO5, EtOH
67% avg (from enal)

1. NaH, PhH, EtO2C


O
2 EtOH (cat)
R R2
2. HCl
R1 R1
77% avg EtO2C

Substrates studied:
R1 = R2 = n-Bu
R1 = R2= Ph Leahy, J.Org.Chem., 1994, 59, 5496
R1 = Ph, R2 = Me
R1 = Ph, R2 = H

Tandem Aldol / Aldol / Homoaldol Reaction


One-pot synthesis of a 6,7,6-tricycle

OH 1. CH3MgBr, THF, 0°C OH


Relative stereochemistry verified
OEt 2. OLi OH by x-ray crystallography
H
67%
H
-78 → 25 °C O

A possible mechanism:
O
OH CH MgBr OMgBt
3 O
OEt OEt MgBr

O
O
O
O OH OH O Mg O O MgBr

Helquist, J.Am.Chem.Soc., 1986, 108, 8313

30A-19 12/1/03 11:45 AM


Synthetic Examples
Depresosterol
OH
Me CHO Me OiPr
Me O TiCl2OiPr Me
H H O
Me i
PrO Me

H H 60%, > 6:1 dr H H


AcO AcO
• trichlorotitanium homoenolate is not reactive enough to add to hindered aldehyde

Nu
H Me
H O
Via

Nakamura, J.Am.Chem.Soc., 1985, 107, 2138

Synthetic Examples
Depresosterol - completion of the formal synthesis

O
OH 1. MsCl, Et3N O
Me 2. KOH, MeOH, ∆
CO2iPr 3. aqueous HCl
Me
R 4. TBSCl, Et3N
R = steroid ring system (reprotects steroid OH) R
84% 1. LDA, -78 → -10 °C
OAc 2. CH2O (g)
OAc 3. aqueous HCl
70%, ~10:1 dr
Me Me
Me
O
H MeOH
Me 1. MeLi, THF, 45 °C O OH
2. Aqueous AcOH, rt Me
H H 3. Ac2O, pyr., rt
46% R
AcO
Triacetyldepresosterol
Kashman

Depresosterol
Nakamura, J.Am.Chem.Soc., 1985, 107, 2138
Kashman, Tetrahedron, 1981, 37, 2397
30A-20 12/1/03 11:46 AM
Synthetic Examples
Depresosterol - Maximizing Homoenolate Functionality

O
OH
O Me OH
Me O Me Me
Me Me Me
H O H MeOH

Nakamura, J.Am.Chem.Soc., 1985, 107, 2138

Synthetic Examples
Pumiliotoxin 251D

Me O TiCl3
1.Cp2Ti Al 6.2 eq
OMe Me Me
Cl EtO
N N 49%
Cbz 2. 1 N HCl Cbz
O O
87%
Me OH Me OH
H H
O
3 steps H2, Pd/C
100% N OEt
N Gallagher N CbzH
Me OH
Me O "single diastereomer"
Me
Pumiliotoxin 251D

Nu

via:
N O
H Ti
CBz Ln Nu

Barrett, J.Org.Chem., 1999, 64, 1410


Gallagher, J.Am.Chem.Soc., 1991, 113, 2652
30A-21 12/1/03 12:44 PM
Synthetic Examples
EtO O (±)-Cortisone

6 steps

O Me
Me 2.6 Me
Me equiv. O
O i
PrO Zn
2 (±) OH
(±) - OTMS CuBr•DMS, 0.15 equiv. O
HMPA, 2.6 equiv. H
H BF3•OEt2, 4.8 equiv. i • reaction in absence of
>95:5 dr, >80%y PrO lewis acid was unselective

OH

O
Me
OH 7 steps
Me

H H
O
(±)-cortisone Nakamura, J.Org.Chem., 1986, 51, 4324

Synthetic Examples
(±)-Ginkgolide B

O
O
CO2Et
EtO2C O EtO 2 Zn O
1.2 equiv.
(±)
TMSCl, 2.4 equiv.
t CuBr•DMS, 0.25 equiv.
OTES Bu HMPA, 2.4 equiv. OTES tBu
THF, 23 °C, 4 h
82% hν (366 nm)
hexanes
100%, 98:2 dr
O O HO OH O O CO2Et
O
Me O
HO
O O t
Bu t
O TESO Bu
(±)-Ginkgolide B

Crimmins, J.Am.Chem.Soc., 1999, 121, 10249

30A-22 12/1/03 12:45 PM


D. A. Evans Carbocations: Stability & Structure Chem 206

Other Relevant Background Reading


http://www.courses.fas.harvard.edu/~chem206/
March, Advanced Organic Chemistry, 4th Ed. Chapter 5, pp165-174.
Lowery & Richardson, Mech. & Theory in Org, Chem., 3rd Ed. pp
383-412.
Chemistry 206 Arnett, Hoeflich, Schriver in Reactive Intermediates Vol 3, Wiley, 1985,
Chapter 5, p 189.
Advanced Organic Chemistry Saunders, M. and H. A. Jimenez-Vazquez (1991). “Recent studies of
carbocations.” Chem. Rev. 91: 375.

Lecture Number 30 Stang, P. J. (1978). “Vinyl Triflate Chemistry: Unsaturated Cations and
Carbenes.” Acc. Chem. Res. 11: 107.

Introduction to Carbonium Ions Olah, G. A. and G. Rasul (1997). “Chemistry in superacids .26. From Kekule's
tetravalent methane to five-, six- and seven-coordinate protonated methanes.”
Acc. Chem. Res. 30(6): 245-250.
■ Carbocation Stabilization
Olah, G. A. (1995). “My search for carbocations and their role in chemistry
■ Carbocation Structures by X-ray Crystallography (Nobel lecture).” Angew. Chem., Int. Ed. Engl. 34, 1393-1405

■ Vinyl & Allyl Carbonium Ions Qumulative Exam Question Fall, 2001. The reaction illustrated below was recently
reported by Snider and co-workers (Org. Lett. 2001, 123, 569-572). Provide a mechanism for
Reading Assignment for this Lecture: this transformation. Where stereochemical issues are present, provide clear three
dimensional drawings to support your answer.
Carey & Sundberg, Advanced Organic Chemistry, 4th Ed. O O
Part A Chapter 5, "Nucleophilic Substitution", 263-350 . R EtAlCl2
R
Me
CH2Cl2, 0 °C
Birladeanu, L. (2000). "The Story of the Wagner-Meerwein Rearrangement.” Me
J. Chem. Ed. 2000, 77, 858. (handout) Me Me
Me Me
Olah, G. A. (2001). “100 Years of Carbocations and their Significance in
Chemistry.” J. Org. Chem. 2001, 66, 5944-5957. (handout) Carey & Sundberg-A, p 337: Provide mechanisms for the following reactions.
Walling, C. (1983). “An Innocent Bystander Looks at the 2-Norbornyl Cation.” OH
Acc. Chem. Res. 1983, 16, 448. (handout) CHO
NH2
NaNO2
Laube (1995). “X-Ray Crystal Structures of Carbocations Stabilized by Bridging OH O
or Hyperconjugation.” Acc. Chem. Res.1995, 28,: 399 (handout) HOAc/H2O NH2
NaNO2

Wednesday, HOAc/H2O
D. A. Evans December 3, 2003
CMe3 CMe3

31-00-Cover Page 12/3/03 8:35 AM


D. A. Evans. B. Breit Carbocations: Stability Chem 206

Carbocation Subclasses Hydride ion affinities (HI)


–27 –18
Carbon-substituted Heteroatom–stabilized Me CH2 Me2 CH Me3 C
R1 ⊕ R ⊕ R 249
R1 O N 276 231

R3 ⊕ R2 R3 R2 R3 R2 +21 +81
H 3C CH2 H 2C CH HC C
R–R3 = alkyl or aryl R–R3 = alkyl or aryl R–R3 = alkyl or aryl
276 287 386
The following discussion will focus on carbocations unsubstitutred with heteroatoms

Classical vs nonclassical carbonium ions –37 –20


C C Ph CH2 Me CH2 H 2C CH CH2
C C
⊕ ⊕ ⊕ ⊕
C 239 276 256
C C C C C C C
open hyperconjugation unsymmetrical symmetrical The effect of beta substituents: Rationalize
trivalent no bridging bridging bridging
–7
Me CH2 Me–CH2 CH2
increasing nonclassical character
classical nonclassical 276 270

Stability: Stabilization via alkyl substituents (hyperconjugation)

Order of carbocation stability: 3˚>2˚>1˚ Hydride ion affinities versus Rates of Solvolysis
R H H H
> H C > H C Due to increasing number of substituents PhCH2–Br CH=CH–CH2–Br Me2CH–Br
R C > R C
capable of hyperconjugation
R R R H
rel rate 100 52 0.7
The relative stabilities of various carbocations 239 256 249
Hydride ion HI
can be measured in the gas phase by their affinities
affinity for hydride ion. ∆-HI 0 +17 +10
CH3+ 314
CH3CH2+ 276
R + H R–H + HI
(CH3)2CH + 249 Relative Solvolysis rates in 80% EtOH, 80 °C
Hydride Affinity = –∆G° (CH3)3C +
231 A. Streitwieser, Solvolytic Displacement Reactions, p75

∆HI increases → C(+) stability decreases H2C=CH+ 287


H C C+ 386
Note: As S-character increases, cation stability PhCH2+ 239
decreases due to more electronegative carbon. Conclusion:
Gas phase stabilities do not always correlate with rates of solvolysis
J. Beauchamp, J. Am. Chem. Soc. 1984, 106, 3917. Carey & Sundberg–A, pp 276-

31-01 Carbocations-1 12/3/03 8:08 AM


M. Shair, D. Evans Carbocation Generation & Stability Chem 206

Carbocation Stability: The pKR+ value Removal of an energy-poor anion from a neutral precursor via Lewis Acids

Definition: R + + H 2O ROH + H+
R 3C X + LA R 3C + LA–X
aROH ⋅ aH+
KR+ = a = activity
aR+ ⋅ aH2O
LA: Ag , AlCl3, SnCl4, SbCl5, SbF5, BF3, FeCl3, ZnCl2, PCl3, PCl5, POCl3
...
pKR+ = – log KR+ X: F, Cl, Br, I, OR
Carey & Sundberg, A, p 277

Table: pKR+ values of some selected carbenium salts Acidic dehydratization of secondary and tertiary alcohols

(4-MeO-C6H4)3C Ph3C (3-Cl-C6H4)3C Ph2CH least stable


0.82 – 6.63 – 11.0 – 13.3 - H2O R 3C
R3C OH + H–X + X

CH2 CHPh CHPh R CPh2 R: Aryl + other charge stabilizing substituents


Fe Fe
Cr(CO)3 Co2(CO)6 X: SO42-, ClO4-, FSO3-, CF3SO3-
0.40 0.75 –10.4 –7.4
From neutral precursors via heterolytic dissociation (solvolysis) - First
H7C3 step in SN1 or E1 reactions
+ C 3H 7 most stable solvent
R 3C X R 3C + X
H7C3
7.2 4.77 Carey & Sundberg, A, pp 276-
Ability of X to function as a leaving group:
-N2+ > -OSO2R' > -OPO(OR')2 > -I ≥ -Br > Cl > OH2+ ...
Carbocation Generation

Hydride abstraction from neutral precursors Addition of electrophiles to π -systems


R R H R R
H chemistry
R 3C H + Lewis-Acid R 3C R R R R

H R
R R R chemistry
H H RS R2N
H H H
R 3C H = etc.
H H H H Provide a Mechanism of this transformation
RS R 2N

H2SO4
Lewis-Acid: Ph3C BF4, BF3, PCl5 HC C CH2OH J.C.S.,CC 1971, 556
Br Me

31-02-carbocation-2 12/3/03 8:14 AM O


D. A. Evans, B. Breit Carbocations: Structure Chem 206

Carbocation Stabilization Through Hyperconjugation Physical Evidence for Hyperconjugation: The Adamantyl Cation

Bonds participating in the hyperconjugative interaction, e.g C–R,


R + R+
will be lengthened while the C(+)–C bond will be shortened.
H H
H C C C C
H H
H H First X-ray Structure of an Aliphatic Carbocation

■ FMO Description 1.431 Å


Take linear combination of σ C–R (filled) and C pz-orbital (empty): [F5Sb–F–SbF5]–

E
+ +
C
σ∗ C–R σ∗ C–R
Me
1.608 Å Me
100.6 °
Me
+ +
H H
C C
H H

T. Laube, Angew. Chem. Int. Ed. 1986, 25, 349


σ C–R σ C–R

Syn-planar orientation between interacting orbitals

C–H versus C–C Hyperconjugation: The Adamantane Reference


Calculated carbocation (MM-2)
The t-Pentyl Cation agrees with solution
1.528 Å
structure
H

Me +
Me 110 °
H C C 1.582 Å 1.092 Å
Me Me 1.530 Å
Me
H
+ Me

1.107 Å
R. P von Schleyer in
Stable Carbocation Chemistry, 1997, p 46-47

31-03 Carbocations-3 12/3/03 8:14 AM


D. A. Evans, K. Scheidt Carbocations: Structure Chem 206

Me
F SbF5 Me + –
[F5Sb–F–SbF5]– Me F F5Sb F SbF5
C Me
+ Me
C Me
Me 2 SbF5
Me
Me Me
Me Me

1.439 Å
1.421 Å C–C1: 1.439 Å
1.466 Å + C–C2: 1.446 Å
+ C–C3: 1.442 Å

1.622 Å
1.621 Å 98.2 °

1.608 Å


F5Sb F SbF5

1.551 Å

T. Laube, Angew. Chem. Int. Ed. 1986, 25, 349

1.508 Å
reference structure:
CSP3–Csp2 bond length
1.342 Å
T. Laube, JACS 1993, 115, 7240

31-04 Carbocations-4 12/3/03 8:15 AM


D. A. Evans, K. Scheidt Carbocations: Structure Chem 206

Cl Cl Cl Cl Me
SbF5 SbF5 Me +
+ Me F C Ph + SbF6–
Cl + SbClF5– Me
Ph

122 °
1.668 Å

Cl 1.432 Å
1.408 Å
+ +
1.422 Å
1.432 Å
1.371 Å
Cl 1.725 Å 1.491 Å

117.8 °
1.442 Å

1.446 Å
121.0 °
+
1.505 Å
1.439 Å
121.2 °

1.508 Å
T. Laube, JACS 1993, 115, 7240
reference structure: 1.342 Å
CSP2–Csp2 bond length
reference structure:
CSP3–Csp2 bond length

31-05 Carbocations-5 12/3/03 8:15 AM


D. A. Evans, K. Scheidt Carbonium Ion X-ray Structures: Bridged Carbocations Chem 206

Ph Cl Ph
+ C
Me AgSbF6
Me Me
Me F –
[F5Sb–F–SbF5]
+ Me Me
Me 2 SbF5 Me –
H H
F5Sb F SbF5

Me Me
Me Me

T. Laube, Angew. Chem. Int. Ed. 1987, 26, 560

1.495 Å
1.467 Å
2.092 Å
1.739 Å**

+
1.855 Å
1.467 Å +
1.442 Å

**One of the longest documented C–C bond lengths.


1.503 Å

C
C
⊕ ⊕
C C C C
T. Laube, JACS 1989, 111, 9224
hyperconjugation unsymmetrical
no bridging bridging

31-06 Bridged cations 12/3/03 8:15 AM


D. A. Evans, K. Scheidt Carbonium Ion X-ray Structures: A Summary Chem 206

(ref 1.513 Å)Ph–C(Me)=CH2


1.442 Å
1.446 Å 1.408 Å
+ 2.092 Å 1.739 Å
+

1.439 Å
1.371 Å 1.432 Å 1.467 Å
1.491 Å +
1.442 Å

1.508 Å
1.668 Å

1.342 Å Cl 1.432 Å
+
1.422 Å
Cl
1.725 Å
1.421 Å 1.467 Å 1.495 Å
1.466 Å
+ +
1.855 Å
1.621 Å 98.2 ° 1.622 Å

1.608 Å

Nomenclature: classical vs nonclassical 1.503 Å


C C
1.551 Å C C
⊕ ⊕ ⊕ ⊕
C C C C C C C C
open hyperconjugation unsymmetrical symmetrical
trivalent no bridging bridging bridging

increasing nonclassical character


classical nonclassical

31-07 C(+) Summary 12/3/03 8:16 AM


D. A. Evans, B. Breit Vinyl & Allyl Carbocations Chem 206

Vinyl & Phenyl Cations: Highly Unstable Allyl & Benzyl Carbocations

Evidence suggests that vinyl cations are linear. Carbocation Stabilization via π-delocalization

OTf HOSolv OSolv

R
R
R
R
As ring size decreases, the rate of hydrolysis also diminishes. Implying that the
formation of the linear vinyl cation is disfavored due to increasing ring strain.
allyl cation

R
R R H+ C C R Hyperconjugation
D ■ Stabilization by Phenyl-groups
D OTf
The Benzyl cation is as stable as a t-Butylcation. This is shown in the
A secondary kinetic isotope effect was measured to be KH/KD = 1.5 (quite subsequent isodesmic equations:
large) indicating strong hyperconjugation and an orientation of the vacant p
orbital as shown above. ∆H0r
[kcal/mol]
P. J. Stang J. Am. Chem Soc. 1971, 93, 1513; P. J. Stang J.C.S. PT II 1977, 1486.
(CH3)3C + PhCH3 (CH3)3CH + PhCH2
3.8
Hydride ion affinities (HI)
(CH3)3C + PhCH2Cl (CH3)3CCl + PhCH2 - 0.8
+21 +81
H 3C CH2 H 2C CH HC C
–8
276 287 386 Ph CH2 Me3 C
Hydride ion affinities (HI)
239 231
+11
H 2C CH Phenyl Cations Hydride ion affinities versus Rates of Solvolysis
287
298 PhCH2–Br Me2CH–Br CH=CH–CH2–Br
The ring geometry opposes rehybridization (top) so the vacant orbital retains 100 0.7 52
sp2 character. Additionally, the empty orbital lies in the nodal plane of the
HI 239 249 256
ring, effectively prohibiting conjugative stabilization.
∆-HI 0 +10 +17
Relative Solvolysis rates in 80% EtOH, 80 °C
A. Streitwieser, Solvolytic Displacement Reactions, p75

31-08 Vinyl/allyl (+) 12/3/03 8:17 AM


D. A. Evans, B. Breit Cyclopropyl-carbinyl & Bridgehead Carbocations Chem 206

Carbocation Stabilization via Cyclopropylgroups Solvolysis rates represent the extend of that cyclopropyl orbital overlap
contributing to the stabiliziation of the carbenium ion which is involved as a
reactive intermediate:

C OTs
Me OTs
Cl
Me
See Lecture 5, slide 5-05 for discussion of Walsh orbitals krel = 1
krel = 1 krel = 1
Me Why??
A rotational barrier of about NMR in super acids OTs
Me
13.7 kcal/mol is observed in δ(CH3) = 2.6 and 3.2 ppm OTs
following example:
H Cl
O
X-ray Structures support this orientation 1.222 Å krel = 106 krel = 108 krel = 10-3
1.517 Å
R Carey–A, p 286

1.302 Å 1.478 Å 1.474 Å Bridgehead Carbocations

Me
1.464 Å Me OTs why so
Me reactive?
1.541 Å TsO TsO
TsO

1.409 Å 1 10-7 10-13 104


1.444 Å
Bridgehead carbocations are highly disfavored due to a strain increase in
achieving planarity. Systems with the greatest strain increase upon passing
1.534 Å from ground state to transition state react slowest.

24 ° why so reactive?

–TsO

TsO

R. F. Childs, JACS 1986, 108, 1692

31-09 Cyclopropyl-carbinyl C+ 12/3/03 9:11 AM


D. A. Evans, J.Tedrow A Stable Hypervalent Carbon Compound ? Chem 206

"The Synthesis and Isolation of Stable Hypervalent Carbon Compound (10-C-5) Bearing a 1,8-Dimethoxyanthrecene Ligand"

Akibe, et al. JACS 1999, 121, 10644-10645

+
MeO OMe
CO2Me Me Me
OMe OMe Me3O+BF4– O C O

B2F7–

"The relevant C–O distances are longer than a covalent C–O bond
(1.43 Å) but shorter than the sum of the van der Waals radii (3.25 Å)."

2.428 Å

2.428 Å 1.483 Å
2.452 Å

2.452 Å

For a recent monograph on hypervalent Compounds see:


"Chemistry of Hypervalent Compounds", K. Akiba, Wiley-VGH, 1999

31-10-5 coord (C+) 12/3/03 8:17 AM


D. A. Evans, J.Tedrow Transition Metal-Stablized Carbocations-1 Chem 206

Carbocation Stabilization - d(π ) stabilization Ph


via Transition metal Fragments
+
■ Transition metal not bound directly to carbenium ion: Ferrocenes
Fe Ph

C CH CF CO H C CH
3 2 2
Fe Fe
stable in CF3COOH

1+ 1.385 Å
R R
C C
R R 1.376 Å
Fe Fe
1.391 Å

an important resonance
contribution
1.509 Å

Cyclopropene
MM 2
1.297 Å
R. L. Sime, etal. JACS 1974, 96, 892
1.389 Å 1.428 Å

+
1.412 Å
■ Transition metal not bound directly to carbenium ion: Colbalt-Acetylenes

R
OH 1) Co2(CO)8 R
R C C R
R'' 2) HX
R'
(OC)3Co Co(CO)3

isolable,
basis of the Nicholas reaction
J. Lukasser, etal. Organometallics 1995, 14, 5566-5578
See Houk etal. JACS 1999, 121, 3596-3606

31-11-transition-M C(+) 12/3/03 8:18 AM


D. A. Evans, J. Tedrow Transition Metal-Stablized Carbocations-Pd Catalysis Chem 206

Carbocation Stabilization - d(π ) stabilization Nu =


AcO Nu Nu =
CH(CO2Me)2 BnNH
via Transition metal Fragments
Nu,
CH2Cl2, -20 °C 94% ee 91% ee
Transition metals bound to carbenium ions: π–Allyl Pd(II) Complexes

R OAc 1b Nu
H 1+ R O Me
L4Pd(0) Nu, Ar2P
Nu
R CH2Cl2, -20 °C 94% ee 91% ee
inversion S CHMe2
OAc inversion
Nu CMe3
Pd
OAc– L OAc Nu
L 1b, Ar = α-naphthyl
Nu,
O Me
OAc Nu Ar2P CH2Cl2, -20 °C
96% ee 97% ee
2 mol% [C3H5-PdCl]2, 1
Ph Ph Nu, CH2Cl2, –20 °C Ph Ph S CHMe2 1b
OCO2Et Nu
R Nu,
Nu = CH(CO2Me)2 98% ee
Nu = BnNH 95% ee 1a Ar = Ph CH2Cl2, -20 °C 94% ee 94% ee
BocN BocN conditions: 2 mol% [C3H5-PdCl]2, 2b

Me
Me
CHMe2
O CHMe2
Ph O
Ph
P S P S
P Ph Pd CMe3 Ph CMe3
H H Pd
S C1 C2
Ph C1 C2 Ph
Selected Bond Lengths:
Selected Bond Lengths:
Pd-C1, 2.17Å; Pd-C2, 2.26Å
Pd-C1, 2.16Å; Pd-C2, 2.28Å
Pd-P, 2.25 Å; Pd-S, 2.36 Å
Pd-P, 2.29 Å; Pd-S, 2.38 Å
C1 C2

Nu
Pd-C1, 2.16Å C1 C2
Pd-C2, 2.28Å
Evans, Campos,Tedrow, Michael, Gagné,
JACS 2000, 122, 7905 Pd-C1, 2.17Å
Nu
Pd-C2, 2.26Å

31-12-transition-M C(+)-2 12/3/03 8:18 AM


D. A. Evans Carbocations: Stability, Structure, & Rearrangements Chem 206

Here is a typical carbonium ion question that you should be able to handle by
the end of the course. Write out a mechanism for the following transformation.
http://www.courses.fas.harvard.edu/~chem206/
Me
Me Me
BF3•OEt2
Chemistry 206 Me
O CH2Cl2 H
CH2
Me Me 60% yield
Advanced Organic Chemistry O

Lecture Number 32 Me
Me OBF4
Me
Introduction to Carbonium Ions–2
Me
■ Allyl- & Vinylsilanes: The β-Silicon Effect Me Me
OBF4
■ Carbonium Ion Rearrangements
A. Srikrishna, Chem Commun 1994, 2259
Reading Assignment for this Lecture:
Question 13. Final Exam, 1999. During Corey's synthesis of Aspidophytine
Carey & Sundberg, Advanced Organic Chemistry, 4th Ed. (JACS, 1999, 121, 6771), the pivotal intermediate 3 was assembled by the
Part A Chapter 5, "Nucleophilic Substitution", 263-350 . union of 1 and 2 under the specified conditions. Provide a mechanism for this
single-pot transformation.
Walling, C. (1983). “An Innocent Bystander Looks at the 2-Norbornyl Cation.”
Acc. Chem. Res. 16: 448. (handout) CHO
Birladeanu (2000). “The Story of the Wagner-Meerwein Rearrangement.”
OHC
J. Chem. Ed. 77: 858. (handout) 1) mix at room
Ra Rb 3) excess
CO2R temp, 5 min NaBH3CN
Lambert, (1999). “The β effect of silicon and related manifestations of σ N
conjugation.” Acc. Chem. Res. 32, 183-190. (handout) 2) 2 equiv.
Me3Si + Rc Rd
2 NH2 TFAA, 0 °C
Other Relevant Background Reading
Saunders, M. and H. A. Jimenez-Vazquez (1991). “Recent studies of
carbocations.” Chem. Rev. 91: 375. N
1
N CO2R
MeO
OMe Me
Friday,
MeO N H
D. A. Evans December 5 , 2003 3
MeO Me
32-00-Cover Page 12/5/03 8:52 AM
M. Shair, D. Evans Carbocation Rearrangements-1

Carbocation [1,2] Sigmatropic Rearrangements


Pinacol rearrangement (vicinal diol): Driving force is the
1,2 Sigmatropic shifts are the most commonly encountered cationic rearrangements.
When either an alkyl substituent or a hydride is involved, the term Wagner-Meerwein gen. of C=O
shift is employed to identify this class of rearrangments. OH O
+
H
Birladeanu (2000). “The Story of the Wagner-Meerwein Rearrangement.” J.
Chem. Ed. 77: 858. (handout)
OH

OH
CH2 NH2 CH2
Stereoelectronic requirement for migration.... Deamination
" NO+ "
retention of stereochemistry
Demjanov-rearrangement (Driving force: relief of ring strain)

C C
A D A D Wagner-Meerwein Rearrangements: Application in Total
B bridging T.S. B Synthesis
Me H
Me
Me
H2SO4 H
Me Me Me
C
A D Me OH H Me
B A

2-electron Huckel transition state Me H OH


Me
Me
H
If migration accompanies ionization, the migration terminus will be inverted. Overlap α-caryophyllene alcohol H equiv to Me Me
between the σ C-C (migration origin) and the σ* C-X (migration terminus) will be HO Me
maximized in an antiperiplanar arrangement.
Me H Me
Preparation of A:
MsO H H H O
OH H O
H OH
OH Me Me H
Et2 AlCl RO OH Me Me
TBSO RO hυ MeMgBr
H CH2 Cl2, -78˚C H
H Me [2+2] Me
O H H
O
32-01-carbocation Rearrang-1 12/5/03 11:13 AM
S. L. Schreiber et al Tetrahedron Lett. 1989, 30, 3765.
M. Shair, D. Evans Carbocation Rearrangements-2 Chem 206

Synthesis of (±)-Isocomene: Pirrung, JACS 1979, 7130; 1981, 82.

Me Me HOAc
Me Me Me Me
Me
OTs
H+
OTs H
Me Me Me non-classical
Me
Me krel = 95 carbonium ion
Me Me
Me
(±)-Isocumene
R. G. Lawton, JACS 1961, 2399
OTs krel = 1
Carbocations: Neighboring Group Participation
■ Groups with accessable electron density (heteroatoms, arenes) and the The Cram Phenonium Ion Experiments: Cram, JACS 1949, 71, 3865
correct stereoelectronic orientation (anti-periplanar) can "assist" in the H Me
ionization of a leaving group. TsO – AcO
OTs H H Ph
L-Threo
H Me
X: X Me Me
R R
X:
R H H Me
Me
R OAc
R R R R R R
R Y- R Ph
Y
Nuc: Nuc H H Me
Me D-Threo
L-Threo
98% chemical fidelity
Ph
Me H
TsO– AcO
OTs H Me Ph L-Erythro
H Me
OTs Me H Me H
O
+3 O HOAc H Me
krel = 10 Me OAc
O Me O
Ph
HMe Me
H L-Erythro
L-Erythro
98% chemical fidelity
Ph

Physical Evidence for Neighboring Group Participation


OTs OAc tetrahedral
O HOAc O Cl carbon in C13 NMR (68.8 ppm)
krel = 1
O Me O Me Phenonium ion
HF-SbF5
SO2 ClF G. Olah JACS 1976, 98, 6784.

See Lowry & Richardson, pp 434-439 for discussion of this controversy


xx-02-carbocation Rearrang-2 12/5/03 9:19 AM
M. Shair, D. Evans Allyl- & Vinylsilanes: The β-Silicon Effect Chem 206

References: Lambert Acc. Chem. Res. 1999, 32, 183-190 Magnitude of the β-Silicon Effect
Lambert, JACS 1990, 112, 8120; 1996, 118, 7867.
SiMe3 Me
Fleming, Organic Reactions 1989, 37, 54.
Fleming, Chem. Rev., 1997, 2063. Solvolysis (CF3CH2OH)
Me3C H Me3C H
H H
k1
Allyl– & Vinylsilanes react with electrophiles H OCOCF3 = 2.4 x 10+12 H OCOCF3
(trapped by solution Nu) k2 2
1
E E
R3Si "R3Si+"
H H
E + Solvolysis (CF3CH2OH)
SiMe3 "R3Si " SiMe3
E Me3C Me
H Me3C H
k3
Mechanism - the simple picture: β-Silicon stabilizes carbocation H OCOCF3 = 4 x 10+4
k4 H OCOCF3
3 4
E Nu
R3Si R3Si E E "These figures established the β-effect as one of the kinetically
strongest in organic chemistry": J. Lambert
Nu
SiMe3 H 2C SiMe3 Data provide no distinction between open and bridged intermediates
E

E E
Fleming, Organic Reactions 1989, 37, 54.
Proof for a stepwise mechanism provided the following protodesilylation
β-Silicon Effect: the origin of regioselectivity experiment:

σocc E
Me3Si SiMe2Ph SiMe2Ph
Si pz
pz σSi–C → pz empty H
σSiC Me3Si SiMe2Ph

H
Me3Si SiMe2Ph Me3Si

H3Si H 3C
CH2 versus CH2 B
A H
H
H
H
both silanes yield the same
product mixture.
Hence, the reaction
Calculation: A more stable than B by 38 kcal/mol. proceeds most
likely via a common
Jorgensen JACS 1985, 107, 1496. intermediate, a
carbeniumion

32-03-beta Si-Effect 12/5/03 8:20 AM


M. Shair, D. Evans Allyl- & Vinylsilanes: The β-Silicon Effect-2 Chem 206

The stereochemical consequences for the major product are:


General: Allylsilanes are more nucleophilic than alkenes
❐ trans-alkene:
HOMO is higher in energy due to negative hyperconjugation ❐ anti-addition of E+ with respect to SiR3
Examples:
E π* Ph
t-BuCl, TiCl4 Ph
t-Bu
π (π
π *) σC–Si Me3Si H
H Me CH2Cl2
Me
Houk, JACS 1982, 104, 7162.
But JACS 1982, 104, 4962.
π
σSi–C π Ph
HF Ph
R R
σSi–C Me3Si H Protodesilylation
E:Z
88:12!
Eschenmoser, Helv. Chim. Acta 1979, 62,
1922.
Electrophile Addition - Stereoelectronics
Carbonyl Addition of Allylsilanes: Open Transition States
Me3Si– is not sufficiently Lewis acidic to activate C=O through pre-association; however
R3 R2
(RO)2MeSi– is Lewis acidic enough to activate C=O through pre-association. These
Si 3 2
allylsilanes add to RCHO througl closed transition states
R R H R
σC–Si R1 X nM R
R 1 σC–Si O
R R1 R2 R1 R2
Si OR
H
Antiperiplanar TS Synclinal TS
A1,3-strain O H
H R
minimized
X nM
SiMe3
E anti addition observed E
SiMe3
Calculations by Houk et al. show that the relative energy differences between the
antiperiplanar and and synclinal transition states are negligible. Both the antiperiplanar
R3 E and synclinal models predict a syn selectivity for the newly formed stereogenic centers.
Si R2
R3 1
R H R R2
σCSi R1 O OH
R E TiCl4
+ Me3Si Me
> 95:5 syn
H Si R H CH2Cl2 R
+ Nu - NuSiMe3 + Nu - NuSiMe3 Me

O OH OH
E
R3 TiCl4
R3 R21 R2
+ Me3Si
R H R H CH2Cl2 R R
R R1
R Me Me Me
H E Hayashi, TL 1983, 2865. syn
minor (cis) ca. 65 : 35
major (trans)
Catalytic Enantioselective Addition of Allylic Organometallic Reagents to
Aldehydes and Ketones, Denmark and Jiping Fu, Chem. Rev. 2003, 103, 2763-2793 (handout)
32-04-beta Si-Effect-2 12/5/03 8:26 AM
B. Breit, D. Evans Allylsilanes: Reactions with Electrophiles Chem 206

Allylsilanes add to aldehydes and acetals under Lewis acid promotion Reactions Proceedilng through Silicon-Migration

OH Si migration may be promoted by using hindered Si substituents


O
Me3Si Ph TiCl4
+ n-C3H7 OTiCl4 OTiCl4
H Me
Ph O
OH TiCl4 Me Me
O
Me3Si TiCl4 Ph Me CH2Cl2
+ n-C3H7
H Me
Ph Si(iPr)3 Si
(Pri)3Si (iPr)3 ??
regioselectivity: Allyl inversion
Felkin Selectivity also holds with this class of nucleophiles
A. I. Meyers, J. Org. Chem. 1998, 63, 5517 Me O

Acetals can be used as well diastereoselection: 97:3


85% yield (Pri)3Si
OCH3
OCH3
Me3Si Me TiCl4
+ n-C4H9
H3CO n-C4H9 (80%)
Me OZrCl4–
Me Me OZrCl4–
MeO2C CO2Me ZrCl4 CO2Me
OCH3 OCH3 MeO CO2Me
Me3Si TiCl4 MeO
+ Me CH2Cl2
H3CO n-C4H9 (83%) n-C4H9 Ar Ar
Me Me Ar SiR3
Me
(Pri)3Si SiR3

The Sakurai Reaction (Enone Conjugate Addition) O Ph(Pri)2Si

OTiCl4 Me MeO2C
O diastereoselection: 96:4 SiR3
TiCl4 Me 68-70% yield MeO2C
Me CH2Cl2 75%
Ar
SiMe3 O Can you work out the mechanism??
Me3Si Me
Me3Si O CO2Me
Me
Fleming, Org. Reactions 1989, 37, 127-133 CO2Me Me BF3°OEt2 Me
17% H PhMe2Si
PhMe2Si rt 8 h
CHO
Me SiMe3 Me
EtAlCl2 Me
78% diastereoselection: >30:1
CH2Cl2
93% yield
O O
Panek, J. Org. Chem. 1993, 58, 2345
Majetich, Tetrahedron 1987, 43, 5621

32-05-allyllsilanes 12/5/03 8:27 AM


B. Breit, D. Evans Vinylsilanes: Reactions with Electrophiles Chem 206

Stereochemistry of Electrophile Addition to Vinylsilanes Summary Statements


1. Me3C+ is more stable than Me3Si+ in spite of the fact that Si is less
Vinyl/Allylsilanes in Organic Synthesis - Selected Examples electronegative than C.
Fleming, Org. Reactions 1989, 37, 54. H
Me H
Me
O Si C
R1 R1 H Si C C–Si bond length: 1.87 Å
Me Me H
H H H
N I R2 R2 RETENTION H
SiMe3 I H H
Me Me
O C C
H C C C–C bond length: 1.54 Å
Me Me H
• Rotation in direction a favored - H
El + Nu H
(avoidance of eclipsing interactions,
- NuSiMe3 C–Si hyperconjugation is less pronounced than the anaologous C–C hyperconjugation
• Principle of least motion do to the impact of the longer C–Si bond lengths.
El El
R1 2. Carbonium ions α to Si are less stabilized than carbonium ions β to Si.
R1 El
El a H
H R2 R2 ≡ R1 R2 Me Me
R1 R2
Me3Si H SiMe3
H C((+) α to silicon Me Si C H H
SiMe3 a Me Si C
SiMe3 H H
Me Me
Et Et
+ TiCl4 Me3Si Me3Si
Et ClCH(OMe)2 Et OMe
SiMe3 - 78 °C C((+) β to silicon C C H C C H
H H
(73%)
OMe H H
H H

H R C–Si hyperconjugation is less pronounced than the anaologous C–C hyperconjugation


TiCl4 R Me Me do to the impact of the longer C–Si bond lengths.
SiMe3 OH
Fleming p 289 H 3. According to Lambert, silicon has a propensity to stabilize β
Me carbonium ion via hyperconjugation (vertical stabilization) rather than
bridging (nonvertical stabilization.
O Me
Me3Si Me3Si
C((+) β to silicon C C H C C H
H H
H H
H H
(CH2O)n Me3Si
NH N
N SiMe3 N
H TsOH H hyperconjugation more C C H
important than bridging H
Fleming p 148 H
Me Me H
4. Silicon has a lower propensity to undergo Wagner–Meerwein like
rearrangements than carbon.
32-06-allyl/vinylsilanes 12/5/03 8:31 AM
M. Shair, D. Evans Stabilized Cations: Iminium-Ions 1 Chem 206

Ph
R1 R3
TFA
Iminium Ions N X-
Me3Si Ph
R2 R4 N
(Z)
Common Methods of Generation: rel rates: 7000/1 N
Ph H H
R3 TFA
R1 R3 H+, -H2O R1 (E)
O H N N N
R4 Overman et al. TL 1984, 25, 5739.
R2 or Lewis Acid R4
R2
Me3Si
OR2
H+, -ROH Ph Ph
N R1 H SiMe3
N R1 or Lewis Acid N H N H
H H
SiMe3 H
Oxidation of Amines Hg(0) X–
Hg X (Z) vinylsilane) (E) vinylsilane)
Me
HgX2 N
N rds N
Me H Me Only in the case of the (Z) vinylsilane is the emerging p orbital coplanar with
H
C-Si bond. Full stabilization of the empty orbital cannot occur with the (E)
H vinylsilane.....hence the rate difference.
X– HX
Stereoelectronic Effects on Nu Addition to Iminium Ions
N Me R
N H O TMS Me TMS
Hg(OAc)2/EDTA PPTS, MeOH
NaBH4 R N
N H 80˚C
Me
H H N H H MeOH
MeO2C OH
N H
R H Me R
H H
N H
MeO2C OH
H N steps
pumiliotoxin A
H one diastereomer
71 %
H CO2Me OH one double bond isomer
H OH
Nu (favored) Stork et al. JACS 1972, 94, 5109. Me
"Least Motion Argument"

C=N Stereoelectronic Effects: Lecture 20 Overman et al. JOC 1989, 54, 2591.

32-07-iminium ions 12/5/03 8:34 AM


D. A. Evans, M. Calter The Aza-Cope Rearrangement Chem 206

Review:
The 3-aza-Cope Rearrangement
Heimgartner, H. In "Iminium Salts in Organic Chemistry";
Bohme, H., Viehe, H., Eds.; Wiley: New York, 1979; Part 2,
pp 655-732. First Neutral Case: Hill TL 1967, 1421.

The 3-aza-Cope Rearrangement:


Me Me
Neutral Variant: Me 250oC, Me
N N "Practically quantitative", no real
2 2 1 hr Me yields given.
3 1 3 Me
R 1 R
N [3,3] N
Exothermic as written by ~7-10kcal/mole.
First Cationic Case: Elkik Compt. Rend. 1968, 267, 623.

Ammonium Variant: Me + Me
Me 80 oC, Me +
R N N
p.
p. R Me No yields given.
[3,3] Even more exothermic than the neutral 2-3 hr Me
R N N Me Me
version, since enamine lacks resonance
R and iminium salt has stronger p-Bond than
H 2O
imine does.
2-aza-Cope Rearrangement: Me
OHC
R R In the simplest case, degenerate. Steric Me
2 2
3 N 1 1
effects, conjugation, or selective trapping of
[3,3] N
a particular isomer, will drive equilibrium.
As with the 3-aza-Cope, the cationic Good way to allylate aldehydes: Opitz Angew. Chem. 1960, 72, 169.
version proceeds under much milder
conditions. R'' H OHC R' -H2O R'' R'
N + N +
1-aza-Cope Rearrangement: X R'''
R'' R R'' R
2 2
1 3 3 The 3-aza-Cope rearrangement can be
R [3,3] R 1
N N driven in reverse by judicious choice of
substrates(i.e., incorporating the imine into H
a strained ring or by making R an acyl R' R' R'
R'' R''
group). O H 2O N N
R ∆ R
R R''
R''
R''' R''' [3,3] R'''

32-08 Aza Cope-1 12/5/03 8:41 AM


D. A. Evans, M. Calter The Aza-Cope Rearrangement Chem 206

The 2-aza-Cope Rearrangement Mechanism for Yohimbane Analog Formation:

First Reported Case: Horowitz JACS 1950, 72, 1518.


NH N
HCHO,
H H H2N
H+, -H2O
Ph NH2 HCHO Ph N Ph N H 2O
2-aza-Cope N N
HCOOH H
100oC, 2hr. + H
PhCHO 2-aza-Cope, driven by
A B conjugation
Equilibrium between A and B driven towards B by conjugation of iminium double bond
to the aromatic ring in B. N
N
Application to Yohimbine Analog Synthesis: Winterfeldt Chem. ber. 1968, 101, 2938.
N
N
H OMe
H
MeOH
..
p.
p. N N H

OH
H H H H N-Acyliminium Ion Rearrangements: Hart JOC 1985, 50, 235.
N N CO2Me
Hart observed an unusual product while trapping the intermediates of N-acyliminium olefi
H H
cyclizations.
Yohimbane Yohimbine
OH
TFA
NH NH N
POCl3 N

O O CF3CO2
O NaBH4 C 3H 7 C 3H 7
N N
H H N
Et3SiH O
NaBH4 C 3H 7

N
N HCHO, MeOH,
Cat. H+, 85% O
NH C 3H 7
H OMe 2-Aza Cope rearrangements add to
N complexity of cyclization process
H N
N N
H
15-Methoxy-isoyohimbane
O O
C 3H 7 40:60 ratio C3H7
32-09 Aza Cope-2 12/5/03 8:43 AM
M. Shair, D. Evans Stabilized Cations: AcylIminium-Ions Chem 206

N-Acyliminium Ion Rearrangements OH


SiMe3
SiMe3 H
Synthesis of (-)-hastanecine: Hart JOC 1985, 50, 235. CH2
N TFA
N 67% N
O Me
AcO O Me
Me Me O Me
BnO NH2 +
The origin of the modest [3,3] ???
O O O diastereoselection has not
SiMe3 H
been attributed to H
81% CH2
2-aza-Cope process.

29% N
N
BnO BnO
OAc O Me
OAc O O Me
OH
OBn HCO2H NaBH4,
N Gelas-Mailhe, Tet. Lett, 1992, 33, 73
Me OAc Me MeOH, Me N
N 83%
Me O Me Me O
O
Competing 2-Aza-Cope and Pinacol Rearrangements:
HCO2H Which Dominates??
[3,3]

Ph Ph
BnO BnO :OH
OAc OAc OH
OBn Ph Ph
Me HCO2H Me CSA, 60oC, cyclization
Me N OAc
HCO2
N N 1.5 hr, 79% Me N
Me Me Me N
Me O H Ph
O O Ph Me Me
O Pinacol
[3,3]

HO BnO N Me
OH OH Ph O
Me Ph
Me Homo-chiral Ph
(-)-hastancine HO Mannich Ph
N N OH
Me
Me N Me N
O
Me Me
racemic product
Conclusion: 2-aza-Cope rearrangements afford a low-barrier to competing processes

32-10-iminium ions 12/5/03 8:46 AM


M. Shair, D. Evans Stabilized Cations: Iminium-Ions 2 Chem 206

2-Aza-Cope-Mannich sequence: Another aza-Cope-Mannich sequence:

OR OR OR
O
Ar
O OH
(CH2O)n, Na2SO4 HO Ar
N N [3,3] N O
CH2O/HCl BF3
H MeCN, 80˚C
HO N N
HO NHBn
HO Bn Bn
NR2 NR2
NR2 O [3,3]

O
Axial Attack O HO
H Ar
H2/Pd-C [3,3]
98 %!! Mannich
Rxn CH2O/HCl Mannich
N N
H Bn 97% Bn
N
ROH2C
O O
N
O OR
O O
NR2 O O H
equivalent to H 67% O
steps NR2 Pictet-Spengler
N cyclization N
H
H
steps

N O

HO O
H
N Overman et al. JACS 1995, 117, 5776. Overman et al. JOC 1991, 56, 5005
H HO N
H Pancracine
O
HO
strychnine

32-11-iminium ions-2 12/3/03 5:15 PM


D. Evans, E. Shaughnessy The Prins-Pinacol Reaction Chem 206

References
Evidence for Prins-Pinacol Mechanism
Prins reaction: Adams, D.R.; Bhaynagar, S. D. Synthesis 1977, 661 Ph
Prins & carbonyl ene reactions: Snider, Comprehensive Organic Synthesis, 1991, Vol. 2 O
Me O Ph
Me SnCl4, CH2Cl2
Me
The Prins Process: O Me
Me
O
Me >95% ee
Me O Me
HX OH X
R2
R1 H R2
R1
Ph Me Ph Me
X– Me Prins
-
Cl4SnO
-
Cl4SnO Me
O O
H Me + Me
O OH Me Me
R2 R2
R1 H R1
[3,3] pinacol enantiopure
R1CHO - H+
R2
O Ph
OH Ph Me
O O
R2 H Aldol (fast)
R2 R1 Me Me
- Me
R1 Cl4SnO O racemic
+ Me O Me
Me
The Prins-Pinacol Variant: Me >95% ee
Ph If a [3,3] rearrangement were intervening, the product would be racemic.
O Overman, JACS 2000, 122, 8672
Lewis Acid
Ph >95% ee Overman, Org Lett 2001, 3, 1225
Me O Me SnCl4
Me Me Examples of Stereoselective THF Formation
O Me Me
Me O Me O
Me
SnCl4 Me O Me SnCl4, CH2Cl2
Pinacol
Ph Me Me -70 → -23 °C Me
Ph Me Me O Me O
H 82%
H Me
– Prins syn
Cl4Sn–O Me – Me
O Cl4Sn–O HO Me BF3•OEt2
O
Me + Me (E)-CH=CHPhCHO O
Me Me
Me
OH CH2Cl2, -55 °C Me
97%
Me O Ph
7:1 anti:syn
32-12-Prins-1 12/3/03 5:15 PM
D. Evans, E. Shaughnessy The Prins-Pinacol Reaction Chem 206

Prins-Pinacol Mechanism Me
Overman: Magellanine Synthesis N

Ph Me Ph Me JACS, 1993, 115, 2992 O


SnCl4 H
(-)-Magellanine
- Me Prins - Me
CH2Cl2 Cl4SnO O Cl4SnO O OH
Me Me Me
Ph Me Me
Me
O The pivotal transformation
Me [3,3] pinacol enantiopure
O
Me O TESO H
Me Ph Me O Ph SnCl4
H OMe
Homo-chrial Aldol CH(OMe)2 57%

Cl4SnO Me Me Me
O
Me
MeMe O Me 1. OsO4, HIO4
2. Ph2CHNH3Cl
>95% ee NaBH3CN
(-)-Magellanine Me
Prins cyclization faster than [3,3] rearrngement CHPh2
N
N
O
H O
2-aza-Cope vs. Pinacol: Ph Ph H
:OH OH Steps
OH
Ph Ph OMe
cyclization Me
CSA, 60oC,

1.5 hr, 79% Me N


Me N
Ph
Ph Me Me
O TESO TESO
Pinacol SnCl4
[3,3]
N CH(OMe)2 Me
Me Ph O O
Me Ph
Homo-chiral Ph
Mannich Ph
OH
Me N Me N O R
H O
Me Me
racemic product OMe H

✻ OMe
[3,3] rearrngement faster than Mannich cyclization ✻ mixture of diastereomers

32-13-Prins-2 12/3/03 5:15 PM


D. Evans, E. Shaughnessy The Prins Reaction-3 Chem 206

Overman Synthesis of a Eunicellin Diterpene Overman: Synthesis of trans-Kumausyne


Overman & MacMillan JACS, 1995, 117, 10391 Me
Me AcO
Me
H
(-)-7-Deacetoxy-alcyoninacetate Me H JACS, 1991, 113, 5378
HO AcO O O Et
Br
trans-Kumausyne
Felkin Control (Lecture 20)
Me BnOCH2CHO OH OH
I Me OH OH RSO3H, rt [3,3]
t-BuLi, THF, PPTS, MeOH
69% O O
-78 °C 64% OH
OH TMS
OBn OBn
Me Me OHC
Me Me
O TMS BF3•OEt2 (3 equiv.)
Me CH2Cl2, -55→ -20 °C
OMe Me 79%
Me
O
OHC OTIPS H O H
m-CPBA O
OBn OBn
Me OH Me OH 72%
O O
[3,3] 4:1 regioselectivity
H H
O TMS O TMS
1. H2, Pd-C, 88%
R R 2. Swern, 100%
Me Me Me Me 1. TMS
Felkin Control (Lecture 20)

Et O
O H BF3•OEt2 H
Me O O
CHO -78 °C → rt
CHO
73%
Et O
single stereoisomer O
O TMS H OTBS 2. TBSCl H
ds = 9:1
Me2HC
Me DIBAL HO
-78 °C H
OTIPS
97%
6 steps, 39% yield from (S)-carvone O O Et
OTBS
32-14-Prins-3 12/3/03 5:16 PM
D. A. Evans The Prins Reaction-4 Chem 206

Mukaiyama Aldol–Prins Cascade Application to Leucasandrolide


Me Me
Rychnovsky JACS, 2001, 123, 8420 OH OH

The Basic Process O OMe O O OMe O


SiMe3
O OH HO
El
O O
El
R O R O Me Me The seco acid
Me Me
5.5:1 ratio
El –TMSX The Pivotal Step: Me
Me
SiMe3 SiMe3 H
SiMe3
Prins O OH O
O O O
BF3•OEt2
El El OBn TBSO
R O R O base, 78%
OBn OTBS

Control of hydroxyl center: see Lecture 20


Let El(+) = Lewis acid activated RCHO
R H BF3•OEt2 R R
R
SiMe3 No (little) control over anti selection:
this (❋) stereocenter BnO O BnO OH O ~5–8:1
O
RCHO TMS
HO
Evans et al., JACS 1996, 116, 4322
BF3•OEt2
R O R O ❋ R Aldehyde Synthesis
base
Chiral enolate alkylation: see Lecture 23 Me
F 3B
O Me Me OH O DIBALH
aldol Me Me OH LDA H+
N Ac2O
H R N Ph O
SiMe3 Ph 77%
SiMe3
O Me BnO O Me
OBn
BF3 I diastereoselection >20:1
O Prins OX OTBS
Me
BnO Me
R O R R O ❋ R H SiMe3 OAc
H
OTBS
O
Myers, JACS 1997, 119, 6496 O CH2 BF3•OEt2
base = OBn
OBn
Me3C N CMe3
cyclic oxo-carbenium ion addition: see Lecture 19
32-15-Prins-4 12/3/03 5:16 PM
Donald J.Cram (1919–2001)

Phenonium Ion Experiments: Cram, JACS 1949, 71, 3865


H Me
TsO– AcO
OTs H H Ph
H Me
Me Me
H H Me
Me OAc
Ph
H H Me
Me
98% chemical fidelity
Cram: "Just remember Dave, old deadwood is better than young deadwood." Ph

"A View from the Far Side. Memorable Characters and Interesting Places." Evans, D. A. Tetrahedron 1999, 55, 8589-8608.
Quotes-7 12/5/03 9:31 AM
D. A. Evans Iminium Ions and Their Transformations Chem 206

CHO OR
http://www.courses.fas.harvard.edu/~chem206/
1
OHC N
1) mix at room O
Chemistry 206 CO2R temp, 5 min
RO
N OR
MeO
Me3Si +
Advanced Organic Chemistry 2 OMe Me
TMS
TFA
Lecture Number 33 OR
N
O reversible? N
Introduction to Carbonium Ions–3 O
OR
■ Stabilized Carbocations: Iminium Ions (C=NR2(+)) MeO N N OR
MeO
■ Stabilized Carbocations: Oxo–Carbenium Ions (C=OR(+)) MeO Me
TMS OMe Me
■ Stabilized Carbocations: Addition & Rearrangements TMS
A
H
Reading Assignment for this Lecture:
N N
Carey & Sundberg, Advanced Organic Chemistry, 4th Ed. CO2R CO2R
Part A Chapter 5, "Nucleophilic Substitution", 263-350 .
Question 13. Final Exam, 1999. During Corey's recent synthesis of Aspidophytine (JACS,
1999, 121, 6771), the pivotal intermediate 3 was assembled by the union of 1 and 2 under MeO N H MeO N H
the specified conditions. Provide a mechanism for this single-pot transformation. MeO Me MeO Me
CHO 1) mix at room 3) excess
temp, 5 min
Ra Rb
NaBH3CN The product-determining step could be step A. NaBH3CN
OHC +N
2) 2 equiv.
TFAA, 0 °C Rc Rd
CO2R N
Me3Si NH2 CO2R
+ N
2
CO2R
MeO N H
1 MeO MeO Me
MeO N N H 3
OMe Me MeO Me

D. A. Evans Monday,
December 8 , 2003
33-00-Cover Page 12/7/03 8:55 PM
D. A. Evans, M. Calter The Aza-Cope Rearrangement Chem 206
Review:
The 3-aza-Cope Rearrangement
Heimgartner, H. In "Iminium Salts in Organic Chemistry";
Bohme, H., Viehe, H., Eds.; Wiley: New York, 1979; Part 2,
pp 655-732. First Neutral Case: Hill TL 1967, 1421.

The 3-aza-Cope Rearrangement:


Me Me
Neutral Variant: Me 250oC, Me
N N "Practically quantitative", no real
2 2 1 hr Me yields given.
3 1 3 Me
R 1 R
N [3,3] N
Exothermic as written by ~7-10kcal/mole.
First Cationic Case: Elkik Compt. Rend. 1968, 267, 623.

Ammonium Variant: Me + Me
Me 80 oC, Me +
R N N
p.
p. R Me No yields given.
[3,3] Even more exothermic than the neutral 2-3 hr Me
R N N Me Me
version, since enamine lacks resonance
R and iminium salt has stronger p-Bond than
H 2O
imine does.
2-aza-Cope Rearrangement: Me
OHC
R R In the simplest case, degenerate. Steric Me
2 2
3 N 1 1
effects, conjugation, or selective trapping of
[3,3] N
a particular isomer, will drive equilibrium.
As with the 3-aza-Cope, the cationic Good way to allylate aldehydes: Opitz Angew. Chem. 1960, 72, 169.
version proceeds under much milder
conditions. R'' H OHC R' -H2O R'' R'
N + N +
1-aza-Cope Rearrangement: X R'''
R'' R R'' R
2 2
1 3 3 The 3-aza-Cope rearrangement can be
R [3,3] R 1
N N driven in reverse by judicious choice of
substrates(i.e., incorporating the imine into H
a strained ring or by making R an acyl R' R' R'
R'' R''
group). O H 2O N N
R ∆ R
R R''
R''
R''' R''' [3,3] R'''

33-01 Aza Cope-1 12/4/01 8:19 PM


M. Shair, D. Evans Stabilized Cations: AcylIminium-Ions Chem 206

N-Acyliminium Ion Rearrangements OH


SiMe3
SiMe3 H
Synthesis of (-)-hastanecine: Hart JOC 1985, 50, 235. CH2
N TFA
N 67% N
O Me
AcO O Me
Me Me O Me
BnO NH2 +
The origin of the modest [3,3] ???
O O O diastereoselection has not
SiMe3 H
been attributed to H
81% CH2
2-aza-Cope process.

29% N
N
BnO BnO
OAc O Me
OAc O Me
OH O
OBn HCO2H NaBH4,
N Gelas-Mailhe, Tet. Lett, 1992, 33, 73
Me OAc Me MeOH,
Me
N 83% N
Me O Me O Me O
Competing 2-Aza-Cope and Pinacol Rearrangements:
Which Dominates??
[3,3]

Ph Ph
BnO BnO :OH
OAc OAc OH
OBn Ph Ph
Me HCO2H Me CSA, 60oC, cyclization
Me N OAc
HCO2
N N 1.5 hr, 79% Me N
Me Me Me N
Me O H Ph
O O Ph Me Me
O Pinacol
[3,3]

HO BnO N Me
OH OH Ph O
Me Ph
Me Homo-chiral Ph
(-)-hastancine HO Mannich Ph
N N OH
Me
Me N Me N
O
Me Me
racemic product
Conclusion: 2-aza-Cope rearrangements afford a low-barrier to competing processes

33-02-iminium ions 12/7/03 8:57 PM


M. Shair, D. Evans Stabilized Cations: Iminium-Ions 2 Chem 206

2-Aza-Cope-Mannich sequence:
Another aza-Cope-Mannich sequence:
OR OR OR

O
Ar
(CH2O)n, Na2SO4 N [3,3]
N N
O OH
H MeCN, 80˚C HO Ar
HO O
HO CH2O/HCl BF3
HO
NR2 N N
NR2 NR2 NHBn
Bn Bn

O [3,3]
Axial Attack
O
98 %!! Mannich O HO
Rxn H Ar
H2/Pd-C [3,3]
CH2O/HCl Mannich
N N N
ROH2C
H Bn 97% Bn
N
O OR O O
NR2 O
equivalent to O
O
O H
steps NR2 H 67% O

Pictet-Spengler
N cyclization N
H
H
steps
N

O
H
N Overman et al. JACS 1995, 117, 5776. HO O
H
O Overman et al. JOC 1991, 56, 5005
HO HO N
H Pancracine
strychnine

33-03-iminium ions-2 12/7/03 8:58 PM


D. Evans, E. Shaughnessy The Prins-Pinacol Reaction Chem 206

References
Evidence for Prins-Pinacol Mechanism
Prins reaction: Adams, D.R.; Bhaynagar, S. D. Synthesis 1977, 661 Ph
Prins & carbonyl ene reactions: Snider, Comprehensive Organic Synthesis, 1991, Vol. 2 O
Me O Ph
Me SnCl4, CH2Cl2
Me
The Prins Process: O Me
Me
O
Me >95% ee
Me O Me
HX OH X
R2
R1 H R2
R1
Ph Me Ph Me
X– Me Prins
-
Cl4SnO
-
Cl4SnO Me
O O
H Me + Me
O OH Me Me
R2 R2
R1 H R1
[3,3] pinacol enantiopure
R1CHO - H+
R2
O Ph
OH Ph Me
O O
R2 H Aldol (fast)
R2 R1 Me Me
- Me
R1 Cl4SnO O racemic
+ Me O Me
Me
The Prins-Pinacol Variant: Me >95% ee
Ph If a [3,3] rearrangement were intervening, the product would be racemic.
O Overman, JACS 2000, 122, 8672
Lewis Acid
Ph >95% ee Overman, Org Lett 2001, 3, 1225
Me O Me SnCl4
Me Me Examples of Stereoselective THF Formation
O Me Me
Me O Me O
Me
SnCl4 Me O Me SnCl4, CH2Cl2
Pinacol
Ph Me Me -70 → -23 °C Me
Ph Me Me O Me O
H 82%
H Me
– Prins syn
Cl4Sn–O Me – Me
O Cl4Sn–O HO Me BF3•OEt2
O
Me + Me (E)-CH=CHPhCHO O
Me Me
Me
OH CH2Cl2, -55 °C Me
97%
Me O Ph
7:1 anti:syn
33-04-Prins-1 12/7/03 9:00 PM
D. Evans, E. Shaughnessy The Prins-Pinacol Reaction Chem 206

Prins-Pinacol Mechanism Me
Overman: Magellanine Synthesis N

Ph Me Ph Me JACS, 1993, 115, 2992 O


SnCl4 H
(-)-Magellanine
- Me Prins - Me
CH2Cl2 Cl4SnO O Cl4SnO O OH
Me Me Me
Ph Me Me
Me
O The pivotal transformation
Me [3,3] pinacol enantiopure
O
Me O TESO H
Me Ph Me O Ph SnCl4
H OMe
Homo-chrial Aldol CH(OMe)2 57%

Cl4SnO Me Me Me
O
Me
MeMe O Me 1. OsO4, HIO4
2. Ph2CHNH3Cl
>95% ee NaBH3CN
(-)-Magellanine Me
Prins cyclization faster than [3,3] rearrngement CHPh2
N
N
O
H O
2-aza-Cope vs. Pinacol: Ph Ph H
:OH OH Steps
OH
Ph Ph OMe
cyclization Me
CSA, 60oC,

1.5 hr, 79% Me N


Me N
Ph
Ph Me Me
O TESO TESO
Pinacol SnCl4
[3,3]
N CH(OMe)2 Me
Me Ph O O
Me Ph
Homo-chiral Ph
Mannich Ph
OH
Me N Me N O R
H O
Me Me
racemic product OMe H

✻ OMe
[3,3] rearrngement faster than Mannich cyclization ✻ mixture of diastereomers

33-05-Prins-2 12/7/03 9:00 PM


D. Evans, E. Shaughnessy The Prins Reaction-3 Chem 206

Overman Synthesis of a Eunicellin Diterpene Overman: Synthesis of trans-Kumausyne


Overman & MacMillan JACS, 1995, 117, 10391 Me
Me AcO
Me
H
(-)-7-Deacetoxy-alcyoninacetate Me H JACS, 1991, 113, 5378
HO AcO O O Et
Br
trans-Kumausyne
Felkin Control (Lecture 20-21)
Me BnOCH2CHO OH OH
I Me OH OH RSO3H, rt [3,3]
t-BuLi, THF, PPTS, MeOH
69% O O
-78 °C 64% OH
OH TMS
OBn OBn
Me Me OHC
Me Me
O TMS BF3•OEt2 (3 equiv.)
Me CH2Cl2, -55→ -20 °C
OMe Me 79%
Me
O
OHC OTIPS H O H
m-CPBA O
OBn OBn
Me OH Me OH 72%
O O
[3,3] 4:1 regioselectivity
H H
O TMS O TMS
1. H2, Pd-C, 88%
R R 2. Swern, 100%
Me Me Me Me 1. TMS
Felkin Control (Lecture 20)

Et O
O H BF3•OEt2 H
Me O O
CHO -78 °C → rt
CHO
73%
Et O
single stereoisomer O
O TMS H OTBS 2. TBSCl H
ds = 9:1
Me2HC
Me DIBAL HO
-78 °C H
OTIPS
97%
6 steps, 39% yield from (S)-carvone O O Et
OTBS
33-06-Prins-3 12/7/03 9:02 PM
D. A. Evans The Prins Reaction-4 Chem 206

Mukaiyama Aldol–Prins Cascade Application to Leucasandrolide


Me Me
Rychnovsky JACS, 2001, 123, 8420
OH OH

The Basic Process O OMe O O OMe O


SiMe3
O OH HO
El
O O
El
R O R O Me Me The seco acid
Me Me
5.5:1 ratio
El –TMSX The Pivotal Step: Me
Me
SiMe3 SiMe3 H
SiMe3
Prins O OH O
O O O
BF3•OEt2
El El OBn TBSO
R O R O base, 78%
OBn OTBS

Control of hydroxyl center: see Lecture 20-21


Let El(+) = Lewis acid activated RCHO
R H BF3•OEt2 R R
R
SiMe3 No (little) control over anti selection:
this (❋) stereocenter BnO O BnO OH O ~5–8:1
O
RCHO TMS
HO
Evans et al., JACS 1996, 116, 4322
BF3•OEt2
R O R O ❋ R Aldehyde Synthesis
base
Chiral enolate alkylation: see Lecture 24 Me
F 3B
O Me Me OH O DIBALH
aldol Me Me OH LDA H+
N Ac2O
H R N Ph O
SiMe3 Ph 77%
SiMe3
O Me BnO O Me
OBn
BF3 I diastereoselection >20:1
O Prins OX OTBS
Me
BnO Me
R O R R O ❋ R H SiMe3 OAc
H
OTBS
O
Myers, JACS 1997, 119, 6496 O CH2 BF3•OEt2
base = see Lecture 24 OBn
OBn
Me3C N CMe3
cyclic oxo-carbenium ion addition: see Lecture 20
33-07-Prins-4 12/8/03 8:49 AM
N. Finney, D. A. Evans Squalene Biosynthesis; Squalene Oxide Cyclization Chem 206

Squalene Biosynthesis Squalene Oxide Cyclase


Me Me Me O P P
farnesyl pyrophosphate Prestwilch, et. al. Chem. Rev. 1993, 93, 2189-2206
(C15) Me
Cornforth Proposal: ACIEE, 1968, 903.

Me Me Me Me Me Me
Me
Me O P P Me
O Me Me Me
Enz-X– squalene oxide

Me Me Me Hb Ha Me Me Me Me H Me
Me Me
Me Me Me
O
Enz-X
O P P
Me
Me Me
Me Me Me Ha Me Me Me

Me Me Me H Me
Me Me
presqualene pyrophosphate O P P Me
HO +
Me H Me Me
Me Me Me H Me Me Me H

Me Me
Me H
Me Me
H
HO
NAD H Me
Me H
Me Me Me H Me Me Me Me
H Me Me
Me Me
Me
Me Me
Me Me Me Me Me
Me H
Me
Me HO
Me Me Me lanosterol
squalene (C30) Me
Me H
33-08- Squalene, Lanosterol 12/7/03 9:03 PM
N. Finney, D. A. Evans The Corey Addendum to the Cyclization Process Chem 206

Squalene Oxide Cyclase - Cornforth Proposal: ACIEE, 1968, 903. Corey–Virgil Revision: JACS 1991, 113, 4025-4026; 8171-8172
Me H Me Me Me
Me Me Me Me
HO Me
+
Me Me
H Me O Me
H Me H
H
Me
Me
Me H
Me Me Me
H X-Enz Me Me
HO H Me
O Me squalene oxide H
Me Me
Me O
Me H Me
Me
H Me Me in vivo in vivo
Me O
Me H stereocenter in question Me
Me Me Me H
H Me H
HO Me
Me Me Me Me
Me
Me H Me Me Me
H X-Enz Me lanosterol
HO HO
Me Me Me Me
Me
Me H
Me Me O
H NOT
Me Me Me X Me
HO Me H Me H
Me Me H
Me
Me H
+
Me HO
H
81 kcal/mol (MM2) Me
anti-migration Me H
H Me
–17 kcal/mol H anti-migration
Me
Me Me Sir John Cornforth:
64 kcal/mol
Me Me
Me H "That outpost of empire Australia, If you are anxious for over-exposure,
produces some curious mammalia, to prepublication disclosure,
HO the kangaroo rat, to good food and good drink,
Me Me the blood-sucking bat, without leisure to think,
H lanosterol
and Aurthur J. Birch, inter alia." try IUPAC symposia."
33-09-Revised SqOC 12/7/03 8:45 PM
N. Finney, D. A. Evans Cationic Cyclizations in the Laboratory Chem 206

Biomimetic Polyene Cyclizations


Me
Me Me Me
H2C Me Me Me
Me R Me R Me
TiCl4 Me

–78 °C Me
Me
O SiPhMe2 MeAlCl2
O O Me
O Me Me –78 °C, 57% Me
OH
Me Me
R = H ca. 30%
Johnson, et al. JACS 1987, 109, 2517. R = CH=CH2 77% Me
SiEt3 PrO2iC HO
Me CH2 Me Me
RO Me
CO2iPr AcO E. J. Corey, et al. Tetrahedron Lett. 1994, 35, 9149.
Me R' Me R'

HO Cyclization to Form Simpler Bicyclics


Me Me

R = OH, R' = H 20%, 24 hr. CH2 CH2


Johnson, et al. JACS 1987, 109, 5852. R = OAc, R' = CH=CH2 80%, 1 min. SiEt3
Me Me
Me
Post cyclization transformation: Johnson TiCl4
+
Me –78 °C H H
Me O Me O Me
O3 Me O O
B Base
B HO HO
A B Me Me
O O
Me Me O Me Me
96 : 4
O
Me Me Me Johnson, et al. JACS 1984, 106, 1138.
Me
O3
C D C D
Base
Introduction of chiral auxiliaries for C=O groups

33-10-Synth Appl 12/8/03 8:37 AM


Andrew Ratz Proposed Biosynthesis of the Daphniphyllum Alkaloids Evans Group Seminar, June 1993

CO2H
Me CO2H
CO2Me O
R
Me O
i
Me O Pr Me
Me i
Pr H
HN i HN Me
Pr
N
N
i
Pr HO
Me pyridoxal OP*
Methyl Homodaphniphyllate HN CHO
HO Me N
OP*
Secodaphniphylline + H+
CO2H
OHC Me Me
OHC
Me Me R
CO2H
i
Pr Me
Me Me N
pyridoxamine
i
Me CHO N Pr Me HO
R
N OP*
Squalene Dialdehyde HO
OP* HO Me N
OP* H
Me N
Me N
CO2Me
R
OHC
R
OHC i
R Pr
formal CO2H
O [4+2] N

Ar NH R
R N Ar Methyl Homodaphniphyllate
i
Ar Pr Me
NH R N
NH HN
R O

R R CH2O i
Pr O
formal Me Me
[4+2] Me N
N Me Me
Me N HN
Me Daphnilactone A

Me Me Pure & Appl. Chem. 1989, 61, 289


J. Am. Chem. Soc. 1986, 108, 8274
33-11-Daphniphyllum Biosysyn 12/8/03 8:35 AM
D. A. Evans Polyene Cyclizations Chem 206

http://www.courses.fas.harvard.edu/~chem206/ Several Comp[ex Mechanisms

OBn
Chemistry 206 OBn

Advanced Organic Chemistry OHC Me


1. MeNH2
Me
OHC HN
Handout 33A Me 2. HOAc, NH4OAc Me
75%
Me Me
Angew. Chem. Int. Ed. Engl. 1978, 17, 476.
Introduction to Carbonium Ions–4 J. Org. Chem. 1990, 57, 2544.

■ Stabilized Carbocations: Oxo–Carbenium Ions (C=OR(+))


■ Introduction to Terpene Biosynthesis O Br
■ Squalene, Lanosterol, & Cholesterol Biosynthesis OMe O
TFA, reflux
■ Polyene Cyclizations
NMe O
■ Chiral Acetals as C=O Auxiliaries TIPSO Bu3SnH N O
O Me
Reading Assignment for this Lecture:
A. P. Johnson et al., JCS Chem. Commun. 1994, 6, 765
Carey & Sundberg, Advanced Organic Chemistry, 4th Ed.
Part A Chapter 5, "Nucleophilic Substitution", 263-350 .
"That outpost of empire Australia,
Heathcock, C. H. (1992). “The enchanting alkaloids of Yuzuriha.” produces some curious mammalia,
Angew. Chem., Int. Ed. Engl. 31: 665. (handout)
the kangaroo rat,
Other Excellent References the blood-sucking bat,
and Aurthur J. Birch, inter alia."
Bartlett, P. A. (1984). "Olefin Cyclisation Processes That Form
Carbon-Carbon Bonds". Asymmetric Synthesis. Stereodifferentiating
Reactions, Part B. J. D. Morrison. New York, AP. vol 3: 341. If you are anxious for over-exposure,
to prepublication disclosure,
Thebtaranonth, C. and Y. Thebtaranonth (1994). "Cyclization Reactions". to good food and good drink,
Boca Raton, FL, CRC Press. Sir John Cornforth:
without leisure to think,
Corey & Liu, enzyme "Mechanism for Polycyclic Triterpene Formation,"
Angew. Chem. Int Ed. 2000, 39, 2812-2833
try IUPAC symposia."

D. A. Evans Monday,
December 8 , 2003
33A-00-Cover Page 12/7/03 9:07 PM
D. A. Evans Terpene Biosynthesis Chem 206

Suggested Reading Representative Terpenes

"An Experimental Demonstration of the Stereochemistry of Enzymic H Me


Cyclization of 2,3-Oxidosqualene...." Me Me Me
E. J. Corey, S. C. Virgil JACS, 1991, 113, 4025. Me Me Me
Me
"New Mechanistic and Stereochemical Insights on the Biosynthesis Me H Me H
of Sterols from 2,3-Oxidosqualene Cyclase"
E. J. Corey, et al. JACS 1991, 113, 8171. H H
Me Me
HO
"Enzymatic Cyclization of Squalene and Oxidosqualene to Sterols HO H
and Terpenoids" Abe, et al. Chem. Rev. 1993, 93, 2189. lanosterol cholesterol
Me
"Isoprenoid Biosynthesis. Stereochemistry of the Cyclization of
Allylic Pyrophosphates." D. E. Cane Acc. Chem. Res. 1985, 18, 220.
OAc
"Biomimetic Polyene Cyclizations" Me O
W. S. Johnson Angew. Chem. Int. Ed. Engl, 1976, 15, 9. Me OH
Me
taxol RO
Me

HO
BzO
Interesting Reading AcO O

Science, 1997, v277: O


Me O
"Structure and Function of the Squalene Cyclase", G. Shultz, p. 1811.

"Structural Basis for Cyclic Terpene Biosynthesis by Tobacco O


5-epi-Aristolochene", J. P. Noel, p. 1815. CH2 Me OH
"Crystal Structure of Pentalene Synthase: Mechanistic Insights on Me picrotoxinin
caryophyllene O
Terpenoid Cyclization Reactions in Biology", D. W. Christianson, p. Me O
H 2C Me
1821.

"The Stereochemistry of Allylic Pyrophosphate Metabolism." O


D. E. Cane Chem. Rev. 1980, 36, 1109.
OC
"Biosynthesis of Natural Products" P. Manitto; Wiley&Sons, NY: 1981. HO OH
Me CO2H

gibberellic acid
33A-01-terpene refs 12/8/00 9:00 AM
D. A. Evans Terpenes: Occurance in Nature Chem 206

■ Definition: Natural products whose carbon skeletons are built up ■ Role in insects: hormones, pheromones (communication chemicals)
largely from isoprene subunits:
O
Me O O
Me Me periplanone
Me sex attractant pheromone of the
Me O O American cockroach
H H H2C Me
OH H
isoprene
Me Me ✻ Me
Me Me Me O
menthol (S)-carvone (R)-carvone
caraway spearmint Cecropia junenile hormone OMe
blocks development at larval growth stage
■ Occurance: Plants, insects, higher organisms note that the starred methyls are O
not derived from isoprene ✻ Me
■ Role in plants: hormones, defense etc. Me

example: pyrethrin insecticides Me alarm pheromone for aphids


H Me
Me
Me O
Me
H
H ■ Role in mammals: hormones, pheromones ??
Me Me O
Me OH O H
H chrysanthemic Me Me
O acid Me
Me cholesterol steroid
CHO Me hormones
Me OH
CHO Me H
example: antifeedants such as warburganal
Me Me
H
Me Me HO H
example: flavor constituents Me

Me Me O
Me
Me nepetalactone Me O
H
Me Me H oil of catnip
nootkatone O
grapefruit flavor active at nanomolar-femptomolar concentrations
citronellal
lemon oil is this related to a feline pheromone?
O
H
Me
33A-02-terpene-1 1/25/00 2:49 PM
D. A. Evans Isoprene : Building Block of Nature (2) Chem 206

Me Classification of terpenes ■ Terpene Biosynthesis


tail monoterpenes : 10 C-atoms (2 isoprene units)
head There are two isoprene units which are used to build up terpenes:
sesquiterpenes : 15 C-atoms (3 isoprene units)
2-methyl-1,3-butadiene diterpenes : 20 C-atoms (4 isoprene units)
isoprene Me enzyme Me
triterpenes : 30 C-atoms (6 isoprene units)
OX Me OX
t isopentenyl pyrophosphate γ,γ-dimethylallyl pyrophosphate
h OH (IPP) (DMAP)
OH
t O
OH O O O

h
R O P O P OH ≡ ROX R O S CH3 ≡ ROTs
geraniol citronellol menthol camphor O O O
two components in rose oil
pyrophosphate: tosylate:
O nature's leaving group chemist's leaving group
O
O
O O O The basic process: alkene addition to electrophiles:
Me Me
O
Me -OX - Me
H 2C Me OX
CH2
H Me OX Me
sex attractant pheromone of the cinnamolide
American cockroach antifungicide DMAP

Me Me Me Me
-HB
Me OX Me OX
H H
geranyl pyrophosphate
B-
ß-carotene H2O/OH-
Me

OH

natural rubber n
geraniol
Practice: Recognize the isoprene units in the above structures.
Me Me
33A-03-terpenes-2 12/17/99 8:00 AM
D. A. Evans Isoprene : Building Block of Nature (3) Chem 206

From isoprene to α− pinene and bornene


Me Me The Basic Process: 1-2 rearrangement
Me OX
OX isomerization
Me OX R3 R3
-OX-
R2 R4 + R2
R4 C C C C
Me Me + CH3 R1
DMAP R1 CH3
geranyl pyrophosphate

Me Me Me

-OX- -H+
OX
CH3
Me Me Me Me Me The precursor of bornene could also lead to camphene.
Show the mechanism!
limonene CH3
camphene

Me
Me Me Me
Me
-H+ Tetrahydrocannabinol (THC) Me
≡ H+
Me OH OH
aromatic
substitution
α−pinene
OH HO CH3 HO C5H11
1,2 shift
chrysanthenol olivetol
Me Me
Me
Me Me Me Me
Me -H+ OH OH

Me -H+

Me Me
bornene Me O Me C5H11
C5H11 O
33A-04-terpenes-3 12/17/99 8:03 AM H
N. Finney, D. A. Evans Cyclic and "Irregular" Monoterpenes Chem 206

Cyclic Monoterpenes Biosynthetic Study of Monoterpenes


Me
Me OH 14
Me Me C label O
Me CO2H HO Me
Me
OH O HO2C
α-terpineol HO CO2H OH
Me HO
H Me (±) MVA
Me (±) MVA lactone
Me Me borneol hydroxy-methyl
glutarate cholesterol-lowering
Me drugs operate here
Me Me Me
H
+ Me
Me Me Me OH
Me
Me O P P
α-pinene
GPP
Me
Me
Me Me Me OH
H
Me
Me Me Me O
O
+H carene
Me Me Me camphor α-thujone

Me Me Iridoids and Secoiridoids


terpinolene Me Me OR CH2 OR

O O
HO
+H Me OHC
Me Me OH
Me CO2CH3 CO2CH3
R = β-D-glucose
loganin secologanin
terpinenol Me
Irregular Monoterpenoids
Me Me Me
H Me Me
Me Me
+
CO2H Me
Me O
H
α-thujene chrysanthemic acid
Me Me Me Me Me Me nezukone
33A-05-Monoterpenes 12/8/00 8:52 AM
N. Finney Sesquiterpenes–C15 Terpenoids Chem 206

Me Me

Me O P P Me (E)-FPP

Me Me (Z)-FPP Me Me O P P

Me Me Me

Me CH3 Me
+
Me Me Me Me Me Me

Me Me Me Me Me
Me

Me Me Me CH3
CH3 Me

Me Me Me Me Me Me Me Me Me Me Me
Me

[Ox]

O O Me Me Me Me
Me
Me
O Me
OH
Me CH2 Me
Me OH Me Me
Me Me Me Me
H 3C Me
O cuparene widdrol thujopsene Me
O Me
H 2C Me
picrotoxinin caryophyllene humulene
33A-06-Sesquiterpenes 12/8/00 8:54 AM
N. Finney Squalene Biosynthesis; Squalene Oxide Cyclization Chem 206

Squalene Biosynthesis Squalene Oxide Cyclase


Me Me Me O P P
farnesyl pyrophosphate Prestwilch, et. al. Chem. Rev. 1993, 93, 2189-2206
(C15) Me
Cornforth Proposal: ACIEE, 1968, 903.

Me Me Me Me Me Me
Me
Me O P P Me
O Me Me Me
Enz-X– squalene oxide

Me Me Me Hb Ha Me Me Me Me H Me
Me Me
Me Me Me
O
Enz-X
O P P
Me
Me Me
Me Me Me Ha Me Me Me

Me Me Me H Me
Me Me
presqualene pyrophosphate O P P Me
HO +
Me H Me Me
Me Me Me H Me Me Me H

Me Me
Me H
Me Me
H
HO
NAD H Me
Me H
Me Me Me H Me Me Me Me
H Me Me
Me Me
Me
Me Me
Me Me Me Me Me
Me H
Me
Me HO
Me Me Me lanosterol
squalene (C30) Me
Me H
33A-07- Squalene, Lanosterol 12/7/03 8:45 PM
N. Finney, D. A. Evans The Corey Addendum to the Cyclization Process Chem 206

Squalene Oxide Cyclase - Cornforth Proposal: ACIEE, 1968, 903. Corey–Virgil Revision: JACS 1991, 113, 4025-4026; 8171-8172
Me H Me Me Me
Me Me Me Me
HO Me
+
Me Me
H Me O Me
H Me H
H
Me
Me
Me H
Me Me Me
H X-Enz Me Me
HO H Me
O Me squalene oxide H
Me Me
Me O
Me H Me
Me
H Me Me in vivo in vivo
Me O
Me H stereocenter in question Me
Me Me Me H
H Me H
HO Me
Me Me Me Me
Me
Me H Me Me Me
H X-Enz Me lanosterol
HO HO
Me Me Me Me
Me H
Me Me
H
Me Me Me
HO
Me O
Me H NOT
Me Me
H Me H Me X H
81 kcal/mol (MM2) Me
Me Me H
anti-migration +
–17 kcal/mol HO
Me
Me Me H Me
64 kcal/mol Me H Me
Me Me H anti-migration
Me H

HO
Me Me
H lanosterol
33A-08-Revised SqOC 12/8/03 8:06 AM
N. Finney Cationic Cyclizations in the Laboratory Chem 206

Biomimetic Polyene Cyclizations


Me
Me Me Me
H2C Me Me Me
Me R Me R Me
TiCl4 Me

–78 °C Me
Me
O SiPhMe2 MeAlCl2
O O Me
O Me Me –78 °C, 57% Me
OH
Me Me
R = H ca. 30%
Johnson, et al. JACS 1987, 109, 2517. R = CH=CH2 77% Me
SiEt3 PrO2iC HO
Me CH2 Me Me
RO Me
CO2iPr AcO E. J. Corey, et al. Tetrahedron Lett. 1994, 35, 9149.
Me R' Me R'

HO Cyclization to Form Simpler Bicyclics


Me Me

R = OH, R' = H 20%, 24 hr. CH2 CH2


Johnson, et al. JACS 1987, 109, 5852. R = OAc, R' = CH=CH2 80%, 1 min. SiEt3
Me Me
Me
Post cyclization transformation: Johnson TiCl4
+
Me –78 °C H H
Me O Me O Me
O3 Me O O
B Base
B HO HO
A B Me Me
O O
Me Me O Me Me
96 : 4
O
Me Me Me Johnson, et al. JACS 1984, 106, 1138.
Me
O3
C D C D
Base
Introduction of chiral auxiliaries for C=O groups

33A-09-Synth Appl 12/8/00 8:48 AM


D. A. Evans Chiral Acetal Auxiliaries-1 Chem 206

Me
Me R H R R
Me Me TiCl4
Me Me O O TMS
O OH O O
SnCl4 CH2Cl2, -78oC
O O
C6H6, R.T. 91% yield
H H Me Me Me Me Me Me
Me Me RO RO
P. A. Bartlett & Co-workers,
R = C8H17 Ratio = 88 : 12
W. S. Johnson & Co-workers, J. Am. Chem. Soc., 1983, 105, 2088
Ratio = 72 : 28 de = 84%
J. Am. Chem. Soc., 1976, 98, 6188.
Br
Ph CH3 Ph CH3 Ph CH3
R H R R HBr Br2
D O O O OH O O
LiAlD4 D CCl4
O O O OH HO O
AlCl3 MeO2C CO2Me MeO2C CO2Me MeO2C CO2Me
Me Me Me Me Me Me G. Castaldi & Co-workers,
Angew. Chem. I. E., 1986, 25, 259. 94 % yield de = 86 %
R ratio
W. J. Richter, J. Org. Chem., 1981, 46, 5119.
Et 01:99 ( CH2)6 ( CH2)6
t-Bu 22:78
Ph 14:86
Zn-Cu, CH2I2
R Me R R
Et2O, ∆
Me Me O O O O
O O LiAlH4
O OH HO O
AlCl3
MOMO OMOM MOMO OMOM
Me Me Me Me Me Me
K. A. Nelson & E. A. Mash,
J. Org. Chem., 1986, 51, 2721. de = >95 %
R ratio
W. J. Richter, J. Org. Chem. ,1981, 46, 5119.
Et 73:24
85:15 R
t-Bu R
Ph 51:46

O H PhCu (1)
O
R H R R CuBr, PBu3
Et Et O O O OH
OTMS SnCl4 (2) LiBr, BF3
O O CH2 C O OH O O Cu Et2O, -78oC
Et CH3CN, -20oC Me Me Me Me

Me Me Me Me Me Me Acetal Alkylating Product


Isomer Agent (R) E/Z % Yield % de
J. M. McNamara, Y. Kishi, R ratio P. Mangeney & Co-workers,
J. Am. Chem. Soc., 1982, 104, 7371. E (1) 100/0 75 95 (R)
Ph 16:1 Tetrahedron Lett., 1987, 28, 2363. E (2) 95/5 70 85 (R)
n-C9H19 03:1 (error in R/S nomenclature in paper) Z (1) 100/0 75 69 (S)

33A-10-chiral acetals-1 12/17/99 8:57 AM


D. A. Evans Chiral Acetal Auxiliaries-2 Chem 206

PhO
C8H17 H C8H17 C8H17
CN CN
TMS TiCl4 H
O O O OH TiCl4 Ar Ar
OH O
TMS C N
CH2Cl2, -78oC O O o HO O O HO
CH2Cl2, 0 C
98% yield
92% yield
Me Me Me
P. A. Bartlett & Co-workers,
J. Am. Chem. Soc., 1983, 105, 2088. Ratio = 87 : 13
W. S. Johnson & Co-workers,
Tetrahedron Lett., 1984, 25, 591. Ratio = 97 : 3, 88% de

C8H17 H C8H17
Me H C6H13 R H
O O TiCl4 RLi, CuBr Me2S
Me TMS O OH + diastereomer O
O O C6H13
CH2Cl2, -78oC Et2O
Me Me 96% yield Me COOH
Me Me O Me
W. S. Johnson & Co-workers,
J. Am. Chem. Soc., 1983, 105, 2904. Ratio = 93 : 7
R Temp.(oC) % Yield % de
S. L. Schreiber & J. Reagan
Tetrahedron Lett., 1986, 27, 2945.
Me 0 58 94
C 6H 5 H C8H17 Ph -30 90 97
C N
O O TiCl4
TMS C N O OH + diastereomer
CH2Cl2, -20oC Me COOH
Me Me C8H17
97% yield Me Me OTBS C8H17
O O 1) TiCl4, CH2Cl2, -780C
W. S. Johnson & Co-workers, HO O
Ratio = 96.5 : 3.5 OtBu
J. Org. Chem., 1983, 48, 2294. 2) CF3COOH, 22oC
Me Me Me
98 % yield Me

c-C6H11 CH3 c-C6H11


J. D. Elliott & Co-workers, Ratio = 98 : 2
Et2O, -20 Co CH3 Tetrahedron Lett., 1985, 26, 2535.
O O Br2AlH O OH + diastereomer
99% yield
Me Me
Me Me
H 3C
H. Yamamoto & Co-workers,
Tetrahedron Lett., 1983, 24, 4581. Ratio = 23 : 1 1) Me3Al
O O 2) Ac20 - Py
OXc
Et Bu Me2NCO CONMe2
Et Bu
O O DIBAL
OH O + diastereomer H. Yamamoto & Co-workers, 97% yield de = 77%
CH2Cl2, 0oC Tetrahedron Lett., 1984, 25, 5911.
Me Me 90 % yield
Me Me
H. Yamamoto & Co-workers,
Tetrahedron Lett., 1986, 27, 983. Ratio = 97 : 3
diastereoselection >99%

33A-11-chiral acetals-2 12/17/99 8:59 AM


Andrew Ratz Daphniphyllum Alkaloids Evans Group Seminar, June 1993

Me Daphniphyllum Alkaloids "Classical" Total Synthesis of Methyl


O Homodaphniphyllate
O
O
Me CO2Me
CO2Me CO2Me
AcO Me
R
i i
Pr Pr *
i i
N N Pr Pr
* N
N N H
*
Daphniphylline Methyl Homodaphniphyllate
*
Me Me

Me
O Me
R BnO O
O
O Me O
Me CO2Me N N
O O
H O O
Me Me
i
Pr i
Pr R
Me Me
HN HN

Secodaphniphylline Methyl Homosecodaphniphyllate


O O
O O
O O BnO O
N N
i
Pr O
Me Me O O
N N

Daphnilactone A Bukittinggine

• Isolated from bark and leaves of Yuzuriha tree

• Used to cure asthma and vermicide in the early 20th century


O
O O O O
• Compounds have been known since 1909. First structure solved in 1965. CO2H +
Me N Me NH2
H
• 38 members in this alkaloid family O O
J. Org. Chem. 1992, 57, 2531-2594
Kyoto Igaku Zasshi 1909, 6, 208 J. Org. Chem. 1992, 57, 2531
Tet. Lett. 1965, 965 J. Am. Chem. Soc. 1975, 97, 6116
Angew. Chem. Int. Ed. Eng. 1992, 31, 665

33B-01 12/11/00 11:53 AM


Andrew Ratz Daphniphyllum Alkaloids Evans Group Seminar, June 1993

Synthesis of Methyl Homodaphniphyllate Construction of the Methyl


Homodaphniphyllate Skeleton
HO2C
OBn
O O
O Et3N O O pTsOH, tol.

ClCO2Et Me N ∆ O OBn O
89% H 83% N
O Me
O O O O
1. Et3OBF4 O
H 2N Me Me N
O 2. Et3N
LDA, THF N
80%
BnO Br O
1 isomer
S
73% Me
O 4:1
1. Me3OBF4
2. NaBH4
BnO 3. HOAc, HCl
87%
BnO O
O LDA, THF
Me
O Me O OBn OBn
N
N X O i
Pr Me O N
O O O
i O O
O Pr 8:1
O
R 74% Me 1. LTMP, PhSeCl Me
O O
N 2. MCPBA N
60%
Lawesson's
H H
Reagent
O O

H2SO4,
BnO Acetone
BnO

BnO O 1. H3O+
2. HO(CH2)2OH
+
OBn
Me O S S OBn
N 3. LDA, THF N N
S O
O S Me
R O Me
O O
R R=iPr 10:1
46% 14% N NaOMe
R=Me 83:17
H MeOH N
70% 2 steps O
J. Am. Chem. Soc. 1986, 108, 5650
O H
J. Org. Chem. 1992, 57, 2531
J. Am. Chem. Soc. 1968, 90, 6177 HO O
Nouv. J. Chem. 1980, 4, 47

33B-02 12/8/00 9:51 AM


Andrew Ratz Daphniphyllum Alkaloids Evans Group Seminar, June 1993

Synthesis of Methyl Homodaphniphyllate cont'd Proposed Biosynthesis of the Daphniphyllum Alkaloids


OBn

O
Me Me Me
O R R
N H30+
LDA, THF OHC Me
H O O OHC
MeCHO Me Me R
O 100% HO H N H N
O
Me Me
O Me Me
O pyridoxamine

OBn Me CHO R N
OBn OBn
Squalene Dialdehyde HO
1. Me2CuLi
Me OP*
i (EtO)2POCl H
Pr 83%
Me N
+ Me N
(EtO)2PO2 N 2. LDA, THF, HMPA O N
(EtO)2POCl O
73%
O2P(OEt)2 O HO
68% 13%
Li/EtNH2 R
64% OHC
R
OH OH OH OHC
R
O
i 120 atm H2 i
Pr H
Pr
Pd(OH)2 i Ar NH R
Pr R N Ar
H + N Ar
N EtOH, 120°C N N NH R
H 81% H H NH

H H 1:1 ratio H
1. Jones R
?? 2. MeOH, H+
Me R R
CO2Me
R Me Me
Me H N Me Me
Me N HN
Tet. Lett. 1969, 2145. i
Pr Me
N
H
N Me Me Pure & Appl. Chem. 1989, 61, 289
H
H J. Am. Chem. Soc. 1986, 108, 8274
H

33B-03 12/11/00 11:27 AM


Andrew Ratz Daphniphyllum Alkaloids Evans Group Seminar, June 1993

Proposed Biosynthesis cont'd


Retrosynthesis of Methyl Homosecodaphniphyllate
CO2H

CHO
3 15 HO
Me i
Pr OP*
Me14 Me + OR OR
Me 10 HN CO2Me
Me
7

Squalene CO2H
i
Pr Me Me Me
HN +N
HN
H
i H
Pr Me
+N

HO
CO2H
OP*

Me N
CO2H OR
H+ i
Pr Me OR
+N

i
Pr Me HO
+N + Me
OP* HN +N
Me H
HO Me N
OP* H CO2Me Me Me
Me N
OR
i
Pr
N OHC
CO2H

Methyl Homodaphniphyllate OHC


Me
i
Pr Me Me Me
O
HN
i
Pr O OR Me
Me + + I
N R”
R’ Me Me
O
Daphnilactone A O
33B-04 12/11/00 11:31 AM
Andrew Ratz Daphniphyllum Alkaloids Evans Group Seminar, June 1993

Synthesis of Methyl Homosecodaphnipyllate The Vollhardt "Ammonia" Incident


I
OBn Me OBn
+ +
MeO2C OBn
N Me Me
O LDA, THF OHC 1. MeNH2
-78°C to 25°C Me
2. HOAc, NH4OAc Me
OHC 75% Me HN
Me
OBn OBn OBn
Me Me
N N N

O O O
MeO2C MeO2C MeO2C
Me Me Me

Me Me Me Me Me Me OBn
87% OBn
4% 8%
1. DIBAL 83%
2. KOH, H2O,
95°C +
3. HCl,H2O 100% CH3 N Me
+
Me H N
OBn R
O Me Me

OHC 1. LAH 96% O


OHC 2. Swern Me
Me
Me Me OBn OBn
Me Me
1. NH3
2. AcOH, NH4OAc
OR 70°C 82-95%
Me + Me
CO2Me +N N
CH3 H
1. H2, Pd/C

Me 2. Jones i
3. MeOH, H+
Pr Me
HN
HN
Angew. Chem. Int. Ed. Engl. 1978, 17, 476.
J. Org Chem. 1989, 54, 1215. J. Org. Chem. 1990, 57, 2544.
Tet. Lett. 1986, 27, 959.
Top. Stereochem. 1990, 20, 87.

33B-05 12/11/00 11:36 AM


Andrew Ratz Daphniphyllum Alkaloids Evans Group Seminar, June 1993

Stepwise vs Concerted Cyclization


One-Pot Pentacyclization Reaction: Formation of
Protodaphniphylline
R’
R’

+
N Stepwise R Me Me Me LDA, THF Me Me
H N+ OtBu
H I CH3CO2tBu Me
Me 83%
R O
Me 1. LDA, Br(CH2)3CH(OMe)2
2. H3O+ 83%

R’ R’
Me Me
R Concerted
Me R Me
+N +N Me Me CO2tBu Me
70%
H H CHO
Me OH tBuO2C
Me
BnO
CO2tBu Me Me Me
R’ R’ 1. MsCl, Et3N Me
OHC 2. DBU
NH3 AcOH OR 3. DIBAL
OHC AcOH 70°C
Me 80%
44% N 25°C N 1. Swern
95%
90% 2. NH3 Me
Me Me OH 3. AcOH, 80°C
R Me Me 15%
Me Me
Me HN Me Me
Me Me
HN
Me Me
OH
R’ R’
1. Swern
R’ 2. MeNH2 Me
Stepwise R Me Me Me 3. HOAc, 80°C
N + N 65% Me
OHC H Me H
R’
OHC
Me
Me Me Other amines:
Me Me R’ HN
Glycine 38%y
(S)-(+)-Alanine 32%y 1-2%ee
Me Me Me (S)-(+)-Valine 13%y 20-25%ee
Concerted N Me (+)-α-Phenylethyl amine no reaction
Me Me
Me HN
79% isolated yield

33B-06 12/11/00 11:43 AM


Andrew Ratz Daphniphyllum Alkaloids Evans Group Seminar, June 1993

Synthesis of (-)-Secodaphniphylline Rationale for Stereoselectivity in Michael Additions


Me
O OBn
1. LDA, THF
O CO2Me
O COX OBn 2. CO2Me N
Me
Me O N O
O Me + i
Pr 3. MeO2C
Me Me
HN Me
i
Pr O I
Me Me Me
HN Me Me

BnO OBn
Me
Me O Me
O Me LAH O
Me 1. Weinreb Me Me
O Me H
Darvon Alc. O H
HO2C O 93% O N
2. Me Li
Me Me O Me
Li OH O
92% ee N O
OMe LiO
KAPA
1,3-diaminoprop. OMe
87%
Me CH2OH Me
HgSO4 O
HOCH2 O + Me O Me
O Me O Me
H2SO4 O
95% Me
Me OBn
1 : 5 OH OMe
Me Me
N N
1. RuCl3, NaIO4
2. CH2N2 O H Me
Li O O
MeO2C
Me CO2Me MeO OLi H Me
1. separate COCl
OBn
MeO2C O Me O 2. KOH EtOH
O Me + Me O Me Me
O Me O Me
3. (COCl)2, CH2Cl2
31%
i
Pr
92% ee Me
HN
90% ee
Me OBn
Me Interference with
O Me
O Me MeO Me Li cluster ?? H
N
O N O
O O
O H Li Me
Me O
Me O MeO2C
NaCN, DMSO Me
J. Org. Chem. 1980, 45, 582 150°C MeO2C BnO
Tet. Lett. 1981, 22, 4171
J. Org. Chem. 1981, 46, 3936 iPr Me Me
Me 43% overall i
Tet Lett. 1967, 8, 215 Pr Tet Lett 1986, 27, 959
HN Me J. Org. Chem. 1990, 55, 132
HN Topics in Stereochemistry 1989, 227

99.6% ee
33B-07 12/11/00 11:47 AM
Andrew Ratz Daphniphyllum Alkaloids Evans Group Seminar, June 1993

Retrosynthesis of Daphnilactone A Synthesis of Daphnilactone A


Br
Br O O
OH EtO
CO2H LDA, homogeranyliodide 1. LAH
O
O 85% 2. Br
EtO2C Me
Br
Me COBr
i
Pr O i
Pr i
Pr
Me Me Me 3. H3O+ Me Me
N HN HN OEt Me Me 95%
CO2Et Zn, THF, HMPA
89%

Br
H
O O O OZnBr

O O O i O
Pr O O
Me
HN Me Me
O N
Me Me Me Me Me Me
Me Me Me Me
LAH
90%

HO O i O
1. Swern Pr
2. NH3 O H2, Pt Me
Br
HO Me HN
Br O O EtO 3. HOAc, ∆ HN 100%
Me
87%
O
EtO2C Me Me
Me Me OEt DIBAL
tol. ∆
CO2Et 87%
Me Me Me Me

OH HO
Org. React. 1975, 22, 423
Ang. Chem. Int. Ed. Eng. 1967, 6, 1
Ang. Chem. Int. Ed. Eng. 1969, 8, 535
J. Org. Chem. 1992, 57, 2585
Angew. Chem. Int. Ed. Engl. 1992, 31, 665 i
i Pr
Pr Me + Me
HN HN

4.5 : 1
33B-08 12/11/00 11:51 AM
Andrew Ratz Daphniphyllum Alkaloids Evans Group Seminar, June 1993

Total Synthesis of Daphnilactone and


Conversion to the Daphniphylline Skeleton

OH

O
1. CrO3, H2SO4
2. CH2O, pH7 i
Pr O
i Me
Pr Me
50% N
HN

CO2H
O-

i O
Pr Me i
Pr Me
HN +N

OH
CO2Me

i
Pr 1. CrO3, H2SO4 i
Me Pr Me
2. MeOH, H+
HN HN
60%

PhNCO

CO2Me
CO2Me

HCO2H
i
i
Pr Pr Me
50% N
N O
HN
Ph

33B-09 12/11/00 11:52 AM


Carfbene–Olefin Cycloaddition: The FMO Analysis

HOMO LUMO

C C C C

H H
C C
R R

Carbene demo 12/10/03 9:12 AM


D. A. Evans Introduction to Carbenes & Carbenoids-1 Chem 206

http://www.courses.fas.harvard.edu/~chem206/ Useful References to the Carbene Literature


Books:
Chemistry 206 Modern Catalytic methods for Organic Synthesis with Diazo Compounds;
M. P. Doyle, Wiley, 1998.

Advanced Organic Chemistry Carbene Chemistry, 2nd ed. Academic Press, Kirmse, W., 1971.

Lecture Number 34

Provide a mechanism for the following transformations.


Introduction to Carbenes & Carbenoids-1
■ Carbene Structure & Electronics
■ Methods for Generating Carbenes EtO2C
■ Simmons-Smith Reaction O
H 3C N CO2Me
■ Carbene-Olefin Insertions N CO2Et Rh2(OAc)4
■ Carbene Rearrangements CH3 O
N2 MeO2C CO2Me CO2Me
Reading Assignment for this Lecture: O
Carey & Sundberg, Advanced Organic Chemistry, 4th Ed. O
Part B Chapter 10, "Reactions Involving Highly Reactive
Electron-Deficient Intermediates", 595–680.

Handout 09A Simmons-Smith Reaction: Enantioselective Variants O


O
Handout 26B Synthetic Applications of α-Diazocarbonyl Compounds O
1. Rh2(OAc)4
N CO2Et EtO2C N
Chiral DirhodiumCarboxamidates: Catalysts for Highly Enantioselective Syntheses of CH3 CH3
Lactones and Lactams, Aldrichchimica Acta. 1996, 29, 3 (handout) MeO2C
2. CO2Me
N2 MeO2C CO2Me
Doyle, Catalytic Methods for Metal Carbene Transformations, Chem. Rev. 1986, 86, O
919-939 (electronic handout)
JACS 1990 2037
McKervey, Organic Synthesis with α-Diazocarbonyl Compounds, Chem. Rev. 1994, 94,
1091-1160 (electronic handout)
Muller, Catalytic Enantioselective Aziridinations & Asymmetric Nitrene Insertions, Chem. 150˚C
Rev. 2003, 103, 2905-2919 (electronic handout) H H
O
(71%) O
Monday, N2
December 10 , 2003
D. A. Evans

34-00-Cover Page 12/10/03 8:12 AM


D. A. Evans Carbenes: An Introduction Chem 206

Suggested Reading: ■ Carbene Configuration: Triplet vs. Singlet

Doyle, Chem Rev. 1988, 86, 919. p


Kodadek, Science, 1992, 256, 1544.

Recent Review Article: S1 σ


Chemistry of Diazocarbonyls: McKervey et al. Chem Rev. 1994, 94, 1091.

Energy
8–10 kcal/mol
singlet
Books:
Modern Catalytic methods for Organic Synthesis with Diazo Compounds; T1 p σ
M. P. Doyle, Wiley, 1998.
triplet
Carbenes and Nitrenes in "Reactive Molecules: The Neutral Reactive
Intermediates in Organic Chemistry", Wentrup, C. W. 1984, Wiley, p. 162.
Due to electron repulsion, there is an energy cost in pairing both electrons in the σ
Rearrangements of Carbenes and Nitrenes in Rearrangements in Ground & orbital. If a small energy difference between the σ and p orbitals exists, the electrons
Excited States, Academic Press, DeMayo ed., Jones, W. M. 1980, p. 95. will remain unpaired (triplet). If a large gap exists between the σ and p orbitals the
electrons will pair in the σ orbital (singlet).
Carbene Chemistry, 2nd ed. Academic Press, Kirmse, W., 1971.
■ the History of the Singlet-Triplet Gap
S–T Splitting
Carbenes: Electronic Structure Year Method Author HCH Angle Grnd State
kcal/mol

1932 Qual. Muliken 90-100° singlet ––


■ Carbene Configuration: Triplet vs. Singlet
1.078 Å 1947 Thermochem Walsh 180° triplet
p p small

1957 Qual. QM Gallup 160° triplet 30


H H
133.8˚ σ σ Harrison
1969 Ab initio 138° triplet >33
H H
1971 Kinetics Hase –– triplet 8–9
Triplet (two unpaired e-) Singlet (all e- paired)
1971 SCF Pople 132° triplet 19
Often has radical-like character Often has electrophilic or
nucleophilic character: A-type 1974 MINDO Dewar 134° triplet 8.7
(Ambiphilic)
empty 1976 Expt Lineberger 138° triplet 19.5
■ Nitrene empty
filled H
R N H 1976 An Initio Schaeffer ––– triplet 19.7
filled R N
filled 1978 Expt Zare ––– triplet 8.1
Singlet (all e- paired) 1982 Expt Haydon ––– triplet 8.5
■ Nitrenium ion (Wentrup)
34-01-carbenes intro 12/10/03 7:39 AM
M. Shair, D. A. Evans Carbenes: Structure and Generation Chem 206

Heteroatom-Substituted Carbenes: Singlets ■ Bamford-Stevens Reaction: See Lecture 28 on Hydrazones


Shapiro Org. Rxns. 1976, 23, 405.
The p orbital of carbenes substituted with p-donor atoms (N, O, halogen) is raised high
enough in energy to make the pairing of the electrons in the σ orbital energetically
favorable. As a result, these carbenes are often in the singlet state. Me Me Me Me Me Me
OR

p donor p H
orbital Me –Ts Me Me
N N N N N
Energy Ts N
Ts
Me Me Me Me

σ –N2

Heteroatom- π-donor
triplet substituted
Me Me
heteroatom H
carbene carbene
R1 R1 R1
Cl H ■ diazo compounds N N N N
hν or heat
C + N2
Examples: C Singlet C Singlet R2 R2 R2
Cl C6 H 5
diazirines
R1 N hν or heat

Methods of Synthesis R2 N Chem. Soc. Rev. 1982, 11, 127.

■ metal-catalyzed decomposition Doyle Chem Rev. 1986, 86, 919 (handout)


■ Alkyl Halides: Cl
Cl Me Me
Cl C H OH C
Cl Cl O
O O
O
R Rh R1 Rh
Rh Rh2(OAc)4 Rh
R –N2
OH C R1 R2
R C H N
N N (ligands omitted for
R N
Cl clarity)
R2 Me
Me
■ ketenes
O carbenoid
O O
R R1 O
heat or hν Rh
C C O R 2C + CO Rh R1 Rh
Rh
R
R2
R2
34-02-carbenes intro 12/10/03 8:49 AM
M. Shair, D. A. Evans Carbenes: Structure and Generation Chem 206

■ "Stable Carbenes"
■ Cyclopropanation The Skell Rule:
"Stable Carbenes–Illusion or reality"?
Regitz, M. Angew. Chem. Int. Ed. Engl. 1991, 30, 674 (handout) R R
1 CH +
2
Cl
N NaH, THF N R R
H
N cat. tBuOK N
Singlet carbenes add to olefins stereospecifically;
(89%)
R R R
Arduengo et al. J. Am. Chem. Soc. 1991, 113, 361; 1992, 114, 5530. 3 CH ISC
2 +
Arduengo et al. J. Am. Chem. Soc. 1994, 116, 6812, Neutron diffraction study:
R H 2C R H 2C R

N N N
R R R
N N N ISC R

Arduengo argues that these resonance structures are not players based on electron R H 2C R H 2C R
distribution from neutron diffraction. R
These are nucleophilic carbenes which display high stability. Triplet carbenes add non-stereospecifically
Skell and Woodworth JACS, 1956, 78, 4496.
For reviews on the subject, see:
Regitz, M. Angew. Chem. Int. Ed. Engl. 1996, 35, 725.
Regitz, M. Angew. Chem. Int. Ed. Engl. 1991, 30, 674.
Synthetic Applications
Me Me
N F ■ Simmons-Smith Cyclopropanation (See Tedrow hanndout 10B)
F
Au X–ray Structure Simmons, H.; Smith, R. J. Am. Chem. Soc., 1958, 80, 5323.
S
F OH OH
F
F CH2I2
>99:1 diastereoselectivity
Zn(Cu)

The intermediate organometallic reagent: I–CH2–Zn–I

Zn(Cu)
H. G. Raubenheimer OH OH
CH2I2
Chem. Comm. 1990, 1722.
150:1 cis : trans 75% yield

Winstein & Sonnenberg, JACS 1961, 91, 3235


34-03-carbenes intro 12/10/03 8:20 AM
M. Shair, D. A. Evans The Simmons-Smith Reaction Chem 206

■ The Furakawa Simmons-Smith Variant ■ Catalytic Asymmetric Cyclopropanation:


For a recent general review of the Simmons-Smith reaction see: Charette & R3 R3
Beauchemin, Organic Reactions, 58, 1-415 (2001) ZnEt2, CH2I2
R2 OH R2 OH
Et2Zn, CH2I2 NHSO2R
R1 R1
Solvent (0.12 eq.)
NHSO2R 66-82 % ee
Et–Zn–Et + I–CH2–I 2 I–CH2–Zn–Et
Kobayashi, et al. Tetrahedron Lett. 1994, 35, 7045.

Furukawa, J.; Kawabata, N.; Nishimura, J. Tetrahedron, 1968, 24, 53 For a Lewis Acid catalyzed process in which the rate of the catalyzed process is
faster than the uncatalyzed, see: Charette, A. B. JACS 1995, 117, 11367.
Furukawa, J.; Kawabata, N.; Fujita, T. Tetrahedron, 1970, 26, 243
■ Applications in Synthesis
Et2Zn, PhCHI2 syn : anti O New SS variant:
Ph 94 : 6 see Shi, Tet. Lett 1998, 39,
ether, rt 69% Me O 8621
MeO NH O
■ Hydroxyl directivity is a powerful atribute of the S–S Rxn
O
Cl
Me OH CH2I2 Me OH O O Me
>99:1 H Et2Zn
diastereoselectivity Me
Me Zn(Cu) CF3COOH
Me Calipeltoside A CH2I2
Me
OH OH O
CH2I2 50:1
no H OH O diastereoselectivity
Me Me Zn(Cu) Me Me diastereoselectivity MeO O O O
Cl
OMe OMe
Me
CH2I2 >99:1 Evans, Burch
diastereoselectivity Cl Org. Lett. 2001, 3, 503
Zn(Cu) Me
R O
For an review of the directed Simmons-Smith, see: HN
Evans, D. A.; Hoveyda, A.; Fu, G. Chem. Rev. 1993, 93, 1307. O
Me
Me FR-900848 N
Me U-106305
O OBn O
O Me H
Me OBn N
HO R= NH R= OH
Me Me
Et2Zn O OBn O O OH
O
Me OBn
CH2I2 HO Charrette, A. B.; J. Am. Chem. Soc. Falck J. Am. Chem. Soc.
1996, 118, 10327. 1996, 118, 6096.
Charette, A. B. JACS 1991, 113, 8166. >50 : 1 diastereoselection
Barrett, JOC, 1996, 61,
34-04-carbenes intro 12/9/03 8:47 PM 3280.
D. A. Evans Simmons-Smith Reaction the Shi Variant Chem 206

34-04a-Shi/SS summary 12/9/03 9:24 PM


M. Shair, D. A. Evans Carbenoids: Cyclopropanation Chem 206

■ Synthetic Applications
■ Buchner Reaction
H
CO2Me
H CO2Me
cat.
O N2 Cu(I) CO2Me
TBSO H
TBSO N2
Me
Me O
O O
Me Me
Rh2(OAc)4
H Et2AlCl O
1. Br2 H
H TBSO (84%)
TBSO 1,3-shift Me H H AcO
2. DBU Me AcO Me N2
O O Me O
O O
OH
Me Me
H H
H O H
Corey & Myers JACS 1985, 107, 5574.
HO O
Me O
CO2Me Me OTBS Me
O H
O
Antheridic Acid
McKervey et al. JCS PTI, 1991, 2565. confertin

O O
CO2Me
O O
N2 CO2Me
Cu Powder
130˚C, Xylenes
■ Wolff Rearrangement
retention
OMe OMe O OMe OMe
THPO THPO
O2N N2 O2N
CO2H
O AgOBz
Me Me
O H 2O
CO2Me OMe OMe
CuLi
2 prostaglandins
Evans et al. J. Org. Chem. 1993, 58, 471. (+) Macbecin
Et2O, -12˚C
THPO Corey and Fuchs JACS 1972, 94, 4014.

34-05-carbenes 12/9/03 8:49 PM


D. A. Evans Carbenes: Enantioselective Cyclopropanation Chem 206

■ Mechanism
Characterization of metal carbenoid intermediates: not much data!
There is no definitive evidence for metal-catalyzed cyclopropanation and the
possibility that metallacyclobutane intermediates are involved cannot be
CO2Et Rh2(OAc)4 cat. ruled out.
CO2Et
Ph N2 Me3C + Me3C +
Ph
R O CO2R O
N Me N2CHCO2R R N Me
For a detailed mechanistic study which provides supporting evidence for the Cu Cu
intermediacy of a Rh carbene, see: Kodakek, Science, 1992, 256, 1544. N Me H N Me
-N2 H
O O –
OTf
SiMe3 Me3C Me3C
SiMe3 –
OTf
Me3C N
CH2 CO2Me Me3C N CO2Me
Cu
Me3C N2 Cu
N CH2 Me3C Reductive
Ph N Ph R Elimination
SiMe3
SiMe3
spectroscopically observed Me3C +
H
"Copper(I) Carbenes: The Synthesis of Active Intermediates inCu-Catalyzed CO2R CO2R O
H N Me
Cyclopropanation" P. Hoffmann et al, Angew. Chem. Int. Ed. 2001, 40, 1288-1290 R Cu
H H N Me
R H O
■ Catalytic Asymmetric Variants: Me Me Me3C –
OTf
Chiral Cu(I) Complexes O O
L* =
N N ■ Catalytic Asymmetric Variants:Chiral Rh(II) Complexes
Me3C CMe3
O
(5 mol %)
CO2BHT Ph N
L* CO2BHT H
Ph nPr O
Ph N2 CuOTf N H nPr
99% ee CO2Me
94:6 trans/cis Rh Rh
H
R O O H
CuOTf R
+ EtO2C N2 N2 CH2Cl2
ent-6b CO2Et
R O O
R a, R = Ph, >99% ee
b, R = Me, >99% ee Doyle et al. Tetrahedron Lett. 1995, 36, 7579. 95% ee

Evans, et al. J. Am. Chem. Soc. 1991, 113, 726.


How do these complexes really work??
34-06-carbenes 12/10/03 8:38 AM
D. A. Evans Carbenes: Enantioselective Cyclopropanation Chem 206

■ Catalytic Asymmetric Variants: Chiral Rh(II) Complexes ■ The Carbene Complex


O Doyle, JACS 1993, 115, 9968
(5 mol %) Molecular mechanics: favored by 3 kcal/mol
Ph N
H
nPr O
N CO2Me H nPr
Rh Rh
H
O O H
N2 CH2Cl2
O O
95% ee
Doyle et al. Tetrahedron Lett. 1995, 36, 7579.
variable ligand

O CO2Me
H 27A
N H
N Rh O H O CO2Me
H
MeO2C O Rh N N H
N CO2Me N Rh O H
H O O Rh N
CO2Me MeO2C
N C H CO2Me
H O
CO2Me CO2Me

Note N–O trans influence

styrene
N
2 4
O Rh N
1 3 Numbers designate increasing steric H CO2Me
O hindrance in each quadrant
Ph H
48%ee
Doyle, JACS 1993, 115, 9968 Ph CO2Me 27B

H 86%ee H
34-06a-carbenes-Doyle 12/10/03 8:43 AM
M. Shair, D. A. Evans Carbenes: Rearrangements Chem 206

■ Carbene-Carbene Rearrangements ■ Other Rearrangements

H 150˚C
H H
O
(71%) O
N2

H H
O O

Wu, Tetrahedron Lett. 1973, 3903.

H Schecter, J. Am. Chem. Soc. 1971, 93, 5940.

H
N2
O
200˚C
O O
(92%)
O
■ Skattebol Rearrangement O O

Sammes, Chem. Comm. 1975, 328.

C13
■ Vinylidenes
Br
C BuLi C Corey-Fuchs: Danishefsky et al.
Br J. Am. Chem. Soc. 1996, 118, 9509.
TIPS
TIPS

[1,2] C13
Teoc O Ph
N Teoc O Ph
N
Ph 2 eq. BuLi
Tetrahedron Lett. 1973, 2283. O Ph
O
Br -78˚C
(81%)
OTBS Br
OTBS

34-07-carbenes 12/9/03 8:52 PM


M. Shair, D. A. Evans Carbenes: Rearrangements Chem 206

■ Carbene Rearrangements
O O
O OLi O SO2Ar
BuLi N NaSO2Ar N
H P(OEt)2 P(OEt)2 OTf
R R
–78 °C I+Ph
N2 C N N
CH2Cl2, 20˚C I+Ph
N2
RCHO
H
–N2
O O
R
R C C H C -PhI SO2Ar N
N
H
vinylidene
(63%)
carbene
O O OMOM
P(OEt)2 Stang et al. J. Am. Chem. Soc. 1994, 116, 93.
Me + MeO2C CHO
N2
OMOM
K2CO3 carbene intermediates are accessible at high temperatures, more later!
MeO2C
MeOH
(83%)
25˚C
Bestmann, et al. Synlett 1996, 521.
MeH O MeH O MeH H
Me Me Me
Me Me 620˚C
O Me O O
KO-t-Bu Me H H
O H P(OEt)2 O Me Me Me
Me
H –78 °C capnellene
N2 CH insertion
O 68%
Me
MeH O
Gilbert, JOC 1983, 48, 5251 MeH O Me
Me H
620˚C

H H
H
Me
Me

34-08-carbenes 12/10/03 8:45 AM


M. Shair, D. A. Evans Carbenes: Rearrangements Chem 206

■ C–H Insertions continued... ■ N–H Insertions are also possible...

N2 O
Rh2(OAc)4 HO N2
O H H HO
H H Rh2(OAc)4 H H
CO2Me Me CO2PNB Me
CO2Me
Me (83%) N–H O O
Me N
O
O
Electrophilic carbenes are very sensitive to electronic effects CO2PNB

Stork Tetrahedron Lett. 1988, 29, 2283.


HO
H H
Me
S NH3 HO
CO2Me N H H
CO2Me O
CO2 Me
N2 CuOTf H O
thienamycin N
O O
N N CO2PNB
O Salzmann, JACS, 1980, 102, 6163.
(75%) O
O
Insertions (X-H): Stereochemical outcome
Sulikowski, J. Org. Chem. 1995, 60, 2326. R1
R1 R1
H
R2 δ+ δ + R2 δ+ δ+ R2
Enantioselective C-H Insertion H
H A
O R3 A R3 B
R3 A
(5 mol %) B
Ph N B
Retention
H
O N CO2Me H
O Rh Rh O O
O
N2 MeO2C Rh2(OAc)4
O O H CO2Me
CH2Cl2 H
N2
99%, 97% ee Ph Ph
Doyle, JACS 1994, 116, 4507 Me Me
Chiral DirhodiumCarboxamidates: Catalysts for Highly Enantioselective Syntheses of
Lactones and Lactams, Aldrichchimica Acta. 1996, 29, 3 (handout) Taber JACS, 1996, 107, 196.

34-09-carbenes 12/9/03 8:54 PM


M. Shair, D. A. Evans Carbenes: Rearrangements Chem 206

Ring Opening Problem 332: Wolff Rearrangement Stoltz ACS 2003, 125, 13624

H H O
O O O MeO MeO O
hν R C
R N2 AgOBz, Et3N
N2 EtOH 95% yield
H H H THF, 45 ° Me
H O HOEt Me

OH AgOBz, Et3N –N2


H O
R CO2Et Tetrahedron Lett. 1990, 31, 6589.
O C
Wolff Rearr H Me
(88% H
H H
Me
H
H H
Ring Contraction carbene intermediate

N2 H O
H H
S hν S
N
O
N iPr2NH N
O
O CO2Me CO2Me
(81%) Vinylolgous Wolff Rearrangement Doyle pp520-521
Moore et al. J. Org. Chem. 1983, 48, 3365.
Me Me Me
Me
Cu(I)
Wolff-[2+2] N2
N2 O MeOH CO2Me
Me H C O
Me O δ+ Me Me

Me
Me Me Me Me Me
Me O
O
(74%) C
J. Org. Chem. 1980, 45, 2708.
Me Me
O H
Me

34-10-carbenes 12/9/03 9:16 PM


D. A. Evans Introduction to Carbenes & Carbenoids-2 Chem 206

http://www.courses.fas.harvard.edu/~chem206/ Useful References to the Carbene Literature

Chemistry 206 Recent Review Article:


Chemistry of Diazocarbonyls: McKervey et al. Chem Rev. 1994, 94, 1091.

Advanced Organic Chemistry Books:


Modern Catalytic methods for Organic Synthesis with Diazo Compounds;
M. P. Doyle, Wiley, 1998.
Lecture Number 35
Carbenes and Nitrenes in "Reactive Molecules: The Neutral Reactive
Intermediates in Organic Chemistry", Wentrup, C. W. 1984, Wiley, p. 162.
Introduction to Carbenes & Carbenoids-2 Rearrangements of Carbenes and Nitrenes in Rearrangements in Ground &
Excited States, Academic Press, DeMayo ed., Jones, W. M. 1980, p. 95.
■ Thermally Induced Carbene Rearrangements
Carbene Chemistry, 2nd ed. Academic Press, Kirmse, W., 1971.
■ Carbonyl Ylides and their Reactions

R O R O R R O R
C ••
R R R R R R R
■ Oxonium & Sulfonium Ylides and their Reactions The Automerization of Naphthalene (The Cume Question from Hell!)
R R R R
C •• X C X
R R ∆
R R Rationalize

Reading Assignment for this Lecture:


α–13C-labeled C10H8 is isomerized into β–13C-labeled C10H8 at 1035 °C
Carey & Sundberg, Advanced Organic Chemistry, 4th Ed.
Part B Chapter 10, "Reactiona Involving Highly Reactive L. T. Scott, JACS 1991, 113, 9692.
Electron-Deficient Intermediates", 263-350 .

Handout 10B Simmons-Smith Reaction: Enantioselective Variants


Handout 27B Synthetic Applications of α-Diazocarbonyl Compounds Provide a Mechanism for this Transformation
Handout 35A The Use of Fischer Carbenes in Organic Synthesis

1000° C
Corannulene 10%
≤ 10-4 Torr
Friday,
D. A. Evans December 12 , 2003
Scott, L.T., et. al., JACS 113 7082 (1991)
35-00-Cover Page 12/12/03 8:16 AM
D. A. Evans Carbenes: Enantioselective Cyclopropanation Chem 206

■ Catalytic Asymmetric Variants: Chiral Rh(II) Complexes ■ The Carbene Complex


O Doyle, JACS 1993, 115, 9968
(5 mol %) Molecular mechanics: favored by 3 kcal/mol
Ph N
H
nPr O
N CO2Me H nPr
Rh Rh
H
O O H
N2 CH2Cl2
O O
95% ee
Doyle et al. Tetrahedron Lett. 1995, 36, 7579.
variable ligand

O CO2Me
H 27A
N H
N Rh O H O CO2Me
H
MeO2C O Rh N N H
N CO2Me N Rh O H
H O O Rh N
CO2Me MeO2C
N C H CO2Me
H O
CO2Me CO2Me

Note N–O trans influence

styrene
N
2 4
O Rh N
1 3 Numbers designate increasing steric H CO2Me
O hindrance in each quadrant
Ph H
48%ee
Doyle, JACS 1993, 115, 9968 Ph CO2Me 27B

H 86%ee H
34-06a-carbenes-Doyle 12/10/03 8:43 AM
M. Shair, D. A. Evans Carbenes: Rearrangements Chem 206

■ Carbene-Carbene Rearrangements ■ Other Rearrangements

H 150˚C
H H
O
(71%) O
N2

H H
O O

Wu, Tetrahedron Lett. 1973, 3903.

H Schecter, J. Am. Chem. Soc. 1971, 93, 5940.

H
N2
O
200˚C
O O
(92%)
O
■ Skattebol Rearrangement O O

Sammes, Chem. Comm. 1975, 328.

C13
■ Vinylidenes
Br
C BuLi C Corey-Fuchs: Danishefsky et al.
Br J. Am. Chem. Soc. 1996, 118, 9509.
TIPS
TIPS

[1,2] C13
Teoc O Ph
N Teoc O Ph
N
Ph 2 eq. BuLi
Tetrahedron Lett. 1973, 2283. O Ph
O
Br -78˚C
(81%)
OTBS Br
OTBS

34-07-carbenes 12/9/03 8:52 PM


M. Shair, D. A. Evans Carbenes: Rearrangements Chem 206

■ Carbene Rearrangements
O O
O OLi O SO2Ar
BuLi N NaSO2Ar N
H P(OEt)2 P(OEt)2 OTf
R R
–78 °C I+Ph
N2 C N N
CH2Cl2, 20˚C I+Ph
N2
RCHO
H
–N2
O O
R
R C C H C -PhI SO2Ar N
N
H
vinylidene
(63%)
carbene
O O OMOM
P(OEt)2 Stang et al. J. Am. Chem. Soc. 1994, 116, 93.
Me + MeO2C CHO
N2
OMOM
K2CO3 carbene intermediates are accessible at high temperatures, more later!
MeO2C
MeOH
(83%)
25˚C
Bestmann, et al. Synlett 1996, 521.
MeH O MeH O MeH H
Me Me Me
Me Me 620˚C
O Me O O
KO-t-Bu Me H H
O H P(OEt)2 O Me Me Me
Me
H –78 °C capnellene
N2 CH insertion
O 68%
Me
MeH O
Gilbert, JOC 1983, 48, 5251 MeH O Me
Me H
620˚C

H H
H
Me
Me

34-08-carbenes 12/10/03 8:45 AM


M. Shair, D. A. Evans Carbenes: Rearrangements Chem 206

■ C–H Insertions continued... ■ N–H Insertions are also possible...

N2 O
Rh2(OAc)4 HO N2
O H H HO
H H Rh2(OAc)4 H H
CO2Me Me CO2PNB Me
CO2Me
Me (83%) N–H O O
Me N
O
O
Electrophilic carbenes are very sensitive to electronic effects CO2PNB

Stork Tetrahedron Lett. 1988, 29, 2283.


HO
H H
Me
S NH3 HO
CO2Me N H H
CO2Me O
CO2 Me
N2 CuOTf H O
thienamycin N
O O
N N CO2PNB
O Salzmann, JACS, 1980, 102, 6163.
(75%) O
O
Insertions (X-H): Stereochemical outcome
Sulikowski, J. Org. Chem. 1995, 60, 2326. R1
R1 R1
H
R2 δ+ δ + R2 δ+ δ+ R2
Enantioselective C-H Insertion H
H A
O R3 A R3 B
R3 A
(5 mol %) B
Ph N B
Retention
H
O N CO2Me H
O Rh Rh O O
O
N2 MeO2C Rh2(OAc)4
O O H CO2Me
CH2Cl2 H
N2
99%, 97% ee Ph Ph
Doyle, JACS 1994, 116, 4507 Me Me
Chiral DirhodiumCarboxamidates: Catalysts for Highly Enantioselective Syntheses of
Lactones and Lactams, Aldrichchimica Acta. 1996, 29, 3 (handout) Taber JACS, 1996, 107, 196.

34-09-carbenes 12/9/03 8:54 PM


M. Shair, D. A. Evans Carbenes: Rearrangements Chem 206

Ring Opening Problem 332: Wolff Rearrangement Stoltz ACS 2003, 125, 13624

H H O
O O O MeO MeO O
hν R C
R N2 AgOBz, Et3N
N2 EtOH 95% yield
H H H THF, 45 ° Me
H O HOEt Me

OH AgOBz, Et3N –N2


H O
R CO2Et Tetrahedron Lett. 1990, 31, 6589.
O C
Wolff Rearr H Me
(88% H
H H
Me
H
H H
Ring Contraction carbene intermediate

N2 H O
H H
S hν S
N
O
N iPr2NH N
O
O CO2Me CO2Me
(81%) Vinylolgous Wolff Rearrangement Doyle pp520-521
Moore et al. J. Org. Chem. 1983, 48, 3365.
Me Me Me
Me
Cu(I)
Wolff-[2+2] N2
N2 O MeOH CO2Me
Me H C O
Me O δ+ Me Me

Me
Me Me Me Me Me
Me O
O
(74%) C
J. Org. Chem. 1980, 45, 2708.
Me Me
O H
Me

34-10-carbenes 12/9/03 9:16 PM


D. A. Evans, D. Guterman Thermal Generation of Carbene Intermediates Chem 206

Carbenes are Accessible via Sigmatropic Rearrangement The Automerization of Naphthalene (The Cume Question from Hell!)

■ [1,2] Shifts: Alpha-Alkynone Cyclizations ∆


Rationalize
O O O
∆ H C-H α–13C-labeled C10H8 is isomerized into β–13C-labeled C10H8 at 1035 °C
H [1,2]
insertion
H H ••
H ■ Mechanism-1: L. T. Scott, JACS 1977, 99, 4506;
Conditions: 620° C, 12-16 Torr, Quartz filled Quartz Tube
π2s + π2a
σ2s + σ2a
S.M. 3° 2° 1° Recovery
O O O O
_ 80%

60 22 18 σ2s + σ2a π2s + π2a


O O O
_ 78%
87 13
■ For the azulene–naphthalene Isomerization: ∆G° = –30.7 kcal/mol (298K)
O O O ■ The Activation energy for the isomerization: ∆G± = +86 kcal/mol
_ 89%
54 46 ■ Mechanism-2,3: L. T. Scott, JACS 1991, 113, 9692.

O O •• H
H H
_ 8 90% H X H X H
92 Option–2
O

Karpf, M., Dreiding, A., Helv. Chim. Acta. 65 13 (1982) 900 °C


H BF B + A
H 21% + 79%
620° C BF H
Me 14 Torr Me Me H H H
H H •• H
N2 , 1 Hr
O •• H
O
Me Clovene
80% (+ 19% isomers)
Me Option–3
Karpf, M., Dreiding, A.S., Helv. Chim. Acta. 67 1963 (1984) B ∆(HB–HA) = –3.4 kcal/mol (MNDO) A

35-01-carbenes/heat 12/11/03 9:03 PM


D. A. Evans, D. Guterman Thermal Generation of Carbene Intermediates Chem 206

Provide a Mechanism for this Transformation Carbenes: Reaction with Heteroatoms

Suggested Reading
1000° C
Corannulene 10%
≤ 10-4 Torr
Houk and Wu J. Org. Chem. 1991, 56, 5657.

Padwa and Hornbuckle Chem. Rev. 1991, 91, 263.


Scott, L.T., et. al., JACS 113 7082 (1991)
Review Articles

Padwa and Krumpe Tetrahedron 1992, 48, 5385.


Hoffman, R. W. Angew. Chem. Int. Ed. Engl. 1979, 18, 563.
0000000
00
00
00
00
0000000
00
00
00 00000000
00
00 00 00

McKervey et al. Chem. Rev. 1994, 94, 1091.


000

00000 00

Ylide Formation by the Interaction of Carbeneoids


0000
00

00 00
00
00
00
00
00

0000

with Carbonyl Lone Pairs


00

000
000

R O
00

00
00

R O R R O R
0
0

000

••
00
00

Internal Ring: 6 e–
00
00

R R R R R R R
External Ring: 14 e–

H H Generally, the carbene precursor of choice is a diazoalkane or, more frequently,


H H an α-diazocarbonyl reagent. These can be decomposed via thermolysis or
1000° C •• C C ••
photolysis. However, the most common method involves catalytic amounts of
≤ 10-4 Torr transition metals, such as copper or rhodium.

Dipolar Cycloaddition (See Lecture 18)

H H
C–H Insertion R O R
X Y R O R
R R
R R
X Y

35-02-carbenes/heat-2 12/11/03 9:06 PM


D. A. Evans Thermal Generation of Carbene Intermediates Chem 206

Web Problem 88. Please provide a mechanism for the following high temperature reaction that was Web Problem 172. Scott has recently reported the transformation illustrated below (Tetrahedron Lett.
reported by Yranzo and co-workers (J. Org. Chem. 1998, 63, 8188). 1997, 38, 1877) which is implemented by flash vacuum pyrolysis (FVP) at the indicated temperature.

Ph H Ph Cl CH2
N
N flash pyrolysis
+ N2
820 °C FVP, 900 °C
Ph –2 HCl
Draw a plausible mechanism for the transformation to the major product isomer. Do not invoke any
radical intermediates in your answer.
Cl CH2
H H
Ph N 1,5-H shift Ph Ph thermal Provide a concise mechanism for this reaction in the space below.
N
isomerization
N N
H
Ph Ph N2 Web Problem 198. Provide a mechanism for the thermal conversion of triquinacene to azulene (JACS,
Ph
Ph 1973, 2724).

C-H insertion
700° C

Ph Ph Ph
H
1,5-H shift 1,5-H shift
H

Web Problem 135. Anderson has reported the transformation illustrated below (Aust J. Chem. 1990,
43, 1137) which is implemented by flash vacuum pyrolysis (FVP). As indicated, this reaction proceeds
through intermediate A.
Web Problem 282. Hoffmann has reported the mechanistically interesting thermally induced
O
transformation illustrated below (Chem. Ber. 1985, 634.).
O
FVP, 900 °C
O 95% yield
–CO2 H
–CO NC N
Ph 400 °C

–CO2 NC N
–CO H
A
Provide a plausible mechanism for this reaction.
Provide a mechanism for this reaction and identify intermediate A in your answer.

35-03-Carbenes/heat-3 12/12/03 8:37 AM


D. A. Evans, D. Barnes Carbonyl Ylids: Dipolar Cycloaddition Chem 206

Tandem Intramolecular Cyclization–Intermolecular Cycloaddition


Reactions of Diazoimides: [3+2] addition

R R R H Me
O Rh2(OAc)4 N C CO2Et O O O
O N
CHN2 O Me Rh2(OAc)4
N Me COMe
CO2Et O
O Bn N2 PhCH3, 110 °C N
O O O
Bn O
NPh RCHO 74%
–N2
O DMAD Me
R O COMe
R H O O
O N
R
R Bn O
O NPh CO2CH3
O H
O H CH3
O H O Me O
CO2CH3 O H OH
Et3SiH / BF3•Et2O
O Me
Me 68%
O CH2Cl2
Dipolar-Dipolarophile Cycloadditions: HOMO–LUMO N N O
Energies Bn O H
Bn
Carbonyl Ylides have very small HOMO-LUMO gaps Maier, M. E.; Evertz, K. Tetrahedron Lett. 1988, 29, 1677-1680.

O
O O O H
Rh2(OAc)4 Me
Energy O
N Me
PhH, reflux O
N2 N
88% H

ylide dipolarophile O
Me O
Therefore, either raising the dipolarophile HOMO (electron-donating H
substituents) or lowering the LUMO (electron-withdrawing) will accelerate O O O O N
the reaction. N
LUMO N Me
O N H
N2
H

O HOMO "high yield"

Padwa et. al. Tetrahedron Lett. 1992, 33, 4731-4734.


35-04-carbonyl ylids-2 12/11/03 9:06 PM
D. A. Evans, K. Beaver Carbonyl Ylids: Dipolar Cycloaddition–2 Chem 206

Dipolar Cycloadditions: Carbonyl Ylides Reactions of Diazoimides: [3+2] addition – [4+2] retroaddition

Me Me
OAc Me R1
Me Me AcO O O
H AcO Me Rh2(OAc)4
H Z
H H (86%) N
O H H PhCH3, 110 °C Z
N2 H H 2
R1 R2 R Me N2 R2 O
Rh2(OAc)4 R2
O
CO2Et O
H – Me–N=C=O
O CO2Et
H H CO2Et R1
O O R1
Dauben, JOC 1993 7635 Tigilane Skeleton
+O – Z
2
R Z
R2 N R2 O
Me O R2 N
Me O
Me O OM O R1 = H, OBn
N O MeO2C MeO2C Me R2 = H, Me Yields = 44-63%
N
N Me O Z = COMe, CO2Me
O N2 OMe O (93%)
Rh2(pfb)4 H H Maier, M. E.; Schöffling, B. Chem. Ber. 1989, 122, 1081-1087.
N
Bz
N
N
Padwa, JOC 1995 2704 Bz O
Bz O O O
Rh2(OAc)4, MeOH O
Lysergic Acid Skeleton COCH3
N Me MeO N
PhH, 80 °C H
N2
MeO2C CO2Me
85%
MeO2C CO2Me
O O O O
N CO2Me
Me N N CO2Me
Et N O O CO2Me
2 O OH
N O Et O Me O Me MeO2C
Rh2(OAc)4 MeO2C Me
N O MeOH O
(95%) N O N O
O Et Me CO2Me N CO2Me
Padwa, JOC 1995 6258 N O Vindoline Skeleton
Me CO2Me
Padwa, A.; Hertzog, D. L.; Chinn, R. L. Tetrahedron Lett. 1989, 30, 4077-4080.

35-05-carbonyl ylids-2 12/11/03 9:07 PM


D. A. Evans, D. Barnes Carbonyl Ylids: Dipolar Cycloaddition (and More!)–3 Chem 206

CO2Et
The Carbonyl Ylide - Azomethine "Dipole Cascade" The Synthesis of Furans
O O
O
CHOCH3
Me O CuSO4 Me
O Rh2(OAc)4 O
N N Me Me
H EtO CHN2 160 °C
CHN2 HC C CO2CH3
Me O Me CO2Me
Me CO2CH3 Me CO2CH3
~30%
H O
O not observed N O
N
Me O hydrolysis
Me O Me decarboxylation
MeO2C Me
rearrange HC C CO2CH3 Methyl vinhaticoate
O
O
Me CO2CH3
O N OH Spencer, T. A., et. al. JACS 1967, 89, 5497.
N N O

Me O H Me Me CO2Me Can you propose a rational mechanism for this transformation?


CO2Me
O
O O
The 1,3-proton shift is catalyzed by trace amounts of water. Azomethine ylide O O
formation requires a proton at the tertiary center. O
Padwa, A.; Dean, D. C.; Zhi, L. J. Am. Chem. Soc. 1989, 111, 6451-6452. O Cu(acac)2
Padwa, A.; Dean, D. C.; Zhi, L. J. Am. Chem. Soc. 1992, 114, 593-601. N2 89%
CH3O
CH3O
CO2CH3 HO CO2CH3
The Synthesis of Furans
Intermolecular addition to α,β-unsaturated carbonyls
OMe OMe
O CuSO4 O O
Et O O
O CHN2 O O
160 °C
O EtO2C O
O O
2-methoxymethylenecholestanone-3 CH3O CH3O
CO2CH3 CO2CH3
EtO2C
O Hildebrandt, Tetrahedron Lett. 1988, 29, 2045-2046.
Spencer Tetrahedron Lett. 1967, 1865-1867.
29%
35-06-carbonyl ylids-3 12/11/03 9:07 PM
D. A. Evans, D. Barnes Carbene Heteroatom Transformations: Ylide Formation Chem 206

Ylid Formation versus Cyclopropanation Tandem O-H Insertion/Claisen Rearrangement


O O O O O Me
catalyst MLn
EtO2C O O OH
EtO CHN2 EtO
Rh2(OAc)4
+ HO O
Me OMe Me
1 PhH, ∆, 20 min
N2 Me OCH3
(66%)
98% ee O
95% ee
H+
EtO EtO O
O O OEt X = H, Rh(II) ??
O O
2 O X
O
O X
catalyst Temp (°C) %1 %2 Me O [3,3]
Me Me O
Pd2Cl2(C3H5)2 RT 53 3 fast Me
CO2Me
Cu(PhCOCHCOMe)2 80 50 13 CO2Me
Z-Enol Transition State
(MeO)3PCuI RT 3 35 Wood, JACS 1997 9641
Wood, JACS 1999, 121, 1748
Rh2(OAc)4 RT 1 58 slow
None 80 15 54 Me The opposite
O O enantiomer is
it is evident that these are reactions of metal carbenoids Me observed!
Me OMe O O
Bien, S.; Gillon, A.; Kohen, S. J. Chem. Soc. Perkin Trans. I. 1976, 489-492 HO HO
Me O PhH, ∆, 18 hrs Me
(75%) Me OCH3
CO2Me
O O O
O O CH2t-Bu
Rh2(pfb)4 E-Enol Transition State 47% ee
Me N
Me N t-Bu + insertion
PhH, reflux O
products
N2 CO2Et Merck Thienamycin Process
pfb = perfluorobutyrate EtO
OH OH
H H H H
O NO2 Rh2(oct)4 Me
Me
OH
O O OH O NH O PhH, 80 °C N
O N2 O
Me CH2t-Bu CH2t-Bu 100% CO2p-NB
N H transfer Me N O
L4Rh2
- O+ O OH
H H NH3

EtO H EtO
S Thienamycin
N
Doyle, M. P., et al. J. Org. Chem. 1991, 56, 820-829.
Doyle, M. P.; Taunton, J.; Pho, H. Q. Tetrahedron Lett. 1989, 30, 5397. O Salzmann, JACS 1980 6161
CO2
35-07-carbonyl ylids-4 12/11/03 9:09 PM
M. Shair, K. Beaver Carbene Heteroatom Transformations: Sulfonium & Oxonium Ylids Chem 206

Ring expansion reactions have been investigated


Ylide Formation Methods based on sulfur ylides: (review) Vedejs, Accts. Chem. Res. 1984, 17, 358
O O
R 2X
R R CO2Et
OEt catalyst OEt Cu(I)
X S S 50%
N2 S
R R CO2Et CO2Et
N2
Reviews: Padwa, Chem. Rev. 1991 263
Padwa, Chem. Rev. 1996 223
Barnes, Evening Seminar, March 16, 1993 KOt-Bu TfO CO2Et
72%
S S DBU S
2 3
X is generally S, O or N and can be sp or sp hybridized
72%
Ylides often undergo sigmatropic rearrangements or cycloadditions Me
EtO2C
O

[2,3]-Sigmatropic rearrangement: OH Methynolide has been synthesized by Vedejs


Me
Ph HO using this ring-expansion methodology
O
S Me Vedejs, JACS 1989, 111, 8430
SPh O Me
O O E Et O
Rh2(OAc)4
OMe OMe O OMe O
N2 CO2Me
CO2Me
Rh2(OAc)4
Kido and Kato, JCS Perkins 1 1992 229 N2
MeO O MeO O Me
SPh Cl Me
E Me Me Cl
Acorenone B O

Pirrung et al JACS, 1991, 113, 8561. OMe O OH

Stevens Rearrangement ([1,2] alkyl shift): O


MeO O
O Cl Me
O AcO O
R2 Rh2(OAc)4
N N2 AcO Me Me griseofulvin
N Cu(I)
R1 O N R1
R1 R2 or Rh(II) O
R2 O
O
H H
West, JACS 1993 1177 N2 AcO O
Me
Clark et al.
Tetrahedron Lett. 1996, 37, 5605. O
35-08-ylide formation 12/11/03 9:10 PM
Chemistry 206

Advanced Organic Chemistry

Handout–35A

The Use of Fischer Carbenes in Organic


Synthesis

"...every synthetic chemist is well advised to follow this fascinating field with appropriate
attention."

- Schmalz, H.-G., ACIEE, 1994, 303.

Brian Connell

Evans Group Seminar, February, 1999

Friday ,
D. A. Evans
December 12, 2003

00-Cover page 12/12/03 8:12 AM


B. Connell Fischer Carbenes Chem 206

The Use of Fischer Carbenes in Organic Synthesis


"...every synthetic chemist is well advised to follow this fascinating field with appropriate attention."
- Schmalz, H.-G., ACIEE, 1994, 303.

Brian Connell
Evans Group Seminar
2/12/99

Outline
• Introduction and Fundamentals

• Reactions
• Cyclopropanation

• Diels-Alder Cycloaddition

• Other Cycloadditions

• Dotz Reaction and Analogs

• Photochemistry

• Conjugate Additions

• Other Reactions

General References:
Wulff,Organometallics, 1998, 3116.
Wulff,Comprehensive Organic Synthesis, Vol. 5, Chap 9.2: Metal Carbene Cycloadditions, Pergamon Press, 1991.
Wulff, Comprehensive Organometallic Chemistry II, Vol. 12, Chap 5.3: Transition Metal Carbene Complexes: Alkyne
and Vinyl Ketene Chemistry, Pergamon Press, 1994.
Hegedus, Comprehensive Organometallic Chemistry II, Vol. 12, Chap 5.4: Transition Metal Carbene Complexes:
Photochemical Reactions of Carbene Complexes, Pergamon Press, 1994.

Introduction

• Definition: electrophilic, heteroatom stabilized complexes having formal metal-to-carbon double bonds
• Group 6 metals (Cr, Mo, W) are the most common metals used.
• First prepared by Fischer (ACIEE, 1964, 580):

O O
RLi Li (CH3)4NBr N(CH3)4 can be prepared on
Cr(CO)6
large scale and stored
(CO)5Cr R (CO)5Cr R for long periods

"hard" alkylating reagents


(CH3)3OBF4, CH3COBr
• excellent yields for all steps
• cheap starting materials (20-50¢/mmol)
OCH3
(CO)5Cr
R

• air, silica stable


• crystalline, easy to handle
• colored (yellow to red)

01/02 12/20/99 3:49 PM


B. Connell Fischer Carbenes Chem 206

Nitrogen Analogs

O CH3
O
N(CH3)4 O NR2
H3C Br
O NHR2
(CO)5Cr (CO)5Cr
(CO)5Cr R
R R

O OTMS
O NR2
K2Cr(CO)5 TMSCl
R 1 R1 NR2 (CO)5Cr
R1 NR2 NR2
(CO)5Cr
(CO)5Cr R1

Dotz, Synlett, 1991, 381.

Selected Physical Data

H3C H3C
OCH3
N CH3 N CH3
(CO)5Cr
(CO)5Cr O
CH3
CH3 CH3

Bond Lengths Bond Lengths Bond Lengths


Ccarbene - O = 1.33 Å Ccarbene - N = 1.31 Å Ccarbonyl - N = 1.29 Å
Ccarbene - Cr = 2.04 Å Ccarbene - Cr = 2.16 Å Ccarbonyl - O = 1.23 Å
Cr - COcis = 1.86 - 1.91 Å Cr - COcis = 1.90 Å
Cr - COtrans = 1.87 Å Cr - COtrans = 1.85 Å

IR Frequencies IR Frequencies IR Frequencies


vCO ~2070, 1992, 1953 cm-1 vCO = ~2060, 1970, 1940 cm-1 vCO = ~1650 cm-1

1 1 1
H NMR H NMR H NMR
Ccarbene - CH3 = ~5 Ccarbene - CH3 = ~3.2 Ccarbonyl - CH3 = ~2.1

13 13
C NMR C NMR 13
C NMR
Ccarbene 320-360 ppm Ccarbene = 250-290 ppm
Ccarbonyl = ~165 ppm

03/04 12/20/99 4:21 PM


Major Contributors
Ernst Otto Fischer
I was born in Solln, near Munich, on 10 November 1918 as the third child of the Professor of Physics at the Technical College of Munich, Dr. Karl T.
Fischer (died 1953), and his wife, Valentine, née Danzer (died 1935). After completing four years at elementary school I went on to grammar school in
1929, from which I graduated in 1937 with my Abitur. Following a subsequent period of "work service" and shortly before the end of my two years'
compulsory military service, the Second World War broke out. I served in Poland, France and Russia. In the winter of 1941/2 I began to study
Chemistry at the Technical College in Munich during a period of study leave. I was released by the Americans in the autumn of 1945, and resumed my
study of Chemistry in Munich after the reopening of the Technical College in 1946. I graduated in 1949. I took up a position as scientific assistant to
Professor Walter Hieber in the Inorganic Chemistry Department, and under his guidance I dedicated myself to working on my doctoral thesis, "The
Mechanisms of Carbon Monoxide Reactions of Nickel II Salts in the Presence of Dithionites and Sulfoxylates". After receiving my doctorate in 1952, I
was invited by Professor Hieber to continue my activities at the college and consequently chose to specialise in the study of transition metal and
organo-metallic chemistry. I wrote my university teaching thesis on "The Metal Complexes of Cyclopentadienes and Indenes". I was appointed a
lecturer at the Technical College in 1955 and in 1956 I completed a scientific sojourn of many months in the United States. In 1957 I was appointed
Professor at the University of Munich. After turning down an offer of the Chair of Inorganic Chemistry at the University of Jena I was appointed Senior
Professor at the University of Munich in 1959 . In 1957 I was awarded the Chemistry Prize by the Göttingen Academy of Sciences. The Society of
German Chemists awarded me the Alfred Stock Memorial Prize in 1959. In 1960 I refused an appointment as Senior Professor in the Department of
Inorganic Chemistry at the University of Marburg. In 1964 I took the Chair of Inorganic Chemistry at the Technical College of Munich, which had been
vacated by Professor Hieber. In the same year I was elected a member of the Mathematics/Natural Science section of the Bavarian Academy of
Sciences; in 1969 I was appointed a member of the German Academy of Scientists Leopoldina. In 1972 I was given an honorary doctorate by the
Faculty of Chemistry and Pharmacy of the University of Munich.
Lectures on my fields, particularly those on metallic complexes of cyclopentadienes and indenes, metal-pie-complex s of six-ringed aromatics, mono-,
di- and oligo-olefins and most recently metalcarbonyl carbene and carbyne complexes, led me on lecture tours of the United States, Australia,
Venezuela, Brazil, Israel and Lebanon, as well as numerous European countries, including the former Soviet Union. In 1969 I was Firestone Lecturer at
the University of Wisconsin, Madison,Wisconsin, USA; in 1971 Visiting Professor at the University of Florida, Gainesville, USA, as well as the first
Inorganic Chemistry Pacific West Coast Lecturer. In the spring of 1973 I held lectures as the Arthur D. Little Visiting Professor at the Massachusetts
Institute of Technology, Cambridge, Massachusetts, USA; and that was followed by a period when I was Visiting Distinguished Lecturer at the
University of Rochester, Rochester, New York, USA.

Nobel lecture: On the way to carbene and carbyne complexes. Angew. Chem. (1974), 86(18), 651-63.

Karl Heinz Dotz


Kekule-Institut fur Organische Chemie und Biochemie der Universitat Bonn
Born1943
Ph.D. Technical University of Munich (E.O. Fischer) 1971.
Habilitation Technical University of Munich 1980.
Professor of Organometallic Chemistry University of Marburg 1986-1992, Dean of the Faculty 1990-1991.
Professor of Organic Chemistry University of Bonn since 1992.

Karl's research is focused on the following areas:


• Synthetic Organometallic Chemistry (metal carbenes and planar-chiral arene complexes)
• Physical Organic and Organometallic Chemistry
(distorted fused arenes and cyclophanes: synthesis and structure-chiroptics correlation,transition metal NMR spectroscopy, Ab
initio-calculations on organometallic complexes and intermediates)
• Metal-Mediated Organic Synthesis (stereoselective C-C formation via metal carbenes, diastereoselective benzannulation and
cyclopentannulation)
• Organometallic Catalysis (chromium-catalyzed cyclopropanation, axial-chiral and redox-active biaryl ligands)
• Transition Metal Modified Sugars (metal glycosylcarbenes and glycosylidenes: synthesis and application in C-glycosidation,
disaccharide mimetics,conformation of acyclic metal modified sugars)

01a contributors1 2/13/99 8:42 AM


Major Contributors

Claude F. Bernasconi Jose Barluenga


Charles P. Casey William D. Wulff
University of California at Santa University of Oviedo
Cruz University of University of Chicago
Wisconsin-Madison Born Eau Claire, Jose Barluenga obtained his Louis S. Hegedus
Claude was born in Zurich, Born 1942, St. Louis, MO Wisconsin, 1949 Ph.D. degree (solvomercuration
B.S. 1963, St. Louis B. S. 1971, University of Colorado State University
Switzerland. He received his of dienes) at the University of Born Cleveland, Ohio, 1943
undergraduate and Ph.D. University Wisconsin-Eau Claire
Ph.D. 1968, MIT Zaragoza in 1966 under the B.S, 1965, Penn State University
degrees from the Swiss Federal Ph.D. 1979, Iowa State direction of Professor V.
Institute of Technology (ETH) University M.A. 1966, Penn State University
with Heinrich Zollinger. Chuck received a Ph.D. in Gomez-Aranda. He spent 3.5 Ph.D., 1970, Harvard
Following a postdoctoral year organic chemistry from MIT years as a postdoctoral fellow at
in 1968, where he studied Professor Wulff received Max Planck Institut Fur
with Manfred Eigen at the Max his Ph.D. degree from Lou was born in 1943 in
Planck Institute for Biophysical organocopper chemistry Kohlenforschung, Mulheim, in the Cleveland, Ohio, but grew up in
Chemistry in Gottingen, he under the direction of Iowa State University in group of Professor Hoberg
Professor George M. 1979 with Professor rural Ohio, away from big city
joined the chemistry faculty at studying aluminum chemistry. In temptations. He did his
the University of California at Whitesides. After spending Thomas Barton. After NIH 1970 he took a position as a
6 months at Harvard as an postdoctoral work with undergraduate studies at
Santa Cruz in 1967, where he research associate at the Pennsylvania State University,
has been a professor of NSF postdoc with Paul D. Martin Semmelhack at
Bartlett he joined the faculty University of Zaragoza, where he where studied aqueous chromium
chemistry since 1977. His main Princeton University, he was promoted to Associate
research interests are in at Wisconsin. Chuck is
accepted a position at the redox chemistry with Professor
physical organic chemistry and interested in studying the Professor in 1972. In 1975 he Albert Haim. After Ph.D. studies
mechanisms of University of Chicago in moved to the University of
center on problems of 1980. Professor Wulff's at Harvard on nickel carbonyl
mechanism, structure-reactivity organometallic reactions Oviedo as Professor of Organic chemistry with E. J. Corey (1970),
relationships, intrinsic barriers of and in developing an research interests are in Chemistry in the Department of
understanding of the applications of and a NIH postdoctoral year at
reactions, and catalysis in Organometallic Chemistry. His Stanford with J. P. Collman
organic and organometallic homogeneous catalysis. In organometallics in organic major research interest is
addition, he is trying to synthesis as both reagents studying polymer-supported
reactions, particularly proton focused on the development of homogeneous catalysis, he
transfer reactions, nucleophilic design new organometallic and catalysts.
reagents for synthesis and new synthetic methods in the moved to Colorado State
addition to electrophilic alkenes, area of heterocyclic chemistry
nucleophilic vinylic substitution new heterobimetallic University, where he remains
and reactions of Fischer catalysts. and functionalized systems. today as a professor of chemistry.
carbene complexes. His research interests center on
the use of transition metals in
organic synthesis.

01b contributors2 2/13/99 8:40 AM


B. Connell Fischer Carbenes Chem 206
Recurring Themes

The Wall of CO Resonance

CO OCH3 OCH3
CO OCH3 (CO)5Cr (CO)5Cr
OC Cr R R

OC R
CO
•Rotation about heteroatom carbene
bond is restricted by 14-25 kcal/mol
• 53Cr NMR is consistent with strong
• (CO)5Cr is sterically very large resonance contribution

Hegedus and Dotz JACS, 1988, 8413.

Kinetic Electrophilicity

OCH3 OCH3
HOCH3
(CO)5Cr (CO)5Cr OCH3

CH3 CH3 H

•Formation of tetrahedral intermediate is 109 faster than CH3O– addition to BnO2CCH3.

Bernasconi
Chem. Soc. Rev., 1997, 299.
JACS, 1998, 8632.

pKa Data

Thermodynamic Acidity

H3C H3C
OCH3
N CH3 N CH3
(CO)5Cr
(CO)5Cr O
CH3
CH3 CH3

• pKa(THF) = 8 • pKa = 20.4 (DMSO) • pKa = 35 (DMSO)


• pKa(H2O) = 12.3

equivalent to p-cyanophenol

Casey, JACS, 1974, 1230.

05/06 12/20/99 4:22 PM


B. Connell Fischer Carbenes Chem 206

Sensitivity to Acid

OCH3 H3CO
pyridine
(CO)5Cr + (CO)5Cr•pyr
CH3

pyridine reductive elimination

H OCH3
OCH3 pyridinium
(CO)5Cr (CO)5Cr

CH2 CH2

See Hegedus, JACS, 1990, 6255.

Metal Removal and Functionalization

O
OCH3
CH3Li, ClCH2I Barluenga, TL, 1994, 9471.
(CO)5Cr
Ar CH3
Ar
80-90%

OCH3 OCH3
CH2N2
(CO)5Cr Casey, TL, 1973, 1421.
R
R

OCH3
OCH3 Ph3PCH2
H2C Casey, JACS, 1972, 6543.
(CO)5Cr
Ar
Ar

Via similar intermediates:

OCH3 OCH3 OCH3


(CO)5Cr Ar (CO)5Cr R (CO)5Cr Ar

Ph3P
Cl N2

07/08 12/20/99 4:22 PM


B. Connell Fischer Carbenes Chem 206
Selected Reactions of Unsaturated Fischer Carbene Complexes

OCH3 OCH3
OH
R1 R2 (CO)5Cr R2 3
R3 R R
(CO)5Cr Nu
R1 R3
R1
R OCH3
R4 Nu
4
R
R2 = H O
3
R R 5 HN OCH3 R2
R2 R
(CO)5M R2
5 R R 3
R4 R
2 3
R ,R ≠H R1
R1 R3
H3CO N R4 OCH3
R R4

Bu3SnH
R2 = NR2
R3 R1
R OCH3
OCH3
OCH3 R2

Bu3Sn R2
R4
3
O R
R R1 R3

Selected Reactions of Saturated Fischer Carbene Complexes


OCH3
OCH3
CH3 H3C
O
CH3 R2
O
R3 O
XR6 R1 R1

(CO)5M OH R1
OCHR2R3

R
OCHR2R3 O
RCHO
(CO)5M XR6 R
R O
XR6
O
(CO)5M O
CH3
CH3 R
R OCH3
XR6 = OCH3, N(R5)2
O RX R1
N

XR 6 R3SnCl R2
R2 O
(CO)5M
N
R XR6
6
R3Sn XR
R1 CH3

09/10 12/20/99 3:51 PM


B. Connell Fischer Carbenes Chem 206

Cyclopropanation

CO2CH3

OCH3
n-Bu
(CO)5Mo
THF, 65 °C CO2CH3
n-Bu OCH3
78%
Harvey, TL, 1990, 2529.
E:Z
1.9:1

OCH3
OTBS
(CO)5Mo
CH3

TBSO
25 °C, 49% OCH3
CH3 >95% cis
Wulff
Pure Appl. Chem., 1988, 137.
JACS, 1988, 2653.

Cyclopropanation
Reaction with Alkynes
R2
2
R
OCH3
R1 OCH3 OCH3
(CO)5Cr
R2 M(CO)4
R1 M(CO)4
R1

OCH3
n-Bu
(CO)5Cr
n-Bu
OCH3
O O
PhH, 100 °C
81% Harvey, JACS, 1992, 8424.

PhH
OCH3
OCH3 reflux
97% Hoye, JACS, 1988, 2676.
(CO)5Cr

11/12 12/20/99 3:54 PM


B. Connell Fischer Carbenes Chem 206
Selective Cyclopropanation

OCH3
Ph OCH3
Ph (CO)5Cr Ph OCH3
Ph H
Ph Ph H SiO2
N H O
THF, 80 °C, 3 h H N
H t-Bu H
H t-Bu

OCH3

(CO)5Cr n-Bu OCH3


n-Bu
n-Bu OCH3
O3
H
O
DMF, 152 °C, 4% BHT H
Fe
30 min H
88%, 97% de Fe

Barluenga
Chem. Commun., 1995, 665.
JACS, 1997, 7591.

Cyclopropanation
Reaction with Alkynes

PhH
O H
CH3 65 °C, 55%
O O CH3 CO2Et
Mo(CO)5 CO2Et

O
CH3
CO2Et

Harvey, JOC, 1992, 5559.

13/14 12/20/99 3:55 PM


B. Connell Fischer Carbenes Chem 206
[4 + 3] Annulation
Cr(CO)5
Ph NHt-Bu

NHt-Bu Ph OCH3

Et N -78 °C to -40 °C
H3CO N Et
H 91%

[2 + 1]

Ph Ph NHt-Bu
Barluenga
[3,3] Chem. Commun., 1994, 321.
HN
JACS, 1995, 9419.
OCH3 NHt-Bu
JACS, 1996, 695.
Et H3CO N Et Chem. Eur. J., 1996, 88.
H

Diels-Alder Cycloaddition

H3CO

OCH3 O
CH3 70:30
O regioselectivity
25 °C, 7 mo
54%
H3C

H3CO
OCH3 (CO)5Cr
CH3
4
10 times faster than (CO)5Cr 92:8
methyl acrylate 25 °C, 3 h regioselectivity
70%

H3C

Wulff, JACS, 1990, 4550.

15/16 12/20/99 3:56 PM


B. Connell Fischer Carbenes Chem 206
Diels-Alder Cycloadditions

(OC)5Cr
Me
CH3
H3CO Cr(CO)5
25 °C, 2 h
85% H3CO

O Me
CH3
O
H3CO 170 °C, 1 h
22% H3CO

Wulff
JACS 1983, 6726.
JACS 1990, 3642.
O
W(CO)5
Si(CH3)2
Si(CH3)2
O H3CO
H3CO O
Si(CH3)2
H
H
O
120 °C, 48 h 25 °C, 2 h
H3CO O
addition of LA causes decomp 100% H3CO W(CO)5

Diels-Alder Cycloadditions

CH3
Me Cr(CO)5
+
25 °C, 2 h
Me OCH3
H3CO Cr(CO)5 87% H3CO Cr(CO)5 9:1

H3C Me
+ Cr(CO)5
Me
H3CO (CO)5Cr 25 °C, 2 h
OCH3
88% H3CO Cr(CO)5
8.5:1.5

Wulff, JACS, 1990, 3642.

17/18 12/20/99 3:57 PM


B. Connell Fischer Carbenes Chem 206
Asymmetric Exo-Selective Diels-Alder Reaction
CH3 CH3
CH3 CH3 N
N TBSO
O
O
OCH3 N
N Ph
Ph exo:endo
(CO)4Cr CH3
(CO)4Cr 25 °C, 12 h, 80% >96:4

H3CO
CH3

TBSO OTBS

O
O non-oxygenated dienes give ~85:15 exo:endo
H3C C
O
HC
C
OCH3
M

C O
N
O
NCH3
Ph

H3C
Wulff, JACS, 1997, 6438.
Proposed Transition State

[2 + 2] Cycloaddition

O
H3C
H3CO W(CO)5 H3C
O W(CO)5
DMSO O O

25 °C, 6 h 10 min
CH3 95%
97% CH3
CH3

corresponding ester
does not react

O
H3C
H3CO O
O
O O CO2CH3
+
180 °C, 16 h CO2CH3
O

O OCH3 H3CO

36% 24%

Wulff, JACS, 1988, 8727.

19/20 12/20/99 3:58 PM


B. Connell Fischer Carbenes Chem 206
[3 + 2] Cycloaddition

OCH3 OCH3
H3CO M(CO)5 CH3
(OC)5M CH3 O
TMSCHN2 CAN

25 °C, 2 h 97% HN
HN
76 - 87% N 30 min N
CH3
>300:1 regioselection

OCH3 H3CO
H3CO O
O CH3 H3C O
TMSCHN2
+
70 °C, 5 d HN HN
N N
CH3
25% 48%
Wulff, JACS, 1986, 6726.
Barluenga TL, 1998, 4887.
Barluenga JCS Perkin I, 1997, 2267.

Dotz Reaction
Thermal Reaction of Unsaturated Carbene Complexes
"...one of the most utilized reactions in natural product synthesis involving an organometallic
process."

OH
OCH3
RL
(CO)5Cr ∆
RL RS
–CO RS
(CO)3Cr OCH3

Observed Connectivity:

RL

RS
H3CO (CO)5Cr
Dotz
ACIEE, 1975, 644.
New J. Chem., 1990, 433.
ACIEE, 1984, 587.

21/22 12/20/99 3:59 PM


B. Connell Fischer Carbenes Chem 206
Dotz Proposed Reaction Mechanism

OCH3
OCH3 OCH3 RL RS
–CO (CO)4Cr
(CO)5Cr (CO)4Cr RL

RS
∆S‡ = +6.2 e.u.
∆H‡ = +26 kcal/mol
rate-limiting CO dissociation

OCH3 OCH3

RS OCH3 (CO)4Cr
(CO)3Cr (CO)4Cr

RL RS RL RS
O RL

O
800 psi CO HO OCH3
OCH3 HO OCH3

RL RS
RL RS RL RS
Cr(CO)3 +
Cr(CO)3
Cr(CO)6

Barluenga, JACS, 1994, 11191.

Dotz Alternative Workup Procedures


air
HO OCH3
or FeCl3

RL RS

HO OCH3 (NH4)2Ce(NO3)6
O O
H 2O
RL RS RS
Cr(CO)3 RL

(NH4)2Ce(NO3)6 OCH3
O
OCH3
CH3OH
RL RS

See Wulff, JOC, 1984, 2293.


23/24 12/20/99 4:00 PM
B. Connell Fischer Carbenes Chem 206
Dotz: Nitrogen Analog

N
H Ph
N
(CO)5Cr THF, 60 °C
then SiO2 Ph CO2Et
CO2Et OH
95%
Wulff, JOC, 1995, 4566.
Barluenga, JOC, 1998, 7588.

Dotz: Large Scale Applicability

OAc
1.5 equiv.
OCH3 n-Bu
n-Bu
(CO)5Cr
THF, reflux, 45-60 min
Ph
1.1 equiv. Ac2O
1.1 equiv. NEt3 OCH3
0.16 equiv. DMAP
U-66,858 Timko
lipoxygenase inhibitor TL, 1988, 2513.
400 g scale Org. Synth., 1992, 72.
68%

Biaryl Synthesis

OCH3
2 equiv. (CO)5Cr H R1 R1
+

R2 R3

OCH3

R1 R3

R1 R2
HO OCH3
2 3OH
R R

The concept works, in moderate to low yield, but the reactions must be run stepwise.
Occasionally CO insertion is suppressed and five-membered rings are formed.

Wulff, JACS, 1996, 2166.


25/26 12/20/99 4:00 PM
B. Connell Fischer Carbenes Chem 206
Intercepted Intermediates

(CO)6Cr CH3 O
H3C O
Et Et
H3CO Et

37% CH3
H3C OH
Et OCH3

via HO

CH3 O
H3C O
O H
Et
CH3 OCH3
CH3

Et Et Et OCH3
Cr(CO)3

HO OCH3

Et Et Wulff, Chem. Commun., 1996, 1863.


Cr(CO)3

Synthetic Uses

OCH3
OCH3
OCH3 OH
CH3 OCH3
TMS
TMS
CH3
H3CO2C
OCH3
CH3CN, 45 °C, 24 h
OCH3 Cr(CO)5 CO2CH3
66% OCH3 OCH3

O O
OCH3
CH3

OH O OCH3

menogaril
antitumor antibiotic Wulff, JOC, 1998, 840.
27/28 12/20/99 4:05 PM
B. Connell Fischer Carbenes Chem 206
Synthetic Uses

1) Ot-Bu OTf
AcO
AcO
O
OCH3
CH2Cl2, 45 °C, 24 h O
H3CO Cr(CO)4 2) Tf2O, pyr
CH3O OCH3 Ot-Bu
67%

OCH3 OH
H
O OH
disaccaride

O CH3
R O

OH OH O trisaccaride

R = CH3 Chromomycin
R = H, Olivomycin

antitumor antibiotics Wulff, Synthesis, 1999, 80.

Application to Synthesis

TMS 8 equiv. OTMS


TMS OTMS

(CO)5Cr
(CO)5Cr O
1.5 equiv.
H3CO H3CO

O
O
O O
TMS
OTMS

50 °C, THF, 3 d
~25%
OCH3

O O
O OH
CH3 OTMS
O
OTMS
OH

OCH3 O OH OH
OCH3
Daunomycinone
Wulff, JACS, 1984, 434 & 7565.
29/30 12/20/99 4:05 PM
B. Connell Fischer Carbenes Chem 206
Dihydrofluorene Synthesis

OCH3
H3CO W(CO)5 H3C
H3C
-20 °C to rt
+
12 h N
N
>95% O
O Ph

OCH3 OCH3
OCH3
H3C H3C
W(CO)5 W(CO)5 H3C

N N
N
O O
O

Barluenga, Chem Commun., 1995, 1973.

Alkyne-Alkene Reactions
1)
CH3
O O
H3C
OCH3
(OC)5Cr H3C

CH3 CH3CN, 70 °C, 3 h


CH3
83%
2) H+

H+

O H3C O H3C
H3C
OCH3
(OC)5Cr OCH3
H3C OCH3
H3C
H3C Cr(CO)3 CH3
CH3
CH3

Wulff, JOC, 1993, 5571.


31/32 12/20/99 4:06 PM
B. Connell Fischer Carbenes Chem 206

Ketene Cyclizations

OCH3 OCH3
hν OH
Cr(CO)5

60 - 90%

Via:
CH3O
Cr(CO)4

Merlic, JACS, 1992, 5602.

"Asymmetric" Benzopentaannulation

CH3
R *O
H3C Ph Li
H3C 90%, 2:1 diastereoselectivity
then
O CH3 MeOTf quench
Ph
(CO)5W Ph

R *O

H3C
W(CO)5
Ph

OR* Ph R *O
R *O
Ph [1,3] MeOTf
(CO)5W H3C

Ph
(CO)5W W(CO)5
Ph
Ph

Barluenga, JACS, 1998, 12129.

33/34 12/20/99 4:07 PM


B. Connell Fischer Carbenes Chem 206
Furan Synthesis

R3
CH3
1) OCH3 O

O (CO)5Cr O
R4
3
CH3 R
R4
66 - 84%
R1
+
2) H3O R1 R2
R2

H3O+

R4 R2 R4 R2 R2
R1
R1
O R1
O R4
(CO)4Cr O R3
R3 (CO)4Cr
R3

H3C H3C
H3C OCH3
OCH3
OCH3
Herndon, JOC, 1998, 4564.

"A Versatile [4 + 2 + 1 - 2] Cycloaddition"

O
OCH3
RL RS RL
(CO)5Cr 40 - 90%
99:1 dioxane:H2O
8 h, ∆
RS OCH3

(CO)3Cr OCH3 (CO)3Cr


(CO)4Cr OCH3
RL
O RS O OCH3 Cr
RL RS (CO)3
CH3O
RS
RL RL RS
loss of
H2C CH2

O O

RL
OCH3 RL Cr0, H2O
same OCH3
(CO)5W
conditions OCH3
RS OCH3 RS
65% O RS
Herndon
RL JACS, 1988, 3334.
TL, 1989, 295.
JOC, 1990, 786.
JACS, 1991, 7808.
JACS, 1992, 8394.

35/36 12/20/99 4:08 PM


B. Connell Fischer Carbenes Chem 206
Photochemistry

• Electronic absorption consists of three low-lying bands:


• ~500 nm: spin-forbidden M → carbene π* charge transfer transition
• 360-450 nm: spin allowed M → carbene π* charge transfer transition (visible)
• 300-350 nm: ligand field transition

• In addition, all carbene complexes absorb strongly below 300 nm.

• Exposure to light leads to a reversible CO insertion:

OCH3 H3CO CH3


OCH3 hν
(CO)4Cr
(CO)4Cr CH3 (CO)4Cr

CH3
OC O O

Geoffroy, JACS, 1983, 3064.

Molecular Orbital Diagram


OCH3 OCH3
(CO)5Cr (CO)5Cr
CH3 CH3

LUMO is
carbene carbon
p-orbital centered

HOMO is
metal-d-orbital
centered

• Photolysis results in a formal, reversible,


one electron oxidation of the metal.

Geoffroy, JACS, 1983, 3064.

37/38 12/20/99 4:09 PM


B. Connell Fischer Carbenes Chem 206
Ketene [2 + 2]

S
H3C H
OCH3 S
(CO)5Cr N
H3CO
CH3 hν N
81% O
Hegedus
Tetrahedron, 1985, 5833.
JOC, 1997, 3586.

OCH3 Ph
Ph Ph
(CO)5Cr C8H17 C H
H3CO 8 17
H3CO MCPBA
C8H17 N O
N O N O
92% O
hν O O
O
O 84%, ≥97% de
O

Hegedus
JOC, 1995, 3787. TBAF 90%
JOC, 1996, 6121.
Organometallics, 1997, 2313.
JOC, 1998, 4691 & 8012. H3CO C8H17
O O
1) NaOCl 30% O
H2N C8H17
2) NH3 100%
O
O
(+)-Cerulenin

Ene Carbamate Synthesis

OH Ph

HN 2 equiv. NaH
(CO)5Cr Ph O N O
CH3
PhO OPh O

88%

O O O
O O
PhONa PhOH O
N N
Ph Ph N
Ph
(CO)4Cr (CO)4Cr (CO)4Cr
CH3 CH2
H CH2

Hegedus, JACS, 1990, 6255.


39/40 12/20/99 4:10 PM
B. Connell Fischer Carbenes Chem 206
Amino Acid Synthesis
O
H3C

H3C
hν, t-BuOH R Ot-Bu 60-80%
N ≥97% de
N CH3
(CO)5Cr Ph Ph

CH2R CH3

• Mutiply-labeled amino acids are readily


prepared from (13CO)6CO and CH3OD:

O
13
C
R 13
C Ot-Bu
Ph
N
D Hegedus
CH3 JACS, 1990, 2264.
JACS, 1992, 5602.
CH3 JACS, 1993, 87.
Acc. Chem. Res., 1995, 299.
JOC 1995, 5831.
JACS, 1995, 3697.
JOC 1997, 7704.

Peptide Synthesis

H3C O CH3
hν H3C
H3C N CO2t-Bu 68 - 88% yield
N H
NH2 Ph N CH3 80-96% de
(CO)5Cr Ph
CH3 H3C CO2t-Bu CH3

Solid Support
Merrifield Resin: Acc. Chem. Res., 1995, 299.
PEG: JOC 1995, 5831; JOC 1997, 7704.

• Reactions have been performed and do work, but are not as practical because the failure to achieve
100% yields and high diastereoselectivity limit this method.
• hard to work with because the polymer "sticks to everything, making quantitative transfer difficult".
Acc. Chem. Res., 1995, 299.

41/42 12/20/99 4:10 PM


B. Connell Fischer Carbenes Chem 206
Aryl Glycines

Ph
Ph H H
Ar N Ph Ar N Ph

HN (CO)5Cr
OH
(CO)5Cr Ph
HO Ph O O
Ar O

H2, PdCl2

Aryl glycines cannot be made via


the previous route due to instability H3N COO
of the required carbenes:
Ar
H3C
40 - 95% yield
H3C 56 - 98% ee
N
(CO)5Cr Ph
Ar

Hegedus, JOC, 1992, 6914.

Tertiary Amino Acids

O Ph
H3C O
O
N H3C N
H3C O
N H3C
(CO)5Cr Ph hν N
H O
1) 0.2 M HCl

2) phosgene

Ph
Ph O
O Ph
R O
R N
N O
O HCl, CH3OH O 1) KHMDS N
O O
O
N 2) RX, DMF
CH3O2C H
O N
O

tertiary amino acids

Hegedus, JOC, 1993, 5918.

43/44 12/20/99 4:11 PM


B. Connell Fischer Carbenes Chem 206
Zwitterionic Aza Cope Rearrangement

OCH3
(CO)5Cr
CH3 Bn
N OCH3
N hν
Bn CH3
71% O
91:8
81:19

H3CO CH3 H3CO CH3


[3,3]
(CO)4Cr

O N O N
Bn Bn Hegedus, JOC, 1996, 2871.

Michael Reaction

(CO)4
Cr O
(CO)4Cr O O R
OLi
N NCH3
N NCH3
-40 °C

R
R Yield syn:anti
Ph 81% 99.5:0.5
CH3 92% 98.3:1.7

Wulff, JACS, 1993, 4602.

O O
N(CH3)2 a) n-BuLi
(CO)5Cr (CO)5Cr DMSO O
b) O
CH3 71%
(CH3)2N (CH3)2N

-78 °C, 20 min


Heathcock, JOC, 1986, 279.
85%

45/46 12/20/99 4:12 PM


B. Connell Fischer Carbenes Chem 206
Asymmetric Michael Addition

(CO)4
OLi O CH3 Cr O
(CO)4Cr O

Ph H3C N NCH3
N NCH3
-20 °C
Ph CH3
H3C Ph CH3 90%
3:1diastereoselection

(CO)4
(CO)4Cr O O R2 Cr O
a) n-BuLi

H3C N NCH3 R1 N NCH3


b) O

Ph CH3 R1 R2 Ph CH3
80%
>96:4 diastereoselection

(CO)4
O CH3 Cr O
O CH3 O
1) CAN
R1 N NCH3
R1 OCH3
2) NaOCH3
Ph CH3
Wulff, Chem. Commun., 1996, 2601.

Asymmetric Michael Reaction

OLi

OCH3
OR* Ph O
CH3
(CO)5Cr OCH3

CH3
74%, 84% de
CH3
Ph

H3C

O CH3

(CO)5Cr Ph

OLi
OR* Ph O
H3C
Ph
(CO)5Cr Ph

CH3

89%, 99% de
Barluenga, Chem. Eur. J., 1995, 236.

47/48 12/20/99 4:12 PM


B. Connell Fischer Carbenes Chem 206

Michael Reactions: Selectivity

CH3

OC CO

OC Cr O CO H3C CH3
H CO CO O
R1 H
N NCH3 OC Cr
C
2
R OC
O
H
Ph CH3 CO
R

Open Transition States are Postulated

Pyridine Synthesis

H
EtO W(CO)5 HN O CH3
N O
H3C CH3

then HBF4 CH3


Ph Ph
75%

HBF4

CH3
HN O
(CO)5W
CH3
Ph

W(OC)5 W(OC)5
EtO W(OC)5 EtO
EtO HN
(CO)5W H
H HN
H N NH CH3
EtO CH3
CH3

O CH3 H O
O
H H3C
Ph H3C O H3C
CH3
Aumann, Synlett, 1993, 669.

49/50 12/20/99 4:13 PM


B. Connell Fischer Carbenes Chem 206
Alkylation

OCH3 OCH3
a) n-BuLi
(CO)5Cr (CO)5Cr Hegedus Org. Synth., 1987, 140.
CH3 b) MeI
CH2 CH3
22%

OCH3 OCH3 OCH3


a) n-BuLi
(CO)4Cr (CO)4Cr + (CO)4Cr
b) EtI
Bu3P CH3 Bu3P Bu3P
CH3
65% CH3
CH3
7:3 Bu3P substitution often leads to
different reactivity:
TL, 1995, 8159.
OCH3 OCH3
a) n-BuLi
(CO)5Cr (CO)5Cr
CH3
b)
OTf Wulff, JOC, 1987, 3263.
80%

N a) n-BuLi
N
(CO)5Cr b) EtI Wulff, TL, 1989, 4061.
(CO)5Cr CH3
CH3 87%

"Aldol" Reaction
OCH3 OCH3
a) n-BuLi
(CO)5Cr (CO)5Cr OH Wulff, JACS, 1985, 503.
CH3 b) 10 equiv.
PhCHO/BF3•OEt2 Ph
81%

a) n-BuLi
N N
b) 1.1 equiv. (CO)5Cr OH
(CO)5Cr Wulff, JACS, 1989, 5485.
PhCHO/BF3•OEt2
CH3 96%
Ph
Ketones work as well.

(CO)4Cr O
OH O O
a)n-BuLi
H3C N NCH3 b) Bu3B H3C
N NCH3
c) i-PrCHO Wulff, JOC, 1994, 6882.
CH3
Ph CH3 then CAN Ph CH3
74%
ds: 98.6: 1.4 oxazolidinone complexes were unstable:
Hegedus, JACS, 1990, 6255.
in absense of Bu3B, ds: 91:9
51/52 12/20/99 4:13 PM
B. Connell Fischer Carbenes Chem 206

Ene Reaction

OCH3 OCH3
(CO)5W CH3
H3CO W(CO)5 (CO)5W CH3
OR

25 °C, 4 h RO
+
RO +
R = CH3 90%
R = TBS 96%
CH3
R = CH3 23: 77
R = TBS 90:10

Wulff, JACS, 1990, 6419.

Reaction of Ketene Acetals

H3C OCH3 O
1)
CH3
OCH3 O 24:1
80 °C, 3 d trans:cis
O
2) aq. HCl H3C
(CO)5Cr
CH3
CH3
50%
H3C

(±)-eldanolide
sex pheromone

H3C
CH3
CH3 H3C
H3C
CH3
H
O
O OCH3 O
H3C OCH3
(CO)5Cr OCH3 (CO)5Cr
CH3 CH3 OCH3 OCH3
OCH3
Wulff, JACS, 1992, 10665.

53/54 12/20/99 4:14 PM


B. Connell Fischer Carbenes Chem 206
Vinylsilane Synthesis

RCH2Li or RCH2MgX
CeCl3 R = i-Pr, 94%, 96:4 E:Z
R R = Bu, 86%, 96:4 E:Z
H3CPh2Si
-78 °C, cold quench R = i-Bu, 85%, 97:3 E:Z
OCH3
(CO)5Mo
SiPh2CH3
RCH2Li or RCH2MgX
R = i-Pr, 76%, 92:8 Z:E
CeCl3, HMPA H3CPh2Si R = Bu, 79%, 97:3 Z:E
-78 °C to rt R = i-Bu, 61%, 97:3 Z:E
R

Iwasawa, Chem. Lett., 1994, 231.

Vinylsilane Synthesis
Proposed Mechanism

Li H OCH3
OCH3 OCH3 H 2O
n-BuLi
(CO)5Mo (CO)5Mo n-Bu
(CO)5Mo n-Bu -78 °C
SiPh2CH3 SiPh2CH3 SiPh2CH3

warm – HOCH3
– LiOCH3

n-Bu
Li n-Bu
CH3OLi (CO)5Mo
(CO)5Mo (CO)5Mo SiPh2CH3
SiPh2CH3 SiPh2CH3

hydrogen
migration

H
SiPh2CH3
(CO)5Mo

SiPh2CH3 SiPh2CH3

55/56 12/20/99 4:14 PM


B. Connell Fischer Carbenes Chem 206
Allyl Stannane Synthesis

W(CO)5 SnBu3
Bu3SnH, pyr
H3C OCH3 H3C OCH3
73%

Ph Cr(CO)5 Ph SnBu3
Bu3SnH, pyr

H3CO OCH3 69% H3CO OCH3

1,1 addition is usually observed, however 1,3 addition is possible:

W(CO)5

Bu3SnH, pyr E:Z


OCH3 Bu3Sn OCH3
1:1
88% TMS
TMS

W(CO)5
Bu3SnH, pyr
Bu3Sn OMOM
MOM
66% Me
Me Merlic, TL, 1995, 1007.

Halban-White Cyclizations
PhO
OPhO O
O Ph OPh O Ph
Ph Ph
Ph O hν Ph Ph

Ph Ph O
PhCO COPh PhCO COPh O
O O
Andres, A. Dissertation, Strasbourg, 1911.
van Halban, Helv. Chim. Acta, 1948, 1899.

O R6
R2
R5
OCH3 R1 O
(CO)5M R6 R2 O R4 yields 30-70%
Dotz byproduct
R4 R5 R1 R3

6
O R2 R R5
O

R4
1
R H3CO
Wulff
JACS, 1990, 1645.
JACS, 1991, 5459.
57/58 12/20/99 4:15 PM
D. A. Evans Introduction to Organosilicon Chemistry Chem 206

http://www.courses.fas.harvard.edu/~chem206/
Problems to Contemplate
Chemistry 206 Me
Explain what drives this rearrangement. TBS = Si CMe3
Advanced Organic Chemistry
Me

Lecture Number 36 TBSO OH OH OTBS


KHMDS
Bu3Sn Bu3Sn
THF, -78 °C
Introduction to Organosilicon Chemistry OMe Me 94% OMe
■ Silicon Bonding Considerations Calter, M. A. Ph. D. Thesis, Harvard University, 1993.
■ The Silicon–Proton Analogy
■ C=O Addition of Organosilanes
The C=O addition illustrated in eq 1 proceeds while the carbon analogue (eq 2)
■ Sigmatropic Rearrangements of Organosilanes does not. Explain
■ Anionic (Brook) Rearrangements
■ Peterson Olefination Reaction TMS
O
SiR3 OTMS
O O (1)
■ Survey of Silicon (and related) Protecting Groups O R P
RO P R P
Reading Assignment for this Lecture: OR RO OR
RO OR
Carey & Sundberg, Advanced Organic Chemistry, 4th Ed. Part B O
Chapter 9, " C–C Bond Forming Rxns of Boron, Silicon & Tin",
595–680. R H Me Me OMe
fails! O
O O (2)
Fleming, I.; Barbero, A.; Walter, D. "Stereochemical control in organic synthesis O R P
RO P R P
using silicon-containing compounds." Chem. Rev. 1997, 97, 2063-2192. (Web) RO OR
OR RO OR
Moser, W. H. "The Brook Rearrangement in Tandem Bond Formation
Strategies," Tetrahedron 2001, 57, 2065-2084 (handout)
Masse, C. E.; Panek, J. S. "Diastereoselective reactions of chiral allyl- and Provide a mechanism for the indicated transformation
allenylsilanes with activated C-X pi-bonds." Chem. Rev. 1995, 95, 1293-1316.
Ager, D. J. "The Peterson olefination reaction." Org. Reactions 1990, 38, 1-224 O
OLi H
Me3Si
Colvin, E. "Silicon in Organic Synthesis," Butterworths, 1981
O Me OSiMe3
Bois, et al. "SiliconTethered Reactions" Chem. Rev. 1995, 95, 1253-1277. Me
(Handout)
X H
Monday, X
D. A. Evans December 15, 2003 Takeda, Org. Lett, 2000, 2, 903-1905
36-00-Cover Page 12/14/03 8:46 PM
D. A. Evans Bonding Considerations: Carbon vs Silicon Chem 206

Bonding Considerations: Carbon vs Silicon Hypervalent 5-Coordinate Silicon Compounds


Average Bond dissociation eneregies (Kcal/mol) Akiba, "Chemistry of hypervalent Compounds" Wiley-VCH, Chapters 4-5, 1999

C–C C–Si Si–Si C–F Si–F C–O Si–O Penta-coordinate silicates are commonly observed

83 76 53 116 135 86 108


MeSiF4 NEt4 Ph3SiF2 NR4
Average Bond Lengths (Å) C–H Si–H
C–C C–Si C–O Si–O 83 76 Nucleophilic substitution at Silicon
1.54 1.87 1.43 1.66
RO–SiMe3 F RO + F–SiMe3
C C C C better than C Si C Si

σ∗ C–C OSiMe3 O K
σ∗ C–Si
THF
KOCMe3 Me3Si–OCMe3
–20 ° 2h
Si-SP3

C-SP3 C-SP3 C-SP3 Duhamel et al. J. Org. Chem. 1996, 61, 2232
σ C–Si
σ C–C
H3C–CH3 BDE = 83 kcal/mol H3C–SiH3 BDE ~ 76 kcal/mol OSiMe3 O Li

Bond length = 1.534 Å Bond length = 1.87 Å THF


MeLi Me3Si–Me

This trend is even more dramatic with pi-bonds:


Stork et al. JACS. 1968, 90, 4462, 4464
π C–C = 65 kcal/mol π C–Si = 36 kcal/mol π Si–Si = 23 kcal/mol
Thermal Rearrangements One may readily access divalent intermediates
δ– δ+
C Si Me
thermolysis Me Colvin, pp 7-9
Me Si Si CH2 H2C CH2
Group IV Electronegativities (Pauling) Me
Carbon Silicon Germanium Tin Lead Me
Me Me Me
2.55 1.90 2.01 1.96 2.33 Si Si Me
thermolysis Si Me
+2 Oxidation state becones H
increasingly more stable H

36-01-Si intro 12/14/03 8:46 PM


K. Scheidt, D. A. Evans Hypervalent Silicon Ate-Complexes Chem 206

F3C CF3

O
Ph
Si
1.689 Å Ph
F

1.647 Å

CF3

F Inorg. Chem. 1 9 8 4, 23, 1378


Ph J. Am. Chem.Soc. 1987, 109, 476
F Si S(NMe2)3
Ph
F

Cl

2.198 Å
1.668 Å
Me

1.604 Å 2.104 Å
1.597 Å F
Ph
F Si
F
J. Organomet.Chem. 1981, 221, 137. F
Acta Crystallogr. Sect. C 1 9 8 4, 4 0, 476
36-01a-5 coord Si(-) 12/14/03 8:48 PM
D. A. Evans Bonding Considerations: Carbon vs Silicon Chem 206

Carbonyl addition Reactions The prospect of catalysis was investigated


1970 DAE Objective: Develop a reagent that will transform aldehydes ZnI2
into protected cyanohydrins in one step Me3Si CN OSiMe3
1-5 min
C5H11 H reaction was instaneous
+ O
CN – and quantiltative
CN
O SiR3 OSiR3 OSiR3 Me H
LiNR2 1-5 min
+
G R G R G
R H
H Li Principle established that normally inaccessible cyanohydrin derivatives
G = carbanion-stabilizing FG Carbonyl Anion Equivalent may now be accessed
Me Me
Me TMSO CN O
OTMS >95% yield
Me Me
R3Si G Candidates Carbonyl Adducts only 1,2-addition
CN Me (CN– catalysis)
OSiR3 92% yield
>95% yield only 1,2-addition TMSO CN
R3Si CN R CN (ZnI2 catalysis) (ZnI2 catalysis)
H
with Truesdale, Carroll, Chem Commun. 1973, 55; J. Org. Chem.. 1974, 39, 914
Tetrahedron Lett 1973, 4929 (first discussion of Nu catalysis)
OSiR3
R3Si OSO2Ar R SO2Ar
"The Silicon Advantage"
H
From the preceding case, it is clear that ∆HSi is more exothermic than ∆HH
OSiR3
R3Si OPR2 R POR2 O R O–X
H + X CN ∆HSi > ∆HH
R R R CN

Thermal C=O addition of TMSCN is not a clean reaction O O


Nucleophilic Catalysis R R
Me3Si CN R R CN
OSiMe3 OSiMe3
+ O 50 C°
C5H11 H + C N
2-5 hr C4H9
H Me3Si CN
Me H CN
ratio: 65:35
OTMS
C N + R R
CN
36-02-Si-C=O addition 12/14/03 8:50 PM
D. A. Evans Carbonyl Additon Reactions-2 Chem 206

Explain the following observations "The Proton–Silicon Correlation"


O OH
■ Organosilanes undergo carbonyl addition processes in direct analogy
THF/H2O
with their proton counterparts but with an attendant greater exothermicity.
+ C N 1-4 addition
CN
■ Organosilanes undergo a range of thermal rearrangements processes in
O OH direct analogy with their proton counterparts.
O O
X
benzene X
+ TMSCN rt
1-2 addition
C N k(Si) =10+6 K(H)
H H
O TMSO CN
A. J. Ashe III, JACS 1970. 92, 1233

SR O OSiMe3 Me3Si
ZnI2 RS– O O
SiMe3
R1 R2 R1 R2 heat O
R1 R2 O
SR + SR Me Me Me
O C Colvin, pp 37-8
Me3Si SR
with Truesdale, Grimm, Nesbitt, MeMe Me Me Me O
JACS 1975, 97, 3229
JACS 1977, 99, 5009 SiR3
O O
O SiR3 ∆G* 15-22kcal/mol
O OTMSO R N R N
Me3Si – –
CN or F
OEt SiR3 SiR3 Si transfer is intramolecular
R H 1-5 min R OEt
N2
N2 Yoder et al., JACS 1974. 96, 4283
with Truesdale, Grimm
Non-catalyzed processes may also occur if a proper
JOC 1976, 41, 3335
geometry for atom transfer can be achieved Me3Si C N C N SiMe3

TMS ■ Organosilicon hydrides undergo transition metal catalyzed hydrosilylation


O SiR3 OTMS
O processes in direct analogy with normal hydrogenation reactions
O O
O
R H RO P R P R P
Me H–SiR3 Me Me
OR RO OR RO OR H–H
+ SiR3 H
R R R
O R OTMS
RO Rh(I) catalysis Rh(I) catalysis
R O P R
RO RO
P O "Hydrosilylation of C–C Bonds". T. Hayashi In Comprehensive Asymmetric
TMSO
OR with Hurst, Takacs Catalysis, Jacobsen, E. N.; Pfaltz, A.; and Yamamoto, H. Editors; Springer Verlag:
JACS 1978, 100, 3467 Heidelberg, 1999; Vol I, 319-332.

36-03-Si-C=O addition-2 12/15/03 8:09 AM


D. A. Evans Carbonyl [1,2] & [1,3] Sigmatropic Rearrangements Chem 206

[1,3]-Sigmatropic Rearrangements "The Brook Rearrangement(s)"


A. G. Brook Accts. Chem. Research 1974, 7, 77-84
R3Si R3Si
R3Si Ph SiR3
Y C X Y C X Et2NH Ph C O
Ph C OH
R R Ph DMSO H
Y = C; X = O Et2NH Ea ~ 8-11 kcal/mol
Me
Me SiR3
O Complete retention of Si R3Si
110 °C Si
Si Np O stereochemistry was noted. Ph
Np Ph Ph C O C O
Ph
Ph Ea = 28 kcal/mol Ph Ph
Ph H H H
H
N N
Ph A. G. Brook Accts. Chem. Research 1974, 7, 77-84 Et Et
Et Et
O
Me Brook has documented that retention at Silicon & inversion at Carbon occur.
Si
Brook speculates that a hypervalent Si intermediate
Ph might be involved in the rearrangement.
Np
Transformations Involving the Brook Rearrangement
Y = C; X = C Moser, W. H. "The Brook Rearrangement in Tandem Bond Formation
Strategies," Tetrahedron 2001, 57, 2065-2084
Me
Me CH2 Inversion of Si
500 °C Si Acylsilanes
Si Ph stereochemistry was noted.
Np O Cu(I) O R2BH
H Np
R Li SiR3 R R SiR3
Ph H 2C H Ea = 48 kcal/mol
Cl SiR3 [Ox]

H. Kwart et al., JACS 1973. 95, 8678

R3Si
O El(+) O El
Theoretical calculations lead to the conclusion that the concerted [1,3] R
Li
sigmatropic rearrangement with retention of Si-configuration should R SiR3 R R
represent the lower energy pathway.

Yamabe, JACS 1997, 119, 808 Li


O [1,2] Si R3Si Li
O
At the present time these rearrangements are not well studied, R
R
R3Si R R
El(+)

36-04-Si/thermal Rearrangement 12/14/03 8:52 PM


D. A. Evans Transformations Involving the Brook Rearrangements Chem 206

Intramolecular alkylations may be carried out:


Transformations Involving the Brook Rearrangement
Moser, W. H. "The Brook Rearrangement in Tandem Bond Formation O OTMS
Strategies," Tetrahedron 2001, 57, 2065-2084 H3O+ O
R SiR3 R C
R3Si
O El(+) O El R
R
Li Li (CH2)4–I
R SiR3 R R Reich JACS 1980, 102, 1423

Li
O [1,2] Si R3Si Li
OSiMe3
O O
R OLi
R
R3Si R R SiR3
R OH
El(+)
PhS PhS R

s-BuLi Li El(+)
O R O R
O R R3Si SiR3 OSiMe3
R3Si R3Si LiO
El [1,2] Si

PhS O R PhS O R
O R
Li
SiR3 Takeda JACS 1993, 115, 9351; Synlett 1994, 178; SynLett 1997, 255

O
O These reagents are useful CH2(–)
R homoenolate anion equivalents R OSiMe3
O OLi

"Metalated Allylic Ethers as Homoenolate Anion Equivalents". SiR3


Evans, D. A.; Andrews, G. C.; Buckwalter, B. JACS 1974, 96, 5560. Me3Si
OLi
Si–Variant: Still & MacDonald JACS 1974, 96, 5561 PhS R
R

Brook Equilibrium Reich JACS 1980, 102, 1423 (see footnote 8)


[3,3]

Me3Si SiMe3 LiO SiR3 OSiMe3


THF Ph OSiMe3
C OLi Ph C O [1,2] Si
Ph
Ph Li
PhS O PhS O
Me3Si SiMe3 PhS OLi
THF Ph
C OLi H C O R R
Ph R
H Li Tetrahedron 2001, 57, 2065-2084, footnote 16
36-05-Brook Rearrangments-2 12/14/03 8:54 PM
D. A. Evans Transformations Involving the Brook Rearrangements Chem 206

The natural product target: The key reaction nBuLi


Me3Si CH2 SMe Me3Si CH SMe
RO TMEDA Li
OLi
H
Me3Si
O MeS H
Me CHO Me O Me3Si CH SMe C
Li Ph Ph
X Ph Ph
Me carbanion-stabiizing groups facilitate elimination
Me O
H
Elimination could also be effected with dilute acid
Me OSiMe3
Takeda, Org. Lett, 2000, 2, 903-1905 The β-Effect (lecture 32)
H Me3Si OH
10% H2SO4 R
X
The Peterson Olefination Reaction R Me3Si OH
R rt R
Ager, D. J. "The Peterson olefination reaction."
Org. Reactions 1990, 38, 1-224
C N analogy provided by Whitmore et al. JACS 1947, 69, 1551
The key paper: Peterson, J. Org. Chem. 1968, 33, 780-784
Si P
It was Peterson's intent to find a silicon analog to the Wittig rxn.
The reaction concept is outlined below: Mechanistic aspects of Beta-OH Elimination

O Me3Si OM Me3Si H H H
Pr H+
Me3Si CH2M Anti Elimination
R R R H
R Pr OH Pr Pr
The β-Effect (lecture 32)
M KH
Me3Si OM Me3Si O
R OK Pr H
R Me3Si
R H Syn Elimination
H H Pr
R Pr
Pr
Magnesium alkoxides: Stable Hudrlik et al. JACS 1975, 97, 1464 Colvin chapter 12, pp 141
O Me3Si O–MgCl
Me3Si CH2MgCl
these adducts are
R R R quite stable
R R3Si
O OH H H
Na & K alkoxides: Eliminate H R2CuLi H KH
H H
R Me3Si R Pr
Me3Si OH KH Pr R Pr
Me3Si OK (Na)
R rt R reaction is stereospecific
R note site of nu attack. Why?
36-06-Brook Rearrangments-3 12/15/03 8:18 AM
D. A. Evans The Peterson Olefination Reaction Chem 206

■ Simple Examples: Taken from Organic Rxns review Me Miyakolide (+)-1


OH
O CH2 Evans et al. JACS 1999, 121, 6816-6826.
Me
SiMe3
Me3Si CH2MgBr >90%
NaH orTsOH
O O
Me Miyakolide presents an interesting
This reagent is better that H2C=PPh3 for hindered ketones O O olefin geometry challenge
H OH OH
Me Me Me
Me O O

OH H H
H Boeckman, Tet. Lett 1973, 3437 Me OH
H O OMe
O H2C
Me Me
O O
O
Me Me
Xp 1 TMS Xp 1
O OMe OMe OMe
O
LiN(TMS)2 O O
Me3Si OEt >95% Conditions
OEt –78 °C
O 9 O 9
Nozaki, JACS 1974, 96, 1620
OPMB OPMB CO2Me
19-E
t-BuLi OH entry base solvent E:Z
Me3Si MgBr2 SiMe3 SOCl2 1 LDA THF 73 : 27
>90%
2 NaHMDS THF 18 : 82
3 LDA Et2O 66 : 33
Chan, Tet. Lett 1978, 2383
4 LDA PhMe 66 : 33

O OH O OH OH
Me R Me3Si Me3Si
Me OH KH H Me3Si
CMe3 CMe3
O Me RCHO OLi
n-Hexyl n-Hexyl
Me3Si B R n-Hexyl
O Me BF3•OEt2 KH
SiMe3 H+
CMe3
Chan, Chem. Commun 1982, 969 R
O OH
Hudrlik, JACS 1981, 103, 6251
n-Hexyl
CMe3 CMe3
n-Hexyl

36-07-Peterson Olefination 12/14/03 8:57 PM


D. A. Evans The Peterson Olefination Reaction Chem 206

OTMS
TMSO
Bunnelle-Peterson Allylsilane Synthesis
Me O Me3Si O O
N
Me O Me N SiMe3
Me O
O silica gel
OTMS KN(TMS)2 R SiMe3 SiMe3
R O R R
(E):(Z) = 13:1 OH
OTMS 2 Me3Si CH2M
TMSO O O M = Li → M = CeCl2 THPO
Me N
Me SiMe3 SiMe3
O Me N Me Ph
O Me
OTMS 90% 93% Me

t-BuLi (E):(Z) = 1:7


Application to Leucasandrolide: Rychnovsky JACS, 2001, 123, 8420
OTMS Me Me
TMSO O (t-Bu)Me Si O O OH OH
Me 2 N
Me O Me N O OMe O O OMe O
Me
O
OTMS O OH HO
Bell et al. Tetrahedron 1994, 50, 6643
Reaction may be altered significantly with an attendant change in stereoselection O O

Me Me The seco acid


Ireland Enolate Claisen Coupled to Peterson Olefination
Me Me
5.5:1 ratio
The Pivotal Step: Me
R SiMe3 OH Me
LiN(TMS)2
R syn:anti = 96:4 H
O TMSCl CO2Me SiMe3
OH O OH O
CH2N2 SiMe3 O O O
O BF3•OEt2
(See Lecture 15) OBn TBSO
KH BF3•OEt2 base, 78%
OBn OTBS

CO2Me
R R
CO2Me O OEt
SiMe3
TMSCH2MgCl
TMSO TMSO
CeCl3 87%
Sato et al. Chem. Lett 1986, 1553 TBSO TBSO
silica gel

36-08-Peterson Olefination 12/15/03 8:20 AM


J. Leighton, D. A. Evans ROH Protecting Groups Chem 206

Silyl Ethers:
Relative stabilities:
Me Et Me
Me Si OR Et Si OR t-Bu Si OR TES ~102 times more stable to acidic hydrolysis than TMS
TBS ~104 times more stable to acidic hydrolysis than TMS
Me Et Me
trimethylsilyl triethylsilyl tert-butyldimethylsilyl
(TMS) (TES) (TBS or TBDMS)

1% HCl in EtOH
t-Bu t-Bu Me OSiR3 Me OH
Ph i-Pr Si 22.5 °C
O O
t-Bu Si OR i-Pr Si OR SiR3 Half-life
Ph i-Pr R n R’ TBS < 1 min
tert-butyldiphenylsilyl triisopropylsilyl di-tert-butyldimethylsilylene
TIPS 18 min
(TBDPS) (TIPS) (DTBS)
TBDPS 244 min

5% NaOH in EtOH
Me OSiR3 Me OH
90 °C

SiR3 Half-life
Formation: TBS 1h

TIPS 14 h
By far the two most common methods:
TBDPS <4h

R3Si-Cl, imidazole
R OH R OSiR3
DMF, R.T.

2 equiv of imidazole are required relative to R3SiCl.


OSiR3 OH

Corey, E. J.; Venkateswarlu, A. J. Am. Chem. Soc. 1972, 94, 6190. 2 equiv TBAF
THF, 22.5 °C

R3Si-OTf, 2,6-lutidine SiR3 Half-life


R OH R OSiR3
CH2Cl2, 0 °C
TBS 76 min
Corey, E. J. et al., Tetrahedron Lett. 1981, 22, 3455. TIPS 137 min
Cunico, R. F.; Bedell, L. J. Org. Chem. 1980, 45, 4797-4798.

36-09-Si Protecting Grps-1 12/14/03 8:58 PM


J. Leighton, D. A. Evans ROH Protecting Groups-2 Chem 206

Selective Protection: Me

OH OH OH OTBDPS CO2Me
TBDPSCl, imidazole TBSOTf, 2,6-lutidine OTBS
Me Me
DMF, RT Me CH2Cl2, -78 °C OTBS
Me Me Me 95% Me Me Me 71% Me HO
CO2Me Me
Me
OTBS OTBS
OH
Me HO
Me White, J. D. et al.,
O OH OH O OTESOH Me
OTBS J. Am. Chem. Soc. 1989, 111, 790.
TESCl, DMAP
Me Me
MeO CH2Cl2, 0 °C MeO
OTIPS 77% OTIPS

Evans et al.JACS 1999, 121, 7540-7552. HO

CO2Me TMS-NEt2
Me acetone, -45 °C

HO Me OH HO
Me Me CO2Me
OH OH Yankee, E. W.; Bundy, G. L. Me
J. Am. Chem. Soc. 1972, 94, 3651.
H H TMSO Me OH
TBSOTf, 2,6-lutidine
TMS-NEt2 has been reported to selectively protect equatorial
CH2Cl2, -78 °C alcohols in the presence of axial alcohols:
OH 91% OTBS Weisz, I. et al. Acta. Chim. Acad. Sci. Hung. 1968, 58, 189.
OTMS OTMS
Askin, D.; Angst, D.; Danishefsky, S. J. Org. Chem. 1987, 52, 622.
OH OH O O
TBSOTf, 2,6-lutidine
HO HO N O CH2Cl2, 0 °C
SO2Ph SO2Ph 92%
TBSCl, imid. Me Me Me
Bn O
DMF, RT OTBSOH O
80%
HO TBSO Evans, Ng JACS 1993, 115, 11446 N O
Me Me Me
Donaldson, R. E.; Fuchs, P. L. J. Am. Chem. Soc. 1981, 103, 2108.

36-10-Si Protecting Grps-2 12/14/03 9:00 PM


J. Leighton, D. A. Evans ROH Protecting Groups-2 Chem 206

Selective Protection: Selective Deprotection:


OH OH O O
TBSOTf, 2,6-lutidine Me Me OMe
Me
N O CH2Cl2, 0 °C
> 80% TMSO O
Me Me Me Calter, M. A. Ph. D. Thesis,
Me O Harvard University, 1993
Bn O
OTBSOH O
Evans, Ng JACS 1993, 115, 11446 Me OTMS
N O
Me OMe Me
Me Me Me
Bn Me Me OMe

TMSO O
K2CO3
OH OH O O
MeOH Me O
TBSOTf, 2,6-lutidine
Me 100%
N O CH2Cl2, -10 °C OH
83%
Me Me Me
Me OMe Me
Bn O
OH OTBSO
Evans, D. A.; Dart, M. J. Unpublished Me
N O
Me Me Me OTBS
Bn
O
(NCCH2CH2O)2P Me Me Me
O O
Me OH
Evans, Gage, Leighton
Me Me OTBS JACS 1992, 114, 9434
O
O Me TESCl, imidazole
O O O DMAP PivO OMe
Me H Me H H O O CH2Cl2, -78 °C
HO MeO OH
Me OTES O
Me Me
(NCCH2CH2O)2P Me Me Me
Me O O
O HF•pyr, pyridine
O O O 98%
Me H Me H H O O THF
HO MeO OTBS
94% O
Evans, Ratz JACS 1995, 117, 3448
PivO OMe

36-11-Si Protecting Grps-3 12/14/03 9:01 PM


J. Leighton, D. A. Evans ROH Protecting Groups-2 Chem 206

Selective Deprotection: 1,2-Migration:

OTBDPS OH
K2CO3
OH OTBDPS
Me Me Me Me Me Me Me Me MeOH Me
100%
CO2Me Mulzer, J.; Schollhorn, B. Angew. Chem., Int. Ed. Eng. 1990, 29, 431-432.
O O
TBSO O N3 O OTBS
Me Me Me Me 1,3-Migration:
OTBSOH OH OTBS
Bu3Sn KHMDS Bu3Sn
TBAF, THF
THF, -78 °C
RT
OMe Me 94% OMe Me
90%
Calter, M. A. Ph. D. Thesis, Harvard University, 1993.

Me Me Me Me Me Me Me

CO2Me
O O
HO O N3 O OTBS
Me Me Me Me OTBS OH OTBS
H ? Bu3Sn
Nakaba, T.; Fukui, M.; Oishi, T. Tetrahedron Lett. 1988, 29, 2219, 2223. OPiv
O Me OMe Me

MgBr KHMDS
66% Bu3Sn 94%
THF, -78 °C
TESO TESO Me
H H OTBSOPiv OTBSOH
CO2H AcOH:THF:H2O CO2H
4 (8:8:1) 4 Bu3Sn 1. MeOTf,t-Bu N t-Bu Bu3Sn
C5H11 4 h, 20 °C C5H11 2. DIBAl-H, CH2Cl2, -78 °C
OH Me OMe Me
H 76% H 80%
TESO OTBS HO OTBS 4:1

Hart, T. W.; Metcalfe, D. A.; Scheinmann, F.


J. Chem. Soc., Chem. ommun. 1979, 156.
Calter, M. A. Ph. D. Thesis, Harvard University, 1993.

36-12-Si Protecting Grps-4 12/14/03 9:02 PM


J. Leighton, D. A. Evans ROH Protecting Groups Chem 115

Principle Methods for Benzylation of Alcohols: Priciple Methods for Deprotection:


1.
1. NaH, DMF, 0 °C R OBn
R OH
2. BnBr/PMBBr R PMB 1. Hydrogenation
10% Pd on C, H2
NH R OBn R OH
2. EtOH/EtOAc/AcOH
Ar O CCl3 R OBn See Greene, p. 49.
R OH
cat. TfOH R PMB
Ar = Ph: CH2Cl2

Ar = 4-MeO-Ph: Et2O

Ar = Ph: Iversen, T.; Bundle, K. R. 2. Transfer Hydrogenation


J. Chem. Soc., Chem. Commun. 1981, 1240.
10% Pd on C
R OBn R OH
Ar = 4-MeO-Ph: Yonemitsu, O. et al., Tetrahedron Lett. 1988, 29, 4139. Hydrogen Source

3. Hydrogen Source Ref.


1. (Bu3Sn)2O, PhMe, ↑↓
R OH R OBn Cyclohexene Synthesis 1981, 396.
2. BnBr, N-methylimidazole
Cyclohexadiene J. Org. Chem. 1978, 43, 4194.
Cruzado, C.; Bernabe, M.; Martin-Lomas, M. J. Org. Chem. 1989, 54, 465.
HCO2H J. Org. Chem. 1979, 44, 3442.
Review: David, S.; Hanessian, S. Tetrahedron, 1985, 41, 643-663.
i-PrOH Tetrahedron Lett. 1986, 27, 2497
4.
Ag2O, BnBr
R OH R OBn
DMF 3. Lewis Acids

Van Hijfte, L.; Little, R. D. J. Org. Chem. 1985, 50, 3940.


R OBn R OH

Via Benzylidene Acetal:


Ar H Reagents Ref.
O O DIBAl-H OH O Ar BF3•OEt2, EtSH Tetrahedron Lett. 1989, 30, 5713.
CH2Cl2, 0 °C J. Am. Chem. Soc. 1989, 111, 1923.
R1 R2 R1 R2
1. BCl3, -78 °C to 0 °C.
n n 2. MeOH, -78 °C.
Takano, S. et al., Synthesis 1986, 811-817.
TMSBr, C6H5SMe Chem. Pharm. Bull. 1987, 35, 3880.
36A-13 Benzylic 12/7/01 8:58 AM
J. Leighton, D. A. Evans ROH Protecting Groups Chem 115
PMB Deprotection: DDQ is incompatible with: N

C Me
OMe
DDQ R
ROH + ArCHO R
O 20:1 CH2Cl2:H2O
R
Me Me
-
-1 e
styryl extended conjugated polyenes

OMe OMe
+
O • O
R O R
Cl CN OPMB O
H OH
DDQ = DDQ
-H+ Cl CN
H 2O R R’ 20:1 CH2Cl2:H2O R R’
O
OMe OMe

O O
R • R
+
H Selective Benzylation:
OMe

O -1 e-
Br Br
R • Me Me
H
NaH, BnBr
Ar DMF, -70 °C
H 97% H
* OH OH
OH OPMB O O OH OBn
DDQ, CH2Cl2
3 Å mol sieves Fukuzawa, A. et al. Tetrahedron Lett. 1987, 28, 4303.
R R
n n

Yonemitsu, O. et al., Tetrahedron 1986, 42, 3021.

O O
OH 1. (Bu3Sn)2O, PhMe, ↑↓ OH
Other Oxidants: NBS, Br2, CAN ((NH4)2Ce(NO3)6).
O O
Acta Chem. Scand. Ser. B, 1984, B38, 419. HO OH 2. BnBr, N-methylimidazole BnO OBn
J. Chem. Soc., Perkin Trans. I, 1984, 2371. 92%
Cruzado, C.; Bernabe, M.; Martin-Lomas, M. J. Org. Chem. 1989, 54, 465.
36A-14 Benzyl Protect 12/7/01 8:18 AM
J. Leighton, D. A. Evans ROH Protecting Groups Chem 115

Me Me CO2H
O OH O OH O OH 1
5 OH 17
Me 33 H 1 8
O I 11
H O O
OH H Me Me Me Me Me Me Me
O 25
H 8
OH
Me Me
H
O CO2R'
11
Me OH OR O O OH 1
20 17
HO 14 10 4
Et Me X H
8
Rutamycin B Me Me Me Me Me Me
Me
(Streptomyces aureofaciens) 17
C9-C17 Subunit C1-C8 Subunit
Ng, H. P. Ph. D. Thesis,
Harvard University, 1993

O
14
Me Ph H

Me 33 H Me
O O O O OH O O
H OH
OH H 10 (c-hex)2BCl, EtNMe2 14 10
O 25 N O Ph Xq
H Spiroketal Fragment Et2O, 0 °C
Me Me Me Me Me Me
H (Xq) 84%
Bn

Et 20 B(OH)2 NaBH(OAc)3, AcOH


81%

CO2H OH OH O
OH O OH O OH 1
17 14 10
8 Ph Xq
I 11
Me Me Me
Me Me Me Me Me Me

Polypropionate Fragment

36A-15 Rutamycin 12/7/01 8:18 AM


J. Leighton, D. A. Evans ROH Protecting Groups Chem 115
HO O
OH OH O 1

14 10
Ph Xq Me
5 OTBS
Me Me Me Me 33 H Me
O
H OTES O
TBSO H Me
TBSO PMBO O HF•pyr O 25
H 8
pyridine, THF 99% OTBS
? 14 10 H
Me Me
Ph H
O
Me Me Me Me
11
20
HO O TBSO
Selective silylation was unsuccessful. 1
Et Me

Me Me
17
5 OTBS
Me 33 H Me
O
OH OH O OH OH OH H OH O
TBSO H Me
14 10 LiBH4, H2O 14 10 O 25
Ph Xq Ph H 8
OTBS
Et2O, 0 °C Me Me
Me Me Me Me Me Me H
94% O
Me
OMe 11
20
CSA, DMF TBSO
OMe Et Me
91%
PMP MeO PMP Me
17
TBSO O O OH O O 1. Cl3C6H2COCl
14 10 TBSOTf, 2,6-lut. 14 10 DMAP, Et3N, benzene
Ph CH2Cl2, 0 °C Ph
2. (aq.) HF, CH3CN, H2O Me Me
Me Me Me 100% Me Me Me
O OH
Me H
O
DIBAl-H H O O
100% OH H Me
CH2Cl2, 0 °C O
H OH
Me Me
H
84% O
TBSO PMBO OH TBSO PMBO O Me
14 10
Swern Ox. 14 10 HO
Ph 97% Ph H Et Me
Me Me Me Me Me Me
Rutamycin B Me
36A-16 Rutamycin 12/7/01 8:18 AM
J. Leighton, D. A. Evans ROH Protecting Groups Chem 115

OH O Me
37
O
MeO 33 N 30 TBSO O
H
Me2N OH N
26 MeO OMe No Reaction
25 Me2N TBSO
(+) Calyculin A
O
(aq.) HF, CH3CN, H2O
Me Me Me
N O 24 h
(HO)2 P O 17 TESO O OH O
C1 Me Me Me
21 OH
13 O MeO OMe MeO OMe
8
Me2N TESO Me2N OH
Me Me OH OH OMe
Evans, D. A.; Gage, J. R.; Leighton, J. L.
J. Am. Chem. Soc. 1992, 114, 9434-9453. After protonation of the amine, coulombic repulsion insulates
against formation of another cationic site in the vicinity.

(aq.) HF, CH3CN, H2O


70% OTBS
92 h

Me Me Me
N O
PMBO 17
C1 Me Me Me
21 OTBS
13 O
8
TESO O Me
Me Me OTBS OTBS OMe
37
O
MeO 33 N 30
H DDQ
Me2N OTES N
26
X
20:1 CH2Cl2:H2O
25
O
OTBS
(PMBO)2P Me Me Me
N O
O 17
C1 Me Me Me
21 OTBS Me Me Me DDQ
13 O O
8 RO 17 20:1 CH2Cl2:H2O
Me Me R = PMB
Me Me OTBS OTBS OMe 21 OTBS
PivO 13 O
R=H
94%
OTBS OTBS OMe
36A-17 Calyculin 12/7/01 8:18 AM
J. Leighton, D. A. Evans ROH Protecting Groups Chem 115
Me

Me
O
O
H H OH
O
Me O OH Me
H OMe N
O
OH H O O Me
H H
H Me Me Rapamycin H
O H OH
Me
O
HO O
OH OH H
H
Cytovaricin H Me OMe
MeO O OH
Me O Me
OH OH Me
H O OH
H
Me OMe Me Me
Evans, D. A.; Kaldor, S. W.; Jones, T. K.; Clardy, J.; Stout, T. J.
J. Am. Chem. Soc. 1990, 112, 7001-7031. Romo, D.; Meyer, S. D.;
Johnson, D. D.; Schreiber, S. L.
J. Am. Chem. Soc. 1993, 115, 7906-7907.
1. Pd(Ph3P)4, HCO2NH4, THF
Et 2. Dess-Martin Periodinane 30% overall
HF•pyr, pyridine
74% DEIPS = Si i-Pr
THF, RT 3. HF•pyr, pyridine, THF O
Et
Aloc =
O
Me

OAloc
Me
OTBS
H
O Me
Me O OTES
H OMe O N
DEIPSO H O O Me
H H
Me H Me Me
H O O
TBSO t-Bu
TESO Si O
O H
O t-Bu H Me OMe
H OAloc ODEIPS
MeO
Me O Me Me
TESO OTES OAloc OTIPS
H
H Me OMe Me Me
36A-18 Cytovaricin 12/7/01 8:18 AM
J. Leighton, D. A. Evans ROH Protecting Groups Chem 115
O
OH
O O OH OH
1. DDQ, t-BuOH-CH2Cl2; Ac2O, DMAP, pyr.
HO OH
OH
H 2N
2. aq. HClO4, THF, 8 days OH
O OH
OH
3. LiOH, H2O/MeOH/THF, RT, 20 h
Me
4. TBAF, THF/DMF, RT, 90 h HO OH
OH OH
5. AcOH, H2O, RT, 36 h OH
35% OH
OH
HO OH O
O Me OH Me HO OH
~97.5% per protecting group OH OH
HO O
OH OH OH
HO HO
Me
OH
OH O
Me O
O Me OH HO
Me OH
OH
O
OH
O HO OH OH
O HO OH
O O Me TBSO OTBS
OH
Me TBSO OTBS
O
H N
OTBS Palytoxin Carboxylic Acid
O O TBSO
OTBS Kishi, Y. et al., J. Am. Chem. Soc. 1989, 111, 7525, 7530.
Me3Si(CH2)2O
Me
TBSO OTBS
OTBS OAc
OTBS
OTBS
OPMB
PMBO OPMB O
O Me OTBSMe PMBO OTBS

OPMB OTBS
MeO O
OAc OAc OPMB
PMBO TBSO
Me
OTBS
OPMB O
Me O
O Me OPMB TBSO
Me OTBS
OMe
O
OTBS
BzO OAc OAc
BzO OBz
OBz
36A-19 Palytoxin 12/7/01 8:19 AM
Shair Common Terpenes and Chiral Pool Molecules Chem 215

CH3 CH3 CH3 CH3 CH3


H3 C H3C H3C H3C H3C
OH
H
CH3 H3C H3C CH3
O OH
(+)-c-pinene (–)-d-pinene (1R)-(+)-camphor (+)-borneol (S)-cis-verbenol

H3 C OH H3 C OH H3C OH

CH3 CH3 CH3 CH3 CH3 CH3 CH3 CH3 CH3


geraniol farnesol geranylgeraniol

CH3 CH3 CH3 CH3


O O OH
O
OH O O
OH O
CH3 H3C CH3 CH3 H3C CH3

(+)-carvone (+)-menthol (R)-(+)-limonene (+)-pulegone 1,2:5,6-di-O-cyclohexylidene-


D-mannitol

O O CH3 R
R
H3 C HO OCH3 O
OH OH H3 N OH
H2N
OH OH O O

(L)-(+)-lactic acid (S)-(+)-mandelic acid Roche ester amino acids amino alcohols
And a Few Others Useful Ones...

O OH
O
O O CH3
O
H3C HO OCH3
OH OH OH O
OH OH O

1,2:5,6-di-O-cyclohexylidene-
(L)-(+)-lactic acid (S)-(+)-mandelic acid Roche ester
D-mannitol

H3 C OH H3C OH H3C OH

CH3 CH3 CH3 CH3 CH3 CH3 CH3 CH3 CH3

geraniol farnesol geranylgeraniol


Inexpensive Chiral Starting Materials and Resolving Agents: Amino Acids
NH
Me COOH
COOH H2N COOH H2N COOH HO COOH
H2N N
NH2 H O NH2
NH2 O NH2 O NH2
L-alanine L-arginine L-aspartic acid
L-asparagine D-asparagine

O Me
COOH H2N COOH
HS COOH COOH COOH
HO
NH2 NH2 L-isoleucine NH NH2 L-lysine NH2
2
L-cysteine L-glutamic acid L-leucine

COOH
S COOH COOH COOH COOH
Me H2N
NH2
L-methionine NH2 L-ornithine NH2 NH2 NH2
L-phenylalanine
L-phenylglycine D-phenylglycine

COOH COOH
COOH HS COOH
N O COOH
H N NH2
H NH2
HN NH2 HO
L-proline L-serine
L-pyroglutamic acid
L-tryptophan L-tyrosine

COOH COOH
L-valine is very expensive!
NH2 NH2
t-leucine

Lecture
© André B. Charette, Université de Montréal 1.10
Inexpensive Chiral Starting Materials and Resolving Agents:
Hydroxyacids

O Me
Me COOH Me COOH COOH
HOOC
O
OH OH OH n
L-lactic acid D-lactic acid (S)-Malic acid (Poly)-3-(R)-hydroxybutyrate

OH OH OH
OH
COOH COOH COOH
COOH Me
HOOC HOOC Me
OH NH2 NH2
OH
L-Tartaric acid D-Tartaric acid D-Threonine L-Threonine

Lecture
© André B. Charette, Université de Montréal 1.11
Inexpensive Chiral Starting Materials and Resolving Agents: Terpenes
H3C CH3 H3C CH3
CH3 H3C CH3 H3C CH3
H3C
CH3
CH3 CO2H
Br CO2H
OH H3C O HO3S O
CH3 H3C H3C
O
l-(-)-Borneol endo-3-Bromo-d (+)-Camphene D-(+)-Camphoric d-10-Camphorsulfonic
-camphor d-(+)-Camphor
acid acid
CH3 CH3 CH3
CH3 CH3 (+)-Isomenthol
O CH3
CHO O
H3C
(+)-Citronellal H3C OH
O
H3C (+)-Carene l-(-)-Carvone l-(+)-Fenchone
d-(-)-Fenchone
CH3 H3C CH2 H3C CH3
CH3 CH3
CH3 CH3 CH3
CH2OH

OH OH
d-(+)-Limonene l-(-)-Limonene O H3C
H3C CH3 H3C CH3
H3C CH2 H3C CH2 H3C CH3 H3C
d-(+)-Menthol l-(-)-Menthol (-)-Nopol
CH3
CH3 CH3 l-(-)-Menthone
CH3
H3C
CH3 H3C H3C H3C O
(-)- -Phellandrene H3C (-)- -Pinene (R)-(+)-Pulegone
H3C (+)- -Pinene H3C (-)- -Pinene
H3C CH3

Lecture
© André B. Charette, Université de Montréal 1.12
Inexpensive Chiral Starting Materials and Resolving Agents: Carbohydrates
OH OH OH
OH HO
O O HO HO O
HO O O O HO
HO CH2OH HO HO
HO HO HO
OH
OH OH OH OH OH OH O NH2 OH
OH OH
D-Arabinose L-Arabinose D-Galactose D-Gluconic acid, -lactone D-Glucosamine
D-Fructose
O OH
H3C H
HO HO O
OH O HO O
O O HO O H3C
HO HO O O
HO HO HO
CH3 CCl3
OH OH OH OH OH HO O
D-Mannose O CH3 -Chloralose
D-Glucose D-Xylose HO
Diacetone-D-glucose
HO
O OH O H OH HO H
H O O OH O
COOH HO O HO O
H2N HO2C
O O OH
L-Glutamine NH2
D-Glucurone HO OH HO OH HO OH
D-Isoascorbic acid D-Saccharic 1,4-lactone
HO H D-Ribose
O OH
HO O
HO O OH
OH OH OH
OH HO
HO OH OH HO OH O
O O
L-Ascorbic acid
HO OH HO HO
HO HO
HO OH HO OH OH
COOH OH OH
HO HO OH HO
OH OH OH OH HO
OH D-Glucoheptonic acid
O OH OH OH OH OH
D-Quinic acid -lactone
D-Glucoheptonic acid D-Gluconic acid D-Mannitol D-Sorbitol L-Sorbose

Lecture
© André B. Charette, Université de Montréal 1.13
Inexpensive Chiral Starting Materials and Resolving Agents: Alkaloids

H N N H
N N
HO HO
H H
HO HO
MeO
N N

N N
MeO
(+)-Cinchonine Quinidine Quinine
(-)-Cinchonidine

N H
Hydroquinidine N
(dihydroquinidine) HO
N H
HO
CH3 MeO
N Hydroquinine N
l-(-)-Nicotine (dihydroquinine)
N
MeO

OH OH OH OH
H H H H
N N N N
CH3 H CH3 CH3
CH3 CH3 CH3 CH3
D-(+)-Ephedrine (+)-Norephedrine L-(-)-Pseudoephedrine
D-(+)-Pseudoephedrine

Lecture
© André B. Charette, Université de Montréal 1.14
Inexpensive Chiral Starting Materials and Resolving Agents:
Other Ligands

OH OH O O

NH2 NH2

L-(+)-2-Amino-1-phenyl-1,3-propanediol (4S,5S)-(+)-5-Amino-2,2-dimethyl-4-phenyl-1,3-dioxane

H3C OH H3C OH
NH2 CH3

L-(-)-2-Amino-1-butanol (-)-2-Methyl-1-butanol

Lecture
© André B. Charette, Université de Montréal 1.15
And a Few Others Useful Ones...

O OH
O
O O CH3
O
H3C HO OCH3
OH OH OH O
OH OH O

1,2:5,6-di-O-cyclohexylidene-
(L)-(+)-lactic acid (S)-(+)-mandelic acid Roche ester
D-mannitol

H3C OH H3C OH H3C OH

CH3 CH3 CH3 CH3 CH3 CH3 CH3 CH3 CH3

geraniol farnesol geranylgeraniol


D. A. Evans An Introduction to Frontier Molecular Orbital Theory-1 Chem 206

http://www.courses.fas.harvard.edu/colgsas/1063 ! Problems of the Day


The molecule illustrated below can react through either Path A or Path B to
form salt 1 or salt 2. In both instances the carbonyl oxygen functions as the
http://evans.harvard.edu/problems/ nucleophile in an intramolecular alkylation. What is the preferred reaction
path for the transformation in question?
Chemistry 206 O
Path A +
O 1 Br
N O
Advanced Organic Chemistry Br Br H Br

N O
H O
Path B
Lecture Number 1 2
+ Br
N O
H Br –
Introduction to FMO Theory
This is a "thought" question posed to me by Prof. Duilo Arigoni at the ETH in
Zuerich some years ago
! General Bonding Considerations

! The H2 Molecule Revisited (Again!) FIRST HOUR EXAM, 2005. The oxidation of acetals by electrophilic ozone
is known to be sensitive to structure. Two striking examples of different
! Donor & Acceptor Properties of Bonding & Antibonding States reactivity are detailed in the questions below. Using clear three-dimensional
drawings provide a rationale for the observation that rigid glycoside A
! Hyperconjugation readily undergoes oxidation but glycoside B does not. Be sure to indicate
all relevant stereoelectronic interactions.
H +
_ O
H
O O
! Reading Assignment for week:
O O O O
Kirby, Stereoelectronic Effects H O O
O H
A
Carey & Sundberg: Part A; Chapter 1
H +
_ O H
O O
Fukui,Acc. Chem. Res. 1971, 4, 57. (pdf)
Alabugin & Zeidan, JACS 2002, 124, 3175 (pdf)
O O O O
Robertson, Org. Letters 2005, 7, 5007 (pdf) H O O
O H
B
Deslongchamps, Can. J. Chem. 1974, 3651-3664.
Monday,
D. A. Evans September 18, 2006
D. A. Evans An Introduction to Frontier Molecular Orbital Theory-1 Chem 206

Universal Effects Governing Chemical Reactions ! Stereoelectronic Effects


There are three: Geometrical constraints placed upon ground and transition states
! Steric Effects by orbital overlap considerations.

Nonbonding interactions (Van der Waals repulsion) between Fukui Postulate for reactions:
substituents within a molecule or between reacting molecules
"During the course of chemical reactions, the interaction of
Me Me the highest filled (HOMO) and lowest unfilled (antibonding)
SN2
C Br Br: – molecular orbital (LUMO) in reacting species is very important
Nu: Nu C R
R
R R to the stabilization of the transition structure."

RO RO H ! General Reaction Types


O O major
H
H Radical Reactions (~10%): A• + B• A B
Me
Me2CuLi RO Me
O
minor Polar Reactions (~90%): A(:) + B(+) A B
H
H
Lewis Acid
! Electronic Effects (Inductive Effects): Lewis Base

The effect of bond and through-space polarization by FMO concepts extend the donor-acceptor paradigm to
heteroatom substituents on reaction rates and selectivities non-obvious families of reactions
Inductive Effects: Through-bond polarization ! Examples to consider
Field Effects: Through-space polarization
H2 + 2 Li(0) 2 LiH
Me +
SN1 R
CH3–I + Mg(0) CH3–MgBr
C Br C Me + Br:–
R R
R "Organic chemists are generally unaware of the impact of
electronic effects on the stereochemical outcome of reactions."
rate decreases as R becomes more electronegative "The distinction between electronic and stereoelectronic effects is
not clear-cut."
D. A. Evans Steric Versus Electronic Effects; A time to be careful!! Chem 206

! Steric Versus electronic Effects: Some Case Studies


When steric and electronic (stereoelectronic) effects O OSiR3
lead to differing stereochemical consequences R3SiO
TiCl4 diastereoselection
Woerpel etal. JACS 1999, 121, 12208. >94:6
EtO Nu
O O OSiR3 OSiR3
OAc SnBr4 O R3Si

O O
Me Me H
Me
AlCl3 diastereoselection
SiMe3 stereoselection 99:1 93:7

stereoselection >95:5 H
OSiR3 OSiR3
O OAc O O
SnBr4
Danishefsky et al JOC 1991, 56, 387
BnO BnO BnO

O
EtO2C O AcO
(R)2CuLi diastereoselection H
AcO H
O 8:1 N
EtO2C Bu Ph N
N N
OTBS O
N only diastereomer
O O
EtO2C N
OTBS Bu3Al Ph
only diastereomer O O
OAc
H OAc
Bu
R3 OTBS Ph N H OAc
OAc H
O Al O
O H O
Yakura's 60-94%
EtO rationalization:
N
Ph
O
R O TBS
Al Yakura et al
R Tetrahedron 2000, 56, 7715 Mehta et al, Acc Chem. Res. 2000, 33, 278-286
R
TRANSITION STATE
HYPERCONJUGATION

LUMO

DR = 95:5

HOMO
NU CSP2
D. A. Evans The H2 Molecular Orbitals & Antibonds Chem 206

Linear Combination of Atomic Orbitals (LCAO): Orbital Coefficients


The H2 Molecule (again!!)
! Rule Two:
Let's combine two hydrogen atoms to form the hydrogen molecule. Each MO is constructed by taking a linear combination of the
Mathematically, linear combinations of the 2 atomic 1s states create individual atomic orbitals (AO):
two new orbitals, one is bonding, and one antibonding:
Bonding MO " = C1!1 + C2!2
! Rule one: A linear combination of n atomic states will create n MOs.
Antibonding MO "# = C*1!1 – C*2!2
!" (antibonding)
The coefficients, C1 and C2, represent the contribution of each AO.

#E
! Rule Three: (C1)2 + (C2)2 = 1
Energy

H 1s 1s H The squares of the C-values are a measure of the electron


population in neighborhood of atoms in question
$1 $2
#E ! Rule Four: bonding(C1)2 + antibonding(C*1)2= 1

! (bonding) In LCAO method, both wave functions must each contribute


one net orbital

Consider the pi–bond of a C=O function: In the ground state pi-C–O


Let's now add the two electrons to the new MO, one from each H atom: is polarized toward Oxygen. Note (Rule 4) that the antibonding MO
is polarized in the opposite direction.
!" (antibonding)

#E1 C O !" (antibonding)


Energy

H 1s 1s H Energy

$1 $2
#E2 C

! (bonding) O

Note that #E1 is greater than #E2. Why? ! (bonding)


C O
D. A. Evans Bonding Generalizations Chem 206

! Bond strengths (Bond dissociation energies) are composed of a ! Orbital orientation strongly affects the strength of the resulting bond.
covalent contribution (! Ecov) and an ionic contribution (! Eionic). For " Bonds:
A Better A B
B
Bond Energy (BDE) = ! Ecovalent + ! Eionic than

When one compares bond strengths between C–C and C–X, where X
is some other element such as O, N, F, Si, or S, keep in mind that
covalent and ionic contributions vary independently. Hence, the For ! Bonds:
A B
Better A B
mapping of trends is not a trivial exercise. than

Useful generalizations on covalent bonding This is a simple notion with very important consequences. It surfaces
in the delocalized bonding which occurs in the competing anti
! Overlap between orbitals of comparable energy is more effective
(favored) syn (disfavored) E2 elimination reactions. Review this
than overlap between orbitals of differing energy.
situation.
For example, consider elements in Group IV, Carbon and Silicon.
We know that C-C bonds are considerably stronger by Ca. 20 kcal ! Anti orientation of filled and unfilled orbitals leads to better overlap.
mol-1 than C-Si bonds.
This is a corrollary to the preceding generalization.
There are two common situations.
C C C C better than C Si C Si
Case-1: Anti Nonbonding electron pair & C–X bond
!" C–C
!" C–Si

X X X
!* C–X lone pair !* C–X
Si-SP3 LUMO HOMO LUMO
A C A C Better A C
C-SP3 C-SP3 C-SP3 than
•• lone pair
! C–Si
HOMO
! C–C
H3C–CH3 BDE = 88 kcal/mol H3C–SiH3 BDE ~ 70 kcal/mol
Bond length = 1.534 Å Bond length = 1.87 Å Case-2: Two anti sigma bonds

This trend is even more dramatic with pi-bonds: Y


X X
X !* C–X Better ! C–Y !* C–X
! C–C = 65 kcal/mol ! C–Si = 36 kcal/mol ! Si–Si = 23 kcal/mol LUMO HOMO LUMO
than
! Weak bonds will have corresponding low-lying antibonds. A C C C C C
! C–Y
Formation of a weak bond will lead to a corresponding low-lying antibonding Y HOMO
orbital. Such structures are reactive as both nucleophiles & electrophiles Y
D. A. Evans Donor-Acceptor Properties of Bonding and Antibonding States Chem 206

Donor Acceptor Properties of C-C & C-O Bonds Hierarchy of Donor & Acceptor States
Consider the energy level diagrams for both bonding & antibonding Following trends are made on the basis of comparing the bonding and
orbitals for C–C and C–O bonds. antibonding states for the molecule CH3–X where X = C, N, O, F, & H.
!* C-C
!-bonding States: (C–X)
!* C-O
CH3–CH3 CH3–H

C-SP3 C-SP3 CH3–NH2


very close!!
O-SP3 CH3–OH

decreasing !-donor capacity CH3–F

! C-C poorest donor


! C-O
! The greater electronegativity of oxygen lowers both the bonding
!-anti-bonding States: (C–X)
& antibonding C-O states. Hence:
For the latest views, please read
! ! C–C is a better donor orbital than ! C–O CH3–H Alabugin & Zeidan, JACS 2002, 124, 3175 (pdf)
! !"C–O is a better acceptor orbital than !"C–C CH3–CH3
CH3–NH2
Donor Acceptor Properties of CSP3-CSP3 & CSP3-CSP2 Bonds CH3–OH

!* C–C CH3–F
!* C–C better acceptor Increasing !"-acceptor capacity best acceptor

The following are trends for the energy levels of nonbonding states of
C-SP3 C-SP3 several common molecules. Trend was established by photoelectron
spectroscopy.
C-SP2
Nonbonding States

! C–C
H3P:
better donor ! C–C H2S:
! The greater electronegativity of CSP2 lowers both the bonding & H3N:
antibonding C–C states. Hence: H2O:
HCl:
! ! CSP3-CSP3 is a better donor orbital than ! CSP3-CSP2 decreasing donor capacity poorest donor
! !"CSP3-CSP2 is a better acceptor orbital than !"CSP3-CSP3
Radial Electron Density of S-States

1S 2S 3S
Electron Probability

Distance from Nucleus


Radial Electron Density of S- & P-States

View of Nucleus
2S
Electron Probability

+ 2S-State

2P + 2P-State

Distance from Nucleus


D. A. Evans Hybridization vs Electronegativity Chem 206

Electrons in 2S states "see" a greater effective nuclear charge There is a linear relationship between %S character &
than electrons in 2P states. Pauling electronegativity

This becomes apparent when the radial probability functions for S 5

and P-states are examined: The radial probability functions for the NSP
hydrogen atom S & P states are shown below. 4.5

100 % 100 %

Pauling Electronegativity
4
Radial Probability

1 S Orbital

Radial Probability
NSP2
NSP3
3.5
C
SP

2 S Orbital 2 S Orbital
CSP2
2.5
CSP3
2 P Orbital

2
20 25 30 35 40 45 50 55
Å Å % S-Character

There is a direct relationship between %S character &


3 S Orbital 3 P Orbital hydrocarbon acidity

60

CH4 (56)
55

S-states have greater radial penetration due to the nodal properties of the wave
function. Electrons in S-states "see" a higher nuclear charge. 50

Pka of Carbon Acid


45
Above observation correctly implies that the stability of nonbonding electron
C6H6 (44)
pairs is directly proportional to the % of S-character in the doubly occupied orbital
40

Least stable Most stable


35
CSP3 CSP2 CSP PhCC-H (29)
30

The above trend indicates that the greater the % of S-character


25
at a given atom, the greater the electronegativity of that atom. 20 25 30 35 40 45 50 55
% S-Character
D. A. Evans Hyperconjugation: Carbocation Stabilization Chem 206

! The interaction of a vicinal bonding orbital with a p-orbital is referred Physical Evidence for Hyperconjugation
to as hyperconjugation.
This is a traditional vehicle for using valence bond to denote charge ■ Bonds participating in the hyperconjugative interaction, e.g. C–R,
delocalization. will be lengthened while the C(+)–C bond will be shortened.
R + R
H H First X-ray Structure of an Aliphatic Carbocation
H C C H C C
H H
H H

The graphic illustrates the fact that the C-R bonding electrons can
"delocalize" to stabilize the electron deficient carbocationic center. + 1.431 Å [F5Sb–F–SbF5]–

Note that the general rules of drawing resonance structures still hold: +
the positions of all atoms must not be changed. C

1.608 Å Me
100.6 ° Me
Stereoelectronic Requirement for Hyperconjugation:
Me
Syn-planar orientation between interacting orbitals

The Molecular Orbital Description


!" C–R !" C–R T. Laube, Angew. Chem. Int. Ed. 1986, 25, 349

+ + The Adamantane Reference


H H (MM-2) 1.528 Å
C C
H H
H

110 °
! C–R ! C–R Me 1.530 Å
Me

Me
! Take a linear combination of ! C–R and CSP2 p-orbital:

"The new occupied bonding orbital is lower in energy. When you


stabilize the electrons is a system you stabilize the system itself."
D. A. Evans Hyperconjugation, The Anomeric Effect, and More Chem 206
Useful LIterature Reviews
http://www.courses.fas.harvard.edu/colgsas/1063 Kirby, A. J. (1982). The Anomeric Effect and Related Stereoelectronic Effects at
Oxygen. New York, Springer Verlag.
Box, V. G. S. (1990). “The role of lone pair interactions in the chemistry of the
Chemistry 206 monosaccharides. The anomeric effect.” Heterocycles 31: 1157.

Box, V. G. S. (1998). “The anomeric effect of monosaccharides and their


derivatives. Insights from the new QVBMM molecular mechanics force field.”
Advanced Organic Chemistry Heterocycles 48(11): 2389-2417.

Graczyk, P. P. and M. Mikolajczyk (1994). “Anomeric effect: origin and


Lecture Number 2 consequences.” Top. Stereochem. 21: 159-349.
Juaristi, E. and G. Cuevas (1992). “Recent studies on the anomeric effect.”
Stereoelectronic Effects-2 Tetrahedron 48: 5019 (PDF)

Carey & Sundberg: Part A; Chapter 3 pp 151-156


! "Positive" and "Negative" Hyperconjugation
Database problem 307
! Anomeric and Related Effects
Natural bond order (NBO) analysis is a powerful computational tool that allows
! Peracid & Dioxirane Epoxidation (Stereoelectronics) one to quantitatively estimate the energy of hyperconjugative effects (Alabugin
& Zeiden, JACS 2002, 124, 3175). The stabilizing interactions for the H1 and
H4 equatorial hydrogens for 1,3-dioxane and 1,3-dithiane are provided below.
It is interesting that the two indicated hyperconjugative interactions in dioxane
are approximately the same (4.5 vs 4.2) but that the same interactions in
dithiane are quite different (6.5 vs 1.8). In fact, the authors refer to C–S bonds
Kirby, Stereoelectronic Effects Chapters 1-5 as "one directional" acceptor orbitals.
numbers refer to interaction
Carey & Sundberg: Part A; Chapter 1, Chapter 3 +4.2 +1.8 energies in kcal/mol at the
Fukui,Acc. Chem. Res. 1971, 4, 57. (pdf) B3LYP/6-31+G*** level
H4 O H1 S H1
H4
Alabugin & Zeidan, JACS 2002, 124, 3175 (pdf) O S
+4.5 +6.5
!C–H " !*C–O !C–H " !*C–S

There is one quite reasonable qualitative argument that can rationalize the
D. A. Evans Wednesday, "one directional" character of !*C–S. Please provide your argument in the
September 20, 2006 space provided below.
D. A. Evans Hyperconjugation: Carbocation Stabilization Chem 206
! The interaction of a vicinal bonding orbital with a p-orbital is referred
to as hyperconjugation. Physical Evidence for Hyperconjugation
This is a traditional vehicle for using valence bond to denote charge
delocalization. ■ Bonds participating in the hyperconjugative interaction, e.g. C–R,
will be lengthened while the C(+)–C bond will be shortened.
R R
H H
H C C H C C
H H First X-ray Structure of an Aliphatic Carbocation
H H

The graphic illustrates the fact that the C-R bonding electrons can
"delocalize" to stabilize the electron deficient carbocationic center.
+ 1.431 Å [F5Sb–F–SbF5]–
Note that the general rules of drawing resonance structures still hold:
the positions of all atoms must not be changed. +
C

Stereoelectronic Requirement for Hyperconjugation: 100.6 ° 1.608 Å Me


Me
Syn-planar orientation between interacting orbitals
Me

The Molecular Orbital Description


!" C–R !" C–R
T. Laube, Angew. Chem. Int. Ed. 1986, 25, 349

+ +
H H The Adamantane Reference
C C (MM-2) 1.528 Å
H H

! C–R ! C–R 110 °


Me 1.530 Å
Me

! Take a linear combination of ! C–R and CSP2 p-orbital: Me

"The new occupied bonding orbital is lower in energy. When you


stabilize the electrons is a system you stabilize the system itself."
D. A. Evans "Negative" Hyperconjugation Chem 206

antibonding !" C–R


! Delocalization of nonbonding electron pairs into vicinal antibonding Syn Orientation
orbitals is also possible
R:– R
R ""
""
filled
R "" R ""
C X H C X+ H C X hybrid orbital
H
H C X H H C X H H H H
H H H H

This decloalization is referred to as "Negative" hyperconjugation antibonding !" C–R


Anti Orientation

R R:– R
C X+ filled
H C X H
H C X hybrid orbital
Since nonbonding electrons prefer hybrid orbitals rather that P H
""
H
H ""

orbitals, this orbital can adopt either a syn or anti relationship


to the vicinal C–R bond.
! Overlap between two orbitals is better in the anti orientation as
The Molecular Orbital Description stated in "Bonding Generalizations" handout.

!" C–R
The Expected Structural Perturbations
Change in Structure Spectroscopic Probe
X Nonbonding e– pair
!! ! Shorter C–X bond X-ray crystallography

! Longer C–R bond X-ray crystallography

! Stronger C–X bond Infrared Spectroscopy


As the antibonding C–R orbital
! C–R decreases in energy, the magnitude ! Weaker C–R bond Infrared Spectroscopy
of this interaction will increase
! Greater e-density at R NMR Spectroscopy
Note that ! C–R is slightly destabilized
! Less e-density at X NMR Spectroscopy
D. A. Evans Lone Pair Delocalization: N2F2 Chem 206
The interaction of filled orbitals with adjacent antibonding orbitals can
have an ordering effect on the structure which will stabilize a particular Now carry out the same analysis with the same 2
The trans Isomer orbitals present in the trans isomer.
geometry. Here are several examples:

Case 1: N2F2 This molecule can exist as either cis or filled


trans isomers N-SP2 F
antibonding !" N–F
F F F N N !" N–F (LUMO)
N N N N F filled
N-SP2
F (HOMO)
There are two logical reasons why the trans isomer should be more
stable than the cis isomer. ! In this geometry the "small lobe" of the filled N-SP2 is required to
overlap with the large lobe of the antibonding C–F orbital. Hence, when
! The nonbonding lone pair orbitals in the cis isomer will be destabilizing the new MO's are generated the new bonding orbital is not as stabilizing
due to electron-electron repulsion. as for the cis isomer.
! The individual C–F dipoles are mutually repulsive (pointing in same Conclusions
direction) in the cis isomer.
! Lone pair delocalization appears to override electron-electron and
dipole-dipole repulsion in the stabilization of the cis isomer.
In fact the cis isomer is favored by 3 kcal/ mol at 25 °C. ! This HOMO-LUMO delocalization is stronger in the cis isomer due
to better orbital overlap.
Let's look at the interaction with the lone pairs with the adjacent C–F
antibonding orbitals.
Important Take-home Lesson

The cis Isomer Orbital orientation is important for optimal orbital overlap.

F !" N–F
F (LUMO)
antibonding
N N !" N–F A B forms stronger pi-bond than A B
filled
filled N-SP2
N-SP2
(HOMO)
A forms stronger
B A B
! Note that by taking a linear combination of the nonbonding and sigma-bond than
antibonding orbitals you generate a more stable bonding situation.
! Note that two such interactions occur in the molecule even though This is a simple notion with very important consequences. It surfaces in
only one has been illustrated. the delocalized bonding which occurs in the competing anti (favored)
syn (disfavored) E2 elimination reactions. Review this situation.
D. A. Evans The Anomeric Effect: Negative Hyperconjugation Chem 206
! Since the antibonding C–O orbital is a better acceptor orbital than the
The Anomeric Effect antibonding C–H bond, the axial OMe conformer is better stabilized by
this interaction which is worth ca. 1.2 kcal/mol.
It is not unexpected that the methoxyl substituent on a cyclohexane ring Other electronegative substituents such as Cl, SR etc also participate in
prefers to adopt the equatorial conformation. anomeric stabilization.
H
"" H
H 1.781 Å
OMe O H
Cl H O O
OMe
! Gc° = +0.6 kcal/mol Cl
What is unexpected is that the closely related 2-methoxytetrahydropyran This conformer 1.819 Å Cl
prefers the axial conformation: preferred by 1.8 kcal/mol Why is axial C–Cl bond longer ?
H
axial O lone pair!"# C–Cl
O H "# C–Cl
OMe
O ""
OMe
! Gp° = –0.6 kcal/mol H
O
That effect which provides the stabilization of the axial OR O
conformer which overrides the inherent steric bias of the Cl HOMO
substituent is referred to as the anomeric effect.

Let anomeric effect = A " C–Cl

$ Gp° = $ Gc° + A The Exo-Anomeric Effect


A = $ Gp° – $ Gc°
! There is also a rotational bias that is imposed on the exocyclic
A = –0.6 kcal/mol – 0.6 kcal/mol = –1.2 kcal/mol C–OR bond where one of the oxygen lone pairs prevers to
be anti to the ring sigma C–O bond

Principal HOMO-LUMO interaction from each conformation is


illustrated below: O H
O R O
H
!! O O R O
R
OMe O H favored
O
!! OMe A. J. Kirby, The Anomeric and Related Stereoelectronic Effects at Oxygen,
Springer-Verlag, 1983
E. Jurasti, G. Cuevas, The Anomeric Effect, CRC Press, 1995
axial O lone pair!"# C–H axial O lone pair!"# C–O
D. A. Evans The Anomeric Effect: Carbonyl Groups Chem 206

Do the following valence bond resonance structures Aldehyde C–H Infrared Stretching Frequencies
have meaning?

R Prediction: The IR C–H stretching frequency for aldehydes is lower


R
than the closely related olefin C–H stretching frequency.
C O C O For years this observation has gone unexplained.
!!

X X
R R R
C O C C
Prediction: As X becomes more electronegative, the IR frequency H H R
should increase ! C–H = 2730 cm -1 ! C–H = 3050 cm -1

O O O Sigma conjugation of the lone pair anti to the H will weaken the bond.
This will result in a low frequency shift.
Me CH3 Me CBr3 Me CF3
!C=O (cm-1) 1720 1750 1780 Infrared evidence for lone pair delocalization into
vicinal antibonding orbitals.
The N–H stretching frequency of cis-methyl diazene is 200 cm-1 lower
than the trans isomer.
Prediction: As the indicated pi-bonding increases, the X–C–O Me H
bond angle should decrease. This distortion improves overlap. Me H
N N antibonding
N N "# N–H
filled
N-SP2
R R ! N–H = 2188 cm -1
C O C O Me
Me antibonding
N N "# N–H
X N N
X H filled ..
N-SP2 H
! N–H = 2317 cm -1
!* C–X "O lone pair
! The low-frequency shift of the cis isomer is a result of N–H bond
Evidence for this distortion has been obtained by X-ray crystallography weakening due to the anti lone pair on the adjacent (vicinal)
nitrogen which is interacting with the N–H antibonding orbital. Note
that the orbital overlap is not nearly as good from the trans isomer.
Corey, Tetrahedron Lett. 1992, 33, 7103-7106
N. C. Craig & co-workers JACS 1979, 101, 2480.
D. A. Evans The Anomeric Effect: Nitrogen-Based Systems Chem 206

Observation: C–H bonds anti-periplanar to nitrogen lone pairs are


CMe3
spectroscopically distinct from their equatorial C–H bond counterparts
Me3C N N
N CMe3 Me3C N N
N
Me3C
!" C–H Me3C
H N
H !G° = – 0.35kcal/mol
H
H
H N A. R. Katritzky et. al., J. Chemm. Soc. B 1970 135
HOMO

! C–H Favored Solution Structure (NMR)


Spectroscopic Evidence for Conjugation Me
MeN NMe
Me N N
Infrared Bohlmann Bands MeN NMe N N Me
Characteristic bands in the IR between 2700 Me
and 2800 cm-1 for C-H4, C-H6 , & C-H10 stretch
J. E. Anderson, J. D. Roberts, JACS 1967 96 4186
Bohlmann, Ber. 1958 91 2157
Reviews: McKean, Chem Soc. Rev. 1978 7 399 B Favored Solid State Structure (X-ray crystallography)
L. J. Bellamy, D. W. Mayo, J. Phys.
Chem. 1976 80 1271 1.484
1.453 Bn
Me N N
NMR : Shielding of H antiperiplanar to N lone pair N N Me
Bn Bn Me
H10 (axial): shifted furthest upfield N N N N
1.453
H6, H4: !" = " Haxial - " H equatorial = -0.93 ppm Me 1.459 1.457
Protonation on nitrogen reduces !" to -0.5ppm Bn
A B

H. P. Hamlow et. al., Tet. Lett. 1964 2553 Rationalize why B might be more stable than A.
J. B. Lambert et. al., JACS 1967 89 3761
A. R. Katrizky et. al., J. C. S. Perkin II 1980 1733
D. A. Evans Carboxylic Acids (& Esters): Anomeric Effects Again? Chem 206

! Hyperconjugation: Let us now focus on the oxygen lone pair in the hybrid
! Conformations: There are 2 planar conformations.
orbital lying in the sigma framework of the C=O plane.
O O (Z) Conformer
(Z) Conformer R' (E) Conformer R !* C–O
R O R O In the (Z) conformation this
C O
•• O lone pair is aligned to overlap O
O O R'
Specific Case: R with !* C–O.
Methyl Formate Me !G° = +4.8 kcal/mol R
H O H O
R O
Me
The (E) conformation of both acids and esters is less stable by 3-5 kcal/mol. If (E) Conformer
this equilibrium were governed only by steric effects one would predict that the R
(E) conformation of formic acid would be more stable (H smaller than =O). C O In the (E) conformation this
Since this is not the case, there are electronic effects which must also be R O lone pair is aligned to overlap
considered. These effects will be introduced shortly. •• with !* C–R.
!* C–R
O
! Rotational Barriers: There is hindered rotation about the =C–OR bond.
R
These resonance structures suggest O C Since !* C–O is a better acceptor than !* C–R R O
hindered rotation about =C–OR bond. O (where R is a carbon substituent) it follows that
barrier ~ 10-12
This is indeed observed: kcal/mol R the (Z) conformation is stabilized by this interaction. R
O O O
R' R'
R O R O R O
Energy

O
R R
Rotational barriers are ~ 10-12 R O !G° ~ 2-3 Esters versus Lactones: Questions to Ponder. O
kcal/mol 1
kcal/mol. This is a measure of the Et
strength of the pi bond. CH3CH2 O
Esters strongly prefer to adopt the (Z) conformation while O
small-ring lactones such as 2 are constrained to exist in the
(Z) conformation. From the preceding discussion explain the O
following: 2
! Lone Pair Conjugation: The oxygen lone pairs conjugate with the C=O.
1) Lactone 2 is significantly more susceptible to nucleophilic
versus
attack at the carbonyl carbon than 1? Explain.
•• R The filled oxygen p-orbital interacts with pi (and pi*) 2) Lactone 2 is significantly more prone to enolization than 1?
C O
•• O C=O to form a 3-centered 4-electron bonding system. In fact the pKa of 2 is ~25 while ester 1 is ~30 (DMSO). Explain.
R 3) In 1985 Burgi, on carefully studying O O O
! " ! " ! "
the X-ray structures of a number of
SP2 Hybridization
lactones, noted that the O-C-C (!) & O O
O
O-C-O (") bond angles were not equal.
! Oxygen Hybridization: Note that the alkyl oxygen is Sp2. Rehybridization Explain the indicated trend in bond
is driven by system to optimize pi-bonding. angle changes. !#" = 12.3 ° !#" = 6.9 ° !#" = 4.5 °
D. A. Evans Anomeric Effects in DNA Phosphodiesters Chem 206
Calculated Structure of ACG–TGC Duplex The Phospho-Diesters Excised from Crystal Structure

Guanine

Cytosine 1B

Cytosine

2B

1A

Phosphate-1A Phosphate-1B
Thymine

Adenine
The Anomeric Effect
Acceptor orbital hierarchy: !* P–OR * > !* P–O–

R R

O !– O
O
!– O O
P O P
R R
!– O
! –
O
Gauche-Gauche conformation

O Phosphate-2A Phosphate-2B
!– O R !– O R O
P O O
P
R R Oxygen lone pairs may establish a simultaneous hyperconjugative
!– O !– O relationship with both acceptor orbitals only in the illustrated
Anti-Anti conformation conformation.
Plavec, et al. (1996). “How do the Energetics of the Stereoelectronic Gauche &
Gauche-Gauche conformation affords a better donor-acceptor relationship Anomeric Effects Modulate the Conformation of Nucleos(t)ides?
” Pure Appl. Chem. 68: 2137-44.
D. A. Evans Olefin Epoxidation via Peracids: An Introduction Chem 206

! The General Reaction:


■ The transition state:
R R O R R O
+ "
OH " +
R O R OH
R R R R

HOMO LUMO note labeled oxygen is transferfed


!C–C "*O–O O-O bond energy: ~35 kcal/mol
R R

HOMO–LUMO Interactions for Peracid Epoxidation


R R
Since 2 C–O bonds are formed in the epoxidation reaction, there are two
HOMO–LUMO pairs that should be considered. They are illustrated below. View from below olefin

! Reaction rates are governed by olefin nucleophilicity. The rates of


epoxidation of the indicated olefin relative to cyclohexene are provided
below: OH
OH OAc

LUMO "*O–O HOMO O lone pair 1.0 0.6 0.05 0.4


! The indicated olefin in each of the diolefinic substrates may be
oxidized selectively.
Me Me Me

Me

Me
H
Me Me Me Me

HOMO !C–C LUMO !#C–C


D. A. Evans Olefin Epoxidation with Dioxiranes Chem 206

R R R R
Asymmetric Epoxidation with Chiral Ketones
R O O
+ ! + Review: Frohn & Shi, Syn Lett 2000, 1979-2000 (PDF)
R !
O R R
R R R R Me
Me
HOMO1 LUMO1 note labeled oxygen is transferfed
O
O
O
!C–C #*O–O O-O bond energy: ~35 kcal/mol chiral catalyst
LUMO2 HOMO2
O O
!"C–C O lone pr
Me O
R2 O R2
Me
Transition State for the Dioxirane Mediated Olefin Epoxidation
R1 R2 oxone, CH3CN-H2O R1 R2
pH 7-8
O R O R
Me
planar R R
O O Ph Me Ph
spiro Ph Ph Ph
rotate 90° >95% ee 84% ee 92% ee

Question: First hour Exam 2000 (Database Problem 34)


stabilizing Olp ! "* C=C
cis olefins react ~10 times faster than trans Question 4. (15 points). The useful epoxidation reagent dimethyldioxirane (1) may be
prepared from "oxone" (KO3SOOH) and acetone (eq 1). In an extension of this epoxidation
concept, Shi has described a family of chiral fructose-derived ketones such as 2 that, in the
Houk, JACS, 1997, 12982. presence of "oxone", mediate the asymmetric epoxidation of di- and tri-substituted olefins
with excellent enantioselectivities (>90% ee) (JACS 1997, 119, 11224).
! Synthesis of the Dioxirane Oxidant
H Me
Me Me Me
O R O KO3SOOH O
O K+ –O R O O O
(1) O O
S O R O CH3CN-H2O O
O O H R "
O Me pH 10.5 Me 2
R R SO3
1
(Oxone) R2 O R2 O
1 equiv 2 O
(2) O
oxone, Me
Synthetically Useful Dioxirane Synthesis R1 R2
CH3CN-H2O R1 R2
Me
pH 10.5 >90% ee
O oxone O O co-distill to give
~0.1 M soln of Part A (8 points). Provide a mechanism for the epoxidation of ethylene with
Me Me Me Me dioxirane in acetone dimethyldioxirane (1). Use three-dimensional representations, where relevant, to illustrate
the relative stereochemical aspects of the oxygen transfer step. Clearly identify the
O oxone co-distill to give frontier orbitals involved in the epoxidation.
O O
~0.6 M soln of dioxirane
F3C CF3 in hexafluoroacetone Part B (7 points). Now superimpose chiral ketone 2 on to your mechanism proposed
F3C CF3 above and rationalize the sense of asymmetric induction of the epoxidation of trisubstituted
olefins (eq 2). Use three-dimensional representations, where relevant, to illustrate the
absolute stereochemical aspects of the oxygen transfer step.
D. A. Evans The Baeyer-Villiger Reaction: Stereoelectronic Effects Chem 206
CMe3
Let RL and RS be Sterically large and small substituents. Me
O O Me O
O O O Me3C
+ RCO3H O - MeCO2H
+ RCO3H O
C RL C + C RS Me CMe3 O O
RL RS – RCO2H O RS RL O R
H O H
major minor O R

The major product is that wherein oxygen has been inserted into
the RL–C=O bond. Favored Migrating group
H
kR O
R kR / KMe CMe3
R C O
O Me 72 O
CH3CH2 CMe3
O major Me OH Me O
O
C + CF3CO3H CH3(CH2)2 150 O
R Me
Conformer A R
O (CH3)3C 830
kMe C Me
R O >2000
PhCH2
minor
H
RS OH Disfavored Migrating group

The Intermediate C O R Me
RL O
O
O
O Me
Me3C OH
O CMe3
O
O

The important stereoelectronic components to this rearrangement: R


The destabilizing
Conformer B gauche interaction
1. The RL–C–O–O dihedral angle must be180° due to the HOMO
LUMO interaction σ-RL–C with σ∗−O–O.
Steric effects destabilize Conformer B relative to Conformer A;
2. The C–O–O–C' dihedral angle will be ca. 60° due to the gauche hence, the reaction is thought to proceed via a transition
effect (O-lone pairsσ∗−C–O). state similar to A.

This gauche geometry is probably reinforced by intramolecular For relevant papers see:
hydrogen bonding as illustrated on the opposite page: Crudden, Angew. Chem. Int. Ed 2000, 39, 2852-2855 (pdf)
Kishi, JACS 1998, 120, 9392 (pdf)
D. A. Evans Stereoelectronic Effects Part 3 Chem 206

"Rules for Ring Closure: Baldwin's Rules"


http://www.courses.fas.harvard.edu/colgsas/1063

Chemistry 206

Advanced Organic Chemistry

Lecture Number 3

Stereoelectronic Effects-3

! The SN2 Reaction: Stereoelectronic Effects Sir Jack Baldwin


! Baldwin's Rules for Ring Closure The Primary Literature
Baldwin, J. Chem. Soc., Chem. Comm. 1976, 734, 736.
Baldwin, J. Chem. Soc., Chem. Comm. 1977 233.
Read Kirby, Chapter 5, Cary & Sundberg, Chapter 5 Baldwin, J. Org. Chem. 1977, 42, 3846.
Baldwin, Tetrahedron 1982, 38, 2939.
Useful LIterature Reviews
Johnson, C. D. (1993). “Stereoelectronic effects in the formation of 5-
and 6-membered rings: the role of Baldwin's rules.” Problem 689. Predict the product of the reaction below. Include a
Acc. Chem. Res. 26: 476-82. (Pdf) rationalization based on a detailed analysis of competing transition
state geometries.
Beak, P. (1992). “Determinations of transition-state geometries by the
endocyclic restriction test: mechanisms of substitution at
nonstereogenic atoms.” Acc. Chem. Res. 25: 215. (Pdf) MeO O O Base, THF
Friday, Cl
D. A. Evans OEt 64%
September 22, 2006
D. A. Evans Three-Center Bonds: A Review Chem 206

Consider the linear combination of three atomic orbitals. The resulting Case 3: 2 p-Orbitals; 1 s-orbital
molecular orbitals (MOs) usually consist of one bonding, one nonbonding
and one antibonding MO. antibonding

Case 1: 3 p-Orbitals 2
+ nonbonding
pi-orientation antibonding

bonding
Energy

3 nonbonding

Case 4: 2 s-Orbitals; 1 p-orbital Do this as an exercise

bonding

Note that the more nodes there are in the wave function, the higher its energy. Examples of three-center bonds in organic chemistry
+
H2C CH CH2 Allyl carbonium ion: both pi-electrons in bonding state A. H-bonds: (3–center, 4–electron)
! O H O
H2C CH CH2 Allyl Radical: 2 electrons in bonding obital plus one in The acetic acid dimer is
CH3 CH3
nonbonding MO. stabilized by ca 15 kcal/mol
O H O
– Allyl Carbanion: 2 electrons in bonding obital plus 2 in
H2C CH CH2 nonbonding MO.
B. H-B-H bonds: (3-center, 2 electron)

H H H H H
B B B H H B
Case 2: 3 p-Orbitals H H H H H

sigma-orientation diborane stabilized by 35 kcal/mol


antibonding

3 C. The SN2 Transition state: (3–center, 4–electron)


Energy

The SN2 transition state approximates a case 2


nonbonding H situation with a central carbon p-orbital
The three orbitals in reactant molecules used are:
Nu C Br 1 nonbonding MO from Nucleophile (2 electrons)
1 bonding MO ! C–Br (2 electrons)
H H 1 antibonding MO !* C–Br
bonding
D. A. Evans Substitution Reactions: General Considerations Chem 206

Why do SN2 Rxns proceed with backside displacement? Electrophilic substitution at saturated carbon may occur
with either inversion of retention

R R R
!– !– Inversion
Nu: – C X Nu C X Nu C – ‡
H H X: Ra Ra
H Ra
H H H !+ !+
El(+) C M Nu C M Nu C M+
H H
Rb H Rb
Rb
Given the fact that the LUMO on the electrophile is the C–X antibonding
orblital, Nucleophilic attack could occur with either inversion or retention. Retention
!+ ‡
Inversion Retention Ra Ra M Ra
R R C M C
El(+) H H H
C El M+
Rb Rb El !+
Nu C C X Rb
!!
X
H H !!

H H Ra
HOMO LUMO Ra
Nu
El(+) C !!
M
Constructive overlap between Overlap from this geometry results C !!
M
in no net bonding interaction H
Nu & !*C–X H
LUMO HOMO Rb
Rb

Inversion
Expanded view of !*C–X
El(+)
Retention
LUMO C X Examples
antibonding bonding

!! Br2 CO2
HOMO H Li CO2Li
Br H H
Nu
predominant inversion predominant retention
Fleming, page 75-76 Stereochemistry frequently determined by electrophile structure
See A. Basu, Angew. Chem. Int. Ed. 2002, 41, 717-738 (PDF)
D. A. Evans SN2 Reaction: Stereoelectronic Effects Chem 206
Related Cases: " The Endocyclic Restriction Test"

R R R
+ –
!– !– SO3CH3 SO3
Nu: – C X Nu C X Nu C X: – (CH3)2N (CH3)3N
H H
H H H
H
X
What is the absolute requirement for the Nu–C–X bond angle?? exclusively
180° +/– ?? Me O
N H S intermolecular
Me C O O
H
Eschenmoser: Tether Nu and X H
R
R
!– !– King, et al. 1982 Chem Commun. 175
Nu: – C X Nu C X King, et al. 1982 Tetrahedron Lett. 23, 4465
H
H H H


"tethered reactants" "constrained transition state" SO3CH3 SO3
+
N(CH3)2 N(CH3)3
Eschenmoser, Helv. Chim Acta 1970, 53, 2059 (PDF)

O O
S CH3 – Me
SO3
O
N H 16% intramolecular
Nu: –
CH3 Me 84% intermolecular
Nu H C
O S
O O H
O
O
S
O 10-membered transition structure
X
H
C H Intramolecular option Hence, the Nu–C–X 180 ° transition state bond angle must be rigidly
is not tenable maintained for the reaction to take place.
Nu: H

"Determinations of transition-state geometries by the endocyclic restriction test:


The reaction illustrated above proceeds exclusively through bimolecular mechanisms of substitution at nonstereogenic atoms",
pathway in contrast to the apparent availability of the intramolecular path. Peter Beak,
Acc. Chem. Res.; 1992 25, 215-222 (PDF)
D. A. Evans, J. Johnson Rules for Ring Closure: Introduction Chem 206

Ring Closure and Stereoelectronic Considerations C. Nucleophilic ring closures sub-classified according to hybridization
An Examination of Baldwin's Rules state of electrophilic component:
(tetrahedral = tet; trigonal = trig; digonal = dig)
"Baldwin's Rules" provides a qualitative set of generalizations on the
probability of a given ring closure. D. Nucleophilic ring closures further subclassified according to size of
the fomed ring. For example:
There are circumstances where the "rules" don't apply.
! They do not apply to non-first-row elements participating in the 5-exo-tet
cyclization event. The longer bond lengths and larger atomic radii of X X
2nd row elements result in relaxed geometrical constraints. Y Y–

For example, a change in a heteroatom from O to S could result in


relaxation of a given geometric constraint. 5-exo-trig
X X
Y Y–
endo
X = O vs S
X Y X 5-exo-dig
••

Y
X
X
! The "rules" do not apply to electrocyclic processes.
Y Y–
Required trajectories (Baldwin):
Nomenclature
! !
Classes of Ring Closing Processes ! = 180 "
X Y X Y
A. Exo-cyclization modes identified by the breaking bond
being positioned exocyclic to the forming cycle.
exo X
! X !
X ! = 109 " Y
X
••

Y
Y Y
B. Endo-cyclization modes identified by the breaking bond Will come
being positioned endocyclic to the forming cycle. back to this
X case later
! " X
endo * ! # 120 * ! !
X Y X !
••

Y Y Y
X = first-row element
N, O Baldwin, J. Chem. Soc., Chem. Commun., 1976, 734.
D. A. Evans, J. Johnson Rules for Ring Closure: SP3 Carbon & Related Systems Chem 206

FÜRST-PLATTNER RULE
Tetrahedral Carbon
All exo cyclization modes are allowed: (n-exo-tet, n = 3!) In this simple model, the transition-state leading to 1 involves the
diaxial orientation of nucleophile and leaving group. This orientation
affords the best overlap of the anti-bonding C–Y orbital and the
exo nonbonding electron pairs on the nucleophile O–.

X C X C
••

In the formation of the diastereomeric epoxide 2, the proper


Y Y–
alignment of orbitals may only be achieved by cyclization through the
less-favored boat conformer. Accordingly, while both cyclizations are
There are stereoelectronic issues to consider for n-exo-tet cyclizations
"allowed", there are large rate differences the the rates of ring closure.
Formation of 3-Membered Rings (3-exo-tet)
While the FÜRST-PLATTNER RULE deals wilth the microscopic
H H H ‡ reverse, in the opening of epoxides by nucleophiles, the
C Y X CH2 stereoelectronic arguments are the same.
••

X X C Y
+ Y–
C C H
C
H2 H2 H2
Stereoelectronic Effects in Epoxide Ring Cleavage
Nu
Nu-
Conformational Effects in Epoxide Ring Formation/cleavage Me3C Me3C H
H
O
Those stereoelectronic effects that operate in ring cleavage also H H HO
influence ring formation. Consider a rigid cyclohexene oxide system:
HO
!– O
Y
Y ‡ Nu-
Me3C Me3C H
faster H H
H
H H H H H Nu
!– O
1 O
O–
Y !– ‡ Me Me
Nu
H slower Nu-
O
Y H
H HO
O– O H H
H
O !–
chair boat 2
D. A. Evans, J. Johnson Rules for Ring Closure: SP3 Carbon & Related Systems Chem 206

Tetrahedral Carbon Case 2: King, J.C.S. Chem. Comm., 1979, 1140.


Endo cyclization modes that are disallowed O O
O
(n-endo-tet, n = 3!"9) S 8-endo-tet S
O
disfavored _
O O
Me
NMe2
Rxn exclusively
NMe3+
endo intermolecular

X X Y
••

••
8-endo-tet
disfavored SO3–
C C(SP3) SO2OMe

NMe2 NMe3+
Rxn exclusively
The stereoelectronic requirement for a 180° X–C–Y bond angle is only intermolecular
met when the endo cyclization ring size reaches 9 or 10 members.

9-endo-tet SO3–
Case 1: Eschenmoser, Helvetica Chim. Acta 1970, 53, 2059. SO2OMe
borderline
NMe3+
O O NMe2
O O 84% intermolecular,
S S 16% intramolecular
O O-
NaH
CX3
CX3 Conclusions
6-endo-tet
O S O disfavored O S O
Allowed endo cyclization modes will require transition state ring sizes
of at least nine members.
Rxn exclusively
intermolecular Intramolecular epoxidation has also been evaluated
(lecture 2)
CY3 CY3 Beak, JACS 1991, 113, 6281.
O
Cl
8-endo-tet
Cyclization exclusively intermolecular. However the exocyclic analog disfavored Cl CO2H
O–OH
is exclusively intramolecular
O O O O
S n n
S O
O O-
n = 1: rxn exclusively intermolecular
CX2I NaH CX2 n = 9: rxn is intramolecular
6-exo-tet
O S O
favored O S O Beak states that the conclusions made with carbon
substitution also hold for oxygen atom transfer.
Rxn exclusively
intramolecular Beak, P. (1992). “Determinations of transition-state geometries by the
CY3 CY3 endocyclic restriction test: mechanisms of substitution at nonstereogenic
atoms.” Acc. Chem. Res. 25: 215.
D. A. Evans Olefin Epoxidation via Peracids: An Intramolecular Case Chem 206

■ The General Reaction:


Per-arachidonic acid Epoxidation
R R O R R O
O
+ ●
OH ● + Me
R O !
R R R R R OH
O H
HOMO LUMO note labeled oxygen is transferfed
πC–C σ*O–O O-O bond energy: ~35 kcal/mol

■ The transition state:

View from below olefin

Per-arachidonic acid Epoxidation O


Me H
O ! O
Me !
O H E. J. Corey, JACS 101, 1586 (1979)

Which olefin is selectively epoxidized?? For a more detailed study see P. Beak, JACS 113, 6281 (1991)

For theoretical studies of TS see R. D. Bach, JACS 1991, 113, 2338


R. D. Bach, J. Org. Chem 2000, 65, 6715
D. A. Evans, J. Johnson Rules for Ring Closure: SP3 Carbon Versus Silicon Chem 206

Tetrahedral Silicon Brook Rearrangements


Me
Explain what drives this rearrangement. TBS = Si CMe3
All Endo-Tet cyclization modes are allowed
Me
TBSO OH OH OTBS
KHMDS
endo Bu3Sn Bu3Sn
THF, -78 °C
X X Y OMe Me OMe
••

Y 94%

••
Si Si(SP3) Calter, M. A. Ph. D. Thesis, Harvard University, 1993.

Silicon may readily proceed through a hypervalent intermediate. TMS SiR3 OTMS
O
O O (1)
O R P
RO P R P
OR RO OR
RO OR
O
F
Ph R H Me Me OMe
F Si fails! O
Ph O O (2)
O R P
RO P R P
F RO OR
OR RO OR

Moser, W. H. "The Brook Rearrangement in Tandem Bond Formation


Inorg. Chem. 1984, 23, 1378 Strategies," Tetrahedron 2001, 57, 2065-2084
R3Si
O El(+) O El
R
Li
R SiR3 R R

3S +P
Li
O [1,2] Si R3Si Li
3D O
R
R
R3Si R R
El(+)
D. A. Evans, J. Johnson Rules for Ring Closure: SP2 Carbon & Related Systems Chem 206
Trigonal Carbon
Endo cyclization modes that are disallowed
(3 to 5-endo-trig) MeO2C CO2Me

n-endo-trig NH2

X Y X Y–
••

C C

X = first-row element
The 5-endo-trig cyclization is a watershed case

Case 1: Baldwin, J. Chem. Soc., Chem. Commun., 1976, 734.


distance from reacting centers: 2.77 Å
CO2Me base CO2Me
X
OH 5-endo-trig O
Disfavored It is possible that a "nonvertical"
however
trajectory is operational like that
CO2Me base CO2Me suspected in C=O addition

SH S

Second row atom relaxes the cyclization geometrical requirement

Case 2: Baldwin, J. Chem. Soc., Chem. Commun., 1976, 736.

MeO2C CO2Me MeO2C CO2Me


X
NH2 HN
5-endo-trig
0%
MeO2C
5-exo-trig
HN 100%
O
D. A. Evans, J. Johnson Rules for Ring Closure: SP2 Carbon & Related Systems Chem 206

Case 2: continued...
Apparent exceptions to disallowed 5-endo-trig cyclization process
MeO2C CO2Me MeO2C CO2Me
O
X
NH2 HN
+
N CH3CO2H N OH N O
5-endo-trig
0%
MeO2C Filer, J. Am. Chem. Soc. 1979, 44, 285.
5-exo-trig
HN 100% CO2Me CO2Me CO2Me
O
1
R HC KOtBu R1 R1
R2
Control experiment: Intermolecular reaction favors conjugate addtion. N HN R2 HN CO2Me
CO2Me CO2Me 3:1 R2
Me Me Me
PhCH2NH2 H H R1 = aryl, R2 = aryl, alkyl
Ph N N Ph
CO2Me CO2Me Grigg, J. Chem. Soc., Chem. Commun. 1980, 648.
O 0%
100%

Does the illustrated R (CH2OH)2 R O


Case 3:
O ketalization process O
NH2NH2 Ph O necessarily violate "the H+ O
R R
Ph OMe rules"?
65 oC HN NH

R (CH2OH)2 R R R
O
1) EtO2CCl, pyridine Ph O OH H+ OH
O R
2) NH2NH2 ( )2 ( )2
Ph OK R HO O –H2O O
NH +
H2N

MeI H+ disfavored ? 5-endo-trig


200 oC X 5-endo-trig
O
R
R OH 5-exo-tet R O
Ph OMe +
Ph Ph ( )2
CO2Me O H2O O favored ?
NH2NH2 R O
HN HN NH
65 oC NH2
5-exo-trig Johnson, C. D. (1993). “Stereoelectronic effects in the formation of 5- and 6-
membered rings: the role of Baldwin's rules.”
Acc. Chem. Res. 26: 476-82.
D. A. Evans, J. Johnson Rules for Ring Closure: SP2 Carbon & Related Systems Chem 206

Trigonal Carbon: Exocyclic Enolate Alkylation


distance between reacting
exo centers: 3.37Å

C C C C
–O Y-
C Y O C
Br
X
■ By definition, an exo-tet cyclization, but stereoelectronically
MO
behaves as an endo trig. O

Me Me The overlap for C-alkylation is poor due to


Br Me geometrical constraints of 5-membered ring
Me Me (1)
O
X Me
MO
only observed O
product distance between reacting
centers: 3.04Å
However:
Me Me The relaxed geometrical constraint
KOt-Bu or LDA
provided by the added CH2 group
Me Me > 95% by NMR now renders the 6-membered
O Me Br O cyclization possible

Baldwin, J. Chem. Soc., Chem. Commun. 1977, 233.

■ Given the failure of the enolate alkylation shown above (eq 1),
explain why these two cyclizations are successful. MO Br O

Br
base
NHAr N
O O Ar
Favorskii Rearrangement (Carey, Pt B, pp 609-610)
R Your thoughts on the mechanism
R R O
O R O CO2Me
base
N Cl MeO– MeO–
NH OMs O
Ar Ar
–HCl
D. A. Evans, J. Johnson Rules for Ring Closure: SP2 & SP Carbon & Related Systems Chem 206

Trigonal Carbon: Intramolecular Aldol Condensations Digonal Carbon: Cyclizations on to Acetylenes


Baldwin, Tetrahedron 1982, 38, 2939
DIGONAL: Angle of approach for attack on triple bonds

(Enolendo)-Exo-trig Baldwin:
MO X
O
X Nu-
YM
R Y R 120°
- 3 and 4-Exo-dig are disfavored
Favored: 6-7-(enolendo)-exo-trig - 5 to 7-Exo-dig are favored
Disfavored: 3-5-(enolendo)-exo-trig 120°
- 3 to 7-Endo-dig are favored
E+

X
(Enolexo)-exo-trig X
Ab initio SCF 4-31G calculations for the interaction of
MO O YM
Y
hydride with acetylene:
R R _
H
Favored: 3-7-(enolexo)-exo-trig H
2.13 127 o 148o H 4-31G basis set
H H C C Houk, J.ACS.1979, 101, 1340.
O 5-(Enolendo)-Exo-trig 6-(Enolendo)-Exo-trig H 156o 1.22
Me Me O
Me O
O H STO-3G minimal basis set
1.5-2.0 110o -120o
I Me Dunitz, Helv Chim. Acta
favored H C C H 1978, 61, 2538.
Me
Me O O Me O
Me Me
III
O Crystal Structures do not support Baldwin
O Statistical Distribution, (I + II)/III = 2:1
II
Experimental Distribution, = 0:100
N
N 2.92
(KOH, MeOH, r.t., 5 min, 77% y.) 104o
O- N + O N
2.44
N+ 93o 86o
Caution: Baldwin's conclusions assume that the RDS is ring closure; N
however, it is well known (by some!) that the rate determining step is
dehydration in a base-catalyzed aldol condensation.
J. Dunitz and J. Wallis J. C. S. Chem. Comm. 1984, 671.
D. A. Evans, J. Johnson Rules for Ring Closure: SP Carbon & Related Systems Chem 206

! Indole synthesis:
Endo Digonal versus Endo Trigonal Cyclizations
CH3 CH2R
2 equiv. LDA

5-endo-trig 2 equiv. RX
N+ -78 oC N+
C- C-
R = Me, Bu, CO2Me
Y
LiTMP

X:
R
In-plane approach; Out-of-plane approach;
nucleophile lone pair is nucleophile lone pair can't Saegusa, J. Am. Chem. Soc. 1977, 99, 3532. _
orthogonal to !* achieve Bürgi-Dunitz angle
N Li+
! Spiro dihydrofuranones:
5-endo-dig
Li
O
HO O
:X MeO OMe KOtBu
Y Allowed due to in-plane pi orbitals OMe

n
X n = 1,2
n n

For an opposing viewpoint to Baldwin's view of nucleophile trajectories, see Developing negative charge on the central allenic carbon is
Menger's article on directionality in solution organic chemistry: in the same plane as the OMe group
Tetrahedron 1983, 39, 1013.
Magnus, J. Am. Chem. Soc. 1978, 100, 7746.
O O
Me
NaOMe Me
Me
MeOH Me
HO Ph O Ph
5-endo-dig
5-exo-dig
O O
NaOMe Li
R X R Ph Li
OH 5-endo-trig O
Ph
R = H, OMe Ph
however, the acid catalyzed version does cyclize 4-endo-dig Li
Baldwin, J. Chem. Soc., Chem. Commun., 1976, 736. X
Johnson, Can. J. Chem. 1990, 68, 1780 Ph
Li
J. Am. Chem. Soc. 1983, 105, 5090
J. Chem. Soc., Chem. Commun. 1982, 36.
D. A. Evans, J. Johnson Rules for Ring Closure: SP Carbon & Related Systems Chem 206

Digonal Cyclizations: Interesting Examples


Conclusions and Caveats

! Baldwin's Rules are an effective first line of analysis in OTBS


LiCH2NC;
evaluating the stereoelectronics of a given ring closure O TBS-Cl OTBS 1) RCOCl
2) AgBF4

71% Me
! Baldwin's Rules have provided an important foundation for the Me Me 86% N+
study of reaction mechanism N+ C
R
C-
! Competition studies between different modes of cyclization only O
give information about relative rates, and are not an absolute
indicator of whether a process is "favored" or "disfavored" Works for varying ring sizes and R groups; acylnitrilium
ion can also work as an electophile in a Friedel-Crafts
! Structural modifications can dramatically affect the cyclization type of reaction 5-endo-dig
mode; beware of imines and epoxides
! Livinghouse, Tetrahedron 1992, 48, 2209.

EXO ENDO
O
Tet Trig Dig Tet Trig Dig
H Me
3 ! ! X X X R
4 ! X N
! X X
O
5 ! ! ! X X !
6 ! ! X !
! !
7 ! ! ! X ! !

MeO2C Et3N, Toluene, reflux


CN Me OH
12 h, 65-70% y. H
CN
Me
5-exo-dig
O
O O CO2Me HO2C
H
! Trost, JACS 1979, 101, 1284 DAE undergraduate, UCLA, 1972
D. A. Evans Acyclic Conformational Analysis-1 Chem 206

http://www.courses.fas.harvard.edu/colgsas/1063 Problem 61. The following stereoselective hydroboration has been reported by Kishi in
his synthesis of monensin (JACS 1979, 101, 259). Provide the stereostructure of the
major product and rationalize the stereochemical outcome as indicated in the directions.

Chemistry 206
BH3, THF
The product ?
Advanced Organic Chemistry O OCH2Ph
H2O2, -OH Stereoselection: 8/1
Me Me

Lecture Number 4
Problem 68. The following stereoselective enolate alkylation has been reported by Kim
(Tetrahedron Lett. 1986, 27, 943). Provide the stereostructure of the major product and
Conformational Analysis-1 rationalize the stereochemical outcome as indicated in the directions.

Me
LiNR2 The product ?
TsO
! Ethane, Propane, Butane & Pentane Conformations CO2Me Stereoselection: >40:1

C4H9
! Simple Alkene Conformations

Problem 722. Carbonium ion A has been calculated to be 38


! Reading Assignment for week kcal/mol more stable than carbonium ion B (Jorgensen JACS 1985,
R3Si CH2 CH2
107, 1496). The profound stabilization of carbonium ions by silicon in A
A. Carey & Sundberg: Part A; Chapters 2 & 3 this fashion is referred to as the "beta-silicon effect". For example,
vs
the SN1 solvolysis reaction of 1 is 10+12 times as fast as the
R. W. Hoffmann, Angew. Chem. Int. Ed. Engl. 2000, 39, 2054-2070 corresponding reaction of 2. The solvolysis of 2 leads to the olefin.
Conformation Design of Open-Chain Compounds (handout) For a good review see: Lambert Acc. Chem. Res. 1999, 32, 183-190 R3 C CH2 CH2

The Ethane Barrier Problem B


F. Weinhold, Nature 2001, 411, 539-541
SiMe3 Me
"A New Twist on Molecular Shape" (handout) Solvolysis (CF3CH2OH)
Me3C H Me3C H
F. M. Bickemhaupt & E. J. Baerends, Angew. Chem. Int. Ed. 2003, 42, 4183- H H
k1
4188,"The Case for Steric Repulsion Causing the Staggered Conformation H OCOCF3 = 2.4 x 10+12 H OCOCF3
1 k2 2
in Ethane" (handout)
F. Weinhold,, Angew. Chem. Int. Ed. 2003, 42, 4188-4194,"Rebuttal of the
Bikelhaupt–Baerends Case for Steric Repulsion Causing the staggered Part A: Identify the HOMO– 1-LUMO 2-LUMO
Connformation of Ethane" (handout) LUMO interactions in the SN1
reactions of 1 and 2. 1-HOMO 2-HOMO
Monday,
D. A. Evans September 25, 2006
D. A. Evans Acyclic Conformational Analysis-1 Chem 206

Ethane Rotational Barrier: The FMO View


F. Weinhold, Angew. nature 2001, 411, 539-541"A New Twist on Molecular Shape"

One explanation for the rotational barrier in ethane is that better overlap is
possible in the staggered conformation than in the eclipsed conformation as
shown below.

In the staggered conformation there are 3 anti-periplanar C–H Bonds

H !" C–H
H !* C–H
LUMO
C C C C
! C–H
HOMO
H H ! C–H

In the eclipsed conformation there are 3 syn-periplanar C–H Bonds

H H !" C–H
H H
C C ! C–H !* C–H
HOMO LUMO
C C
! C–H

Following this argument one might conclude that:


For purposes of analysis, each eclipsed conformer may be broken up ! The staggered conformer has a better orbital match between
into its component destabilizing interactions. bonding and antibonding states.
Incremental Contributions to the Barrier. ! The staggered conformer can form more delocalized molecular
Eclipsed atoms " E (kcal mol -1) orbitals.
Structure
J. P. Lowe was the first to propose this explanation
ethane 3 (H!H) +1.0 kcal mol -1 "A Simple Molecuar Orbital Explanation for the Barrier to Internal
Rotation in Ethane and Other Molecules"
propane 2 (H!H) +1.0 kcal mol -1 J. P. Lowe, JACS 1970, 92, 3799
1 (H!Me) +1.4 kcal mol -1 Me
Me Estimate the rotational barrier about the C1-C2 bond
Me in isobutane
D. A. Evans Acyclic Conformational Analysis: Butane Chem 206

Butane
Relationship between !G and Keq and pKa
Using the eclipsing interactions extracted from propane & ethane we should
! G° = –RT Ln K be able to estimate all but one of the eclipsed butane conformations
Recall that: or
Me Me
! G˚ = –2.3RT Log10K H H H
H eclipsed
staggered H
C C conformation
conformation H H
!E=?
At 298 K: 2.3RT = 1.4 (!G in kcal Mol–1 ) Me Me
H

! G˚298 = –1.4 Log10Keq Eclipsed atoms ! E (kcal mol -1)

1 (H"H) +1.0 kcal mol -1


Since pKeq = – Log10Keq 2 (H"Me) +2.8 kcal mol -1
# E est = 3.8 kcal mol -1

The estimated value of +3.8 agrees quite well with the value of +3.6
reported by Allinger (J. Comp. Chem. 1980, 1, 181-184)

! G˚298 = 1.4 pKeq


n-Butane Torsional Energy Profile

Hence, pK is proportional to the free energy change HH


H
H
E2 C
Keq pKeq !G˚ E1
H Me
H MeMe
1.0 0 0 H
C
energy

10 –1 –1.4 Me
H H
H
H
Me
100 –2 –2.8 kcal /mol H H C
+5.1
H Me
C
H H Me G Barrier?
+3.6
Me A
+0.88
Ref = 0
D. A. Evans Acyclic Conformational Analysis: Butane Chem 206

Butane continued Me Me Me
H H H Me Me H
From the torsional energy profile established by Allinger, we should be able to Nomenclature for C
C C
extract the contribution of the Me"Me eclipsing interaction to the barrier: staggered conformers: H H H H
H H
Me H H
H Me trans or t gauche(+) gauche(-)
H Me H Me
staggered C
H eclipsed or (anti) or g+ or g-
C
conformation Me H conformation Conformer
H population 70% 15% 15%
H H
! E = +5.1 kcal mol-1 at 298 K:
(Klyne, Prelog, Experientia 1960, 16, 521.)
Let's extract out the magnitide of the Me–Me interaction RR

2 (H!H) + 1 (Me!Me) = +5.1 C

1 (Me!Me) = +5.1 – 2 (H!H) 0°


R R
R R
1 (Me!Me) = +3.1 C C
-60° sp +60°
Incremental Contributions to the Barrier. sc sc
Eclipsed atoms " E (kcal mol -1)

2 (H!H) +2.0 ac ac
+3.1 R -120° +120° R
1 (Me!Me) ap
R C C

Eclipsed Butane R
conformation 180°

R
From the energy profiles of ethane, propane, and n-butane, one may extract
C
the useful eclipsing interactions summarized below:
R
Hierarchy of Eclipsing Interactions n-Butane
Torsion angle Designation Symbol Conformer
-1
X Y ! E kcal mol Energy Maxima 0 ± 30° ± syn periplanar ± sp E2
X Y Energy Minima +60 ± 30° + syn-clinal + sc (g+) G
H H +1.0
+120 ± 30° + anti-clinal + ac E1
H C C H +1.4 180 ± 30° antiperiplanar ap (anti or t) A
H Me -120 ± 30° - anti-clinal - ac E1
H H Me Me +3.1 -60 ± 30° - syn-clinal - sc (g-) G
D. A. Evans Acyclic Conformational Analysis: Pentane Chem 206

n-Pentane
The double-gauche pentane
Rotation about both the C2-C3 and C3-C4 bonds in either direction (+ or -):
conformation
Me Me

Me H H Me
g+g- The new high-energy conformation: (g+g–)
H Me Me H
g+t
Me Me tg- Me Me

H H Me Me H H Estimate of 1,3-Dimethyl Eclipsing Interaction


t,t Me
g+g+ g-g-
Me

Me Me H Me Y
Me H X

H H g-t
tg+
g-g+

Δ G° = +5.5 kcal mol -1


Δ G = X + 2Y where:

X = 1,3(Me−Me) & Y = 1,3(Me−H)

1,3(Me−H) = Skew-butane = 0.88 kcal mol-1

1,3(Me-Me) = ΔG – 2Y = 5.5 –1.76 = + 3.7 kcal mol-1


1,3(Me!Me) = + 3.7 kcal mol -1

Estimates of In-Plane 1,2 &1,3-Dimethyl Eclipsing Interactions

Me Me Me Me Me Me Me Me

3.1 ~ 3.7 ~3.9 ~ 7.6

It may be concluded that in-plane 1,3(Me!Me) interactions are Ca +4


kcal/mol while 1,2(Me!Me) interactions are destabliizing by Ca 3 kcal/mol.
D. A. Evans Acyclic Conformational Analysis: Natural Products Chem 206

The syn-Pentane Interaction - Consequences Lactol & Ketol Polyether Antibioitics


R. W. Hoffmann, Angew. Chem. Int. Ed. Engl. 2000, 39, 2054-2070
Conformation Design of Open-Chain Compounds (handout)
Me Me Me Me
O OH O
Et
R R' R R' Me Me HO O O O
! or OH H H Me OH Et
R Me Me Et OH
Me Me Me H H Me H R R' H
tt g-g-
Ferensimycin B, R = Me
R R' R Me Me R' Lysocellin, R = H
! or
Me Me Me H H R' H H R H
tg
The conformation of these structures are strongly influenced by the
gt
acyclic stereocenters and internal H-bonding

Consequences for the preferred conformation of polyketide natural products


Alborixin R = Me; X-206 R = H

Analyze the conformation found in the crystal state of a bourgeanic acid derivative!
Internal H-Bonding
Me Me Me Me
Me R
OH
H
Me
OH O O O O O O
H OH OH Me
C Me OH Me OH O
Me Me
OR O OH
OH
Et
Me Me Me
Bourgeanic acid

Metal ion ligation sites (M = Ag, K)

Me Me Me Me
Me R
OH
H
Me
O O O O O
H OH OH Me
C Me OH Me OH O Me
O O
OH
M Et
D. A. Evans Conformational Analysis: Ionophore X-206/X-rays Chem 206

X-ray of Ionophore X-206 ! H2O X-ray of Ionophore X-206 - Ag+ - Complex


Internal H-Bonding
Me Me Me Me
Metal ion ligation sites (M = Ag, K)
Me
OH
H Me Me Me
Me Me R Me
O O O O O OH
H OH OH Me H
C Me OH Me OH O Me
Me
O OH O O O O O
OH H OH OH Me
Et C Me OH Me OH O Me
O O
OH
M Et

"The Total Synthesis of the Polyether Antibiotic X-206". Evans, D. A.; Bender,
S. L.; Morris, J. J. Am. Chem. Soc. 1988, 110, 2506-2526.
D. A. Evans The Gauche Effect Chem 206

The 1,2-Dihaloethanes Alabugin & Zeidan, JACS 2002, 124, 3175 (pdf)
X H
H H H H X = Cl; !H° = + 0.9–1.3 kcal/mol
C C X = Br; !H° = + 1.4–1.8 kcal/mol
H H H X X = F; !H° = – 0.6-0.9 kcal/mol
X X
Observation: While the anti conformers are favored for X = Cl, Br, the gauche
conformation is prefered for 1,2-difluroethane. Explain.

Relevant Article: Chem. Commun 2002, 1226-1227 (pdf)

!-anti-bonding States: (C–X)


For the latest views, read
CH3–H Alabugin & Zeidan, JACS 2002, 124, 3175 (pdf)
CH3–CH3
CH3–NH2

CH3–OH

CH3–F
Increasing !"-acceptor capacity best acceptor

Your Thoughts on the trend shown below:


!-anti-bonding States: (C–X)
For the latest views, read
Alabugin & Zeidan, JACS 2002, 124, 3175 (pdf) The 1,2-Dihaloethanes
CH3–F
X H
CH3–Cl H H H H X = Cl; ΔH° = + 0.9–1.3 kcal/mol
CH3–Br best acceptor C C X = Br; ΔH° = + 1.4–1.8 kcal/mol
H H H X X = F; ΔH° = – 0.6-0.9 kcal/mol
Increasing !"-acceptor capacity X X
D. A. Evans Stabilized Eclipsed Conformations in Simple Olefins Chem 206

Simple olefins exhibit unusal conformational Butane versus 1-Butene


properties relative to their saturated counterparts Me
H Me H Me
staggered H Me eclipsed
Propane versus Propene C C
conformation H H conformation
109° 120° H
H Me H CH2 H H
! G° = +4 kcal mol-1
H H H
H H Me
H H Me
CH2 CH2
staggered C
H eclipsed
C
Hybridization change opens up the C–C–C bond angle conformation H H conformation
H H H
" = 50 "=0
H ! G° = –0.83 kcal mol-1
! The Propylene Barrier CH2 staggered
C conformation The Torsional Energy Profile
H H
! = 50 ! = 180
H Me
CH2 +2.0 kcal/mol H
H H Me
C H C C H
eclipsed C C H H
H H H
conformation H H
H

!=0 H
New (de)stabilizing effect H +1.33
C C H kcal
H Me Me
H +1.32 kcal
H
H
C C H
H H H ! = 120
+0.49 kcal
X C H H
H X C H !=0 ! = 180
Conforms to ab initio (3-21G) values:
H Wiberg, K. B.; Martin, E. J. Am. Chem. Soc. 1985, 107, 5035.
H
stabilizing conjugation between ! Acetaldehyde exhibits a similar conformational bias
!"–C–X & #–C–H
K. Wiberg, JACS 1985, 107, 5035-5041 O O O O
K. Houk, JACS 1987, 109, 6591-6600 H Me H Me
H H Me Me
H H H H H H H H

The low-energy conformation in each of above cases is eclipsed


Evans, Duffy, & Ripin Conformational Barriers to Rotation: Olefin A-1,2 Interactions Chem 206

5 5
1-butene 2-propen-1-ol

4 H H 4 H H
H H
C C H C C H
H H
! Me !
E (kcal/mol)

OH

E (kcal/mol)
3 3

2
2

1
1

0
-180 -90 0 90 180 0
! (Deg) -180 -90 0 90 180
! (Deg)
The Torsional Energy Profile
The Torsional Energy Profile H
! = 50 ! = 180 H OH
C C H
Me H H
H Me
H C C H H
C C H
H H H ! = 60 ! = 180
H H HO
H
!=0 H C C H
H H
H
! = 120
+1.33 H OH
C C H
kcal +2.00
H Me Me kcal
H +1.32 kcal H
C C H
H !=0 H H
C C H H
H H +1.18 kcal H
+0.49 kcal ! = 120 H
H C C H
!=0 ! = 180 HO
H +0.37 kcal
Conforms to ab initio (3-21G) values:
Wiberg, K. B.; Martin, E. J. Am. Chem. Soc. 1985, 107, 5035. !=0 ! = 180
Evans, Duffy, & Ripin Conformational Barriers to Rotation: Olefin A-1,2 Interactions-2 Chem 206

5
2-methyl-1-butene 5
2-methyl-2-propen-1-ol
4 H H
H 4 H H
C C Me H
H C C Me
H
!
E (kcal/mol)

3
Me
! OH

E (kcal/mol)
3

2
2

1
1

0
0
-180 -90 0 90 180
-180 -90 0 90 180
! (Deg) ! (Deg)

The Torsional Energy Profile


H The Torsional Energy Profile
Me
! = 180 H C C Me H
OH
H H
! = 180 C C Me
H H
! = 50 ! = 60 H
Me HO

H H
C C Me C C Me
H H H
H
H H ! = 120
! = 110 +2.68 OH +2.01
!=0 kcal kcal
H +1.39 kcal Me H
!=0 +1.16 kcal C C Me
H H H H
H
C C Me C C Me H
H Me H H
H C C Me
H H HO
H +0.21 kcal
+0.06 kcal H
!=0 ! = 180 !=0 ! = 180
Evans, Duffy, & Ripin Conformational Barriers to Rotation: Olefin A-1,3 Interactions Chem 206

5 5
(Z)-2-pentene (Z)-2-buten-1-ol
4 4 H H
Me
C C H
H H H
Me
C C H ! OH
E (kcal/mol)

E (kcal/mol)
3 H 3

! Me

2 2

1 1

0 0
-180 -90 0 90 180 -180 -90 0 90 180

! (Deg) ! (Deg)

H The Torsional Energy Profile The Torsional Energy Profile


Me
C C H !=0 !=0 H
H Me
H
Me
C C H
H HO ! = 180
H
H
Me OH
C C H
H
! = 180
+3.88 kcal H
H
! = 90 Me Me +1.44 kcal ! = 120
C C H
Me H OH
+0.86
Me H Me kcal
C C H C C H
H H H
+0.52
H H H
kcal
!=0 ! = 180 !=0 ! = 180
Values calculated using MM2 (molecular mechanics) force fields
via the Macromodel multiconformation search. Review: Hoffman, R. W. Chem. Rev. 1989, 89, 1841.
D. A. Evans Acyclic Conformational Analysis-2 Chem 206

! Problems of the Day:


http://www.courses.fas.harvard.edu/colgsas/1063
Can you predict the stereochemical outcome of this reaction?

O OTs OLi OTs


Chemistry 206 Me
EtO Me
LiNR2 EtO 1 + 2
Advanced Organic Chemistry n-C4H9
H n-C4H9 98:2
H
Lecture Number 5
D. Kim & Co-workers, Tetrahedron Lett. 1986, 27, 943.
Acyclic Conformational Analysis-2

OH
! Introduction to Allylic Strain CH2OBn B2H6 " CH2OBn
O O "
H2O2
Me Me Me Me
diastereoselection 8:1
! Reading Assignment for week
Y. Kishi & Co-workers, J. Am. Chem. Soc. 1979, 101, 259.
A. Carey & Sundberg: Part A; Chapters 2 & 3
R. W. Hoffmann, Angew. Chem. Int. Ed. Engl. 2000, 39, 2054-2070
Conformation Design of Open-Chain Compounds (handout) Me Me
Me Me
R. W. Hoffmann, Chem. Rev. 1989, 89, 1841-1860 PhNCO " "
Allylic 1-3-Strain as a Controlling Element in Stereoselective Transformations O
(handout) Et3N
N
NO2
only one isomer

A. Kozikowski & Co-workers, Tetrahedron Lett. 1982, 23, 2081.


Wednesday,
D. A. Evans September 27, 2006
We call this
Allylic Strain
D. A. Evans Acyclic Conformational Analysis: Allylic Strain Chem 206

The Definition of Allylic Strain Can you predict the stereochemical outcome of this reaction?
F. Johnson, Chem. Rev. 1968, 68, 375; Allylic Strain in Six-Membered Rings D. Kim & Co-workers, Tetrahedron Lett. 1986, 27, 943.
R. W. Hoffmann, Chem. Rev. 1989, 89, 1841-1860 (handout)
Allylic 1-3-Strain as a Controlling Element in Stereoselective Transformations
O OTs OLi OTs
Houk, Hoffmann JACS 1991, 113, 5006 Me
EtO Me
LiNR2 EtO
R2 3 R1 1 + 2
Consider the illustrated general structure Y n-C4H9 n-C4H9
where X & Y are permutations of C, N, and O: H H 98:2
X R large
R3 2 1 O
Me
Typical examples: R small ! Relevant enolate EtO
conformations 1 major
R2 R1 R2 R1 R2 R1 R2 R1 n-C4H9
H
! R large N ! R large N ! R large – N ! R large
R3 R + O +
R small R small R small R small (CH2)4OTs (CH2)4OTs
TsO(H2C)4 H OR Bu OR
OR
Olefin Imine Imonium ion Nitrone Me C C Me C C Me C C
OLi OLi OLi
Bu H
In the above examples, the resident allylic stereocenter (!) and its associated A1 Bu B1 C1 H
substituents frequently impart a pronounced bias towards reactions occuring at
the pi-bond. critical conformations
A(1,3) H Bu
interaction H Bu
Nonbonding interactions between the allylic R2 3 R1 OR OR OR
substituents (Rlarge, Rsmall) & substituents at Y Me C
Me C C C Me C C
the 2- & 3-positions play a critical role in X R large OLi OLi OLi
R3 2 TsO(H2C)4 Bu H (CH2)4OTs
defining the stereochemical course of such 1
(CH2)4OTs
reactions R small
A(1,2) A2 B2 C2
interaction
Representative Reactions controlled by Allylic Strain Interactions
HO HO Me
H H EtO2C
O R O R
Hg(OAc)2 2 minor
Me NaBH4 HO Me n-C4H9
OBn OBn H
diastereoselection 10:1

M. Isobe & Co-workers, Tetrahedron Lett. 1985, 26, 5199.


D. A. Evans Allylic Strain & Enolate Diastereoface Selection Chem 206

O OTs O
Me Ph OM Ph O
Me
EtO LiNR2 EtO NH4Cl
diastereoselection 98:2 Me3Si OMe Me3Si OMe
n-C4H9 n-C4H9 R R
H H R-substituent diastereoselection

R = Me 87:13
O O
Me R = Et 80:20
R = CHMe2 40:60 major diastereomer opposite
EtO LiNR2
EtO to that shown
diastereoselection 89:11 I. Fleming & Co-workers, Chem. Commun. 1985, 318.
MeI n-C4H9
Y. Yamamoto & Co-workers, Chem. Commun. 1984, 904.
n-C4H9 H
H
Ph O Ph O
D. Kim & Co-workers, Tetrahedron Lett. 1986, 27, 943. LiNR2 H
Me3Si OBn Me–CHO Me3Si OBn diastereoselection 90:10 at C3
one isomer at C2
71% yield Me OH
MeO MeO
O O I. Fleming & Co-workers, Chem. Commun. 1986, 1198.
RO2C LiNR2 RO2C
H "one isomer"
Br Me O S Me O S
H H H
95% yield Bn Sn(OTf)2 Bn
CO2Me CO2Me N N S N N S diastereoselection >95%
R–CHO
G. Stork & Co-workers, Tetrahedron Lett. 1987, 28, 2088. Boc Boc
91-95% R OH

T. Mukaiyama & Co-workers, Chem. Letters 1986, 637

Me CH2 Me CH2 O Me O
TBSOCH2 TBSOCH2 Ph(MeS)2C–Li MeS
LiNR2 MeS
Me OMe Me–I OMe diastereoselection 99:1
"one isomer"
MeI Me H 86% MeS Me
H H
CO2Me K. Koga & Co-workers, Tetrahedron Letters 1985, 26, 3031.
CO2Me
T. Money & Co-workers, Chem. Commun. 1986, 288.
I
OLi
CO2Et H
R CO2Et
R O O-t-Bu
R OM R = H: one isomer
MeI KOt-Bu CO2-t-Bu R = Me: > 15 :1
PhMe2Si OEt PhMe2Si OEt THF -78 °C H
R
Me R = Me: diastereoselection 99:1
R = Ph: diastereoselection 97:3 Y. Yamaguchi & Co-workers, Tetrahedron Letters 1985, 26,1723.

I. Fleming & Co-workers, Chem. Commun. 1984, 28.


D. A. Evans Allylic Strain & Olefin Hydroboration Chem 206
! The basic process Hydroborations dominated by A(1,3) Strain
S S ‡ OH
CH2OBn B2H6 CH2OBn
H B H O O
H B H H2B H H2O2
H Me Me Me Me
H
R C C R
R R R R diastereoselection 8:1
C C C C R R
R R R R OMe OMe OH
Me B2H6
O O OH
H2O2
Me Me Me Me Me
OH
diastereoselection 12:1
A
Oxidant Ratio, A:E Reference Y. Kishi & Co-workers, J. Am. Chem. Soc. 1979, 101, 259.
CH2
MCPBA 69:31 JOC, 1967, 32, 1363
Me3C OH
BH3, H2O2 34:66 JOC, 1970, 35, 2654
B2H6
E BnO OH BnO OH
H H2O2
Me Me Me Me Me Me
! Acyclic hydroboration can be controlled by A(1,3) interactions:
Diastereoselection = 3:1
OH
C. H. Heathcock et. al. Tetrahedron Lett 1984 25 243.
RL R2BH RL
OH OH major diastereomer
H2O2
RM Me RM Me
R
Me Me
R Me Me
B H ThexylBH2,
A(1,3) allylic strain RM H TrO OTr
Me
control elements Steric effects; RL vs RM H C C then BH3 OH OH OH
CH2OR OH
Staggered transition states TrO OTr
Diastereoselection; 5:1
RL

OH
Me Me Me Me ThexylBH2, Me Me Me Me
RL Me R2BH RL
OH major diastereomer
H2O2 then BH3 TrO OTr
RM RM Me
OH OH OH OH OH OH OH OH OH
R TrO OTr
R Diastereoselection; 4: 1
B H
RM H
CH2OR Still, W.C.; Barrish, J. C. J. Am. Chem. Soc. 1983, 105, 2487.
See Houk, Tetrahedron 1984, 40, 2257 H C C
Me
RL
D. A. Evans Allylic Strain & Amide Conformation Chem 206

Consider the resonance structures of an amide: R2 3 R1 The selection of amide protecting group may be done with the knowledge that
Y altered conformational preferences may result:
O R1 –O R1 X R large O
C C R3 O
2 1 H H
N R N R R small
R3 R3 + Favored for Favored for
1 1
R = H, alkyl R = COR
R R N N
H H
A(1,3) interactions between the "allylic substituent" and the R1 moiety will R R
strongly influence the torsion angle between N & C1. H
O O
Favored N H Disfavored
O Me N
C R
N Me
H H R
Me H
O
Me O
Disfavored H Favored
N N
O C C R
! conformations of cyclic amides H H
R O
–O R R
C H
R R
R N N N 2 2
+ R C R C O 1 –O 1
A(1,3) interaction between the C2 & amide C R C R
R H
O O strongly favored substituents will strongly influence the torsion
angle between C1 & C2. N N
A(1,3) R R R + R

H Me
Chow H Me As a result, amides afford (Z) enolates under all conditions
Can. J. Chem. 1968, 46, 2821 Me H
N N H ‡
R C R C H
Me H OM
O O strongly favored L L
O C N base O C N Me L
O O N
Me H L Me H L
favored
published X-ray structure of this amide shows chair H L
Me N Me diaxial conformation
(Z)-Enolate
Ph O Quick, J. Org. Chem. 1978, 43, 2705 H ‡
H
OM
" Problem: Predict the stereochemical outcome of this cyclization. L base L
O C N O C N H L
OH H H Me L N
HOCO ! disfavored H Me L
HCO2H ! Me L
D. Hart, JACS 1980, 102, 397 N N identify HOMO-LUMO pair (E)-Enolate

Ph O Ph O
diastereoselection >95%
D. A. Evans Allylic Strain & Amide Conformation Chem 206

A(1,3) Strain and Chiral Enolate Design Polypropionate Biosynthesis: The Acylation Event
O O M O
O O O O OH O
Me LDA
N Me Acylation Reduction
O
or NaNTMS2 N O enolization selectivity R SR
R SR R SR
>100:1 – CO2
Bn O O Me Me
Bn
HO SR
‡ El(+)
H Me
JACS. 1982,104, 1737.
O O
L
O C N Me First laboratory analogue of the acylation event
Me H L N O
Li
El O O O O
O
Bn O !
favored Me
N O N O
enolization geometry Et Cl
Me Me
! In the enolate alkylation process product epimerization is a serious R R
problem. Allylic strain suppresses product enolization through the
intervention of allylic strain with M. Ennis JACS 1984, 106, 1154. Diastereoselection ~ 97 : 3
H
El
L L Me El
O C N O L Why does'nt the acylation product rapidy epimerize at the exocyclic
C N O C N
El Me L stereocenter??
Me H L L
B A C H R H
R R
While conformers B and C meet the stereoelectronic requirement for O C N O C N
enolization, they are much higher in energy than conformer A. Further, as R R
Me H R Me
deprotonation is initiated, A(1,3) destabilization contributes significantly to
favored
reducing the kinetic acidity of the system

These allylic strain attributes are an integral part of the design criteria of
chiral amide and imide-based enolate systems
O O
O CH2OH O Me
Me
Me N O Me
N N
Bn Me OH

Evans Evans Myers


Tetr Lett. 1977, 29, 2495 JACS 1982,104, 1737. JACS 1997, 119, 6496 X-ray structure
D. A. Evans Discodermolide Chem 206

hinge

Me Me Me
16
HO
Me 17

O O H OH O NH2
Me Me
OH O
Me Me
OH
- immunosuppressive activity
- potent microtubule-stabilizing agent
(antitumor activity similar to that of taxol)

The epimers at C16 and C17 have no or almost no biological activity.

The conformation about C16 and C17 is critical to discodermolide's biological activity.

S. L. Schreiber et al. JACS 1996, 118, 11061.


D. A. Evans Conformational Analysis - Discodermolide X-ray 1 Chem 206

Me Me Me
HO
Me
O O H OH O NH2
Me Me
OH O
Me Me
OH
D. A. Evans Conformational Analysis - Discodermolide X-ray 2 Chem 206

Me Me Me
16
HO
Me
O O H OH O NH2
Me Me
OH O
Me Me
OH

16
D. A. Evans Conformational Analysis: Part–3 Chem 206

http://www.courses.fas.harvard.edu/colgsas/1063
Conformational Analysis of Cyclic Systems
Three Types of Strain:
Chemistry 206
Prelog Strain: van der Waals interactions
Advanced Organic Chemistry Baeyer Strain: bond angle distortion away from the ideal

Lecture Number 6 Pitzer Strain: torsional rotation about a sigma bond

Baeyer Strain for selected ring sizes


Conformational Analysis-3
size of ring Ht of Combustion Total Strain Strain per CH2 "angle strain"
(kcal/mol) (kcal/mol) (kcal.mol) deviation from 109°28'
! Conformational Analysis of C4 ! C6 Rings 27.5
3 499.8 9.17 24°44'
4 656.1 26.3 6.58 9°44'
5 793.5 6.2 1.24 0°44'
! Reading Assignment for week 6 944.8 0.1 0.02 -5°16'
7 1108.3 6.2 0.89
A. Carey & Sundberg: Part A; Chapter 3 8 1269.2 9.7 1.21
9 1429.6 12.6 1.40
10 1586.8 12.4 1.24
Eliel & Wilen, "Stereochemistry of Organic Compounds, "Chapter 11, 1743.1 11.3 1.02
11
Configuration and Conformation of Cyclic Molecules, Wiley, 1994 12 1893.4 4.1 0.34
13 2051.9 5.2 0.40
Ribeiro & Rittner, "The Role of Hyperconjugation in the Conformational 14 2206.1 1.9 0.14
Analysis of Methylcyclohexane and Methylheterocyclohexanes" 15 2363.5 1.9 0.13
J. Org. Chem., 2003, 68, 6780-6787 (handout) Eliel, E. L., Wilen, S. H. Stereochemistry of Organic Compounds
Chapter 11, John Wiley & Sons, 1994.
Stability Relationships in Bicyclic Ketones,
Synlett 2005, 2911-2914 (handout) ! Baeyer "angle strain" is calculated from the deviation of the
planar bond angles from the ideal tetrahedral bond angle.
The Application of Cyclobutane Derivatives in Organic Synthesis
Chem Rev. 2003, 103, 1485-1537 (pdf) ! Discrepancies between calculated strain/CH2 and the "angle
de Meijere, "Bonding Properties of Cyclopropane & their Chemical strain" results from puckering to minimize van der Waals or
Characteristics" eclipsing torsional strain between vicinal hydrogens.
Angew Chem. Int. Ed. 1979, 18, 809-826 (pdf)
Problem: Rationalize the regioselectivity of the following reduction
The Application of Cyclobutane Derivatives in Organic Synthesis H H
Chem. Rev. 2003, 103, 1485-1537 (pdf)
O NaBH4 O
O OH
Friday,
D. A. Evans September 29, 2006 Stork, JACS, 1979, 7107.
Evans, Kim, Breit Cyclopropane: Bonding, Conformation, Carbonium Ion Stabilization Chem 206

Carbocation Stabilization via Cyclopropylgroups


Cyclopropane

H ! Necessarily planar.
! Subtituents are therefore eclipsed. C
H ! Disubstitution prefers to be trans.

H
" = 120 °
H ! Almost sp2, not sp3 Me
A rotational barrier of about NMR in super acids
! = 3080 cm-1 Me
13.7 kcal/mol is observed in !(CH3) = 2.6 and 3.2 ppm
following example:
H

Walsh Model for Strained Rings:


! Rather than ! and !* c-c bonds, cyclopropane has sp2 and p-type
orbitals instead.

! (antibonding) ! (antibonding)

" (antibonding)
H
3 Nonbonding
H

side view

" (bonding) " (bonding)

!–1 (bonding)
de Meijere, "Bonding Properties of Cyclopropane & their Chemical Characteristics"
Angew Chem. Int. Ed. 1979, 18, 809-826 (handout)
de Meijere, A.; Wessjohann, L. "Tailoring the Reactivity of Small Ring Building
Blocks for Organic Synthesis." Synlett 1990, 20. (pdf)
Evans, Kim, Breit Conformational Analysis: Cyclic Systems-2 Chem 206

Cyclobutane Cyclopentane
145-155° H H
H H
H H
ax H H
ax ! Eclipsing torsional strain overrides H
H
H H H
H H
eq increased bond angle strain by puckering. H
H H H
eq eq H H H H
eq H H
ax ! Ring barrier to inversion is 1.45 kcal/mol. H H H H H
! = 28 ° ax
CsEnvelope C2 Half-Chair CsEnvelope

! Two lowest energy conformations (10 envelope and 10 half chair conformations
Cs favored by only 0.5 kcal/mol) in rapid conformational flux (pseudorotation)
which causes the molecule to appear to have a single out-of-plane atom "bulge"
which rotates about the ring.
(MM2)
! Since there is no "natural" conformation of cyclopentane, the ring conforms to
minimize interactions of any substituents present.

H
H H
CsEnvelope H
H
(MM2) H
H H H

! A single substituent prefers the equatorial position of the flap of the envelope
(barrier ca. 3.4 kcal/mol, R = CH3).
! !G = 1 kcal/mol favoring R = Me equatorial
X
! 1,2 Disubstitution prefers
! 1,3 Disubstitution prefers cis diequatorial to trans for steric/torsional
trans by 0.58 kcal/mol for di-bromo cmpd. reasons (alkyl groups) and
dipole reasons (polar groups).

Me X

Me ! 1,3 Alkyl Disubstitution: Cis-1,3-dimethyl


! 1,2 Disubstitution prefers trans diequatorial to cyclopentane 0.5 kcal/mol more stable than trans.
H H
cis by 1.3 kcal/mol for diacid (roughly equivalent
to the cyclohexyl analogue.) ! A carbonyl or methylene prefers the planar position of
the half-chair (barrier 1.15 kcal/mol for cyclopentanone).
X
Evans, Kim, Breit Conformational Analysis: Cyclic Systems-3 Chem 206

Methylenecyclopentane and Cyclopentene


Strain trends:
! Decrease in eclipsing strain
more than compensates for the
! > > increase in angle strain.

Relative to cyclohexane derivatives, those of cyclopentane prefer an sp2


center in the ring to minimize eclipsing interactions.

"Reactions will proceed in such a manner as to favor the formation or retention


of an exo double bond in the 5-ring and to avoid the formation or retention of
the exo double bond in the 6-ring systems." Brown, H. C., Brewster, J. H.;
Shechter, H. J. Am. Chem. Soc. 1954, 76, 467.
Examples: H
H O H
NaBH4
OH
H k6 H
H H
H H H k6
H = 23
H k5
NaBH4 H H
O
H k5
OH
H H

Brown, H. C.; Ichikawa, K. Tetrahedron 1957, 1, 221.

Problem: Rationalize the regioselectivity of the following


O O reduction
O O
H hydrolyzes H
13 times faster than
O NaBH4 O
O OH
Conan, J-Y.; Natat, A.; Priolet, D. Bull. Soc. Chim., Fr. 1976, 1935.
Stork, JACS, 1979,
OH 7107.
O
O O

OEt
OEt

95.5:4.5 keto:enol 76:24 enol:keto

Brown, H. C., Brewster, J. H.; Shechter, H. JACS 1954, 76, 467.


"Total Synthesis of the Antifungal Macrolide Antibiotic (+)-Roxaticin," Evans, D. A.; Connell, B. T.
J. Am. Chem. Soc., 2003, 125, 10899-10905

Me Me Me Me
O O OTBSO O O O OTBSO O
Me Me
22 18 18
27 22
Me 27
O XO Me
O XO
Me Me
O 12 OX O 12 OX
X = CMe2 Me Me
X = C(CH2)4

<10% PPTS, rt, MeOH. 63% PPTS, rt, MeOH.

OH OH OH OH OH
Me
22
27
16
Me2CH O HO
12

O 2 OH
Me
Roxiticin

O O O O
hydrolyzes
13 times faster than

Conan, J-Y.; Natat, A.; Priolet, D. Bull. Soc. Chim., Fr. 1976, 1935.
HO OH
Zaragozic Acid C Synthesis OH
OH
CO2H OH
5
7

HO 1 R 5 H
7 HO2C
J. Leighton, J. Barrow 3 O 1 R
OH HO2C O 3
JACS 1994, 116, 12111-12112 O CO2H OH O
HO CO2H

tBuO C
2 CO2tBu tBuO C CO2tBu
O MgBr
2 tBuO C CO2tBu
OBn O OBn 3 steps 2
O
OTBS
O 7
5 O 5
3
CH2Cl2:THF 3 86% O 1
H 1 H 7 3 5
O –78 °C 76% HO H O O
7 OBn
TBSO Ph OTBS Ph H
Chelate Control

91% 3 steps
tBuO C
2 CO2tBu OTBS
OPMB O 7
Li Bn O tBuO C CO2tBu OTBS
2 steps 2
O
H O O 7
O 1
Me tBuO C
2 70% O
3 5
O H O 1 O
65% tBuO C OBn
Ph 2

Me

tBuO C
2 CO2tBu OTBS Me
O
OH OR Ph O Zaragozic acid C
O
5
H O Bn 94%
tBuO C O O OH
2 7
O OAc
Me
Ph 5 H
0:10:1 CH2Cl2:TFA:H2O, 18 h HO2C
Me O 1 4' Ph
HO2C
3 O
R = PMB R = Ac (89%) CO2H Me
HO
Evans, Breit Conformational Analysis: Cyclic Systems-4 Chem 206

Monosubstituted Cyclohexanes: A Values A Values depend on the relative size of the particular substituent.
R H
Keq H H H Me H Me Me Me
H H Me Me
H R !G° = –RTlnKeq

H H H H
! Me–axial has 2 gauche butane interactions more than Me–equatorial.
Expected destabilization: ! 2(0.88) kcal/mol = ~1.8 kcal/mol; A–Value 1.74 1.80 2.15 5.0
Observed: 1.74 kcal/mol
Me Me
H Me H The "relative size" of a substituent and the associated A-value may not correlate.
H
C H C For example, consider the –CMe3 and –SiMe3 substituents. While the –SiMe3
C H
H H substituent has a larger covalent radius, it has a smaller A-value:
H H
! The A– Value, or -!G°, is the preference of the substituent for the Me Me Me
equatorial position. Me Me Me
C Me Si Me Sn Me

H H H

A–Value 4.5-5.0 2.5 1.1

Can you explain these observations?

! The impact of double bonds on A-values:


Lambert, Accts. Chem. Res. 1987, 20, 454

R H

H R

A-value
substituent !"G° (cyclohexane)

R = Me 0.8 1.74
R = OMe 0.8 0.6
R = OAc 0.6 0.71
The Me substituent appears to respond strictly to the decrease in nonbonding
interactions in axial conformer. With the more polar substituents, electrostatic
effects due to the trigonal ring carbon offset the decreased steric environment.
Evans, Breit Conformational Analysis: Cyclic Systems-5 Chem 206

Impact of Trigonal Carbon Polysubstituted Cyclohexane A Values


! Let's now compare look at the carbonyl analog in the 3-position
! As long as the substituents on the ring do not interact in either
Me H conformation, their A-values are roughly additive

H Me 1,4 Disubstitution: A Values are roughly additive.


O O
Me Me
!G° = –1.36 kcal/mol !G° = 0 kcal/mol
versus –1.74 for cyclohexane Me Me

Me
! Let's now compare look at the carbonyl analog in the 2-position Me
Me
Me base H
epimerization Me !G° = –2(1.74) = –3.48 kcal/mol
Me3C H Me3C Me

O O
!G° = –1.56 kcal/mol 1,3 Disubstitution: A Values are only additive in the trans diastereomer
versus –1.74 for cyclohexane
X
H
H X
However, the larger alkyl groups do not follow the expected trend. H !G° = A(Me) – A(X)
Can you explain? (see Eliel, page 732) H
Me Me
CHMe2 base H
epimerization The cis Isomer
H
Me3C H Me3C CHMe2 H
H H
O O X

!G° = –0.59 kcal/mol Me Me X


versus –2.15 for cyclohexane The new interaction!

CMe3 base H For X = Me H


epimerization H
H
Me3C H Me3C CMe3
Me H
H
O O
Me H
+ 0.88 + 0.88
!G° = –1.62 kcal/mol Me Me
versus –5.0 for cyclohexane
!G° = 2(.9) + 1(+3.7)= 5.5 kcal/mol
+ 3.7
Evans, Breit Conformational Analysis: Cyclic Systems-6 Chem 206

Let's now consider geminal substitution Let's now consider vicinal substitution
Me
Me
Me
Ph H
Case 1: H
Ph H Me
Me H
The prediction: !G° = A(Ph) – A(Me) Me

!G° = +2.8 – 1.7 = +1.1 kcal/mol The prediction: !G° = 1 gauche butane – 2A(Me)
!G° = +0.88 – 2(1.74) = +2.6 kcal/mol
Observed: !G° = –0.32 kcal/mol
Observed: !G° = +2.74 kcal/mol
If the added gauche butane destabilization in the di-equatorial
conformer had not been added, the estimate would have been off.

Case 2:
Me
H Me
OH H

H
OH Me
H Me
H
H
The conformer which places the isopropyl group equatorial is much more
strongly preferred than would be suggested by A- Values. This is due to
a syn pentane OH/Me interaction.

Problem:
Can you rationalize the stereochemical outcome of this reaction?
O O
Me

EtO EtO
LiNR2
MeI n-C4H9
n-C4H9
H H

diastereoselection 89:11

D. Kim & Co-workers, Tetrahedron Lett. 1986, 27, 943.


Evans, Breit Conformational Analysis: Cyclic Systems-7 Chem 206
Heteroatom-Substituted 6-Membered Rings A-Values for N-Substituents in Piperidine
! A-values at the 2-position in both the O & N heterocycles are larger than
expected. This is due to the shorter C–O (1.43 Å), and C–N (1.47 Å) bond H The Reference:
lengths relative to carbon (C–C; 1.53 Å). The combination of bond length and N N
bond angle change increases the indicated 1,3-diaxial interaction (see eq 1, 4). H !G° = –0.36 kcal/mol

Me
Me Reference: H
N N
Me !G° = –3.0 kcal/mol
H Me !"G° = 1.74 kcal/mol

! Hydrogen is "bigger" than the N–lone Pair.


! The A-value of N–substituents is slightly larger than the corresponding
Me H cyclohexane value. Rationalize
H
(1) H Me !"G° = 2.86 kcal/mol
O O

Me H
Me H
H
H Me !"G° = 1.43 kcal/mol
(2)
(4) O H O Me !"G° = 2.5 kcal/mol
N H N H

Me H

H Me !"G° = 1.95 kcal/mol


(3) O O

Me H
H
(4) H Me !"G° = 2.5 kcal/mol
N H N H

Me H

N H N Me !"G° = 1.6 kcal/mol


(5)
H H
Me H

H Me !"G° = 1.9 kcal/mol


(6) H N H
N
Evans, Breit Conformational Analysis: Bicyclic Ring Systems Chem 206

Estimate the energy difference between the two methyl-decalins


shown below.
Me Me

H H

Hydrindane Ring System (6/5)


H H

flexible rigid

Decalin Ring System (6/6) H H


!G° = –0.5 kcal/mol (at 23 °C)
H H !G° = 0.0 kcal/mol (at ~200 °C)
mobile H rigid
! The turnover to favor the cis fusion results from the entropic preference for the
H less ordered cis isomer.
The 5-5 Ring System

H H H

H favored

H H
2.4 kcal/mol 0 Relative !G°
!G° = +6.4 kcal/mol
Let's identify the destabilizing gauche butane interactions in the cis isomer
H H
H Me H R
3
Gauche-butane interactions Me H R
C D
C D
H C1 ! C2 A B
A B
H H H
4 2 C1 ! C3 H H
A/B Trans H
C4 ! C3 A/B Cis
1
"G°(est) = 3(0.88) = 2.64 kcal/mol
Rationalize the conformational flexibility of a A/B Trans vs. A/B Cis Steroid!
Evans, Breit Conformational Analysis: Axial vs Equatorial Reactivity Chem 206

Different reactivity for axial and equatorial substituents ! SN2 Reactions (Displacement with Ph–S–)
OTs
Axial substituents are more hindered, thus less reactive in many H
transformations Me3C Me3C
OTs H

! Acetylation with Ac2O/Py k rel 1 31


H OH

OH H

k rel 1 0.13
H OH
The axial diastereomer is not always slower reacting:
Me3C OH Me3C H
! Alcohol Oxidation with Cr(6+)
k rel
1 0.27 H OH

Me3C OH Me3C H
! Acid-catalyzed esterification k rel
1 3.2
H CO2H
Me H Me OH
CO2H H
Me OH
k rel 1 0.04 Me H
Me
k rel Me
H CO2H 1 3.36
Me3C CO2H Me3C H The rate-determining step is breakdown of the chromate ester. This is an
k rel apparent case of strain acceleration
1 0.05

! Ester Saponification
For a more detailed discussion of this topic see:
H CO2Et Eliel, E. L., S. H. Wilen, et al. (1994). Stereochemistry of Organic
Compounds pp 720-726
Me3C CO2Et Me3C H

k rel 20 1
D. A. Evans Conformational Analysis: Part–4 Chem 206

http://www.courses.fas.harvard.edu/colgsas/1063 Problems of the Day:


Predict the stereochemical outcome of this reaction. The
diastereoselection is 99:1
Chemistry 206
O
Advanced Organic Chemistry H
mCPBA
H

Lecture Number 7 N N
Ph Ph
Conformational Analysis-4 O O

Martinelli, et.al. Tett. Lett. 1989, 30, 3935

! Ground State Torsional Effects (Conformational Transmission)


Rationalize the stereochemical outcome of this reaction.
! Conformational Analysis of C6 ! C8 Rings continued
Me
Me O CO2Me Me O CO2Me
! Transition State Torsional Effects LiNR2

Me O Me-I Me O
Me Me
diastereoselection is 8:1.
! Reading Assignment for week
Ladner, et.al. Angew. Chemie, Int. Ed 1982, 21, 449-450
A. Carey & Sundberg: Part A; Chapter 4
"Study & Descrption of Reaction Mechanisms"

K. Houk, Science. 1986, 231, 1108-1117 Which is the more stable C=C isomer in the two THC structures?
Theory & Modeling of Stereoselective Organic Reactions (handout)
Me Me
Still, W. C.; Galynker, I. Tetrahedron 1981, 37, 3981(handout)
OH OH

Me Me
Monday, O C5H11 O C5H11
D. A. Evans Me Me
October 2, 2006
R. W. Kierstead, JACS 1967, 89, 5934
Evans, Breit Conformational Analysis: Cylcoheptane Chem 206

Cycloheptane Cyclooctane

Chair-Boat 5
Lowest-energy conformation

eclipsed 3
1
Chair (2.16 kcal/mol) Twist-Chair (0 kcal/mol) Transannular strain
between C3 & C7

Ring strain originates in eclipsing interactions and transannular


vanderWaals interactions

Methylene position 1 2 3 4 5
pseudoequatorial Me
!G 1.8 2.8 >4.5 -0.3 6.1
pseudoaxial Me (kcal/mol)
Boat (3.02 kcal/mol) Twist-Boat (2.49 kcal/mol)
Underside view of boat-chair C3 & C7 eclipsing interactions
Hendrickson, J. B. JACS 1961, 83, 4537.
See Eliel, page 762+

7
Olefins preferentially oriented to eliminate eclipsing interactions.

1 7

3 3
Evans, Breit Conformational Analysis: Cyclooctane Chem 206

Cyclooctane continued...
5
Chair-Boat (BC)
Lowest-energy conformation

1 3
Transannular strain
between C3 & C7
Chair-Boat (CB) conformation Chair-Chair (CC) conformation
reference structure (+1–1.6 kcal/mol)

Methyl position 1 2 3 4 5
(pseudoeqatorial)
!G 1.8 2.8 >4.5 -0.3 6.1
(pseudoaxial) (kcal/mol)

Cyclooctane derivatives

Boat-Boat (BB) conformation


(>+ 8 kcal/mol)
O
Carbonyl is positioned at C3 or C7 Olefin is positioned at C3-C4 or C6-C7

Me Me
Nu O O
X LINR2 Predict
! stereochemistry
MeI
Me

X Nu Me

O O
LiCuMe2 ! Predict
Disubstitution occurs at C4 or C6 SN2 occurs at C1 and C5
stereochemistry
Me Me
Still, W. C.; Galynker, I. Tetrahedron 1981, 37, 3981.
D. A. Evans Ground State Torsional Effects Chem 206

Torsional Effects Relevant Orbital Interactions:


Torsional Strain: the resistance to rotation about a bond
H
Torsional energy: the energy required to obtain rotation about a bond
H
Torsional Angle: also known as dihedral angle X C H
H X C H
Torsional steering: Stereoselectivity originating from transition state
torsional energy considerations H
H
stabilizing conjugation between
# C–H & ! electrons are !"–C–X & #–C–H
destabilizing
Dorigo, A. E.; Pratt, D. W.; Houk, K. N. JACS 1987, 109, 6591-6600.

ΔG = +3 kcal mol-1

Conformational Preferences: Acetaldehyde


staggered eclipsed
H H
Torsional Strain (Pitzer Strain): Ethane H H
A O C H O C H B

H
H H
CH2 staggered The eclipsed conformation (conformation A) is preferred.
C conformation Polarization of the carbonyl decreases the 4–electron destabilizing
H H Rotational barrier: 1.14 kcal/mol
H
H Houk, JACS 1983, 105, 5980-5988.
CH2 +2.0 kcal/mol
H
C
Conformational Preferences
eclipsed 1-Butene (X = CH2); Propanal (X = O)
conformation H
H
H H H
Wiberg K. B.; Martin, E. J. Amer. Chem. Soc. 1985, 107, 5035-5041. H Me Me
A X C H X C H X C H B

See Lecture 5 for previous discussion Me A' H H

Conformation A is preferred. There is little steric repulsion between the


methyl and the X-group in conformation A'.
D. A. Evans Ground State Torsional Effects: Conformational Transmission Chem 206
Observation:
Relative enolate stability correlates to ring junction stereochemistry
Let ! be defined as the torsion or dihedral angle for butane

Me !
H
A Me
Me
H H H
C B ! = 60° for butane
base H H
C Me
O LiO LiO H
H H H
Let's now consider cyclohexane
ratio: 13 : 87
H H H
! Perfect chair real chair
base H
C ! = 60° ! = 56°
O LiO LiO
H H H H H CCC" 109° 28' CCC" 111°
ratio: 70 : 30
H
House, JOC 1965, 30, 1341
Given cyclohexane with an identified torsion angle !R, if !R either increases
or decreases what effects in angle change are transmitted to !O, !M, and !P?
Observation:
Relative enolate stability seems to be correlated to position of C=C !R = 56° !R = 0° ["]
!O 56 15 –41°
H H H !R !P
!M 56 44 –11°
base
!O !M !P 56 61 +6°
O LiO LiO
H H H ["] = !R( 0°) – !R( 56°)
ratio: 10 : 90
H H H Operation:
56 56 15° 44°
base
–H2
56 56 0* 61°
O LiO LiO
H H H 56
ratio: 92 : 8 56
Hence, relative to cyclohexane, the following notation for torsion angle change
How do we explain the experimental observations shown above? may be denoted:

–11°
+6°
Readings: Velluz etal, Angew. Chemie, Int Ed. 1965, 4, 181-270 (pdf)

For !P For !M
D. A. Evans Ground State Torsional Effects: Conformational Transmission-2 Chem 206
Operation: trans-dimethylcyclohexane
H
Me R Me
H H H
56° 64° 56° C 64°
R Me base
Me
H O LiO LiO
Endocyclic dihedral angle is 56° H H H
Exocyclic dihedral expands correspondingly (+4°): 64° ratio: 10 : 90
effects opposing effects reinforcing
Operation: cis-dimethylcyclohexane
H
Me R H H H H
56° 56° 56° C
R Me
base
Me
Me O LiO LiO
H H H
trans-decalin cis-decalin 56° ratio: 92 : 8
A H B H effects reinforcing effects opposing
A B

60° 60° 56° 56°

H H Question: Which is the more stable C=C isomer in the two THC structures?
Simple Application: Reinforcing Torsional Effects Me Me

OH OH
Which C=C bond isomer below is more stable?
R
1) C=C will open up ring=B torsion angle Me Me
O C5H11 O C5H11
B 2) Ring B will resist increase in its ring fusion torsion angle Me Me

3) Therefore torsion effects are opposed R. W. Kierstead, JACS 1967, 89, 5934
R

R Question: Which enol acetate is more stable?


1) C=C will close down ring=B torsion angle
B 2) Ring B will accomodate decrease in its ring fusion torsion angle OBz OBz OBz
3) Therefore torsion effects are reinforcing Ac2O
R TSOH
or

O AcO AcO
D. A. Evans Transition State Torsional Effects According to Houk Chem 206

Houk: "Torsional effects in transition states are more important than in ground states"
Transition states
30 ° H*
0° H* H* H-radical and H-anion: antiperiplanar !"C–R orbital stabilized the TS
90 ° H illustrated for Nu addition
H H H
H
H2C C H H2C C H H2C C H
RL
H H H !"C-RL
H
lumo !"C-RL
C C
!C-Nu !C-Nu
0° 30° 60° 90° 120° homo
no H* Nu
+1.6
See lecture 5 Forming bond
+4.7 Forming bond
H+ Same trends are observed for H+ addition
Houk, Science 1981, 231, 1108-1117
2 Kcalmol-1 "The Theory and Modeling of Stereoselective Organic Reactions"
0
H• +2.4
0 Nu
H H
+5.3 H !"C-RL
H- Transition state C C H
H !C-Nu
0 RL

0° 30° 60° 90° 120° Nu


Houk, JACS 1981, 103, 2438 H H
!"C-RL
Houk, JACS 1982, 104, 7162 Ground state C
R H
RL
!C-Nu
D. A. Evans Transition State Torsional Effects: Olefin Additions Chem 206

! Olefin Addition Reactions: Case one


! Olefin Addition Reactions: Case two
How do we account for the high exo selectivities in How do we account for the high selectilvities in the oxidation
addition reactions to norbornene? of the indicated olefin?
exo
Highly exo selective for electrophilic, OH O
nucleophilic and cycloaddition reactions OsO4 mCPBA
OH H
H H
Rate enhancement due to strain H
H N N
N Ph
Ph
endo Ph O O
O 98 : 2 99 : 1
diastereoselectivity diastereoselectivity

Nitrogen protecting group does not affect selectivities


The Controversy over origin of high exoselectivities
Steric effects

Least nuclear motion


A
Orbital distortion

Schleyer: torsional steering


B
B
A

Favored

Martinelli, et.al. Tett. Lett. 1989, 30, 3935


Schleyer, P. R. J. Amer. Chem. Soc. 1967, 89, 701.

Addition from indicated olefin face avoids eclipsing A & B H's


Addition from exo face avoids eclipsing A & B hydrogens
(better hyperconjugative stabilization of transition state)
(better hyperconjugative stabilization of transition state)
Martinelli has carried out further studies on related structures...........
D. A. Evans Transition State Torsional Effects: Olefin Additions Chem 206

Martinelli: Torsional steering important in selectivity


O
mCPBA
H
89°

99 : 1 diastereoselectivity
major

O
mCPBA

Me 62°
Me

50 : 50 diastereoselectivity
63°

O
mCPBA
H

major
99 : 1 diastereoselectivity

40°

74°

Authors propose that diastereoselection controlled by


TS torsional effects
Martinelli & Houk, J. Org. Chem. 1994, 59, 2204.
D. A. Evans Transition State Torsional Effects: Olefin Additions Chem 206

Me
Me
O Me
O O O Me
AcO
O AcO
O
H Me Me O
–50°C
Me2HC Me Me
Me2HC Me Me
D. A. Evans Cyclohexanone Addition Reactions: Hydride Reduction Chem 206

Stereoselective Reductions: Cyclic Systems


H OH HA
O Nu: Attack Trajectories for Cyclohexanone
H HE (Torsional Argument)
Me3C [H] OH Me3C
Me3C

H H H

% Axial(-OH) Diastereomer 3
HE This approach favored
0 10 20 30 40 50 60 70 80 90 100 sterically
Nu:
HA

DIBAL-H 72:28 L-Selectride 8:92 The steric hindrance encountered by Nu-attack from the axial C=O face by the axial ring
substituents (hydrogens in this case) at the 3-positions is more severe than the
NaBH4 79:21 K-Selectride 3:97 steric hinderance encountered from Nu-attack from the equatorial C=O face.
Me
LiAlH(Ot-Bu)3 92:8 –
H B C CH2Me
LiAlH4 93:7 M+ H
3
Axial Attack
Observation: Increasingly bulky hydride reagents prefer to attack from H: –
the equatorial C=O face.
O–C–C–He dihedral:
+63.0 °
The most stereoselective Reductions
H OH
O O–C–C–He dihedral:
Me3C [H] Me3C
OH Me3C H +4.0 °
H: –
H H H Equatorial Attack O–C–C–He dihedral:
Reagent –56.0 °
Ratio
KBH(s-Bu)3 03 :97 The Issues Associated with the Reduction Process
Li in NH3 99 :01
Steric Effects: Attack across equatorial C=O face sterically more favorable.
H NHR
NPh Torsional Effects: However, attack across the axial face of the C=O group avoids
[H] NHR Me C H development of eclipsing interactions in the transition state. (Note the
Me3C Me3C 3 dihedral angle sign changes between reactants & products shown
above). These "torsional effects" favor axial attack.
H H H
(R) Reagent Ratio For "small" hydride reagents such as LiAlH4, torsional effects
Prediction are felt to be dominant and this explains the predisposition for
Ganem, Tet. Let 1981, 22, 3447 R = Bn LiBH(s-Bu)3 03 :97 axial attack.
Hutchins, JOC 1983, 48, 3412 R = Ph LiBH(s-Bu)3 01 :99
private communication For "large" hydride reagents such as H–BR4, steric effects
Al/Hg/MeOH ~90 :10 Prediction now are dominant and this explains the predisposition for
equatorial attack.
D. A. Evans Olefin Addition Reactions: Part–1 Chem 206

http://www.courses.fas.harvard.edu/~chem206/ ! Problems of the Day: (To be discussed)


Rationalize the stereochemical outcome of these reactions.
Chemistry 206
OH OH OH
9-BBN
Advanced Organic Chemistry Me2CH H2O2
Me2CH diastereoselection 24:1
Me Me
Lecture Number 8 W. C. Still & J. C. Barrish, J. Am. Chem. Soc. 1983, 105, 2487.

Me NHCONHPh m-CPBA Me NHCONHPh


Olefin Addition Reactions–1 Ph Ph
CH2Cl2, 0 °C
75 % O
! Hydroboration Roush, J. Org. Chem. 1987, 52, 5127. Diastereoselection = 95 : 5

! Epoxidation & Directed Epoxidation

Problem 309. The indicated olefins were subjected to hydroboration/oxidation


! Reading Assignment for week as shown (Synlett, 1993, 696).
Me Me
A. Carey & Sundberg: Part B; Chapter 4 Me Me BH3•THF
"Electrophilic Additions to C–C Multilple Bonds"
Hexanes R * O
R *
O
then H2O2 OH O
K. Houk, Science. 1986, 231, 1108-1117 Substrate A O Me
Me Me
Theory & Modeling of Stereoselective Organic Reactions (Handout) Me
Me Me BH3•THF
Me Me Me
K. Houk, Tetrahedron. 1984, 40, 2257-2274 Hexanes
Theoretical Studies of Stereoselective Hydroboration Reactions (Handout) R O
then H2O2 R * O Me
Substrate B O *
Me OH OH
Me
Hoveyda, Evans, Fu, Chem Rev. 1993, 93, 1307-1370
Substrate-Directable Chemical Reactions Part A. Please predict the major stereoisomer obtained from these reactions
(Electronic Handout) and illustrate your predictions with clear 3D drawings.

Part B. Please also explain the fact that B suffers reductive ring opening while
Wednesday, A does not..
D. A. Evans October 4, 2006
D. A. Evans Olefin Addition Reactions: Introduction Chem 206

Representative Cis-Addition Processes Representative Trans-Addition Processes

! Hydrometallation ! Halogenation
M H Br R
R R H
C C R + M H R C C R C C R + Br Br R C C
H H H H
M = B, Al, etc H H H Br

! Hydrogenation
M-catalyst H H ! Oxy–metallation (M = Hg(II), Tl(III)
R
C C R + H H R C C R
H H RO R
H H R H
C C R + Hg(OR)2 R C C
H H
! Group Transfer (epoxidation) H Hg–OR
O
R
C C R + RO2H R C C R ! Oxy–sulfenation (M = S(II), Se(II)
H H –ROH H H
RO R
R H
! Group Transfer (dihydroxylation) C C R + R–S–X R C C
O H H
O
H S–R
Os
R
C C R + OsO4
O O Attributes:
H H –N2 R C C R
H H
Each process may proceed via an bridged X
intermediate where X is the initiating electrophile R C C R
! Group Transfer (cyclopropanation) R2
H H
C Olefin substitution may disrupt bridging
R M-catalyst
C C R + R2C=N2 R C C R
H H –N2 H H
! Addition of hydrogen halides
! Cycloadditions (one of many!) R O
H R H R
R R
C C R H H
R + H–X R C C
C C R + R2C=C=O R C C R
H H + H C C
H H H X R X
–N2 H H
Attributes:
Attributes:
Process may proceed via an bridged H
intermediate where H+ is the initiating electrophile
Each process adds to the C=C via a stereospecific process R C C R
H H
Olefin substitution, reaction conditions as well as
Intermediates may be involved in some of the indicated reactions halide type may disrupt bridging
D. A. Evans Allylic Strain & Olefin Hydroboration Chem 206
The basic process Hydroborations dominated by A(1,3) Strain
S S ‡ OH
H B CH2OBn B2H6 CH2OBn
H H H O O
H B H2B H2O2
H !– Me Me
H Me Me
R C C R
R R R R diastereoselection 8:1
C C C C R R
R R R !+ R OMe OMe OH
Me B2H6
O H2O2 O OH
Me Me Me Me Me
Response to steric effects: Here is a good calibration system: OH
diastereoselection 12:1
A
Y. Kishi & Co-workers, J. Am. Chem. Soc. 1979, 101, 259.
Oxidant Ratio, A:E Reference
CH2
MCPBA 69:31 JOC, 1967, 32, 1363
OH
Me3C JOC, 1970, 35, 2654
BH3, H2O2 34:66 B2H6
E BnO OH H2O2 BnO OH
H
Me Me Me Me Me Me

Acyclic hydroboration can be controlled by A(1,3) interactions: Diastereoselection = 3:1

OH C. H. Heathcock et. al. Tetrahedron Lett 1984 25 243.


RL B2H6 RL
OH OH major diastereomer
H2O2
RM Me RM Me
Me Me
Me Me
A(1,3) allylic strain ThexylBH2,
control elements TrO OTr
Steric effects; RL vs RM then BH3 OH OH OH
Staggered transition states OH
H TrO OTr Diastereoselection; 5:1
H RM
B H
RM H Me
Me H C C Me Me Me Me ThexylBH2, Me Me Me Me
major H C C minor
CH2OR
CH2OR RL H then BH3 TrO OTr
B H
RL H OH OH OH OH OH
H OH OH OH
TrO OTr
Diastereoselection; 4: 1

Houk, "Theoretical Studies of Stereoselective Hydroboration Reactions"


Tetrahedron 1984, 40, 2257 (Handout) Still, W.C.; Barrish, J. C. J. Am. Chem. Soc. 1983, 105, 2487.
D. A. Evans Allylic Strain & Olefin Hydroboration Chem 206

Hydroborations dominated by A(1,2) Strain ! Case II: Dialkylboranes


R
Me R
R2BH
H B Me
RL OH RM H R2BH
H2O2 TS1 H RL OH
Me Me C C H2O2
R2BH structure is major
RL RM H
a potential variable RM
Me RL favored for R2BH
RM R2BH R
RL OH R
H2O2 B H
RM H Me
RM H R2BH
TS2 C C Me RL OH
minor H H2O2
Houk's rules: Orient RL anti-periplanar to incoming reagents to avoid TS eclipsing:
RM
RL
Midland finds that TS1 favored for R2BH reagents, but TS1 ~ TS2 for BH3
! Case I: Borane H
H Others have found that TS1 favored over TS2 for BH3
B H
RM H Me
B BH3
TS2 H RL OH Representative Examples
C C Me H2O2
major H
RM
RL favored for BH3 Me Me
CH2 CH2OH
H Me Me
A(1,2) Strain H H 9-BBN H
H B Me
A RM H BH3 H H2O2 H
TS1 H RL OH
Me C C H2O2
minor H H
H RM
RL R2BH diastereoselection
from Lecture 5:
The Torsional Energy Profile borane methylsulfide 1:1
M. M. Midland & Co-workers,
H
J. Am. Chem. Soc. 1983, 105, 3725.. thexylborane 4:1
Me
! = 180 H C C Me 9-BBN 14 : 1
H dicyclohexylborane 26 : 1
H
! = 50
OH OH OH
Me 9-BBN R = n-Bu: diastereoselection 11:1
H R H2O2 R = CHMe2: diastereoselection 24:1
C C Me R
H H
Me Me
H
! = 110 +2.68
!=0 kcal Model is consistent if you presume HO = RM: R = RL
H +1.39 kcal Me
H H
C C Me C C Me
H Me H H
H
+0.06 kcal H W. C. Still & J. C. Barrish, J. Am. Chem. Soc. 1983, 105, 2487.
!=0 ! = 180
D. A. Evans Allylic Strain & Olefin Hydroboration Chem 206

! Case I: Dialkylboranes R
! Case II: Borane H
R H
H B Me H B Me
RM H R2BH A RM H BH3
TS1 H RL OH TS1 H RL OH
Me C C H2O2 Me C C H2O2
major minor
H RM H RM
RL RL
R favored for R2BH H
R B H H B H
RM H Me RM H Me
R2BH B BH3
TS2 H
RL OH TS2 H RL OH
C C Me C C Me
minor H H2O2 major H H2O2
RM RM
RL RL
favored for BH3

OMe OMe
Me Me OMe
Me Me
Me Me
A B O
D E OMe
HO2C C Evans, Ratz, Huff, Sheppard, JACS 1995, 117, 3448-3467.
O 9 O O O
OH H H Me H F
Me Me O
Me
Lonomycin A Me OH
C-9 ! C10

OMe
H H
Me Me H
O OH OH H B H H B
RO H RO H
BH3•SMe2 10 H
H
85% XP O 9 C C Me Me C C
OMe H H H
Me Me Me
OMe RL RL
diastereoselection TS1 favored TS2 disfavored
Me Me 92 : 8
O O OH
10 OMe R R
O N O 9
OMe H Me Me R B H H B R
Me Me Me O OH OH RO H RO H
Bn 10 H H
XP O 9 C C Me Me C C
OMe H H H
9-BBN Me Me Me RL RL
60%
diastereoselection TA1 disfavored TA2 favored
> 95 : 5
D. A. Evans Represetative Hydroboration Examples: Acyclic Control Chem 206

Me O
For each of the examples shown below, attempt to rationalize the stereochemical Me CO2H H
1. 9-BBN
outcome of the reaction in terms of one of the models presented in the discussion. O
CH2
2. H2O2, NaOH
Me H
Me Me Me
OH Wolinsky, J.; Eustace, E. J.
J. Org. Chem. 1972, 37, 3376. Diastereoselection = 7:1
B2H6
Me R
Et O O H2O2 Et O
Et H Et Et O
Et OH Et Me CO2H Me
1. 9-BBN
"one isomer" O
Y. Kishi & Co-workers, J. Am. Chem. Soc. 1978, 100, 2933. CH2
2. H2O2, NaOH
Me H
Me
Me Wolinsky, J.; Nelson, D.
Me
O Me Tetrahedron. 1968, 25, 3767. Diastereoselection = 10:1
O Me
Me O BH3.THF
Me O
Me Me

B2H6/[O] H
HO Me Me
Mori, K.
Tetrahedron 1976, 32, 1979 diastereoselection 12:1
OH CH2 OH
OH
Diastereoselection = 19:1
R
R
H
O H Me Me
O HO O
O
CH3 BH3.THF B2H6/[O] H
CH3 Me Me
O CH3 O CH3
OBn OH CH2 OH
OBn OH
Diastereoselection = 32:1
Oikawa et. al. R=H; Diastereoselection = 6.8:1
Tetrahedron Lett. 1983, 19, 1987. R=OBn Diastereoselection = 6.6:1
Me Me

B2H6/[O] H
Me Me
R OH R OH
OH CH2 OH
BH3.THF OH
OH Diastereoselection = 10:1
H
Me
Me
Birtwistle et. al.
R = CH3; Diastereoselction = 6.7:1
Me H
Tetrahedron Lett. 1986, 25, 243. R = isopropyl "One Compound" B2H6/[O] Me
OH CH2
Schulte-Ette, K.H.; Ohloff, G. OH
OH
Helv. Chim. Acta 1967, 50, 153.
Diastereoselection = 4.6:1
D. A. Evans Representative Hydroboration Examples: Cyclic Systems Chem 206

CH2 OH OH
BH3.THF OH H H
BnO O O
H BH3.THF BnO
HO HO H
Me3C Me3C
OH
Diastereoselection = 2.1:1 O O
Me Me
B. Fraser-Reid et. al.
J. Am. Chem. Soc. 1984, 106, 731. Major isomer; no ratio given.

CH2
BH3.THF OH
Me Me
Me3C Me3C CH3
Me Me H CH3
O
Diastereoselection = 1.2:1 N O H
N
H H BH3.THF
H H
CH2 OH
O
O O
O
CH2
BH3.THF OH
Sallay, S. I.
J. Am. Chem. Soc. 1967, 89, 6762. 90% yield, no diastereoselection given
Me3C Me Me3C Me
Diastereoselection = 3.3:1

Me Me
BH3.THF
CH2
BH3.THF OH Me Me OH
O CH2 O H
H H
Me3C Me Me3C Me CO2Me CO2Me
Diastereoselection = 2.4:1
Ley, S. et.al. 55% yield with the diastereomeric alcohol
J. Chem. Soc. Chem. Commun. 1983 630.
produced in an unspecified amount.
Recycling of the minor isomer further
provided 15% of the desired material

CH2
BH3.THF OH N–NHAr N–NHAr
Me Me
BH3.THF
Me Me
Diastereoselection = 4.9:1 Me Me
Y. Senda et. al. (Compare with H.C. Brown's
Tetrahedron 1977, 33, 2933. case, with 9-BBN; 1.5:1) H H
Me Me OH
McMurry, J. E.
J. Am. Chem. Soc. 1968, 90, 6321. Minor diastereomer not detected
D. A. Evans Directed Reactions: An Introduction Chem 206

Stereochemical Control Elements for all reactions Heteroatom-directed Reactions


Mechanism-based: (HO & C=C must be allylic)
! Steric & Electronic Factors
! Stereoelectronic Considerations [3,3]
R
! Associative Substrate-Reagent Interactions
R R
O
OH O CH2 R
! Steric control: A B
disfavored
R Claisen Rearrangement
R R
A via Reagent Ligation
A B C C
H H
H Et2Zn
B
favored CH2
CH2I2
favored product A B R R
R
OH O CH2I
Zn OH
Nonbonding Interactions disfavor the syn diastereoface ? Simmons-Smith Reaction

Directed Reactions
Hydroxyl-directed Reactions
Review: Hoveyda, Evans, Fu Chem. Reviews 1993, 93, 1307 MCPBA
ratio 92 : 8 Henbest
! Associative Substrate-Reagent Interactions Cl CO3H J. Chem. Soc. 1958, (1957)

A B O
X X A B X favored
A OH
A B t-BuOOH
C C Sharpless
VO(acac)2
ratio 98 : 2 JACS 95, 6136, (1973)
B H H OH
H
favored product
disfavored
Noncovalent Interaction favors the syn diastereoface CH2
A B Et2Zn

CH2I2 Winstein
OH
Directed Oxidations JACS 91, 6892, (1969)
ratio 90 : 10
Epoxidation
Hydroboration
Agenda Directed Reductions Me Me
M(I) + H2 (Ir+) Stork
Hydrogenation JACS 105, 1072 (1983)
Hydride reduction
(Rh+) Evans
Directed C–C Bond Constructions OH JACS 106, 3866 (1984)
OH
D. A. Evans Directed Reactions: An Introduction Chem 206

The Directed Peracid Epoxidation


Orientation of the Directing Group
" Transition State Hydrogen Bonding: Substrate as H-bond donor
H
reagent X (Henbest)
H Me Me R R R
X = O, CH2 ‡
Me OH Me OH
O O O O O O

!
O H
‡ ‡ H H !
O
!maj Me H R H
H
H •• •• C O
R!
O H
H
C C H C
Me R C O
R H C R C
Reag OH H C
Reag
OH C H2
O H2
H R C
C C H !min C
Me H2 require allylic or homoallylic alcohol
H Me

TSmajor TSminor " Transition State Hydrogen Bonding: Peracid as H-bond donor (Ganem)

CF3 CF3 ‡
CF3

O O O
Orientation of directing group is not the same for all reactions O O O

H !
O H !
O H
H R
R O
R O
H R C
R H!
O
R
C O C
Reagent Selectivity C C
! Estimate C C H2 H
C C
H2 H H
+5 H2
t-BuO2H, V 71 : 29 ~ 50 °
RCO3H 95 : 5 ~ 120 ° require more acidic peracid both allylic alcohols and ethers OK

CH2I2, Zn–Cu > 99 : 1 ? Syn : Anti Syn : Anti Syn : Anti Syn : Anti
OH (m-CPBA) (CF3CO3H) OTBS (m-CPBA) (CF3CO3H)

24 : 1 50 : 1 1:7 5:1

OH OTBS
The transition state bite angles for the above reactions are either
not known or have been only crudely estimated.
24 : 1 100 : 1 1:8 12 : 1
Me3C Me3C
The "best guesses" are provided. OH OTBS

5:1 100 : 1 1:4 1:6


Me3C Me3C
Ganem Tet. Let. 1985, 26, 4895
D. A. Evans Diastereoselective Peracid Epoxidation Chem 206

Epoxidation of Cyclic Olefins with Amide &Urethane Directing Groups Epoxidation of Cyclic Homoallylic Alcohols

Substrate Major Product Selectivity Major


Substrate Selectivity
O O Product

HN Me HN Me OH O OH 9:1

HO HO "highly Me Me
selective" OH OH
O
HO HO "highly
O O selective"
HO HO
Et Me O
HN Ph HN Ph Et Me

Me Me
"highly
O selective"
16 : 1
HO HO
O O O

O R O R a. R = NH2 3:1 Me Me

5:1 1:1
b. R = NHBn
O HO HO
10 : 1
c. R = NMe2 O
Me Me
Me Me
a. R = OCONHBn >20 : 1
21 : 1
b. R = OCONMe2 >20 : 1 HO HO
R R O
O
HOH2C HOH2C
RO RO
a. R = CONH2 6:1 5:1
b. R = CONHBn >10 : 1 AcO AcO
O
AcO AcO 2:1
O c. R = CONMe2
Conditions: Perbenzoic acid, or meta-chloroperbenzoic acid
Conditions: Perbenzoic acid, or meta-chlorobenzoic acid in benzene. in benzene or cyclopentane.

(Table 11, p1316, from the Evans, Hoveyda, Fu review article) (Table 14, p1318, from the Evans, Hoveyda, Fu review article)
D. A. Evans Sharpless Epoxidation (V+5) Chem 206
The Sharpless Epoxidation ! The literature precedent: Sheng, Zajecek, J. Org. Chem. 1970, 35, 1839
R
OR
RO O+ TBHP
OR
!
O RO RDS
O V RO "
O "
O
V !
O V OH 80 oC OH OH
HO O
O !
O
VO(acac)2 4:1
O O Catalyst
Mo(CO)6 1:1
–ROH
ROOH
! Next step: Sharpless, Michaelson JACS 1973, 95, 6136
OR
RO OH
O !
O OH
HO
V !
O Me
VO(acac)2
Me
HO Regioselection 20:1
"
O
OR Me TBHP Me
80 °C
Me Me
Aldrichimica Acta, 12, 63 (1979)
OH OH
Mo(CO)6
Stereoselection 98:2
TBHP (90 % yield)
O–C2–C3–C4 = 41° 80 °C "
O
The Sharpless estimate: ~50°

Relative Rates (Diastereoselectivities) for the Epoxidation of


Cyclohexene Derivatives JACS 1973, 95, 6136

t-BuOOH t-BuOH krela,b (diastereoselectivityc )


Substrate
HO
HO peracid Mo(CO)6 VO(acac)2
O

1.00 1.00 1.00


2 tBu OR
OH
O-OtBu
1 O O V 0.55 (92 : 8) 4.5 (98 : 2) >200 (98 : 2)
3 RO
4 V O O
O OAc
O
0.046 (37 : 63) 0.07 (40 : 60) --
tBu
OH
O
Chem 3D Transition State slow RO 0.42 (60 : 40) 11.0 (98 : 2) 10.0 (98 : 2)
V O
O
O a,b The relative rate data apply only to a given column.

Values in parenthesis refer to the ratio of syn:anti epoxide.


D. A. Evans Epoxidation of Acyclic Alcohols Chem 206

! Allylic Alcohols:
OH OH OH
Me OH Me OH Reagent
Me OH
Reagent Me Me Me Me + Me Me
+ !
O !
O
Me Me Me
"
O "
O Reagent Ratio
threo erythro m-CPBA 64 : 36
t-BuOOH / VO(acac)2 29 : 71
! Estimate Reagent Ratio
t-BuOOH / Mo(CO)6 62 : 38
~ 120 ° m-CPBA 95 : 5 t-BuOOH / (t-BuO)3Al 64 : 36
40-50 ° t-BuOOH / VO(acac)2 71 : 29
OH OH OH
t-BuOOH / Mo(CO)6 84 : 16
t-BuOOH
R1 R2 R1 R2 + R1 R2
! RCO3H Transition States: ! ~ 120 ° VO(acac)2 !
O !
O
SiMe3 SiMe3 SiMe3
OH H
R1 R2 Yield Ratio
TSmajor H H Oshima, Tetrahedron Lett. 1982, 23, 3387.
CH C H C C H TSminor H Bu 84 % 99 : 1
Me Me Me
C5H11 Me 70 % 99 : 1
Me OH
OH OH
t BuOOH
! V(+) Transition States: ! ~ 45 ° O O
Me VO(acac)2 !
O only isomer
HO O O
EtO OEt Me 60 % EtO OEt Me
TSmajor H Me H Me Me
C C H C C H TSminor OH OH
Me Me H
HO t-BuOOH
H O
!
O
O
VO(acac)2
O O only isomer
EtO OEt Me 60 % EtO OEt Me
Me Me
Depezay, Tetrahedron Lett. 1978, 19, 2869.
Me OH Me OH Me OH
Reagent !
O
+
Me Me Me Me Me Me t-BuOOH
Me Me
!
O !
O VO(acac)2
OH OH
threo erythro HO 60 % HO
Boeckman, JACS 1977, 99, 2805. Diastereoselection = 7 : 1

Reagent Ratio
K. Oshima & Coworkers Me NHCONHPh Me NHCONHPh Me NHCONHPh
Tetrahedron Lett. 1980, 21, 1657, 4843. m-CPBA 95 : 5 m-CPBA
Ph Ph Ph
t-BuOOH / VO(acac)2 86 : 14 CH2Cl2, 0 °C
+
t-BuOOH / Mo(CO)6 95 : 5 75 % !
O !
O
K. B. Sharpless & Coworkers
Tetrahedron Lett. 1979, 20, 4733. t-BuOOH / (t-BuO)3Al 100 : 0 Roush, J. Org. Chem. 1987, 52, 5127. Diastereoselection = 95 : 5
D. A. Evans Epoxidation of Acyclic Homoallylic Alcohols Chem 206
Anti diastereomer
Homoallylic Alcohols (Mihelich, JACS 1981, 103, 7690)
OH OH OH
!
O !
O
!
O !
O t-BuOOH
t-BuOOH R2 R2 + R2
OH OH + OH VO(acac)2
VO(acac)2 R1 Me R1 Me
Me Me Me Me R1 Me
90 % Me Me

Diastereoselection > 400 : 1 R1 R2 Yield Ratio


Control Elements
C6H13 Me 92 % 104 : 1
A(1,3) Strain H L
L' Me i-Pr 97 % > 400 : 1
Directed Rxn HV O R !
O
H O Syn diastereomer
Me !
O H HO
Me Me OH OH OH
H
!
O !
O
Me t-BuOOH
R2 R2 + R2
VO(acac)2
R1 Me R1 Me R1 Me
OH OH OH
!
O R1 R2 Yield Ratio
!
O
t-BuOOH Me 73 % 70 : 1
Me Me + Me C6H13
VO(acac)2 70 % 85 : 1
Et Et Et Me Me
Me C5H11 81 % 16 :1
Diastereoselection 12 : 1

Control Elements OH
!O
OH
!O OH
H L
L' OH
HV t-BuOOH
Directed Rxn O R !
O Me Me + Me
Me O VO(acac)2
! H
O Me Me C5H11 Me C5H11 Me C5H11
Et
Et E. D. Mihelich & Coworkers Diastereoselection = 211 : 1
J. Am. Chem. Soc. 1981, 103, 7690.

Epoxidation of Homoallylic Alcohols with TBHP, VO(acac)2


Prediction Substrate Product Selectivity
L L OH OH
H L' H L' !O
2:1
HV O R HV O R Me
Me
R2 O Anti should be more R2 H O
! H
O !
O H
R1 diastereoselective OH OH !O
H
than syn 4.6 : 1
R1 Me Me
Me Me
Me Me
Anti diastereomer Syn diastereomer OH OH !O
R R
Hex Hex 1.4 : 1
R = (CH2)7CO2Me
D. A. Evans Epoxidation of Acyclic Homoallylic Alcohols Chem 206

Epoxidation & Cyclization of Bishomoallylic Alcohols


Bishomoallylic Alcohols (Kishi, Tet. Lett. 1978, 19, 2741)
R
R Et R Et
OH VO(acac)2 AcOH
OH !
O
!
O Me
t-BuOOH, VO(acac)2 iPr OH Me iPr OH Me iPr O
Me CHMe2 Me CHMe2 Et
C6H6, RT !
OH
Et Et
diastereoselection ~ 9 : 1 The Kishi Lasalocid Synthesis (JACS 1978, 100, 2933)
Me Me
H Et OH O
Et A H

H Me HO O
R Me H Et B
R H CO2H Me Me Et O OH
!
O !
O
V OH Et
O O R Me
Me Et Me

A H
Et VO(acac)2
OH Ar OH Ar O Diastereoselection 8:1
OH TBHP Et
t-BuOOH, VO(acac)2
!
O AcOH !
OH
Me CHMe2 Ar = p-MeOPh Me Et
Me CHMe2
C6H6, RT Me
Et Me Et Me
diastereoselection ~ 20 : 1

Evans X-206 Synthesis JACS 1988, 110, 2506.


H Et
2nd stereocenter Me Et Me Me Me Me
Me Me Me
is reinforcing R H R Me OH
H
A B C D E
!
O
!
O Me
V OH O O O O O
O O R H OH OH Me OH Me F
Me OH O Me
O OH
OH
Et
OH OH
!
O Me O OH
t-BuOOH, VO(acac)2 O OH
Me CHMe2 Me CHMe2 !
O
Ph VO(acac)2
C6H6, RT N Et
Et Me XN Et
Et Me TBHP
O OBn Me C6H6, RT OBn Me
diastereoselection ~ 6 : 1 O
HOAc

Me Et O
Me Et diastereoselection 20 : 1 D Et
H Me (89 %) XN O
R H R Me Me
!
O OBn !
OH
V !
O
O OH
O R
D. A. Evans Olefin Addition Reactions: Part–2 Chem 206

! Problems of the Day:


http://www.courses.fas.harvard.edu/colgsas/1063 Rationalize the stereochemical outcome of the indicated reaction.

N M–H N H Me N OH
Me Me
O
Chemistry 206 OH H

R. Noyori R2AlH 97 : 3
Advanced Organic Chemistry Bull. Chem. Soc. Japan 47, 2617, (1974) LiAlH4 28 : 72

Lecture Number 9 Problem 579. The following publication (J. Org. Chem. 1991, 56, 5553) reported the
surprisingly selective olefin epoxidation illustrated below. In this reaction, olefin B in 1
was found to be much less reactive than olefin A. Using your knowlege of
Olefin Addition Reactions–2 stereoelectronic effects, provide an explanation for the reduced reactivity of olefin B in
diene 1.
Cl H Cl H Cl H
! Curtin-Hammett Principle RCO3H
A
RCO3H
B
O
2 1 3 O
favored disfavored
! Hydrogenation
Problem 313. Overman and co-workers recently reported the indicated selective
epoxidation in conjunction with a synthesis of briarellins A and E, a new family of
diterpenes (JACS 2003, 125, 6650). It should be noted that the Al(t-BuO)3/(t)-BuOOH
! Reading Assignment for week reagent system is both highly diastereoselective and site selective. It is also relevant to
the mechanism of the reaction that the ring-trisubstituted olefin lacking an allylic oxygen
substituent would normally be more prone to epoxidation with a peracid than the acyclic
A. Carey & Sundberg: Part B; Chapter 4 trisubstituted olefin.
"Electrophilic Additions to C–C Multilple Bonds"
Me H
Hoveyda, Evans, & Fu (1993). Substrate-directable chemical reactions. Me Al(t-BuO)3 R Me
Chem. Rev. 93: 1307-70 (pdf) TIPSO H H t-BuOOH H H
Me H H Me H
O 4 Å sieves O R
J. M. Brown, Angew. Chem. Int. Edit. 26, 190-203 (1987) (Handout) O
"
toluene, -20˚C "

H
H. Yamamoto et.al, Angew. Chem. Int. Ed. 2005, 44, 4389-4391 (pdf)
HO HO
Part. Provide a general mechanism illustrating how the Al(t-BuO)3/(t)-BuOOH
reagent epoxidizes olefins. Three-dimensional drawings are recommended.
Part B. Provide a general mechanism illustrating how the above epoxidation
Friday, proceeds and provide the stereochemistry (") of the product epoxide along with a
D. A. Evans October 6, 2006 stereochemical analysis of the noted face selectivity.
A Brief Introduction to the Curtin-Hammett Principle

J. I. Seeman, J. Chem,Ed. 1986, 63, 42-48 The Curtin-Hammett Principle and the Winstein-
Holness Equation

J. I. Seeman, Chem. Rev. 1983, 83, 84-134. Effect of Conformational Change on Reactivity in
Organic Chemistry. Evaluations, Applications, and Extensions of Curtin-Hammet-Winstein-
Holness Kinetics
Curtin–Hammett Conditions

k1 kA k2
Curtin–Hammett Principle PA

You might also like